You are on page 1of 339

‫ﺍﳌﻴﻜﺎﻧﻴﻚ ﺍﻟﺘﺤﻠﻴﻠﻲ‬

-1-

Create PDF files without this message by purchasing novaPDF printer (http://www.novapdf.com)
-2-

Create PDF files without this message by purchasing novaPDF printer (http://www.novapdf.com)
‫ﺍﳉﻤﻬﻮﺭﻳﺔ ﺍﻟﻌﺮﺑﻴﺔ ﺍﻟﺴﻮﺭﻳﺔ‬
‫ﻭﺯﺍﺭﺓ ﺍﻟﺘﻌﻠﻴﻢ ﺍﻟﻌﺎﱄ‬
‫ﺟﺎﻣﻌﺔ ﺗﺸﺮﻳﻦ‬
‫ﻛﻠﻴﺔ ﺍﻟﻌﻠﻮﻡ‬

‫ﺍﳌﻴﻜﺎﻧﻴﻚ ﺍﻟﺘﺤﻠﻴﻠﻲ‬

‫ﺃ‪ .‬ﺩ‪ .‬ﺣﺴﻦ ﻋﺒﺪ ﺍﻟﻜﺮﻳﻢ ﺳﻠﻤﺎﻥ ﺃ‪.‬ﺩ‪ .‬ﺃﲪﺪ ﺷﻔﻴﻖ ﺑﻴﺸﺎﻧﻲ‬
‫ﺃﺳﺘﺎﺫ ﰲ ﻗﺴﻢ ﺍﻟﻔﻴﺰﻳﺎﺀ‬ ‫ﺃﺳﺘﺎﺫ ﰲ ﻗﺴﻢ ﺍﻟﻔﻴﺰﻳﺎﺀ‬

‫ﺩ‪ .‬ﻋﻠﻲ ﳏﻤﻮﺩ‬ ‫ﺩ‪ .‬ﳏ ﻤﺪ ﺣﺴﻦ ﻓﺎﻫﻮﺩ‬


‫ﻣﺪﺭﺱ ﰲ ﻗﺴﻢ ﺍﻟﻔﻴﺰﻳﺎﺀ‬ ‫ﺃﺳﺘﺎﺫ ﻣﺴﺎﻋﺪ ﰲ ﻗﺴﻢ ﺍﻟﻔﻴﺰﻳﺎﺀ‬

‫‪  ‬‬
‫‪  ‬‬
‫‪‬‬
‫‪‬‬
‫‪ 1430- 1429‬ﻫـ‬
‫‪ 2009- 2008‬ﻡ‬

‫‪-3-‬‬

‫)‪Create PDF files without this message by purchasing novaPDF printer (http://www.novapdf.com‬‬
-4-

Create PDF files without this message by purchasing novaPDF printer (http://www.novapdf.com)
‫ﺍﶈﺘﻮﻯ‬

 
‫ ﺃﺳﺲ ﺍﳌﻴﻜﺎﻧﻴﻚ ﺍﻟﺘﺤﻠﻴﻠﻲ‬:‫ﺍﻟﻔﺼﻞ ﺍﻷﻭﻝ‬
11 1
14 2
17 3
21 4
24 5
26 6
29         7 

31 8
32         9 

35 
‫ ﻣﻌﺎﺩﻻﺕ ﺍﳊﺮﻛﺔ‬:‫ﺍﻟﻔﺼﻞ ﺍﻟﺜﺎﻧﻲ‬
47 –        10

49 11
51 12
53 13
54 14
56  q j  q j 15
57 16
60 
‫ ﻣﻌﺎﺩﻟﺔ‬،‫ ﺃﻗﻮﺍﺱ ﺑﻮﺍﺻﻮﻥ‬،‫ ﻣﻌﺎﺩﻻﺕ ﻫﺎﻣﻠﺘﻮﻥ‬: ‫ﺍﻟﻔﺼﻞ ﺍﻟﺜﺎﻟﺚ‬
‫ ﺟﺎﻛﻮﺑﻲ‬،‫ﻫﺎﻣﻠﺘﻮﻥ‬
75 17
78 3.1018

-5-

Create PDF files without this message by purchasing novaPDF printer (http://www.novapdf.com)
81 19
83 20
85 21
89 – 22
94     –       23

97 – 24
99 – 25

103 26
105 27
108 
‫ ﻣﺒﺎﺩﻯء ﺍﻟﺘﻐﲑﺍﺕ‬: ‫ﺍﻟﻔﺼﻞ ﺍﻟﺮﺍﺑﻊ‬
115 28
119 29
120 30

122 – – 31
126 32
129 33
133 34
134 35
137 36
139 37
‫ ﺣﺮﻛﺔ ﺟﺴﻴﻢ ﰲ ﺣﻘﻞ ﻛﻬﺮﻃﻴﺴﻲ‬:‫ﺍﻟﻔﺼﻞ ﺍﳋﺎﻣﺲ‬
‫ﻣﺪﺧﻞ ﺇﱃ ﻧﻈﺮﻳﺔ ﺍﻻﺿﻄﺮﺍﺏ ﺍﻟﺘﻘﻠﻴﺪﻳﺔ‬
149 38
152 39
153 40

156   41
159 42

-6-

Create PDF files without this message by purchasing novaPDF printer (http://www.novapdf.com)
162 43

‫ ﺣﺮﻛﺔ ﺟﺴﻢ ﺻﻠﺐ ﰲ ﺍﻟﻔﻀﺎء‬:‫ﺍﻟﻔﺼﻞ ﺍﻟﺴﺎﺩﺱ‬


174 44
176 45
179 46
183 47
186 – 48
190 49
200 50
201 
207 
‫ ﺍﳌﺮﻭﻧﺔ‬:‫ﺍﻟﻔﺼﻞ ﺍﻟﺴﺎﺑﻊ‬
‫ﺧﻮﺍﺹ ﻣﺮﻭﻧﺔ ﺍﻷﺟﺴﺎﻡ ﺍﻟﺼﻠﺒﺔ‬
218 51
219  52
220 53
222 54
255 55
230 56
235 57
242 58
243 59
 ‫ﺍﻟﻔﺼﻞ ﺍﻟﺜﺎﻣﻦ‬
‫ﺍﳌﺒﺎﺩﻯء ﺍﻷﺳﺎﺳﻴﺔ ﻟﻠﻨﻈﺮﻳﺔ ﺍﻟﻨﺴﺒﻴﺔ ﺍﳋﺎﺻﺔ‬
 251 60
 253  61
 256  62
 258  63
 260  64
 265  65

-7-

Create PDF files without this message by purchasing novaPDF printer (http://www.novapdf.com)
 267  66
 268  – 67
 271 68
 
 274  69
 278  70
 281  
 ‫ﺍﻟﻔﺼﻞ ﺍﻟﺘﺎﺳﻊ‬
‫ﻣﻴﻜﺎﻧﻴﻚ ﺍﻟﻨﻈﺮﻳﺔ ﺍﻟﻨﺴﺒﻴﺔ‬
 285  71
 286  72
 290         73
 
 294 74
 
 296  75
 304  76
 307  77
 309  78
 312  79
 317  
 320 ‫ﻣﻠﺤﻖ‬
337 ‫ﺍﳌﺼﻄﻠﺤﺎﺕ ﺍﻟﻌﻠﻤﻴﺔ‬
 346 ‫ﺍﳌﺮﺍﺟﻊ‬

-8-

Create PDF files without this message by purchasing novaPDF printer (http://www.novapdf.com)
‫ﺍﻟــﻤــﻘــﺪﻣــﺔ‬

‫ﻴﻌﺘﺒﺭ ﺍﻟﻤﻴﻜﺎﻨﻴﻙ ﺍﻟﺘﺤﻠﻴﻠﻲ ﺃﺤﺩ ﻓﺭﻭﻉ ﺍﻟﻤﻴﻜﺎﻨﻴﻙ ﺍﻷﺴﺎﺴﻴﺔ ﻴﻌﺘﻤﺩ‪ ،‬ﺒﺼﻭﺭﺓ‬


‫ﻋﺎﻤﺔ‪ ،‬ﻋﻠﻰ ﺍﻟﺘﺤﻠﻴل ﺍﻟﺭﻴﺎﻀﻲ ﻓﻲ ﺩﺭﺍﺴﺔ ﺴﺎﺌل ﺍﻟﻤﻴﻜﺎﻨﻴﻙ ﺤﻴﺙ ﻴﺘﻡ ﺤﻠﻬﺎ ﺒﻁﺭﻴﻘﺔ‬
‫ﺘﺨﺘﻠﻑ ﻋﻥ ﺍﻟﻤﻴﻜﺎﻨﻴﻙ ﺍﻟﻨﻴﻭﺘﻨﻲ ﺍﻟﺫﻱ ﻴﺭﺘﻜﺯ ﻋﻠﻰ ﺍﻟﻤﺒﺎﺩﻯﺀ ﺍﻟﺜﻼﺜﺔ ﺍﻟﻤﻌﺭﻭﻓﺔ‪.‬‬
‫ﺒﺩﺃﺕ ﺘﻅﻬﺭ ﺍﻟﻤﺒﺎﺩﻯﺀ ﺍﻟﻌﺎﻤﺔ ﻟﻠﻤﻴﻜﺎﻨﻴﻙ ﺍﻟﺘﺤﻠﻴﻠﻲ ﻓﻲ ﺍﻟﻘﺭﻥ ﺍﻟﺜﺎﻤﻥ ﻋﺸﺭ‬
‫ﺒﻔﻀل ﻋﻠﻤﺎﺀ ﻜﺒﺎﺭ ﻤﺜل ﻻﻏﺭﺍﻨﺞ ﻭﻫﺎﻤﻠﺘﻭﻥ ﻭﺒﻭﺍﺼﻭﻥ ﻭﺁﺨﺭﻭﻥ ﺍﻟﺫﻴﻥ ﻭﻀﻌﻭﺍ‬
‫ﺍﻷﺴﺱ ﺍﻟﻌﺎﻤﺔ ﻭﺍﺴﺘﻨﺘﺠﻭﺍ ﻤﻌﺎﺩﻻﺕ ﺤﺭﻜﺔ ﺃﺴﻬل ﻭﺃﺴﻠﺱ ﻤﻥ ﻤﻌﺎﺩﻻﺕ ﻨﻴﻭﺘﻥ‪.‬‬
‫ﻟﻘﺩ ﺤﺎﻭﻟﻨﺎ ﻓﻲ ﺍﻟﻔﺼل ﺍﻷﻭل ﻤﻥ ﻫﺫﺍ ﺍﻟﻜﺘﺎﺏ ﺸﺭﺡ ﺍﻟﻤﺒﺎﺩﻯﺀ ﻭﺍﻟﺘﻌﺎﺭﻴﻑ‬
‫ﺍﻷﺴﺎﺴﻴﺔ ﻟﻠﻤﻴﻜﺎﻨﻴﻙ ﺍﻟﺘﺤﻠﻴﻠﻲ ﻭﺼﻭﻻﹰ ﺇﻟﻰ ﺍﻟﺸﺭﻭﻁ ﺍﻟﻌﺎﻤﺔ ﻟﺘﻭﺍﺯﻥ ﺍﻟﺠﻤل ﺍﻟﻤﺎﺩﻴﺔ‬
‫ﺘﺤﻠﻴﻠﻴﺎﹰ‪ ،‬ﻭﻀﻤﻥ ﺍﻟﻔﺼل ﺘﻤﺎﺭﻴﻥ ﻤﺤﻠﻭﻟﺔ ﻭﻏﻴﺭ ﻤﺤﻠﻭﻟﺔ‪.‬‬
‫ﻓﻲ ﺍﻟﻔﺼل ﺍﻟﺜﺎﻨﻲ ﺍﺴﺘﻨﺘﺠﻨﺎ ﺍﻟﻤﻌﺎﺩﻻﺕ ﺍﻟﻌﺎﻤﺔ ﻟﻠﺤﺭﻜﺔ ﺒﻁﺭﻴﻘﺔ ﻻﻏﺭﺍﻨﺞ ﻭﻗﺩ‬
‫ﺘﻀﻤﻥ ﻫﺫﺍ ﺍﻟﻔﺼل ﻜﺜﻴﺭﺍﹰ ﻤﻥ ﺍﻟﺘﻁﺒﻴﻘﺎﺕ ﻟﻤﺎ ﻟﻬﺎ ﻤﻥ ﺃﻫﻤﻴﺔ ﻓﻲ ﻓﻬﻡ ﺍﻟﻤﻭﻀﻭﻉ‬
‫ﻭﺘﻤﻜﻴﻥ ﺍﻟﻁﺎﻟﺏ ﻤﻥ ﺍﻟﺘﻌﺎﻤل ﺒﻨﻔﺴﻪ ﻤﻊ ﻤﺴﺎﺌل ﺍﻟﻤﻴﻜﺎﻨﻴﻙ‪ ،‬ﺃﻤﺎ ﻓﻲ ﺍﻟﻔﺼل ﺍﻟﺜﺎﻟﺙ ﻓﻘﺩ‬
‫ﺸﺭﺤﻨﺎ ﻁﺭﻗﺎﹰ ﺃﺨﺭﻯ ﻟﻠﻭﺼﻭل ﺇﻟﻰ ﻤﻌﺎﺩﻻﺕ ﺍﻟﺤﺭﻜﺔ‪ ،‬ﻤﻨﺴﻭﺒﺔ ﺇﻟﻰ ﻜل ﻤﻥ ﻫﺎﻤﻠﺘﻭﻥ‬
‫ﻭﺭﺍﻭﺙ ﻭﺠﺎﻜﻭﺒﻲ ﻭﻗﺩ ﺘﻀﻤﻥ ﺍﻟﻔﺼل ﺃﻴﻀﺎﹰ ﻋﺭﻀﺎﹰ ﻷﻗﻭﺍﺱ ﺒﻭﺍﺼﻭﻥ ﻭﺒﻴﺎﻥ‬
‫ﺃﻫﻤﻴﺘﻬﺎ ﻓﻲ ﺇﻴﺠﺎﺩ ﺘﻜﺎﻤﻼﺕ ﺍﻟﺤﺭﻜﺔ ﺜﻡ ﺃﻭﺭﺩﻨﺎ ﺘﻌﻤﻴﻤﺎﹰ ﻟﻬﺫﻩ ﺍﻷﻗﻭﺍﺱ ﻓﻲ ﻤﻴﻜﺎﻨﻴﻙ‬
‫ﺍﻟﻜﻡ ﻭﺨﺘﻤﻨﺎ ﺍﻟﻔﺼل ﺒﺩﺭﺍﺴﺔ ﻤﻌﺎﺩﻟﺔ ﻫﺎﻤﻠﺘﻭﻥ‪-‬ﺠﺎﻜﻭﺒﻲ ﻓﻲ ﺍﻟﻤﻴﻜﺎﻨﻴﻙ ﺍﻟﻤﻭﺠﻲ‬
‫ﻭﺍﻟﻌﻼﻗﺔ ﺒﻴﻥ ﺍﻟﻤﻴﻜﺎﻨﻴﻙ ﺍﻟﺘﻘﻠﻴﺩﻱ‪ ،‬ﺍﻟﺫﻱ ﻴﺘﻌﺎﻤل ﻤﻊ ﺍﻷﺠﺴﺎﻡ ﺍﻟﻤﺎﻜﺭﻭﺴﻜﻭﺒﻴﺔ‬
‫)ﺍﻟﺠﻬﺭﻴﺔ( ﻭﺍﻟﻤﻴﻜﺎﻨﻴﻙ ﺍﻟﻤﻭﺠﻲ‪ ،‬ﺍﻟﺫﻱ ﻴﺘﻌﺎﻤل ﻤﻊ ﺍﻷﺠﺴﺎﻡ ﺍﻟﻤﻴﻜﺭﻭﺴﻜﻭﺒﻴﺔ‬
‫)ﺍﻟﻤﺠﻬﺭﻴﺔ( ﻭﺒﻴﻨﺎ ﺃﻨﻪ ﻴﻤﻜﻥ ﺍﻋﺘﺒﺎﺭ ﺍﻷﻭل ﻜﺤﺎﻟﺔ ﺨﺎﺼﺔ ﻤﻥ ﺍﻟﺜﺎﻨﻲ‪.‬‬

‫‪-9-‬‬

‫)‪Create PDF files without this message by purchasing novaPDF printer (http://www.novapdf.com‬‬
‫ﻭﻓﻲ ﺍﻟﻔﺼل ﺍﻟﺭﺍﺒﻊ ﺸﺭﺤﻨﺎ ﻤﺒﺎﺩﻯﺀ ﺍﻟﺘﻐﻴﺭﺍﺕ )ﺍﻟﻤﺘﻐﺎﻴﺭﺍﺕ( ﺍﻟﺘﻲ ﻴﻤﻜﻥ ﺃﻥ‬
‫ﻨﺴﺘﻨﺘﺞ ﻤﻌﺎﺩﻻﺕ ﺍﻟﺤﺭﻜﺔ ﻤﻥ ﻜل ﻤﻨﻬﺎ‪ ،‬ﻜﻤﺒﺩﺃ ﺍﻟﻔﻌل ﺍﻷﺼﻐﺭ ﻭﺍﻟﻘﺴﺭ ﺍﻷﺼﻐﺭ‬
‫ﻭﻏﻴﺭﻫﺎ‪...‬‬
‫ﻭﻗﺩ ﻜﺎﻥ ﻤﻥ ﺍﻟﻀﺭﻭﺭﻱ‪ ،‬ﻓﻲ ﺍﻟﻔﺼل ﺍﻟﺨﺎﻤﺱ‪ ،‬ﺃﻥ ﻨﺴﺘﻔﻴﺩ ﻤﻥ ﺍﻟﻤﻴﻜﺎﻨﻴﻙ‬
‫ﺍﻟﺘﺤﻠﻴﻠﻲ ﻟﺩﺭﺍﺴﺔ ﺤﺭﻜﺔ ﺠﺴﻴﻡ ﻤﺸﺤﻭﻥ ﻓﻲ ﺤﻘل ﻜﻬﺭﻁﻴﺴﻲ ﻓﻲ ﺇﻁﺎﺭ ﺍﻟﻨﻅﺭﻴﺔ‬
‫ﺍﻟﻨﺴﺒﻴﺔ ﺍﻟﺨﺎﺼﺔ‪ ،‬ﻟﻤﺎ ﻟﻬﺫﻩ ﺍﻟﺤﺭﻜﺔ ﻤﻥ ﺃﻫﻤﻴﺔ ﻓﻲ ﻨﻅﺭﻴﺔ ﺍﻟﻤﺴﺭﻋﺎﺕ ﺍﻟﺨﻁﻴﺔ‬
‫ﻭﺍﻟﺩﺍﺌﺭﻴﺔ‪ .‬ﻭﻗﺩ ﻜﺎﻥ ﻤﻥ ﺍﻟﻤﻔﻴﺩ ﻓﻲ ﻨﻬﺎﻴﺔ ﺍﻟﻔﺼل‪ ،‬ﺃﻥ ﻨﺘﻁﺭﻕ ﻟﻨﻅﺭﻴﺔ ﻫﺎﻤﺔ ﺘﻁﺒﻕ‬
‫ﻓﻲ ﺍﻟﻜﺜﻴﺭ ﻤﻥ ﻓﺭﻭﻉ ﺍﻟﻔﻴﺯﻴﺎﺀ ﺍﻟﺘﻘﻠﻴﺩﻴﺔ ﻭﺍﻟﻜﻭﺍﻨﺘﻴﺔ؛ ﻓﺄﻋﻁﻴﻨﺎ ﻟﻤﺤﺔ ﺴﺭﻴﻌﺔ ﻭﻤﺒﺴﻁﺔ‬
‫ﻋﻥ ﻨﻅﺭﻴﺔ ﺍﻻﻀﻁﺭﺍﺏ ﻤﻥ ﻭﺠﻬﺔ ﻨﻅﺭ ﻜﻼﺴﻴﻜﻴﺔ ﺘﺎﺭﻜﻴﻥ ﺍﻟﻘﺴﻡ ﺍﻟﻜﻭﺍﻨﺘﻲ‬
‫ﻟﻤﻴﻜﺎﻨﻴﻙ ﺍﻟﻜﻡ ﺍﻟﺫﻱ ﺴﻴﺩﺭﺴﻪ ﺍﻟﻁﺎﻟﺏ ﻓﻲ ﺍﻟﺴﻨﻭﺍﺕ ﺍﻟﺘﺎﻟﻴﺔ‪.‬‬
‫ﺘﻀﻤﻥ ﺍﻟﻔﺼل ﺍﻟﺴﺎﺩﺱ ﻤﻭﻀﻭﻋﺎﺕ ﺘﺘﻌﻠﻕ ﺒﺘﺤﺭﻴﻙ ﺍﻟﺠﺴﻡ ﺍﻟﺼﻠﺏ ﻭﺒﻌﺽ‬
‫ﺘﻁﺒﻴﻘﺎﺘﻪ ﺍﻟﺠﻴﺭﻭﺴﻜﻭﺒﻴﺔ‪.‬‬
‫ﺘﻀﻤﻥ ﺍﻟﻔﺼل ﺍﻟﺴﺎﺒﻊ ﺍﻟﻤﺒﺎﺩﻯﺀ ﺍﻟﻌﺎﻤﺔ ﻟﻨﻅﺭﻴﺔ ﺍﻟﻤﺭﻭﻨﺔ ﻭﺒﻌﺽ ﺘﻁﺒﻴﻘﺎﺘﻬﺎ‪.‬‬
‫ﻭﺃﺨﻴﺭﺍﹰ‪ ،‬ﺘﻀﻤﻥ ﺍﻟﻔﺼل ﺍﻟﺜﺎﻤﻥ ﻭﺍﻟﺘﺎﺴﻊ ﻤﻭﻀﻭﻋﺎﺕ ﺘﺘﻌﻠﻕ ﺒﺎﻟﻤﺒﺎﺩﻯﺀ‬
‫ﺍﻷﺴﺎﺴﻴﺔ ﻟﻠﻨﻅﺭﻴﺔ ﺍﻟﻨﺴﺒﻴﺔ ﺍﻟﺨﺎﺼﺔ ﻭﻤﻴﻜﺎﻨﻴﻙ ﺍﻟﻨﻅﺭﻴﺔ ﺍﻟﻨﺴﺒﻴﺔ ﻋﻠﻰ ﺍﻟﺘﻭﺍﻟﻲ‪.‬‬
‫ﻨﺄﻤل ﺃﻥ ﻴﺴﺘﻔﻴﺩ ﻁﻼﺒﻨﺎ ﺍﻷﻋﺯﺍﺀ ﻤﻥ ﻫﺫﺍ ﺍﻟﻜﺘﺎﺏ ﻭﻴﺤﻘﻕ ﺃﻤﺎﻨﻲ ﺍﻟﻁﻠﻴﻌﺔ ﻤﻨﻬﻡ‬
‫ﻓﻲ ﺍﻟﺤﺼﻭل ﻋﻠﻰ ﻤﻌﻠﻭﻤﺎﺕ ﺠﺩﻴﺩﺓ ﻴﺒﺤﺜﻭﻥ ﻋﻨﻬﺎ ﻭﻴﺨﻠﻕ ﻋﻨﺩﻫﻡ ﻤﻠﻜﺔ ﺤل ﻤﺴﺎﺌل‬
‫ﺍﻟﻤﻴﻜﺎﻨﻴﻙ ﻭﺍﻟﺘﻌﺎﻤل ﻤﻌﻬﺎ ﺒﻴﺴﺭ ﻭﺒﺴﻬﻭﻟﺔ‪.‬‬
‫ﺃﺨﻴﺭﺍﹰ‪ ،‬ﻨﺘﻭﺠﻪ ﺇﻟﻰ ﺯﻤﻼﺌﻨﺎ ﺃﻋﻀﺎﺀ ﺍﻟﻬﻴﺌﺔ ﺍﻟﺘﺩﺭﻴﺴﻴﺔ ﻭﺨﺎﺼﺔﹰ ﺃﻭﻟﺌﻙ ﺍﻟﺫﻱ‬
‫ﻴﺘﻌﺎﻤﻠﻭﻥ ﻤﻊ ﻤﻭﺍﻀﻴﻊ ﺍﻟﻤﻴﻜﺎﻨﻴﻙ ﺃﻥ ﻻﻴﺒﺨﻠﻭﺍ ﻋﻠﻨﻴﺎ ﺒﻤﻼﺤﻅﺎﺘﻬﻡ ﺤﻭل ﺍﻟﻜﺘﺎﺏ ﻭﻟﻬﻡ‬
‫ﺠﺯﻴل ﺍﻟﺸﻜﺭ ﻭﺍﻻﻤﺘﻨﺎﻥ‪.‬‬

‫ﺍﳌﺆﻟﻔﻮﻥ‬

‫‪- 10 -‬‬

‫)‪Create PDF files without this message by purchasing novaPDF printer (http://www.novapdf.com‬‬
‫‪ ‬‬

‫ﺃﺳﺲ ﺍﳌﻴﻜﺎﻧﻴﻚ ﺍﻟﺘﺤﻠﻴﻠﻲ‬

‫ﻤﻥ ﺍﻟﻤﻌﻠﻭﻡ ﺃﻥ ﻤﺎﻴﺴﻤﻰ ﻓﻲ ﺍﻟﺭﻴﺎﻀﻴﺎﺕ ﺒﺎﻟﻬﻨﺩﺴﺔ ﺍﻟﺘﺤﻠﻴﻠﻴﺔ ﻴﺴﺘﺨﺩﻡ ﺍﻟﺘﺤﻠﻴل‬


‫ﺍﻟﺭﻴﺎﻀﻲ ﻓﻲ ﺩﺭﺍﺴﺔ ﺍﻟﻬﻨﺩﺴﺔ ﺍﻟﻤﺴﺘﻭﻴﺔ ﻭﺍﻟﻔﺭﺍﻏﻴﺔ ﻓﻬﻭ ﻻﻴﺘﻌﺎﻤل ﻤﻊ ﺍﻟﻤﺴﺘﻘﻴﻡ‬
‫ﻭﺍﻟﺩﺍﺌﺭﺓ ﺃﻭ ﺍﻟﻜﺭﺓ ﺃﻭ ﻤﺠﺴﻡ ﺍﻟﻘﻁﻊ ﺍﻟﻨﺎﻗﺹ ﻭﻏﻴﺭﻫﺎ ﻜﺄﺸﻜﺎل ﻫﻨﺩﺴﻴﺔ ﻓﻘﻁ ﻭﺇﻨﻤﺎ‬
‫ﻴﺩﺭﺴﻬﺎ ﺒﺎﻋﺘﺒﺎﺭﻫﺎ ﺘﺤﻘﻕ ﻤﻌﺎﺩﻻﺕ ﺭﻴﺎﻀﻴﺔ ﻭﻴﺴﺘﻨﺘﺞ ﻜل ﺨﻭﺍﺼﻬﺎ ﺍﻋﺘﻤﺎﺩﺍﹰ ﻋﻠﻰ‬
‫ﺫﻟﻙ‪ .‬ﺃﻻ ﻴﻤﻜﻥ ﺘﻌﻤﻴﻡ ﻫﺫﻩ ﺍﻟﻁﺭﻴﻘﺔ ﻋﻠﻰ ﺍﻟﻤﻴﻜﺎﻨﻴﻙ؟‪.‬‬
‫ﺒﺩﺃ ﻋﻠﻤﺎ ﺍﻟﺭﻴﺎﻀﻴﺎﺕ ﻭﺍﻟﻤﻴﻜﺎﻨﻴﻙ ﺍﻋﺘﺒﺎﺭﺍﹰ ﻤﻥ ﺍﻟﻘﺭﻥ ﺍﻟﺜﺎﻤﻥ ﻋﺸﺭ ﺒﺎﻟﻌﻤل‬
‫ﻋﻠﻰ ﺤل ﻤﺸﺎﻜل ﺍﻟﻤﻴﻜﺎﻨﻴﻙ ﺍﻋﺘﻤﺎﺩﺍﹰ ﻋﻠﻰ ﺍﻟﺘﺤﻠﻴل ﺍﻟﺭﻴﺎﻀﻲ ﻤﻤﺎ ﺴﺎﻫﻡ ﻓﻲ ﻨﺸﻭﺀ‬
‫ﻤﺎﻴﺴﻤﻰ ﺒﺎﻟﻤﻴﻜﺎﻨﻴﻙ ﺍﻟﺘﺤﻠﻴﻠﻲ‪ .‬ﻭﻤﻊ ﺫﻟﻙ ﻟﻡ ﻴﻔﻘﺩ ﻤﻴﻜﺎﻨﻴﻙ ﻨﻴﻭﺘﻥ ﺍﻟﺫﻱ ﺘﻌﺭﻓﻨﺎ ﻋﻠﻴﻪ‬
‫ﻓﻲ ﺍﻟﺴﻨﻭﺍﺕ ﺍﻟﺴﺎﺒﻘﺔ ﻭﺍﻟﺫﻱ ﻴﺭﺘﻜﺯ ﻋﻠﻰ ﺍﻟﻤﺒﺎﺩﻯﺀ ﺍﻟﺜﻼﺜﺔ ﺍﻟﻤﻌﺭﻭﻓﺔ‪ ،‬ﺃﻫﻤﻴﺘﻪ‬
‫ﻭﻻﻨﺴﺘﻁﻴﻊ ﺍﻟﺘﻘﻠﻴل ﻤﻥ ﻤﻨﺠﺯﺍﺘﻪ ﻓﻘﺩ ﺴﺎﻫﻡ ﻓﻲ ﻓﻬﻡ ﻜ ﺜﻴﺭ ﻤﻥ ﺍﻟﻅﻭﺍﻫﺭ ﺍﻟﻤﻴﻜﺎﻨﻴﻜﻴﺔ‬
‫ﻭﺤل ﺍﻟﻤﺸﻜﻼﺕ ﺍﻟﻬﻨﺩﺴﻴﺔ ﺍﻟﻤﺭﺘﺒﻁﺔ ﺒﻌﻠﻡ ﺍﻟﻤﻴﻜﺎﻨﻴﻙ ﻜﺒﻨﺎﺀ ﺍﻟﺠﺴﻭﺭ ﻭﺘﺸﻴﻴﺩ ﺍﻟﻤﺒﺎﻨﻲ‬
‫ﻭﺩﺭﺍﺴﺔ ﺤﺭﻜﺔ ﺍﻟﻘﺫﺍﺌﻑ ﻭﺍﻟﻜﻭﺍﻜﺏ ﺍﻟﺴﻴﺎﺭﺓ ﻭﻏﻴﺭﻫﺎ ﻭﻻﻴﺯﺍل ﺤﺘﻰ ﻭﻗﺘﻨﺎ ﻫﺫﺍ‬
‫ﻤﺭﺠﻌﺎﹰ ﺃﺴﺎﺴﻴﺎﹰ ﻓﻲ ﺤل ﺍﻟﻤﺸﻜﻼﺕ ﺍﻟﻤﺭﺘﺒﻁﺔ ﺒﺎﻟﺼﻭﺍﺭﻴﺦ ﺍﻟﺒﺎﻟﺴﺘﻴﺔ ﻭﺍﻷﻗﻤﺎﺭ‬
‫ﺍﻟﺼﻨﺎﻋﻴﺔ ﻭﺴﻔﻥ ﺍﻟﻔﻀﺎﺀ ﻭﻏﻴﺭﻫﺎ‪.‬‬
‫ﺴﻨﺩﺭﺱ ﻓﻲ ﻫﺫﺍ ﺍﻟﻔﺼل ﺍﻷﺴﺱ ﺍﻟﻌﺎﻤﺔ ﻟﻠﻤﻴﻜﺎﻨﻴﻙ ﺍﻟﺘﺤﻠﻴﻠﻲ ﻓﻨﺫﻜﺭ ﺃﻭﻻﹰ‬
‫ﺒﻌﺽ ﺍﻟﺘﻌﺎﺭﻴﻑ ﺍﻷﺴﺎﺴﻴﺔ ﺍﻟﺘﻲ ﻻﺒﺩ ﻤﻨﻬﺎ ﻜﻲ ﻨﺴﺘﻁﻴﻊ ﻓﻬﻡ ﻭﺍﺴﺘﺨﺩﺍﻡ ﺍﻟﻤﻴﻜﺎﻨﻴﻙ‬
‫ﺍﻟﺘﺤﻠﻴﻠﻲ ﻓﻲ ﺤل ﻤﺴﺎﺌل ﺍﻟﻤﻴﻜﺎﻨﻴﻙ ﻭﺴﻨﺭﻯ ﺃﻨﻪ ﻴﻘﺩﻡ ﻟﻨﺎ ﻁﺭﻴﻘﺔ ﺴﻬﻠﺔ ﻭﺴﻠﺴﺔ‬
‫ﻭﻤﻔﻴﺩﺓ ﻓﻲ ﻫﺫﺍ ﺍﻟﻤﺠﺎل‪.‬‬
‫‪ -1‬ﺍﻻﺭﺗﺒﺎﻃﺎﺕ ) ﺍﻟﻘﻴﻮﺩ ‪:(Constraints‬‬
‫ﺘﻘﺴﻡ ﺍﻟﺠﻤل ﺍﻟﻤﺎﺩﻴﺔ ﺒﺎﻟﻨﺴﺒﺔ ﻟﺘﺄﺜﻴﺭ ﺍﻟﻘﻭﻯ ﻓﻴﻬﺎ ﺇﻟﻰ ﺜﻼﺜﺔ ﺃﻗﺴﺎﻡ‪:‬‬

‫‪- 11 -‬‬

‫)‪Create PDF files without this message by purchasing novaPDF printer (http://www.novapdf.com‬‬
‫ﺃ‪ -‬ﺤﺭﺓ‪ :‬ﻻﺘﺅﺜﺭ ﻋﻠﻴﻬﺎ ﺃﻱ ﻗﻭﻯ ﻤﻁﻠﻘﺎﹰ‪ .‬ﻤﺜﺎل ﺫﻟﻙ ﺴﻔﻴﻨﺔ ﻓﻀﺎﺌﻴﺔ ﺒﻌﻴﺩﺓ ﻋﻥ‬
‫ﺃﻱ ﻤﺠﺎل ﺠﺎﺫﺏ‪ ،‬ﻤﺠﻤﻭﻋﺔ ﺠﺴﻴﻤﺎﺕ ﺨﺎﺭﺝ ﺃﻱ ﻤﺠﺎل ﻗﻭﻯ‪.‬‬
‫ﺏ‪ -‬ﻁﻠﻴﻘﺔ‪ :‬ﺘﺘﺤﺭﻙ ﺘﺤﺕ ﺘﺄﺜﻴﺭ ﻗﻭﻯ ﻓﻌﺎﻟﺔ ﻓﻘﻁ‪.‬‬
‫ﺝ‪ -‬ﻤﻘﻴﺩﺓ‪ :‬ﺘﺅﺜﺭ ﻋﻠﻴﻬﺎ ﺒﺎﻹﻀﺎﻓﺔ ﺇﻟﻰ ﺍﻟﻘﻭﻯ ﺍﻟﻔﻌﺎﻟﺔ ﻗﻭﻯ‪ ،‬ﻨﺎﺘﺠﺔ ﻋﻥ‬
‫ﻤﻼﻤﺴﺘﻬﺎ ﺃﺠﺴﺎﻡ ﻤﺠﺎﻭﺭﺓ‪ ،‬ﺘﺴﻤﻰ ﻗﻭﻯ ﺍﻟﺭﺒﻁ ﻤﺜل ﻗﻭﻯ ﺘﻭﺘﺭ ﺨﻴﻁ ﺃﻭ ﺭﺩ ﻓﻌل‬
‫ﺴﻁﺢ ﺃﻭ ﺴﻠﻙ ﻭﺘﻨﻌﺩﻡ ﻫﺫﻩ ﺍﻟﻘﻭﻯ ﻋﻨﺩﻤﺎ ﺘﺘﺤﺭﺭ ﺍﻟﺠﻤﻠﺔ ﻤﻥ ﺍﻟﻘﻴﺩ ﻓﺘﺼﺒﺢ ﻁﻠﻴﻘﺔ‪.‬‬
‫ﻴﻬﺘﻡ ﺍﻟﻤﻴﻜﺎﻨﻴﻙ ﺍﻟﺘﺤﻠﻴﻠﻲ ﺒﺼﻭﺭﺓ ﺭﺌﻴﺴﻴﺔ ﺒﺩﺭﺍﺴﺔ ﺍﻟﺠﻤل ﺍﻟﻤﻘﻴﺩﺓ ﻭﺴﻨﺭﻯ ﻓﻴﻤﺎ‬
‫ﺒﻌﺩ ﺃﻥ ﻋﺩﺩ ﺍﻟﻤﻌﺎﺩﻻﺕ ﺍﻟﻼﺯﻤﺔ ﻟﻭﺼﻑ ﺍﻟﺠﻤﻠﺔ ﻴﻘل ﻭﺒﺎﻟﺘﺎﻟﻲ ﺘﺼﺒﺢ ﺩﺭﺍﺴﺘﻨﺎ‬
‫ﺃﺴﻬل‪.‬‬
‫ﻜﻴﻑ ﻨﻌﺒﺭ ﻋﻥ ﺍﻟﻘﻴﺩ )ﺍﻻﺭﺘﺒﺎﻁ( ﺍﻟﺫﻱ ﺘﺘﺄﺜﺭ ﻓﻴﻪ ﺍﻟﺠﻤﻠﺔ؟ ﻤﻥ ﺍﻟﻭﺍﻀﺢ ﺃﻨﻪ‬
‫ﻻﺒﺩ ﻤﻥ ﺘﺤﻭﻴﻠﻪ ﺇﻟﻰ ﻤﻌﺎﺩﻟﺔ ﺭﻴﺎﻀﻴﺔ‪.‬‬
‫ﻋﻨﺩﻤﺎ ﺘﺘﺤﺭﻙ ﺍﻟﻨﻘﻁﺔ ﻋﻠﻰ ﺴﻁﺢ ﻓﻼﺒﺩ ﺃﻥ ﺘﺤﻘﻕ ﻤﻌﺎﺩﻟﺔ ﺍﻟﺴﻁﺢ ﻭﻜﺫﻟﻙ‬
‫ﻋﻨﺩﻤﺎ ﺘﺘﺤﺭﻙ ﻋﻠﻰ ﻤﻨﺤﻨﻲ ﻭﻴﻤﻜﻥ ﺘﻌﻤﻴﻡ ﺫﻟﻙ ﻋﻠﻰ ﺃﻱ ﺠﻤﻠﺔ ﻤﺎﺩﻴﺔ‪.‬‬
‫ﻭﻗﺩ ﻴﺘﻌﻠﻕ ﺍﻻﺭﺘﺒﺎﻁ ﺒﺎﻟﺴﺭﻋﺔ ﻭﺍﻟﺯﻤﻥ‪ .‬ﻭﻫﻜﺫﺍ ﻨﻜﺘﺏ ﻤﻌﺎﺩﻟﺔ ﺍﻻﺭﺘﺒﺎﻁ‬
‫ﺒﺼﻭﺭﺓ ﻋﺎﻤﺔ ﻟﺠﻤﻠﺔ ﻤﺎﺩﻴﺔ ﻤﺅﻟﻔﺔ ﻤﻥ ‪ N‬ﻨﻘﻁﺔ ﺒﺎﻟﻌﻼﻗﺔ‪:‬‬
‫)‪f (x 1 , y 1 , z 1 , x 1 , y 1 , z1 ....x N , y N , z N , x N , y N , z N , t )  0 (1.1‬‬

‫ﻭﻟﻨﺄﺨﺫ ﺒﻌﺽ ﺍﻷﻤﺜﻠﺔ‪:‬‬


‫ﻴﺘﺤﺭﻙ ﻋﻠﻰ ﺴﻁﺢ ﻜﺭﺓ‬ ‫) ‪M ( x, y, z‬‬ ‫ﻋﻨﺩﻤﺎ ﺘﺘﺄﻟﻑ ﺍﻟﺠﻤﻠﺔ ﻤﻥ ﺠﺴﻴﻡ ﻭﺍﺤﺩ‬
‫ﻨﺼﻑ ﻗﻁﺭﻫﺎ ‪ a‬ﻓﺈﻥ ﻤﻌﺎﺩﻟﺔ ﺍﻻﺭﺘﺒﺎﻁ ﻫﻲ‪:‬‬
‫‪2‬‬ ‫‪2‬‬ ‫‪2‬‬ ‫‪2‬‬
‫‪x  y z a 0‬‬ ‫)‪(1.2a‬‬
‫ﻭﻗﺩ ﺘﻜﺘﺏ ﺍﻟﻤﻌﺎﺩﻟﺔ ﺍﻟﺴﺎﺒﻘﺔ ﺒﺄﻱ ﺇﺤﺩﺍﺜﻴﺎﺕ ﺃﺨﺭﻯ‪ .‬ﻓﻲ ﺍﻹﺤﺩﺍﺜﻴﺎﺕ ﺍﻟﻜﺭﻭﻴﺔ‬
‫ﻤﺜﻼﹰ‪ ،‬ﻨﻌﺒﺭ ﻋﻥ ﺍﻹﺤﺩﺍﺜﻴﺎﺕ ﺒﺎﻟﻭﺴﻁﺎﺀ )‪ ( r, , ‬ﻭﺘﻭﻀﻊ ﻜﻤﺎ ﻴﻠﻲ‪:‬‬
‫‪ra 0‬‬ ‫)‪(1.2b‬‬
‫ﻭﻋﻨﺩﻤﺎ ﺘﺘﺄﻟﻑ ﺍﻟﺠﻤﻠﺔ ﻤﻥ ‪ N‬ﺠﺴﻴﻡ ﻴﺘﺤﺭﻙ ﻜل ﻤﻨﻬﺎ ﻋﻠﻰ ﺴﻁﺢ ﺍﻟﻜﺭﺓ‬
‫ﺍﻟﺴﺎﺒﻘﺔ ﻓﻴﻤﻜﻥ ﺃﻥ ﻨﻜﺘﺏ ﻤﻌﺎﺩﻻﺕ ﺍﻻﺭﺘﺒﺎﻁ ﻜﻤﺎ ﻴﻠﻲ‪:‬‬
‫‪2‬‬ ‫‪2‬‬ ‫‪2‬‬ ‫‪2‬‬
‫‪x i  y i  x i  a  0 , ri  a  0‬‬ ‫)‪(1.3‬‬

‫‪- 12 -‬‬

‫)‪Create PDF files without this message by purchasing novaPDF printer (http://www.novapdf.com‬‬
‫ﻋﺩﺩ ﻨﻘﻁ ﺍﻟﺠﻤﻠﺔ‪.‬‬ ‫)‪(i  1,2,..., N‬‬ ‫ﺤﻴﺙ‬
‫ﻗﺩ ﺘﺨﻀﻊ ﺍﻟﺠﻤﻠﺔ ﻓﻲ ﺤﺭﻜﺘﻬﺎ ﻟﻌﺩﺓ ﺍﺭﺘﺒﺎﻁﺎﺕ‪ .‬ﻓﻲ ﺍﻟﻤﺜﺎل ﺍﻟﺴﺎﺒﻕ ﻴﻤﻜﻥ ﺃﻥ‬
‫ﻨﺠﺒﺭ ﺍﻟﺠﺴﻴﻡ ﻋﻠﻰ ﺍﻟﺤﺭﻜﺔ ﻋﻠﻰ ﺩﺍﺌﺭﺓ ﺃﻓﻘﻴﺔ ﻋﻠﻰ ﺴﻁﺢ ﺍﻟﻜﺭﺓ ﻭﻋﻨﺩﺌﺫ‪ ‬ﺘﺤﻘﻕ‬
‫ﺇﺤﺩﺍﺜﻴﺎﺘﻪ ﺍﻟﻤﻌﺎﺩﻟﺔ ﺍﻟﺘﺎﻟﻴﺔ‪:‬‬
‫‪  C1‬‬ ‫)‪(1.4‬‬
‫ﻭﻴﻘﺎل ﻓﻲ ﻫﺫﻩ ﺍﻟﺤﺎﻟﺔ ﺃﻥ ﻟﻠﺠﺴﻴﻡ ﺩﺭﺠﺔ ﺤﺭﻴﺔ ﻭﺍﺤﺩﺓ‪ .‬ﻭﻫﻜﺫﺍ ﻨﻌﺭﻑ ﻋﺩﺩ‬
‫ﺩﺭﺠﺎﺕ ﺍﻟﺤﺭﻴﺔ ﺒﺼﻭﺭﺓ ﻋﺎﻤﺔ‪ ،‬ﻭﻫﻭ ﻋﺩﺩ ﺍﻟﻭﺴﻁﺎﺀ ﺍﻟﻤﺴﺘﻘﻠﺔ ﺍﻟﻼﺯﻤﺔ ﻟﺩﺭﺍﺴﺔ‬
‫ﺍﻟﺠﻤﻠﺔ‪ ،‬ﺒﺄﻨﻪ ﻋﺩﺩ ﺍﻹﺤﺩﺍﺜﻴﺎﺕ ﺍﻟﻼﺯﻤﺔ ﻟﻭﺼﻑ ﺍﻟﺠﻤﻠﺔ ﻓﻴﻤﺎ ﻟﻭ ﻜﺎﻨﺕ ﻁﻠﻴﻘﺔ‬
‫ﻤﻁﺭﻭﺤﺎﹰ ﻤﻨﻪ ﻋﺩﺩ ﻤﻌﺎﺩﻻﺕ ﺍﻻﺭﺘﺒﺎﻁ‪ .‬ﻓﺈﺫﺍ ﺘﺄﺜﺭ ﺍﻟﺠﺴﻴﻡ ﺍﻟﻤﺫﻜﻭﺭ ﺴﺎﺒﻘﺎﹰ ﺒﺎﻻﺭﺘﺒﺎﻁ‬
‫ﺍﻟﻤﻌﺒﺭ ﻋﻨﻪ ﺒﺎﻟﻤﻌﺎﺩﻟﺔ )‪ (1.2a‬ﻓﺈﻥ ﻋﺩﺩ ﺩﺭﺠﺎﺕ ﺍﻟﺤﺭﻴﺔ ﻫﻭ ‪ .3-1=2‬ﺃﻤﺎ ﺇﺫﺍ ﺘﺄﺜﺭ‬
‫ﺒﺎﻻﺭﺘﺒﺎﻁﻴﻥ ﺍﻟﻤﻤﺜﻠﻴﻥ ﺒﺎﻟﻤﻌﺎﺩﻟﺘﻴﻥ )‪ (1.2a‬ﻭ )‪ (1.4‬ﻓﻴﺼﺒﺢ ﻫﺫﺍ ﺍﻟﻌﺩﺩ ‪ 3-2=1‬ﺃﻤﺎ ﺇﺫﺍ‬
‫ﺘﺄﺜﺭ ﺍﻟﺠﺴﻴﻡ ﺍﻟﻤﺫﻜﻭﺭ ﺒﺜﻼﺜﺔ ﺍﺭﺘﺒﺎﻁﺎﺕ ﻻﺘﺘﻌﻠﻕ ﺒﺎﻟﺯﻤﻥ ﻓﻼﻴﻭﺠﺩ ﻤﻌﻨﻰ ﻟﻠﻤﺴﺄﻟﺔ ﻷﻥ‬
‫ﺍﻟﺠﺴﻴﻡ ﺴﻴﻜﻭﻥ ﺜﺎﺒﺘﺎﹰ ﻓﻲ ﻫﺫﻩ ﺍﻟﺤﺎﻟﺔ ﻭﻻﻴﺘﺤﺭﻙ‪ .‬ﻤﺜﺎل ﺫﻟﻙ ﺃﻥ ﻨﻀﻊ ﻗﻴﺩﺍﹰ ﺁﺨﺭ‬
‫ﻴﺘﻤﺜل ﺒﺈﺠﺒﺎﺭ ﺍﻟﺠﺴﻴﻡ ﻋﻠﻰ ﺍﻟﺤﺭﻜﺔ ﻋﻠﻰ ﺩﺍﺌﺭﺓ ﺸﺎﻗﻭﻟﻴﺔ ﻤﺭﺴﻭﻤﺔ ﻋﻠﻰ ﺴﻁﺢ ﺍﻟﻜﺭﺓ‬
‫‪   C 2‬ﻭﻋﻨﺩﺌﺫ‪ ‬ﺴﻴﻜﻭﻥ ﻫﺫﺍ ﺍﻟﺠﺴﻴﻡ ﺴﺎﻜﻨﺎﹰ ﺤﺘﻤﺎﹰ ﻓﻲ ﺍﻟﻨﻘﻁﺔ‬ ‫ﻤﻌﺎﺩﻟﺘﻬﺎ‬
‫) ‪ M(a, C1 , C 2‬ﻭﻻﻴﻜﻭﻥ ﻟﻪ ﺃﻱ ﺩﺭﺠﺔ ﺤﺭﻴﺔ‪.‬‬
‫ﻭﻟﺯﻴﺎﺩﺓ ﺍﻹﻴﻀﺎﺡ ﻨﻭﺭﺩ ﺒﻌﺽ ﺍﻷﻤﺜﻠﺔ ﻟﺤﺴﺎﺏ ﻋﺩﺩ ﺩﺭﺠﺎﺕ ﺍﻟﺤﺭﻴﺔ ﻟﺠﻤﻠﺔ‬
‫ﻤﺎﺩﻴﺔ‪:‬‬
‫ﺃ‪ -‬ﺇﻥ ﻋﺩﺩ ﺩﺭﺠﺎﺕ ﺍﻟﺤﺭﻴﺔ ﻟﺠﻤﻠﺔ ﻤﺎﺩﻴﺔ ﻤﺅﻟﻔﺔ ﻤﻥ ‪ N‬ﺠﺴﻴﻡ ﻁﻠﻴﻕ‪ ،‬ﻤﻌﻴﻨﺔ‬
‫ﺒـ ‪ 3N‬ﺍﺤﺩﺍﺜﻲ ﻭﻻﺘﺨﻀﻊ ﻷﻱ ﺍﺭﺘﺒﺎﻁ ﻫﻭ ‪ ،3N‬ﺃﻤﺎ ﺇﺫﺍ ﺘﺄﺜﺭﺕ ﺍﻟﺠﻤﻠﺔ ﺒـ ‪k‬‬
‫ﺍﺭﺘﺒﺎﻁ ﻓﻴﺼﺒﺢ ﺍﻟﻌﺩﺩ ‪.3N-k=s‬‬
‫ﺏ‪ -‬ﻴﻤﻜﻥ ﺍﻟﺒﺭﻫﺎﻥ ﺃﻥ ﺍﻟﺠﺴﻡ ﺍﻟﺼﻠﺏ ﺍﻟﻁﻠﻴﻕ ﻴﺘﻌﻴﻥ ﻓﻲ ﺍﻟﻔﻀﺎﺀ ﺒﻤﻌﺭﻓﺔ‬
‫) ‪M3 (x 3, y3 , z 3‬‬ ‫ﺇﺤﺩﺍﺜﻴﺎﺕ ﺜﻼﺜﺔ ﻨﻘﻁ ﻤﻨﻪ ) ‪، M 2 ( x 2 , y 2 , z 2 ) ، M1 ( x1 , y1 , z1‬‬
‫)ﻭﺴﻁﺎﺀ ﻏﻴﺭ ﻤﺴﺘﻘﻠﺔ( ﻭﻫﻲ ﺘﺭﺘﺒﻁ ﻓﻴﻤﺎ ﺒﻴﻨﻬﺎ ﺒﺜﻼﺙ ﻋﻼﻗﺎﺕ ﺘﻌﺒﺭ ﻋﻥ ﺍﻟﺒﻌﺩ‬
‫ﺍﻟﺜﺎﺒﺕ ﺒﻴﻥ ﻜل ﻨﻘﻁﺘﻴﻥ ﻤﻥ ﺍﻟﺠﺴﻡ ﺍﻟﺼﻠﺏ ﻭﺒﺎﻟﺘﺎﻟﻲ ﻴﻨﺨﻔﺽ ﻋﺩﺩ ﺍﻟﻭﺴﻁﺎﺀ‬

‫‪- 13 -‬‬

‫)‪Create PDF files without this message by purchasing novaPDF printer (http://www.novapdf.com‬‬
‫ﺍﻟﻤﺴﺘﻘﻠﺔ ﺍﻟﻼﺯﻤﺔ ﻟﺩﺭﺍﺴﺘﻪ ﺇﻟﻰ ﺴﺕ ﻓﻘﻁ ﺘﺅﺨﺫ ﻜﻤﺎ ﻴﻠﻲ‪[ x ( c), y( c), z (c )] :‬‬
‫ﺯﻭﺍﻴﺎ ﺃﻭﻟﺭ‪.‬‬ ‫)‪(,  , ‬‬ ‫ﺇﺤﺩﺍﺜﻴﺎﺕ ﻤﺭﻜﺯ ﻜﺘﻠﺘﻪ ‪ c‬ﻭ‬
‫ﻭﻟﻠﺘﺄﻜﺩ ﻤﻥ ﺫﻟﻙ ﻨﻘﻭل ﺃﻥ ﺃﻱ ﻨﻘﻁﺔ ﺃﺨﺭﻯ ‪ M i‬ﻤﻥ ﻫﺫﺍ ﺍﻟﺠﺴﻡ ﺇﺤﺩﺍﺜﻴﺎﺘﻬﺎ‬
‫) ‪ ( x i , y i , z i‬ﺴﻴﻜﻭﻥ ﺒﻌﺩﻫﺎ ﻋﻥ ﻜل ﻤﻥ ﺍﻟﻨﻘﻁ ﺍﻟﺜﻼﺙ ﺍﻟﻤﺫﻜﻭﺭﺓ ﺜﺎﺒﺘﺎﹰ ﻭﻫﺫﺍ ﻴﻌﻁﻲ‬
‫ﺒﺎﻟﻀﺒﻁ ﺜﻼﺙ ﻤﻌﺎﺩﻻﺕ ﻟﺤﺴﺎﺏ ) ‪ ( x i , y i , z i‬ﻭﺒﺎﻟﺘﺎﻟﻲ ﻴﻜﻭﻥ ﻋﺩﺩ ﺩﺭﺠﺎﺕ ﺍﻟﺤﺭﻴﺔ‬
‫ﻟﻬﺫﺍ ﺍﻟﺠﺴﻡ ﺍﻟﺼﻠﺏ ﺍﻟﻁﻠﻴﻕ ﻫﻭ ﺴﺕ ﻓﻘﻁ‪.‬‬
‫ﺝ‪ -‬ﻴﻨﺨﻔﺽ ﻋﺩﺩ ﺩﺭﺠﺎﺕ ﺍﻟﺤﺭﻴﺔ ﺇﻟﻰ ﺜﻼﺙ ﻓﻲ ﺍﻟﻤﺜﺎل ﺍﻟﺴﺎﺒﻕ ﺇﺫﺍ ﺜﺒﺘﺕ‬
‫ﻨﻘﻁﺔ ﻤﻥ ﺍﻟﺠﺴﻡ ﺍﻟﺼﻠﺏ‪ .‬ﻴﻌﺒﺭ ﻋﻥ ﻫﺫﻩ ﺍﻟﺩﺭﺠﺎﺕ ﺍﻟﺜﻼﺜﺔ ﺒﺼﻭﺭﺓ ﻋﺎﻤﺔ‪ ،‬ﺒﺯﻭﺍﻴﺎ‬
‫ﺃﻭﻟﺭ‪.‬‬
‫ﺩ‪ -‬ﻟﻠﺤﺭﻜﺔ ﺍﻟﻤﺴﺘﻭﻴﺔ ﻟﺠﺴﻡ ﺼﻠﺏ ﺜﻼﺙ ﺩﺭﺠﺎﺕ ﺤﺭﻴﺔ ﻫﻲ ﻤﺭﻜﺯ ﻜﺘﻠﺘﻪ‬
‫ﺤﻭل ﻤﺭﻜﺯ ﺍﻟﻜﺘﻠﺔ‪.‬‬ ‫‪‬‬ ‫ﻭﺯﺍﻭﻴﺔ ﺍﻟﺩﻭﺭﺍﻥ‬ ‫)‪x (C), y(C‬‬

‫ﻫـ‪ -‬ﻴﻭﺠﺩ ﺩﺭﺠﺔ ﺤﺭﻴﺔ ﻭﺍﺤﺩﺓ ﻟﺠﺴﻡ ﺼﻠﺏ ﻴﺩﻭﺭ ﺤﻭل ﻤﺤﻭﺭ ﺜﺎﺒﺕ ﻫﻲ‬
‫ﺯﺍﻭﻴﺔ ﺍﻟﺩﻭﺭﺍﻥ ‪.‬‬
‫ﻨﻌﺭﻑ ﺃﺨﻴﺭ ﺍﹰ ﺍﻻﺭﺘﺒﺎﻁ ﺍﻟﺘﺤﻠﻴﻠﻲ ﻭﻨﻘﻭل ﺇﻨﻪ ﺍﻻﺭﺘﺒﺎﻁ ﺍﻟﺫﻱ ﻻﻴﺘﻌﻠﻕ ﺒﻘﺎﻨﻭﻥ‬
‫ﺍﻟﺤﺭﻜﺔ‪ ،‬ﻭﻻﻴﻜﻭﻥ ﺍﻻﺭﺘﺒﺎﻁ ﺘﺤﻠﻴﻠﻴﺎﹰ ﺨﻼﻓﺎﹰ ﻟﺫﻟﻙ‪ ،‬ﻓﻤﺜﻼﹰ ﻻﻴﻜﻭﻥ ﺍﺭﺘﺒﺎﻁ ﺠﺴﻡ ﻤﻌﻠﻕ‬
‫ﺒﻨﺎﺒﺽ ﺘﺤﻠﻴﻠﻴﺎﹰ ﻷﻥ ﻫﺫﺍ ﺍﻻﺭﺘﺒﺎﻁ ﻴﺤﺩﺩ ﻤﻘﺩﻤﺎﹰ ﻗﺎﻨﻭﻥ ﺍﻟﺤﺭﻜﺔ ﺍﻟﺠﻴﺒﻲ ﻟﻠﺠﺴﻡ‪ ،‬ﺃﻤﺎ‬
‫ﺍﻟﺠﺴﻡ ﺍﻟﻤﻭﺠﻭﺩ ﻋﻠﻰ ﺴﻁﺢ ﻓﻴﻤﻜﻥ ﺃﻥ ﻴﺘﺤﺭﻙ ﺒﺄﻴﺔ ﺤﺭﻜﺔ ﺘﺒﻌﺎﹰ ﻟﻠﻘﻭﻯ ﺍﻟﻤﺅﺜﺭﺓ‬
‫ﻋﻠﻴﻪ‪.‬‬
‫‪ -2‬ﺃﻧﻮﺍﻉ ﺍﻻﺭﺗﺒﺎﻃﺎﺕ‪:‬‬
‫ﻴﻤﻜﻥ ﺍﻟﺘﻤﻴﻴﺯ ﺒﻴﻥ ﻋﺩﺓ ﺃﻨﻭﺍﻉ ﻤﻥ ﺍﻻﺭﺘﺒﺎﻁﺎﺕ ﻨﻭﺠﺯﻫﺎ ﻓﻴﻤﺎﻴﻠﻲ‪:‬‬
‫ﺃ‪ -‬ﻴﻘﺴﻡ ﺍﻻﺭﺘﺒﺎﻁ ﻤﻥ ﻨﺎﺤﻴﺔ ﻭﺠﻭﺩ ﺍﻟﺘﻔﺎﻀل ﺇﻟﻰ ﻗﺴﻤﻴﻥ‪:‬‬
‫‪ -‬ﺍﺭﺘﺒﺎﻁ ﻫﻨﺩﺴﻲ‪ :‬ﻻﻴﺤﻭﻱ ﺃﻱ ﺘﻔﺎﻀﻼﺕ‪.‬‬
‫‪ -‬ﺍﺭﺘﺒﺎﻁ ﺘﻔﺎﻀﻠﻲ‪ :‬ﻭﻫﻭ ﺍﻻﺭﺘﺒﺎﻁ ﺍﻟﺫﻱ ﺘﺤﻭﻱ ﻤﻌﺎﺩﻟﺘﻪ ﺍﺸﺘﻘﺎﻗﺎﺕ ﻭﺒﺎﻟﺘﺎﻟﻲ‬
‫ﻓﻬﻭ ﻴﻀﻊ ﻗﻴﻭﺩﺍﹰ ﻋﻠﻰ ﺍﻟﺴﺭﻋﺔ‪.‬‬

‫‪- 14 -‬‬

‫)‪Create PDF files without this message by purchasing novaPDF printer (http://www.novapdf.com‬‬
‫ﻭﻫﻜﺫﺍ ﺘﻜﻭﻥ ﺍﻻﺭﺘﺒﺎﻁﺎﺕ ﺍﻟﻤﻌﺒﺭ ﻋﻨﻬﺎ ﺒﺎﻟﻌﻼﻗﺎﺕ )‪(1.4)،(1.2a)،(1.2‬‬
‫ﻫﻨﺩﺴﻴﺔ‪ .‬ﺃﻤﺎ ﺇﺫﺍ ﻓﺭﻀﻨﺎ ﺃﻥ ﺠﺴﻴﻤﺎﹰ ﻴﺘﺤﺭﻙ ﻓﻲ ﺍﻟﻔﻀﺎﺀ ﻭﻫﻭ ﻴﺨﻀﻊ ﻟﻤﻌﺎﺩﻟﺔ‬
‫ﺍﻻﺭﺘﺒﺎﻁ‪:‬‬
‫‪2‬‬ ‫‪2‬‬ ‫‪2‬‬
‫‪x‬‬ ‫‪y‬‬ ‫‪a 0‬‬ ‫)‪(1.5a‬‬
‫ﺃﻭ ﺍﺭﺘﺒﺎﻁ ﻤﻌﺎﺩﻟﺘﻪ‪:‬‬
‫‪‬‬
‫‪  Const.‬‬ ‫)‪(1.5b‬‬
‫ﻓﻴﺘﻡ ﺍﻋﺘﺒﺎﺭ ﺍﻻﺭﺘﺒﺎﻁﻴﻥ ﺍﻟﺴﺎﺒﻘﻴﻥ ﺤﺭﻜﻴﻴﻥ‪.‬‬
‫ﺏ‪ -‬ﻴﻘﺴﻡ ﺍﻻﺭﺘﺒﺎﻁ ﺒﺎﻟﻨﺴﺒﺔ ﻟﻭﺠﻭﺩ ﺍﻟﺯﻤﻥ ﻓﻴﻪ ﺇﻟﻰ ﻗﺴﻤﻴﻥ‪:‬‬
‫‪ -‬ﺍﺭﺘﺒﺎﻁ ﻤﺴﺘﻘﺭ )ﺜﺎﺒﺕ(‪ :‬ﻻﻴﺤﻭﻱ ﺍﻟﺯﻤﻥ ﺒﺸﻜل ﺼﺭﻴﺢ‪.‬‬
‫‪ -‬ﺍﺭﺘﺒﺎﻁ ﻤﺘﺤﻭل )ﻏﻴﺭ ﻤﺴﺘﻘﺭ(‪ :‬ﻴﺤﻭﻱ ﺍﻟﺯﻤﻥ ﺒﺸﻜل ﺼﺭﻴﺢ‪.‬‬
‫ﻫﺫﺍ ﻤﻊ ﺍﻟﻌﻠﻡ ﺃﻥ ﻜل ﺍﻻﺭﺘﺒﺎﻁﺎﺕ ﻻﺒﺩ ﺃﻥ ﺘﺤﻭﻱ ﺍﻟﺯﻤﻥ ﻤﻥ ﺨﻼل‬
‫ﺍﻻﺤﺩﺍﺜﻴﺎﺕ ﻭﺨﻼﻑ ﺫﻟﻙ ﻴﻜﻭﻥ ﺍﻟﻤﺘﺤﺭﻙ ﺴﺎﻜﻨﺎﹰ‪ .‬ﻭﻟﻜﻥ ﻗﺩ ﻴﻅﻬﺭ ﺒﺸﻜل ﺼﺭﻴﺢ ﺃﻭ‬
‫ﻴﻜﻭﻥ ﺘﺎﺒﻌﺎﹰ ﻀﻤﻨﻴﺎﹰ ﻟﻠﺯﻤﻥ‪ .‬ﻭﻫﻜﺫﺍ ﺘﻜﻭﻥ ﻜل ﺍﻻﺭﺘﺒﺎﻁﺎﺕ )‪ (1.2‬ﻭ )‪ (1.4‬ﻤﺴﺘﻘﺭﺓ‪.‬‬
‫ﺃﻤﺎ ﺇﺫﺍ ﻓﺭﻀﻨﺎ ﻨﻭﺍﺴﺎﹰ ﺒﺴﻴﻁﺎﹰ ﺸﻜل )‪ (1.1‬ﻴﻤﺭ ﺨﻴﻁ ﺘﻌﻠﻴﻘﻪ ﻋﻠﻰ ﺒﻜﺭﺓ‬
‫ﻭﻴﺴﺤﺏ ﺍﻟﺨﻴﻁ ﺒﺴﺭﻋﺔ ﻤﻨﺘﻅﻤﺔ )‪ (v‬ﻓﺈﻥ ﻤﻌﺎﺩﻟﺔ ﺍﺭﺘﺒﺎﻁﻪ ﻫﻲ‪:‬‬
‫‪   0  vt‬‬ ‫)‪(1.6‬‬
‫ﺤﻴﺙ ‪  0‬ﻁﻭل ﺍﻟﺨﻴﻁ ﺍﻟﻜﻠﻲ‪ ،‬ﻫﻭ ﺍﺭﺘﺒﺎﻁ ﻤﺘﺤﻭل ﻭﻴﻤﻜﻥ ﺃﻥ ﻴﻘﺎل ﺍﻟﺸﻲﺀ‬
‫ﻨﻔﺴﻪ ﻋﻥ ﺍﻟﺜﻘل ﺍﻟﻤﻌﻠﻕ ﺒﻁﺭﻑ ﺤﺒل ﺭﺍﻓﻌﺔ ﺘﻨﻘﻠﻪ ﻤﻥ ﻤﻜﺎﻥ ﺇﻟﻰ ﺁﺨﺭ‪.‬‬

‫ﺍﻟﺸﻜل )‪(1.1‬‬

‫‪- 15 -‬‬

‫)‪Create PDF files without this message by purchasing novaPDF printer (http://www.novapdf.com‬‬
‫ﺝ‪ -‬ﻴﻘﺴﻡ ﺍﻻﺭﺘﺒﺎﻁ ﺒﺎﻟﻨﺴﺒﺔ ﻹﻤﻜﺎﻨﻴﺔ ﺘﺤﺭﺭﻩ ﻤﻥ ﺍﻟﻘﻴﺩ ﺇﻟﻰ ﻗﺴﻤﻴﻥ‪:‬‬
‫‪ -‬ﺍﺭﺘﺒﺎﻁ ﻤﺤﺭﺭ ) ﺃﺤﺎﺩﻱ ﺍﻟﺠﺎﻨﺏ(‪ :‬ﻴﻤﻜﻥ ﻟﻠﻤﺘﺤﺭﻙ ﺃﻥ ﻴﺘﺤﺭﺭ ﻤﻥ ﺍﻟﻘﻴﺩ‪.‬‬
‫ﻤﺜﻼﹰ ﺠﺴﻴﻡ ﻴﺘﺤﺭﻙ ﻋﻠﻰ ﺴﻁﺢ ﻓﻬﻭ ﻤﺭﺘﺒﻁ ﺒﺎﻟﺴﻁﺢ ﻭﻴﺤﻘﻕ ﻤﻌﺎﺩﻟﺘﻪ ﻁﺎﻟﻤﺎ‬
‫ﺒﻘﻲ ﻤﺘﺤﺭﻜﺎﹰ ﻋﻠﻴﻪ ﻭﻟﻜﻥ ﻋﻨﺩﻤﺎ ﻴﺘﺤﺭﺭ ﻤﻥ ﺍﻟﺴﻁﺢ ﻴﺼﺒﺢ ﻁﻠﻴﻘﺎﹰ ﻭﻻﻴﺨﻀﻊ ﻷﻱ‬
‫ﺍﺭﺘﺒﺎﻁ‪.‬‬
‫‪ -‬ﺍﺭﺘﺒﺎﻁ ﻤﻘﻴﺩ )ﺜﻨﺎﺌﻲ ﺍﻟﺠﺎﻨﺏ(‪ :‬ﻻﻴﻤﻜﻥ ﻟﻠﻤﺘﺤﺭﻙ ﺃﻥ ﻴﺘﺤﺭﺭ ﻤﻥ ﺍﻟﻘﻴﺩ ﻤﺜﺎل‬
‫ﺫﻟﻙ ﺤﻠﻘﺔ ﻤﻌﺩﻨﻴﺔ ﺘﺘﺤﺭﻙ ﻋﻠﻰ ﺴﻠﻙ ﻤﻌﻘﻭﻑ ﻤﻊ ﻁﺭﻓﻴﻪ ﺒﺤﻴﺙ ﻻﻴﺴﺘﻁﻴﻊ ﻤﻐﺎﺩﺭﺘﻪ‪.‬‬
‫ﺩ‪ -‬ﺃﺨﻴﺭﺍﹰ ﻴﻘﺴﻡ ﺍﻻﺭﺘﺒﺎﻁ ﺍﻟﺘﻔﺎﻀﻠﻲ ﺇﻟﻰ ﻗﺴﻤﻴﻥ‪:‬‬
‫‪ -‬ﻫﻭﻟﻭﻨﻭﻤﻲ )‪ :(Holonomic‬ﻭﻫﻭ ﺍﺭﺘﺒﺎﻁ ﺘﻔﺎﻀﻠﻲ ﻴﻤﻜﻥ ﺍﺴﺘﻜﻤﺎﻟﻪ‬
‫ﻭﺒﺎﻟﺘﺎﻟﻲ ﻴﺅﻭل ﺇﻟﻰ ﺍﺭﺘﺒﺎﻁ ﻫﻨﺩﺴﻲ‪ ،‬ﻓﺎﻻﺭﺘﺒﺎﻁ ﺍﻟﻤﻌﺒﺭ ﻋﻨﻪ ﺒﺎﻟﻤﻌﺎﺩﻟﺔ )‪ (1.5b‬ﻴﻤﻜﻥ‬
‫ﺍﺴﺘﻜﻤﺎﻟﻪ ﻭﺒﺎﻟﺘﺎﻟﻲ ﻓﻬﻭ ﻫﻭﻟﻭﻨﻭﻤﻲ‪.‬‬
‫‪ -‬ﻻﻫﻭﻟﻭﻨﻭﻤﻲ )‪ :(Nonholonomic‬ﻭﻫﻭ ﺍﺭﺘﺒﺎﻁ ﺘﻔﺎﻀﻠﻲ ﻻﻴﻤﻜﻥ‬
‫ﺍﺴﺘﻜﻤﺎﻟﻪ ﻭﺒﺎﻟﺘﺎﻟﻲ ﻻﻴﺅﻭل ﺇﻟﻰ ﺍﺭﺘﺒﺎﻁ ﻫﻨﺩﺴﻲ ﻭﻻﺒﺩ ﻤﻥ ﺍﻟﺘﻌﺎﻤل ﻤﻌﻪ ﻋﻠﻰ ﻫﺫﺍ‬
‫ﺍﻷﺴﺎﺱ‪ ،‬ﻤﺜﺎل ﺫﻟﻙ ﺍﻻﺭﺘﺒﺎﻁ )‪.(1.5a‬‬
‫ﻟﻨﺄﺨﺫ ﻤﺜﺎﻻﹰ ﺁﺨﺭ ﻋﻠﻰ ﺍﻻﺭﺘﺒﺎﻁ ﺍﻟﻬﻭﻟﻭﻨﻭﻤﻲ ﻓﻨﻔﺭﺽ ﺩﻭﻻﺒﺎﹰ ﻴﺘﺩﺤﺭﺝ ﻋﻠﻰ‬
‫ﻁﺭﻴﻕ ﺃﻓﻘﻴﺔ ﺒﺤﻴﺙ ﻴﺒﻘﻰ ﻤﻼﺯﻤﺎﹰ ﻟﻠﻁﺭﻴﻕ ) ﺍﺭﺘﺒﺎﻁ ﺜﻨﺎﺌﻲ ﺍﻟﺠﺎﻨﺏ( ﺸﻜل )‪.(1.2‬‬
‫ﻴﺘﻌﻴﻥ ﻤﺭﻜﺯ ﺍﻟﺩﻭﻻﺏ ﺒﺎﻻﺤﺩﺍﺜﻴﻴﻥ )‪ y(c‬ﻭ )‪.x(c‬‬

‫ﺍﻟﺸﻜل )‪(1.2‬‬

‫‪- 16 -‬‬

‫)‪Create PDF files without this message by purchasing novaPDF printer (http://www.novapdf.com‬‬
‫ﻤﻥ ﺍﻟﻭﺍﻀﺢ ﺃﻨﻪ ﻴﻤﻜﻥ ﻜﺘﺎﺒﺔ ﻤﻌﺎﺩﻟﺔ ﺍﺭﺘﺒﺎﻁ ﺍﻟﺩﻭﻻﺏ ﺒﺎﻟﻁﺭﻴﻕ ﻜﻤﺎﻴﻠﻲ‪:‬‬
‫‪y  (c )  0‬‬ ‫)‪(1.7a‬‬
‫ﻭﻫﻭ ﺍﺭﺘﺒﺎﻁ ﺘﻔﺎﻀﻠﻲ ﻴﻤﻜﻥ ﺍﺴﺘﻜﻤﺎﻟﻪ ﻟﻴﺼﺒﺢ‪:‬‬
‫‪y(c )  Const.‬‬ ‫)‪(1.7b‬‬
‫ﻭﺒﺎﻟﺘﺎﻟﻲ ﻴﺅﻭل ﺇﻟﻰ ﺍﺭﺘﺒﺎﻁ ﻫﻨﺩﺴﻲ‪.‬‬
‫ﺍﻟﻤﻌﺎﺩﻟﺘﺎﻥ )‪ (1.7a,b‬ﻤﺘﻜﺎﻓﺌﺘﺎﻥ ﻭﻴﻤﻜﻥ ﺃﻥ ﻨﺄﺨﺫ ﺃﻴﺎﹰ ﻤﻨﻬﻤﺎ‪.‬‬
‫ﻤﻥ ﺍﻟﻤﻔﻴﺩ ﻓﻲ ﻨﻬﺎﻴﺔ ﻫﺫﻩ ﺍﻟﻔﻘﺭﺓ ﺃﻥ ﻨﺫﻜﺭ ﻨﻭﻋﺎﹰ ﺁﺨﺭ ﻤﻥ ﺍﻻﺭﺘﺒﺎﻁ ﻴﻌﺒﺭ ﻋﻨﻪ‬
‫ﺒﻤﺘﺭﺍﺠﺤﺔ ﻓﺈﺫﺍ ﻓﺭﻀﻨﺎ ﻤﺠﻤﻭﻋﺔ ﺠﺴﻴﻤﺎﺕ ﺘﺘﺤﺭﻙ ﻋﻠﻰ ﺍﻟﺴﻁﺢ ﺍﻟﺨﺎﺭﺠﻲ ﻟﻜﺭﺓ‬
‫ﻨﺼﻑ ﻗﻁﺭﻫﺎ )‪ (a‬ﻓﻴﻤﻜﻥ ﻜﺘﺎﺒﺔ ﻤﻌﺎﺩﻻﺕ ﺍﺭﺘﺒﺎﻁﻬﺎ‪:‬‬
‫‪ri  a‬‬ ‫)‪(1.8a‬‬
‫ﺃﻤﺎ ﺇﺫﺍ ﺘﺤﺭﻜﺕ ﺍﻟﺠﺴﻴﻤﺎﺕ ﻋﻠﻰ ﺍﻟﺴﻁﺢ ﺍﻟﺩﺍﺨﻠﻲ ﻟﻠﻜﺭﺓ ﻤﻤﻜﻥ ﺃﻥ ﻨﻀﻊ‬
‫ﻤﻌﺎﺩﻻﺕ ﺍﻻﺭﺘﺒﺎﻁ‪:‬‬
‫‪ri  a‬‬ ‫)‪(1.8b‬‬
‫ﻋﺩﺩ ﺍﻟﺠﺴﻴﻤﺎﺕ ﻋﻠﻰ ﺍﻟﻜﺭﺓ‪.‬‬ ‫)‪(i  1,2,..., N‬‬ ‫ﺤﻴﺙ‬
‫‪ -3‬ﺍﻻﻧﺘﻘﺎﻝ ﺍﻻﻓﱰﺍﺿﻲ ‪:‬‬
‫ﻨﻜﻤل ﻓﻲ ﻫﺫﻩ ﺍﻟﻔﻘﺭﺓ ﻭﺍﻟﺘﻲ ﺘﻠﻴﻬﺎ ﺸﺭﺡ ﺍﻟﺘﻌﺎﺭﻴﻑ ﻭﺍﻟﻤﻔﺎﻫﻴﻡ ﺍﻟﻀﺭﻭﺭﻴﺔ‬
‫ﻟﺩﺭﺍﺴﺔ ﺘﻭﺍﺯﻥ ﺍﻟﺠﻤل ﺍﻟﻤﺎﺩﻴﺔ ﺍﻋﺘﻤﺎﺩﺍﹰ ﻋﻠﻰ ﺍﻟﻤﻴﻜﺎﻨﻴﻙ ﺍﻟﺘﺤﻠﻴﻠﻲ ﻓﻨﻌﺭﻑ ﺃﻭﻻﹰ‬
‫‪‬‬
‫ﺍﻻﻨﺘﻘﺎل ﺍﻻﻓﺘﺭﺍﻀﻲ ﻭﻨﻘﻭل ﺃﻨﻪ ﺍﻨﺘﻘﺎل ﻨﺭﻤﺯ ﻟﻪ ‪) r‬ﺘﻤﻴﻴﺯﺍﹰ ﻋﻥ ﺍﻻﻨﺘﻘﺎل ﺍﻟﺤﻘﻴﻘﻲ‬
‫‪‬‬
‫‪ ( dr‬ﻴﺤﺩﺙ ﺒﺜﺒﺎﺕ ﺍﻟﺯﻤﻥ‪.‬‬
‫ﻟﻨﺒﺤﺙ ﻋﻥ ﺍﻟﻌﻼﻗﺔ ﺍﻟﺘﻲ ﻴﺤﻘﻘﻬﺎ ﻫﺫﺍ ﺍﻻﻨﺘﻘﺎل ﻓﻨﻔﺭﺽ ﺠﺴﻴﻤﺎﹰ ﻴﺘﺤﺭﻙ ﻋﻠﻰ‬
‫ﺴﻁﺢ ﻤﻌﺎﺩﻟﺘﻪ )ﻤﻌﺎﺩﻟﺔ ﺍﻻﺭﺘﺒﺎﻁ(‪:‬‬
‫‪f (x, y, z , t )  C‬‬ ‫)‪(1.9‬‬
‫‪‬‬
‫ﻭﻟﻨﻔﺭﺽ ﺍﻨﺘﻘﺎﻻﹰ ﺍﻓﺘﺭﺍﻀﻴﺎﹰ ﺍﺨﺘﻴﺎﺭﻴﺎﹰ ‪ r‬ﻤﺭﻜﺒﺎﺘﻪ ) ‪ (x , y, z‬ﻴﺤﺩﺙ ﻓﻲ‬
‫ﺍﻟﻠﺤﻅﺔ ‪ t‬ﺒﺤﻴﺙ ﻴﺒﻘﻰ ﺍﻟﺠﺴﻴﻡ ﺨﺎﻀﻌﺎﹰ ﻟﻼﺭﺘﺒﺎﻁ )‪ (1.9‬ﻭﺒﺎﻟﺘﺎﻟﻲ ﺘﺘﺤﻘﻕ ﺍﻟﻌﻼﻗﺔ‪:‬‬
‫‪f ( x  x, y  y, z  z , t )  C‬‬ ‫)‪(1.10a‬‬
‫ﻭﺒﺎﻟﻨﺸﺭ ﻨﺠﺩ‪:‬‬

‫‪- 17 -‬‬

‫)‪Create PDF files without this message by purchasing novaPDF printer (http://www.novapdf.com‬‬
‫‪f‬‬ ‫‪f‬‬ ‫‪f‬‬
‫‪f ( x, y, z ) ‬‬ ‫‪x ‬‬ ‫‪y ‬‬ ‫‪z  C‬‬ ‫)‪(1.10b‬‬
‫‪x‬‬ ‫‪y‬‬ ‫‪z‬‬
‫ﻭﻤﻨﻪ‪:‬‬
‫‪f‬‬ ‫‪f‬‬ ‫‪f‬‬ ‫‪‬‬
‫‪x  y  z  f .d r  0‬‬ ‫)‪(1.11a‬‬
‫‪x‬‬ ‫‪y‬‬ ‫‪z‬‬
‫‪‬‬
‫ﺃﻱ ﺃﻥ ‪ r‬ﻴﺘﻌﺎﻤﺩ ﻤﻊ ﺍﻟﻨﺎﻅﻡ ﻋﻠﻰ ﺍﻟﺴﻁﺢ ﺍﻟﺫﻱ ﻤﻌﺎﺩﻟﺘﻪ )‪.(1.9‬‬
‫ﺃﻤﺎ ﻋﻨﺩﻤﺎ ﻴﻜﻭﻥ ﺍﻻﺭﺘﺒﺎﻁ ﻤﺤﺭﺭﺍﹰ ﺤﻴﺙ ﻴﻤﻜﻥ ﻟﻠﺠﺴﻴﻡ ﺃﻥ ﻴﻨﻔﻙ ﻋﻥ ﺍﻟﺴﻁﺢ‬
‫ﻭﻻﺘﺘﺤﻘﻕ ﺍﻟﻌﻼﻗﺔ )‪ (1.11a‬ﻭﻴﺴﺎﻭﻱ ﻋﻨﺩﺌﺫ‪ ‬ﺍﻟﻁﺭﻑ ﺍﻟﺜﺎﻨﻲ ﻤﻥ )‪(1.10b‬‬
‫‪ C  C‬ﻭﺒﺎﻟﺘﺎﻟﻲ ﻨﺠﺩ ﻋﻭﻀﺎﹰ ﻋﻥ )‪ (1.11a‬ﺍﻟﻌﻼﻗﺔ ﺍﻟﺘﺎﻟﻴﺔ‪:‬‬
‫‪‬‬
‫‪f . r  C‬‬ ‫)‪(1.11b‬‬
‫‪‬‬ ‫‪‬‬
‫ﺃﻤﺎ ﺍﻟﻌﻼﻗﺔ ﺍﻟﺘﻲ ﻴﺤﻘﻘﻬﺎ ﺍﻻﻨﺘﻘﺎل ﺍﻟﺤﻘﻴﻘﻲ ‪ d r‬ﺍﻟﺫﻱ ﻴﺤﺩﺙ ﺒﻔﻌل ﺍﻟﻘﻭﺓ ‪F‬‬
‫ﺍﻟﻤﺅﺜﺭﺓ ﻋﻠﻰ ﺍﻟﺠﺴﻴﻡ‪ ،‬ﻓﺘﺨﺘﻠﻑ ﻋﻤﺎ ﺴﺒﻕ‪ .‬ﻭﻤﻥ ﺍﻟﺴﻬل ﺃﻥ ﻨﻨﺸﺭ )‪ (1.10a‬ﺒﻔﺭﺽ‬
‫‪‬‬ ‫‪‬‬
‫ﺃﻥ ﺍﻻﻨﺘﻘﺎل ‪ d r‬ﻭﻟﻴﺱ ‪ . r‬ﻓﻨﺠﺩ ﺃﺨﻴﺭ ﺍﹰ‪.‬‬
‫‪f‬‬ ‫‪f‬‬ ‫‪f‬‬ ‫‪f‬‬
‫‪dx  dy  dz  dt  0‬‬ ‫)‪(1.12a‬‬
‫‪x‬‬ ‫‪y‬‬ ‫‪z‬‬ ‫‪t‬‬
‫ﺃﻭ‪:‬‬
‫‪‬‬ ‫‪f‬‬
‫‪.d r ‬‬ ‫‪dt  0‬‬ ‫)‪(1.12b‬‬
‫‪t‬‬
‫ﻭﻫﻲ ﺘﺨﺘﻠﻑ ﻋﻥ )‪ (1.11a‬ﻭﻟﻜﻥ ﻴﻤﻜﻥ ﺃﻥ ﺘﺘﻁﺎﺒﻕ ﺍﻟﻌﻼﻗﺘﺎﻥ ﻋﻨﺩﻤﺎ‬
‫ﻻﺘﺤﻭﻱ ﻤﻌﺎﺩﻟﺔ ﺍﻻﺭﺘﺒﺎﻁ )‪ (1.9‬ﺍﻟﺯﻤﻥ ﺒﺸﻜل ﺼﺭﻴﺢ ﻭﺒﺎﻟﺘﺎﻟﻲ ﻴﻤﻜﻥ ﺃﻥ ﻨﺠﺩ‬
‫‪‬‬ ‫‪‬‬
‫ﺍﻨﺘﻘﺎﻻﹰ ﺍﻓﺘﺭﺍﻀﻴﺎﹰ ‪ r‬ﻴﻨﻁﺒﻕ ﻋﻠﻰ ﺍﻨﺘﻘﺎل ﺤﻘﻴﻘﻲ ‪. d r‬‬
‫ﻨﻌﻤﻡ ﺍﻟﻌﻼﻗﺎﺕ ﺍﻟﺴﺎﺒﻘﺔ ﻓﻨﻔﺭﺽ ﺠﻤﻠﺔ ﻤﺎﺩﻴﺔ ﻤﺅﻟﻔﺔ ﻤﻥ ‪ N‬ﺠﺴﻴﻡ ﺍﺤﺩﺍﺜﻴﺎﺕ‬
‫ﻜل ﻤﻨﻬﺎ ) ‪ (i  1,..., N ) ( x i , y i , z i‬ﺘﺘﺄﺜﺭ ﺒـ ‪ k‬ﺍﺭﺘﺒﺎﻁ ﻫﻭﻟﻭﻨﻭﻤﻲ ﺜﻨﺎﺌﻲ ﺍﻟﺠﺎﻨﺏ‬
‫ﻴﻌﺒﺭ ﻋﻨﻬﺎ ﺒﺎﻟﻤﻌﺎﺩﻻﺕ‪:‬‬
‫‪f   (.... x i , y i , z i ,...., t )  C ‬‬ ‫)‪(  1,2,..., k‬‬ ‫)‪(1.13‬‬
‫‪‬‬
‫ﻭﻟﻨﻌﻁ ﻟﻠﺠﺴﻴﻡ ) ‪ M i (x i , y i , z i‬ﺍﻨﺘﻘﺎﻻﹰ ﺍﻓﺘﺭﺍﻀﻴﺎﹰ ‪ ri‬ﻓﻨﺠﺩ‪:‬‬

‫‪- 18 -‬‬

‫)‪Create PDF files without this message by purchasing novaPDF printer (http://www.novapdf.com‬‬
‫‪f  (...x i  x i , y i  y i , z i  z i ,..., t )  0‬‬ ‫)‪(1.14‬‬
‫ﻭﺒﺎﻟﻨﺸﺭ ﻭﺍﻻﺨﺘﺼﺎﺭ ﻨﺠﺩ‪:‬‬
‫‪N‬‬ ‫‪‬‬
‫‪  i f  . ri‬‬ ‫‪0‬‬ ‫)‪(1.15‬‬
‫‪i 0‬‬
‫ﻭﻫﻲ ﻋﺒﺎﺭﺓ ﻋﻥ ‪ k‬ﻋﻼﻗﺔ ﺘﺤﻘﻘﻬﺎ ﺍﻻﻨﺘﻘﺎﻻﺕ ﺍﻻﻓﺘﺭﺍﻀﻴﺔ ﻟﻠﺠﺴﻴﻤﺎﺕ‪ .‬ﺃﻤﺎ‬
‫ﺍﻻﻨﺘﻘﺎﻻﺕ ﺍﻟﺤﻘﻴﻘﻴﺔ ﻓﺘﺤﻘﻕ ﺍﻟﻌﻼﻗﺎﺕ‪:‬‬
‫‪N‬‬ ‫‪‬‬ ‫‪f‬‬
‫‪  i .f  .d r1 ‬‬ ‫‪t‬‬
‫‪dt  0‬‬ ‫)‪(1.16‬‬
‫‪i 1‬‬
‫ﺃﻤﺎ ﻋﻨﺩﻤﺎ ﻴﻜﻭﻥ ﺍﻻﺭﺘﺒﺎﻁ ﻤﺤﺭﺭﺍﹰ ﻓﻴﺠﺏ ﺃﻥ ﻨﻀﻴﻑ ﺇﻟﻰ ﻁﺭﻓﻲ ﺍﻟﻌﻼﻗﺘﻴﻥ‬
‫)‪ (1.15‬ﻭ )‪ C  (1.16‬ﻭ ‪ dC ‬ﻤﻥ ﺍﻟﺠﻬﺔ ﺍﻟﻴﻤﻨﻰ‪ ،‬ﻋﻠﻰ ﺍﻟﺘﺭﺘﻴﺏ‪.‬‬
‫ﻤﻥ ﺍﻟﻤﻔﻴﺩ‪ ،‬ﻗﺒل ﺨﺘﺎﻡ ﻫﺫﻩ ﺍﻟﻔﻘﺭﺓ‪ ،‬ﺃﻥ ﻨﺫﻜﺭ ﻁﺭﻴﻘﺔ ﺤﺴﺎﺏ ﺍﻻﻨﺘﻘﺎل‬
‫ﺍﻻﻓﺘﺭﺍﻀﻲ ﺍﻟﺘﻲ ﻨﺤﺘﺎﺠﻬﺎ ﻓﻲ ﺤل ﺍﻟﻤﺴﺎﺌل‪ ،‬ﻓﻨﻘﻭل ﺃﻨﻪ ﻴﺤﺴﺏ ﻜﺎﻟﺘﻔﺎﻀل ﺍﻟﺤﻘﻴﻘﻲ‬
‫ﺘﻤﺎﻤﺎﹰ ﻭﻟﻜﻥ ﺒﻭﻀﻊ ‪ ‬ﺒﺩﻻﹰ ﻤﻥ ‪ d‬ﻭﺍﻻﻨﺘﺒﺎﻩ ﺇﻟﻰ ﺃﻨﻪ ﻴﺤﺩﺙ ﺒﺜﺒﺎﺕ ﺍﻟﺯﻤﻥ ﻓﺈﺫﺍ ﻜﺎﻥ‬
‫ﻟﺩﻴﻨﺎ ﺍﻟﺘﺎﺒﻊ‪:‬‬
‫) ‪z  f ( x, y, t‬‬ ‫)‪(1.17‬‬
‫ﻓﺈﻥ‪:‬‬
‫‪f‬‬ ‫‪f‬‬
‫‪z ‬‬ ‫‪x  y‬‬ ‫)‪(1.18‬‬
‫‪x‬‬ ‫‪y‬‬

‫ﻭﻟﺘﻭﻀﻴﺢ ﻤﻔﻬﻭﻡ ﺍﻻﻨﺘﻘﺎل ﺍﻻﻓﺘﺭﺍﻀﻲ ﻭﺍﻻﻨﺘﻘﺎل ﺍﻟﺤﻘﻴﻘﻲ ﻨﻭﺭﺩ ﺍﻟﻤﺜﺎل ﺍﻟﺘﺎﻟﻲ‪:‬‬


‫ﺘﺘﺤﺭﻙ ﻨﻘﻁﺔ ﻤﺎﺩﻴﺔ )ﺤﻠﻘﺔ( ﻋﻠﻰ ﻤﺤﻭﺭ ∆ ﻴﺩﻭﺭ ﺒﺴﺭﻋﺔ ﺯﺍﻭﻴﺔ ﺜﺎﺒﺘﺔ ً ﺤﻭل‬
‫ﺍﻟﻤﺤﻭﺭ ﺍﻟﺸﺎﻗﻭﻟﻲ ﺍﻟﺜﺎﺒﺕ ‪ oz‬ﺍﻟﻤﺘﻌﺎﻤﺩ ﻤﻊ ﻤﺴﺘﻭﻱ ﺍﻟﺸﻜل )‪ .(1.3‬ﺍﻜﺘﺏ ﻤﻌﺎﺩﻻﺕ‬
‫ﺍﻻﺭﺘﺒﺎﻁ ﻭﻋﻴﻥ ﻜل ﻤﻥ ﺍﻻﻨﺘﻘﺎﻟﻴﻥ ﺍﻻﻓﺘﺭﺍﻀﻲ ﻭﺍﻟﺤﻘﻴﻘﻲ‪.‬‬

‫‪- 19 -‬‬

‫)‪Create PDF files without this message by purchasing novaPDF printer (http://www.novapdf.com‬‬
‫ﺍﻟﺸﻜل )‪(1.3‬‬
‫ﺍﳊﻞ ‪:‬‬
‫ﻨﺤﺩﺩ ﻤﻭﻀﻊ ﺍﻟﺤﻠﻘﺔ ‪ M‬ﻓﻲ ﺍﻟﻔﻀﺎﺀ ﺒﺎﻻﺤﺩﺍﺜﻴﺎﺕ ﺍﻻﺴﻁﻭﺍﻨﻴﺔ ) ‪. M (, , z‬‬
‫ﻴﺠﺏ ﺃﻥ ﺘﻌﺒﺭ ﻤﻌﺎﺩﻟﺔ ﺍﻻﺭﺘﺒﺎﻁ ﺍﻷﻭﻟﻰ ﻋﻥ ﺃﻥ ﺍﻟﺤﻠﻘﺔ ﺘﺒﻘﻰ ﻓﻲ ﺍﻟﻤﺴﺘﻭﻱ‬
‫‪ xoy‬ﻭﺒﺎﻟﺘﺎﻟﻲ ﻴﻜﻭﻥ‪:‬‬
‫‪z=0‬‬
‫ﺃﻤﺎ ﺍﻟﻤﻌﺎﺩﻟﺔ ﺍﻟﺜﺎﻨﻴﺔ ﻓﻴﺠﺏ ﺃﻥ ﺘﺅﻜﺩ ﺃﻥ ‪ M‬ﺘﺒﻘﻰ ﻋﻠﻰ ﺍﻟﻤﺤﻭﺭ ∆ ﻓﻬﻲ‬
‫ﺘﺤﻘﻕ ﻤﻌﺎﺩﻟﺘﻪ ﺃﻱ‪:‬‬
‫‪  t  0‬‬
‫‪‬‬
‫ﻭﺃﻤﺎ ﺍﻻﻨﺘﻘﺎل ﺍﻻﻓﺘﺭﺍﻀﻲ ‪  r‬ﻓﻤﺭﻜﺒﺎﺘﻪ ‪  ,  , z‬ﻭﻫﻲ ﻜﻤﺎ ﻴﻠﻲ‪:‬‬
‫‪  0 , z  0 ,   0‬‬
‫‪‬‬
‫‪ d r‬ﺴﻴﺨﺘﻠﻑ ﻋﻥ ﺍﻻﻓﺘﺭﺍﻀﻲ‪ ،‬ﻷﻥ ﻤﻌﺎﺩﻟﺔ‬ ‫ﻭﻟﻜﻥ ﺍﻻﻨﺘﻘﺎل ﺍﻟﺤﻘﻴﻘﻲ‬
‫ﺍﻻﺭﺘﺒﺎﻁ ﺍﻟﺜﺎﺒﺘﺔ ﺘﺘﺒﻊ ﺍﻟﺯﻤﻥ ﺒﺸﻜل ﺼﺭﻴﺢ‪ ،‬ﻭﻫﻭ‪:‬‬
‫‪d  dt  0 , dz  0 , d  0‬‬
‫‪‬‬
‫ﻭﺒﺎﻟﺘﺎﻟﻲ ﻴﻜﻭﻥ ﻟﻪ ﻤﺭﻜﺒﺘﺎﻥ ﺍﻷﻭﻟﻰ ﻋﻠﻰ ‪) e ‬ﻤﺘﺠﻪ ﺍﻟﻭﺍﺤﺩﺓ ﺍﻟﻤﻘﺎﺒل ﻟﺘﺯﺍﻴﺩ‬
‫‪‬‬
‫‪ (‬ﻭﻫﻲ ‪ d‬ﻭﺍﻟﺜﺎﻨﻴﺔ ﻋﻠﻰ ‪) e ‬ﻤﺘﺠﺔ ﺍﻟﻭﺍﺤﺩﺓ ﺍﻟﻤﻘﺎﺒل ﻟﺘﺯﺍﻴﺩ ‪ (‬ﻭﻫﻲ ‪. d‬‬
‫‪‬‬ ‫‪‬‬
‫ﻭﻫﻜﺫﺍ ﻴﻤﻜﻥ ﺭﺴﻡ ﻜل ﻤﻥ ‪ r‬ﻭ ‪) dr‬ﻤﺘﺠﻪ ﺍﻟﻭﺍﺤﺩﺓ ﺍﻟﻤﻘﺎﺒل ﻟﺘﺯﺍﻴﺩ ‪ (‬ﻋﻠﻰ‬

‫‪- 20 -‬‬

‫)‪Create PDF files without this message by purchasing novaPDF printer (http://www.novapdf.com‬‬
‫ﺍﻟﺸﻜل )‪ (1.3‬ﻭﻫﻨﺎ ﻴﻜﻭﻥ ﻋﺩﺩ ﺩﺭﺠﺎﺕ ﺍﻟﺤﺭﻴﺔ ‪ 3-2=1‬ﻭﻴﻜﻔﻲ ﺤﺴﺎﺏ ) ‪( t‬‬
‫ﻟﻤﻌﺭﻓﺔ ﻗﺎﻨﻭﻥ ﺍﻟﺤﺭﻜﺔ‪.‬‬
‫‪ -4‬ﺍﻟﻌﻤﻞ ﺍﻻﻓﱰﺍﺿﻲ – ﺍﻻﺭﺗﺒﺎﻁ ﺍﳌﺜﺎﱄ‪:‬‬
‫‪‬‬
‫ﻨﻌﺭﻑ ﺍﻟﻌﻤل ﺍﻻﻓﺘﺭﺍﻀﻲ ﺍﻟﻌﻨﺼﺭﻱ‪ ،‬ﻜﻤﺎ ﻋﺭﻓﻨﺎ ﺍﻟﻌﻤل ﺍﻟﺤﻘﻴﻘﻲ ﻟﻠﻘﻭﺓ ‪F‬‬
‫ﺍﻟﻤﺅﺜﺭﺓ ﻓﻲ ﺠﺴﻴﻡ ) ‪ ، M (x, y, z‬ﺒﺎﻟﻌﻼﻗﺔ‪:‬‬
‫‪ ‬‬
‫‪A  F . r  Fx x  Fy y  Fz z‬‬ ‫)‪(1.19‬‬
‫‪‬‬
‫‪dr‬‬ ‫ﻭﻫﻲ ﺘﺘﻁﺎﺒﻕ ﻤﻊ ﻋﺒﺎﺭﺓ ﺍﻟﻌﻤل ﺍﻟﻌﻨﺼﺭﻱ ﺍﻟﺤﻘﻴﻘﻲ ﺒﻌﺩ ﺘﺒﺩﻴل‬
‫‪‬‬
‫ﺒـ ‪ .  r‬ﻭﻋﻠﻰ ﻀﻭﺀ ﺫﻟﻙ ﻨﺴﺘﻁﻴﻊ ﺃﻥ ﻨﻜﺘﺏ ﺍﻟﻌﻤل ﺍﻻﻓﺘﺭﺍﻀﻲ ﺍﻟﻌﻨﺼﺭﻱ ﻟﻠﻘﻭﺓ‬
‫‪‬‬
‫‪ F‬ﺍﻟﻤﺅﺜﺭﺓ ﻓﻲ ﺍﻟﺤﻠﻘﺔ ‪ M‬ﻓﻲ ﺍﻟﻤﺜﺎل ﺍﻟﺴﺎﺒﻕ‪:‬‬
‫‪‬‬
‫‪A ( F )  F .‬‬ ‫)‪(1.20‬‬
‫ﺃﻤﺎ ﺍﻟﻌﻤل ﺍﻟﻌﻨﺼﺭﻱ ﺍﻟﺤﻘﻴﻘﻲ ﻓﻬﻭ‪:‬‬
‫‪‬‬ ‫‪‬‬ ‫‪‬‬
‫‪dA ( F )  F .d r  F d  F d  Fd  F  dt‬‬ ‫)‪(1.21‬‬
‫ﺃﻤﺎ ﺍﻟﻌﻤل ﺍﻟﻌﻨﺼﺭﻱ ﺍﻻﻓﺘﺭﺍﻀﻲ ﻟﺭﺩ ﺍﻟﻔﻌل ﻓﻬﻭ‪:‬‬
‫‪‬‬ ‫‪‬‬ ‫‪‬‬
‫‪A ( R )  R . r‬‬ ‫)‪(1.22‬‬
‫ﻨﻌﺭﻑ ﺃﺨﻴﺭ ﺍﹰ ﺍﻻﺭﺘﺒﺎﻁ ﺍﻟﻤﺜﺎﻟﻲ ﻓﻨﻘﻭل ﺇﻨﻪ ﺍﻻﺭﺘﺒﺎﻁ ﺍﻟﺫﻱ ﻴﻜﻭﻥ ﻓﻴﻪ ﺍﻟﻌﻤل‬
‫ﺍﻻﻓﺘﺭﺍﻀﻲ ﺍﻟﻌﻨﺼﺭﻱ ﻟﺭﺩ ﺍﻟﻔﻌل ﻴﺴﺎﻭﻱ ﺍﻟﺼﻔﺭ‪.‬‬
‫ﺃﻣﺜﻠﺔ‪:‬‬
‫ﺃ‪ -‬ﺇﺫﺍ ﻻﺯﻤﺕ ﻨﻘﻁﺔ ﻤﺎﺩﻴﺔ ﺴﻁﺤﺎﹰ ﺼﻘﻴﻼﹰ ﻴﻜﻭﻥ ﺭﺩ ﺍﻟﻔﻌل ﻨﺎﻅﻤﻲ ﺩﻭﻤﺎﹰ ﻋﻠﻰ‬
‫ﺍﻟﺴﻁﺢ ﺃﻤﺎ ﺍﻻﻨﺘﻘﺎل ﺍﻻﻓﺘﺭﺍﻀﻲ ﻓﻘﺩ ﺭﺃﻴﻨﺎ ﺃﻨﻪ ﻤﺘﻌﺎﻤﺩ ﻤﻊ ﺍﻟﺘﺩﺭﺝ ﺃﻱ ﻤﺘﻌﺎﻤﺩ ﻤﻊ‬
‫‪‬‬ ‫‪‬‬
‫ﺍﻟﻨﺎﻅﻡ ﻭﺒﺎﻟﺘﺎﻟﻲ ﻴﻨﻌﺩﻡ ﺍﻟﻤﻘﺩﺍﺭ ‪ R . r‬ﻭﻫﻜﺫﺍ ﻴﻜﻭﻥ ﺍﻻﺭﺘﺒﺎﻁ ﻤﺜﺎﻟﻴﺎﹰ ﻓﻲ ﻫﺫﻩ ﺍﻟﺤﺎﻟﺔ‪.‬‬
‫ﺃﻤﺎ ﺇﺫﺍ ﻟﻡ ﻴﻜﻥ ﺍﻟﺴﻁﺢ ﺼﻘﻴﻼﹰ ﻓﻼﺒﺩ ﻤﻥ ﻭﺠﻭﺩ ﻤﺭﻜﺒﺔ ﻤﻤﺎﺴﻴﺔ ﻟﺭﺩ ﺍﻟﻔﻌل ﻻﻴﻨﻌﺩﻡ‬
‫ﻋﻤﻠﻬﺎ ﺍﻻﻓﺘﺭﺍﻀﻲ ﻭﺒﺎﻟﺘﺎﻟﻲ ﻻﻴﻤﺜل ﺍﺭﺘﺒﺎﻁ ﺍﻟﻨﻘﻁﺔ )ﺍﻟﺠﺴﻴﻡ( ﺒﺎﻟﺴﻁﺢ ﺍﻟﺨﺸﻥ‬
‫ﺍﺭﺘﺒﺎﻁﺎﹰ ﻤﺜﺎﻟﻴﺎﹰ‪.‬‬

‫‪- 21 -‬‬

‫)‪Create PDF files without this message by purchasing novaPDF printer (http://www.novapdf.com‬‬
‫ﻭﻨﻼﺤﻅ ﺃﻥ ﺍﻟﻌﻤل ﺍﻟﺤﻘﻴﻘﻲ ﻫﻨﺎ ﻴﺴﺎﻭﻱ ﺍﻟﺼﻔﺭ ﺃﻴﻀﺎﹰ ﻋﻨﺩﻤﺎ ﻴﻜﻭﻥ ﺍﻟﺴﻁﺢ‬
‫ﺼﻘﻴﻼﹰ ﻭﻟﻜﻨﻪ ﻻﻴﺴﺎﻭﻱ ﺍﻟﺼﻔﺭ ﻋﻨﺩﻤﺎ ﻴﻜﻭﻥ ﺍﻻﺭﺘﺒﺎﻁ ﻤﺘﺤﻭﻻﹰ ﻭﺍﻟﻤﺜﺎل ﺍﻟﺘﺎﻟﻲ‬
‫ﻴﻭﻀﺢ ﺫﻟﻙ‪.‬‬
‫ﺏ ‪ -‬ﻴﺘﺤﺭﻙ ﻤﺴﺘﻭﻱ ﺼﻘﻴل ﺒﺎﺘﺠﺎﻩ ﺍﻟﻤﺤﻭﺭ ‪ oz‬ﺍﻟﺸﺎﻗﻭﻟﻲ ﺒﺴﺭﻋﺔ ﺜﺎﺒﺘﺔ ‪u‬‬
‫ﺒﻴﻨﻤﺎ ﻴﺘﺤﺭﻙ ﻋﻠﻰ ﺍﻟﻤﺴﺘﻭﻱ ﺠﺴﻴﻡ‪ .‬ﺃﻭﺠﺩ ﺍﻟﻌﻤل ﺍﻻﻓﺘﺭﺍﻀﻲ ﻭﺍﻟﺤﻘﻴﻘﻲ ﻟﺭﺩ ﺍﻟﻔﻌل‪.‬‬
‫ﺇﻥ ﻤﻌﺎﺩﻟﺔ ﺍﻻﺭﺘﺒﺎﻁ ﻫﻲ‪:‬‬
‫‪z  ut  z  ut  0‬‬
‫‪z  0 , dz  u.dt  0‬‬
‫ﻭﺒﻤﺎ ﺃﻥ ﺭﺩ ﺍﻟﻔﻌل ﻴﺘﺠﻪ ﺒﺎﺘﺠﺎﻩ ‪ oz‬ﻴﻜﻭﻥ ﺍﻟﻌﻤل ﺍﻻﻓﺘﺭﺍﻀﻲ ﺍﻟﻌﻨﺼﺭﻱ‪:‬‬
‫‪ ‬‬
‫‪A  R . r  Rz  0‬‬
‫ﺃﻤﺎ ﺍﻟﻌﻤل ﺍﻟﺤﻘﻴﻘﻲ ﺍﻟﻌﻨﺼﺭﻱ ﻟﺭﺩ ﺍﻟﻔﻌل‪:‬‬
‫‪ ‬‬
‫‪dA  R . dr  R .dz  Rudt  0‬‬ ‫)‪(1.23‬‬
‫ﺝ‪ -‬ﻫل ﻴﻌﺘﺒﺭ ﺍﻻﺭﺘﺒﺎﻁ ﺍﻟﺩﻭﻻﺏ )ﻋﻠﻰ ﺍﻟﺸﻜل ‪ (1.2‬ﺒﺎﻟﻁﺭﻴﻕ ﻤﺜﺎﻟﻴﺎﹰ؟‬
‫ﻴﻭﺠﺩ ﺭﺩ ﻓﻌل ﻓﻲ ﺍﻟﻨﻘﻁﺔ ‪ p‬ﺤﻴﺙ ﻴﻼﻤﺱ ﺍﻟﺩﻭﻻﺏ ﺍﻟﻁﺭﻴﻕ ﻭﻫﻭ ﻤﺅﻟﻑ ﻤﻥ‬
‫ﻤﺭﻜﺒﺘﻴﻥ ﻨﺎﻅﻤﻴﺔ ‪ N‬ﻭﻤﻤﺎﺴﻴﺔ ‪) T‬ﻗﻭﺓ ﺍﻻﺤﺘﻜﺎﻙ( ﺘﺅﻤﻥ ﺘﺩﺤﺭﺝ ﺍﻟﺩﻭﻻﺏ )ﺭﺍﺠﻊ‬
‫ﺍﻟﺸﻜل ‪.(1.2‬‬
‫ﺃﻤﺎ ﻤﻌﺎﺩﻟﺔ ﺍﻻﺭﺘﺒﺎﻁ ﻓﻬﻲ ‪ y(c )  a‬ﺤﻴﺙ ‪ a‬ﻫﻲ ﻨﺼﻑ ﻗﻁﺭ ﺍﻟﺩﻭﻻﺏ‬
‫ﺃﻤﺎﻋﻤل ﺭﺩ ﺍﻟﻔﻌل ﻓﻬﻭ‪:‬‬
‫‪ ‬‬
‫‪A  R . r  TPT  NPN‬‬ ‫)‪(1.24‬‬
‫ﻭﻁﺎﻟﻤﺎ ﺃﻥ ﺍﻟﺩﻭﻻﺏ ﻴﺒﻘﻰ ﻤﻼﺯﻤﺎﹰ ﻟﻠﻁﺭﻴﻕ ﻓﺈﻥ ‪ . PN  0‬ﻭﻜﺫﻟﻙ‬
‫‪ PT  0‬ﻷﻥ ﺍﻟﻨﻘﻁﺔ ‪ P‬ﻫﻲ ﺍﻟﻤﺭﻜﺯ ﺍﻵﻨﻲ ﻟﻠﺩﻭﺭﺍﻥ ﻭﺒﺎﻟﺘﺎﻟﻲ ﺘﻨﻌﺩﻡ ﺴﺭﻋﺘﻬﺎ‪.‬‬
‫ﻭﻫﻜﺫﺍ ﻴﻜﻭﻥ ﺍﻟﻌﻤل ﺍﻻﻓﺘﺭﺍﻀﻲ ﻟﺭﺩ ﺍﻟﻔﻌل ﻤﻌﺩﻭﻤﺎﹰ ﻭﺍﻻﺭﺘﺒﺎﻁ ﻤﺜﺎﻟﻴﺎﹰ‪ .‬ﻭﺍﻟﺠﺩﻴﺭ‬
‫ﺒﺎﻟﺫﻜﺭ ﺃﻥ ﻫﺫﺍ ﺍﻻﺭﺘﺒﺎﻁ ﻟﻥ ﻴﻜﻭﻥ ﻤﺜﺎﻟﻴﺎﹰ ﺇﺫﺍ ﺤﺩﺙ ﺍﻨﺯﻻﻕ ﻤﻊ ﺍﻟﺘﺩﺤﺭﺝ ﻷﻥ ﺍﻟﻨﻘﻁﺔ‬
‫‪ P‬ﺘﻜﺘﺴﺏ ﺴﺭﻋﺔ ﺃﻓﻘﻴﺔ ﺃﺜﻨﺎﺀ ﺍﻟﺘﺯﺤﻠﻕ ﻭﺒﺎﻟﺘﺎﻟﻲ ‪. PT  0‬‬

‫‪- 22 -‬‬

‫)‪Create PDF files without this message by purchasing novaPDF printer (http://www.novapdf.com‬‬
‫ﺩ‪ -‬ﻋﺎﺭﻀﺔ ﺘﺭﺘﻜﺯ ﻤﻥ ﻤﻨﺘﺼﻔﻬﺎ ﻋﻠﻰ ﻨﻘﻁﺔ ﺜﺎﺒﺘﺔ‪ .‬ﻫل ﺍﻻﺭﺘﺒﺎﻁ ﻤﺜﺎﻟﻲ؟‬
‫ﺸﻜل )‪.(1.4‬‬

‫ﺍﻟﺸﻜل )‪(1.4‬‬
‫ﻭﺒﻤﺎ ﺃﻥ ‪ ro  0‬ﻷﻥ ‪ o‬ﺘﺒﻘﻰ ﺜﺎﺒﺘﺔ ﻓﺭﻀﺎﹰ ﻭﺒﺎﻟﺘﺎﻟﻲ ﻴﻜﻭﻥ ﺍﻻﺭﺘﺒﺎﻁ ﻤﺜﺎﻟﻴﺎﹰ‪.‬‬
‫ﻫـ ‪ -‬ﺘﺘﺩﺤﺭﺝ ﻋﺎﺭﻀﺔ ﺩﻭﻥ ﺍﻨﺯﻻﻕ ﻋﻠﻰ ﺴﻁﺢ ﻜﺭﺓ ﺜﺎﺒﺘﺔ ﺸﻜل )‪.(1.5‬‬
‫‪ -‬ﻫل ﺍﻻﺭﺘﺒﺎﻁ ﻤﺜﺎﻟﻲ؟‬
‫‪ -‬ﻤﺎﻫﻭ ﻋﺩﺩ ﺍﻟﻭﺴﻁﺎﺀ ﺍﻟﻤﺴﺘﻘﻠﺔ )ﻋﺩﺩ ﺩﺭﺠﺎﺕ ﺍﻟﺤﺭﻴﺔ(؟‬
‫ﻤﻥ ﺍﻟﻭﺍﻀﺢ ﺃﻥ ﺴﺭﻋﺔ ﺍﻟﻨﻘﻁﺔ ‪ ،P‬ﺤﻴﺙ ﻴﺅﺜﺭ ﺭﺩ ﺍﻟﻔﻌل‪ ،‬ﺘﺴﺎﻭﻱ ﺍﻟﺼﻔﺭ‬
‫ﻷﻥ ﺍﻟﺘﺩﺤﺭﺝ ﻴﺘﻡ ﺩﻭﻥ ﺍﻨﺯﻻﻕ ﻭﺒﺎﻟﺘﺎﻟﻲ ﻴﻜﻭﻥ ﺍﻟﻌﻤل ﺍﻟﻌﻨﺼﺭﻱ ﻟﺭﺩ ﺍﻟﻔﻌل‪:‬‬
‫‪ ‬‬
‫‪A  R . P  0‬‬ ‫)‪(1.26‬‬
‫ﻓﺎﻻﺭﺘﺒﺎﻁ ﻤﺜﺎﻟﻲ‪.‬‬

‫ﺍﻟﺸﻜل )‪(1.5‬‬

‫‪- 23 -‬‬

‫)‪Create PDF files without this message by purchasing novaPDF printer (http://www.novapdf.com‬‬
‫ﻟﻘﺩ ﺭﺃﻴﻨﺎ ﺃﻥ ﻟﻠﺠﺴﻡ ﺍﻟﺼﻠﺏ ﺍﻟﺫﻱ ﻴﺘﺤﺭﻙ ﺤﺭﻜﺔ ﻤﺴﺘﻭﻴﺔ ﺜﻼﺙ ﺩﺭﺠﺎﺕ‬
‫ﺤﺭﻴﺔ ﻓﻲ ﺍﻟﺤﺎﻟﺔ ﺍﻟﻌﺎﻤﺔ‪ .‬ﺃﺜﻨﺘﺎﻥ ﻟﻤﺭﻜﺯ ﻜﺘﻠﺘﻪ )‪ x (C), y(C‬ﻭﺜﺎﻟﺜﺔ ‪ ‬ﻟﺩﻭﺭﺍﻨﻪ‪ .‬ﻭﻓﻲ‬
‫ﺩﻭﺭﺍﻥ ﺍﻟﻌﺎﺭﻀﺔ ﺒﻭﺍﺴﻁﺔ ﺍﻟﺯﺍﻭﻴﺔ ‪ ‬ﺒﻴﻥ ﺍﻟﻤﺤﻭﺭ‬ ‫ﺤﺎﻟﺘﻨﺎ ﻫﺫﻩ ﻴﻤﻜﻥ ﻤﻌﺭﻓﺔ‬
‫ﺍﻟﺸﺎﻗﻭﻟﻲ ﻭﻨﺼﻑ ﺍﻟﻘﻁﺭ ﺍﻟﻤﺘﻌﺎﻤﺩ ﻤﻊ ﻤﺴﺘﻭﻱ ﺍﻟﻌﺎﺭﻀﺔ ﺍﻟﻤﺎﺭ ﻤﻥ ﻤﺭﻜﺯ ﻜﺘﻠﺘﻬﺎ‪.‬‬
‫ﺃﻤﺎ ﺇﺤﺩﺍﺜﻴﺎﺕ ﻤﺭﻜﺯ ﻜﺘﻠﺘﻬﺎ ﻓﻴﺘﻌﻴﻥ ﺒﺎﻟﺯﺍﻭﻴﺔ ﻨﻔﺴﻬﺎ ‪ ‬ﻭﻟﺒﺭﻫﺎﻥ ﺫﻟﻙ ﻨﻼﺤﻅ ﺃﻥ‪:‬‬
‫‪‬‬ ‫‪‬‬ ‫‪‬‬ ‫‪‬‬
‫‪OC  OP  PC' C' C‬‬ ‫)‪(1.27a‬‬
‫ﻓﺈﺫﺍ ﻓﺭﻀﻨﺎ ﺃﻨﻪ ﻓﻲ ﺒﺩﺍﻴﺔ ﺍﻟﺤﺭﻜﺔ ﺍﻨﻁﺒﺎﻕ '‪ C‬ﻋﻠﻰ ﺫﺭﻭﺓ ﺍﻟﻜﺭﺓ ﻓﺈﻥ‬
‫‪ PC'  a‬ﺤﻴﺙ ‪ a‬ﻨﺼﻑ ﻗﻁﺭ ﺍﻟﻜﺭﺓ‪ ،‬ﻭﻓﺭﻀﻨﺎ ﺃﻴﻀﺎﹰ ﺃﻥ ﺴﻤﺎﻜﺔ ﺍﻟﻌﺎﺭﻀﺔ ‪2h‬‬
‫ﻓﻴﻤﻜﻥ ﺃﻥ ﻨﺠﺩ ﺒﺈﺴﻘﺎﻁ )‪ (1.27a‬ﻋﻠﻰ ﺍﻟﻤﺤﻭﺭﻴﻥ ‪ oy , ox‬ﺍﻟﻤﻭﻀﺤﻴﻥ ﻋﻠﻰ‬
‫ﺍﻟﺸﻜل )‪ (1.5‬ﻤﺎﻴﻠﻲ‪:‬‬
‫‪x (C)  a sin   a cos   h sin  ‬‬
‫‪‬‬
‫‪‬‬ ‫)‪(1.27b‬‬
‫‪y(C)  a cos   a sin   h cos ‬‬
‫ﻭﺒﺎﻟﺘﺎﻟﻲ ﻴﻜﻭﻥ ﻟﻠﻤﺴﺄﻟﺔ ﺩﺭﺠﺔ ﺤﺭﻴﺔ ﻭﺍﺤﺩﺓ ﻫﻭ ‪) ‬ﻭﺴﻴﻁ ﻭﺍﺤﺩ ﻤﺴﺘﻘل(‪.‬‬
‫‪ -5‬ﺍﳌﺒﺪﺃ ﺍﻟﺘﻮﺍﺯﻧﻲ ﻟﻼﻧﺘﻘﺎﻻﺕ ﺍﻻﻓﱰﺍﺿﻴﺔ‪:‬‬
‫ﻴﻤﻜﻨﻨﺎ ﺍﻵﻥ ﻋﻠﻰ ﻀﻭﺀ ﺍﻟﻔﻘﺭﺍﺕ ﺍﻷﺭﺒﻊ ﺍﻟﺴﺎﺒﻘﺔ‪ ،‬ﺩﺭﺍﺴﺔ ﺘﻭﺍﺯﻥ ﺍﻟﺠﻤل‬
‫ﺍﻟﻤﺎﺩﻴﺔ ﺍﻋﺘﻤﺎﺩﺍﹰ ﻋﻠﻰ ﺍﻟﻤﻴﻜﺎﻨﻴﻙ ﺍﻟﺘﺤﻠﻴﻠﻲ‪ ،‬ﻟﻬﺫﺍ ﻨﺒﺭﻫﻥ ﺃﻭﻻﹰ ﺍﻟﻨﻅﺭﻴﺔ ﺍﻷﺴﺎﺴﻴﺔ‬
‫ﺍﻟﺘﺎﻟﻴﺔ‪:‬‬
‫‪ -‬ﺍﻟﺸﺭﻁ ﺍﻟﻼﺯﻡ ﻭﺍﻟﻜﺎﻓﻲ ﻟﺘﻭﺍﺯﻥ ﺠﻤﻠﺔ ﻤﺎﺩﻴﺔ ﺨﺎﻀﻌﺔ ﻻﺭﺘﺒﺎﻁﺎﺕ ﻤﺜﺎﻟﻴﺔ‬
‫ﺜﻨﺎﺌﻴﺔ ﺍﻟﺠﺎﻨﺏ ﻭﻤﺴﺘﻘﺭﺓ ﻫﻭ ﺃﻥ ﻴﻜﻭﻥ ﻤﺠﻤﻭﻉ ﺍﻷﻋﻤﺎل ﺍﻻﻓﺘﺭﺍﻀﻴﺔ ﺍﻟﺠﺯﺌﻴﺔ ﻟﻠﻘﻭﻯ‬
‫ﺍﻟﻔﻌﺎﻟﺔ ﻴﺴﺎﻭﻱ ﺍﻟﺼﻔﺭ ﻭﺒﺎﻟﺘﺎﻟﻲ ﺘﺘﺤﻘﻕ ﺍﻟﻌﻼﻗﺔ‪:‬‬
‫‪N  ‬‬
‫‪A   Fi . ri  0‬‬ ‫)‪(1.28‬‬
‫‪i 1‬‬

‫‪- 24 -‬‬

‫)‪Create PDF files without this message by purchasing novaPDF printer (http://www.novapdf.com‬‬
‫ﺍﻟﱪﻫﺎﻥ‪:‬‬
‫ﺃ‪ -‬ﻟﺯﻭﻡ ﺍﻟﺸﺭﻁ‪:‬‬
‫‪N  ‬‬
‫‪. Fi .ri‬‬
‫‪‬‬ ‫ﻨﻔﺭﺽ ﺃﻥ ﺍﻟﺠﻤﻠﺔ ﻤﺘﻭﺍﺯﻨﺔ ﻭﺒﺎﻟﺘﺎﻟﻲ ﻴﺠﺏ ﺍﻟﺒﺭﻫﺎﻥ ﺃﻥ ‪ 0‬‬
‫‪i 1‬‬
‫ﺇﻥ ﺘﻭﺍﺯﻥ ﺍﻟﺠﻤﻠﺔ ﻜﻜل ﻴﻌﻨﻲ ﺃﻥ ﻜل ﻨﻘﻁﺔ ‪ M i‬ﻤﻥ ﻨﻘﻁﻬﺎ ﺘﺘﺄﺜﺭ ﺒﻘﻭﺘﻴﻥ‪:‬‬
‫‪‬‬ ‫‪‬‬
‫ﺍﻷﻭﻟﻰ ﻤﺤﺼﻠﺔ ﺍﻟﻘﻭﻯ ﺍﻟﻔﻌﺎﻟﺔ ‪ F i‬ﻭﺍﻟﺜﺎﻨﻴﺔ ‪ R i‬ﻤﺤﺼﻠﺔ ﺍﻟﻘﻭﻯ ﺍﻟﻨﺎﺘﺠﺔ ﻋﻥ‬
‫ﺍﻻﺭﺘﺒﺎﻁ‪ .‬ﻭﺒﻤﺎ ﺃﻥ ﺍﻟﺠﻤﻠﺔ ﻤﺘﻭﺍﺯﻨﺔ ﻴﻜﻭﻥ‪:‬‬
‫‪‬‬ ‫‪‬‬
‫‪Fi  R i  0‬‬
‫‪‬‬
‫ﻭﺒﺎﻟﻀﺭﺏ ﺒـ ‪ ri‬ﺜﻡ ﺍﻟﺠﻤﻊ ﺒـ ‪ i‬ﻨﺠﺩ‪:‬‬
‫‪N   ‬‬
‫‪‬‬ ‫‪(Fi  R i ). ri‬‬ ‫‪0‬‬ ‫)‪(1.29‬‬
‫‪i 1‬‬
‫ﻭﺒﻤﺎ ﺃﻥ ﺍﻻﺭﺘﺒﺎﻁ ﻤﺜﺎﻟﻲ ﻓﻼﺒﺩ ﺃﻥ ﻴﻨﻌﺩﻡ ﺍﻟﺤﺩ ﺍﻟﺜﺎﻨﻲ ﺃﻱ‪:‬‬
‫‪N  ‬‬
‫‪‬‬ ‫‪R i . ri‬‬ ‫‪0‬‬
‫‪i 1‬‬
‫‪N  ‬‬
‫ﻭﻫﻭ ﺍﻟﻤﻁﻠﻭﺏ‪.‬‬ ‫‪‬‬ ‫‪Fi . rt‬‬ ‫ﻭﺒﺎﻟﺘﺎﻟﻲ ﻴﻜﻭﻥ ‪ 0‬‬
‫‪i 1‬‬
‫‪N  ‬‬
‫ﻭﻴﻁﻠﺏ ﺍﻟﺒﺭﻫﺎﻥ ﺃﻥ‬ ‫‪‬‬ ‫‪Fi . rt‬‬ ‫ﺏ‪ -‬ﻜﻔﺎﻴﺔ ﺍﻟﺸﺭﻁ‪ :‬ﺘﺘﺤﻘﻕ ﺍﻟﻌﻼﻗﺔ ‪ 0‬‬
‫‪i 1‬‬
‫ﺍﻟﺠﻤﻠﺔ ﻤﺘﻭﺍﺯﻨﺔ‪.‬‬
‫ﻨﺒﺭﻫﻥ ﻜﻔﺎﻴﺔ ﺍﻟﺸﺭﻁ ﺒﻁﺭﻴﻘﺔ ﻨﻘﺽ ﺍﻟﻔﺭﺽ ﻓﻨﻘﻭل ﺇﻤﺎ ﺃﻥ ﺘﻜﻭﻥ ﺍﻟﺠﻤﻠﺔ‬
‫ﻤﺘﻭﺍﺯﻨﺔ ﻭﻴﺘﺤﻘﻕ ﺍﻟﻤﻁﻠﻭﺏ ﻭﺇﻤﺎ ﻏﻴﺭ ﻤﺘﻭﺍﺯﻨﺔ ﻭﻋﻨﺩﺌﺫ‪ ‬ﻓﻼﺒﺩ ﺃﻥ ﺘﺘﺤﺭﻙ ﺍﺤﺩﻯ‬
‫‪ ‬‬ ‫‪‬‬
‫ﻨﻘﻁﻬﺎ ‪ M i‬ﺘﺤﺕ ﺘﺄﺜﻴﺭ ﺍﻟﻘﻭﺓ ‪ Fi‬ﻭﺘﻌﻁﻲ ﻋﻤﻼﹰ ﺤﻘﻴﻘﻴﺎﹰ ﻤﻭﺠﺒﺎﹰ ‪ . Fi . dri  0‬ﻭﻗﺩ‬
‫‪‬‬ ‫‪‬‬
‫ﺭﺃﻴﻨﺎ ﺃﻨﻪ ﻴﻤﻜﻥ ﺃﻥ ﻨﺠﺩ ﺍﻨﺘﻘﺎﻻﹰ ﺍﻓﺘﺭﺍﻀﻴﺎﹰ ‪ ri‬ﻴﻨﻁﺒﻕ ﻋﻠﻰ ﺍﻨﺘﻘﺎل ﺤﻘﻴﻘﻲ ‪dri‬‬
‫‪ ‬‬
‫)ﺭﺍﺠﻊ ﺍﻟﻔﻘﺭﺓ ‪ (3‬ﻭﺒﺎﻟﺘﺎﻟﻲ ﻴﻜﻭﻥ ‪ Fi . ri  0‬ﻭﻟﻤﺎ ﻜﺎﻨﺕ ﺍﻻﺭﺘﺒﺎﻁﺎﺕ ﻤﺜﺎﻟﻴﺔ ﻓﺈﻨﻨﺎ‬

‫‪- 25 -‬‬

‫)‪Create PDF files without this message by purchasing novaPDF printer (http://www.novapdf.com‬‬
‫‪N  ‬‬
‫ﺨﻼﻑ ﺍﻟﻔﺭﺽ ﻭﺒﺎﻟﺘﺎﻟﻲ ﻻﺒﺩ ﺃﻥ ﺘﻜﻭﻥ ﺍﻟﺠﻤﻠﺔ ﻤﺘﻭﺍﺯﻨﺔ‬ ‫‪ Fi .ri‬ﻭﻫﺫﺍ‬
‫‪‬‬ ‫ﻨﺠﺩ‪ 0 :‬‬
‫‪i 1‬‬
‫ﻭﻫﻭ ﺍﻟﻤﻁﻠﻭﺏ‪.‬‬
‫ﻜﻴﻑ ﻴﺼ ﺒﺢ ﺸﺭﻁ ﺍﻟﺘﻭﺍﺯﻥ ﺍﻟﺴﺎﺒﻕ )‪ (1.28‬ﻋﻨﺩﻤﺎ ﺘﺘﺄﺜﺭ ﺍﻟﺠﻤﻠﺔ ﺒﺎﺭﺘﺒﺎﻁﺎﺕ‬
‫ﺃﺤﺎﺩﻴﺔ ﺍﻟﺠﺎﻨﺏ ﻴﻤﻜﻥ ﺃﻥ ﺘ ﺘﺤﺭﺭ ﻨﻘﻁﻬﺎ ﻤﻥ ﺍﻟﻘﻴﺩ؟‬
‫ﺘﺘﺤﺭﺭ ﺍﻟﻨﻘﻁﺔ ﻤﻥ ﺍﻟﻘﻴﺩ ﺒﺎﺘﺠﺎﻩ ﺭﺩ ﺍﻟﻔﻌل ﻭﺒﺎﻟﺘﺎﻟﻲ ﻴﻜﻭﻥ ﻋﻤل ﺭﺩ ﺍﻟﻔﻌل‬
‫‪N  ‬‬
‫ﻭﻁﺒﻘﺎﹰ ﻟـ )‪ (1.29‬ﻨﺠﺩ‪:‬‬ ‫‪‬‬ ‫‪R i . ri‬‬ ‫ﻤﻭﺠﺒﺎﹰ ﺃﻱ ﺃﻥ ‪ 0‬‬
‫‪i 1‬‬
‫‪N  ‬‬
‫‪ Fi .ri  0‬‬ ‫)‪(1.30‬‬
‫‪i 1‬‬
‫ﻭﻫﻭ ﺸﺭﻁ ﺘﻭﺍﺯﻥ ﺍﻟﺠﻤل ﺍﻟﻤﺎﺩﻴﺔ ﺍﻟﻤﺤﺭﺭﺓ‪ .‬ﻴﻤﻜﻥ ﺠﻤﻊ ﺍﻟﺸﺭﻁﻴﻥ )‪(1.29‬‬
‫ﻭ )‪ (1.30‬ﺒﺸﺭﻁ ﻭﺍﺤﺩ ﻫﻭ‪:‬‬
‫‪N  ‬‬
‫‪‬‬ ‫‪Fi . ri‬‬ ‫‪0‬‬
‫‪i 1‬‬
‫ﻋﻠﻰ ﺃﻥ ﺘﺄﺨﺫ ﺇﺸﺎﺭﺓ ﺍﻟﻤﺴﺎﻭﺍﺓ ﻟﻼﺭﺘﺒﺎﻁﺎﺕ ﺍﻟﺜﻨﺎﺌﻴﺔ ﻭﺍﻟﻤﺘﺭﺍﺠﺤﺔ ﻟﻸﺤﺎﺩﻴﺔ‪.‬‬
‫ﺗﻮﺍﺯﻥ ﻧﻘﻄﺔ ﻣﺎﺩﻳﺔ ﺍﻋﺘﻤﺎﺩﺍ ً ﻋﻠﻰ ﻣﻀﺎﺭﻳﺐ ﻻﻏﺮﺍﻧﺞ‪:‬‬
‫‪-6‬‬
‫ﺴﻨﺩﺭﺱ ﻓﻲ ﻫﺫﻩ ﺍﻟﻔﻘﺭﺓ ﻭﺍﻟﺘﻲ ﺘﻠﻴﻬﺎ ﻁﺭﻴﻘﺔ ﺜﺎﻨﻴﺔ ﻻﻴﺠﺎﺩ ﺘﻭﺍﺯﻥ ﻨﻘﻁﺔ ﻤﺎﺩﻴﺔ‬
‫)ﺠﺴﻴﻡ( ﻭﺠﻤﻠﺔ ﻤﺎﺩﻴﺔ‪ ،‬ﻴﻤﻜﻥ ﺍﻋﺘﻤﺎﺩﺍﹰ ﻋﻠﻴﻬﺎ‪ ،‬ﺃﻥ ﻨﺤﺴﺏ ﺭﺩﻭﺩ ﺍﻷﻓﻌﺎل ﺍﻟﻨﺎﺘﺠﺔ ﻋﻥ‬
‫ﺍﻻﺭﺘﺒﺎﻁ‪.‬‬
‫ﻟﺘﻜﻥ ﻨﻘﻁﺔ ﻤﺎﺩﻴﺔ ‪ M‬ﺘﺘﺤﺭﻙ ﻋﻠﻰ ﺴﻁﺢ ﻤﻌﺎﺩﻟﺘﻪ )ﺍﺭﺘﺒﺎﻁ ﻤﻘﻴﺩ(‪:‬‬
‫‪f ( x, y, z )  0‬‬
‫ﻭﺒﺎﻟﺘﺎﻟﻲ ﻜﻤﺎ ﺭﺃﻴﻨﺎ ﺴﺎﺒﻘﺎﹰ ﺘﺘﺤﻘﻕ ﺍﻟﻌﻼﻗﺔ )‪ ،(1.11a‬ﻭﻟﻨﻜﺘﺏ ﺸﺭﻁ ﺘﻭﺍﺯﻥ‬
‫ﺍﻟﻨﻘﻁﺔ ‪ M‬ﺍﻋﺘﻤﺎﺩﺍﹰ ﻋﻠﻰ ﻤﺒﺩﺃ ﺍﻟﻌﻤل ﺍﻻﻓﺘﺭﺍﻀﻲ ﺜﻡ ﻨﻀﺭﺏ )‪ (1.11a‬ﺒﻤﻀﺭﻭﺏ‬
‫ﺍﺨﺘﻴﺎﺭﻱ ‪) ‬ﻴﺴﻤﻰ ﻤﻀﺭﻭﺏ ﻻﻏﺭﺍﻨﺞ( ﻭﻨﻀﻴﻑ ﺍﻟﻤﻌﺎﺩﻟﺔ ﺍﻟﻨﺎﺘﺠﺔ ﺇﻟﻰ ﺸﺭﻁ‬
‫ﺍﻟﺘﻭﺍﺯﻥ ﻓﻨﺤﺼل ﻋﻠﻰ ﺍﻟﻤﻌﺎﺩﻟﺔ‪:‬‬
‫‪ ‬‬ ‫‪‬‬
‫)‪F . r f . r  0 (1.31‬‬

‫‪- 26 -‬‬

‫)‪Create PDF files without this message by purchasing novaPDF printer (http://www.novapdf.com‬‬
‫ﺃﻭ‪:‬‬
‫‪ f‬‬ ‫‪f‬‬ ‫‪f‬‬ ‫‪‬‬
‫‪Fx .x  Fy y  Fz  z   x  y  z )  0 ‬‬
‫‪ x‬‬ ‫‪y‬‬ ‫‪z‬‬ ‫‪‬‬
‫ﻭﻤﻨﻪ‪:‬‬
‫‪f‬‬ ‫‪f‬‬ ‫‪f‬‬
‫‪(Fx  ‬‬ ‫)‪)x  (Fy   )y  (Fz   )z  0 (1.32‬‬
‫‪x‬‬ ‫‪y‬‬ ‫‪z‬‬
‫ﻭﻟﻜﻲ ﺘﺘﺤﻘﻕ ﻫﺫﻩ ﺍﻟﻤﻌﺎﺩﻟﺔ ﻤﻬﻤﺎ ﻜﺎﻨﺕ ‪ ‬ﻻﻴﻤﻜﻥ ﺃﻥ ﻨﻘﻭل ﻤﺒﺎﺸﺭﺓﹰ ﺃﻥ ﺃﻤﺜﺎل‬
‫ﻜل ﻤﻥ ‪ z , y, x‬ﺘﺴﺎﻭﻱ ﺍﻟﺼﻔﺭ ﻷﻥ ﺍﻻﻨﺘﻘﺎﻻﺕ ﺍﻟﻤﺫﻜﻭﺭﺓ ﻏﻴﺭ ﻤﺴﺘﻘﻠﺔ ﻓﻬﻲ‬
‫ﺘﺭﺘﺒﻁ ﻓﻴﻤﺎ ﺒﻴﻨﻬﺎ ﺒﺎﻟﻤﻌﺎﺩﻟﺔ )‪.(1.11a‬‬
‫‪‬‬
‫ﻭﻟﻜﻥ ﺇﺫﺍ ﻋﻠﻤﻨﺎ ﺃﻥ ‪ ‬ﻫﻭ ﻤﻀﺭﻭﺏ ﺍﺨﺘﻴﺎﺭﻱ )ﺘﺘﺤﻘﻕ ﺍﻟﻌﻼﻗﺔ ‪f . r  0‬‬
‫ﻤﻬﻤﺎ ﻜﺎﻨﺕ ‪ (‬ﻓﻴﻤﻜﻥ ﻨﺯﻉ ﺍﺨﺘﻴﺎﺭﻴﺔ ﻫﺫﺍ ﺍﻟﻤﻀﺭﻭﺏ ﻭﺘﺤﺩﻴﺩ ﻗﻴﻤﺘﻪ ﻋﻥ ﻁﺭﻴﻕ‬
‫ﺍﻨﻌﺩﺍﻡ ﺃﺤﺩ ﺍﻷﻗﻭﺍﺱ )ﻭﻟﻴﻜﻥ ﺍﻟﺜﺎﻟﺙ( ﻓﻲ )‪ (1.32‬ﺃﻱ ﺃﻥ‪:‬‬
‫‪f‬‬
‫‪Fz  ‬‬ ‫)‪ 0 (1.33‬‬
‫‪z‬‬
‫ﻭﺒﻤﺎ ﺃﻥ ﻟﻠﻤﺴﺄﻟﺔ ﺩﺭﺠﺘﻲ ﺤﺭﻴﺔ ﻓﻴﻤﻜﻥ ﺍﻋﺘﺒﺎﺭ ‪ y, x‬ﺍﻨﺘﻘﺎﻟﻴﻥ ﻤﺴﺘﻘﻠﻴﻥ‬
‫ﻭﺒﺎﻟﺘﺎﻟﻲ ﻴﻤﻜﻥ ﺃﻥ ﻨﻌﺩﻡ ﺃﻤﺜﺎﻟﻬﻤﺎ ﻭﻫﻜﺫﺍ ﻨﺠﺩ ﺒﺎﻻﻀﺎﻓﺔ ﺇﻟﻰ )‪ (1.33‬ﺍﻟﻤﻌﺎﺩﻟﺘﻴﻥ‪:‬‬
‫‪f‬‬ ‫‪f‬‬
‫‪Fx  ‬‬ ‫‪ 0 , Fy  ‬‬ ‫)‪ 0 (1.34‬‬
‫‪x‬‬ ‫‪y‬‬
‫ﻨﻀﻴﻑ ﺇﻟﻰ ﺍﻟﻤﻌﺎﺩﻻﺕ )‪ (1.33‬ﻭ )‪ (1.34‬ﻤﻌﺎﺩﻟﺔ ﺍﻻﺭﺘﺒﺎﻁ ﻓﻨﺤﺼل ﻋﻠﻰ‬
‫ﺃﺭﺒﻊ ﻤﻌﺎﺩﻻﺕ ﻜﺎﻓﻴﺔ ﻟﺤﺴﺎﺏ ﺃﺭﺒﻌﺔ ﻤﺠﺎﻫﻴل ﻫﻲ ﺍﺤﺩﺍﺜﻴﺎﺕ ﻤﻭﻀﻊ ﺍﻟﺘﻭﺍﺯﻥ‬
‫) ‪ ( x, y, z‬ﻭ ‪ ‬ﻤﻀﺭﻭﺏ ﻻﻏﺭﺍﻨﺞ‪.‬‬
‫ﻨﻌﻤﻡ ﺍﻟﻁﺭﻴﻘﺔ ﺍﻟﺴﺎﺒﻘﺔ ﻋﻨﺩﻤﺎ ﻴﺘﺤﺭﻙ ﺍﻟﺠﺴﻴﻡ ﻋﻠﻰ ﻤﻨﺤﻥﹴ ﻨﺎﺘﺞ ﻋﻥ ﺘﻘﺎﻁﻊ‬
‫ﺍﻟﺴﻁﺤﻴﻥ‪:‬‬
‫)‪f i ( x, y, z )  0 , (i  1 , 2) (1.35‬‬
‫ﻭﻋﻨﺩﺌﺫ‪ ‬ﺘﺘﺤﻘﻕ ﺍﻻﻨﺘﻘﺎﻻﺕ ﺍﻻﻓﺘﺭﺍﻀﻴﺔ ﺍﻟﻤﻌﺎﺩﻟﺘﻴﻥ‪:‬‬
‫‪f i‬‬ ‫‪f i‬‬ ‫‪f i‬‬
‫‪x ‬‬ ‫‪y ‬‬ ‫)‪z  0 (1.36‬‬
‫‪x‬‬ ‫‪y‬‬ ‫‪z‬‬

‫‪- 27 -‬‬

‫)‪Create PDF files without this message by purchasing novaPDF printer (http://www.novapdf.com‬‬
‫ﻨﻜﺘﺏ ﺍﻵﻥ ﺸﺭﻁ ﺍﻟﺘﻭﺍﺯﻥ )‪ (1.28‬ﻭﻨﻀﻴﻑ ﻟﻪ ﺍﻟﻤﻌﺎﺩﻟﺘﻴﻥ )‪ (1.36‬ﺒﻌﺩ‬
‫ﺍﻟﻀﺭﺏ ﺒـ ‪)  i‬ﻤﻀﺭﻭﺒﻴﻥ ﺍﺨﺘﻴﺎﺭﻴﻴﻥ( ﻓﻨﺤﺼل ﺃﺨﻴﺭ ﺍﹰ ﻋﻠﻰ ﺍﻟﻤﻌﺎﺩﻟﺔ‪:‬‬

‫‪f1‬‬ ‫‪f‬‬ ‫‪f‬‬ ‫‪f‬‬


‫‪(Fx  1‬‬ ‫‪  2 2 )x  (Fy  1 1   2 2 )y ‬‬
‫‪x‬‬ ‫‪x‬‬ ‫‪y‬‬ ‫‪y‬‬
‫‪f‬‬ ‫‪f‬‬
‫‪(Fz   1   2 2 )z  0‬‬ ‫)‪(1.37‬‬
‫‪z‬‬ ‫‪z‬‬
‫ﻨﻨﺯﻉ ﺍﺨﺘﻴﺎﺭﻴﺔ ﺍﻟﻤﻀﺭﻭﺒﻴﻥ ‪ 1‬ﻭ ‪  2‬ﻓﻨﻌﻴﻨﻬﻤﺎ ﺒﻤﺴﺎﻭﺍﺓ ﻗﻭﺴﻴﻥ ﻤﻥ‬
‫ﺍﻷﻗﻭﺍﺱ ﺍﻟﺜﻼﺜﺔ ﺍﻟﺴﺎﺒﻘﺔ ﺒﺎﻟﺼﻔﺭ ﻓﻨﺤﺼل ﻋﻠﻰ ﻤﻌﺎﺩﻻﺕ ﺍﻟﺘﻭﺍﺯﻥ ﺍﻟﺘﺎﻟﻴﺔ )ﺘﻌﻤﻴﻡ‬
‫ﻟﻠﻤﻌﺎﺩﻻﺕ )‪:((1.34) ،(1.33‬‬
‫‪f1‬‬ ‫‪f‬‬ ‫‪‬‬
‫‪Fx  1‬‬ ‫‪  2 2  0‬‬
‫‪x‬‬ ‫‪x‬‬ ‫‪‬‬
‫‪‬‬
‫‪f‬‬ ‫‪f‬‬ ‫‪‬‬
‫)‪Fy  1 1   2 2  0 (1.38‬‬
‫‪y‬‬ ‫‪y‬‬ ‫‪‬‬
‫‪‬‬
‫‪‬‬
‫‪f1‬‬ ‫‪f 2‬‬
‫‪Fz  1‬‬ ‫‪ 2‬‬ ‫‪ 0‬‬
‫‪z‬‬ ‫‪z‬‬ ‫‪‬‬
‫ﻨﻀﻴﻑ ﺇﻟﻰ ﺍﻟﻤﻌﺎﺩﻻﺕ )‪ (1.38‬ﻤﻌﺎﺩﻟﺘﻲ ﺍﻻﺭﺘﺒﺎﻁ )‪ (1.35‬ﻓﻨﺤﺼل ﺃﺨﻴﺭ ﺍﹰ‬
‫ﻋﻠﻰ ﺨﻤﺱ ﻤﻌﺎﺩﻻﺕ ﻟﺘﻌﻴﻴﻥ ﻤﻭﻀﻊ ﺍﻟﺘﻭﺍﺯﻥ ) ‪ ( x, y, z‬ﻭﺤﺴﺎﺏ ‪.  2 ، 1‬‬
‫ﻭﻤﻥ ﺍﻟﻤﻔﻴﺩ ﻓﻲ ﻨﻬﺎﻴﺔ ﻫﺫﻩ ﺍﻟﻔﻘﺭﺓ ﺘﻭﻀﻴﺢ ﺍﻟﻤﻔﻬﻭﻡ ﺍﻟﻔﻴﺯﻴﺎﺌﻲ ﻟﻤﻀﺎﺭﻴﺏ‬
‫ﻻﻏﺭﺍﻨﺞ‪ ،‬ﻟﻬﺫﺍ ﻨﻜﺘﺏ ﻤﻌﺎﺩﻻﺕ ﺍﻟﺘﻭﺍﺯﻥ ﺍﻟﻨﻴﻭﺘﻨﻴﺔ ﻟﺠﺴﻴﻡ ﻴﺘﺤﺭﻙ ﻋﻠﻰ ﺴﻁﺢ‬
‫ﻭﻨﻘﺎﺭﻨﻬﺎ ﻤﻊ ﺍﻟﻤﻌﺎﺩﻻﺕ ﺍﻟﺘﻲ ﺤﺼﻠﻨﺎ ﻋﻠﻴﻬﺎ ﻋﻥ ﻁﺭﻴﻕ ﺍﻟﻤﻴﻜﺎﻨﻴﻙ ﺍﻟﺘﺤﻠﻴﻠﻲ‪ .‬ﻓﻲ‬
‫ﺍﻷﻭﻟﻰ ﻨﺠﺩ‪:‬‬
‫‪ ‬‬
‫)‪F  R  0  Fx  R x  0, Fy  R y  0, Fz  R z  0 (1.39‬‬

‫ﻭﻓﻲ ﺍﻟﺜﺎﻨﻴﺔ ﻨﺠﺩ ﺍﻟﻤﻌﺎﺩﻻﺕ )‪ (1.33‬ﻭ )‪ (1.35‬ﻭﺒﺎﻟﻤﻘﺎﺭﻨﺔ ﻨﺤﺼل ﻋﻠﻰ‬


‫ﺭﺩ ﺍﻟﻔﻌل ﺍﻟﺘﺎﻟﻲ‪:‬‬
‫‪f‬‬ ‫‪f‬‬ ‫‪f‬‬
‫‪Rx  ‬‬ ‫‪, Ry  ‬‬ ‫‪, Rz  ‬‬
‫‪x‬‬ ‫‪y‬‬ ‫‪z‬‬

‫‪- 28 -‬‬

‫)‪Create PDF files without this message by purchasing novaPDF printer (http://www.novapdf.com‬‬
‫ﺃﻱ ﺃﻥ‪:‬‬
‫‪‬‬
‫)‪R  f   grad f (1.40‬‬
‫ﻭﻫﻜﺫﺍ ﻨﺠﺩ ﺃﻥ ﺍﻟﻤﻀﺭﻭﺏ ‪ ‬ﻴﺭﺘﺒﻁ ﺒﺭﺩ ﺍﻟﻔﻌل‪.‬‬
‫ﻭﻋﻨﺩﻤﺎ ﻴﺘﺤﺭﻙ ﺍﻟﺠﺴﻴﻡ ﻋﻠﻰ ﻤﻨﺤﻨﻲ ﻨﺤﺼل ﻋﻠﻰ ﺭﺩ ﺍﻟﻔﻌل ﺍﻟﺘﺎﻟﻲ‪:‬‬
‫‪‬‬
‫‪(1.41) R  1f1   2f 2‬‬
‫ﻓﻜﺄﻨﻨﺎ ﻗﺴﻤﻨﺎ ﺭﺩ ﻓﻌل ﺍﻟﻤﻨﺤﻨﻲ )ﺍﻟﻤﺅﻟﻑ ﻤﻥ ﺘﻘﺎﻁﻊ ﺴﻁﺤﻴﻥ( ﺇﻟﻰ ﻗﺴﻤﻴﻥ‪:‬‬
‫ﺍﻷﻭل ‪ 1f1‬ﻴﻨﺘﺞ ﻋﻥ ﺍﻟﺴﻁﺢ ﺍﻷﻭل ﻭﺍﻟﺜﺎﻨﻲ ‪  2 f 2‬ﻋﻥ ﺍﻟﺴﻁﺢ ﺍﻟﺜﺎﻨﻲ‪.‬‬
‫‪ -7‬ﺗﻮﺍﺯﻥ ﲨﻠﺔ ﻣﺎﺩﻳﺔ ﺑﻄﺮﻳﻘﺔ ﻣﻀﺎﺭﻳﺐ ﻻﻏﺮﺍﻧﺞ – ﻣﻌﺎﺩﻻﺕ‬
‫ﻻﻏﺮﺍﻧﺞ )ﺍﻟﻨﻮﻉ ﺍﻷﻭﻝ(‪:‬‬
‫ﻨﻌﻤﻡ ﻤﺎﺴﺒﻕ ﻋﻠﻰ ﺘﻭﺍﺯﻥ ﺠﻤﻠﺔ ﻤﺎﺩﻴﺔ ﻤﺅﻟﻔﺔ ﻤﻥ ‪ N‬ﻨﻘﻁﺔ ﺨﺎﻀﻌﺔ‬
‫ﻻﺭﺘﺒﺎﻁﺎﺕ ﻫﻭﻟﻭﻨﻭﻤﻴﺔ ﺜﻨﺎﺌﻴﺔ ﺍﻟﺠﺎﻨﺏ ﻤﻥ ﺍﻟﻨﻭﻉ )‪.(1.13‬‬
‫ﻴﻤﻜﻥ ﻭﻀﻊ ﺭﺩ ﺍﻟﻔﻌل ﺍﻟﻤﺅﺜﺭ ﻋﻠﻰ ﺍﻟﻨﻘﻁﺔ ‪ M i‬ﻤﻥ ﺍﻟﺠﻤﻠﺔ ﻜﺘﻌﻤﻴﻡ ﻟﻠﻌﻼﻗﺔ‬
‫)‪ (1.41‬ﺤﻴﺙ ﻨﺠﺩ‪:‬‬
‫‪‬‬ ‫‪k‬‬
‫‪R i  11f1   2 1f 2 .... k  i f k ‬‬ ‫‪   i f ‬‬ ‫)‪(1.42‬‬
‫‪ 1‬‬
‫ﻭﻋﻨﺩﺌﺫ‪ ‬ﻨﻜﺘﺏ ﻤﻌﺎﺩﻻﺕ ﺍﻟﺘﻭﺍﺯﻥ‪:‬‬
‫‪f ‬‬ ‫‪‬‬
‫‪Xi  a‬‬ ‫‪ 0‬‬
‫‪‬‬
‫‪x i‬‬ ‫‪‬‬
‫‪‬‬
‫‪f ‬‬ ‫‪‬‬
‫‪Yi    a‬‬ ‫‪0‬‬ ‫)‪(1.43‬‬
‫‪‬‬
‫‪y i‬‬ ‫‪‬‬
‫‪‬‬
‫‪f ‬‬ ‫‪‬‬
‫‪Zi    a‬‬ ‫‪0‬‬
‫‪‬‬
‫‪z i‬‬ ‫‪‬‬
‫ﺤﻴﺙ ) ‪ (X i , Yi , Z i‬ﻫﻲ ﻤﺭﻜﺒﺎﺕ ﻤﺤﺼﻠﺔ ﺍﻟﻘﻭﻯ ﺍﻟﻔﻌﺎﻟﺔ ﺍﻟﻤﺅﺜﺭﺓ ﻋﻠﻰ‬
‫ﺍﻟﻨﻘﻁﺔ ‪ ، M i‬ﻤﻊ ﺍﻟﻌﻠﻡ ﺃﻥ ﻋﺩﺩ ﺍﻟﻤﺠﺎﻫﻴل ﻫﻨﺎ ﻫﻭ ‪ 3N  k‬ﻤﺠﻬﻭﻻﹰ‪ .‬ﻭﻟﻜﻲ ﻨﺤﺴﺏ‬
‫ﻜل ﻫﺫﻩ ﺍﻟﻤﺠﺎﻫﻴل ﻻﺒﺩ ﻤﻥ ﺍﻻﺴﺘﻔﺎﺩﺓ ﻤﻥ ‪ k‬ﻤﻌﺎﺩﻟﺔ ﺍﺭﺘﺒﺎﻁ ﻭﻫﻜﺫﺍ ﻨﺤﺼل ﻋﻠﻰ‬

‫‪- 29 -‬‬

‫)‪Create PDF files without this message by purchasing novaPDF printer (http://www.novapdf.com‬‬
‫ﻤﻭﻀﻊ ﺘﻭﺍﺯﻥ ﺍﻟﻨﻘﻁﺔ‬ ‫) ‪( x i , y i , zi‬‬ ‫ﻤﻌﺎﺩﻟﺔ ﻜﺎﻓﻴﺔ ﻟﺤﺴﺎﺏ‬ ‫) ‪(3N  k‬‬
‫‪ (i  1,2,..., N i )M i‬ﻭ ‪ k‬ﻤﻀﺭﻭﺏ ﻻﻏﺭﺍﻨﺞ ‪. (  1,2,..., k ) ‬‬
‫ﻴﺴﻬل ﺍﻵﻥ ﺍﻟﺤﺼﻭل ﻋﻠﻰ ﻤﺎﻴﺴﻤﻰ ﻤﻌﺎﺩﻻﺕ ﻻﻏﺭﺍﻨﺞ )ﺍﻟﻨﻭﻉ ﺍﻷﻭل(‬
‫‪‬‬
‫ﺒﻭﻀﻊ ﻤﻌﺎﺩﻟﺔ ﺤﺭﻜﺔ ﺍﻟﻨﻘﻁﺔ ‪ M i‬ﺫﺍﺕ ﺍﻟﺘﺴﺎﺭﻉ ‪ Wi‬ﻭﺍﻟﻤﺘﺄﺜﺭﺓ ﺒﻘﻭﻯ ﻤﺤﺼﻠﺘﻬﺎ‬
‫‪‬‬ ‫‪‬‬
‫‪ Fi‬ﻭﺭﺩﻭﺩ ﻓﻌل ﻤﺤﺼﻠﺘﻬﺎ ‪ R i‬ﻤﻌﻁﺎﺓ ﺒﺎﻟﻌﻼﻗﺔ )‪ (1.42‬ﺤﻴﺙ ﻨﺠﺩ‪:‬‬
‫‪‬‬ ‫‪‬‬
‫)‪m i . Wi  Fi      i f  (1.44‬‬
‫‪‬‬
‫ﺍﻟﺘﻲ ﺘﺼﺒﺢ ﺒﺎﻻﺴﻘﺎﻁ ﻋﻠﻰ ﺍﻟﻤﺤﺎﻭﺭ ﺍﻹﺤﺩﺍﺜﻴﺔ‪:‬‬
‫‪‬‬ ‫‪f  ‬‬
‫‪mi x i  Xi    ‬‬ ‫‪‬‬
‫‪‬‬
‫‪x i ‬‬
‫‪‬‬
‫‪‬‬ ‫‪f  ‬‬
‫‪m i yi  Yi    ‬‬ ‫)‪ (1.45‬‬
‫‪‬‬
‫‪y i ‬‬
‫‪‬‬
‫‪‬‬ ‫‪f  ‬‬
‫‪m i z i  Zi    ‬‬ ‫‪‬‬
‫‪‬‬
‫‪z i ‬‬

‫ﻭﻫﻲ ﻋﺒﺎﺭﺓ ﻋﻥ ‪ 3N‬ﻤﻌﺎﺩﻟﺔ ﺘﻔﺎﻀﻠﻴﺔ ﻤﻥ ﺍﻟﻤﺭﺘﺒﺔ ﺍﻟﺜﺎﻨﻴﺔ ﻏﻴﺭ ﻜﺎﻓﻴﺔ ﻟﺤﺴﺎﺏ‬


‫ﻗﻭﺍﻨﻴﻥ ﺍﻟﺤﺭﻜﺔ ) ‪ x i , y i , z i‬ﺒﺩﻻﻟﺔ ﺍﻟﺯﻤﻥ ﻭﺜﻭﺍﺒﺕ ﺍﻟﺤﺭﻜﺔ(‪ .‬ﻟﺫﻟﻙ ﻨﻀﻴﻑ ﻟﻬﺎ ‪k‬‬
‫‪‬‬
‫ﻤﻌﺎﺩﻟﺔ ﺍﺭﺘﺒﺎﻁ ﻓﻨﺤﺼل ﻋﻠﻰ ‪ 3N  k‬ﻤﺠﻬﻭﻻﹰ ﻜﺎﻓﻴﺔ ﻟﺤﺴﺎﺏ ‪ ri‬ﺒﺩﻻﻟﺔ ﺍﻟﺯﻤﻥ‬
‫)ﻗﻭﺍﻨﻴﻥ ﺍﻟﺤﺭﻜﺔ( ﻭﻫﻲ ﻤﻥ ﺍﻟﺸﻜل‪:‬‬
‫‪   ‬‬ ‫‪‬‬
‫) ‪ri  ri ( t , C1 , C 2 ,...., C N‬‬ ‫)‪(1.46‬‬

‫ﻭ ‪ k‬ﻤﻀﺭﻭﺏ ﻻﻏﺭﺍﻨﺞ ‪  ‬ﺍﻟﺫﻱ ﻴﺭﺘﺒﻁ ﺒﺭﺩ ﺍﻟﻔﻌل ﻁﺒﻘﺎﹰ ﻟﻠﻤﻌﺎﺩﻟﺔ‬


‫)‪.(1.42‬‬

‫ﻤﻥ ﺍﻟﻭﺍﻀﺢ ﺃﻥ ﻋﺩﺩ ﺍﻟﻤﻌﺎﺩﻻﺕ ﺍﻟﺴﺎﺒﻘﺔ ﻜﺒﻴﺭ ﺠﺩﺍﹰ ﻭﺒﺎﻟﺘﺎﻟﻲ ﻴﺼﻌﺏ ﺤﻠﻬﺎ‬
‫ﻴﺩﻭﻴﺎﹰ ﻭﺒﺎﻟﺭﻏﻡ ﻤﻥ ﺫﻟﻙ ﻗﺩ ﻴﻜﻭﻥ ﻤﻥ ﺍﻟﻀﺭﻭﺭﻱ ﺍﻻﺴﺘﻔﺎﺩﺓ ﻤﻨﻬﺎ ﻓﻲ ﺍﻻﻨﺸﺎﺀﺍﺕ‬

‫‪- 30 -‬‬

‫)‪Create PDF files without this message by purchasing novaPDF printer (http://www.novapdf.com‬‬
‫ﺍﻟﻬﻨﺩﺴﻴﺔ ﻜﺒﻨﺎﺀ ﺍﻟﺠﺴﻭﺭ ﻭﺍﻷﺒﺭﺍﺝ ﻭﻏﻴﺭﻫﺎ‪ ،‬ﻭﻋﻨﺩﺌﺫ‪ ‬ﺘﻭﻀﻊ ﺒﺭﺍﻤﺞ ﺤﺎﺴﻭﺒﻴﺔ ﻟﺤل‬
‫ﺍﻟﻤﻌﺎﺩﻻﺕ ﻭﺤﺴﺎﺏ ﺍﻟﻤﺠﺎﻫﻴل‪.‬‬

‫ﺃﻤﺎ ﻋﻨﺩﻤﺎ ﻻﻨﻬﺘﻡ ﺒﺭﺩﻭﺩ ﺍﻟﻔﻌل ﻓﻨﺴﺘﺨﺩﻡ ﻤﻌﺎﺩﻻﺕ ﻻﻏﺭﺍﻨﺞ ﻤﻥ ﺍﻟﻨﻭﻉ ﺍﻟﺜﺎﻨﻲ‬
‫ﺍﻟﺘﻲ ﻋﺩﺩﻫﺎ ﺃﻗل ﻭﻫﻭ ) ) ‪ (3N  k‬ﻤﻌﺎﺩﻟﺔ ﻓﻘﻁ‪ ،‬ﻭﺒﺎﻟﺘﺎﻟﻲ ﻴﺴﻬل ﺤﻠﻬﺎ‪ ،‬ﻭﺍﻟﺘﻲ‬
‫ﺴﻨﺭﺍﻫﺎ ﻓﻲ ﺍﻟﻔﺼل ﺍﻟﺜﺎﻨﻲ‪.‬‬
‫ﺍﳊﺼﻮﻝ ﻋﻠﻰ ﺗﻜﺎﻣﻞ ﺍﻟﻄﺎﻗﺔ ﺍﻧﻄﻼﻗﺎ ً ﻣﻦ ﻣﻌﺎﺩﻻﺕ ﻻﻏﺮﺍﻧﺞ‪:‬‬
‫‪-8‬‬
‫ﻴﻤﻜﻥ ﺍﻟﺤﺼﻭل ﻋﻠﻰ ﺘﻜﺎﻤل ﺍﻟﻁﺎﻗﺔ ﺍﻨﻁﻼﻗﺎﹰ ﻤﻥ ﺍﻟﻤﻌﺎﺩﻻﺕ ﺍﻟﺴﺎﺒﻘﺔ‪ ،‬ﻭﻟﻬﺫﺍ‬
‫ﻨﻀﺭﺏ ﺃﻁﺭﺍﻑ ﺍﻟﻤﻌﺎﺩﻻﺕ )‪ (1.45‬ﺒـ ‪ dz i , dyi , dx i‬ﻋﻠﻰ ﺍﻟﺘﺭﺘﻴﺏ ﻭﻨﺠﻤﻊ‬
‫ﺒـ ‪ i‬ﺜﻡ ﻨﺠﻤﻊ ﺍﻟﻤﻌﺎﺩﻻﺕ ﺍﻟﺜﻼﺙ ﺍﻟﻨﺎﺘﺠﺔ ﻁﺭﻓﺎﹰ ﺇﻟﻰ ﻁﺭﻑ ﻓﻨﺠﺩ‪:‬‬
‫‪N‬‬ ‫‪N‬‬
‫‪‬‬ ‫‪‬‬ ‫‪‬‬
‫) ‪ (m i x i dx i  m i y i dyi  m i z i dzi )   (X i dx i  Yi dyi  Zi dzi‬‬
‫‪i 1‬‬ ‫‪i 1‬‬
‫‪N ‬‬ ‫‪‬‬ ‫‪f ‬‬ ‫‪f‬‬ ‫‪f‬‬ ‫‪‬‬
‫‪(1.47)      ‬‬ ‫‪dx i     dyi     dz i  ‬‬
‫‪x i‬‬ ‫‪ yi‬‬ ‫‪z i‬‬ ‫‪‬‬
‫‪i 1   ‬‬ ‫‪‬‬
‫ﻨﻀﻊ ﺍﻟﺤﺩﻭﺩ ﺍﻟﺜﻼﺜﺔ ﺍﻷﻭﻟﻰ ﺒﺎﻟﺼﻴﻐﺔ ﺍﻟﺘﺎﻟﻴﺔ‪:‬‬
‫‪ dx ‬‬ ‫‪‬‬ ‫‪‬‬
‫‪  i‬‬ ‫‪ dy i‬‬ ‫‪ dz i ‬‬ ‫‪d 1 N‬‬ ‫‪2‬‬ ‫‪2‬‬ ‫‪2‬‬
‫) ‪ i  i dt i dt i dt  dt 2  m i ( x i  y i  z i‬‬
‫‪m‬‬ ‫‪x‬‬ ‫‪‬‬ ‫‪y‬‬ ‫‪‬‬ ‫‪z‬‬ ‫‪‬‬
‫‪i‬‬ ‫‪‬‬ ‫‪‬‬ ‫‪i 1‬‬
‫‪d 1‬‬ ‫‪2‬‬
‫‪‬‬
‫‪dt 2 i‬‬
‫‪‬‬ ‫)‪mi v i  dT (1.48‬‬

‫‪‬‬
‫ﺃﻤﺎ ﺍﻟﺤﺩﻭﺩ ﺍﻟﺜﻼﺜﺔ ﺍﻟﺘﻲ ﺘﻠﻴﻬﺎ ﻓﻴﻤﻜﻥ ﻭﻀﻌﻬﺎ ﻜﻤﺎﻴﻠﻲ ﺒﻔﺭﺽ ﺃﻥ ﺍﻟﻘﻭﺓ ‪Fi‬‬
‫ﺘﺸﺘﻕ ﻤﻥ ﻜﻤﻭﻥ ‪:V‬‬
‫)‪(1.49‬‬
‫‪ V‬‬ ‫‪V‬‬ ‫‪V‬‬ ‫‪‬‬
‫‪ X dx‬‬‫‪i‬‬ ‫‪i‬‬ ‫‪ Yi dyi  Z i dz i   ‬‬ ‫‪dx i ‬‬ ‫‪dy i ‬‬ ‫‪dz i   dV‬‬
‫‪ i x i‬‬ ‫‪y i‬‬ ‫‪z i‬‬ ‫‪‬‬
‫ﺃﻤﺎ ﺍﻟﺤﺩﻭﺩ ﺍﻟﺜﻼﺜﺔ ﺍﻷﺨﻴﺭﺓ )ﺭﺍﺠﻊ )‪ (1.16‬ﺒﻌﺩ ﻀﺭﺒﻬﺎ ﺒـ ‪  ‬ﻭﺠﻤﻌﻬﺎ‬
‫‪ ‬ﻓﺘﺴﺎﻭﻱ‪:‬‬

‫‪- 31 -‬‬

‫)‪Create PDF files without this message by purchasing novaPDF printer (http://www.novapdf.com‬‬
‫‪ f ‬‬ ‫‪f ‬‬ ‫‪f ‬‬ ‫‪‬‬ ‫‪‬‬ ‫‪‬‬
‫‪    x i y i z i      i  i ‬‬
‫‪‬‬ ‫‪‬‬ ‫‪dx‬‬ ‫‪‬‬ ‫‪dy‬‬ ‫‪‬‬ ‫‪dz‬‬ ‫‪‬‬ ‫‪‬‬ ‫‪‬‬ ‫‪‬‬ ‫‪‬‬ ‫‪f‬‬ ‫‪.‬‬ ‫‪dr‬‬
‫‪‬‬ ‫‪i ‬‬ ‫‪i‬‬ ‫‪i‬‬ ‫‪i‬‬ ‫‪  i‬‬ ‫‪‬‬
‫‪f ‬‬
‫‪   ‬‬ ‫)‪dt (1.50‬‬
‫‪‬‬
‫‪t‬‬
‫ﻭﺒﺘﻌﻭﻴﺽ )‪ (1.50) – (1.48‬ﻓﻲ )‪ (1.47‬ﻨﺠﺩ‪:‬‬
‫‪f ‬‬
‫‪dT  dV    ‬‬ ‫)‪dt (1.51‬‬
‫‪‬‬
‫‪t‬‬
‫ﻓﺈﺫﺍ ﻓﺭﻀﻨﺎ ﺃﻥ ﺍﻻﺭﺘﺒﺎﻁﺎﺕ ﻤﺴﺘﻘﺭﺓ )ﻻﺘﺤﻭﻱ ﺍﻟﺯﻤﻥ ﺒﺸﻜل ﺼﺭﻴﺢ( ﻓﺈﻨﻨﺎ‬
‫ﻨﺤﺼل ﻤﻥ )‪ (1.51‬ﻋﻠﻰ ﺘﻜﺎﻤل ﺍﻟﻁﺎﻗﺔ ﺍﻟﺘﺎﻟﻲ‪:‬‬
‫)‪d (T  V )  0  T  V  E  Const. (1.52‬‬
‫ﺗﻮﺟﻴﻬﺎﺕ ﳊﻞ ﻣﺴﺎﺋﻞ ﺍﻟﺘﻮﺍﺯﻥ ﺍﻋﺘﻤﺎﺩﺍ ً ﻋﻠﻰ ﺍﳌﺒﺪﺃ ﺍﻟﺘﻮﺍﺯﻧﻲ‬
‫‪-9‬‬
‫ﻟﻼﻧﺘﻘﺎﻻﺕ ﺍﻻﻓﱰﺍﺿﻴﺔ‪:‬‬
‫‪ -1‬ﺭﺴﻡ ﺍﻟﺸﻜل ﺠﻴﺩﺍﹰ ﻭﺘﻁﺒﻴﻕ ﺍﻟﻘﻭﻯ ﺍﻟﻤﺅﺜﺭﺓ ﻋﻠﻴﻪ‪.‬‬
‫‪ -2‬ﺍﺨﺘﻴﺎﺭ ﻭﺴﻁﺎﺀ ﺍﻟﻤﺴﺄﻟﺔ ﺍﻟﻤﺴﺘﻘﻠﺔ ﻭﻫﻲ ﻋﺩﺩ ﺍﻹﺤﺩﺍﺜﻴﺎﺕ ﺍﻟﻤﺴﺘﻘﻠﺔ‬
‫ﺍﻟﻼﺯﻤﺔ )ﻟﻭﺼﻑ( ﺍﻟﺠﻤﻠﺔ ﺍﻟﻤﺎﺩﻴﺔ‪ ،‬ﻭﻗﺩ ﻴﻜﻭﻥ ﻫﺫﺍ ﺍﻟﺒﻨﺩ ﺃﻫﻡ ﻤﻥ ﻏﻴﺭﻩ ﻷﻥ ﺍﻟﻨﺠﺎﺡ‬
‫ﻓﻲ ﺤل ﺍﻟﻤﺴﺎﻟﺔ ﻴﺘﻭﻗﻑ ﺒﺼﻭﺭﺓ ﺃﺴﺎﺴﻴﺔ ﻋﻠﻰ ﺼﺤﺔ ﻫﺫﺍ ﺍﻻﺨﺘﻴﺎﺭ‪ .‬ﻭﻟﻌل ﻤﻥ ﺍﻟﻤﻔﻴﺩ‬
‫ﺃﻥ ﻨﺫﻜﺭ ﻁﺭﻴﻘﺔ ﺒﺴﻴﻁﺔ ﻟﺫﻟﻙ ﺘﻌﺘﻤﺩ ﻋﻠﻰ ﺘﺜﺒﻴﺕ ﺃﺠﺯﺍﺀ ﺍﻟﺠﻤﻠﺔ ﺒﺎﻟﺘﺘﺎﻟﻲ ﻋﻠﻰ ﺃﻥ‬
‫ﺘﺜﺒﺕ ﻓﻲ ﻜل ﻤﺭﺓ ﻭﺴﻴﻁﺎﹰ ﻭﺍﺤﺩﺍﹰ ﻓﻴﻜﻭﻥ ﻋﺩﺩ ﻤﺭﺍﺕ ﺍﻟﺘﺜﺒﻴﺕ ﻫﻭ ﺒﺎﻟﻀﺒﻁ ﻋﺩﺩ‬
‫ﺍﻻﺤﺩﺍﺜﻴﺎﺕ ﺍﻟﻤﺴﺘﻘﻠﺔ ﺍﻟﻼﺯﻤﺔ ﻟﺩﺭﺍﺴﺔ ﺍﻟﺠﻤﻠﺔ‪ .‬ﻭﻟﻨﺫﻜﺭ ﺒﻌﺽ ﺍﻷﻤﺜﻠﺔ ﻋﻠﻰ ﺫﻟﻙ‪:‬‬
‫ﺃ‪ -‬ﺇﻥ ﻋﺩﺩ ﺍﻻﺤﺩﺍﺜﻴﺎﺕ ﺍﻟﻼﺯﻤﺔ ﻟﺩﺭﺍﺴﺔ ﻨﻘﻁﺔ ﻤﺎﺩﻴﺔ ﻁﻠﻴﻘﺔ ﻻﺘﺨﻀﻊ ﻷﻱ‬
‫ﺃﺭﺒﻁﺔ‪ ،‬ﻫﻲ ﺜﻼﺙ؛ ﻷﻥ ﻫﺫﻩ ﺍﻟﻨﻘﻁﺔ ﻴﻤﻜﻥ ﺃﻥ ﺘﺘﺤﺭﻙ ﻓﻲ ﺜﻼﺜﺔ ﺍﺘﺠﺎﻫﺎﺕ ﻤﻭﺍﻓﻘﺔ‬
‫ﻟﺜﻼﺜﺔ ﻤﺤﺎﻭﺭ ﺇﺤﺩﺍﺜﻴﺔ ‪ .oxyz‬ﻓﺈﺫﺍ ﺜﺒﺘﻨﺎ ﺍﻟﺤﺭﻜﺔ ﻋﻠﻰ ‪ ox‬ﻓﺈﻥ ﺍﻟﻨﻘﻁﺔ ﻟﻥ ﺘﺜﺒﺕ‬
‫)ﺍﻟﻭﺴﻴﻁ ﺍﻷﻭل( ﺜﻡ ﺇﺫﺍ ﺜﺒﺘﻨﺎ ﺍﻟﺤﺭﻜﺔ ﻋﻠﻰ ‪ oy‬ﻓﺈﻥ ﺍﻟﻨﻘﻁﺔ ﻟﻥ ﺘﺜﺒﺕ ﺃﻴﻀﺎﹰ )ﺍﻟﻭﺴﻴﻁ‬
‫ﺍﻟﺜﺎﻨﻲ( ﻭﺃﺨﻴﺭﺍﹰ ﺇﺫﺍ ﺜﺒﺘﻨﺎ ﺍﻟﺤﺭﻜﺔ ﻋﻠﻰ ‪ oz‬ﻓﺈﻥ ﺍﻟﺤﺭﻜﺔ ﺘﺜﺒﺕ ﺘﻤﺎﻤﺎﹰ )ﺍﻟﻭﺴﻴﻁ ﺍﻟﺜﺎﻟﺙ‬
‫ﻭﺍﻷﺨﻴﺭ( ﻭﻫﻜﺫﺍ ﻴﻜﻭﻥ ﻟﻠﻨﻘﻁﺔ ﺍﻟﻤﺎﺩﻴﺔ ﺍﻟﻁﻠﻴﻘﺔ ﺜﻼﺙ ﻭﺴﻁﺎﺀ ﻨﺭﻤﺯ ﻟﻬﺎ ﺒـ ‪ z‬ﻭ ‪y‬‬
‫ﻭ ‪ x‬ﻓﻲ ﺍﻻﺤﺩﺍﺜﻴﺎﺕ ﺍﻟﺩﻴﻜﺎﺭﺘﻴﺔ ﻭﻴﻜﻭﻥ ﺸﺭﻁ ﺘﻭﺍﺯﻨﻬﺎ ﻫﻭ ﺍﻟﺘﺎﻟﻲ‪:‬‬

‫‪- 32 -‬‬

‫)‪Create PDF files without this message by purchasing novaPDF printer (http://www.novapdf.com‬‬
‫‪ ‬‬ ‫‪ ‬‬
‫)‪ Fi . ri  0  F . r  0  Fx x  Fy y  Fz z  0 (1.53‬‬
‫ﻭﻟﻜﻲ ﺘﺘﺤﻘﻕ ﺍﻟﻤﻌﺎﺩﻟﺔ ﺍﻟﺴﺎﺒﻘﺔ ﻻﺒﺩ ﺃﻥ ﻴﻜﻭﻥ ﺃﻤﺜﺎل ﻜل ﻤﻥ ‪ x‬ﻭ ‪ y‬ﻭ‬
‫‪ z‬ﺘﺴﺎﻭﻱ ﺍﻟﺼﻔﺭ ﻷﻥ ﺠﻤﻴﻊ ﻫﺫﻩ ﺍﻻﻨﺘﻘﺎﻻﺕ ﻤﺴﺘﻘﻠﺔ ﻓﻲ ﻫﺫﻩ ﺍﻟﺤﺎﻟﺔ‪ .‬ﻭﻫﻲ‬
‫ﺍﻟﻨﺘﻴﺠﺔ ﻨﻔﺴﻬﺎ ﺍﻟﺘﻲ ﻨﻌﺭﻓﻬﺎ ﻤﻥ ﺍﻟﻁﺭﻴﻘﺔ ﺍﻟﻌﺎﻤﺔ ﺍﻟﻤﺒﻨﻴ ﺔ ﻋﻠﻰ ﻤﻴﻜﺎﻨﻴﻙ ﻨﻴﻭﺘﻥ ﻓﻲ‬
‫ﺍﻟﺘﻭﺍﺯﻥ‪.‬‬
‫ﺏ‪ -‬ﺇﺫﺍ ﺘﺤﺭﻜﺕ ﺍﻟﻨﻘﻁﺔ ﺍﻟﻤﺫﻜﻭﺭﺓ ﻓﻲ ﺍﻟﻤﺜﺎل ﺍﻟﺴﺎﺒﻕ ﻋﻠﻰ ﺴﻁﺢ ﻤﺎ ﻤﻌﺎﺩﻟﺘﻪ‬
‫‪ f ( x, y, z )  0‬ﻓﺈﻥ ﻋﺩﺩ ﺍﻟﻭﺴﻁﺎﺀ ﺍﻟﻤﺴﺘﻘﻠﺔ ﻴﻨﻘﺹ ﺇﻟﻰ )‪ (2‬ﻓﻘﻁ ﻷﻥ ﻋﺩﺩ‬
‫ﺍﻟﻭﺴﻁﺎﺀ ﺍﻟﻼﺯﻤﺔ ﻟﺩﺭﺍﺴﺔ ﻫﺫﻩ ﺍﻟﻨﻘﻁﺔ ﻓﻴﻤﺎ ﻟﻭ ﻜﺎﻨﺕ ﻁﻠﻴﻘﺔ ﻴﺴﺎﻭﻱ ﺜﻼﺜﺔ ) ‪( x, y, z‬‬
‫ﻭﻫﻲ ﻟﻴﺴﺕ ﻤﺴﺘﻘﻠﺔ ﺍﻵﻥ ﻷﻨﻬﺎ ﺘﺭﺘﺒﻁ ﺒﻤﻌﺎﺩﻟﺔ ﺍﻟﺴﻁﺢ‪ ،‬ﻭﻫﻜﺫﺍ ﻴﻜﻔﻲ ﻤﻌﺭﻓﺔ ﺍﺜﻨﻴﻥ‬
‫ﻤﻥ ﺍﻹﺤﺩﺍﺜﻴﺎﺕ ﻷﻥ ﺍﻟﺜﺎﻟﺙ ﻴﻌﺭﻑ ﻤﻥ ﻤﻌﺎﺩﻟﺔ ﺍﻟﺴﻁﺢ‪ .‬ﺃﻤﺎ ﺇﺫﺍ ﺘﺤﺭﻜﺕ ﺍﻟﻨﻘﻁﺔ ﻋﻠﻰ‬
‫ﻤﻨﺤﻨﻲ ﻴﻤﻜﻥ ﺃﻥ ﻴﻔﻬﻡ ﻋﻠﻰ ﺃﻨﻪ ﺘﻘﺎﻁﻊ ﺴﻁﺤﻴﻥ ﻓﻲ ﺍﻟﻔﺭﺍﻍ ﻫﻤﺎ‪:‬‬
‫‪f 1 ( x , y, z)  0 , f 2 ( x, y, z )  0‬‬ ‫)‪(1.54‬‬
‫ﻭﻫﻜﺫﺍ ﻴﻜﻔﻲ ﻤﻌﺭﻓﺔ ﻭﺴﻴﻁ ﻭﺍﺤﺩ )‪ x‬ﺃﻭ ‪ y‬ﺃﻭ ‪ (z‬ﻟﻜﻲ ﻨﻌﺭﻑ ﺍﻟﻭﺴﻴﻁﻴﻥ‬
‫ﺍﻟﺒﺎﻗﻴﺘﻴﻥ ﻤﻥ ﻤﻌﺎﺩﻟﺘﻲ ﺍﻟﺴﻁﺤﻴﻥ‪.‬‬
‫ﺝ‪ -‬ﻋﻠﻰ ﺍﻟﺸﻜل )‪ (1.3‬ﻴﻭﺠﺩ ﻟﻠﻤﺴﺄﻟﺔ ﻭﺴﻴﻁﺎﻥ ﻤﺴﺘﻘﻼﻥ ﻫﻤﺎ ‪ ‬ﻭ ‪‬‬
‫ﻭﺴﺒﺏ ﺫﻟﻙ ﺃﻨﻪ ﺇﺫﺍ ﺜﺒﺘﻨﺎ ﺩﻭﺭﺍﻥ ﺍﻟﻤﺤﻭﺭ )ﺘﺜﺒﻴﺕ ‪ ،(‬ﻓﺈﻥ ﺍﻟﺠﻤﻠﺔ ﺍﻟﻤﺎﺩﻴﺔ ﻟﻥ ﺘﺜﺒﺕ‬
‫ﻜﻜل ﻷﻥ ﺍﻟﺤﻠﻘﺔ ﻴﻤﻜﻥ ﺃﻥ ﺘﺘﺤﺭﻙ ﻋﻠﻰ ﺍﻟﻤﺤﻭﺭ ∆ ﻭﻟﻜﻥ ﺇﺫﺍ ﺜﺒﺘﻨﺎ ﺃﻴﻀﺎﹰ ﺍﻟﺤﻠﻘﺔ‬
‫ﻋﻠﻰ ∆ )ﺃﻱ ﺜﺒﺘﻨﺎ ‪ (‬ﻓﺈﻥ ﺍﻟﺠﻤﻠﺔ ﺘﺜﺒﺕ ﻜﻠﻬﺎ ﻭﻫﻜﺫﺍ ﻴﻜﻭﻥ ﻟﻠﻤﺠﻤﻭﻋﺔ ﻭﺴﻴﻁﺎﻥ‬
‫ﻤﺴﺘﻘﻼﻥ ﻫﻤﺎ ‪ ‬ﻭ ‪ ‬ﻭﻟﻜﻥ ﺃﺤﺩﻫﻤﺎ ﻤﻌﻠﻭﻡ ﻤﻥ ﻨﺹ ﺍﻟﻤﺴﺄﻟﺔ ﻭﻫﻭ ‪ ‬ﻭﺒﺎﻟﺘﺎﻟﻲ‬
‫ﻴﻜﻔﻲ ﻤﻌﺎﺩﻟﺔ ﻭﺍﺤﺩﺓ ﻟﺩﺭﺍﺴﺘﻬﺎ‪.‬‬
‫ﺩ‪ -‬ﻋﻠﻰ ﺍﻟﺸﻜل )‪) (1.6‬ﺍﻨﻅﺭ ﻤﺴﺎﺌل ﺍﻟﻔﺼل( ﻴﻜﻭﻥ ﻟﻠﻤﺴﺄﻟﺔ ﻭﺴﻴﻁ ﻭﺍﺤﺩ‬
‫ﻜﻤﺎ ﺭﺃﻴﻨﺎ ﻭﻴﻤﻜﻥ ﺒﺭﻫﺎﻥ ﺫﻟﻙ ﺒﺴﻬﻭﻟﺔ ﻁﺒﻘﺎﹰ ﻟﻁﺭﻴﻘﺔ ﺍﻟﺘﺜﺒﻴﺕ ﺍﻟﻤﺫﻜﻭﺭﺓ‪ ،‬ﻭﺒﺎﻟﻔﻌل ﺇﺫﺍ‬
‫ﺜﺒﺘﻨﺎ ﺍﻟﺫﺭﺍﻉ ‪) oA‬ﺘﺜﺒﻴﺕ ﺍﻟﺯﺍﻭﻴﺔ ‪ (‬ﻓﺈﻥ ﺍﻟﺠﻤﻠﺔ ﺘﺘﺜﺒﺕ ﻜﻠﻬﺎ ﻭﻫﻜﺫﺍ ﻴﻜﻭﻥ ﻟﻠﺠﻤﻠﺔ‬
‫ﻭﺴﻴﻁ ﻭﺍﺤﺩ ﻫﻭ ‪ ‬ﻭﻴﻜﻔﻲ ﺍﺨﺘﻴﺎﺭ ﻫﺫﺍ ﺍﻟﻭﺴﻴﻁ ﺒﻤﺜﺎﺒﺔ ﺍﻟﺒﻌﺩ ﺒﻴﻥ ‪ O‬ﻭﺍﻟﻤﻜﺒﺱ ‪B‬‬
‫)ﺃﻱ ‪ (x‬ﻷﻥ ‪ x‬ﻭ ‪ ‬ﺘﺭﺘﺒﻁﺎﻥ ﺒﺎﻟﻌﻼﻗﺔ )‪.(2‬‬

‫‪- 33 -‬‬

‫)‪Create PDF files without this message by purchasing novaPDF printer (http://www.novapdf.com‬‬
‫ﻫـ‪ -‬ﻋﺎﺭﻀﺔ ﺘﺘﺤﺭﻙ ﻓﻲ ﻤﺴﺘﻭ ﺸﺎﻗﻭﻟﻲ ﺘﺤﺕ ﺘﺄﺜﻴﺭ ﺜﻘﻠﻬﺎ‪ .‬ﻤﺎﻫﻭ ﻋﺩﺩ‬
‫ﺍﻟﻭﺴﻁﺎﺀ ﺍﻟﻤﺴﺘﻘﻠﺔ؟‬
‫ﺇﻥ ﻋﺩﺩ ﺍﻟﻭﺴﻁﺎﺀ ﺍﻟﻼﺯﻤﺔ ﻟﺩﺭﺍﺴﺔ ﺤﺭﻜﺘﻬﺎ ﻫﻭ ﺜﻼﺜﺔ ﻷﻥ ﺘﺜﺒﻴﺕ ﻤﺭﻜﺯ‬
‫ﻜﺘﻠﺘﻬﺎ ‪ C‬ﻫﻨﺎ ﺜﺒﺘﻨﺎ ﻭﺴﻴﻁﻴﻥ ﻤﺭﺓ ﻭﺍﺤﺩﺓ ﻫﻤﺎ ))‪ x(C‬ﻭ )‪ (y(C‬ﻻﻴﺅﺩﻱ ﺇﻟﻰ ﺇﻴﻘﺎﻑ‬
‫ﺤﺭﻜﺘﻬﺎ ﻜﻜل ) ﺃﻥ ﻫﺫﻩ ﺍﻟﻌﺎﺭﻀﺔ ﻴﻤﻜﻥ ﺃﻥ ﺘﺩﻭﺭ ﺤﻭل ﻤﺤﻭﺭ ﻴﻤﺭ ﻤﻥ ﻤﺭﻜﺯ‬
‫ﻜﺘﻠﺘﻬﺎ‪ .‬ﻓﺈﺫﺍ ﺜﺒﺘﻨﺎ ﺍﻟﺩﻭﺭﺍﻥ )ﺘﺜﺒﻴﺕ ﺍﻟﺯﺍﻭﻴﺔ ‪ (‬ﻓﺈﻥ ﻫﺫﺍ ﻴﺅﺩﻱ ﺇﻟﻰ ﺜﺒﺎﺕ ﺍﻟﺠﻤﻠﺔ ﻜﻜل‬
‫ﻭﺒﺎﻟﺘﺎﻟﻲ ﻴﻜﻭﻥ ﻟﻠﻤﺴﺄﻟﺔ ﺜﻼﺜﺔ ﻭﺴﻁﺎﺀ ﻫﻲ )‪ x(C‬ﻭ )‪ y(C‬ﻭ ‪ .‬ﻭﻓﻲ ﺍﻟﺤﺎﻟﺔ‬
‫ﺍﻟﺨﺎﺼﺔ ﻋﻨﺩﻤﺎ ﺘﺜﺒﺕ ﻨﻘﻁﺔ ‪ O‬ﻤﻥ ﺍﻟﻌﺎﺭﻀﺔ ﻓﺈﻨﻪ ﻴﻜﻔﻲ ﻭﺴﻴﻁﺎﹰ ﻭﺍﺤﺩﺍﹰ ﻟﺩﺭﺍﺴﺘﻬﺎ‬
‫ﺒﺴﺒﺏ ﺇﻤﻜﺎﻨﻴﺔ ﺍﻟﺩﻭﺭﺍﻥ ﺤﻭل ‪ O‬ﺒﺯﺍﻭﻴﺔ ‪ ‬ﺘﺅﺨﺫ ﻜﻭﺴﻴﻁ ﻤﺴﺘﻘل ﻟﻠﻤﺴﺄﻟﺔ‪.‬‬
‫‪ -3‬ﺤﺴﺎﺏ ﺍﻟﻌﻤل ﺍﻻﻓﺘﺭﺍﻀﻲ ﻟﻠﻘﻭﻯ ﺍﻟﻔﻌﺎﻟﺔ ﺍﻟﻤﺅﺜﺭﺓ ﻓﻲ ﺍﻟﺠﻤﻠﺔ‪ ،‬ﻭﻴﺠﺏ ﺃﻥ‬
‫ﻴﺘﻡ ﻫﺫﺍ ﺍﻟﺤﺴﺎﺏ ﺤﺼﺭ ﺍﹰ ﺒﺩﻻﻟﺔ ﻭﺴﻁﺎﺀ ﺍﻟﻤﺴﺄﻟﺔ ﺍﻟﻤﺴﺘﻘﻠﺔ ﺍﻟﻤﺨﺘﺎﺭﺓ‪ ،‬ﻭﺒﺎﻻﺴﺘﻔﺎﺩﺓ ﻤﻥ‬
‫ﺍﻟﻤﻌﺎﺩﻟﺔ )‪ (1.28‬ﻨﺤﺼل ﻋﻠﻰ ﻤﺠﻤﻭﻋﺔ ﻤﻌﺎﺩﻻﺕ ﻋﺩﺩﻫﺎ ﻴﺴﺎﻭﻱ ﻋﺩﺩ ﺩﺭﺠﺎﺕ‬
‫ﺍﻟﺤﺭﻴﺔ )ﻋﺩﺩ ﺍﻟﻭﺴﻁﺎﺀ ﺍﻟﻤﺴﺘﻘﻠﺔ ﻟﻠﻤﺴﺄﻟﺔ( ﻫﻲ ﺒﺎﻟﻀﺒﻁ ﻤﻌﺎﺩﻻﺕ ﺍﻟﺘﻭﺍﺯﻥ ﺍﻟﺘﻲ‬
‫ﻴﻤﻜﻥ ﺃﻥ ﻨﺤﺴﺏ ﻤﻨﻬﺎ ﻤﺎﻫﻭ ﻤﻁﻠﻭﺏ ﻓﻲ ﺍﻟﻤﺴﺄﻟﺔ‪.‬‬

‫‪- 34 -‬‬

‫)‪Create PDF files without this message by purchasing novaPDF printer (http://www.novapdf.com‬‬
‫ﻣﺴﺎﺋﻞ ﻭﲤﺎﺭﻳﻦ‬

‫‪ -1‬ﻟﺘﻜﻥ ﺍﻟﺠﻤﻠﺔ ﺸﻜل )‪ (1.6‬ﺤﻴﺙ ‪ B‬ﻤﻜﺒﺱ ﻴﺘﺤﺭﻙ ﻀﻤﻥ ﺍﺴﻁﻭﺍﻨﺔ ﻭﻫﻭ‬


‫ﻤﺘﺼل ﺒﺫﺭﺍﻉ ‪ CA‬ﻁﻭﻟﻪ ‪ ‬ﻴﻨﺘﻬﻲ ﺒﺼﺎﻤﻭﻟﺔ ‪ A‬ﻭﻗﺩ ﺜﺒﺕ ﻤﻔﺼﻠﻴﺎﹰ ﺒﺎﻟﺼﺎﻤﻭﻟﺔ‬
‫ﺫﺭﺍﻉ ﺁﺨﺭ ‪ OA‬ﻁﻭﻟﻪ ‪ a‬ﻁﺭﻓﻪ ﺍﻵﺨﺭ ﻤﺜﺒﺕ ﺒﻨﻘﻁﺔ ﺜﺎﺒﺘﺔ ‪) O‬ﺠﻤﻠﺔ ﺘﺤﻭﻴل‬
‫ﺍﻟﺤﺭﻜﺔ ﺍﻻﻨﺴﺤﺎﺒﻴﺔ ﺇﻟﻰ ﺤﺭﻜﺔ ﺩﻭﺭﺍﻨﻴﺔ ﻭﻫﻲ ﻤﻭﺠﻭﺩﺓ ﻓﻲ ﻜل ﺍﻟﻤﺤﺭﻜﺎﺕ ﺫﺍﺕ‬
‫ﺍﻻﺤﺘﺭﺍﻕ ﺍﻟﺩﺍﺨﻠﻲ(‪ .‬ﻴﻁﻠﺏ ﺍﻴﺠﺎﺩ ﻤﻌﺎﺩﻻﺕ ﺍﻻﺭﺘﺒﺎﻁ ﻭﺤﺴﺎﺏ ﺍﻻﻨﺘﻘﺎﻻﺕ‬
‫ﺍﻻﻓﺘﺭﺍﻀﻴﺔ‪ ،‬ﺒﻔﺭﺽ ﺃﻥ ﺍﻟﺠﻤﻠﺔ ﻤﺅﻟﻔﺔ ﻤﻥ ﻨﻘﻁﺘﻴﻥ ﻓﻘﻁ ﻫﻤﺎ ﺍﻟﻤﻜﺒﺱ ‪ B‬ﻭﺍﻟﺼﺎﻤﻭﻟﺔ‬
‫‪.A‬‬

‫ﺍﻟﺸﻜل )‪(1.6‬‬
‫ﺍﳊﻞ‪:‬‬
‫ﻤﻥ ﺍﻟﻭﺍﻀﺢ ﺃﻥ ﺍﻟﺤﺭﻜﺔ ﺘﺤﺩﺙ ﻓﻲ ﻤﺴﺘﻭﻱ ﻭﺒﺎﻟﺘﺎﻟﻲ ﻴﻤﻜﻥ ﺘﻌﻴﻴﻥ ﻜل ﻤﻥ ‪A‬‬
‫ﻭ ‪ B‬ﺒﺎﺤﺩﺍﺜﻴﻴﻥ ﻓﻘﻁ ﻫﻤﺎ )‪ B( x, y‬ﻭ )‪) A(, ‬ﺍﻨﻅﺭ ﺍﻟﺸﻜل(‪ .‬ﺃﻤﺎ ﻤﻌﺎﺩﻻﺕ‬
‫ﺍﻻﺭﺘﺒﺎﻁ ﻓﻬﻲ‪:‬‬
‫)‪y  0 ,   a (1‬‬
‫ﻭﻻﺒﺩ ﻤﻥ ﻭﺠﻭﺩ ﻤﻌﺎﺩﻟﺔ ﺜﺎﻟﺜﺔ ﻷﻥ ﺍﻟﺠﻤﻠﺔ ﻜﻜل ﺘﺜﺒﺕ ﺒﺘﺜﺒﻴﺕ ﺍﻟﺯﺍﻭﻴﺔ ‪‬‬
‫ﻭﺒﺎﻟﺘﺎﻟﻲ ﻴﻜﻭﻥ ﻟﻬﺎ ﻭﺴﻴﻁ ﻭﺍﺤﺩ‪ .‬ﻭﻨﻼﺤﻅ ﻤﻥ ﺍﻟﺸﻜل ﺃﻥ‪:‬‬
‫)‪x  a cos    cos   const. (2‬‬

‫‪- 35 -‬‬

‫)‪Create PDF files without this message by purchasing novaPDF printer (http://www.novapdf.com‬‬
‫ﻭﻫﻨﺎ ﺃﻀﻔﻨﺎ ﻭﺴﻴﻁﺎﹰ ﺠﺩﻴﺩﺍﹰ ﻫﻭ ‪ ‬ﻴﻤﻜﻥ ﺤﺴﺎﺒﻪ ﺒﺩﻻﻟﺔ ‪ ‬ﺤﻴﺙ ﻨﺠﺩ ﻤﻥ‬
‫ﺍﻟﺸﻜل ‪:‬‬
‫‪a2‬‬
‫‪a sin    sin   cos   1 ‬‬
‫‪2‬‬
‫‪sin 2 ‬‬ ‫)‪(3‬‬
‫‪‬‬
‫ﻨﺒﺩل ‪ ‬ﺒﻘﻴﻤﺘﻬﺎ ﻤﻥ )‪ (3‬ﻓﻲ )‪ (2‬ﻓﻨﺠﺩ‪:‬‬
‫‪a2‬‬
‫‪x  a cos   l 1 ‬‬ ‫)‪sin 2   Const. (4‬‬
‫‪2‬‬
‫‪‬‬
‫ﻭﻫﻲ ﻤﻌﺎﺩﻟﺔ ﺍﻻﺭﺘﺒﺎﻁ ﺍﻟﺜﺎﻟﺜﺔ ﻭﺒﺎﻟﺘﺎﻟﻲ ﻴﻜﻭﻥ ﻋﺩﺩ ﻭﺴﻁﺎﺀ ﺍﻟﻤﺴﺄﻟﺔ ﺍﻟﻤﺴﺘﻘﻠﺔ‬
‫ﻫﻭ ﻭﺴﻴﻁ ﻭﺍﺤﺩ ﻨﺨﺘﺎﺭﻩ ﺍﻟﺯﺍﻭﻴﺔ ‪ ،‬ﺃﻱ ﺃﻥ ﻟﻠﻤﺴﺄﻟﺔ ﺩﺭﺠﺔ ﺤﺭﻴﺔ ﻭﺍﺤﺩﺓ‪.‬‬
‫ﺃﻤﺎ ﺍﻻﻨﺘﻘﺎﻻﺕ ﺍﻻﻓﺘﺭﺍﻀﻴﺔ ﻓﻬﻲ‪:‬‬
‫)‪  0 , y  0 ,   0 (5‬‬
‫ﺃﻤﺎ ‪ x‬ﻓﺘﺤﺴﺏ ﺒﺩﻻﻟﺔ ‪ ‬ﻤﻥ )‪ (4‬ﺤﻴﺙ ﻨﺠﺩ‪:‬‬
‫‪‬‬ ‫‪‬‬
‫‪‬‬ ‫‪a2‬‬ ‫‪‬‬
‫‪‬‬ ‫‪sin  cos  ‬‬
‫‪x   a sin   ‬‬ ‫)‪ (6‬‬
‫‪‬‬ ‫‪2‬‬ ‫‪‬‬
‫‪a‬‬
‫‪‬‬ ‫‪1‬‬ ‫‪sin 2  ‬‬
‫‪‬‬ ‫‪2‬‬ ‫‪‬‬

‫‪ -2‬ﺍﺩﺭﺱ ﺘﻭﺍﺯﻥ ﺍﻟﻌﺎﺭﻀﺔ ﺍﻟﻤﻭﻀﺤﺔ ﻋﻠﻰ ﺍﻟﺸﻜل )‪ (1.5‬ﺒﻔﺭﺽ ﺃﻥ ﺍﻟﻘﻭﺓ‬


‫ﺍﻟﻔﻌﺎﻟﺔ ﺍﻟﻭﺤﻴﺩﺓ ﺍﻟﻤﺅﺜﺭﺓ ﻫﻲ ﺜﻘل ﺍﻟﻌﺎﺭﻀﺔ ﺍﻟﻤﺭﻜﺯ ﻓﻲ ﻤﺭﻜﺯ ﺜﻘﻠﻬﺎ ‪.C‬‬
‫ﺍﳊﻞ ‪:‬‬
‫ﺍﻥ ﺸﺭﻁ ﺍﻟﺘﻭﺍﺯﻥ ﻫﻭ‪:‬‬
‫‪‬‬ ‫‪‬‬ ‫‪‬‬ ‫‪‬‬
‫)‪F . r  0  m g . C  0 (1‬‬
‫ﻭﻤﻨﻪ‪:‬‬
‫)‪ mg y( C)  0 (2‬‬
‫ﻟﺤﺴﺎﺏ )‪ y(C‬ﻨﺴﺘﻔﻴﺩ ﻤﻥ )‪.(1.27b‬‬
‫ﺤﻴﺙ ﻨﺠﺩ‪:‬‬

‫‪- 36 -‬‬

‫)‪Create PDF files without this message by purchasing novaPDF printer (http://www.novapdf.com‬‬
‫)‪ y( C)  a sin    a sin   a cos    h sin   (3‬‬

‫ﻓﺈﺫﺍ ﺒﺩﻟﻨﺎ ﻓﻲ )‪ (2‬ﻨﺠﺩ ﺍﻟﻌﻼﻗﺔ ﺍﻟﺘﻲ ﺘﺤﻘﻘﻬﺎ ‪  0‬ﻓﻲ ﻭﻀﻊ ﺍﻟﺘﻭﺍﺯﻥ‪:‬‬

‫‪( a 0 cos  0  h sin  0 )  0‬‬

‫ﻭﻤﻨﻪ ﻨﺤﺼل ﻋﻠﻰ ﻗﻴﻤﺔ ‪ ‬ﺍﻟﺘﺎﻟﻴﺔ‪:‬‬


‫‪a‬‬
‫‪tg   tg  0 ‬‬ ‫)‪ (4‬‬
‫‪h 0‬‬

‫ﻭﻤﻥ ﺍﻟﻭﺍﻀﺢ ﺃﻥ ﻫﺫﻩ ﺍﻟﻌﻼﻗﺔ ﻻﺘﺘﺤﻘﻕ ﺇﻻ ﻋﻨﺩﻤﺎ ‪ .  0  0‬ﻭﺍﻟﺠﺩﻴﺭ ﺒﺎﻟﺫﻜﺭ‬


‫ﺃﻨﻪ ﻋﻨﺩﻤﺎ ﻨﺼل ﺴﻤﺎﻜﺔ ﺍﻟﻌﺎﺭﻀﺔ ‪ h  0‬ﻓﻠﻥ ﺘﺘﺤﻘﻕ ﺍﻟﻌﻼﻗﺔ ﺍﻟﺴﺎﺒﻘﺔ ﺍﻻ ﻋﻨﺩﻤﺎ‬
‫‪ .  0  0‬ﻭﻫﻲ ﻨﺘﻴﺠﺔ ﻤﻨﻁﻘﻴﺔ ﻷﻥ ﺍﻟﺘﻭﺍﺯﻥ ﻴﺤﺩﺙ ﻋﻨﺩﻤﺎ ﺘﻜﻭﻥ ﺍﻟﻌﺎﺭﻀﺔ ﺃﻓﻘﻴﺔ‬
‫ﻭﺘﺭﺘﻜﺯ ﻋﻠﻰ ﺫﺭﻭﺓ ﺍﻟﻜﺭﺓ )‪. ( 0  0‬‬

‫‪ - 3‬ﻟﺘﻜﻥ ﺍﻟﺠﻤﻠﺔ ﺍﻟﻤﻭﻀﺤﺔ ﻋﻠﻰ ﺍﻟﺸﻜل )‪ (1.7‬ﺤﻴﺙ ‪ A‬ﺒﻜﺭﺓ ﺜﻘﻠﻬﺎ ‪P‬‬


‫ﻴﻤﻜﻨﻬﺎ ﺍﻟﺩﻭﺭﺍﻥ ﺤﻭل ﻤﺤﻭﺭ ﺃﻓﻘﻲ ﺜﺎﺒﺕ ﻭﻗﺩ ﻋﻠﻕ ﻓﻲ ﻤﺤﺯﻫﺎ ﺨﻴﻁ ﻴﻨﺘﻬﻲ ﻁﺭﻓﻪ‬
‫ﺍﻷﻭل ﺒﺜﻘل ‪ P3‬ﺒﻴﻨﻤﺎ ﻴﻨﺘﻬﻲ ﻁﺭﻓﻪ ﺍﻟﺜﺎﻨﻲ ﺒﺒﻜﺭﺓ ﺜﺎﻨﻴﺔ ‪ C‬ﺜﻘﻠﻬﺎ ‪ P4‬ﻴﻤﻜﻨﻬﺎ ﺍﻟﺘﺩﺤﺭﺝ‬
‫ﺩﻭﻥ ﺍﻨﺯﻻﻕ ﻋﻠﻰ ﻤﺴﺘﻭﹴ ﻴﻤﻴل ﻋﻠﻰ ﺍﻷﻓﻕ ﺒﺯﺍﻭﻴﺔ ‪ ،‬ﻭﻗﺩ ﻋﻠﻕ ﻓﻲ ﻤﺤﺯﻫﺎ ﺨﻴﻁ‬
‫ﺁﺨﺭ ﻴﻨﺘﻬﻲ ﺃﺤﺩ ﻁﺭﻓﻴﻪ ﺒﻜﺘﻠﺔ ‪ D‬ﺜﻘﻠﻬﺎ ‪ P2‬ﺒﻴﻨﻤﺎ ﻴﻨﺘﻬﻲ ﺍﻟﻁﺭﻑ ﺍﻵﺨﺭ ﺒﺜﻘل ‪E‬‬
‫ﻗﺩﺭﻩ ‪ P1‬ﻤﻭﻀﻭﻉ ﻋﻠﻰ ﻤﺴﺘ ﻭﹴ ﻴﻤﻴل ﻋﻠﻰ ﺍﻷﻓﻕ ﺒﺯﺍﻭﻴﺔ ‪ ‬ﺃﻴﻀﺎﹰ ﺸﻜل )‪(1.7‬‬
‫ﺇﺤﺴﺏ ﻗﻴﻤﺔ ﺍﻟﺜﻘل ‪ P3‬ﻭﺍﻟﺯﺍﻭﻴﺔ ‪ ‬ﺒﺩﻻﻟﺔ ﺍﻷﺜﻘﺎل ‪ P4 , P2 , P1‬ﻓﻲ ﺤﺎﻟﺔ ﺘﻭﺍﺯﻥ‬
‫ﺍﻟﺠﻤﻠﺔ‪.‬‬

‫‪- 37 -‬‬

‫)‪Create PDF files without this message by purchasing novaPDF printer (http://www.novapdf.com‬‬
‫ﺍﻟﺸﻜل )‪(1.7‬‬

‫ﺍﳊﻞ ‪:‬‬
‫ﺇﺫﺍ ﺍﺘﺒﻌﻨﺎ ﻁﺭﻴﻘﺔ ﺍﻟﺘﺜﺒﻴﺕ ﺍﻟﻤﺫﻜﻭﺭﺓ ﻓﻲ ﺍﻟﻔﻘﺭﺓ )‪ (9‬ﻟﻤﻌﺭﻓﺔ ﻋﺩﺩ ﺍﻟﻭﺴﻁﺎﺀ‬
‫ﺍﻟﻤﺴﺘﻘﻠﺔ ﻨﻼﺤﻅ ﺃﻥ ﻋﻨﺩ ﺘﺜﺒﻴﺕ ﺍﻟﺜﻘل ‪ P3‬ﻟﻥ ﺘﺜﺒﺕ ﺍﻟﺠﻤﻠﺔ ﻷﻥ ﺍﻟﺒﻜﺭﺓ ‪ C‬ﻴﻤﻜﻨﻬﺎ‬
‫ﺍﻟﺩﻭﺭﺍﻥ‪ .‬ﺃﻤﺎ ﺇﺫﺍ ﺜﺒﺘﻨﺎ ﻫﺫﺍ ﺍﻟﺩﻭﺭﺍﻥ ﻓﺈﻥ ﺍﻟﺠﻤﻠﺔ ﻜﻜل ﺘﺜﺒﺕ ﻭﺒﺎﻟﺘﺎﻟﻲ ﻴﻭﺠﺩ ﺩﺭﺠﺘﺎ‬
‫ﺤﺭﻴﺔ‪.‬‬
‫ﺴﻨﺄﺨﺫ ﺍﻟﻭﺴﻴﻁﻴﻥ ﺍﻟﺘﺎﻟﻴﻴﻥ ﺍﻟﻤﻘﺎﺒﻠﻴﻥ ﻟﻬﻤﺎ‪ :‬ﻫﺒﻭﻁ ﺍﻟﺜﻘل ‪ P3‬ﺃﻱ ‪ y B‬ﻭﻫﺒﻭﻁ‬
‫ﺍﻟﺜﻘل ‪ D‬ﺃﻱ ‪ . y D‬ﻤﻥ ﺍﻟﻤﻔﻴﺩ ﺃﻥ ﻨﺫﻜﺭ ﻁﺭﻴﻘﺔ ﺜﺎﻨﻴﺔ ﻟﺘﺤﺩﻴﺩ ﻋﺩﺩ ﺍﻟﻭﺴﻁﺎﺀ ﺍﻟﻤﺴﺘﻘﻠﺔ‬
‫ﻓﻨﻘﻭل ﺃﻨﻪ ﻤﻭﻀﻊ ﺍﻟﺜﻘل ‪ E‬ﻴﺘﻌﻴﻥ ﺒﻤﻌﺭﻓﺔ ﺒﻌﺩﻩ ﻋﻥ ﻨﻬﺎﻴﺔ ﺍﻟﻤﺴﺘﻭﻱ ﺍﻟﻤﺎﺌل ﺃﻤﺎ‬
‫ﺍﻟﺒﻜﺭﺓ ﺍﻟﺜﺎﻨﻴﺔ ‪ C‬ﻓﻴﻤﻜﻥ ﺘﻌﻴﻴﻥ ﻤﻭﻀﻌﻬﺎ ﺒﻭﺴﻴﻁﻴﻥ ﻓﻘﻁ ﻫﻤﺎ ﺩﻭﺭﺍﻨﻬﺎ ﻭﺍﻨﺴﺤﺎﺏ‬
‫ﻤﺭﻜﺯ ﻜﺘﻠﺘﻬﺎ‪ .‬ﻭﻴﺘﻌﻴﻥ ﻤﻜﺎﻥ ﺍﻟﺜﻘل ‪ D‬ﺒﺎﺤﺩﺍﺜﻴﻪ ﺍﻟﺸﺎﻗﻭﻟﻲ ‪ y D‬ﺃﻤﺎ ﺍﻟﺒﻜﺭﺓ ‪ ، A‬ﺍﻟﺘﻲ‬
‫ﺘﺩﻭﺭ ﺤﻭل ﻨﻘﻁﺔ ﺜﺎﺒﺘﺔ‪ ،‬ﻓﻴﺘﻌﻴﻥ ﻤﻭﻀﻌﻬﺎ ﺒﺯﺍﻭﻴﺔ ﻤﺎ ﻭﺃﺨﻴﺭ ﺍﹰ ﻨﻌ ﻴﻥ ﻤﻭﻀﻊ ﺍﻟﺜﻘل ‪B‬‬
‫ﺒﺎﺤﺩﺍﺜﻴﻪ ﺍﻟﺸﺎﻗﻭﻟﻲ ‪ y B‬ﻭﻴﻜﻭﻥ ﻤﺠﻤﻭﻉ ﺍﻟﻭﺴﻁﺎﺀ ﻏﻴﺭ ﺍﻟﻤﺴﺘﻘﻠﺔ ﻫﻭ ﺴﺘﺔ ﻭﺴﻁﺎﺀ‪.‬‬
‫ﻓﺈﺫﺍ ﻋﻠﻤﻨﺎ ﺃﻥ ﺤﺭﻜﺔ ﺍﻟﺜﻘل ‪ E‬ﻭﺩﻭﺭﺍﻥ ﺍﻟﺒﻜﺭﺓ ‪ C‬ﻴﺘﻌﻴﻨﺎﻥ ﺒﻬﺒﻭﻁ ﺍﻟﺜﻘل ‪ D‬ﻭﺃﻥ‬
‫ﺤﺭﻜﺔ ﻤﺭﻜﺯ ﻜﺘﻠﺘﻪ ﺍﻟﺒﻜﺭﺓ ‪ C‬ﻭﺩﻭﺭﺍﻥ ﺍﻟﺒﻜﺭﺓ ‪ A‬ﻴﺘﻌﻴﻨﺎﻥ ﺒﻬﺒﻭﻁ ﺍﻟﺜﻘل ‪ B‬ﻓﺴﻨﺠﺩ ﺃﻥ‬

‫‪- 38 -‬‬

‫)‪Create PDF files without this message by purchasing novaPDF printer (http://www.novapdf.com‬‬
‫ﺍﻟﻭﺴﻁﺎﺀ ﺍﻟﺴﺘﺔ ﻏﻴﺭ ﺍﻟﻤﺴﺘﻘﻠﺔ ﺘﻨﺨﻔﺽ ﺒﻤﻘﺩﺍﺭ ﺃﺭﺒﻌﺔ ﻭﺒﺎﻟﺘﺎﻟﻲ ﻴﻜﻭﻥ ﻋﺩﺩ ﺍﻟﻭﺴﻁﺎﺀ‬
‫ﺍﻟﻤﺴﺘﻘﻠﺔ ﻫﻭ )‪. (6  4  2‬‬
‫ﻨﺫﻜﺭ ﺃﺨﻴﺭﺍﹰ ﻁﺭﻴﻘﺔ ﺜﺎﻟﺜﺔ ﻟﺘﻌﻴﻴﻥ ﻋﺩﺩ ﺍﻟﻭﺴﻁﺎﺀ ﺍﻟﻤﺴﺘﻘﻠﺔ ﺘﺘﻠﺨﺹ ﺒﻜﺘﺎﺒﺔ‬
‫ﻤﻌﺎﺩﻻﺕ ﺍﻻﺭﺘﺒﺎﻁ ﺍﻷﺭﺒﻊ ﺍﻟﺘﺎﻟﻴﺔ‪:‬‬
‫ﺃ‪ -‬ﻁﻭل ﺍﻟﺨﻴﻁ ﺍﻷﻭل ﺜﺎﺒﺕ‪.‬‬
‫ﺏ‪ -‬ﻁﻭل ﺍﻟﺨﻴﻁ ﺍﻟﺜﺎﻨﻲ ﺜﺎﺒﺕ‪.‬‬
‫ﺝ‪ -‬ﻤﻌﺎﺩﻟﺔ ﺍﻟﻤﺴﺘﻭﻱ ﺍﻟﻤﺎﺌل ﺍﻷﻭل‪.‬‬
‫ﺩ‪ -‬ﻤﻌﺎﺩﻟﺔ ﺍﻟﻤﺴﺘﻭﻱ ﺍﻟﻤﺎﺌل ﺍﻟﺜﺎﻨﻲ‪.‬‬
‫ﻭﻫﻜﺫﺍ ﻴﺼﺒﺢ ﻋﺩﺩ ﺍﻟﻭﺴﻁﺎﺀ ﺍﻟﻤﺴﺘﻘﻠﺔ ‪ 6-4=2‬ﻜﻤﺎ ﺭﺃﻴﻨﺎ ﺴﺎﺒﻘﺎﹰ‪.‬‬
‫ﻟﻨﻨﺘﻘل ﺍﻵﻥ ﻻﻴﺠﺎﺩ ﺍﻟﻤﺠﻬﻭﻟﻴﻥ ﺍﻟﻤﻁﻠﻭﺒﻴﻥ ﻭﻟﻬﺫﺍ ﻨﺤﺴﺏ ﺍﻟﻌﻤل ﺍﻻﻓﺘﺭﺍﻀﻲ‬
‫ﺒﻌﺩ ﻤﻼﺤﻅﺔ ﺃﻨﻪ ﻋﻨﺩﻤﺎ ﻴﻬﺒﻁ ﺍﻟﺜﻘل ‪ B‬ﻴﺠﺭ ﻤﻌﻪ ﺍﻟﺜﻘل ‪ D‬ﺍﻟﺫﻱ ﻴﺒﺩﻭ ﻜﺄﻨﻪ ﻴﺘﺤﺭﻙ‬
‫ﻋﻠﻰ ﻤﺴﺘﻭﹴ ﻤﺎﺌل ﺒﺯﺍﻭﻴﺔ ‪ ‬ﻭﻫﻜﺫﺍ ﻨﺠﺩ‪:‬‬
‫‪ P1 sin y D  P2yD  P1 sin y B  P2 sin y B  P4 sin y B  P3y B  0‬‬
‫)‪(1‬‬
‫ﻭﻤﻥ ﺍﻟﺤﺩﻴﻥ ﺍﻷﻭﻟﻴﻥ ﻨﺠﺩ‪:‬‬
‫‪P2  P1 sin ‬‬ ‫)‪(2‬‬
‫ﻭﻤﻥ ﺍﻟﺤﺩﻭﺩ ﺍﻷﺭﺒﻊ ﺍﻟﺒﺎﻗﻴﺔ ﻨﺠﺩ‪:‬‬
‫)‪(P1  P2  P4 ) sin   P3 (3‬‬
‫ﻓﺈﺫﺍ ﺒﺩﻟﻨﺎ ‪ sin ‬ﺒﻘﻴﻤﺘﻬﺎ ﻤﻥ )‪ (2‬ﻓﻲ )‪ (3‬ﻨﺤﺼل ﻋﻠﻰ ﻗﻴﻤﺔ ﺍﻟﺜﻘل ‪ P3‬ﺍﻟﺫﻱ‬
‫ﻴﺅﻤﻥ ﺍﻟﺘﻭﺍﺯﻥ ﻭﻫﻲ‪:‬‬
‫‪P‬‬
‫) ‪P3  2 (P1  P2  P4‬‬
‫‪P1‬‬
‫‪ -4‬ﺘﺘﺤﺭﻙ ﺤﻠﻘﺔ ﺜﻘﻴﻠﺔ ﻋﻠﻰ ﺴﻠﻙ ﺩﺍﺌﺭﻱ ﺜﺎﺒﺕ ﻨﺼﻑ ﻗﻁﺭﻩ ‪ a‬ﻴﻘﻊ ﻓﻲ‬
‫ﻤﺴﺘﻭﹴ ﺸﺎﻗﻭﻟﻲ ﺸﻜل )‪ .(1.8‬ﺍﺩﺭﺱ ﺘﻭﺍﺯﻥ ﻫﺫﻩ ﺍﻟﺤﻠﻘﺔ ﻋﻠﻤﺎﹰ ﺃﻨﻬﺎ ﺘﺘﺄﺜﺭ ﺒﻘﻭﺓ ﺜﺎﺒﺘ ﺔ‬
‫‪‬‬ ‫‪‬‬
‫‪) F  k BM‬ﺍﻨﻅﺭ ﺍﻟﺸﻜل( ﻭﺃﻥ ﺍﻻﺭﺘﺒﺎﻁ ﻤﺜﺎﻟﻲ‪.‬‬

‫‪- 39 -‬‬

‫)‪Create PDF files without this message by purchasing novaPDF printer (http://www.novapdf.com‬‬
(1.8) ‫ﺍﻟﺸﻜل‬
: ‫ﺍﳊﻞ‬
:‫ﺸﺭﻁ ﺍﻟﺘﻭﺍﺯﻥ ﻫﻭ‬
 
 Fi . ri  0 (1)
i
:‫ﻨﺴﻘﻁ ﻋﻠﻰ ﺍﻟﻤﻤﺎﺱ ﻓﻨﺠﺩ ﺃﻥ ﺍﻟﻌﻤل ﺍﻻﻓﺘﺭﺍﻀﻲ ﻴﺴﺎﻭﻱ‬
FT rT  PT rT  0 (2)
:‫ ﻨﻼﺤﻅ ﺃﻥ‬.‫ﻤﻊ ﺍﻟﻌﻠﻡ ﺃﻥ ﻋﻤل ﺍﻟﻘﻭﻯ ﺍﻟﻨﺎﻅﻤﻴﺔ ﻴﺴﺎﻭﻱ ﺍﻟﺼﻔﺭ‬
  
FT  F cos  k.2a sin . cos  ak sin  (3)
2 2 2
PT  P cos(  )   P cos  (4)

rT  a (5)

:‫( ﻓﻨﺠﺩ‬2) ‫( ﻓﻲ‬5) ‫( ﻭ‬4) ‫( ﻭ‬3) ‫ﻨﺒﺩل‬

( ka 2 sin   P cos )  0

:‫ ﺍﻟﻤﻘﺎﺒﻠﺔ ﻟﻭﻀﻊ ﺍﻟﺘﻭﺍﺯﻥ‬ 0 ‫ﻭﻤﻨﻪ ﻨﺤﺴﺏ ﺍﻟﺯﺍﻭﻴﺔ‬


P
tg 0 
2
ka

- 40 -

Create PDF files without this message by purchasing novaPDF printer (http://www.novapdf.com)
‫‪ -5‬ﺘﺘﺤﺭﻙ ﺤﻠﻘﺔ ﺜﻘﻴﻠﺔ ‪ M‬ﻋﻠﻰ ﺴﻠﻙ ﻟﻭﻟﺒﻲ ﻤﻌﺎﺩﻟﺘﻪ‪:‬‬

‫‪x  a cos  , y  a sin  , z  b‬‬

‫ﻭﺘﺘﺄﺜﺭ ﺒﻘﻭﺓ ﻤﺭﻭﻨﺔ‪ ،‬ﻨﺎﺘﺠﺔ ﻋﻥ ﺭﺒﻁ ﺍﻟﺤﻠﻘﺔ ﺒﻨﺎﺒﺽ ﻁﺭﻓﻪ ﺍﻟﺜﺎﻨﻲ ﻤﺜﺒﺕ ﻓﻲ‬
‫‪‬‬ ‫‪2 ‬‬
‫ﻤﺭﻜﺯ ﺍﻻﺤﺩﺍﺜﻴﺎﺕ‪ ،‬ﻫﻲ ‪ F  m OM‬ﻴﻁﻠﺏ ﺩﺭﺍﺴ ﺔ ﺘﻭﺍﺯﻥ ﺍﻟﺤﻠﻘﺔ ‪M‬‬
‫ﻭﺤﺴﺎﺏ ﺭﺩ ﺍﻟﻔﻌل ﻓﻲ ﻤﻭﻀﻊ ﺍﻟﺘﻭﺍﺯﻥ‪.‬‬

‫‪ -6‬ﻴﺘﻡ ﺭﻓﻊ ﺜﻘل ‪ P‬ﻋﻥ ﻁﺭﻴﻕ ﺍﻟﺘﺄﺜﻴﺭ ﺒﻘﻭﺓ ‪ F‬ﻋﻠﻰ ﺫﺭﺍﻉ ﻁﻭﻟﻪ ‪ ‬ﺸﻜل‬
‫)‪ (1.9‬ﻴﺩﻭﺭ ﺼﺎﻤﻭﻟﺔ ﺨﻁﻭﺘﻬﺎ ‪ b‬ﺘﺭﻓﻊ ﺒﺩﻭﺭﻫﺎ ﺍﻟﺜﻘل ﺇﻟﻰ ﺃﻋﻠﻰ )ﺭﺍﻓﻌﺔ ﺍﻟﺴﻴﺎﺭﺓ(‪.‬‬
‫‪‬‬ ‫‪‬‬
‫ﺍﺤﺴﺏ ﺍﻟﻌﻼﻗﺔ ﺒﻴﻥ ﺍﻟﻘﻭﺘﻴﻥ‪ :‬ﺍﻟﻘﻭﺓ ﺍﻟﻤﺤﺭﻜﺔ ‪ F‬ﻭﺍﻟﻘﻭﺓ ﺍﻟﻤﻘﺎﻭﻤﺔ ‪ P‬ﻟﻜﻲ ﻴﺘﻡ‬
‫ﺘﻭﺍﺯﻥ ﺍﻟﺠﻤﻠﺔ‪.‬‬

‫ﺍﻟﺸﻜل )‪(1.9‬‬

‫‪ -7‬ﺍﺩﺭﺱ ﺘﻭﺍﺯﻥ ﻨﻘﻁﺔ ﻤﺎﺩﻴﺔ ﺜﻘﻴﻠﺔ ﻋﻠﻰ ﻜﺭﺓ ﺒﻁﺭﻴﻘﺔ ﻤﻀﺎﺭﻴﺏ ﻻﻏﺭﺍﻨﺞ‪.‬‬
‫‪‬‬ ‫‪‬‬
‫‪ -8‬ﺍﻭﺠﺩ ﺍﻟﻌﻼﻗﺔ ﺒﻴﻥ ﺍﻟﻘﻭﺘﻴﻥ ‪ F‬ﻭ ‪ P‬ﻟﻜﻲ ﻴﺘﻡ ﺘﻭﺍﺯﻥ ﺍﻟﺠﻤﻠﺔ ﺍﻟﻤﻭﻀﺤﺔ‬
‫ﻋﻠﻰ ﺍﻟﺸﻜل )‪ (1.10‬ﻤﻊ ﺍﻟﻌﻠﻡ ﺃﻥ ﻜﺎﻓﺔ ﺍﻷﺫﺭﻉ ﻤﺘﺠﺎﻨﺴﺔ ﻭﻤﺘﺴﺎﻭﻴﺔ ﺍﻟﻁﻭل ﻭﺃﻥ‬
‫ﺍﻟﺒﻌﺩ ‪ AB‬ﺜﺎﺒﺕ‪.‬‬

‫‪- 41 -‬‬

‫)‪Create PDF files without this message by purchasing novaPDF printer (http://www.novapdf.com‬‬
‫ﺸﻜل )‪(1.10‬‬

‫‪ - 9‬ﻋﺎﺭﻀﺘﺎﻥ ﺜﻘﻴﻠﺘﺎﻥ ﻤﺘﺠﺎﻨﺴﺘﺎﻥ ‪ OA‬ﻭ ‪ AB‬ﻤﺘﻤﻔﺼﻠﺘﺎﻥ ﻓﻲ ‪ ،A‬ﻭ ‪O‬‬


‫ﻤﻔﺼل ﺜﺎﺒﺕ‪ .‬ﺘﺅﺜﺭ ﻋﻠﻰ ﺍﻟﻨﻬﺎﻴﺔ ‪ B‬ﻗﻭﺓ ﻨﺼﻨﻊ ﺯﺍﻭﻴﺔ ‪ ‬ﻤﻊ ﺍﻷﻓﻕ ﺍﺤﺴﺏ ﺍﻟﻘﻭﺓ‬
‫‪‬‬
‫‪ F‬ﻓﻲ ﻭﻀﻊ ﺍﻟﺘﻭﺍﺯﻥ ﺸﻜل )‪.(1.11‬‬

‫ﺍﻟﺸﻜل )‪(1.11‬‬

‫‪ -10‬ﺍﻋﺩ ﺤل ﺍﻟﻤﺴﺄﻟﺔ ﺍﻟﺴﺎﺒﻘﺔ ﻋﻨﺩﻤﺎ ﻴﻜﻭﻥ ﻟﺩﻴﻨﺎ ﺜﻼﺜﺔ ﻋﻭﺍﺭﺽ‬


‫‪ BC, AB, OA‬ﻤﺘﻤﻔﺼﻠﺔ ﻓﻲ ‪ A‬ﻭ ‪ B. O‬ﻤﻔﺼل ﺜﺎﺒﺕ‪ .‬ﺘﺅﺜﺭ ﻋﻠﻰ ‪) C‬ﻨﻬﺎﻴﺔ‬
‫‪‬‬
‫‪ (BC‬ﻗﻭﺓ ﺃﻓﻘﻴﺔ ‪. F‬‬

‫‪- 42 -‬‬

‫)‪Create PDF files without this message by purchasing novaPDF printer (http://www.novapdf.com‬‬
‫‪ -11‬ﻤﺴﺩﺱ ‪ ABCDEF‬ﻤﺅﻟﻑ ﻤﻥ ﺴﺘﺔ ﺃﻀﻼﻉ ﺜﻘﻴﻠﺔ ﻤﺘﻤﺎﺜﻠﺔ ﻭﻤﺘﺠﺎﻨﺴﺔ‬
‫‪‬‬
‫ﻁﻭل ﻜل ﻤﻨﻬﺎ ‪ a‬ﻭﺜﻘﻠﻪ ‪ P‬ﺘﺅﺜﺭ ﻋﻠﻰ ﺍﻟﻀﻠﻊ ‪ ED‬ﻗﻭﺓ ﺸﺎﻗﻭﻟﻴﺔ ﺘﺠﻌل ﺍﻟﻤﺴﺩﺱ‬
‫ﻴﺘﻭﺍﺯﻥ ﻓﻲ ﻭﻀﻌﻪ ﺍﻟﻤﻨﺘﻅﻡ‪ .‬ﺍﺤﺴﺏ ﻫﺫﻩ ﺍﻟﻘﻭﺓ‪.‬‬

‫ﺍﻟﺸﻜل )‪(1.12‬‬
‫‪ -12‬ﻜﺘﻠﺘﺎﻥ ﻤﺎﺩﻴﺘﺎﻥ ﺜﻘﻴﻠﺘﺎﻥ ﻴﻤﻜﻨﻬﻤﺎ ﺍﻟﺤﺭﻜﺔ ﻋﻠﻰ ﻤﺴﺘﻭﻴﻴﻥ ﻤﺎﺌﻠﻴﻥ ﻭﻫﻤﺎ‬
‫ﻤﺭﺒﻭﻁﺘﺎﻥ ﺒﺨﻴﻁ ﻴﻤﺭ ﻋﻠﻰ ﺒﻜﺭﺓ ﻤﺜﺒﺘﺔ ﻓﻲ ﺍﻟﻨﻘﻁﺔ ‪) A‬ﻨﻘﻁﺔ ﺘﻼﻗﻲ ﺍﻟﻤﺴﺘﻭﻴﻴﻥ(‪.‬‬
‫ﺍﻭﺠﺩ ﺸﺭﻁ ﺍﻟﺘﻭﺍﺯﻥ‪.‬‬

‫‪ -13‬ﺃﻋﺩ ﺤل ﺍﻟﻤﺴﺄﻟﺔ ﺍﻟﺴﺎﺒﻘﺔ ﻋﻨﺩﻤﺎ ﻴﺴﺘﺒﺩل ﺍﻟﺨﻴﻁ ﺒﺨﻴﻁ ﻤﺭﻥ ﺒﺤﻴﺙ ﺘﺘﺄﺜﺭ‬
‫‪‬‬ ‫‪‬‬
‫ﻜﻠﺘﺎ ﺍﻟﻜﺘﻠﺘﻴﻥ ﺒﺎﻹﻀﺎﻓﺔ ﺇﻟﻰ ﺜﻘﻠﻬﻤﺎ ﺒﻘﻭﺘﻲ ﻤﺭﻭﻨﺔ ‪. F2 , F1‬‬

‫‪ -14‬ﺍﻭﺠﺩ ﻤﻭﻀﻊ ﺘﻭﺍﺯﻥ ﺤﻠﻘﺔ ﺜﻘﻴﻠﺔ ﺘﺘﺤﺭﻙ ﻋﻠﻰ ﺴﻠﻙ ﺜﺎﺒﺕ ‪ AB‬ﻴﻘﻊ ﻓﻲ‬
‫‪‬‬
‫ﻤﺴﺘﻭﹴ ﺸﺎﻗﻭﻟﻲ ﻭﺘﺘﺄﺜﺭ ﺒﻘﻭﺓ ﺜﺎﺒﺘﺔ ‪ F‬ﺘﻤﺭ ﻤﻥ ﺍﻟﻨﻘﻁﺔ ‪ A‬ﺸﻜل )‪.(1.13‬‬

‫‪- 43 -‬‬

‫)‪Create PDF files without this message by purchasing novaPDF printer (http://www.novapdf.com‬‬
‫ﺍﻟﺸﻜل )‪(1.13‬‬

‫‪‬‬ ‫‪‬‬
‫‪ -15‬ﺃﻋﺩ ﺤل ﺍﻟﻤﺴﺄﻟﺔ )‪ (4‬ﻋﻨﺩﻤﺎ ﺘﺘﺄﺜﺭ ﺍﻟﺤﻠﻘﺔ ﺒﺎﻟﻘﻭﺓ ‪. F   k AM‬‬

‫‪ -16‬ﺘﺘﺤﺭﻙ ﺤﻠﻘﺔ ﺜﻘﻴﻠﺔ ‪ M‬ﻋﻠﻰ ﻤﺤﻭﺭ ‪ AB‬ﻴﻘﻊ ﻓﻲ ﻤﺴﺘﻭﹴ ﺸﺎﻗﻭﻟﻲ ﻴﺼﻨﻊ‬


‫ﺯﺍﻭﻴﺔ ﺜﺎﺒﺘﺔ ‪ ‬ﻤﻊ ﺍﻟﻤﺴﺘﻭﻱ ‪ .xoz‬ﻴﺅﺜﺭ ﻓﻲ ﺍﻟﺤﻠﻘﺔ‪ ،‬ﺒﺎﻻﻀﺎﻓﺔ ﺇﻟﻰ ﺜﻘﻠﻬﺎ‪ ،‬ﻗﻭﺓ ﺩﻓﻊ‬
‫‪‬‬ ‫‪‬‬
‫)‪ F  k OM (k  0‬ﺸﻜل )‪ .(1.14‬ﺍﺩﺭﺱ ﺘﻭﺍﺯﻥ ﺍﻟﺤﻠﻘﺔ‪.‬‬

‫ﺍﻟﺸﻜل )‪(1.14‬‬

‫‪- 44 -‬‬

‫)‪Create PDF files without this message by purchasing novaPDF printer (http://www.novapdf.com‬‬
‫‪ -1‬ﺍﻜﺘﺏ ﻤﻌﺎﺩﻻﺕ ﺍﻻﺭﺘﺒﺎﻁ ﻭﺒﺭﻫﻥ ﺃﻥ ﻟﻠﻤﺴﺄﻟﺔ ﻭﺴﻴﻁ ﻭﺍﺤﺩ‪.‬‬

‫‪ -2‬ﺍﺩﺭﺱ ﺘﻭﺍﺯﻥ ﺍﻟﺤﻠﻘﺔ ﺍﻋﺘﻤﺎﺩﺍﹰ ﻋﻠﻰ ﻤﺒﺩﺃ ﺍﻟﻌﻤل ﺍﻻﻓﺘﺭﺍﻀﻲ‪.‬‬

‫‪- 45 -‬‬

‫)‪Create PDF files without this message by purchasing novaPDF printer (http://www.novapdf.com‬‬
- 46 -

Create PDF files without this message by purchasing novaPDF printer (http://www.novapdf.com)
‫‪‬‬

‫ﻣـﻌـﺎﺩﻻﺕ ﺍﻟـﺤـﺮﻛـﺔ‬

‫ﺭﺃﻴﻨﺎ ﻓﻲ ﺍﻟﻔﺼل ﺍﻟﺴﺎﺒﻕ ﻜﻴﻑ ﻨﺤل ﻤﺴﺄﻟﺔ ﺘﻭﺍﺯﻥ ﻤﺠﻤﻭﻋﺔ ﻤﺎﺩﻴﺔ ﻋﻥ‬
‫ﻁﺭﻴﻕ ﺍﻟﻤﻴﻜﺎﻨﻴﻙ ﺍﻟﺘﺤﻠﻴﻠﻲ‪ .‬ﻭﺴﻨﺘﺎﺒﻊ ﻫﺫﺍ ﺍﻟﻤﺴﻌﻰ ﻓﻨﺭﻯ ﻜﻴﻑ ﻴﻤﻜﻥ ﻜﺘﺎﺒﺔ ﻤﻌﺎﺩﻻﺕ‬
‫ﺍﻟﺤﺭﻜﺔ ﺃﻴﻀﺎﹰ ﻭﺍﻴﺠﺎﺩ ﻗﻭﺍﻨﻴﻨﻬﺎ‪ ،‬ﻭﻴﺘﻡ ﺫﻟﻙ ﺒﻌﺩﺓ ﻁﺭﻕ ﻤﻥ ﺃﻫﻤﻬﺎ ﻤﻌﺎﺩﻻﺕ ﻻﻏﺭﺍﻨﺞ‬
‫ﺍﻟﺘﻲ ﺴﻨﺭﺍﻫﺎ ﻓﻲ ﻫﺫﺍ ﺍﻟﻔﺼل‪.‬‬
‫‪ - 10‬ﺍﳌﺒﺪﺃ ﺍﻟﺪﻳﻨﺎﻣﻴﻜﻲ )ﺍﻟﺘﺤﺮﻳﻜﻲ( ﻟﻼﻧﺘﻘﺎﻻﺕ ﺍﻻﻓﱰﺍﺿﻴﺔ )ﻣﻌﺎﺩﻟﺔ‬
‫ﺩﺍﳌﺒﲑ ‪ -‬ﻻﻏﺮﺍﻧﺞ(‪:‬‬
‫ﻟﻨﻜﺘﺏ ﻤﻌﺎﺩﻟﺔ ﺤﺭﻜﺔ ﺍﻟﻨﻘﻁﺔ ‪ M i‬ﻤﻥ ﺍﻟﻤﺠﻤﻭﻋﺔ ﺍﻟﻤﺎﺩﻴﺔ ﺍﻟﻤﺩﺭﻭﺴﺔ ﻁﺒﻘﺎﹰ ﻟـ‬
‫‪‬‬
‫)‪ (1.44‬ﺜﻡ ﻨﻀﺭﺏ ﻁﺭﻓﻲ ﺍﻟﻤﻌﺎﺩﻟﺔ ﺍﻟﻨﺎﺘﺠﺔ ﺒـ ‪  ri‬ﻭﻨﺠﻤﻊ ﺃﺨﻴﺭﺍﹰ ﺒـ ‪ i‬ﻓﻨﺤﺼل‬
‫ﻋﻠﻰ ﺍﻟﻤﻌﺎﺩﻟﺔ ﺍﻟﺘﺎﻟﻴﺔ‪:‬‬
‫‪N‬‬ ‫‪‬‬ ‫‪   N  ‬‬
‫‪  i i i . ri   R i . ri  0‬‬
‫‪‬‬ ‫‪F‬‬ ‫‪‬‬ ‫‪m‬‬ ‫‪W‬‬ ‫)‪(2.1‬‬
‫‪i 1‬‬ ‫‪‬‬ ‫‪i 1‬‬
‫ﻭﺇﺫﺍ ﻓﺭﻀﻨﺎ ﺃﻥ ﺍﻟﻤﺠﻤﻭﻋﺔ ﺍﻟﻤﺎﺩﻴﺔ ﺘﺘﺄﺜﺭ ﺒﺎﺭﺘﺒﺎﻁﺎﺕ ﻤﺜﺎﻟﻴﺔ ﻫﻭﻟﻭﻨﻭﻤﻴﺔ ﺜﻨﺎﺌﻴﺔ‬
‫ﺍﻟﺠﺎﻨﺏ ﻻﻏﻴﺭ ﻭﺭﻤﺯﻨﺎ ﻟﻠﻤﻘﺩﺍﺭ ‪ m i Wi‬ﺒـ ‪) J i‬ﻗﻭﺓ ﺍﻟﻌﻁﺎﻟﺔ( ﻓﺈﻥ ﺍﻟﻤﻌﺎﺩﻟﺔ )‪(2.1‬‬
‫ﺘﻭﻀﻊ ﻜﻤﺎﻴﻠﻲ‪:‬‬
‫‪N‬‬ ‫‪‬‬ ‫‪  ‬‬ ‫‪N   ‬‬
‫‪‬‬ ‫‪m‬‬ ‫‪W‬‬ ‫‪‬‬
‫‪  i i i  i  (Fi  J i ). ri  0‬‬
‫‪F‬‬ ‫‪‬‬ ‫‪.‬‬‫‪‬‬ ‫‪r‬‬ ‫‪‬‬ ‫)‪(2.2‬‬
‫‪i 1 ‬‬ ‫‪‬‬ ‫‪i 1‬‬
‫ﺃﻭ ﺒﺎﻟﺸﻜل‪:‬‬
‫)‪.x i   Yi  m i y i y i   Z  m i z i z i   0 (2.3‬‬
‫‪ ‬‬ ‫‪‬‬ ‫‪‬‬
‫‪  X i  m i x i‬‬ ‫‪‬‬ ‫‪‬‬ ‫‪‬‬ ‫‪‬‬ ‫‪ ‬‬
‫‪‬‬

‫ﺘﺴﻤﻰ ﻫﺫﻩ ﺍﻟﻤﻌﺎﺩﻟﺔ ﺃﺤﻴﺎﻨﺎﹰ ﻤﻌﺎﺩﻟﺔ ﺩﺍﻟﻤﺒﻴﺭ‪-‬ﻻﻏﺭﺍﻨﺞ ﻭﻫﻲ ﺘﻌﺒﺭ ﻋﻤﺎ ﻴﺴﻤﻰ‬
‫ﺍﻟﻤﺒﺩﺃ ﺍﻟﺩﻴﻨﺎﻤﻴﻜﻲ ﻟﻼﻨﺘﻘﺎﻻﺕ ﺍﻻﻓﺘﺭﺍﻀﻴﺔ ﺍﻟﺫﻱ ﻴﻨﺹ ﻋﻠﻰ ﻤﺎﻴﻠﻲ‪:‬‬

‫‪- 47 -‬‬

‫)‪Create PDF files without this message by purchasing novaPDF printer (http://www.novapdf.com‬‬
‫"ﺘﺠﺭﻯ ﺤﺭﻜﺔ ﻤﺠﻤﻭﻋﺔ ﻤﺎﺩﻴﺔ ﺨﺎﻀﻌﺔ ﻻﺭﺘﺒﺎﻁﺎﺕ ﻤﺜﺎﻟﻴﺔ ﻫﻭﻟﻭﻨﻭﻤﻴﺔ ﺒﺤﻴﺙ‬
‫ﻴﻜﻭﻥ ﻤﺠﻤﻭﻉ ﺍﻷﻋﻤﺎل ﺍﻻﻓﺘﺭﺍﻀﻴﺔ ﻟﻜﺎﻓﺔ ﺍﻟﻘﻭﻯ ﺍﻟﻤﺅﺜﺭﺓ ﻋﻠﻴﻬﺎ )ﺍﻟﻘﻭﻯ ﺍﻟﻔﻌﺎﻟﺔ‬
‫ﻭﻗﻭﻯ ﺍﻟﻌﻁﺎﻟﺔ( ﻴﺴﺎﻭﻱ ﺍﻟﺼﻔﺭ"‪.‬‬
‫ﻭﻤﻥ ﺍﻟﻭﺍﻀﺢ ﺃﻨﻪ ﻟﻜﻲ ﺘﺘﺤﻘﻕ ﺍﻟﻤﻌﺎﺩﻟﺔ )‪ (2.3‬ﻓﻲ ﺍﻟﺤﺎﻟﺔ ﺍﻟﻌﺎﻤﺔ ﻻﻴﻤﻜﻥ ﺃﻥ‬
‫ﻨﻘﻭل ﺇﻥ ﺃﻤﺜﺎل ﺍﻻﻨﺘﻘﺎﻻﺕ ﺍﻻﻓﺘﺭﺍﻀﻴﺔ ‪ zi , y i , x i‬ﻴﺠﺏ ﺃﻥ ﺘﺴﺎﻭﻱ ﺍﻟﺼﻔﺭ‬
‫ﻷﻥ ﻫﺫﻩ ﺍﻻﻨﺘﻘﺎﻻﺕ ﻏﻴﺭ ﻤﺴﺘﻘﻠﺔ ﻓﻬﻲ ﺘﺭﺘﺒﻁ ﻓﻴﻤﺎ ﺒﻴﻨﻬﺎ ﺒـ ‪ k‬ﻤﻌﺎﺩﻟﺔ ﻤﻥ ﺍﻟﻨﻭﻉ‬
‫)‪.(1.15‬‬
‫ﻟﻨﺒﺤﺙ‪ ،‬ﻋﻠﻰ ﺴﺒﻴل ﺍﻟﻤﺜﺎل‪ ،‬ﻋﻥ ﻤﻌﺎﺩﻟﺔ ﺤﺭﻜﺔ ﻨﻘﻁﺔ ﻤﺎﺩﻴﺔ ﻭﺍﺤﺩﺓ ﻁﻠﻴﻘﺔ ‪M‬‬
‫ﺍﻋﺘﻤﺎﺩﺍﹰ ﻋﻠﻰ ﺍﻟﻤﻌﺎﺩﻟﺔ )‪ (2.3‬ﺤﻴﺙ ﺘﻜﻭﻥ ﻭﺴﻁﺎﺀ ﺍﻟﻤﺴﺄﻟﺔ ﺍﻟﻤﺴﺘﻘﻠﺔ ﻫﻲ ﺇﺤﺩﺍﺜﻴﺎﺕ‬
‫ﺍﻟﻨﻘﻁﺔ ‪ M‬ﺃﻱ ) ‪ ( x, y, z‬ﻭﺘﻜﻭﻥ ﻜﺎﻓﺔ ﺍﻻﻨﺘﻘﺎﻻﺕ ‪ x‬ﻭ ‪ y‬ﻭ ‪ z‬ﻤﺴﺘﻘﻠﺔ‬
‫ﻭﺒﺎﻟﺘﺎﻟﻲ ﻟﻜﻲ ﺘﺘﺤﻘﻕ ﺍﻟﻤﻌﺎﺩﻟﺔ )‪ (2.3‬ﻴﺠﺏ ﺃﻥ ﻴﻜﻭﻥ‪:‬‬
‫‪‬‬ ‫‪‬‬ ‫‪‬‬
‫‪X  mx‬‬ ‫‪ 0 , Y  my‬‬ ‫‪ 0 , Z  mz‬‬ ‫‪0‬‬
‫‪‬‬ ‫‪‬‬
‫ﻭﻤﻨﻪ ‪ m W  F‬ﻭﻫﻲ ﻤﻌﺎﺩﻟﺔ ﺤﺭﻜﺔ ﻨﻘﻁﺔ ﻤﺎﺩﻴﺔ ﻁﻠﻴﻘﺔ ﻜﻤﺎ ﻴﺭﺍﻫﺎ ﻨﻴﻭﺘﻥ‪.‬‬
‫ﻟﻨﻌﻤﻡ ﺍﻟﻤﻌﺎﺩﻟﺔ )‪ (2.2‬ﻋﻠﻰ ﺍﻻﺭﺘﺒﺎﻁﺎﺕ ﺍﻟﻤﺤﺭﺭﺓ‪ ،‬ﻭﻟﺫﺍ ﻨﺘﺫﻜﺭ ﻤﺎ ﺭﺃﻴﻨﺎﻩ ﻓﻲ‬
‫ﺍﻟﻔﺼل ﺍﻟﺴﺎﺒﻕ ﺍﻥ ﺍﻻﻨﺘﻘﺎل ﻴﺤﺩﺙ ﺒﺘﺄﺜﻴﺭ ﻗﻭﻯ ﺍﻟﺭﺒﻁ ﻭﻟﻬﺫﺍ ﻴﻜﻭﻥ ﻋﻤﻠﻬﺎ ﻤﻭﺠﺒ ﺎﹰ‬
‫ﻭﻋﻨﺩﺌﺫ‪ ‬ﻨﺠﺩ ﻤﻥ ﺍﻟﻤﻌﺎﺩﻟﺔ )‪ (2.1‬ﺃﻥ‪:‬‬
‫‪N‬‬ ‫‪‬‬ ‫‪  ‬‬
‫‪ F  m W . r  0‬‬
‫‪ i i i  i‬‬ ‫)‪(2.4‬‬
‫‪i 1‬‬ ‫‪‬‬
‫ﻭﺒﺩﻤﺞ ﺍﻟﻤﻌﺎﺩﻟﺘﻴﻥ )‪ (2.4) ،(2.2‬ﻤﻊ ﺒﻌﻀﻬﻤﺎ ﻨﺤﺼل ﻋﻠﻰ ﺍﻟﺘﻌﺒﻴﺭ‬
‫ﺍﻟﺭﻴﺎﻀﻲ ﻟﻠﻤﺒﺩﺃ ﺍﻟﺩﻴﻨﺎﻤﻴﻜﻲ ﻟﻼﻨﺘﻘﺎﻻﺕ ﺍﻻﻓﺘﺭﺍﻀﻴﺔ‪:‬‬
‫‪N‬‬ ‫‪‬‬ ‫‪‬‬ ‫‪‬‬ ‫‪‬‬
‫‪  Fi  m i Wi . ri‬‬ ‫‪0‬‬ ‫)‪(2.5‬‬
‫‪i 1‬‬ ‫‪‬‬
‫"ﺃﻱ ﺃﻥ ﻤﺠﻤﻭﻉ ﺍﻷﻋﻤﺎل ﺍﻻﻓﺘﺭﺍﻀﻴﺔ ﺍﻟﺠﺯﺌﻴﺔ ﻟﻠﻘﻭﻯ ﺍﻟﻔﻌﺎﻟﺔ ﻭﻗﻭﻯ ﺍﻟﻌﻁﺎﻟﺔ‬
‫)ﺃﻭ ﻤﺎﻴﺴﻤﻰ ﺃﺤﻴﺎﻨﺎﹰ ﻤﺤﺼﻠﺔ ﺍﻟﻘﻭﻯ ﺍﻟﻀﺎﺌﻌﺔ( ﺍﻟﻤﺅﺜﺭﺓ ﻓﻲ ﻤﺠﻤﻭﻋﺔ ﻤﺎﺩﻴﺔ ﻻﻴﻤﻜﻥ‬
‫ﺃﻥ ﻴﻜﻭﻥ ﻤﻭﺠﺒﺎﹰ‪.‬‬

‫‪- 48 -‬‬

‫)‪Create PDF files without this message by purchasing novaPDF printer (http://www.novapdf.com‬‬
‫ﻭﺍﻟﺠﺩﻴﺭ ﺒﺎﻟﺫﻜﺭ ﺃﻥ ﺒﻌﺽ ﺍﻟﻤﺅﻟﻔﻴﻥ ﻻ ﻴﺄﺨﺫﻭﻥ ﺒﻌﻴﻥ ﺍﻻﻋﺘﺒﺎﺭ ﺍﻻﺭﺘﺒﺎﻁﺎﺕ‬
‫ﺍﻷﺤﺎﺩﻴﺔ‪ ،‬ﺇﺫ ﺃﻥ ﺃﻴﺔ ﻨﻘﻁﺔ ﻤﺎﺩﻴﺔ ﺘﺨﻀﻊ ﻻﺭﺘﺒﺎﻁﺎﺕ ﺃﺤﺎﺩﻴﺔ ﺘﻌﺘﺒﺭ ﻁﻠﻴﻘﺔ ﻋﻨﺩﻤﺎ‬
‫ﺘﺘﺤﺭﺭ ﻤﻥ ﺍﻟﻘﻴﺩ ﻭﺘﻌﺎﻤل ﻤﻌﺎﻤﻠﺔ ﺍﻟﻨﻘﻁﺔ ﺍﻟﻁﻠﻴﻘﺔ‪ ،‬ﻭﻁﺎﻟﻤﺎ ﺃﻨﻬﺎ ﻤﻭﺠﻭﺩﺓ ﻋﻠﻰ ﺍﻟﻘﻴﺩ‬
‫ﻓﺈﻨﻨﺎ ﻨﻌﺘﺒﺭ ﺍﺭﺘﺒﺎﻁﻬﺎ ﺜﻨﺎﺌﻴﺎﹰ )ﺍﻨﻅﺭ ﺍﻟﻤﺭﺠﻊ ‪.(6‬‬
‫‪ - 11‬ﺍﻻﺣﺪﺍﺛﻴﺎﺕ ﺍﳌﻌﻤﻤﺔ‪ ،‬ﺍﻟﺴﺮﻉ ﺍﳌﻌﻤﻤﺔ‪:‬‬
‫ﻟﻨﺒﺤﺙ ﻋﻥ ﻤﻌﺎﺩﻻﺕ ﺤﺭﻜﺔ ﻤﺠﻤﻭﻋﺔ ﻤﺎﺩﻴﺔ ﻤﺅﻟﻔﺔ ﻤﻥ ‪ N‬ﻨﻘﻁﺔ ﺘﺨﻀﻊ ﻟـ‬
‫‪ k‬ﺍﺭﺘﺒﺎﻁﺎﹰ ﻤﻘﻴﺩﺍﹰ‪ ،‬ﻟﺫﺍ ﻨﺨﺘﺎﺭ ﻋﺩﺩ ﺍﹰ ‪ n‬ﻤﻥ ﺍﻻﺤﺩﺍﺜﻴﺎﺕ ﺍﻟﺠﺩﻴﺩﺓ ﻨﺭﻤﺯ ﻟﻬﺎ ﺒﺎﻟﺭﻤﺯ ‪q j‬‬

‫ﺤﻴﺙ ) ‪. ( j  1, 2, ...,3N  k‬‬


‫ﻨﺴﻤﻲ ﻫﺫﻩ ﺍﻻﺤﺩﺍﺜﻴﺎﺕ ﺍﻟﺘﻲ ﻋﺩﺩﻫﺎ ‪) n  3N  k‬ﺒﻌﺩﺩ ﺩﺭﺠﺎﺕ ﺍﻟﺤﺭﻴﺔ‬
‫ﻟﻠﻤﺠﻤﻭﻋﺔ ﺍﻟﻤﺎﺩﻴﺔ( ﻨﺴﻤﻴﻬﺎ ﺍﻻﺤﺩﺍﺜﻴﺎﺕ ﺍﻟﻤﻌﻤﻤﺔ ﻭﻨﺨﺘﺎﺭﻫﺎ ﺒﺤﻴﺙ ﺘﺤﻘﻕ ﺸﺭﻁﻴﻥ‬
‫ﺃﺴﺎﺴﻴﻴﻥ‪:‬‬
‫‪ -1‬ﺃﻥ ﻴﻌﺒﺭ ﻋﻥ ﺍﻹﺤﺩﺍﺜﻴﺎﺕ ﺍﻟﻘﺩﻴﻤﺔ ‪ x i‬ﻭ ‪ yi‬ﻭ ‪ zi‬ﺒﺩﻻﻟﺔ ﺍﻹﺤﺩﺍﺜﻴﺎﺕ‬
‫ﺍﻟﺠﺩﻴﺩﺓ ﺒﺘﻭﺍﺒﻊ ﻤﺴﺘﻤﺭﺓ ﻭﻭﺤﻴﺩﺓ ﺍﻟﺘﻌﻴﻴﻥ ﻭﻗﺎﺒﻠﺔ ﻟﻼﺸﺘﻘﺎﻕ ﺒﻘﺩﺭ ﻤﺎﻨﺭﻴﺩ ﻭﻴﻤﻜﻥ ﺃﻥ‬
‫ﺘﺤﻭﻱ ﺍﻟﺯﻤﻥ ﺒﺸﻜل ﺼﺭﻴﺢ‪.‬‬
‫‪ -2‬ﺃﻥ ﺘﺤﻘﻕ ﻤﻌﺎﺩﻻﺕ ﺍﻻﺭﺘﺒﺎﻁ ﺘﻁﺎﺒﻘﺎﹰ‪.‬‬
‫ﻣﺜﺎﻝ‪:‬‬
‫ﻋﻨﺩﻤﺎ ﺘﺘﺤﺭﻙ ﻨﻘﻁﺔ ﻋﻠﻰ ﻜﺭﺓ ﻨﺼﻑ ﻗﻁﺭﻫﺎ ‪ a‬ﻓﺈﻥ ﻤﻌﺎﺩﻟﺔ ﺍﻻﺭﺘﺒﺎﻁ ﻫﻲ‪:‬‬
‫‪ f (x, y, z)  x  y  z  a  0‬ﻭﻴﻜﻭﻥ ﻋﺩﺩ ﺩﺭﺠﺎﺕ ﺍﻟﺤﺭﻴﺔ ﻫﻭ‬
‫‪2‬‬ ‫‪2‬‬ ‫‪2‬‬ ‫‪2‬‬

‫‪ . 3  1  2‬ﻨﺨﺘﺎﺭ ﺍﻻﺤﺩﺍﺜﻴﻴﻥ ﺍﻟﻤﻌﻤﻤﻴﻥ ﺍﻟﺘﺎﻟﻴﻴﻥ ‪ q 2  y‬ﻭ ‪ q1  x‬ﻭﻋﻨﺩﺌﺫ‪ ‬ﻴﻤﻜﻥ‬


‫ﺤﺴﺎﺏ ‪ z‬ﻤﻥ ﻤﻌﺎﺩﻟﺔ ﺍﻟﻜﺭﺓ ﺤﻴﺙ ﻨﺠﺩ‪ z  a 2  q12  q 22 :‬ﻭﺠﻤﻴﻌﻬﺎ ﺘﻭﺍﺒﻊ‬
‫ﻤﺴﺘﻤﺭﺓ ﻭﻭﺤﻴﺩﺓ ﺍﻟﺘﻌﻴﻴﻥ‪.‬‬
‫ﺃﻤﺎ ﻤﻌﺎﺩﻟﺔ ﺍﻻﺭﺘﺒﺎﻁ ﻓﺘﺤﻘﻕ ﺘﻁﺎﺒﻘﺎﹰ ﺃﻱ ﺃﻥ‪:‬‬
‫‪2‬‬ ‫‪2‬‬ ‫‪2‬‬ ‫‪2‬‬ ‫‪2‬‬ ‫‪2‬‬ ‫‪2‬‬ ‫‪2‬‬ ‫‪2‬‬ ‫‪2‬‬
‫‪x  y  z  a  0  q1  q 2  a  q1  q 2  a  0‬‬
‫ﻫﺫﺍ ﻭﻴﻤﻜﻥ ﺍﺨﺘﻴﺎﺭ ﺍﻻﺤﺩﺍﺜﻴﺎﺕ ﺍﻟﻤﻌﻤﻤﺔ ﺍﻟﺘﺎﻟﻴﺔ ‪ q1  , q 2  ‬ﻭﻋﻨﺩﺌﺫ‪‬‬
‫ﻴﻌﺒﺭ ﻋﻥ ﺍﻻﺤﺩﺍﺜﻴﺎﺕ ﺍﻟﻘﺩﻴﻤﺔ ‪ x , y, z‬ﻟﻠﻨﻘﻁﺔ ﺒﺩﻻﻟﺔ ﺍﻻﺤﺩﺍﺜﻴﺎﺕ ﺍﻟﻤﻌﻤﻤﺔ ﺒﺎﻟﺸﻜل‪:‬‬

‫‪- 49 -‬‬

‫)‪Create PDF files without this message by purchasing novaPDF printer (http://www.novapdf.com‬‬
‫‪x  a sin q1 cos q 2 , y  a sin q1 sin q 2 , z  a cos q1‬‬

‫ﻭﻫﻲ ﺘﻭﺍﺒﻊ ﻤﺴﺘﻤﺭﺓ ﻭﻭﺤﻴﺩﺓ ﺍﻟﺘﻌﻴﻴﻥ ﻭﻗﺎﺒﻠﺔ ﻟﻼﺸﺘﻘﺎﻕ ﺩﻭﻤﺎﹰ‪ ،‬ﺃﻤﺎ ﻤﻌﺎﺩﻟﺔ‬
‫ﺍﻻﺭﺘﺒﺎﻁ ﻓﺘﻜﻭﻥ‪:‬‬
‫‪x 2  y 2  z 2  a 2  a 2 sin 2  cos 2   a 2 sin 2  sin 2   a 2 cos 2   a 2  0‬‬
‫ﻭﻫﻲ ﺘﺤﻘﻕ ﺘﻁﺎﺒﻘﺎﹰ ﻜﻤﺎ ﻨﺭﻯ‪.‬‬

‫ﻭﻫﻜﺫﺍ ﻨﻌﺒﺭ ﻋﻥ ﺍﻻﺤﺩﺍﺜﻴﺎﺕ ‪ x1‬ﻭ ‪ yi‬ﻭ ‪ zi‬ﻓﻲ ﺍﻟﺤﺎﻟﺔ ﺍﻟﻌﺎﻤﺔ ﺒﺩﻻﻟﺔ‬


‫ﺍﻻﺤﺩﺍﺜﻴﺎﺕ ﺍﻟﺠﺩﻴﺩﺓ ﻭﺍﻟﺯﻤﻥ ﺒﺎﻟﻌﻼﻗﺎﺕ‪:‬‬
‫‪ ‬‬
‫) ‪ri  ri (q1 , q 2 ,...., q n , t ), (i  1,2,..., N‬‬ ‫)‪(2.6‬‬

‫ﺃﻭ‪:‬‬
‫‪x i  x i ( q1, q 2 ,..., q n , t ), y i  y i (q1, q 2 ,..., q n , t ),‬‬
‫) ‪. z i  z i (q1, q 2 ,..., q n , t‬‬ ‫)‪(2.7‬‬

‫ﻟﻨﻌﺒﺭ ﻋﻥ ﺴﺭﻋﺔ ﺍﻟﻨﻘﻁﺔ ‪ M i‬ﻓﻲ ﺍﻹﺤﺩﺍﺜﻴﺎﺕ ﺍﻟﺠﺩﻴﺩﺓ ﻭﻟﻬﺫﺍ ﻨﺸﺘﻕ )‪(2.6‬‬


‫ﺒﺎﻟﻨﺴﺒﺔ ﻟﻠﺯﻤﻥ‪:‬‬
‫)‪(2.8‬‬
‫‪‬‬ ‫‪‬‬ ‫‪‬‬ ‫‪‬‬ ‫‪‬‬ ‫‪‬‬ ‫‪‬‬
‫‪‬‬ ‫‪d ri  ri   ri ‬‬ ‫‪ ri   ri‬‬ ‫‪n r‬‬
‫‪i‬‬ ‫‪  ri‬‬
‫‪vi ‬‬
‫‪dt‬‬
‫‪‬‬
‫‪q1‬‬
‫‪q1 ‬‬
‫‪q 2‬‬
‫‪q 2  ... ‬‬
‫‪q n‬‬
‫‪qn ‬‬
‫‪t‬‬
‫‪‬‬
‫‪q j‬‬
‫‪qj ‬‬
‫‪t‬‬
‫‪‬‬
‫‪j1‬‬

‫ﺘﺴﻤﻰ ﻤﺸﺘﻘﺎﺕ ﺍﻻﺤﺩﺍﺜﻴﺎﺕ ﺍﻟﻤﻌﻤﻤﺔ ﺒﺎﻟﻨﺴﺒﺔ ﻟﻠﺯﻤﻥ‪ ،‬ﺃﻱ ‪ ، q j‬ﺒﺎﻟﺴﺭﻉ‬


‫‪‬‬

‫ﺍﻟﻤﻌﻤﻤﺔ‪.‬‬

‫ﻭﻴﺒﺩﻭ ﺒﻭﻀﻭﺡ ﻤﻥ )‪ (2.8‬ﻋﻨﺩ ﺍﺸﺘﻘﺎﻗﻬﺎ ﺠﺯﺌﻴﺎﹰ ﺒﺎﻟﻨﺴﺒﺔ ﻟـ ‪ q j‬ﺃﻥ‪:‬‬


‫‪‬‬

‫‪‬‬ ‫‪‬‬
‫‪ vi‬‬ ‫‪ ri‬‬
‫‪‬‬
‫‪‬‬ ‫)‪(2.9‬‬
‫‪q j‬‬ ‫‪q j‬‬

‫‪- 50 -‬‬

‫)‪Create PDF files without this message by purchasing novaPDF printer (http://www.novapdf.com‬‬
‫‪ - 12‬ﻣﻌﺎﺩﻻﺕ ﺍﳊﺮﻛﺔ‪:‬‬
‫ﻟﻠﺤﺼﻭل ﻋﻠﻰ ﻤﻌﺎﺩﻻﺕ ﺍﻟﺤﺭﻜﺔ ﻴﻨﺒﻐﻲ "ﺘﺭﺠﻤﺔ" ﺍﻟﻤﺒﺩﺃ ﺍﻟﺩﻴﻨﺎﻤﻴﻜﻲ‬
‫ﻟﻼﻨﺘﻘﺎﻻﺕ ﺍﻻﻓﺘﺭﺍﻀﻴﺔ )‪ (2.2‬ﺇﻟﻰ ﻟﻐﺔ ﺍﻹﺤﺩﺍﺜﻴﺎﺕ ﺍﻟﻤﻌﻤﻤﺔ‪ ،‬ﻭﻟﺫﺍ ﻨﺤﺴﺏ ﺤﺩﻭﺩﻩ‬
‫‪‬‬
‫ﺤﺩﺍﹰ ﻓﺤﺩﺍﹰ‪ ،‬ﻨﺤﺴﺏ ﺃﻭﻻﹰ ‪  ri‬ﻓﻨﺠﺩ‪:‬‬
‫‪n‬‬
‫‪ri‬‬
‫‪ri  ‬‬ ‫‪q j‬‬ ‫)‪(2.10‬‬
‫‪j1‬‬ ‫‪q j‬‬
‫‪N ‬‬
‫ﻓﻴﻭﻀﻊ ﻋﻨﺩﺌﺫ‪ ‬ﺒﺎﻟﺸﻜل‪:‬‬ ‫ﺃﻤﺎ ﺍﻟﻌﻤل ﺍﻻﻓﺘﺭﺍﻀﻲ ﻟﻠﻘﻭﻯ ﺍﻟﻔﻌﺎﻟﺔ ‪ Fi . ri‬‬
‫‪‬‬

‫‪i‬‬
‫‪‬‬
‫‪N ‬‬ ‫‪N‬‬ ‫‪ ‬‬
‫‪‬‬ ‫‪ ri‬‬
‫‪‬‬
‫‪ r ‬‬
‫‪‬‬ ‫‪Fi . ri   F i .‬‬ ‫)‪q j    Fi . i q j   Q j q j (2.11‬‬
‫‪i 1‬‬ ‫‪i‬‬ ‫‪j‬‬ ‫‪q j‬‬ ‫‪j‬‬ ‫‪i ‬‬ ‫‪q j ‬‬ ‫‪j‬‬
‫‪‬‬ ‫‪‬‬
‫ﺤﻴﺙ ‪ Q j‬ﺘﻌﻁﻰ ﺒﺎﻟﻌﻼﻗﺔ‪:‬‬
‫‪ ‬‬
‫‪ r ‬‬
‫‪Q j    Fi . i  ,‬‬ ‫) ‪( j  1,2,..., n‬‬ ‫)‪(2.12‬‬
‫‪i ‬‬
‫‪q j ‬‬
‫‪‬‬ ‫‪‬‬
‫ﻭﺘﺴﻤﻰ ﺍﻟﻘﻭﻯ ﺍﻟﻤﻌﻤﻤﺔ‪.‬‬
‫‪N‬‬ ‫‪‬‬ ‫‪‬‬
‫ﻨﻜﺘﺏ‪:‬‬ ‫ﻭﻟﺤﺴﺎﺏ ﺍﻟﺤﺩ ﺍﻟﺒﺎﻗﻲ ﺃﻱ ‪ m i Wi . ri‬‬
‫‪i 1‬‬
‫‪‬‬ ‫‪‬‬
‫‪n‬‬ ‫‪   ‬‬ ‫‪‬‬
‫‪‬‬
‫‪‬‬ ‫‪‬‬ ‫‪d v i  ri‬‬ ‫‪d‬‬ ‫‪ ri  ‬‬ ‫‪d  ri ‬‬
‫)‪ v i . q    v i . dt q q j (2.13‬‬
‫‪Wi . ri  ‬‬ ‫‪.‬‬ ‫‪q j  ‬‬ ‫‪‬‬
‫‪j‬‬ ‫‪dt q j‬‬ ‫‪j  dt ‬‬ ‫‪j‬‬ ‫‪j‬‬
‫‪ ‬‬ ‫‪ ‬‬ ‫‪‬‬
‫‪‬‬
‫ﻭﻤﻥ ﺍﻟﺴﻬل ﺤﺴﺎﺏ ﺍﻟﺤﺩ ﺍﻷﺨﻴﺭ ﺇﺫﺍ ﻋﻠﻤﻨﺎ ﺃﻥ ‪ ri‬ﺘﺨﻀﻊ ﻻﺸﺘﻘﺎﻗﻴﻥ ﺍﻷﻭل‬
‫ﺒﺎﻟﻨﺴﺒﺔ ﻟـ ‪ q j‬ﻭﺍﻟﺜﺎﻨﻲ ﺒﺎﻟﻨﺴﺒﺔ ﻟﻠﺯﻤﻥ ﻭﻴﻤﻜﻥ‪ ،‬ﻀﻤﻥ ﺸﺭﻭﻁ ﺍﻟﻤﺴﺄﻟﺔ‪ ،‬ﺍﻟﺘﺒﺩﻴل ﺒﻴﻥ‬
‫ﺍﻻﺸﺘﻘﺎﻗﻴﻥ ﺤﻴﺙ ﻨﺠﺩ‪:‬‬
‫‪‬‬ ‫‪‬‬
‫‪d  ‬‬ ‫‪ d ri  v i‬‬
‫‪ri ‬‬ ‫‪‬‬ ‫)‪(2.14‬‬
‫‪dt q j‬‬ ‫‪q j dt‬‬ ‫‪q j‬‬

‫ﻭﻁﺒﻘﺎﹰ ﻟـ )‪ (2.9‬ﻭ )‪ (2.14‬ﻴﻜﻭﻥ‪:‬‬

‫‪- 51 -‬‬

‫)‪Create PDF files without this message by purchasing novaPDF printer (http://www.novapdf.com‬‬
‫‪‬‬ ‫‪‬‬
‫‪‬‬ ‫‪‬‬
‫‪‬‬ ‫‪‬‬ ‫‪d    vi‬‬ ‫‪‬‬ ‫‪ v‬‬
‫‪Wi . ri    v i . ‬‬ ‫‪i‬‬
‫‪   v i . q q j‬‬ ‫)‪(2.15‬‬
‫‪j dt ‬‬ ‫‪q j‬‬ ‫‪j‬‬ ‫‪j‬‬
‫‪‬‬ ‫‪‬‬
‫ﻭﺒﺎﻟﻀﺭﺏ ﺒـ ‪ mi‬ﺜﻡ ﺍﻟﺠﻤﻊ ﺒـ ‪ i‬ﻤﻥ ‪ 1‬ﺤﺘﻰ ‪) N‬ﻜل ﻨﻘﻁ ﺍﻟﻤﺠﻤﻭﻋﺔ(‬
‫ﻨﺠﺩ‪:‬‬
‫‪‬‬ ‫‪‬‬ ‫‪‬‬ ‫‪   ‬‬
‫‪ ‬‬
‫‪ d    vi ‬‬ ‫‪‬‬ ‫‪ v ‬‬
‫‪‬‬ ‫‪m i Wi . ri ‬‬ ‫)‪  dtm i  v i .     m i  v i . i q j (2.16‬‬
‫‪i‬‬ ‫‪j  i‬‬ ‫‪‬‬ ‫‪q i  i‬‬ ‫‪‬‬ ‫‪q i ‬‬
‫‪‬‬ ‫‪‬‬ ‫‪‬‬ ‫‪‬‬ ‫‪‬‬
‫ﻭﻴﺒﺩﻭ ﺒﻭﻀﻭﺡ ﺃﻥ ﺍﻟﺤﺩ ﺍﻷﻭل ﻫﻭ ﻤﺸﺘﻕ ﺍﻟﻁﺎﻗﺔ ﺍﻟﺤﺭﻜﻴﺔ ﺍﻟﻜﻠﻴﺔ ﺒﺎﻟﻨﺴﺒﺔ‬
‫ﻟﻠﺴﺭﻉ ﺍﻟﻤﻌﻤﻤﺔ ‪ . q j‬ﺃﻤﺎ ﺍﻟﺜﺎﻨﻲ ﻓﻬﻭ ﻤﺸﺘﻕ ﻫﺫﻩ ﺍﻟﻁﺎﻗﺔ ﺒﺎﻟﻨﺴﺒﺔ ﻟﻺﺤﺩﺍﺜﻴﺎﺕ‬
‫‪‬‬

‫ﺍﻟﻤﻌﻤﻤﺔ ‪ q j‬ﺃﻱ ﺃﻥ‪:‬‬


‫‪N‬‬ ‫‪‬‬ ‫‪‬‬
‫‪‬‬ ‫‪n‬‬ ‫‪‬‬
‫‪ d T T ‬‬
‫‪ m i Wi  ri    dt   q q j‬‬ ‫)‪(2.17‬‬
‫‪i 1‬‬ ‫‪j1‬‬ ‫‪q j‬‬ ‫‪j‬‬
‫‪‬‬ ‫‪‬‬
‫ﻭﻋﻨﺩﺌﺫ‪ ‬ﻨﺠﺩ ﻤﻥ )‪ (2.11‬ﻭ )‪ (2.17‬ﻭ )‪ (2.2‬ﺃﻥ‪:‬‬
‫‪‬‬ ‫‪‬‬
‫‪ ‬‬ ‫‪ d T T ‬‬
‫‪  F  m‬‬ ‫‪Wi  ri   Q j q j   ‬‬ ‫‪‬‬ ‫)‪q j  0 (2.18‬‬
‫‪i‬‬
‫‪i‬‬ ‫‪i‬‬
‫‪‬‬ ‫‪j‬‬
‫‪‬‬ ‫‪‬‬
‫‪j  dt q j‬‬ ‫‪q j ‬‬

‫ﻭﻤﻨﻪ‪:‬‬
‫‪n‬‬ ‫‪‬‬ ‫‪‬‬
‫‪ d T‬‬ ‫‪T‬‬ ‫‪‬‬
‫‪  dt   q  Q j q j  0‬‬ ‫)‪(2.19‬‬
‫‪j 1‬‬ ‫‪q j‬‬ ‫‪j‬‬ ‫‪‬‬
‫‪‬‬ ‫‪‬‬
‫ﻭﻟﻜﻲ ﺘﺘﺤﻘﻕ ﻫﺫﻩ ﺍﻟﻤﻌﺎﺩﻟﺔ ﻤﻬﻤﺎ ﻜﺎﻨﺕ ‪ q j‬ﻴﻨﺒﻐﻲ ﺃﻥ ﺘﻨﻌﺩﻡ ﺃﻤﺜﺎل ‪q j‬‬

‫ﻷﻥ ﺍﻻﻨﺘﻘﺎﻻﺕ ‪ q j‬ﻤﺴﺘﻘﻠﺔ ﺒﺴﺒﺏ ﻁﺭﻴﻘﺔ ﺍﺨﺘﻴﺎﺭ ﻫﺫﻩ ﺍﻹﺤﺩﺍﺜﻴﺎﺕ ﺍﻟﻤﻌﻤﻤﺔ ‪. q j‬‬
‫ﻭﻋﻨﺩﺌﺫ‪ ‬ﻨﺠﺩ ﻤﻥ )‪ (2.19‬ﻤﻌﺎﺩﻻﺕ ﻻﻏﺭﺍﻨﺞ ﻤﻥ ﺍﻟﻨﻭﻉ ﺍﻟﺜﺎﻨﻲ ﺍﻟﺘﺎﻟﻴﺔ‪:‬‬
‫‪d T T‬‬
‫‪‬‬ ‫‪ Qj‬‬ ‫)‪(2.20‬‬
‫‪dt q  q j‬‬
‫‪j‬‬

‫ﻭﻫﻲ ﻋﺒﺎﺭﺓ ﻋﻥ ‪ n‬ﻤﻌﺎﺩﻟﺔ ﺘﻔﺎﻀﻠﻴﺔ ﻤﻥ ﺍﻟﻤﺭﺘﺒﺔ ﺍﻟﺜﺎﻨﻴﺔ ﻜﺎﻓﻴﺔ ﻟﺤﺴﺎﺏ ‪n‬‬


‫ﻤﺠﻬﻭﻻﹰ ﻤﻥ ‪ q j‬ﺒﺩﻻﻟﺔ ﺍﻟﺯﻤﻥ ﻭﺒﺎﻟﺘﺎﻟﻲ ﺍﻴﺠﺎﺩ ﻗﻭﺍﻨﻴﻥ ﺍﻟﺤﺭﻜﺔ‪.‬‬

‫‪- 52 -‬‬

‫)‪Create PDF files without this message by purchasing novaPDF printer (http://www.novapdf.com‬‬
‫‪ - 13‬ﺗﺎﺑﻊ ﻻﻏﺮﺍﻧﺞ‪:‬‬
‫ﻴﻤﻜﻥ ﺃﻥ ﺘﺄﺨﺫ ﻤﻌﺎﺩﻻﺕ ﻻﻏﺭﺍﻨﺞ )‪ (2.20‬ﺸﻜﻼﹰ ﺁﺨﺭ ﻗﺩ ﻴﻜﻭﻥ ﺃﺴﻬل‬
‫ﺍﺴﺘﻌﻤﺎﻻﹰ ‪ ،‬ﺴﻨﺒﺤﺙ ﻋﻨﻪ ﺍﻵﻥ‪.‬‬
‫‪‬‬
‫ﻨﻌﺒﺭ ﻋﻥ ﺍﻟﻘﻭﺓ ﺍﻟﻤﺅﺜﺭﺓ ‪ Fi‬ﻓﻲ ﺍﻟﻨﻘﻁﺔ ‪ M i‬ﺒﺩﻻﻟﺔ ﺘﺎﺒﻊ ﺍﻟﻜﻤﻭﻥ ‪) V‬ﻭﺫﻟﻙ‬
‫ﺒﻔﺭﺽ ﺃﻥ ﺍﻟﻘﻭﻯ ﺍﻟﻤﺅﺜﺭﺓ ﻓﻲ ﺍﻟﻤﺠﻤﻭﻋﺔ ﺍﻟﻤﺎﺩﻴﺔ ﻫﻲ ﻗﻭﻯ ﻜﻤﻭﻨﻴﺔ( ﻁﺒﻘﺎﹰ ﻟﻠﻌﻼﻗﺔ‬
‫‪‬‬
‫ﺍﻟﻤﻌﺭﻭﻓﺔ ‪ . Fi  grad i V‬ﻭﻋﻨﺩﺌﺫ‪ ‬ﻨﺤﺴﺏ ﻋﺒﺎﺭﺓ ﺍﻟﻘﻭﻯ ﺍﻟﻤﻌﻤﻤﺔ ‪ Q j‬ﺍﻋﺘﻤﺎﺩﺍﹰ‬
‫ﻋﻠﻰ ﺘﻌﺭﻴﻔﻬﺎ )‪ (2.12‬ﻓﻨﺠﺩ‪:‬‬
‫‪‬‬ ‫‪‬‬
‫‪ r‬‬ ‫‪‬‬ ‫‪r‬‬
‫‪Q j  Fi . i   grad i V. i ‬‬
‫‪‬‬ ‫‪‬‬
‫‪i‬‬
‫‪q i‬‬ ‫‪i‬‬
‫‪q j‬‬
‫‪V x i V y i V z i‬‬ ‫‪V‬‬
‫‪ ‬‬ ‫‪‬‬ ‫‪‬‬ ‫‪‬‬ ‫)‪(2.21‬‬
‫‪x i q i y i q j z i q j‬‬ ‫‪q j‬‬

‫ﻭﺒﺎﻟﺘﺒﺩﻴل ﻓﻲ )‪ (2.20‬ﻨﺠﺩ ﺍﻟﻤﻌﺎﺩﻟﺔ‪:‬‬


‫‪d T T‬‬ ‫‪V‬‬
‫‪‬‬ ‫‪‬‬
‫‪dt q  q j‬‬ ‫‪q j‬‬
‫‪j‬‬

‫ﻭﻤﻨﻪ‪:‬‬
‫)‪d T  (T  V‬‬
‫‪‬‬ ‫‪0‬‬ ‫)‪(2.22‬‬
‫‪dt q ‬‬ ‫‪q j‬‬
‫‪j‬‬

‫ﻭﺇﺫﺍ ﻻﺤﻅﻨﺎ ﺃﻥ ﺘﺎﺒﻊ ﺍﻟﻜﻤﻭﻥ ﻴﺘﻌﻠﻕ ﺒﻤﻭﻀﻊ ﺍﻟﻨﻘﻁﺔ ﺍﻟﻤﺎﺩﻴﺔ ﺃﻱ ﺒﺈﺤﺩﺍﺜﻴﺎﺘﻬﺎ‬


‫ﻭﺒﺎﻟﺘﺎﻟﻲ ﻓﺈﻥ ﻁﺭﺡ ﺍﻟﻤﻘﺩﺍﺭ‬ ‫‪ q j‬ﻭﻻﻴﺘﻌﻠﻕ ﺒﺎﻟﺴﺭﻉ ﺍﻟﻤﻌﻤﻤﺔ ‪ q j‬ﻓﺈﻥ ‪ 0‬‬
‫‪V‬‬ ‫‪‬‬
‫‪‬‬
‫‪q j‬‬

‫ﻤﻥ ﺍﻟﺤﺩ ﺍﻷﻭل ﻓﻲ ﺍﻟﻤﻌﺎﺩﻟﺔ ﺍﻟﺴﺎﺒﻘﺔ ﻻﻴﻐﻴﺭ ﻤﻥ ﺍﻷﻤﺭ ﺸﻴﺌﺎﹰ ﻭﻫﻜﺫﺍ ﻨﻀﻊ‬
‫‪V‬‬
‫‪‬‬
‫‪q j‬‬

‫ﺍﻟﻤﻌﺎﺩﻟﺔ )‪ (2.22‬ﺒﺎﻟﺸﻜل‪:‬‬
‫) ‪d (T  V ) (T  V‬‬
‫‪‬‬ ‫‪0‬‬ ‫)‪(2.23‬‬
‫‪dt q ‬‬ ‫‪q j‬‬
‫‪j‬‬

‫‪- 53 -‬‬

‫)‪Create PDF files without this message by purchasing novaPDF printer (http://www.novapdf.com‬‬
‫ﻟﻨﺭﻤﺯ ﺒـ ‪ L‬ﻟﻠﻤﻘﺩﺍﺭ ) ‪ (T-V‬ﺍﻟﺫﻱ ﻴﺴﻤﻰ ﺘﺎﺒﻊ ﻻﻏﺭﺍﻨﺞ ﻭﻫﻭ ﻴﺴﺎﻭﻱ‬
‫ﺘﻔﺎﻀل ﺍﻟﻁﺎﻗﺘﻴﻥ ﺍﻟﺤﺭﻜﻴﺔ ﻭﺍﻟﻜﺎﻤﻨﺔ ﺃﻱ ﺃﻥ‪:‬‬
‫‪LTV‬‬ ‫)‪(2.24‬‬
‫ﺜﻡ ﻨﺒﺩل ‪ T  V‬ﺒﻘﻴﻤﺘﻬﺎ ﻫﺫﻩ ﻓﻲ ﺍﻟﻤﻌﺎﺩﻟﺔ )‪ (2.23‬ﻓﻨﺤﺼل ﻋﻠﻰ ﺸﻜل ﺁﺨﺭ‬
‫ﻟﻤﻌﺎﺩﻻﺕ ﻻﻏﺭﺍﻨﺞ ﻫﻭ ﺍﻟﺘﺎﻟﻲ‪:‬‬
‫‪d L L‬‬
‫‪‬‬ ‫‪0‬‬ ‫)‪(2.25‬‬
‫‪dt q  q j‬‬
‫‪j‬‬

‫ﻭﻫﻲ ﻋﺒﺎﺭﺓ ﻋﻥ ‪ n‬ﻤﻌﺎﺩﻟﺔ ﺘﻔﺎﻀﻠﻴﺔ ﻤﻥ ﺍﻟﻤﺭﺘﺒﺔ ﺍﻟﺜﺎﻨﻴﺔ ﻜﺎﻓﻴﺔ ﻟﺤﺴﺎﺏ ‪n‬‬


‫ﺍﺤﺩﺍﺜﻲ ﻤﻌﻤﻡ ‪ q j‬ﺒﺩﻻﻟﺔ ﺍﻟﺯﻤﻥ ﻭﺒﺎﻟﺘﺎﻟﻲ ﺍﻴﺠﺎﺩ ﻗﻭﺍﻨﻴﻥ ﺍﻟﺤﺭﻜﺔ‪.‬‬

‫ﺘﺄﺨﺫ ﺍﻟﻤﻌﺎﺩﻻﺕ )‪ (2.25‬ﺸﻜﻼﹰ ﺁﺨﺭ ﻋﻨﺩﻤﺎ ﺘﺘﺄﺜﺭ ﺍﻟﻤﺠﻤﻭﻋﺔ ﺒﻘﻭﻯ ﻏﻴﺭ‬


‫ﻜﻤﻭﻨﻴﺔ ﺤﻴﺙ ﻨﺠﺩ ﺘﻌﻤﻴﻤﺎﹰ ﻟﻬﺎ ﻫﻭ‪:‬‬
‫‪d L L‬‬
‫)'‪(2.25‬‬
‫'‬
‫‪‬‬ ‫‪ Qj‬‬
‫‪dt q‬‬ ‫‪‬‬ ‫‪q j‬‬
‫‪j‬‬

‫ﺤﻴﺙ ‪ Q j‬ﻫﻲ ﺍﻟﻘﻭﻯ ﺍﻟﻤﻌﻤﻤﺔ ﻏﻴﺭ ﺍﻟﻤﺸﺘﻘﺔ ﻤﻥ ﻜﻤﻭﻥ‪ ،‬ﻭﻫﻲ ﺍﻟﺘﻲ ﺘﻨﺘﺞ ﻋﻥ‬
‫'‬

‫ﺭﺩﻭﺩ ﺃﻓﻌﺎل ﺃﻭ ﺘﻭﺘﺭﺍﺕ ﺨﻴﻭﻁ ﺃﻭ ﻗﻭﻯ ﺍﺤﺘﻜﺎﻙ ﺃﻭ ﻗﻭﻯ ﻤﻘﺎﻭﻤﺔ ﺍﻟﻭﺴﻁ ﻭﻏﻴﺭﻫﺎ‪.‬‬

‫‪ - 14‬ﻃﺮﻳﻘﺔ ﺍﺳﺘﺨﺪﺍﻡ ﻣﻌﺎﺩﻻﺕ ﻻﻏﺮﺍﻧﺞ ﰲ ﺣﻞ ﺍﳌﺴﺎﺋﻞ‪:‬‬

‫ﺁ‪ -‬ﺍﳌﻌﺎﺩﻻﺕ‪:( 2.20) :‬‬


‫‪ -‬ﻨﺨﺘﺎﺭ ﺍﻻﺤﺩﺍﺜﻴﺎﺕ ﺍﻟﻤﻌﻤﻤﺔ ﺒﺸﻜل ﻤﻨﺎﺴﺏ‪ ،‬ﺒﺤﻴﺙ ﻴﻜﻭﻥ ﻋﺩﺩﻫﺎ ﻴﺴﺎﻭﻱ‬
‫ﻋﺩﺩ ﺩﺭﺠﺎﺕ ﺍﻟﺤﺭﻴﺔ ﻟﻠﺠﻤﻠﺔ ﺃﻱ ‪. 3N  k‬‬
‫‪ -‬ﻨﺤﺴﺏ ﺍﻟﻁﺎﻗﺔ ﺍﻟﺤﺭﻜﻴﺔ ﺒﺩﻻﻟﺔ ﺍﻻﺤﺩﺍﺜﻴﺎﺕ ﺍﻟﻤﻌﻤﻤﺔ ﻭﺍﻟﺴﺭﻉ ﺍﻟﻤﻌﻤﻤﺔ‪.‬‬
‫‪ -‬ﻨﺤﺴﺏ ﺍﻟﻘﻭﻯ ﺍﻟﻤﻌﻤﻤﺔ ﻭﺫﻟﻙ ﺒﺤﺴﺎﺏ ﺍﻟﻌﻤل ﺍﻻﻓﺘﺭﺍﻀﻲ ﻓﺘﻜﻭﻥ ﺃﻤﺜﺎل‬
‫ﺍﻻﻨﺘﻘﺎﻻﺕ ﺍﻻﻓﺘﺭﺍﻀﻴﺔ ﻟﻼﺤﺩﺍﺜﻴﺎﺕ ﺍﻟﻤﻌﻤﻤﺔ )ﺃﻱ ﺃﻤﺜﺎل ‪ ( q j‬ﻫﻲ ‪. Q j‬‬
‫‪ -‬ﻨﺤﺴﺏ ﻤﺸﺘﻘﺎﺕ ﺍﻟﻁﺎﻗﺔ ﺍﻟﺤﺭﻜﻴﺔ ﺒﺎﻟﻨﺴﺒﺔ ﺇﻟﻰ ‪ q j‬ﻭ ‪ q j‬ﺜﻡ ﻨﺸﻜل‬
‫‪‬‬

‫ﺍﻟﻤﻌﺎﺩﻻﺕ )‪.(2.20‬‬

‫‪- 54 -‬‬

‫)‪Create PDF files without this message by purchasing novaPDF printer (http://www.novapdf.com‬‬
‫ﺏ‪ -‬ﺍﳌﻌﺎﺩﻻﺕ‪:(2.25) :‬‬
‫‪ -‬ﻨﺨﺘﺎﺭ ﺍﻹﺤﺩﺍﺜﻴﺎﺕ ﺍﻟﻤﻌﻤﻤﺔ ﻜﻤﺎ ﻓﻲ ﺍﻟﺤﺎﻟﺔ ﺍﻷﻭﻟﻰ‪.‬‬
‫‪ -‬ﻨﺤﺴﺏ ﺘﺎﺒﻊ ﺍﻟﻜﻤﻭﻥ ﺍﻋﺘﻤﺎﺩ ﺍﹰ ﻋﻠﻰ ﺘﻌﺭﻴﻔﻪ ﺒﺩﻻﻟﺔ ﺍﻹﺤﺩﺍﺜﻴﺎﺕ ﺍﻟﻤﻌﻤﻤﺔ ﺜﻡ‬
‫ﻨﺤﺴﺏ ﺍﻟﻁﺎﻗﺔ ﺍﻟﺤﺭﻜﻴﺔ ﺒﺩﻻﻟﺔ ‪ q j‬ﻭ ‪. q j‬‬
‫‪‬‬

‫‪ -‬ﻨﺤﺴﺏ ﺘﺎﺒﻊ ﻻﻏﺭﺍﻨﺞ ‪ L  T  V‬ﺜﻡ ﻨﺤﺴﺏ ﻤﺸﺘﻘﺎﺘﻪ ﺒﺎﻟﻨﺴﺒﺔ ﻟـ ‪ q j‬ﻭ‬


‫‪.q j‬‬
‫‪‬‬

‫‪ -‬ﻨﺸﻜل ﻤﻌﺎﺩﻻﺕ ﻻﻏﺭﺍﻨﺞ )‪.(2.25‬‬


‫ﻣﺜﺎﻝ ‪:‬‬
‫ﻴﻁﻠﺏ ﺍﻟﺤﺼﻭل ﻋﻠﻰ ﻤﻌﺎﺩﻻﺕ ﻨﻴﻭﺘﻥ ﻟﻨﻘﻁﺔ ﻤﺎﺩﻴﺔ ﻁﻠﻴﻘﺔ ﺍﻨﻁﻼﻗﺎﹰ ﻤﻥ ﺘﺎﺒﻊ‬
‫ﻻﻏﺭﺍﻨﺞ‪.‬‬
‫ﺍﳊﻞ ‪:‬‬
‫ﻨﺨﺘﺎﺭ ﺍﻹﺤﺩﺍﺜﻴﺎﺕ ﺍﻟﻤﻌﻤﻤﺔ ‪) q 3  z , q 2  y , q1  x‬ﻤﻊ ﺍﻟﻌﻠﻡ ﺃﻥ ﻋﺩﺩﻫﺎ‬
‫ﻴﺠﺏ ﺃﻥ ﻴﻜﻭﻥ ﺜﻼﺜﺔ ﻷﻥ ﺍﻟﻨﻘﻁﺔ ﻁﻠﻴﻘﺔ( ﻭﻴﻜﻭﻥ‪m(x  y  z ) :‬‬
‫‪1‬‬ ‫‪2‬‬ ‫‪2‬‬ ‫‪2‬‬
‫‪T‬‬
‫‪2‬‬
‫‪‬‬ ‫‪‬‬
‫ﺜﻡ ﻨﺤﺴﺏ ﺍﻟﻌﻤل ﺍﻻﻓﺘﺭﺍﻀﻲ ‪ A  F . r  F x.x  Fy y  Fz z‬ﺃﻱ ﺃﻥ‪:‬‬
‫‪ . Q x  Fx , Q y  Fy , Qz  Fz‬ﻟﻨﺤﺴﺏ ﺍﻟﻤﺸﺘﻘﺎﺕ‪:‬‬
‫‪T‬‬ ‫‪‬‬ ‫‪T‬‬ ‫‪‬‬ ‫‪T‬‬ ‫‪‬‬ ‫‪T T T‬‬
‫‪ mx ,‬‬ ‫‪ my ,‬‬ ‫‪ mz ,‬‬ ‫‪‬‬ ‫‪‬‬ ‫‪0‬‬
‫‪‬‬ ‫‪‬‬ ‫‪‬‬ ‫‪x y z‬‬
‫‪x‬‬ ‫‪y‬‬ ‫‪z‬‬
‫ﻨﺸﻜل ﺍﻵﻥ ﻤﻌﺎﺩﻻﺕ ﻻﻏﺭﺍﻨﺞ ﻤﻥ ﺍﻟﻨﻭﻉ )‪ (2.20‬ﻓﻨﺠﺩ‪:‬‬
‫‪‬‬ ‫‪‬‬
‫)‪mx   Fx , my   Fy , mz    Fz  m W  F (2.26‬‬

‫ﻭﻫﻲ ﻤﻌﺎﺩﻟﺔ ﻨﻴﻭﺘﻥ ﻟﻨﻘﻁﺔ ﻤﺎﺩﻴﺔ ﻁﻠﻴﻘﺔ‪.‬‬


‫ﺃﻤﺎ ﺇﺫﺍ ﺍﺴﺘﺨﺩﻤﻨﺎ ﺍﻟﻤﻌﺎﺩﻟﺔ )‪ (2.25‬ﻓﺈﻨﻨﺎ ﻨﺠﺩ‪:‬‬
‫‪1  2‬‬ ‫‪2‬‬ ‫‪2‬‬
‫‪L  TV ‬‬ ‫)‪m x  y  z   V (x , y, z‬‬
‫‪2 ‬‬ ‫‪‬‬
‫‪L‬‬ ‫‪ L‬‬ ‫‪ L‬‬ ‫‪‬‬
‫‪ mx ,‬‬ ‫‪ my ,‬‬ ‫‪ mz‬‬
‫‪‬‬ ‫‪‬‬ ‫‪‬‬
‫‪x‬‬ ‫‪y‬‬ ‫‪z‬‬

‫‪- 55 -‬‬

‫)‪Create PDF files without this message by purchasing novaPDF printer (http://www.novapdf.com‬‬
L V L V L V
  Fx ,   Fy ,   Fz
x x y y x z
(2.26) ‫( ﻓﻨﺠﺩ ﺍﻟﻤﻌﺎﺩﻻﺕ‬2.25) ‫ﻨﺸﻜل ﻤﻌﺎﺩﻻﺕ ﻻﻏﺭﺍﻨﺞ ﻤﻥ ﺍﻟﻨﻭﻉ‬
.‫ﻨﻔﺴﻬﺎ‬
:‫ ﰲ ﺍﳊﺎﻟﺔ ﺍﻟﻌﺎﻣﺔ‬q j ‫ ﻭ‬q j ‫ ﺍﻟﺘﻌﺒﲑ ﻋﻦ ﺍﻟﻄﺎﻗﺔ ﺍﳊﺮﻛﻴﺔ ﺑﺪﻻﻟﺔ‬- 15

:‫ﻤﻥ ﺍﻟﻤﻌﻠﻭﻡ ﺃﻥ ﺍﻟﻁﺎﻗﺔ ﺍﻟﺤﺭﻜﻴﺔ ﻟﺠﻤﻠﺔ ﻤﺎﺩﻴﺔ ﺘﻌﻁﻰ ﺒﺎﻟﻌﻼﻗﺔ‬


1 N
T  m i v12
2 i 1

:‫( ﻓﻨﺠﺩ‬2.8) ‫ ﺒﻘﻴﻤﺘﻬﺎ ﻤﻥ‬v i ‫ﻟﻨﺤﺴﺏ ﻫﺫﻩ ﺍﻟﻁﺎﻗﺔ ﺒﻌﺩ ﺘﺒﺩﻴل‬
2
  
1   ri   ri 
T   m i  qj   
2 i  q j  t
 j 
 
2
    
  
1  r  r  r  r r
 i 
  m i  i . i q j q k  2 i . i q j  
2 i q j q k j q j t  t  
 j k   
 T2  T1  T0 (2.27)
:‫ﺤﻴﺙ‬
  
1 n   
N  ri  ri 
T2   a jk q j q k ,  a jk  a kj   mi .  (2.28)
2 j, k  i 1
q j q k 
 
 
n   N 
  ri  ri 
T1   b j q j ,  b j   m i . (2.29)
j ,k  i 1 q j t 
 
2
N
 
1 r 
T0   m i  i  (2.30)
2 i  t 
 

- 56 -

Create PDF files without this message by purchasing novaPDF printer (http://www.novapdf.com)
‫ﻭﻴﺒﺩﻭ ﺒﻭﻀﻭﺡ ﺃﻥ ‪ T2‬ﻴﺤﻭﻱ ﺍﻟﺴﺭﻉ ﺍﻟﻤﻌﻤﻤﺔ ﻤﻥ ﺍﻟﻤﺭﺘﺒﺔ ﺍﻟﺜﺎﻨﻴﺔ ﻭ ‪T1‬‬
‫ﻴﺤﻭﻱ ﻫﺫﻩ ﺍﻟﺴﺭﻉ ﻤﻥ ﺍﻟﻤﺭﺘﺒﺔ ﺍﻷﻭﻟﻰ ﻭ ‪ T0‬ﻴﺤﻭﻱ ﺍﻟﺴﺭﻉ ﺍﻟﻤﻌﻤﻤﺔ ﻤﻥ ﺍﻟﻤﺭﺘﺒﺔ‬
‫ﺼﻔﺭ ﺃﻱ ﺃﻨﻪ ﻻﻴﺤﻭﻱ ﺍﻟﺴﺭﻉ ﺍﻟﻤﻌﻤﺔ ﺍﻁﻼﻗﺎﹰ‪.‬‬
‫ﻭﻓﻲ ﺍﻟﺤﺎﻟﺔ ﺍﻟﺨﺎﺼﺔ ﻋﻨﺩﻤﺎ ﻻﺘﺤﻭﻱ ﺍﻟﻤﻌﺎﺩﻟﺔ )‪ (2.8‬ﺍﻟﺯﻤﻥ ﺒﺸﻜل ﺼﺭﻴﺢ‬
‫‪‬‬
‫‪ ri‬‬
‫ﻭﺒﺎﻟﺘﺎﻟﻲ ‪ T  T2‬ﻭﻋﻨﺩﺌﺫ‪ ‬ﻴﻌﺒﺭ ﻋﻥ ﺍﻟﻁﺎﻗﺔ ﺍﻟﺤﺭﻜﻴﺔ ﺍﻟﻜﻠﻴﺔ ﻟﻠﺠﻤﻠﺔ‬ ‫ﻴﻜﻭﻥ ‪ 0‬‬
‫‪t‬‬
‫ﺍﻟﻤﺎﺩﻴﺔ ﺒﺘﺎﺒﻊ ﻤﺘﺠﺎﻨﺱ ﻤﻥ ﺍﻟﺩﺭﺠﺔ ﺍﻟﺜﺎﻨﻴﺔ ﺒﺎﻟﻨﺴﺒﺔ ﻟﻠﺴﺭﻉ ﺍﻟﻤﻌﻤﻤﺔ ‪. q j‬‬
‫‪‬‬

‫ﻭﻁﺒﻘﺎﹰ ﻟﻨﻅﺭﻴﺔ ﺃﻭﻟﺭ ﻓﻲ ﺍﻟﺘﻭﺍﺒﻊ ﺍﻟﻤﺘﺠﺎﻨﺴﺔ ﻴﻜﻭﻥ‪:‬‬


‫‪T ‬‬ ‫‪T ‬‬
‫‪‬‬ ‫‪‬‬
‫‪q j   2 q j  2T  2T2‬‬
‫‪‬‬
‫)‪(2.31‬‬
‫‪j q j‬‬ ‫‪j q j‬‬

‫ﻭﻴﻤﻜﻥ ﺍﻟﺒﺭﻫﺎﻥ ﺃﻥ ﺍﻟﻤﻌﺎﺩﻟﺔ )‪ (2.31‬ﺘﻜﻭﻥ ﺼﺤﻴﺤﺔ ﺩﻭﻤﺎﹰ ﻋﻨﺩﻤﺎ ﻻﺘﺘﻌﻠﻕ‬


‫ﺍﻻﺭﺘﺒﺎﻁﺎﺕ ﺍﻟﻤﻁﺒﻘﺔ ﻋﻠﻰ ﺍﻟﺠﻤﻠﺔ ﺍﻟﻤﺎﺩﻴﺔ ﺒﺎﻟﺯﻤﻥ‪ ،‬ﺃﻱ ﻋﻨﺩﻤﺎ ﺘﻜﻭﻥ ﻫﺫﻩ‬
‫ﺍﻻﺭﺘﺒﺎﻁﺎﺕ ﻤﺴﺘﻘﺭﺓ‪.‬‬
‫‪ - 16‬ﻣﻼﺣﻈﺎﺕ ﻋﻠﻰ ﺗﺎﺑﻊ ﻻﻏﺮﺍﻧﺞ‪:‬‬
‫‪ -1‬ﻋﻨﺩﻤﺎ ﺘﻨﻌﺩﻡ ﺍﻟﻘﻭﻯ ﺍﻟﻤﻌﻤﻤﺔ ‪ Q k‬ﻏﻴﺭ ﺍﻟﻤﺸﺘﻘﺔ ﻤﻥ ﻜﻤﻭﻥ ﻭﻻﻴﺤﻭﻱ‬
‫'‬

‫ﻭﻋﻨﺩﺌﺫ‪‬‬ ‫ﺘﺎﺒﻊ ﻻﻏﺭﺍﻨﺞ ﺃﺤﺩ ﺍﻹﺤﺩﺍﺜﻴﺎﺕ ﺍﻟﻤﻌﻤﻤﺔ ﻭﻟﻴﻜﻥ ‪ q k‬ﻤﺜﻼﹰ ﻓﺈﻥ‪ 0 :‬‬
‫‪L‬‬
‫‪q k‬‬
‫ﻨﺤﺼل ﻤﻥ ﻤﻌﺎﺩﻻﺕ ﻻﻏﺭﺍﻨﺞ )‪ (2.26‬ﻋﻠﻰ ﺍﻟﻌﻼﻗﺔ‪:‬‬
‫‪d L‬‬ ‫‪L‬‬
‫‪0‬‬ ‫‪ Const.‬‬ ‫)‪(2.32‬‬
‫‪dt q ‬‬ ‫‪‬‬
‫‪q k‬‬
‫‪k‬‬
‫ﻭﻫﻭ ﺘﻜﺎﻤل ﻟﻠﺤﺭﻜﺔ‪.‬‬
‫ﻭﺇﺫﺍ ﻟﻡ ﻴﺤﻭ ﺘﺎﺒﻊ ﻻﻏﺭﺍﻨﺞ ﻋﺩﺩﹰﺍ ‪ S‬ﻤﻥ ﺍﻹﺤﺩﺍﺜﻴﺎﺕ ﺍﻟﻤﻌﻤﻤﺔ ﻓﺈﻨﻨﺎ ﻨﺤﺼل‬
‫ﻋﻠﻰ ‪ S‬ﺘﻜﺎﻤﻼﹰ ﻤﻥ ﻫﺫﺍ ﺍﻟﻨﻭﻉ‪.‬‬
‫ﻨﺴﻤﻲ ﺍﻹﺤﺩﺍﺜﻴﺎﺕ ‪ q k‬ﻓﻲ ﻫﺫﻩ ﺍﻟﺤﺎﻟﺔ ﺇﺤﺩﺍﺜﻴﺎﺕ ﺩﻭﺭﻴﺔ ﻭﺴﻨﻌﻭﺩ ﺇﻟﻰ ﺫﻟﻙ‬
‫ﻓﻴﻤﺎ ﺒﻌﺩ ﻭﻨﺤﺼل ﻋﻠﻰ ﻤﻌﺎﺩﻻﺕ ﺠﺩﻴﺩﺓ ﻟﻠﺤﺭﻜﺔ‪.‬‬

‫‪- 57 -‬‬

‫)‪Create PDF files without this message by purchasing novaPDF printer (http://www.novapdf.com‬‬
‫‪ -2‬ﻟﻨﻀﺭﺏ ﻁﺭﻓﻲ ﺍﻟﻤﻌﺎﺩﻟﺔ )‪ (2.25‬ﺒـ ‪ q j‬ﺜﻡ ﻨﺠﻤﻊ ﺒـ ‪ j‬ﻋﻠﻰ ﺃﻥ‬
‫‪‬‬

‫ﻴﺸﻤل ﺍﻟﺠﻤﻊ ﻜﺎﻓﺔﹰ ﺍﻹﺤﺩﺍﺜﻴﺎﺕ ﺍﻟﻤﻌﻤﻤﺔ ﺃﻱ ﻤﻥ ‪ 1‬ﺤﺘﻰ ‪:n‬‬


‫‪n‬‬ ‫‪n‬‬
‫‪d L‬‬ ‫‪L ‬‬
‫)‪(2.33‬‬
‫‪‬‬
‫‪q j‬‬ ‫‪dt q‬‬ ‫‪‬‬
‫‪‬‬ ‫‪q‬‬
‫‪q j j‬‬
‫‪j 1‬‬ ‫‪j‬‬ ‫‪j‬‬ ‫‪‬‬ ‫‪1‬‬

‫ﻓﻨﺠﺩ‪:‬‬ ‫ﻨﻀﻴﻑ ﺇﻟﻰ ﺍﻟﻁﺭﻓﻴﻥ ﺍﻟﻤﻘﺩﺍﺭ ‪q‬‬


‫‪L  ‬‬
‫‪‬‬ ‫‪ j‬‬
‫‪q j‬‬

‫‪n‬‬ ‫‪‬‬ ‫‪ n ‬‬ ‫‪‬‬


‫‪q  d L  L q    ‬‬ ‫)‪ L q   L q    (2.34‬‬
‫‪  j dt q‬‬ ‫‪ j    q‬‬ ‫‪j‬‬ ‫‪ j ‬‬
‫‪j 1‬‬ ‫‪j q j‬‬ ‫‪‬‬ ‫‪j 1  j‬‬ ‫‪q j‬‬ ‫‪‬‬
‫‪‬‬ ‫‪‬‬ ‫‪‬‬
‫ﻭﺇﺫﺍ ﻋﻠﻤﻨﺎ ﺃﻥ ﺘﺎﺒﻊ ﻻﻏﺭﺍﻨﺞ ﻴﺘﺒﻊ‪ ،‬ﺒﺼﻭﺭﺓ ﻋﺎﻤﺔ‪ q j ،‬ﻭ ‪ q j‬ﻭ ‪ t‬ﺃﻱ‬
‫‪‬‬

‫) ‪ ، L  L(q j , q j , t‬ﺜﻡ ﺒﺎﺸﺘﻘﺎﻕ ﻫﺫﺍ ﺍﻟﺘﺎﺒﻊ ﺒﺎﻟﻨﺴﺒﺔ ﻟﻠﺯﻤﻥ ﻨﺠﺩ‪:‬‬


‫‪‬‬

‫‪dL‬‬ ‫‪L  L   L‬‬


‫‪‬‬ ‫‪q ‬‬ ‫‪q ‬‬ ‫)‪(2.35‬‬
‫‪dt‬‬ ‫‪q j j q  j‬‬ ‫‪t‬‬
‫‪j‬‬

‫ﻭﻤﻨﻪ ﻨﺤﺴﺏ ﺍﻟﺤﺩ ﺍﻟﺜﺎﻨﻲ ﻤﻥ )‪ ، (2.34‬ﻓﻨﺠﺩﻩ ﻤﺴﺎﻭﻴﺎﹰ ﺍﻟﻔﺭﻕ ﺒﻴﻥ ﺍﻟﻤﺸﺘﻘﻴﻥ‬


‫ﺍﻟﻜﻠﻲ ﻭﺍﻟﺠﺯﺌﻲ ﻟﺘﺎﺒﻊ ﻻﻏﺭﺍﻨﺞ‪.‬‬
‫‪‬‬ ‫‪‬‬
‫‪‬‬ ‫‪L  ‬‬
‫ﺃﻤﺎ ﺍﻟﻁﺭﻑ ﺍﻷﻴﺴﺭ ﻓﻬﻭ ﺍﻟﻤﺸﺘﻕ ﺍﻟﻜﻠﻲ ﻟﻠﻤﻘﺩﺍﺭ ‪    q j ‬ﻭﺒﺎﻟﺘﻌﻭﻴﺽ‬
‫‪ q j‬‬ ‫‪‬‬
‫‪‬‬ ‫‪‬‬
‫ﻓﻲ )‪ (2.34‬ﻨﺤﺼل‪ ،‬ﺒﻌﺩ ﺩﻤﺞ ﺍﻟﻤﺸﺘﻘﻴﻥ ﺍﻟﻜﻠﻴﻴﻥ ﺒﺎﻟﻨﺴﺒﺔ ﻟﻠﺯﻤﻥ‪ ،‬ﻋﻠﻰ ﺍﻟﻤﻌﺎﺩﻟﺔ‬
‫ﺍﻟﺘﺎﻟﻴﺔ‪:‬‬
‫‪‬‬ ‫‪‬‬
‫‪d  L ‬‬ ‫‪L‬‬
‫‪‬‬ ‫‪q j  L  ‬‬ ‫)‪(2.36‬‬
‫‪dt  q ‬‬ ‫‪‬‬ ‫‪t‬‬
‫‪‬‬ ‫‪j‬‬ ‫‪‬‬
‫ﻭﻴﻨﻌﺩﻡ ﺍﻟﻁﺭﻑ ﺍﻟﺜﺎﻨﻲ ﺇﺫﺍ ﻟﻡ ﻴﺤﻭ ﺘﺎﺒﻊ ﻻﻏﺭﺍﻨﺞ ﺍﻟﺯﻤﻥ ﺒﺸﻜل ﺼﺭﻴﺢ‬
‫ﻭﻋﻨﺩﺌﺫ‪ ‬ﻨﺤﺼل ﻋﻠﻰ ﺘﻜﺎﻤل ﺠﺩﻴﺩ ﻟﻠﺤﺭﻜﺔ ﻫﻭ ﺍﻟﺘﺎﻟﻲ‪:‬‬
‫‪L ‬‬
‫‪‬‬ ‫‪q  L  Const.‬‬
‫‪ j‬‬
‫)‪(2.37‬‬
‫‪j q j‬‬

‫‪- 58 -‬‬

‫)‪Create PDF files without this message by purchasing novaPDF printer (http://www.novapdf.com‬‬
‫ﻭﺴﻨﺒﺭﻫﻥ ﻓﻴﻤﺎﻴﻠﻲ ﺃﻥ ﻫﺫﺍ ﺍﻟﺘﻜﺎﻤل ﻫﻭ ﺘﻜﺎﻤل ﺍﻟﻁﺎﻗﺔ‪.‬‬
‫‪ - 3‬ﻟﻨﺤﺴﺏ ﺍﻟﺤﺩ ﺍﻷﻭل ﻓﻲ )‪ (2.37‬ﻓﻨﻼﺤﻅ ﺃﻭﻻﹰ ﺃﻥ ﺘﺎﺒﻊ ﺍﻟﻜﻤﻭﻥ ‪V‬‬
‫ﻻﻴﺤﻭﻱ ﺍﻟﺴﺭﻉ ﺍﻟﻤﻌﻤﻤﺔ ﻓﻬﻭ ﻴﺘﺒﻊ ﺍﻹﺤﺩﺍﺜﻴﺎﺕ ﺍﻟﻤﻌﻤﻤﺔ ‪ q j‬ﻻﻏﻴﺭ ﻭﺒﺎﻟﺘﺎﻟﻲ ﻴﻜﻭﻥ‬

‫ﻭﻋﻨﺩﺌﺫ‪ ‬ﻨﺠﺩ ﻁﺒﻘﺎﹰ ﻟـ )‪:(2.31‬‬


‫‪L‬‬ ‫) ‪(T  V‬‬ ‫‪T‬‬
‫‪‬‬ ‫‪‬‬
‫‪‬‬ ‫‪‬‬ ‫‪‬‬
‫‪q j‬‬ ‫‪q j‬‬ ‫‪q j‬‬
‫‪n‬‬ ‫‪n‬‬
‫‪L ‬‬ ‫‪T ‬‬
‫‪  j‬‬ ‫‪q‬‬ ‫‪‬‬ ‫‪L‬‬ ‫‪‬‬ ‫‪  q j  L  2T  T  V  E‬‬ ‫)‪(2.38‬‬
‫‪j 1 q j‬‬ ‫‪j 1 q j‬‬

‫ﻭﺒﺎﻟﺘﺩﺒﻴل ﻓﻲ )‪ (2.36‬ﻨﺤﺼل ﻋﻠﻰ ﻗﺎﻨﻭﻥ ﻤﺼﻭﻨﻴﺔ ﺍﻟﻁﺎﻗﺔ ﺍﻟﺘﺎﻟﻲ‪:‬‬


‫‪E  E0‬‬ ‫)‪(2.39‬‬
‫ﻁﺒﻘﺎﹰ‬ ‫ﻭﻨﺭﻴﺩ ﺍﻟﺘﺄﻜﻴﺩ ﻫﻨﺎ ﺃﻥ ﺘﺤﻘﻕ ﺍﻟﻘﺎﻨﻭﻥ )‪ (2.39‬ﻴﺘﻁﻠﺏ ﺃﻥ ﻴﻜﻭﻥ ‪ 0‬‬
‫‪L‬‬
‫‪t‬‬
‫ﻟـ )‪ ،(2.36‬ﻭﻟﻤﺎ ﻜﺎﻥ ‪ L  T  V‬ﻭﻜﺎﻨﺕ ﺍﻟﻁﺎﻗﺔ ﺍﻟﺤﺭﻜﻴﺔ ﺘﺎﺒﻌﺎﹰ ﻤﺘﺠﺎﻨﺴﺎﹰ ﻤﻥ‬
‫‪‬‬
‫‪ ri‬‬
‫ﻭﻫﺫﺍ ﺒﺩﻭﺭﻩ ﻴﺘﺤﻘﻕ ﻋﻨﺩﻤﺎ‬ ‫ﺍﻟﺩﺭﺠﺔ ﺍﻟﺜﺎﻨﻴﺔ ﺒﺎﻟﻨﺴﺒﺔ ﻟﻠﺴﺭﻉ ﺍﻟﻤﻌﻤﻤﺔ )ﻷﻥ ‪ 0‬‬
‫‪t‬‬
‫ﺘﻜﻭﻥ ﺍﻻﺭﺘﺒﺎﻁﺎﺕ ﻤﺴﺘﻘﺭﺓ ﻜﻤﺎ ﺫﻜﺭﻨﺎ ﺁﻨﻔﺎﹰ( ﻭﻻﻴﺤﻭﻱ ﺍﻟﺯﻤﻥ ﺒﺸﻜل ﺼﺭﻴﺢ ﻓﻲ ﻫﺫﻩ‬
‫ﺍﻟﺤﺎﻟﺔ ﻓﺈﻥ‪:‬‬
‫‪L‬‬ ‫‪ (T  V) V‬‬
‫‪‬‬ ‫‪‬‬ ‫‪‬‬ ‫)‪(2.40‬‬
‫‪t‬‬ ‫‪t‬‬ ‫‪t‬‬
‫ﻭﻤﻨﻪ ﻨﺴﺘﻨﺘﺞ ﺃﻥ ﺍﻟﻁﺎﻗﺔ ﺍﻟﻜﻠﻴﺔ ﺘﻜﻭﻥ ﺜﺎﺒﺘﺔ )ﺘﻜﺎﻤل ﻟﻠﺤﺭﻜﺔ( ﻋﻨﺩﻤﺎ ﻻﻴﺘﻌﻠﻕ‬
‫ﺍﻟﻜﻤﻭﻥ ﺒﺼﻭﺭﺓ ﺼﺭﻴﺤﺔ ﺒﺎﻟﺯﻤﻥ‪ ،‬ﻭﻫﻲ ﺍﻟﻨﺘﻴﺠﺔ ﺍﻟﻤﻌﺭﻭﻓﺔ ﻤﻥ ﻤﻴﻜﺎﻨﻴﻙ ﻨﻴﻭﺘﻥ‬
‫)ﺭﺍﺠﻊ ﻤﻭﺍﻀﻴﻊ ﺍﻟﻤﻴﻜﺎﻨﻴﻙ ﺍﻟﻔﻴﺯﻴﺎﺌﻲ(‪.‬‬

‫‪- 59 -‬‬

‫)‪Create PDF files without this message by purchasing novaPDF printer (http://www.novapdf.com‬‬
‫ﻣﺴﺎﺋﻞ ﺍﻟﻔﺼﻞ ﺍﻟﺜﺎﻧﻲ‬

‫‪ - 1‬ﻨﻭﺍﺱ ﻤﺴﺘﻭ ﻜﺘﻠﺘﻪ ‪ m 2‬ﻭﻨﻘﻁﺔ ﺘﻌﻠﻴﻘﻪ ﺫﺍﺕ ﺍﻟﻜﺘﻠﺔ ‪ m1‬ﺘﺤﺭﻙ ﺒﺤﺭﻜﺔ ﻤﺎ‬
‫ﻋﻠﻰ ﺍﻟﻤﺤﻭﺭ ﺍﻷﻓﻘﻲ ‪.ox‬‬
‫ﺁ‪ -‬ﺍﺤﺴﺏ ﺘﺎﺒﻊ ﻻﻏﺭﺍﻨﺞ ﻟﻠﺠﻤﻠﺔ‪.‬‬
‫ﺏ‪ -‬ﺍﻜﺘﺏ ﻤﻌﺎﺩﻻﺕ ﻻﻏﺭﺍﻨﺞ ﻟﻬﺫﻩ ﺍﻟﺠﻤﻠﺔ ﻭﺍﺤﺴﺏ ﺩﻭﺭ ﺍﻻﻫﺘﺯﺍﺯﺍﺕ‬
‫ﺍﻟﺼﻐﻴﺭﺓ ﻟﻬﺎ‪.‬‬

‫ﺍﻟﺸﻜﻞ )‪(2.1‬‬

‫ﺍﳊﻞ ‪:‬‬
‫ﻨﻼﺤﻅ ﺃﻭﻻﹰ ﺃﻥ ﻟﻠﺠﻤﻠﺔ ﻭﺴﻴﻁﻴﻥ ﻤﺴﺘﻘﻠﻴﻥ ﻨﺨﺘﺎﺭﻫﻤﺎ ﻜﻤﺎ ﻴﻠﻲ‪:‬‬
‫‪ :x‬ﻓﺎﺼﻠﺔ ﺍﻟﻨﻘﻁﺔ ‪ M1‬ﻋﻠﻰ ﺍﻟﻤﺤﻭﺭ ‪.ox‬‬
‫‪ :‬ﺍﻟﺯﺍﻭﻴﺔ ﺍﻟﺘ ﻲ ﻴﺼﻨﻌﻬﺎ ‪ M1M 2‬ﻤﻊ ﺍﻟﺸﺎﻗﻭل ﺸﻜل )‪.(2.1‬‬
‫ﻭﻟﺤﺴﺎﺏ ﺘﺎﺒﻊ ﻻﻏﺭﺍﻨﺞ ﻨﺤﺴﺏ ﻜﻼﹰ ﻤﻥ ﺍﻟﻁﺎﻗﺔ ﺍﻟﺤﺭﻜﻴﺔ ﻭﺘﺎﺒﻊ ﺍﻟﻜﻤﻭﻥ ﻭﻻﺒﺩ‬
‫ﺃﻭﻻﹰ ﻤﻥ ﺤﺴﺎﺏ ﺴﺭﻋﺔ ‪ M 2‬ﻭﻟﻬﺫﺍ ﻨﺤﺴﺏ ﺇﺤﺩﺍﺜﻴﺎﺘﻬﺎ ﻓﻨﺠﺩ‪:‬‬

‫‪- 60 -‬‬

‫)‪Create PDF files without this message by purchasing novaPDF printer (http://www.novapdf.com‬‬
  
OM 2  OM1  M1M 2 (1)
:‫ﻭﻤﻨﻪ‬
x 2  x   sin   x 2  x    cos  .  

 (2)

y 2   cos   y 2   sin .   
1
 
1

T2  m 2 x 22  y 22  m 2 x  2    2  2  2 x    cos 
2 2

( 3)
1 2
T1  m1x (4)
2
:‫ﻭﻜﺫﻟﻙ ﻨﺠﺩ‬
 
V    F . dr   mgy 2  m 2 g cos  (5)
:‫ﻭﻴﻜﻭﻥ ﺘﺎﺒﻊ ﻻﻏﺭﺍﻨﺞ‬
L  T1  T2  V 
1 1 (6)
 (m1  m 2 ) x  2  m 2 (l 2  2  2 x    cos )  m 2 g cos 
2 2
‫ ﻟﻠﺤﺼﻭل ﻋﻠﻰ ﻤﻌﺎﺩﻻﺕ ﻻﻏﺭﺍﻨﺞ ﻴﺠﺏ ﺍﻻﺸﺘﻘﺎﻕ ﺒﺎﻟﻨﺴﺒﺔ ﻟﻠﺴﺭﻉ‬-‫ﺏ‬
:‫ ﺤﻴﺙ ﻨﺠﺩ‬x ‫ ﻭ‬ ‫ ﻭﻟﻺﺤﺩﺍﺜﻴﺎﺕ ﺍﻟﻤﻌﻤﻤﺔ‬x ‫ ﻭ‬ ‫ﺍﻟﻤﻌﻤﻤﺔ‬
 
(7)

L L 
 (m1  m 2 )x   m 2    cos , 0 
x  x 


L L 
 m 2  2    m 2 x  cos ,  m 2 x    sin   m 2 g sin  
  
:‫ﻭﻤﻨﻪ ﻨﺤﺼل ﻋﻠﻰ ﺍﻟﻤﻌﺎﺩﻻﺕ ﺍﻟﻤﻁﻠﻭﺒﺔ ﺍﻟﺘﺎﻟﻴﺔ‬

- 61 -

Create PDF files without this message by purchasing novaPDF printer (http://www.novapdf.com)
(8)
d
dt
 
m1  m 2 x   m 2  cos   0  (m1  m 2 ) x   m 2  cos   0



d
dt
 
m 2 2    m 2 x  cos   m 2 g sin   m 2x    sin   0 

‫ﻤﻊ ﺍﻟﻌﻠﻡ ﺃﻨﻨﺎ ﺍﺨﺘﺭﻨﺎ ﺸﺭﻭﻁﺎﹰ ﺍﺒﺘﺩﺍﺌﻴﺔ ﻤﻨﺎﺴﺒﺔ ﺒﺤﻴﺙ ﻴﻨﻌﺩﻡ ﺜﺎﺒﺕ ﺍﻟﺘﻜﺎﻤل ﻓﻲ‬
.(8) ‫ﺍﻟﻤﻌﺎﺩﻟﺔ ﺍﻷﻭﻟﻰ ﻤﻥ‬
‫ ﻤﻥ ﺍﻷﻭﻟﻰ ﺤﻴﺙ‬x ‫ﻭﻟﻠﺤﺼﻭل ﻋﻠﻰ ﺩﻭﺭ ﺍﻻﻫﺘﺯﺍﺯﺍﺕ ﺍﻟﺼﻐﻴﺭﺓ ﻨﺤﺴﺏ‬

:‫ﻨﺠﺩ‬
m 2  

x  cos  (9)
m1  m 2
:‫ ﻋﻠﻰ ﺍﻟﻤﻌﺎﺩﻟﺔ‬m 2 2 ‫ﺜﻡ ﻨﻌﻭﻀﻬﺎ ﻓﻲ ﺍﻟﺜﺎﻨﻴﺔ ﻓﻨﺤﺼل ﺒﻌﺩ ﺍﻻﺨﺘﺼﺎﺭ ﻋﻠﻰ‬
d   m 2   cos 2   g m  2
   sin   2 sin  cos   0 ( 10)
dt  m1  m 2   m1  m 2

:‫ﻭﺒﺎﺠﺭﺍﺀ ﺍﻻﺸﺘﻘﺎﻕ ﻨﺠﺩ‬


 m 2 cos 2    
1    m 2   2 sin  cos   g sin   0 ( 11)
 m1  m 2  m1  m 2 

 2  0, cos   l, sin    ‫ﻭﻋﻨﺩﻤﺎ ﺘﻜﻭﻥ ﺍﻻﻫﺘﺯﺍﺯﺍﺕ ﺼﻐﻴﺭﺓ‬
:‫ ﻋﻠﻰ ﺍﻟﻤﻌﺎﺩﻟﺔ‬،(11) ‫ ﻜﺤﺎﻟﺔ ﺨﺎﺼﺔ ﻤﻥ‬،‫ﻨﺤﺼل ﺃﺨﻴﺭ ﺍﹰ‬
m1 g
      0      2   0 (12)
m1  m 2 
:‫ﻭﻫﻲ ﻤﻌﺎﺩﻟﺔ ﺤﺭﻜﺔ ﺘﻭﺍﻓﻘﻴﺔ ﺍﻫﺘﺯﺍﺯﻴﺔ ﺩﻭﺭﻫﺎ‬
2 m1
  2 (13)
 m1  m 2 g

- 62 -

Create PDF files without this message by purchasing novaPDF printer (http://www.novapdf.com)
‫‪ - 2‬ﺍﻭﺠﺩ ﺍﻟﻤﻌﺎﺩﻻﺕ ﺍﻟﺘﻔﺎﻀﻠﻴﺔ ﻟﺤﺭﻜﺔ ﻨﻘﻁﺔ ﻤﺎﺩﻴﺔ ﺘﺘﺤﺭﻙ ﺘﺤﺕ ﺘﺄﺜﻴﺭ ﻗﻭﻯ‬
‫ﻓﻌﺎﻟﺔ ﻤﺤﺼﻠﺘﻬﺎ ‪ F‬ﻤﻊ ﺍﻟﻌﻠﻡ ﺃﻥ ﺍﻟﻭﺴﻁ ﺍﻟﺫﻱ ﺘﻭﺠﺩ ﻓﻴﻪ ﻴﺩﻭﺭ ﺒﺴﺭﻋﺔ ﺯﺍﻭﻴﺔ ﺜﺎﺒﺘﺔ‬
‫‪ ‬ﺤﻭل ﻤﺤﻭﺭ ﺸﺎﻗﻭﻟﻲ ﺜﺎﺒﺕ ‪.oz‬‬
‫ﺍﳊﻞ ‪:‬‬
‫ﺴﻨﺤل ﻫﺫﻩ ﺍﻟﻤﺴﺄﻟﺔ ﺒﺎﺴﺘﺨﺩﺍﻡ ﻤﻌﺎﺩﻻﺕ ﻻﻏﺭﺍﻨﺞ )‪ (2.20‬ﻓﻨﺄﺨﺫ ﻤﺤﺎﻭﺭ‬
‫ﺇﺤﺩﺍﺜﻴﺔ ﺜﺎﺒﺘﺔ ‪ oxyz‬ﻭﻤﺤﺎﻭﺭ ﺃﺨﺭﻯ ‪ OXYZ‬ﻤﺘﻤﺎﺴﻜﺔ ﻤﻊ ﺍﻟﻭﺴﻁ ﺍﻟﻤﺘﺤﺭﻙ ﺒﺤﻴﺙ‬
‫ﻴﻨﻁﺒﻕ ﺍﻟﻤﺤﻭﺭ ‪ OZ‬ﻋﻠﻰ ﺍﻟﻤﺤﻭﺭ ﺍﻟﺜﺎﺒﺕ ‪ ،oz‬ﺃﻤﺎ ‪ x‬ﻭ ‪ y‬ﻓﺘﺤﺴﺏ ﺒﺩﻻﻟﺔ ‪ X‬ﻭ ‪Y‬‬
‫ﻁﺒﻘ ﹰﺎ ﻟﻠﻌﻼﻗﺎﺕ ﺍﻟﻤﻌﺭﻭﻓﺔ ﻭﻴﻜﻭﻥ‪:‬‬
‫)‪x  X cos   Y sin , y  X sin   Y cos , z  Z,    t (1‬‬

‫ﻭﻟﻜﻲ ﻨﺤﺴﺏ ﺍﻟﻁﺎﻗﺔ ﺍﻟﺤﺭﻜﻴﺔ ﻴﻨﺒﻐﻲ ﺤﺴﺎﺏ ﺍﻟﺴﺭﻋﺔ ﺃﻭﻻﹰ ﻭﻟﻬﺫﺍ ﻨﺸﺘﻕ‬
‫ﺍﻟﻌﻼﻗﺎﺕ ﺍﻟﺴﺎﺒﻘﺔ ﻓﻨﺠﺩ‪:‬‬
‫‪x   X  cos   Y  sin     X sin     Y cos ‬‬ ‫)‪(2‬‬
‫‪y  X  sin   Y  cos   X  cos   Y   sin ‬‬ ‫)‪(3‬‬
‫ﻭﺒﺎﻟﺘﺭﺒﻴﻊ ﻨﺠﺩ ﺃﺨﻴﺭﺍﹰ‪:‬‬
‫‪2‬‬ ‫‪2‬‬ ‫‪2‬‬ ‫‪2 ‬‬ ‫‪2‬‬ ‫‪2 2‬‬ ‫‪‬‬ ‫‪ ‬‬
‫‪x‬‬ ‫‪y‬‬ ‫‪  X‬‬ ‫‪Y‬‬ ‫‪   X  Y   2(XY  YX )‬‬
‫‪‬‬ ‫‪ ‬‬ ‫‪‬‬
‫ﻭﺒﺎﻟﺘﺎﻟﻲ ﻓﺎﻟﻁﺎﻗﺔ ﺍﻟﺤﺭﻜﻴﺔ ‪:T‬‬
‫‪1  2‬‬ ‫‪2‬‬ ‫‪2‬‬ ‫‪1‬‬ ‫‪2‬‬ ‫‪2‬‬ ‫‪2‬‬ ‫‪2‬‬ ‫‪2‬‬
‫‪T‬‬ ‫‪m x  y  z   m X  Y    X  Y  ‬‬
‫‪2 ‬‬ ‫‪ 2 ‬‬ ‫‪ ‬‬ ‫‪‬‬
‫‪‬‬ ‫‪‬‬ ‫‪2‬‬ ‫‪‬‬
‫)‪ 2(XY  YX )  Z  , (  ‬‬
‫‪‬‬
‫)‪(4‬‬

‫ﻭﻟﺤﺴﺎﺏ ﺍﻟﻘﻭﻯ ﺍﻟﻤﻌﻤﻤﺔ ﻨﺤﺴﺏ ﺍﻟﻌﻤل ﺍﻻﻓﺘﺭﺍﻀﻲ‪:‬‬


‫‪‬‬ ‫‪‬‬
‫‪W  F . r  FX X  FY Y  FZ Z‬‬ ‫)‪(5‬‬
‫ﺃﻱ ﺃﻥ‪:‬‬
‫‪Q X  FX , Q Y  FY , Q Z  FZ‬‬ ‫)‪(6‬‬
‫ﺃﻥ ﻤﺸﺘﻘﺎﺕ ‪ T‬ﺒﺎﻟﻨﺴﺒﺔ ﻟﻠﺴﺭﻉ ﺍﻟﻤﻌﻤﻤﺔ ﻭﺍﻻﺤﺩﺍﺜﻴﺎﺕ ﺍﻟﻤﻌﻤﻤﺔ ﻓﻬﻲ‪:‬‬
‫‪T‬‬ ‫‪‬‬ ‫‪T‬‬ ‫‪2‬‬ ‫‪‬‬
‫‪‬‬
‫‪ mX  mY,‬‬ ‫‪ m X  mY‬‬ ‫)‪(7‬‬
‫‪X‬‬ ‫‪X‬‬

‫‪- 63 -‬‬

‫)‪Create PDF files without this message by purchasing novaPDF printer (http://www.novapdf.com‬‬
T T

 mY   mX ,  m 2 Y  mX  (8)
Y Y
T  T

 mZ , 0 (9)
Z Z
‫( ﻨﺤﺼل ﻋﻠﻰ‬9) ‫( ﻭ‬8) ‫( ﻭ‬7) ‫( ﻭ‬6) ‫ﻭﺒﺎﻻﻋﺘﻤﺎﺩ ﻋﻠﻰ ﺍﻟﻤﻌﺎﺩﻻﺕ‬
:‫ﻤﻌﺎﺩﻻﺕ ﺍﻟﺤﺭﻜﺔ ﺍﻟﺘﺎﻟﻴﺔ‬
mX   m 2 X  2m Y'FX (10)
 2 
mY  m Y  2mX  FY (11)

mZ  FZ (12)
‫ﻫﺫﺍ ﻭﻴﻤﻜﻥ ﺍﻟﻭﺼﻭل ﺇﻟﻰ ﺍﻟﻨﺘﻴﺠﺔ ﻨﻔﺴﻬﺎ ﺒﻁﺭﻴﻘﺔ ﺜﺎﻨﻴﺔ ﺍﻨﻁﻼﻗﺎﹰ ﻤﻥ ﻤﻌﺎﺩﻻﺕ‬
:(‫ﺍﻟﺤﺭﻜﺔ ﺍﻟﻨﺴﺒﻴﺔ ﻭﻫﻲ ﺍﻟﺘﺎﻟﻴﺔ )ﺭﺍﺠﻊ ﻤﻭﺍﻀﻴﻊ ﺍﻟﻤﻴﻜﺎﻨﻴﻙ ﺍﻟﻔﻴﺯﻴﺎﺌﻲ‬
   
m Wr  F  J e  J c (13)
:‫ﺤﻴﺙ‬
           
J e  m W e   m W 0     OM    (  OM
   
    2   2 
  m    OM    OM    m OM (14)
   

:‫ ﻓﺘﻌﻁﻰ ﺒﺎﻟﻤﻌﻴﻥ‬J C ‫ﺃﻤﺎ ﻗﻭﺓ ﺍﻟﻌﻁﺎﻟﺔ ﺍﻟﺠﺭﻴﺔ‬
  
I J K
  
J c  2 m   v r  O O  (15)
X Y Z

 
‫( ﺘﻡ ﺍﻻﺴﻘﺎﻁ ﻋﻠﻰ ﻤﺤﺎﻭﺭ‬15) ‫( ﻭ‬14) ‫ ﺒﻘﻴﻤﺘﻴﻬﻤﺎ ﻤﻥ‬J c ‫ ﻭ‬J e ‫ﻭﺒﺘﺒﺩﻴل‬
‫( ﻭ‬10) ‫ ﻨﺤﺼل ﻋﻠﻰ ﺍﻟﻤﻌﺎﺩﻻﺕ‬OXYZ ‫ﺍﻟﺜﻼﺜﻴﺔ ﺍﻟﻤﺘﺤﺭﻜﺔ ﺍﻟﻤﺘﻤﺎﺴﻜﺔ ﻤﻊ ﺍﻟﻭﺴﻁ‬
.‫( ﻨﻔﺴﻬﺎ‬12) ‫( ﻭ‬11)

- 64 -

Create PDF files without this message by purchasing novaPDF printer (http://www.novapdf.com)
‫‪‬‬
‫‪ - 3‬ﻴﺩﻭﺭ ﻤﻨﻅﻡ ﺘﺤﺕ ﺘﺄﺜﻴﺭ ﻋﺯﻡ ‪ ، ‬ﻭﺯﻥ ﻜل ﻤﻥ ﻜﺭﺘﻲ ﺍﻟﻤﻨﻅﻡ ‪N ،M‬‬
‫ﻴﺴﺎﻭﻱ ‪ P1‬ﻭﻫﻤﺎ ﻤﺜﺒﺘﺎﻥ ﻓﻲ ﻁﺭﻓﻲ ﺫﺭﺍﻋﻴﻥ ‪ OM‬ﻭ ‪ ON‬ﻁﻭل ﻜل ﻤﻨﻬﻤﺎ ‪ ،a‬ﺃﻤﺎ‬
‫ﺍﻟﺤﻠﻘﺔ ‪ B‬ﺫﺍﺕ ﺍﻟﻭﺯﻥ ‪ P2‬ﻓﻴﻤﻜﻥ ﺃﻥ ﺘﻨﺯﻟﻕ ﻋﻠﻰ ﻤﺤﻭﺭ ﺸﺎﻗﻭﻟﻲ ﺩﻭﻥ ﺍﺤﺘﻜﺎﻙ‬
‫ﻭﻫﻲ ﻤﺭﺒﻭﻁﺔ ﺒﻨﺎﺒﺽ ﺜﺒﺘﺕ ﻨﻬﺎﻴﺘﻪ ﺍﻟﺜﺎﻨﻴﺔ ﻓﻲ ﻨﻘﻁﺔ ‪ ، B0‬ﻭﻗﺩ ﻭﺼﻠﺕ ﺍﻟﺤﻠﻘﺔ ‪B‬‬

‫ﺒﺫﺭﺍﻋﻴﻥ ﺁﺨﺭﻴﻥ ‪ BA‬ﻭ ‪ BD‬ﺒﺤﻴﺙ ﻴﺘﺸﻜل ﻤﻌﻴﻥ ‪ OABD‬ﻁﻭل ﻀﻠﻌﻪ ‪ ‬ﺸﻜل‬


‫)‪.(2.2‬‬

‫ﺍﻟﺸﻜﻞ )‪(2.2‬‬

‫ﺍﻭﺠﺩ ﺍﻟﻤﻌﺎﺩﻻﺕ ﺍﻟﺘﻔﺎﻀﻠﻴﺔ ﻟﺤﺭﻜﺔ ﺍﻟﻤﻨﻅﻡ ﻋﻠﻤ ﺎﹰ ﺃﻥ ﺍﻟﻜﺘﻠﺘﻴﻥ ‪ M‬ﻭ ‪N‬‬


‫ﺘﻌﺘﺒﺭﺍﻥ ﻨﻘﻁﻴﺘﻴﻥ ﻭﺃﻥ ﻋﺯﻡ ﻋﻁﺎﻟﺔ ﺍﻟﺤﻠﻘﺔ ‪ B‬ﺤﻭل ﻤﺤﻭﺭﻫﺎ ﻴﺴﺎﻭﻱ ‪ I‬ﻭﺃﻥ ﻜﺘﻠﺔ‬
‫ﺍﻷﺫﺭﻋﺔ ﻤﻬﻤﻠﺔ‪.‬‬

‫‪- 65 -‬‬

‫)‪Create PDF files without this message by purchasing novaPDF printer (http://www.novapdf.com‬‬
‫ﺍﳊﻞ ‪:‬‬
‫ﺴﻨﺤل ﻫﺫﻩ ﺍﻟﻤﺴﺄﻟﺔ ﺍﻨﻁﻼﻗ ﺎﹰ ﻤﻥ ﺍﻟﻤﻌﺎﺩﻻﺕ )‪ (2.20‬ﻟﺫﺍ ﻴﺠﺏ ﺃﻭﻻﹰ ﺍﺨﺘﻴﺎﺭ‬
‫ﺍﻹﺤﺩﺍﺜﻴﺎﺕ ﺍﻟﻤﻌﻤﻤﺔ ﺜﻡ ﺤﺴﺎﺏ ﺍﻟﻁﺎﻗﺔ ﺍﻟﺤﺭﻜﺔ ﻭﺍﻟﻘﻭﻯ ﺍﻟﻤﻌﻤﻤﺔ‪.‬‬
‫ﻨﺴﺘﺨﺩﻡ ﺍﻹﺤﺩﺍﺜﻴﺎﺕ ﺍﻟﻜﺭﻭﻴﺔ ﻓﻨﻌﻴﻥ ﺍﻟﻨﻘﻁﺔ ‪ M‬ﺒﺜﻼﺜﺔ ﻭﺴﻁﺎﺀ )‪ (r, , ‬ﺤﻴﺙ‬
‫‪ . r  a‬ﻭﻴﺴﻬل ﺘﻌﻴﻴﻥ ﺍﻟﻨﻘﻁﺔ ‪ N‬ﺒﻌﺩ ﺫﻟﻙ ﺒﺈﻀﺎﻓﺔ ‪ ‬ﺇﻟﻰ ﺍﻟﺯﺍﻭﻴﺔ ‪) ‬ﺍﻨﻅﺭ ﺍﻟﺸﻜل(‬
‫ﻭﻫﻜﺫﺍ ﻴﻜﻭﻥ )‪ N(a , ,   ), M (a , , ‬ﻤﻊ ﺍﻟﻌﻠﻡ ﺃﻥ ‪.     ‬‬
‫ﻭﺘﻜﻭﻥ ﺍﻟﻁﺎﻗﺔ ﺍﻟﺤﺭﻜﻴﺔ ﻟﻬﺎ‪:‬‬
‫‪1‬‬
‫‪‬‬ ‫‪‬‬ ‫‪‬‬
‫‪T  2 m1a 2   2    2 sin 2   m1a 2  2   2 sin 2 ‬‬
‫‪2‬‬
‫‪‬‬ ‫)‪(1‬‬

‫ﻭﻤﻥ ﺍﻟﺴﻬل ﺤﺴﺏ ﺍﻟﻁﺎﻗﺔ ﺍﻟﺤﺭﻜﻴﺔ ﻟﻠﻨﻘﻁﺔ ‪ B‬ﺤﻴﺙ ﻴﺒﺩﻭ ﻤﻥ ﺍﻟﺸﻜل ﺃﻥ‪:‬‬
‫‪OB  z ( B)  2 cos ‬‬ ‫)‪(2‬‬
‫ﻭﻤﻨﻪ ﻨﺤﺼل ﻋﻠﻰ ﺍﻟﺴﺭﻋﺔ ﺍﻻﻨﺴﺤﺎﺒﻴﺔ ﻟﻠﺤﻠﻘﺔ ‪ B‬ﺫﺍﺕ ﺍﻟﻜﺘﻠﺔ ‪: m 2‬‬
‫‪v(B)  z  (B)  2  sin   ‬‬
‫ﺃﻤﺎ ﺍﻟﻁﺎﻗﺔ ﺍﻟﺤﺭﻜﻴﺔ ﻟﻬﺫﻩ ﺍﻟﺤﻠﻘﺔ ﻓﺘﺘﺄﻟﻑ ﻤﻥ ﻁﺎﻗﺘﻴﻥ‪ :‬ﺍﻷﻭﻟﻰ ﻫﻲ ﺍﻻﻨﺴﺤﺎﺒﻴﺔ‬
‫ﺍﻟﺘﺎﻟﻴﺔ‪:‬‬
‫‪1‬‬ ‫‪1‬‬
‫‪Tt (B) ‬‬ ‫)‪mv2 (B)  4m 2 sin 2    2  2m  2   2 sin 2  (3‬‬
‫‪2‬‬ ‫‪2‬‬
‫ﻭﺍﻟﺜﺎﻨﻴﺔ ﻫﻲ ﺍﻟﻁﺎﻗﺔ ﺍﻟﺤﺭﻜﻴﺔ ﺍﻟﺩﻭﺭﺍﻨﻴﺔ ﺤﻭل ﺍﻟﻤﺤﻭﺭ ‪ oz‬ﺍﻟﺘﻲ ﺘﺴﺎﻭﻱ‬
‫ﺃﻱ ﺃﻥ‪:‬‬
‫‪1 2‬‬
‫‪I‬‬
‫‪2‬‬
‫‪1 2‬‬
‫‪T(B)  Tt (B)  Trot (B)  2m  2   2 sin 2  ‬‬ ‫‪I‬‬ ‫)‪(4‬‬
‫‪2‬‬
‫ﺃﻤﺎ ﺍﻟﻁﺎﻗﺔ ﺍﻟﺤﺭﻜﻴﺔ ﻟﻜل ﺍﻟﻤﻨﻅﻡ ﻓﺘﺴﺎﻭﻱ‪:‬‬

‫‪‬‬ ‫‪‬‬ ‫‪1‬‬


‫)‪T  m1a 2   2    2 sin 2   2m 2  2   2 sin 2   I  2  (5‬‬
‫‪2‬‬
‫‪‬‬ ‫‪‬‬ ‫‪1‬‬
‫‪‬‬
‫‪ m1a 2  2m 2  2 sin 2    2  I  2m1a 2 sin 2    2‬‬
‫‪2‬‬
‫‪‬‬

‫‪- 66 -‬‬

‫)‪Create PDF files without this message by purchasing novaPDF printer (http://www.novapdf.com‬‬
‫ﻟﻨﺤﺴﺏ ﺍﻵﻥ ﺍﻟﻘﻭﻯ ﺍﻟﻤﻌﻤﻤﺔ ﻓﻨﻼﺤﻅ ﺃﻥ ﺍﻟﻘﻭﻯ ﺍﻟﻤﺅﺜﺭﺓ ﻫﻲ ﺃﺜﻘﺎل ﺍﻟﻜﺘل ‪M‬‬
‫ﻭ ‪ N‬ﻭ ‪ B‬ﻭﻗﻭﺓ ﺸﺩ ﺍﻟﻨﺎﺒﺽ ﺒﺎﻻﻀﺎﻓﺔ ﺇﻟﻰ ﺍﻟﻌﺯﻡ ‪ ‬ﺍﻟﺫﻱ ﻴﺩﻭﺭ ﺍﻟﻤﻨﻅﻡ‪ ،‬ﺃﻥ ﻋﻤل‬
‫ﺍﻷﺜﻘﺎل ﻋﻨﺩﻤﺎ ﻴﺤﺩﺙ ﺍﻨﺘﻘﺎل ﻋﻨﺼﺭﻱ ﻓﻬﻭ‪:‬‬
‫‪‬‬ ‫‪‬‬ ‫‪‬‬ ‫‪‬‬ ‫‪‬‬ ‫‪‬‬
‫‪A1  P1 . M  P1 . N  P2 . B  2P1z z (M )  P2z z (B) ‬‬
‫)‪ 2P1z( N )  P2 z (B‬‬ ‫)‪(6‬‬
‫ﺤﻴﺙ‪:‬‬
‫‪z ( M )  a cos ‬‬ ‫‪,‬‬ ‫‪z ( B)  2 cCos ‬‬
‫‪ z( M )   a sin   ,‬‬ ‫‪ z( B)  2  sin  ‬‬ ‫)‪(7‬‬
‫ﻭﺒﺎﻟﺘﻌﻭﻴﺽ ﻓﻲ )‪ (6‬ﻨﺠﺩ‪:‬‬
‫)‪ A1  2P1a sin    2P2 sSin    2( P1a  P2 ) sin   (8‬‬

‫ﻭﻟﺤﺴﺎﺏ ﺍﻟﻌﻤل ﺍﻻﻓﺘﺭﺍﻀﻲ ﺍﻟﻨﺎﺘﺞ ﻋﻥ ﺸﺩ ﺍﻟﻨﺎﺒﺽ ﻴﺠﺏ ﺤﺴﺎﺏ ﺍﺴﺘﻁﺎﻟﺘﻪ‬


‫ﻟﺫﺍ ﻨﻔﺭﺽ ﺃﻥ ﺍﻷﺫﺭﻋﺔ ﻜﺎﻨﺕ ﻤﻨﻁﺒﻘﺔ ﻋﻠﻰ ‪ oz‬ﻋﻨﺩﻤﺎ ﻜﺎﻥ ﺍﻟﻨﺎﺒﺽ ‪ z‬ﻓﻲ ﻭﻀﻊ‬
‫‪‬‬
‫ﺍﻟﺭﺍﺤﺔ‪ ،‬ﻭﺒﺘﺄﺜﻴﺭ ﺍﻟﻌﺯﻡ ‪ ‬ﺘﺒﺩﺃ ﺍﻷﺫﺭﻋﺔ ﺒﺎﻻﺒﺘﻌﺎﺩ ﻋﻥ ‪ oz‬ﻭﻴﺒﺩﻭ ﻤﻥ ﺍﻟﺸﻜل ﺍﻥ‬
‫ﺍﺴﺘﻁﺎﻟﺔ ﺍﻟﻨﺎﺒﺽ ‪ ‬ﺘﺴﺎﻭﻱ‪:‬‬
‫)‪  z (B 0 )  z (B‬‬
‫ﺤﻴﺙ ) ‪ z (B0‬ﻫﻭ ﺭﺍﻗﻡ ﺍﻟﻨﻘﻁﺔ ‪ B‬ﻋﻨﺩﻤﺎ ﻜﺎﻨﺕ ﺍﻷﺫﺭﻋﺔ ﻤﻨﻁﺒﻘﺔ ﻋﻠﻰ ‪oz‬‬

‫ﻭﻫﻭ ﻴﺴﺎﻭﻱ ‪  2 ‬ﻭ )‪ z(B‬ﻫﻭ ﺭﺍﻗﻡ ﺍﻟﻨﻘﻁﺔ ‪ B‬ﺍﻟﺤﺎﻟﻲ ﺃﺜﻨﺎﺀ ﺍﻟﺩﻭﺭﺍﻥ ﻭﻫﻭ‬
‫ﻴﺴﺎﻭﻱ ‪ 2 cos ‬ﺃﻱ ﺃﻥ‪:‬‬
‫)‪  2   2  cos   2(1  cos ‬‬ ‫)‪(9‬‬
‫ﺃﻤﺎ ﺍﻟﻘﻭﺓ ‪ F‬ﺍﻟﻨﺎﺘﺠﺔ ﻋﻥ ﻫﺫﻩ ﺍﻻﺴﺘﻁﺎﻟﺔ ﻓﻬﻲ‪:‬‬
‫)‪F  C   2C (1  cos ‬‬ ‫)‪(10‬‬
‫ﺃﻤﺎ ﺍﻟﻌﻤل ﺍﻟﻌﻨﺼﺭﻱ ﺍﻻﻓﺘﺭﺍﻀﻲ ﺍﻟﻨﺎﺘﺞ ﻋﻥ ﻫﺫﻩ ﺍﻟﻘﻭﺓ ﻓﻬﻭ‪:‬‬
‫‪‬‬ ‫‪‬‬
‫)‪A 2  F . r  Fz z (B)  2C  (1  cos )(2  sin  ‬‬
‫‪ 4C  2 (1  cos ) sin  .‬‬ ‫)‪(11‬‬
‫ﻭﺃﺨﻴﺭﺍﹰ ﻨﺠﺩ ﺃﻥ ﺍﻟﻌﻤل ﺍﻟﻌﻨﺼﺭﻱ ﺍﻟﻨﺎﺘﺞ ﻋﻥ ﺍﻟﻌﺯﻡ ﻴﺴﺎﻭﻱ‪:‬‬

‫‪- 67 -‬‬

‫)‪Create PDF files without this message by purchasing novaPDF printer (http://www.novapdf.com‬‬
‫‪A 3   ‬‬ ‫)‪(12‬‬
‫ﻭﻴﻜﻭﻥ ﺍﻟﻌﻤل ﺍﻻﻓﺘﺭﺍﻀﻲ ﺍﻟﻌﻨﺼﺭﻱ ﺍﻟﻜﻠﻲ‪ :‬ﺍﻟﻨﺎﺘﺞ ﻋﻥ ﺍﻷﺜﻘﺎل ﻭﺍﻟﻨﺎﺒﺽ‬
‫ﻭﺍﻟﻌﺯﻡ‪:‬‬
‫‪‬‬
‫‪A  A1  A 2  A 3  2 P1a  P2  2C  2 (1  cos ) sin     ‬‬ ‫‪‬‬
‫ﺃﻱ ﺃﻥ‪:‬‬
‫‪‬‬
‫‪Q   2 P1a  P2   2C  2 (1  cos ) sin ‬‬ ‫‪‬‬ ‫)‪(13‬‬
‫‪Q  ‬‬ ‫)‪(14‬‬
‫ﻭﻟﻠﺤﺼﻭل ﻋﻠﻰ ﻤﻌﺎﺩﻻﺕ ﻻﻏﺭﺍﻨﺞ ﺍﻟﻤﻁﻠﻭﺒﺔ ﻨﺸﺘﻕ ﺍﻟﻁﺎﻗﺔ ﺍﻟﺤﺭﻜﻴﺔ )‪(5‬‬
‫ﻓﻨﺠﺩ‪:‬‬
‫‪T‬‬
‫‪‬‬ ‫‪‬‬
‫‪‬‬
‫‪ 2 m1a 2  2m 2  2 sin 2   ,‬‬ ‫‪‬‬ ‫‪‬‬
‫‪T‬‬
‫‪‬‬
‫‪‬‬ ‫‪‬‬
‫)‪ 2 I  2m1a 2 sin 2    ( 15‬‬

‫‪T‬‬ ‫‪T‬‬
‫‪ m1a 2   2 sin 2  2m 2  2   2 sin 2,‬‬ ‫‪0‬‬ ‫)‪( 16‬‬
‫‪‬‬ ‫‪‬‬
‫ﻭﻤﻥ )‪ (13‬ﻭ )‪ (14‬ﻭ )‪ (15‬ﻭ )‪ (16‬ﻨﺤﺼل ﻋﻠﻰ ﻤﻌﺎﺩﻻﺕ ﺍﻟﺤﺭﻜﺔ‬
‫ﺍﻟﺘﺎﻟﻴﺔ‪:‬‬
‫‪d‬‬
‫‪dt‬‬
‫‪‬‬ ‫‪ ‬‬
‫‪2m1a 2  2m 2  2 sin 2     m1a 2  2 sin 2  2m 2  2   2 sin 2 ‬‬

‫‪‬‬
‫‪ 2 P1a  P2  2C  2 (1  cos ) sin ‬‬ ‫‪‬‬ ‫)‪(17‬‬
‫‪d‬‬
‫‪dt‬‬
‫‪I  2m a‬‬ ‫‪1‬‬
‫‪2‬‬
‫‪‬‬
‫‪sin 2   '  ‬‬ ‫)‪(18‬‬

‫‪ - 4‬ﻴﻁﻠﺏ ﺍﻟﺤﺼﻭل ﻋﻠﻰ ﻤﻌﺎﺩﻻﺕ ﺤﺭﻜﺔ ﻨﻘﻁﺔ ﻤﺎﺩﻴﺔ ﻁﻠﻴﻘﺔ ﻓﻲ‬


‫ﺍﻹﺤﺩﺍﺜﻴﺎﺕ ﺍﻻﺴﻁﻭﺍﻨﻴﺔ ﻭﺍﻟﻜﺭﻭﻴﺔ ﺍﻨﻁﻼﻗﺎﹰ ﻤﻥ ﺍﻟﻤﻌﺎﺩﻻﺕ )‪ (2.20‬ﻭﺍﻟﻤﻌﺎﺩﻻﺕ‬
‫)‪.(2.25‬‬

‫‪‬‬
‫‪ - 5‬ﻴﻭﻀﻊ ﻨﻭﺍﺱ ﻤﺸﺤﻭﻥ ﺒﺸﺤﻨﺔ ‪ q‬ﻓﻲ ﺤﻘل ﻜﻬﺭﺒﺎﺌﻲ ‪ ‬ﻤﺘﺠﺎﻨﺱ ﻴﺘﺠﻪ‬
‫‪‬‬
‫ﻨﺤﻭ ﺍﻷﻋﻠﻰ ﻓﺈﺫﺍ ﻋﻠﻤﺕ ﺃﻥ ﺜﻘل ﺍﻟﻨﻭﺍﺱ ﻫﻭ ‪ m g‬ﻓﺎﺤﺴﺏ ﺘﺎﺒﻊ ﻻﻏﺭﺍﻨﺞ ﻟﻪ ﺜﻡ‬
‫‪‬‬ ‫‪‬‬
‫ﺃﻭﺠﺩ ﻗﺎﻨﻭﻥ ﺤﺭﻜﺘﻪ‪ .‬ﻨﺎﻗﺵ ﺤﺴﺏ ﺍﻟﻘﻴﻡ ﺍﻟﻤﺨﺘﻠﻔﺔ ﻟﻜل ﻤﻥ ‪ ‬ﻭ ‪. g‬‬

‫‪- 68 -‬‬

‫)‪Create PDF files without this message by purchasing novaPDF printer (http://www.novapdf.com‬‬
‫‪ - 6‬ﻴﺘﺤﺭﻙ ﺠﺴﻴﻡ ﻓﻲ ﺤﻘل ﺍﻟﺜﻘﺎﻟﺔ ﺍﻷﺭﻀﻴﺔ ﻋﻠﻰ ﺍﻟﺴﻁﺢ ﺍﻟﺩﺍﺨﻠﻲ ﻟﻤﺨﺭﻭﻁ‬
‫ﻗﺎﺌﻡ ﺯﺍﻭﻴﺘﻪ ‪ ‬ﻭﻤﺤﻭﺭﻩ ‪ oz‬ﻭﺭﺃﺴﻪ ﻓﻲ ﻤﺒﺩﺃ ﺍﻹﺤﺩﺍﺜﻴﺎﺕ ﻭﻗﺎﻋﺩﺘﻪ ﺃﻋﻠﻰ ﻤﻥ ﺭﺃﺴﻪ‪.‬‬
‫ﺍﻜﺘﺏ ﻤﻌﺎﺩﻻﺕ ﺍﻟﺤﺭﻜﺔ‪.‬‬

‫‪ - 7‬ﺃﻋﺩ ﻨﻔﺱ ﺍﻟﺴﺅﺍل ﻋﻨﺩﻤﺎ ﺘﺘﻡ ﺍﻟﺤﺭﻜﺔ ﻋﻠﻰ ﻨﺼﻑ ﻜﺭﺓ ﻨﺼﻑ ﻗﻁﺭﻫﺎ ‪a‬‬

‫)ﺍﻟﻨﻭﺍﺱ ﺍﻟﻜﺭﻭﻱ(‪.‬‬

‫‪ - 8‬ﺍﻭﺠﺩ ﻗﺎﻨﻭﻥ ﺤﺭﻜﺔ ﻨﻘﻁﺔ ﻤﺎﺩﻴﺔ ﺇﺫﺍ ﺃﻋﻁﻲ ﺘﺎﺒﻊ ﻻﻏﺭﺍﻨﺞ ﺒﻜل ﻤﻥ‬
‫ﺍﻟﻌﻼﻗﺘﻴﻥ ﺍﻟﺘﺎﻟﻴﺘﻴﻥ‪:‬‬
‫‪‬‬
‫‪2‬‬ ‫‪2‬‬ ‫‪2‬‬ ‫‪x‬‬ ‫‪2‬‬
‫‪‬‬ ‫‪x x‬‬ ‫‪ x‬‬ ‫‪y‬‬
‫‪a ) L(x , x )  e‬‬ ‫‪ 2x e‬‬ ‫‪‬‬ ‫‪e‬‬ ‫‪dy‬‬
‫‪0‬‬
‫‪‬‬ ‫‪1 t   2‬‬ ‫‪2 2‬‬
‫‪b) L( x, x , t ) ‬‬ ‫‪e  x   x ‬‬
‫‪2‬‬ ‫‪‬‬ ‫‪‬‬

‫‪2 2‬‬
‫‪ - 9‬ﺠﺴﻴﻡ ﻴﺘﺤﺭﻙ ﻓﻲ ﺍﻟﻜﻤﻭﻥ‪:‬‬
‫‪m x‬‬
‫‪V (x ) ‬‬
‫‪2‬‬
‫ﺁ‪ -‬ﻫل ﻴﻤﻜﻥ ﻤﺒﺎﺸﺭﺓ ﻜﺘﺎﺒﺔ ﺘﻜﺎﻤل ﺃﻭﻟﻲ ﻟﻠﺤﺭﻜﺔ؟ ﻭﻟﻤﺎﺫﺍ؟‬
‫ﺏ‪ -‬ﺃﻭﺠﺩ ﻗﺎﻨﻭﻥ ﺍﻟﺤﺭﻜﺔ ﺒﺎﻟﻁﺭﻴﻘﺔ ﺍﻟﻌﺎﻤﺔ‪.‬‬
‫ﺝ‪ -‬ﺍﺴﺘﺨﺩﻡ ﺘﺎﺒﻊ ﻻﻏﺭﺍﻨﺞ ﻻﻴﺠﺎﺩ ﻗﺎﻨﻭﻥ ﺍﻟﺤﺭﻜﺔ ﻭﻗﺎﺭﻥ ﻤﻊ ﻨﺘﺎﺌﺞ‬
‫ﺍﻟﻁﻠﺏ ﺍﻟﺜﺎﻨﻲ‪.‬‬

‫‪ - 10‬ﺃﻭﺠﺩ ﻗﺎﻨﻭﻥ ﺤﺭﻜﺔ ﺠﺴﻴﻡ ﺇﺫﺍ ﺃﻋﻁﻲ ﺘﺎﺒﻊ ﻻﻏﺭﺍﻨﺞ ﻟﻪ ﺒﺎﻟﻌﻼﻗﺔ‪:‬‬


‫‪2‬‬ ‫‪2‬‬ ‫‪2 2‬‬ ‫‪2 2‬‬
‫‪‬‬ ‫‪‬‬ ‫‪x‬‬ ‫‪y‬‬ ‫‪ x  0 y‬‬
‫‪L( x, x , y, y ) ‬‬ ‫‪ 1‬‬
‫‪2‬‬ ‫‪2‬‬

‫‪- 69 -‬‬

‫)‪Create PDF files without this message by purchasing novaPDF printer (http://www.novapdf.com‬‬
‫‪ - 11‬ﺒﺭﻫﻥ ﺃﻨﻪ ﻋﻨﺩﻤﺎ ﻴﺘﺤﺭﻙ ﺠﺴﻴﻡ ﻓﻲ ﻜﻤﻭﻥ ﻨﺎﺘﺞ ﻋﻥ ﻤﺴﺘﻭﹴ ﻻﻤﺘﻨﺎﻫﻲ‬
‫ﻓﻲ ﺍﻻﺘﺴﺎﻉ ﻓﺈﻥ ﺍﻟﻤﺭﻜﺒﺔ ‪ L z‬ﻭﺍﻟﻤﺭﻜﺒﺘﻴﻥ ‪ Py , Px‬ﺘﺒﻘﻴﺎﻥ ﻤﺼﺎﻨﺘﻴﻥ‪.‬‬

‫‪ - 12‬ﺍﺤﺴﺏ ﺘﻭﺍﺘﺭ ﺍﻻﻫﺘﺯﺍﺯﺍﺕ ﺍﻟﺼﻐﻴﺭﺓ ﻟﻨﻘﻁﺔ ﻤﺎﺩﻴﺔ ﻜﺘﻠﺘﻬﺎ ‪ m‬ﺘﺘﺤﺭﻙ‬


‫‪‬‬
‫ﻋﻠﻰ ﻤﺴﺘﻘﻴﻡ ﻭﻫﻲ ﻤﺸﺩﻭﺩﺓ ﺒﻨﺎﺒﺽ ﻗﺴﺎﻭﺘﻪ ‪ k‬ﻤﻥ ﺍﻟﻨﻘﻁﺔ ‪ A‬ﺒﻘﻭﺓ ‪) F‬ﺸﻜل ‪.(2.3‬‬
‫ﺃﻋﺩ ﺍﻟﺴﺅﺍل ﻨﻔﺴﻪ ﻋﻨﺩﻤﺎ ﺘﺘﺤﺭﻙ ﺍﻟﻨﻘﻁﺔ ﻋﻠﻰ ﺩﺍﺌﺭﺓ ﻨﺼﻑ ﻗﻁﺭﻫﺎ ﻤﻌﻠﻭﻡ‪.‬‬

‫ﺷﻜﻞ )‪(2.3‬‬

‫‪ - 13‬ﺃﻭﺠﺩ ﻗﺎﻨﻭﻥ ﺤﺭﻜﺔ ﺠﺴﻴﻡ ﻴﺠﺒﺭ ﻋﻠﻰ ﺍﻻﻫﺘﺯﺍﺯ ﺘﺤﺕ ﺘﺄﺜﻴﺭ ﻗﻭﺓ‬
‫ﺍﻟﻤﺭﻭﻨﺔ ‪  kx‬ﻭﻗﻭﺓ ﺃﺨﺭﻯ ﺩﻭﺭﻴﺔ ﻓﻲ ﻜل ﻤﻥ ﺍﻟﺤﺎﻻﺕ ﺍﻟﺘﺎﻟﻴﺔ‪:‬‬
‫‪b) F  a t‬‬ ‫‪a ) F  F0  Const.‬‬
‫‪ t‬‬
‫‪c ) F  F0e‬‬ ‫‪d ) F  F0e  t cos  t‬‬
‫)ﺘﻭﺠﻴﻪ‪ :‬ﺍﺴﺘﺨﺩﺍﻡ ﺍﻟﻤﻌﺎﺩﻻﺕ )‪ (2.20‬ﻭﺍﺤﺴﺏ ﺍﻟﻘﻭﻯ ﺍﻟﻤﻌﻤﻤﺔ ﻓﻲ ﻜل ﺤﺎﻟﺔ(‪.‬‬

‫‪ - 14‬ﺍﺤﺴﺏ ﺘﺎﺒﻊ ﻻﻏﺭﺍﻨﺞ ﻟﻠﻨﻭﺍﺱ ﺍﻟﻤﺴﺘﻭﻯ ﺍﻟﺜﻨﺎﺌﻲ ﺍﻟﻤﻭﻀﺢ ﻋﻠﻰ ﺍﻟﺸﻜل‬


‫)‪.(2.4‬‬

‫‪- 70 -‬‬

‫)‪Create PDF files without this message by purchasing novaPDF printer (http://www.novapdf.com‬‬
‫ﺷﻜﻞ )‪(2.4‬‬

‫‪ - 15‬ﺍﺤﺴﺏ ﺘﺎﺒﻊ ﻻﻏﺭﺍﻨﺞ ﻟﻨﻭﺍﺱ ﻤﺴﺘﻭ ﻓﻲ ﺍﻟﺤﺎﻻﺕ ﺍﻟﺜﻼﺙ ﺍﻟﺘﺎﻟﻴﺔ‪:‬‬


‫ﺁ‪ -‬ﺘﺘﺤﺭﻙ ﻨﻘﻁﺔ ﺘﻌﻠﻴﻘﺔ ﺒﺤﺭﻜﺔ ﺠﻴﺒﻴﺔ ﻋﻠﻰ ﺍﻟﻤﺤﻭﺭ ﺍﻟﺸﺎﻗﻭﻟﻲ ‪.oy‬‬
‫ﺏ‪ -‬ﺘﺘﺤﺭﻙ ﻨﻘﻁﺔ ﺘﻌﻠﻴﻘﺔ ﺒﺤﺭﻜﺔ ﺠﻴﺒﻴﺔ ﻋﻠﻰ ﺍﻟﻤﺤﻭﺭ ﺍﻷﻓﻘﻲ ‪.oz‬‬
‫ﺝ‪ -‬ﺘﺘﺤﺭﻙ ﻨﻘﻁﺔ ﺘﻌﻠﻴﻘﺔ ﺒﺴﺭﻋﺔ ﺨﻁﻴﺔ ﺜﺎﺒﺘﺔ ﻋﻠﻰ ﺩﺍﺌﺭﺓ ﻨﺼﻑ ﻗﻁﺭﻫﺎ‬
‫‪ a‬ﺒﺴﺭﻋﺔ ﺯﺍﻭﻴﺔ ‪.‬‬

‫‪ - 16‬ﺃﻭﺠﺩ ﺍﻟﻤﻌﺎﺩﻟﺔ ﺍﻟﺘﻔﺎﻀﻠﻴﺔ ﻟﺤﺭﻜﺔ ﺠﺴﻡ ﺼﻠﺏ ﻴﺩﻭﺭ ﺤﻭل ﻤﺤﻭﺭ ﺜﺎﺒﺕ‬
‫‪‬‬
‫ﺘﺤﺕ ﺘﺄﺜﻴﺭ ﻋﺯﻡ ‪ M‬ﺒﺎﺴﺘﺨﺩﺍﻡ )‪ (2.20‬ﻭ )‪.(2.25‬‬

‫‪ - 17‬ﻴﺘﺤﺭﻙ ﺠﺴﻴﻡ ﻋﻠﻰ ﻤﻨﺤﻥ ﺸﺎﻗﻭﻟﻲ ﻤﻌﺎﺩﻟﺘﺎﻩ ﺍﻟﻭﺴﻴﻁﺘﺎﻥ‪:‬‬


‫)‪x  x ( s‬‬ ‫‪,‬‬ ‫)‪y  y(s‬‬
‫ﺤﻴﺙ ‪ s‬ﻫﻭ ﻁﻭل ﻗﻭﺱ ﺍﻟﻤﻨﺤﻨﻲ‪.‬‬
‫ﺁ‪ -‬ﺍﻜﺘﺏ ﻤﻌﺎﺩﻻﺕ ﻻﻏﺭﺍﻨﺞ ﻟﻬﺫﺍ ﺍﻟﺠﺴﻴﻡ‪.‬‬
‫ﺏ‪ -‬ﺇﺫﺍ ﻜﺎﻨﺕ ﻤﻌﺎﺩﻟﺔ ﺍﻟﻤﻨﺤﻨﻲ ﻤﻥ ﺍﻟﺸﻜل )ﺴﻴﻜﻠﻭﺌﻴﺩ(‪:‬‬
‫)‪x  a (s  sin s ) , y  a (1  cos s‬‬
‫ﻓﺎﻜﺘﺏ ﻤﻌﺎﺩﻻﺕ ﻻﻏﺭﺍﻨﺞ ﻓﻲ ﻫﺫﻩ ﺍﻟﺤﺎﻟﺔ‪.‬‬

‫‪- 71 -‬‬

‫)‪Create PDF files without this message by purchasing novaPDF printer (http://www.novapdf.com‬‬
‫‪ - 18‬ﺘﺘﺤﺭﻙ ﻨﻘﻁﺔ ﺘﻌﻠﻴﻕ ﻨﻭﺍﺱ‪ ،‬ﻁﻭﻟﻪ ‪ ‬ﻭﻜﺘﻠﺘﻪ ‪ m‬ﻋﻠﻰ ﻤﺴﺘﻭﹴ ﻤﺎﺌل ﻋﻠﻰ‬
‫ﺍﻷﻓﻕ ﺒﺯﺍﻭﻴﺔ ‪ .‬ﺍﺨﺘﺭ ﺍﻹﺤﺩﺍﺜﻴﺎﺕ ﺍﻟﻤﻌﻤﻤﺔ ﻭﺍﺴﺘﺨﺩﻡ ﺍﻟﻤﻌﺎﺩﻻﺕ )‪ (2.20‬ﻟﻜﺘﺎﺒﺔ‬
‫ﻤﻌﺎﺩﻻﺕ ﺤﺭﻜﺔ ﻫﺫﺍ ﺍﻟﻨﻭﺍﺱ‪.‬‬

‫‪ - 19‬ﻴﺘﺤﺭﻙ ﺠﺴﻴﻡ ﺘﺤﺕ ﺘﺄﺜﻴﺭ ﻗﻭﺓ ﻭﺤﻴﺩﺓ ﺘﻤﺭ ﻤﻥ ﻨﻘﻁﺔ ﺜﺎﺒﺘﺔ ‪ O‬ﻭﺘﻌﻁﻰ‬
‫ﺒﺎﻟﻌﻼﻗﺔ‪:‬‬
‫‪‬‬ ‫‪2 ‬‬ ‫‪2‬‬
‫‪F  mk OM  mk r‬‬
‫ﺁ‪ -‬ﺒﺭﻫﻥ ﺃﻥ ﻫﺫﺍ ﺍﻟﺠﺴﻴﻡ ﻴﺘﺤﺭﻙ ﻓﻲ ﻤﺴﺘﻭ ﻴﻁﻠﺏ ﺘﻌﻴﻴﻨﻪ‪.‬‬
‫ﺏ‪ -‬ﺃﻭﺠﺩ ﺍﻟﻤﺴﺎﺭ ﻭﻗﺎﻨﻭﻥ ﺍﻟﺤﺭﻜﺔ ﺒﻁﺭﻴﻘﺘﻴﻥ ﺘﺨﺘﺎﺭﻫﻤﺎ‪.‬‬

‫‪ - 20‬ﺘﺘﺤﺭﻙ ﻨﻘﻁﺔ ﻤﺎﺩﻴﺔ ﻋﻠﻰ ﻤﺴﺘﻭ ﻤﻌﺎﺩﻟﺘﻪ‪:‬‬


‫‪2x  3y  z  1  0‬‬
‫ﺁ‪ -‬ﻤﺎﻋﺩﺩ ﻭﺴﻁﺎﺀ ﺍﻟﻤﺴﺄﻟﺔ؟ ﺍﺨﺘﺭ ﻫﺫﻩ ﺍﻟﻭﺴﻁﺎﺀ ﺒﺎﻟﺸﻜل ﺍﻟﺫﻱ ﺘﺭﺍﻩ ﻤﻨﺎﺴﺒﺎﹰ‪.‬‬
‫ﺏ‪ -‬ﺍﻜﺘﺏ ﻤﻌﺎﺩﻻﺕ ﻻﻏﺭﺍﻨﺞ ﻟﻬﺫﻩ ﺍﻟﻨﻘﻁﺔ ﻤﻊ ﺍﻟﻌﻠﻡ ﺃﻨﻬﺎ ﺘﺘﺤﺭﻙ ﺘﺤﺕ ﺘﺄﺜﻴﺭ‬
‫ﻗﻭﺓ ﻭﺤﻴﺩﺓ ﻓﻌﺎﻟﺔ ﻫﻲ ﺜﻘﻠﻬﺎ ﻭﺃﻥ ﺍﺭﺘﺒﺎﻁﻬﺎ ﺒﺎﻟﻤﺴﺘﻭﻯ ﻤﺜﺎﻟﻲ‪.‬‬

‫‪ - 21‬ﺍﻜﺘﺏ ﻤﻌﺎﺩﻻﺕ ﺍﻟﺤﺭﻜﺔ ﻓﻲ ﺍﻟﺤﻘل ﺍﻟﻤﺭﻜﺯﻱ ﺍﻨﻁﻼﻗﺎﹰ ﻤﻥ ﻤﻌﺎﺩﻻﺕ‬


‫ﻻﻏﺭﺍﻨﺞ‪.‬‬

‫‪ - 22‬ﺨﻴﻁ ﺨﻔﻴﻑ ﻋﻠﻘﺕ ﻓﻲ ﻁﺭﻓﻪ ﻜﺘﻠﺔ ‪ 2m‬ﻴﻤﺭ ﻋﻠﻰ ﺒﻜﺭﺓ ﻜﺘﻠﺘﻬﺎ ‪4m‬‬
‫ﻴﻤﻜﻨﻬﺎ ﺍﻟﺩﻭﺭﺍﻥ ﺤﻭل ﻤﺤﻭﺭ ﺃﻓﻘﻲ ﻭﻴﺘﺼل ﻁﺭﻑ ﺍﻟﺨﻴﻁ ﺍﻟﺜﺎﻨﻲ ﺒﺒﻜﺭﺓ ﺜﺎﻨﻴﺔ ﻜﺘﻠﺘﻬﺎ‬
‫‪ m‬ﻴﻤﻜﻨﻬﺎ ﺍﻟﺩﻭﺭﺍﻥ ﺤﻭل ﻤﺤﻭﺭ ﺃﻓﻘﻲ ﺃﻴﻀﺎﹰ‪ ،‬ﻜﻤﺎ ﻴﻤﻜﻨﻬﺎ ﺍﻟﺘﺤﺭﻙ ﻓﻲ ﻤﺴﺘﻭ ﺸﺎﻗﻭﻟﻲ‬
‫ﻭﻗﺩ ﻋﻠﻕ ﻋﻠﻴﻬﺎ ﺨﻴﻁ ﺁﺨﺭ ﻴﻨﺘﻬﻲ ﻤﻥ ﻁﺭﻓﻴﻪ ﺒﻜﺘﻠﺘﻴﻥ ﻫﻤﺎ ‪ 2m‬ﻭ ‪ 11m‬ﻋﻠﻰ‬
‫ﺍﻟﺘﺭﺘﻴﺏ ﺸﻜل )‪ .(2.5‬ﺍﻜﺘﺏ ﻤﻌﺎﺩﻻﺕ ﺍﻟﺤﺭﻜﺔ‪.‬‬

‫‪- 72 -‬‬

‫)‪Create PDF files without this message by purchasing novaPDF printer (http://www.novapdf.com‬‬
‫ﺍﻟﺸﻜﻞ )‪(2.5‬‬

‫‪ - 23‬ﺃﻭﺠﺩ ﺃﻭﻀﺎﻉ ﺘﻭﺍﺯﻥ ﺍﻟﻤﺠﻤﻭﻋﺔ ﺍﻟﻤﺎﺩﻴﺔ ﺍﻟﻤﻭﻀﺤﺔ ﻋﻠﻰ ﺍﻟﺸﻜل‬


‫ﺍﻟﻤﺠﺎﻭﺭ ﺜﻡ ﺍﺤﺴﺏ ﺍﻟﻘﻭﻯ ﺍﻟﻤﻌﻤﻤﺔ ﺍﻟﺸﻜل )‪.(2.6‬‬

‫ﺍﻟﺸﻜﻞ )‪( 2.6‬‬


‫‪ - 24‬ﺍﺤﺴﺏ ﺩﻭﺭ ﺍﻻﻫﺘﺯﺍﺯﺍﺕ ﺍﻟﺼﻐﻴﺭﺓ ﻟﻠﻌﺎﺭﻀﺔ ﺍﻟﻤﺭﺴﻭﻡ ﻋﻠﻰ ﺍﻟﺸﻜل‬
‫)‪ (1.5‬ﻤﻥ ﺍﻟﻔﺼل ﺍﻟﺴﺎﺒﻕ‪.‬‬

‫‪- 73 -‬‬

‫)‪Create PDF files without this message by purchasing novaPDF printer (http://www.novapdf.com‬‬
- 74 -

Create PDF files without this message by purchasing novaPDF printer (http://www.novapdf.com)
‫‪‬‬

‫ﻣﻌﺎﺩﻻﺕ ﻫﺎﻣﻠﺘﻮﻥ‪ ،‬ﺃﻗﻮﺍﺱ ﺑﻮﺍﺻﻮﻥ‪،‬‬


‫ﻣﻌﺎﺩﻟﺔ ﻫﺎﻣﻠﺘﻮﻥ – ﺟﺎﻛﻮﺑﻲ‬

‫‪ - 17‬ﻣﻌﺎﺩﻻﺕ ﻫﺎﻣﻠﺘﻮﻥ )ﺍﳌﻌﺎﺩﻻﺕ ﺍﻟﻘﺎﻧﻮﻧﻴﺔ(‪:‬‬


‫ﺘﻠﻌﺏ ﻤﻌﺎﺩﻻﺕ ﻫﺎﻤﻠﺘﻭﻥ ﺩﻭﺭﺍﹰ ﻫﺎﻤﺎﹰ ﻓﻲ ﺤل ﻤﺴﺎﺌل ﺍﻟﻤﻴﻜﺎﻨﻴﻙ ﻻﻴﻘل ﻋﻥ‬
‫ﺩﻭﺭ ﻤﻌﺎﺩﻻﺕ ﻻﻏﺭﺍﻨﺞ ﺍﻟﺘﻲ ﺭﺃﻴﻨﺎﻫﺎ‪ .‬ﻓﻤﻥ ﺍﻟﻤﻌﻠﻭﻡ ﺃﻥ ﻤﻌﺎﺩﻻﺕ ﻻﻏﺭﺍﻨﺞ ﺍﻟﺘﻲ‬
‫ﻴﺤﻘﻘﻬﺎ ﺘﺎﺒﻊ ﻻﻏﺭﺍﻨﺞ ‪ L‬ﻟﻤﺠﻤﻭﻋﺔ ﻤﺎﺩﻴﺔ ﻤﺤﺎﻓﻅﺔ ﻫﻲ‪:‬‬
‫‪d L L‬‬
‫‪‬‬ ‫)‪ 0 ( j  1,2,...., n ) (3.1‬‬
‫‪dt q  q i‬‬
‫‪j‬‬

‫ﻭﻫﻲ ﻤﺠﻤﻭﻋﺔ ﻤﻌﺎﺩﻻﺕ ﺘﻔﺎﻀﻠﻴﺔ ﻤﻥ ﺍﻟﻤﺭﺘﺒﺔ ﺍﻟﺜﺎﻨﻴﺔ ﻋﺩﺩﻫﺎ ‪ ،n‬ﺤﻴﺙ‬


‫‪ q j , q j‬ﻫﻲ ﺍﻹﺤﺩﺍﺜﻴﺎﺕ ﺍﻟﻤﻌﻤﻤﺔ ﻭﺍﻟﺴﺭﻉ ﺍﻟﻤﻌﻤﻤﺔ ﻋﻠﻰ ﺍﻟﺘﺭﺘﻴﺏ‪.‬‬
‫‪‬‬

‫ﻴﺒﺩﻭ ﺃﻨﻪ ﻤﻥ ﺍﻟﻤﻤﻜﻥ ﺍﻟﺤﺼﻭل ﻋﻠﻰ ﻤﻌﺎﺩﻻﺕ ﺠﺩﻴﺩﺓ ﻋﺩﺩﻫﺎ ‪ 2n‬ﺒﺈﺩﺨﺎل‬


‫ﻤﺘﺤﻭل ﺠﺩﻴﺩ ‪ p j‬ﺒﺩﻻﹰ ﻤﻥ ‪ q j‬ﻴﺴﻤﻰ ﺍﻻﻨﺩﻓﺎﻉ ﺍﻟﻤﻌﻤﻡ‪ ،‬ﻁﺒﻘﺎﹰ ﻟﻠﻌﻼﻗﺔ‪:‬‬
‫‪‬‬

‫‪L‬‬
‫‪pj ‬‬ ‫)‪(3.2‬‬
‫‪q j‬‬

‫ﺜﻡ ﺍﻻﻨﺘﻘﺎل ﻤﻥ ﺍﻹﺤﺩﺍﺜﻴﺎﺕ ‪ q j , q j‬ﺇﻟﻰ ﺍﻹﺤﺩﺍﺜﻴﺎﺕ ‪ q j , p j‬ﺍﻋﺘﻤﺎﺩﺍﹰ ﻋﻠﻰ‬


‫‪‬‬

‫ﺍﻟﻤﻌﺎﺩﻻﺕ )‪ (3.2‬ﺍﻟﺘﻲ ﻋﺩﺩﻫﺎ ‪ ، n‬ﻭﻨﻤﻬﺩ ﺫﻟﻙ ﺭﻴﺎﻀﻴﺎﹰ ﺒﻤﺎﻴﻠﻲ‪:‬‬


‫ﻟﺘﻜﻥ ﺍﻟﺩﺍﻟﺔ )‪ . f (x, y‬ﺇﻥ ﺘﻔﺎﻀل ‪ f‬ﻫﻭ‪:‬‬
‫‪f‬‬ ‫‪f‬‬
‫‪df ‬‬ ‫‪dx  dy  u.dx  v.dy‬‬
‫‪x‬‬ ‫‪y‬‬
‫ﻓﺈﺫﺍ ﺃﺨﺫﻨﺎ ﺩﺍﻟﺔ ﺠﺩﻴﺩﺓ ‪ g‬ﻁﺒﻘﺎﹰ ﻟﻠﻌﻼﻗﺔ‪:‬‬
‫‪g  f  ux‬‬
‫ﺜﻡ ﻓﺎﻀﻠﻨﺎﻫﺎ‪:‬‬

‫‪- 75 -‬‬

‫)‪Create PDF files without this message by purchasing novaPDF printer (http://www.novapdf.com‬‬
‫‪dg  df  udx  xdu  udx  vdy  udx  xdu‬‬
‫ﻭﻤﻨﻪ‪:‬‬
‫)‪dg  vdy  xdu (3.3‬‬
‫ﺤﻴﺙ‪:‬‬
‫‪g‬‬ ‫‪g‬‬
‫‪x‬‬ ‫‪, v‬‬ ‫)‪(3.4‬‬
‫‪u‬‬ ‫‪y‬‬
‫ﻭﻫﻜﺫﺍ ﻨﻜﻭﻥ ﻗﺩ ﻋﺒﺭﻨﺎ ﻋﻥ ‪ g‬ﺒﺩﻻﻟﺔ ﺍﻟﻤﺘﺤﻭﻟﻴﻥ ﺍﻟﺠﺩﻴﺩﻴﻥ ‪ v‬ﻭ ‪.x‬‬
‫ﻭﻟﺯﻴﺎﺩﺓ ﺍﻻﻴﻀﺎﺡ ﻨﺫﻜﺭ ﻤﺜﺎﻻﹰ ﻤﻥ ﺍﻟﻔﻴﺯﻴﺎﺀ ﺍﻟﺤﺭﺍﺭﻴﺔ ﻋﻨﺩﻤﺎ ﻴﻌﺒﺭ ﻋﻥ‬
‫ﺍﻻﻨﺘﺎﻟﺒﻴﺔ ‪ X‬ﺒﺩﻻﻟﺔ ﺍﻻﻨﺘﺭﻭﺒﻴﺔ ‪ S‬ﻭﺍﻟﻀﻐﻁ ‪ p‬ﺒﺎﻟﻌﻼﻗﺔ‪:‬‬
‫‪dX  TdS  Vdp‬‬
‫ﺤﻴﺙ‪:‬‬
‫‪X‬‬ ‫‪X‬‬
‫‪T‬‬ ‫‪, V‬‬
‫‪S‬‬ ‫‪p‬‬
‫ﻏﻴﺭ ﺃﻨﻪ ﻗﺩ ﻴﻜﻭﻥ ﻤﻥ ﺍﻟﻤﻨﺎﺴﺏ ﺍﺴﺘﺨﺩﺍﻡ ﺩﺍﻟﺔ ﺃﺨﺭﻯ ﻫﻲ ﺍﻟﻁﺎﻗﺔ ﺍﻟﺤﺭﺓ ‪G‬‬
‫)ﺩﺍﻟﺔ ﺠﻴﺒﺱ( ﻁﺒﻘﺎﹰ ﻟﻠﻌﻼﻗﺔ‪:‬‬
‫‪G  X  TS‬‬
‫ﻭﺒﺎﻟﺘﺎﻟﻲ‪:‬‬
‫‪dG  SdT  Vdp‬‬
‫ﺃﻱ ﺃﻥ‪:‬‬
‫‪G‬‬ ‫‪G‬‬
‫‪S ‬‬ ‫‪, V‬‬ ‫)‪(3.5‬‬
‫‪T‬‬ ‫‪p‬‬
‫ﻟﻠﺤﺼﻭل ﻋﻠﻰ ﻤﻌﺎﺩﻻﺕ ﻫﺎﻤﻠﺘﻭﻥ ﻨﺴﺘﻔﻴﺩ ﻤﻥ ﺍﻟﺘﺤﻭﻴل ﺍﻟﺴﺎﺒﻕ‪ ،‬ﻓﻨﻌﺭﻑ ﺩﺍﻟﺔ‬
‫ﻫﺎﻤﻠﺘﻭﻥ )ﺘﺎﺒﻊ ﻫﺎﻤﻠﺘﻭﻥ( ﺒﺎﻟﻌﻼﻗﺔ‪:‬‬
‫‪L ‬‬
‫‪H (p j , q i , t )  ‬‬ ‫) ‪q  L (q i , q j , t‬‬
‫‪ j‬‬
‫‪q j‬‬

‫) ‪(3.6)   p jq j  L(q j , q j , t‬‬


‫ﻋﻠﻰ ﺃﻥ ﻨﺒﺩل ﻜل ﺍﻟﺴﺭﻉ ﺍﻟﻤﻌﻤﻤﺔ ‪ q j‬ﺒﺩﻻﻟﺔ ﺍﻻﻨﺩﻓﺎﻋﺎﺕ ﺍﻟﻤﻌﻤﻤﺔ ‪p j‬‬
‫‪‬‬

‫ﻭﺍﻹﺤﺩﺍﺜﻴﺎﺕ ﺍﻟﻤﻌﻤﻤﺔ ‪. q j‬‬

‫‪- 76 -‬‬

‫)‪Create PDF files without this message by purchasing novaPDF printer (http://www.novapdf.com‬‬
‫ﻭﺫﻟﻙ ﺒﺎﻻﺴﺘﻔﺎﺩﺓ ﻤﻥ ﻤﺠﻤﻭﻋﺔ ﺍﻟﻤﻌﺎﺩﻻﺕ )‪ (3.2‬ﺍﻟﺘﻲ ﻋﺩﺩﻫﺎ ‪ n‬ﻤﻌﺎﺩﻟﺔ‪ ،‬ﺜﻡ‬
‫ﻨﻔﺎﻀل ‪ H‬ﻓﻨﺠﺩ‪:‬‬
‫‪L‬‬ ‫‪L‬‬ ‫‪L‬‬
‫‪dH   p jdq j   q j dp j  ‬‬ ‫‪dq j  ‬‬ ‫‪dq j ‬‬ ‫)‪(3.7‬‬
‫‪j‬‬ ‫‪q j‬‬ ‫‪q j‬‬ ‫‪t‬‬

‫ﻭﻁﺒﻘﺎﹰ ﻟـ )‪ (3.2‬ﻴﻨﻌﺩﻡ ﺍﻟﺤﺩﺍﻥ ﺍﻷﻭل ﻭﺍﻟﺜﺎﻟﺙ ﻭﻤﻨﻪ‪:‬‬


‫‪L‬‬ ‫‪L‬‬
‫‪dH   q j dp j  ‬‬ ‫‪dq j ‬‬ ‫)‪(3.8‬‬
‫‪q j‬‬ ‫‪t‬‬

‫ﻭﻨﺤﺼل ﻤﻥ ﺠﻬﺔ ﺜﺎﻨﻴﺔ ﺒﻤﻔﺎﻀﻠﺔ ‪ ،H‬ﺒﺎﻋﺘﺒﺎﺭﻫﺎ ﺘﺎﺒﻌﺎﹰ ﻟﻤﺎ ﻴﺴﻤﻰ ﺍﻟﻤﺘﺤﻭﻻﺕ‬


‫ﺍﻟﻘﺎﻨﻭﻨﻴﺔ )ﺃﻭ ﺍﻟﺩﻴﻨﺎﻤﻴﻜﻴﺔ( ‪ p j‬ﻭ ‪ q i‬ﻋﻠﻰ ﺍﻟﻌﻼﻗﺔ‪:‬‬
‫‪H‬‬ ‫‪H‬‬ ‫‪H‬‬
‫‪dH  ‬‬ ‫‪dp j ‬‬ ‫‪dq j ‬‬ ‫)‪dt (3.9‬‬
‫‪P j‬‬ ‫‪q j‬‬ ‫‪t‬‬

‫ﺍﻟﻤﻭﺠﻭﺩﺓ ﻓﻲ )‪ (3.8‬ﺒﻘﻴﻤﺘﻬﺎ ﻤﻥ ﻤﻌﺎﺩﻻﺕ ﻻﻏﺭﺍﻨﺞ )‪(2.25‬‬ ‫ﻓﺈﺫﺍ ﺒﺩﻟﻨﺎ‬


‫‪L‬‬
‫‪q j‬‬

‫ﻭﻗﺎﺭﻨﺎ )‪ (3.8‬ﻤﻊ )‪ (3.9‬ﻨﺤﺼل ﻋﻠﻰ‬ ‫ﺍﻟﺘﻲ ﻴﻨﺘﺞ ﻤﻨﻬﺎ ﺃﻥ ‪ p j‬‬


‫‪L‬‬ ‫‪d L‬‬
‫‪‬‬
‫‪q j dt q j‬‬

‫ﻤﻌﺎﺩﻻﺕ ﻫﺎﻤﻠﺘﻭﻥ )ﺃﻭ ﺍﻟﻤﻌﺎﺩﻻﺕ ﺍﻟﻘﺎﻨﻭﻨﻴﺔ ﺍﻟﺘﺎﻟﻴﺔ ‪:(Canonic Equations‬‬


‫‪H‬‬ ‫‪H‬‬
‫‪q j ‬‬ ‫‪,  p j ‬‬ ‫) ‪, ( j  1, 2,..., n‬‬ ‫)‪(3.10‬‬
‫‪p j‬‬ ‫‪q j‬‬

‫ﺒﺎﻻﻀﺎﻓﺔ ﺇﻟﻰ ﺍﻟﻤﻌﺎﺩﻟﺔ ﺍﻟﻤﺘﻤﻤﺔ‪:‬‬


‫‪L‬‬ ‫‪H‬‬
‫‪‬‬ ‫)‪(3.11‬‬
‫‪t‬‬ ‫‪t‬‬

‫ﺇﻥ ﻋﺩﺩ ﻤﻌﺎﺩﻻﺕ ﻻﻏﺭﺍﻨﺞ ﻫﻭ ‪ n‬ﻤﻌﺎﺩﻟﺔ ﺘﻔﺎﻀﻠﻴﺔ ﻤﻥ ﺍﻟﻤﺭﺘﺒﺔ ﺍﻟﺜﺎﻨﻴﺔ ﺒﻴﻨﻤﺎ‬


‫ﻋﺩﺩ ﻤﻌﺎﺩﻻﺕ ﻫﺎﻤﻠﺘﻭﻥ ﻫﻭ ‪ 2n‬ﻤﻌﺎﺩﻟﺔ ﺘﻔﺎﻀﻠﻴﺔ ﻤﻥ ﺍﻟﻤﺭﺘﺒﺔ ﺍﻷﻭﻟﻰ ﻭﺒﺎﻟﺘﺎﻟﻲ ﻓﺈﻥ‬
‫ﺍﻻﻨﺘﻘﺎل ﻤﻥ ﺍﻟﻤﺠﻤﻭﻋﺔ ﺍﻷﻭﻟﻰ )ﺍﻟﻼﻏﺭﺍﻨﺠﻴﺔ( ﺇﻟﻰ ﺍﻟﺜﺎﻨﻴﺔ )ﺍﻟﻬﺎﻤﻠﺘﻭﻨﻴﺔ( ﻴﺸﺒﻪ‬
‫ﺘﺤﻭﻴل ﻤﻌﺎﺩﻟﺔ ﺘﻔﺎﻀﻠﻴﺔ ﻤﻥ ﺍﻟﻤﺭﺘﺒﺔ ﺍﻟﺜﺎﻨﻴﺔ )‪ f (y" , y' , x‬ﺇﻟﻰ ﻤﻌﺎﺩﻟﺘﻲ ﻤﻥ ﺍﻟﻤﺭﺘﺒﺔ‬
‫‪f ( z ' , z, x ) , y '  z‬‬ ‫ﺍﻷﻭﻟﻰ ﻫﻤﺎ‪:‬‬

‫‪- 77 -‬‬

‫)‪Create PDF files without this message by purchasing novaPDF printer (http://www.novapdf.com‬‬
‫‪ - 18‬ﻛﻴﻒ ﻧﺴﺘﻔﻴﺪ ﻣﻦ ﺍﳌﻌﺎﺩﻻﺕ )‪ (3.10‬ﰲ ﺣﻞ ﻣﺴﺎﺋﻞ ﺍﳌﻴﻜﺎﻧﻴﻚ؟‬

‫‪ -1‬ﻻﺒﺩ ﺃﻭﻻﹰ ﻤﻥ ﺤﺴﺎﺏ ﺘﺎﺒﻊ ﻻﻏﺭﺍﻨﺞ ﻟﻠﺠﻤﻠﺔ ﺍﻟﻤﺩﺭﻭﺴﺔ ﻭﻟﻬﺫﺍ ﻜﻤﺎ ﻨﻌﻠﻡ‬
‫ﻤﻥ ﺍﻟﻔﺼل ﺍﻟﺴﺎﺒﻕ‪ ،‬ﻴﺠﺏ ﺍﺨﺘﻴﺎﺭ ﺍﻹﺤﺩﺍﺜﻴﺎﺕ ﺍﻟﻤﻌﻤﻤﺔ ﺒﺸﻜل ﻤﻨﺎﺴﺏ‪ ،‬ﻭﺒﻌﺩ ﺫﻟﻙ‬
‫ﺘﺸﻜﻴل ﺘﺎﺒﻊ ﻫﺎﻤﻠﺘﻭﻥ ﻁﺒﻘﺎﹰ ﻟﻠﻤﻌﺎﺩﻟﺔ )‪ (3.6‬ﻭﻤﻥ ﺜﻡ ﻭﻀﻌﻪ ﺒﺸﻜﻠﻪ ﺍﻟﻨﻅﺎﻤﻲ ﺒﺩﻻﻟﺔ‬
‫ﺍﻟﻤﺘﺤﻭﻻﺕ ﺍﻟﻘﺎﻨﻭﻨﻴﺔ ‪. q j , p j‬‬

‫ﻭﺘﺸﻜﻴل ﺍﻟﻤﻌﺎﺩﻻﺕ )‪.(3.10‬‬ ‫‪ -2‬ﺤﺴﺎﺏ ﺍﻟﻤﺸﺘﻘﺎﺕ ﺍﻟﺠﺯﺌﻴﺔ‬


‫‪H H‬‬
‫‪,‬‬
‫‪p j q j‬‬

‫‪ -3‬ﺤل ﺍﻟﻤﻌﺎﺩﻻﺕ ﺍﻟﻨﺎﺘﺠﺔ ﻭﺍﻴﺠﺎﺩ ﻗﺎﻨﻭﻥ ﺍﻟﺤﺭﻜﺔ‪.‬‬

‫ﻣﺜﺎﻝ‪:‬‬

‫ﺍﻜﺘﺏ ﻤﻌﺎﺩﻻﺕ ﻫﺎﻤﻠﺘﻭﻥ ﻟﺠﺴﻴﻡ ﻁﻠﻴﻕ )ﻏﻴﺭ ﺨﺎﻀﻊ ﻷﻱ ﺍﺭﺘﺒﺎﻁﺎﺕ( ﻴﺘﺤﺭﻙ‬


‫ﻓﻲ ﻜﻤﻭﻥ )ﺠﻬﺩ( ﻤﻥ ﺍﻟﺸﻜل )‪. V (r, , ‬‬

‫ﺍﳊﻞ ‪:‬‬

‫ﺇﻥ ﺘﺎﺒﻊ ﻻﻏﺭﺍﻨﺞ ﻟﻬﺫﺍ ﺍﻟﺠﺴﻴﻡ ﻫﻭ‪:‬‬


‫‪1‬‬ ‫‪2‬‬ ‫‪2 2‬‬ ‫‪2‬‬ ‫‪2‬‬ ‫‪2‬‬
‫‪(3.12) L  T.V  m(r‬‬ ‫‪r ‬‬ ‫)‪ r Sin  )  V (r, , ‬‬
‫‪2‬‬
‫ﺃﻤﺎ ﺘﺎﺒﻊ )ﺩﺍﻟﺔ( ﻫﺎﻤﻠﺘﻭﻥ ﻟﻪ ﻓﻬﻭ‪:‬‬
‫‪3‬‬
‫‪L ‬‬ ‫‪L  L  L ‬‬
‫‪H‬‬ ‫‪q L‬‬ ‫‪r ‬‬ ‫‪ ‬‬ ‫‪ ‬‬
‫‪ j‬‬ ‫‪‬‬ ‫‪‬‬ ‫‪‬‬
‫‪j 1 q j‬‬ ‫‪r‬‬ ‫‪‬‬ ‫‪‬‬
‫‪1‬‬ ‫‪2‬‬ ‫‪2 2‬‬ ‫‪2‬‬ ‫‪2‬‬ ‫‪2‬‬
‫‪(3.13)  m(r‬‬ ‫‪r ‬‬ ‫‪ r Sin  )  V(r, , )  T  V‬‬
‫‪2‬‬

‫ﻻﻴﻤﻜﻥ ﺍﻻﺴﺘﻔﺎﺩﺓ ﻤﻥ ‪ ،H‬ﺒﻬﺫﻩ ﺍﻟﺼﻴﻐﺔ‪ ،‬ﻟﺤل ﺍﻟﻤﺴﺎﺌل ﻭﻻﺒﺩ ﻤﻥ ﺘﺤﻭﻴﻠﻪ ﺇﻟﻰ‬


‫ﺸﻜﻠﻪ ﺍﻟﻨﻅﺎﻤﻲ ﺒﺩﻻﻟﺔ ‪ p j , q j‬ﻭﻟﻬﺫﺍ ﻨﺴﺘﻔﻴﺩ ﻤﻥ ﺍﻟﻌﻼﻗﺎﺕ ﺍﻟﺘﺎﻟﻴﺔ‪:‬‬

‫‪- 78 -‬‬

‫)‪Create PDF files without this message by purchasing novaPDF printer (http://www.novapdf.com‬‬
L p
 mr   p r  r   r
 m
r

L p
 mr 2    p       (3.14)
  mr 2

L p
 mr 2 sin   p     
 mr 2 sin 2 
‫( ﻓﻨﺤﺼل ﻋﻠﻰ ﺘﺎﺒﻊ ﻫﺎﻤﻠﺘﻭﻥ ﺒﺸﻜﻠﻪ ﺍﻟﻨﻅﺎﻤﻲ‬3.13) ‫( ﻓﻲ‬3.14) ‫ﻨﺒﺩل‬
:‫ﺍﻟﺘﺎﻟﻲ‬
1  2 p 
2 p 2 
(3.15) H  pr    V (r, , )
2m  r 2
r 2
Sin 2 

 
‫( ﺤﻴﺙ ﻨﺠﺩ ﺍﻟﻤﺠﻤﻭﻋﺔ‬3.10) ‫ﻭﻴﺴﻬل ﺍﻵﻥ ﺘﺸﻜﻴل ﻤﻌﺎﺩﻻﺕ ﻫﺎﻤﻠﺘﻭﻥ‬
:‫ﺍﻷﻭﻟﻰ ﺍﻟﺘﺎﻟﻴﺔ‬
H p r 
  r   p r  mr 
p r m 


H pr  2  
(3.16) 
2
   p   mr  
p  mr 


H p  2 2 
    p   mr sin  
p  mr 2 sin 2  
:‫ﺃﻤﺎ ﺍﻟﻤﺠﻤﻭﻋﺔ ﺍﻟﺜﺎﻨﻴﺔ ﻓﻬﻲ‬

- 79 -

Create PDF files without this message by purchasing novaPDF printer (http://www.novapdf.com)
‫‪H‬‬ ‫‪‬‬ ‫‪1  2‬‬ ‫‪p 2  V‬‬ ‫‪‬‬
‫‪‬‬
‫‪ p r  ‬‬ ‫‪p ‬‬ ‫‪‬‬ ‫‪ pr‬‬
‫‪r‬‬ ‫‪mr 3 ‬‬ ‫‪sin 2   r‬‬ ‫‪‬‬
‫‪‬‬
‫‪‬‬
‫‪‬‬
‫‪2‬‬
‫‪H‬‬ ‫‪p‬‬ ‫‪‬‬ ‫‪cos‬‬ ‫‪‬‬ ‫‪V‬‬ ‫‪‬‬
‫)‪(3.17‬‬ ‫‪ p   ‬‬ ‫‪‬‬ ‫‪0‬‬ ‫‪‬‬
‫‪‬‬ ‫‪mr 2 sin 2  ‬‬ ‫‪‬‬
‫‪‬‬
‫‪‬‬
‫‪H‬‬ ‫‪‬‬ ‫‪V‬‬ ‫‪‬‬ ‫‪‬‬
‫‪ p  ‬‬ ‫‪ p‬‬
‫‪‬‬ ‫‪‬‬ ‫‪‬‬
‫‪‬‬
‫‪‬‬
‫ﻤﻥ ﺍﻟﻭﺍﻀﺢ ﺃﻥ ﻤﺠﻤﻭﻋﺘﻲ ﺍﻟﻤﻌﺎﺩﻻﺕ ﺍﻟﺘﻔﺎﻀﻠﻴﺔ ﺍﻟﺴﺕ )‪ (3.16‬ﻭ )‪(3.17‬‬
‫ﻫﻲ ﻤﻌﺎﺩﻻﺕ ﺤﺭﻜﺔ ﺃﻱ ﺠﺴﻴﻡ ﻓﻲ ﻜﻤﻭﻥ )‪ V (r, , ‬ﻤﻬﻤﺎ ﻜﺎﻥ ﺸﻜل ﻫﺫﺍ ﺍﻟﻜﻤﻭﻥ‪.‬‬
‫ﺃﻤﺎ ﻋﺩﺩ ﻤﻌﺎﺩﻻﺕ ﻻﻏﺭﺍﻨﺞ ﺍﻟﻤﻘﺎﺒﻠﺔ ﻓﻬﻲ ﺜﻼﺙ ﻓﻘﻁ ﻤﻥ ﺍﻟﻤﺭﺘﺒﺔ ﺍﻟﺜﺎﻨﻴﺔ‪ .‬ﻓﻲ ﺍﻟﺤﺎﻟﺔ‬
‫ﺍﻟﺨﺎﺼﺔ ﻋﻨﺩﻤﺎ ﻴﺘﺤﺭﻙ ﺍﻟﺠﺴﻴﻡ ﻓﻲ ﺤﻘل ﻗﻭﻯ ﻜﻤﻭﻨﻲ ﺫﻱ ﺘﻨﺎﻅﺭ ﻤﺭﻜﺯﻱ‪ ،‬ﺤﻴﺙ‬
‫‪‬‬
‫ﻴﻌﺒﺭ ﻋﻥ ﺍﻟﻜﻤﻭﻥ ﺒـ ) ‪ V( r‬ﻭﻟﻴﺱ ﺒـ ) ‪ ، V ( r‬ﻭﻴﻜﻭﻥ ‪ 0‬‬
‫‪V‬‬ ‫‪V‬‬
‫‪0,‬‬
‫‪‬‬ ‫‪‬‬
‫ﻨﺤﺼل ﻋﻨﺩﺌﺫ‪ ‬ﻤﺒﺎﺸﺭﺓﹰ ﻋﻠﻰ ﺘﻜﺎﻤل ﻟﻠﺤﺭﻜﺔ ﻤﻥ ﺍﻟﻤﻌﺎﺩﻟﺔ ﺍﻷﺨﻴﺭﺓ ﻤﻥ ﺍﻟﻤﺠﻤﻭﻋﺔ‬
‫ﺍﻟﺜﺎﻨﻴﺔ ﺤﻴﺙ ﻨﺠﺩ‪:‬‬
‫‪(3.18) p   mr 2 sin 2    Const.‬‬
‫ﻭﻴﻤﻜﻥ ﺍﻟﺤﺼﻭل ﻋﻠﻰ ﺍﻟﻨﺘﻴﺠﺔ ﻨﻔﺴﻬﺎ ﻤﻥ ﻤﻌﺎﺩﻻﺕ ﻻﻏﺭﺍﻨﺞ‪:‬‬
‫‪L‬‬ ‫‪L‬‬
‫‪0‬‬ ‫‪ Const.‬‬
‫‪‬‬ ‫'‬
‫‪‬‬
‫ﻗﺩ ﻴﻜﻭﻥ ﻤﻥ ﺍﻟﻤﻔﻴﺩ ﺍﻹﺸﺎﺭﺓ ﺇﻟﻰ ﺃﻥ ﻫﺫﻩ ﺍﻟﻤﺴﺄﻟﺔ ﺒﺎﻟﺫﺍﺕ ﺘﺼﺒﺢ ﺃﻜﺜﺭ ﺘﻨﺎﻅﺭ ﺍﹰ‬
‫ﻋﻨﺩﻤﺎ ﻨﺴﺘﺨﺩﻡ ﺍﻹﺤﺩﺍﺜﻴﺎﺕ ﺍﻟﺩﻴﻜﺎﺭﺘﻴﺔ ) ‪ ( x, y, z‬ﺒﺩﻻﹰ ﻤﻥ ﺍﻟﻜﺭﻭﻴﺔ ﻭﻋﻨﺩﺌﺫ‪ ‬ﻨﻀﻊ‬
‫ﺘﺎﺒﻊ ﻻﻏﺭﺍﻨﺞ ﺒﺎﻟﺼﻴﻐﺔ‪:‬‬
‫‪1‬‬ ‫‪2‬‬ ‫‪2‬‬ ‫‪2‬‬
‫‪(3.19) L  m(x‬‬ ‫‪y‬‬ ‫‪z‬‬ ‫)‪)  V (x , y, z‬‬
‫‪2‬‬
‫ﺃﻤﺎ ﺘﺎﺒﻊ ﻫﺎﻤﻠﺘﻭﻥ ﻓﻬﻭ‪:‬‬
‫‪1‬‬
‫‪(3.20) H ‬‬ ‫) ‪(p 2x  p 2y  p 2z )  V (x , y, z‬‬
‫‪2m‬‬

‫‪- 80 -‬‬

‫)‪Create PDF files without this message by purchasing novaPDF printer (http://www.novapdf.com‬‬
:‫ﻭﺘﺼﺒﺢ ﻤﻌﺎﺩﻻﺕ ﻫﺎﻤﻠﺘﻭﻥ‬
H p x
  x   p x  mx
p x m

H p y
  y   p y  my (3.21)
p y m

H p z
  z   p z  mz 
p z m
:‫ﻭﻜﺫﻟﻙ‬
H V V
   px  p x    Fx  mx 
x x x

H V V
(3.22)    py  py    Fy  my 
y y y

H V V
   pz  p z    Fz  mz  
z z z
‫ﻭﻤﻥ ﺍﻟﻭﺍﻀﺢ ﺃﻥ ﻫﺫﻩ ﺍﻟﻤﻌﺎﺩﻻﺕ ﺍﻷﺨﻴﺭﺓ ﻫﻲ ﻤﻌﺎﺩﻻﺕ ﻨﻴﻭﺘﻥ ﻨﻔﺴﻬﺎ ﻟﻬﺫﻩ‬
.‫ﺍﻟﺤﺭﻜﺔ‬
:‫ ﺧﻮﺍﺹ ﺗﺎﺑﻊ ﻫﺎﻣﻠﺘﻮﻥ‬- 19
‫( ﻓﻲ ﺤﺎﻟﺔ ﺍﻨﻌﺩﺍﻡ ﺍﻟﻘﻭﻯ ﻏﻴﺭ ﺍﻟﻤﺸﺘﻘﺔ‬2.25) ‫ ﻟﻨﺄﺨﺫ ﻤﻌﺎﺩﻻﺕ ﻻﻏﺭﺍﻨﺞ‬-‫ﺃ‬
:‫ ﻓﻨﺠﺩ‬j ‫ ﺜﻡ ﻨﺠﻤﻊ ﺒـ‬q j ‫( ﻭﻨﻀﺭﺏ ﻁﺭﻓﻲ ﺍﻟﻤﻌﺎﺩﻟﺔ ﺒـ‬Q j  0) ‫ﻤﻥ ﻜﻤﻭﻥ‬
 '

n n
d L L
 q . dt q    q q j (3.23)

j
j1 j j 1 j

:‫ﻓﻨﺠﺩ‬ ‫ﻨﻀﻴﻑ ﺇﻟﻰ ﺍﻟﻁﺭﻓﻴﻥ ﺍﻟﻤﻘﺩﺍﺭ‬


L 
 
qj
q j
n n n
 d L L  L  n L 
(3.24)  q j  qj   qj   q
dt q  j1 q  q  j
j 1 j j j 1 j j 1 q j

:‫ﻭﻤﻨﻪ‬

- 81 -

Create PDF files without this message by purchasing novaPDF printer (http://www.novapdf.com)
‫‪d‬‬ ‫‪ L   dL L‬‬
‫‪ qj  ‬‬
‫‪dt j  q j  dt t‬‬

‫ﺃﻭ‪:‬‬
‫‪‬‬ ‫‪‬‬
‫‪d‬‬ ‫‪L ‬‬ ‫‪ dH‬‬ ‫‪L V‬‬
‫‪(3.25)    q j  L  ‬‬ ‫‪‬‬ ‫‪‬‬
‫‪dt  q‬‬ ‫‪‬‬ ‫‪dt‬‬ ‫‪t‬‬ ‫‪t‬‬
‫‪‬‬ ‫‪j‬‬ ‫‪‬‬

‫ﺃﻱ ﺃﻥ ﻤﺸﺘﻕ ﺘﺎﺒﻊ ﻫﺎﻤﻠﺘﻭﻥ ﺍﻟﻜﻠﻲ ﺒﺎﻟﻨﺴﺒﺔ ﻟﻠﺯﻤﻥ ﻴﺴﺎﻭﻱ ﻤﺸﺘﻕ ﺘﺎﺒﻊ ﺍﻟﻜﻤﻭﻥ‬
‫ﺍﻟﺠﺯﺌﻲ ﺒﺎﻟﻨﺴﺒﺔ ﻟﻠﺯﻤﻥ ﺃﻴﻀﺎﹰ‪.‬‬

‫ﺏ‪ -‬ﻴﺴﻤﻰ ﺘﺎﺒﻊ ﻫﺎﻤﻠﺘﻭﻥ‪ ،‬ﺃﺤﻴﺎﻨﺎﹰ‪ ،‬ﺘﺎﺒﻊ ﺍﻟﻁﺎﻗﺔ ‪ E‬ﻓﻬﻭ ﻴﺴﺎﻭﻱ ‪ E‬ﻋﻨﺩﻤﺎ‬
‫ﻻﺘﺘﻌﻠﻕ ﺍﻻﺭﺘﺒﺎﻁﺎﺕ ﺒﺎﻟﺯﻤﻥ ﺤﻴﺙ ﻴﻜﻭﻥ ‪) T  T2‬ﺭﺍﺠﻊ ﺍﻟﻔﺼل ﺍﻟﺜﺎﻨﻲ( ﻭﺒﺎﻟﺘﺎﻟﻲ‬
‫ﻴﻜﻭﻥ‪:‬‬
‫‪L‬‬ ‫‪‬‬ ‫‪T‬‬ ‫‪‬‬
‫)‪(3.26‬‬ ‫‪‬‬ ‫‪‬‬
‫‪qj  ‬‬
‫‪‬‬
‫‪q j  2T‬‬
‫‪q j‬‬ ‫‪q j‬‬

‫ﻭﻋﻨﺩﺌﺫ‪ ‬ﻨﺠﺩ‪:‬‬
‫‪n‬‬
‫‪L ‬‬
‫‪H‬‬ ‫)‪q  L  2T  (T  V )  T  V  E (3.27‬‬
‫‪ j‬‬
‫‪j 1 q j‬‬

‫ﻭﻟﻜﻥ ‪ H‬ﻻﻴﻜﻭﻥ ﺘﻜﺎﻤﻼﹰ ﻟﻠﺤﺭﻜﺔ ﺇﻻ ﻋﻨﺩﻤﺎ ﻻﻴﺘﻌﻠﻕ ﺍﻟﻜﻤﻭﻥ ﺒﺼﻭﺭﺓ‬


‫ﺼﺭﻴﺤﺔ ﺒﺎﻟﺯﻤﻥ ﻁﺒﻘﺎﹰ ﻟـ )‪.(3.25‬‬

‫ﺝ‪ -‬ﺘﺨﺘﻠﻑ ﺃﺒﻌﺎﺩ ‪ p j‬ﺍﻟﻤﻭﺠﻭﺩ ﻓﻲ ﺘﺎﺒﻊ ﻫﺎﻤﻠﺘﻭﻥ ﺒﺎﺨﺘﻼﻑ ﺍﻹﺤﺩﺍﺜﻴﺎﺕ‬


‫ﺍﻟﻤﻌﻤﻤﺔ ﺍﻟﺘﻲ ﻨﺨﺘﺎﺭﻫﺎ؛ ﻓﺈﺫﺍ ﺃﺨﺫﻨﺎ ‪ q j‬ﺒﺤﻴﺙ ﻴﻜﻭﻥ ﻟﻬﺎ ﺃﺒﻌﺎﺩ ﻁﻭل ) ‪ x,y,z‬ﻤﺜﻼﹰ(‬
‫‪ MLT‬ﻭﻫﻲ ﺃﺒﻌﺎﺩ‬ ‫‪ LT‬ﺃﻤﺎ ﺃﺒﻌﺎﺩ ‪ p j‬ﻓﺘﻜﻭﻥ‬ ‫ﻓﺈﻥ ﺃﺒﻌﺎﺩ ‪ q j‬ﺍﻟﻤﻘﺎﺒﻠﺔ ﺴﺘﻜﻭﻥ‬
‫‪1‬‬ ‫‪1‬‬ ‫‪‬‬

‫ﻜﻤﻴﺔ ﺤﺭﻜﺔ )ﺍﻨﺩﻓﺎﻉ(‪ .‬ﺃﻤﺎ ﺇﺫﺍ ﺃﺨﺫﻨﺎ ‪ q j‬ﺩﻭﻥ ﺃﺒﻌﺎﺩ )ﺯﺍﻭﻴﺔ ﻤﺜﻼﹰ( ﻓﺈﻥ ﺃﺒﻌﺎﺩ ‪q j‬‬
‫‪‬‬

‫‪ ML T‬ﻭﻓﻲ ﺠﻤﻴﻊ ﺍﻟﺤﺎﻻﺕ ﻴﺠﺏ ﺃﻥ ﺘﻜﻭﻥ‬ ‫‪ p j‬ﻫﻲ‬ ‫‪ T‬ﻭﺃﺒﻌﺎﺩ‬ ‫ﺴﺘﻜﻭﻥ‬


‫‪2 1‬‬ ‫‪1‬‬

‫‪. ML T‬‬ ‫ﺃﺒﻌﺎﺩ ‪ p jq j‬ﺃﺒﻌﺎﺩ ﻁﺎﻗﺔ ﺃﻱ‬


‫‪2 2‬‬

‫‪- 82 -‬‬

‫)‪Create PDF files without this message by purchasing novaPDF printer (http://www.novapdf.com‬‬
‫‪ - 20‬ﺍﻹﺣﺪﺍﺛﻴﺎﺕ ﺍﻟﺪﻭﺭﻳﺔ – ﺗﺎﺑﻊ ﺭﺍﻭﺙ‪ ،‬ﻣﻌﺎﺩﻻﺕ ﺭﺍﻭﺙ‪:‬‬
‫‪Cyclic Coordinates, Routh Function, Routh Equations:‬‬
‫ﻟﻴﻜﻥ ﺘﺎﺒﻊ ﻻﻏﺭﺍﻨﺞ ﻟﺠﺴﻴﻡ )‪ M (r, ‬ﻴﺘﺤﺭﻙ ﻓﻲ ﺤﻘل ﻤﺭﻜﺯﻱ ﻤﺘﻨﺎﻅﺭ‪:‬‬
‫‪1‬‬ ‫‪2‬‬ ‫‪2 2‬‬
‫‪(3.28) L  T  V  m(r‬‬ ‫‪r ‬‬ ‫) ‪)  V(r‬‬
‫‪2‬‬
‫ﻭﻫﻭ ﻻﻴﺤﻭﻱ ﺍﻻﺤﺩﺍﺜﻲ ﺍﻟﻤﻌﻤﻡ ‪ ‬ﻭﺒﺎﻟﺘﺎﻟﻲ ﻴﻤﻜﻥ ﺃﻥ ﻨﻜﺘﺏ ﻤﺒﺎﺸﺭﺓﹰ ﺘﻜﺎﻤﻼﹰ‬
‫ﻟﻠﺤﺭﻜﺔ ﺍﻋﺘﻤﺎﺩﺍﹰ ﻋﻠﻰ ﻤﻌﺎﺩﻻﺕ ﻻﻏﺭﺍﻨﺞ ﻫﻭ ﺍﻟﺘﺎﻟﻲ‪:‬‬
‫‪L‬‬ ‫‪2 ‬‬
‫)‪ mr   Const. (3.29‬‬
‫‪‬‬
‫‪‬‬
‫ﻭﻴﻤﻜﻥ ﺘﺒﺩﻴل ‪ ‬ﺒﻘﻴﻤﺘﻬﺎ ﻓﻲ ﺘﺎﺒﻊ ﻻﻏﺭﺍﻨﺞ ﻓﻨﺤﺼل ﻋﻠﻰ ﻤﻌﺎﺩﻟﺔ ﻻﺘﺤﻭﻱ‬
‫‪‬‬

‫‪ ‬ﻭﻻ ‪ ‬ﻭﺒﺎﻟﺘﺎﻟﻲ ﻴﻜﻭﻥ ﻟﺩﻴﻨﺎ ﻤﻌﺎﺩﻟﺔ ﻭﺍﺤﺩﺓ ﻟﻠﺤﺭﻜﺔ ﺘﺘﻌﻠﻕ ﺒﺎﻻﺤﺩﺍﺜﻴﻲ ‪ ،r‬ﻭﻫﻜﺫﺍ‬
‫‪‬‬

‫ﻴﻨﺨﻔﺽ ﻋﺩﺩ ﻤﻌﺎﺩﻻﺕ ﻻﻏﺭﺍﻨﺞ‪.‬‬


‫ﻨﻌﻤ ﻡ ﺫﻟﻙ ﻓﻨﻔﺭﺽ ﺃﻥ ﺘﺎﺒﻊ ﻻﻏﺭﺍﻨﺞ ﻟﺠﻤﻠﺔ ﻤﺎﺩﻴﺔ ﻻﻴﺤﻭﻱ ﻋﺩﺩﺍﹰ ‪ s‬ﻤﻥ‬
‫ﺍﻹﺤﺩﺍﺜﻴﺎﺕ ﺍﻟﻤﻌﻤﻤﺔ ﺍﻟﺘﻲ ﻋﺩﺩﻫﺎ ‪ (s  n ) n‬ﻭﻋﻨﺩﺌﺫ‪ ‬ﻨﺤﺼل ﻋﻠﻰ ‪ S‬ﺘﻜﺎﻤﻼﹰ‬
‫ﻟﻠﺤﺭﻜﺔ ﻫﻲ‪:‬‬
‫‪L‬‬
‫)‪(3.30‬‬ ‫‪ Cj‬‬ ‫)‪( j  1, 2,..., s‬‬
‫‪q j‬‬

‫ﻓﺈﺫﺍ ﻋﻭﻀﻨﺎ ‪ s‬ﻤﻥ ﺍﻟﺴﺭﻉ ﺍﻟﻤﻌﻤﻤﺔ ﺍﻟﻤﺤﺴﻭﺒﺔ ﻤﻥ )‪ (3.30‬ﺒﺩﻻﻟﺔ )‪( n  s‬‬


‫ﻤﻥ ﺍﻟﺴﺭﻉ ﺍﻟﻤﻌﻤﻤﺔ ﺍﻟﺒﺎﻗﻴﺔ ﻓﺈﻨﻨﺎ ﻨﺤﺼل ﻋﻠﻰ ﺘﺎﺒﻊ ﻻﻏﺭﺍﻨﺞ ﻻﻴﺤﻭﻱ )‪ (n  s‬ﻤﻥ‬
‫ﺍﻟﺴﺭﻉ ﺍﻟﻤﻌﻤﻤﺔ ﻭﻤﺜﻠﻬﺎ ﻤﻥ ﺍﻹﺤﺩﺍﺜﻴﺎﺕ ﺍﻟﻤﻌﻤﻤﺔ ﻭﻟﻜﻨﻪ ﻴﺤﻭﻱ ‪ s‬ﺜﺎﺒﺘﺎﹰ ‪ C j‬ﻁﺒﻘﺎﹰ ﻟـ‬
‫)‪ (3.30‬ﻭﻫﻜﺫﺍ ﻨﺠﺩ ﺍﻟﺘﺎﺒﻊ ﺍﻟﺘﺎﻟﻲ‪:‬‬
‫)‪(3.31‬‬ ‫) ‪L  L(q s 1 , q s  2 ,..., q n , q s1 , q s 2 , q n , C1, C 2 ,..., C s‬‬
‫ﻭﻋﻨﺩﺌﺫ‪ ‬ﺘﺸﻜل ﺍﻟﺘﺎﺒﻊ ﺍﻟﺘﺎﻟﻲ ﺍﻟﻤﺴﻤﻰ ﺘﺎﺒﻊ ﺭﺍﻭﺙ‪:‬‬
‫‪S‬‬ ‫‪S‬‬
‫‪L ‬‬
‫‪R (q s 1 ,..., q n , q s1 ,..., q n , C1 ,..., C s )  L  ‬‬ ‫‪q j  L   C j q j‬‬
‫‪‬‬
‫‪j 1 q j‬‬ ‫‪j 1‬‬
‫)‪(3.32‬‬
‫ﻓﻨﺠﺩ‪:‬‬ ‫ﺜﻡ ﻨﻔﺎﻀل ﺍﻟﻁﺭﻓﻴﻥ ﺒﺎﻟﻨﺴﺒﺔ ﺇﻟﻰ‬
‫‪‬‬
‫‪C j,q j ,q j‬‬

‫‪- 83 -‬‬

‫)‪Create PDF files without this message by purchasing novaPDF printer (http://www.novapdf.com‬‬
‫‪n‬‬ ‫‪n‬‬ ‫‪S‬‬
‫‪R‬‬ ‫‪R‬‬ ‫‪R‬‬
‫‪R ‬‬ ‫‪ q q‬‬ ‫‪j ‬‬ ‫‪ q‬‬ ‫‪‬‬
‫‪q j  ‬‬ ‫‪C j ‬‬
‫‪jS 1‬‬ ‫‪j‬‬ ‫‪jS 1‬‬ ‫‪j‬‬ ‫‪j‬‬ ‫‪C j‬‬
‫‪n‬‬ ‫‪S‬‬ ‫‪n‬‬
‫‪L‬‬ ‫‪L ‬‬ ‫‪L  S‬‬ ‫‪S‬‬
‫‪‬‬ ‫)‪ q j  q  j  q  j  j j  C jq j (3.33‬‬
‫‪‬‬‫‪q‬‬ ‫‪‬‬ ‫‪‬‬‫‪q‬‬ ‫‪‬‬ ‫‪‬‬‫‪q‬‬ ‫‪‬‬ ‫‪‬‬‫‪C‬‬ ‫‪q‬‬ ‫‪‬‬
‫‪‬‬
‫‪jS1‬‬ ‫‪j‬‬ ‫‪j1‬‬ ‫‪j‬‬ ‫‪jS 1‬‬ ‫‪j‬‬ ‫‪j‬‬ ‫‪j1‬‬

‫ﻨﻼﺤﻅ ﺃﻥ ﺍﻟﺤﺩﻴﻥ ﺍﻟﺜﺎﻨﻲ ﻭﺍﻷﺨﻴﺭ ﻤﻥ ﺍﻟﻁﺭﻑ ﺍﻟﺜﺎﻨﻲ ﻤﺘﺴﺎﻭﻴﺎﻥ ﻭﻋﻨﺩﺌ ﺫ‪‬‬


‫ﻨﺤﺼل ﺒﺎﻟﻤﻘﺎﺭﻨﺔ ﺒﻴﻥ ﺃﻤﺜﺎل ‪ q j , q j , C j‬ﻓﻲ ﺍﻟﻁﺭﻓﻴﻥ ﻋﻠﻰ ﺍﻟﻤﻌﺎﺩﻻﺕ ﺍﻟﺘﺎﻟﻴﺔ‪:‬‬
‫‪R‬‬ ‫‪L‬‬ ‫‪R‬‬ ‫‪L‬‬ ‫‪R‬‬ ‫‪‬‬
‫‪‬‬ ‫‪,‬‬ ‫‪‬‬ ‫‪,‬‬ ‫‪ q j‬‬ ‫)‪(3.34‬‬
‫‪q j q j q‬‬ ‫‪‬‬ ‫‪‬‬ ‫‪C j‬‬
‫‪j‬‬ ‫‪q j‬‬

‫ﻭﺒﺎﻟﺘﻌﻭﻴﺽ ﻓﻲ ﻤﻌﺎﺩﻻﺕ ﻻﻏﺭﺍﻨﺞ )‪ (3.1‬ﻨﺤﺼل ﻋﻠﻰ ﻤﻌﺎﺩﻻﺕ ﺭﺍﻭﺙ‬


‫ﺍﻟﺘﺎﻟﻴﺔ‪:‬‬
‫‪d R R‬‬
‫‪‬‬ ‫)‪ 0 ( j  1, 2,..., n  s‬‬ ‫)‪(3.35‬‬
‫‪dt q  q j‬‬
‫‪j‬‬

‫ﻭﻫﻲ ﻋﺒﺎﺭﺓ ﻋﻥ )‪ (n  s‬ﻤﻌﺎﺩﻟﺔ ﺘﻔﺎﻀﻠﻴﺔ ﻤﻥ ﺍﻟﻤﺭﺘﺒﺔ ﺍﻟﺜﺎﻨﻴﺔ ﻜﺎﻓﻴﺔ ﻟﺤﺴﺎﺏ‬


‫)‪ (n  s‬ﻤﺠﻬﻭﻻﹰ ﻤﻥ ‪) q j‬ﻗﻭﺍﻨﻴﻥ ﺍﻟﺤﺭﻜﺔ(؛ ﺃﻱ ﺃﻥ ﻋﺩﺩ ﻫﺫﻩ ﺍﻟﻤﻌﺎﺩﻻﺕ ﻴﻨﻘﺹ ‪s‬‬
‫ﻤﻌﺎﺩﻟﺔ ﻋﻥ ﻤﻌﺎﺩﻻﺕ ﻻﻏﺭﺍﻨﺞ ﻤﻤﺎ ﻴﺠﻌﻠﻬﺎ ﺃﺴﻬل ﺍﺴﺘﺨﺩﺍﻤﺎﹰ‪.‬‬
‫ﻭﻹﻴﺠﺎﺩ ﻗﻭﺍﻨﻴﻥ ﺍﻟﺤﺭﻜﺔ ﺍﻟﺨﺎﺼﺔ ﺒﺎﻹﺤﺩﺍﺜﻴﺎﺕ ﺍﻟﻤﻌﻤﻤﺔ ﻏﻴﺭ ﺍﻟﻤﻭﺠﻭﺩﺓ ﻓﻲ‬
‫ﺘﺎﺒﻊ ﻻﻏﺭﺍﻨﺞ ﻭﺍﻟﺘﻲ ﺘﺴﻤﻰ ﺍﻹﺤﺩﺍﺜﻴﺎﺕ ﺍﻟﺩﻭﺭﻴﺔ ﻨﺤﺼل ﻤﻥ )‪ (3.34‬ﻤﺒﺎﺸﺭﺓﹰ‬
‫ﺒﺎﺴﺘﻜﻤﺎﻟﻬﺎ ﻋﻠﻰ ﻗﻭﺍﻨﻴﻥ ﺍﻟﺤﺭﻜﺔ ﺍﻟﺒﺎﻗﻴﺔ ﺍﻟﻤﻁﻠﻭﺒﺔ‪:‬‬
‫‪R‬‬
‫‪q j  ‬‬ ‫‪dt‬‬ ‫)‪( j  1, 2, ..., s‬‬ ‫)‪(3.36‬‬
‫‪C j‬‬

‫ﻭﻫﻲ ﻋﺒﺎﺭﺓ ﻋﻥ ‪ s‬ﻤﻌﺎﺩﻟﺔ ﻜﺎﻓﻴﺔ ﻟﺤﺴﺎﺏ ‪ s‬ﻤﻥ ﺍﻹﺤﺩﺍﺜﻴﺎﺕ ﺍﻟﻤﻌﻤﻤﺔ ﺍﻟﺒﺎﻗﻴﺔ‬


‫ﺒﺩﻻﻟﺔ ﺍﻟﺯﻤﻥ‪.‬‬
‫ﻨﻼﺤﻅ ﺃﻥ ﻤﻌﺎﺩﻻﺕ ﺭﺍﻭﺙ ﺘﺸﺒﻪ ﻤﻌﺎﺩﻻﺕ ﻻﻏﺭﺍﻨﺞ ﺘﻤﺎﻤﺎﹰ‪ ،‬ﻭﻟﻜﻥ ﻫل ﻴﻤﺘﺩ‬
‫ﻫﺫﺍ ﺍﻟﺘﺸﺎﺒﻪ ﻟﻴﺸﻤل ﺍﻟﻌﻼﻗﺎﺕ ﺍﻟﺘﻲ ﻴﺤﻘﻘﻬﺎ ﻜل ﻤﻥ ﺍﻟﺩﺍﻟﺘﻴﻥ؟‬
‫ﻟﻨﻀﺭﺏ ﻁﺭﻓﻲ ﺍﻟﻤﻌﺎﺩﻻﺕ )‪ (3.35‬ﺒـ ‪ q j‬ﺜﻡ ﻨﺠﻤﻊ ﺒـ ‪ j‬ﻓﻨﺠﺩ‪:‬‬
‫‪‬‬

‫‪n‬‬ ‫‪n‬‬
‫‪d R‬‬ ‫‪R‬‬
‫‪‬‬ ‫‪q j‬‬ ‫‪  q j‬‬ ‫)‪(3.37‬‬
‫‪‬‬
‫‪dt q j j s 1 q j‬‬
‫‪j  s 1‬‬

‫‪- 84 -‬‬

‫)‪Create PDF files without this message by purchasing novaPDF printer (http://www.novapdf.com‬‬
‫‪n‬‬
‫ﻜﻤﺎ ﻓﻌﻠﻨﺎ ﻋﻨﺩ ﺍﻟﺤﺼﻭل‬ ‫ﺜﻡ ﻨﻀﻴﻑ ﺇﻟﻰ ﺍﻟﻁﺭﻓﻴﻥ ﺍﻟﻤﻘﺩﺍﺭ‬
‫‪R‬‬
‫‪ q j ‬‬
‫‪j  s 1‬‬ ‫‪q j‬‬

‫ﻋﻠﻰ ﺍﻟﻤﻌﺎﺩﻟﺔ )‪ (3.25‬ﻓﻨﺠﺩ‪:‬‬


‫‪n‬‬ ‫‪n‬‬ ‫‪n‬‬ ‫‪n‬‬
‫‪d R‬‬ ‫‪R‬‬ ‫‪R‬‬ ‫‪R‬‬
‫‪‬‬ ‫‪q j‬‬ ‫‪  q j ‬‬ ‫‪  q j‬‬ ‫‪  q j ‬‬ ‫)‪(3.38‬‬
‫‪j s 1‬‬ ‫‪dt q j j  s 1 q j j  s 1 q‬‬ ‫‪j s 1‬‬ ‫‪‬‬ ‫‪q‬‬ ‫‪‬‬
‫‪j‬‬
‫‪j‬‬

‫ﻭﻤﻨﻪ‪:‬‬
‫‪d n R  dR R‬‬
‫‪‬‬
‫‪dt j  s 1 q j‬‬
‫‪qj ‬‬
‫‪dt‬‬
‫‪‬‬
‫‪t‬‬

‫ﺃﻭ‪:‬‬
‫‪d  n R ‬‬ ‫‪‬‬ ‫‪R‬‬
‫)‪(3.39‬‬ ‫‪‬‬ ‫‪q j  R  ‬‬
‫‪‬‬
‫‪dt  j  s 1 q j‬‬ ‫‪‬‬ ‫‪t‬‬
‫‪‬‬ ‫‪‬‬
‫ﻭﻓﻲ ﺍﻟﺤﺎﻟﺔ ﺍﻟﺨﺎﺼﺔ ﻋﻨﺩﻤﺎ ﻻﻴﺤﻭﻱ ﺘﺎﺒﻊ ﺭﺍﻭﺙ ﺍﻟﺯﻤﻥ ﺒﺸﻜل ﺼﺭﻴﺢ‬
‫ﻨﺤﺼل ﻤﺒﺎﺸﺭﺓﹰ ﻤﻥ )‪ (3.39‬ﻋﻠﻰ ﺘﻜﺎﻤل ﺤﺭﻜﺔ ﻴﻤﻜﻥ ﺍﺴﺘﺨﺩﺍﻤﻪ ﻓﻲ ﺤل ﺍﻟﻤﺴﺎﺌل‬
‫ﻫﻭ ﺍﻟﺘﺎﻟﻲ‪:‬‬
‫‪n‬‬
‫‪R ‬‬
‫)‪(3.40‬‬ ‫‪‬‬ ‫‪q  R  Const.‬‬
‫‪ j‬‬
‫‪j  s 1‬‬ ‫‪‬‬‫‪q‬‬ ‫‪j‬‬

‫‪ - 21‬ﺃﻗﻮﺍﺱ ﺑﻮﺍﺻﻮﻥ ‪Poisson Brackets :‬‬


‫ﻟﻴﺱ ﻤﻥ ﺍﻟﺼﻌﺏ ﻜﺘﺎﺒﺔ ﻤﻌﺎﺩﻻﺕ ﺍﻟﺤﺭﻜﺔ ﺒﻁﺭﻕ ﻤﺨﺘﻠﻔﺔ‪ ،‬ﻭﻟﻜﻥ ﻻﺒﺩ‬
‫ﻟﻠﺤﺼﻭل ﻋﻠﻰ ﻗﺎﻨﻭﻥ ﺍﻟﺤﺭﻜﺔ ﻤﻥ ﺤل ﻫﺫﻩ ﺍﻟﻤﻌﺎﺩﻻﺕ ﻭﺤﺴﺎﺏ ﺍﻹﺤﺩﺍﺜﻴﺎﺕ ‪q j‬‬

‫ﻤﺒﺎﺸﺭﺓﹰ ﺒﺩﻻﻟﺔ ﺍﻟﺯﻤﻥ ﻭﺜﻭﺍﺒﺕ ﺍﻟﺤﺭﻜﺔ‪ .‬ﻭﻗﺩ ﺭﺃﻴﻨﺎ ﺴﺎﺒﻘﺎﹰ ﺃﻨﻪ ﻴﻤﻜﻥ ﺍﻻﺴﺘﻔﺎﺩﺓ ﻤﻥ‬
‫ﺒﻌﺽ ﺍﻟﺘﻜﺎﻤﻼﺕ ﺍﻷﻭﻟﻴﺔ ﻜﺘﻜﺎﻤل ﺍﻟﻁﺎﻗﺔ ﻭﺘﻜﺎﻤل ﺍﻟﻌﺯﻡ ﺍﻟﺤﺭﻜﻲ‪ .‬ﻭﻤﻥ ﺍﻟﻤﻔﻴﺩ ﺸﺭﺡ‬
‫ﻁﺭﻴﻘﺔ ﻻﻴﺠﺎﺩ ﺒﻌﺽ ﻫﺫﻩ ﺍﻟﺘﻜﺎﻤﻼﺕ ﻤﺒﺎﺸﺭﺓﹰ ﻫﻲ ﻁﺭﻴﻘﺔ ﺒﻭﺍﺼﻭﻥ‪.‬‬
‫ﻟﺘﻜﻥ ﺩﺍﻟﺔ ﺍﻟﻤﺘﺤﻭﻻﺕ ﺍﻟﻘﺎﻨﻭﻨﻴﺔ ﻭﺍﻟﺯﻤﻥ ) ‪ f (p j , q i , t‬ﻭﻟﻨﺤﺴﺏ ﺍﻟﻤﺸﺘﻕ‬
‫ﺍﻟﺯﻤﻨﻲ ﺍﻟﻜﻠﻲ ﻟﻬﺎ‪:‬‬
‫‪n ‬‬
‫‪df‬‬ ‫‪f  f   f‬‬
‫)‪(3.41‬‬ ‫‪ ‬‬ ‫‪pj ‬‬ ‫‪qj ‬‬
‫‪dt j1 P j‬‬ ‫‪q j  t‬‬

‫‪- 85 -‬‬

‫)‪Create PDF files without this message by purchasing novaPDF printer (http://www.novapdf.com‬‬
‫ﻭﺒﺎﻻﺴﺘﻔﺎﺩﺓ ﻤﻥ ﻤﻌﺎﺩﻻﺕ ﻫﺎﻤﻠﺘﻭﻥ )‪ (3.10‬ﻴﻤﻜﻥ ﻭﻀﻊ ﺍﻟﻌﻼﻗﺔ ﺍﻟﺴﺎﺒﻘﺔ‬
‫ﺒﺎﻟﺸﻜل‪:‬‬
‫‪n ‬‬
‫‪df‬‬ ‫‪f H‬‬ ‫‪f H  f‬‬ ‫‪f‬‬
‫)‪(3.42‬‬ ‫‪ ‬‬ ‫‪‬‬ ‫‪‬‬ ‫‪ H , f  ‬‬
‫‪‬‬ ‫‪‬‬
‫‪dt j 1 q j p j p j q j  t‬‬ ‫‪t‬‬

‫ﺤﻴﺙ ﺭﻤﺯﻨﺎ ﺒـ ‪ H, f ‬ﻟﻠﻤﻘﺩﺍﺭ‪:‬‬


‫‪n‬‬ ‫‪‬‬ ‫‪H f ‬‬
‫‪H, f     H‬‬ ‫‪f‬‬
‫‪‬‬ ‫)‪(3.43‬‬
‫‪j 1‬‬
‫‪p j q j q j p j ‬‬

‫ﺍﻟﺫﻱ ﻴﺴﻤﻰ ﺃﻗﻭﺍﺱ ﺒﻭﺍﺼﻭﻥ‪.‬‬


‫‪ ‬ﻓﺈﻨﻨﺎ ﻨﺠﺩ ﻤﻥ )‪(3.43‬‬ ‫ﺇﺫﺍ ﻟﻡ ﺘﺤﻭﹺ ﺍﻟﺩﺍﻟﺔ ‪ f‬ﺍﻟﺯﻤﻥ ﺒﺸﻜل ﺼﺭﻴﺢ ‪ 0 ‬‬
‫‪f‬‬ ‫‪‬‬
‫‪ t‬‬ ‫‪‬‬
‫ﺍﻟﻌﻼﻗﺔ‪:‬‬
‫‪df‬‬
‫‪ H, f ‬‬ ‫)‪(3.44‬‬
‫‪dt‬‬
‫ﻭﺒﺎﻟﺘﺎﻟﻲ ﻴﺘﺴﺎﻭﻯ ﺍﻟﻤﺸﺘﻕ ﺍﻟﻜﻠﻲ ﻟﻠﺩﺍﻟﺔ ‪ f‬ﻤﻊ ﺃﻗﻭﺍﺱ ﺒﻭﺍﺼﻭﻥ ﺍﻟﻤﻘﺎﺒﻠﺔ ﻭﺃﻥ‬
‫ﺍﻨﻌﺩﺍﻡ ﻫﺫﻩ ﺍﻷﻗﻭﺍﺱ ﻴﻌﻨﻲ‪ ،‬ﺒﻜل ﺒﺴﺎﻁﺔ‪ ،‬ﺃﻥ ‪ f‬ﻫﻭ ﺒﺎﻟﻀﺒﻁ ﺘﻜﺎﻤل ﺍﻟﺤﺭﻜﺔ‪ ،‬ﻭﻋﻨﺩﺌ ﺫ‪‬‬
‫ﻨﺼل ﺇﻟﻰ ﺍﻟﻨﺘﻴﺠﺔ ﺍﻟﺘﺎﻟﻴﺔ‪:‬‬
‫ﺘﻜﻭﻥ ﺩﺍﻟﺔ ﻤﺎ ) ‪ f (p j , q j‬ﺘﻜﺎﻤﻼﹰ ﻟﻠﺤﺭﻜﺔ ﺇﺫﺍ ﺍﻨﻌﺩﻤﺕ ﺃﻗﻭﺍﺱ ﺒﻭﺍﺼﻭﻥ‬

‫ﺍﻟﻤﺸﻜﻠﺔ ﻤﻨﻬﺎ ﻭﻤﻥ ﺩﺍﻟﺔ ﻫﺎﻤﻠﺘﻭﻥ ‪. H , f   0‬‬


‫ﻟﻨﺄﺧﺬ ﺑﻌﺾ ﺍﻷﻣﺜﻠﺔ‪:‬‬
‫‪ -1‬ﻤﻥ ﺍﻟﻤﻌﻠﻭﻡ ﺃﻥ ﺍﻨﺩﻓﺎﻉ ﻨﻘﻁﺔ ﻤﺎﺩﻴﺔ ﻁﻠﻴﻘﺔ ﻴﻜﻭﻥ ﺜﺎﺒﺘﺎﹰ )ﺘﻜﺎﻤﻼﹰ ﻟﻠﺤﺭﻜﺔ(‬
‫ﺇﺫﺍ ﺍﻨﻌﺩﻤﺕ ﺍﻟﻘﻭﻯ ﺍﻟﻤﺅﺜﺭﺓ ﻋﻠﻴﻬﺎ‪ .‬ﻓﻬل ﻴﻤﻜﻥ ﺍﻟﻭﺼﻭل ﺇﻟﻰ ﺍﻟﻨﺘﻴﺠﺔ ﻨﻔﺴﻬﺎ ﻋﻥ‬
‫ﻁﺭﻴﻕ ﺃﻗﻭﺍﺱ ﺒﻭﺍﺼﻭﻥ؟‬
‫‪‬‬ ‫‪‬‬
‫ﻟﻺﺠﺎﺒﺔ ﻋﻥ ﻫﺫﺍ ﺍﻟﺴﺅﺍل ﻨﺤﺴﺏ ﺃﻗﻭﺍﺱ ﺒﻭﺍﺼﻭﻥ ‪ H , p ‬ﻓﻨﺠﺩ‪:‬‬
‫‪‬‬ ‫‪‬‬
‫‪  ‬‬ ‫‪ ‬‬ ‫‪ ‬‬ ‫‪‬‬
‫)‪(3.45‬‬ ‫‪‬‬ ‫‪H‬‬ ‫‪,‬‬ ‫‪p‬‬ ‫‪‬‬
‫‪ ‬‬ ‫‪H‬‬ ‫‪,‬‬ ‫‪p‬‬ ‫‪i‬‬ ‫‪‬‬
‫‪x  ‬‬ ‫‪H‬‬ ‫‪,‬‬ ‫‪p‬‬ ‫‪j‬‬
‫‪y  ‬‬‫‪‬‬ ‫‪H‬‬ ‫‪,‬‬ ‫‪p‬‬ ‫‪z k‬‬
‫‪‬‬ ‫‪ ‬‬ ‫‪ ‬‬ ‫‪ ‬‬ ‫‪‬‬
‫ﻭﺒﺎﻟﺤﺴﺎﺏ ﻨﺠﺩ‪:‬‬

‫‪- 86 -‬‬

‫)‪Create PDF files without this message by purchasing novaPDF printer (http://www.novapdf.com‬‬
 p x H p x 
H, p x     H 


H

V
 Fx
 p j q j q j p j  x x

:‫ﺃﻱ ﺃﻥ‬
     
 H , p   Fx i  Fy j  Fz k  F
 
 
‫ ﻭﻫﻲ‬p  Const. ‫ ﻭﺒﺎﻟﺘﺎﻟﻲ‬H, p  0 ‫ ﻴﻜﻭﻥ‬F ‫ﻓﺈﺫﺍ ﺍﻨﻌﺩﻤﺕ ﺍﻟﻘﻭﺓ‬
.‫ﺍﻟﻨﺘﻴﺠﺔ ﺍﻟﻤﻌﺭﻭﻓﺔ ﺴﺎﺒﻘﺎﹰ‬


‫ ﻫﻭ ﺘﻜﺎﻤل ﻟﻠﺤﺭﻜﺔ‬L ‫ ﻭﺒﺎﻟﻁﺭﻴﻘﺔ ﻨﻔﺴﻬﺎ ﻨﺒﺭﻫﻥ ﺃﻥ ﺍﻟﻌﺯﻡ ﺍﻟﺤﺭﻜﻲ‬-‫ﺏ‬
:‫[ ﺍﻟﺘﺎﻟﻴﺔ‬H,L] ‫ﻋﻨﺩﻤﺎ ﻴﺘﺤﺭﻙ ﺍﻟﺠﺴﻴﻡ ﻓﻲ ﺤﻘل ﻤﺭﻜﺯﻱ ﻤﺘﻨﺎﻅﺭ ﻭﺫﻟﻙ ﺒﺤﺴﺎﺏ‬
       
 H , L 
  H , L i 
x   H , L j
y   H , L z k (3.46)
       
:‫ﺤﻴﺙ ﻨﺠﺩ‬
 L x H L x 
H, L x     H 

( j  1, 2, 3  x, y, z )
j 
p j q j q j p j 
:‫ﻓﺈﺫﺍ ﺍﺴﺘﻔﺩﻨﺎ ﻤﻥ ﺍﻟﺼﻴﻎ ﺍﻟﻤﻌﺭﻭﻓﺔ ﻟﻤﺭﻜﺒﺎﺕ ﺍﻟﻌﺯﻡ ﺍﻟﺤﺭﻜﻲ ﻨﺠﺩ ﺃﺨﻴﺭﺍﹰ‬
H, L x   V z  V y  (rxV) x  (rxF) x  M x
y z
‫ﻭﻜﺫﻟﻙ ﻨﺠﺩ‬:
H, L z   M z , H, L y   M y
:‫ﺃﻱ ﺃﻥ‬
  
H , L   M (3.47)
 

‫ ﻭﻴﻨﻌﺩﻡ ﻫﺫﺍ ﺍﻟﻌﺯﻡ ﻋﻨﺩﻤﺎ‬،‫ ﻫﻭ ﻋﺯﻡ ﺍﻟﻘﻭﺓ ﺍﻟﻤﺅﺜﺭﺓ ﻋﻠﻰ ﺍﻟﺠﺴﻴﻡ‬M ‫ﺤﻴﺙ‬
 
‫ ﻭﺒﺎﻟﺘﺎﻟﻲ‬H , L  ‫ ﻭﻓﻲ ﻫﺫﻩ ﺍﻟﺤﺎﻟﺔ ﺘﻨﻌﺩﻡ ﺃﻗﻭﺍﺱ ﺒﻭﺍﺼﻭﻥ‬،‫ﺘﻜﻭﻥ ﺍﻟﻘﻭﺓ ﻤﺭﻜﺯﻴﺔ‬
 

- 87 -

Create PDF files without this message by purchasing novaPDF printer (http://www.novapdf.com)
‫ﻓﺎﻟﻌﺯﻡ ﺍﻟﺤﺭﻜﻲ ﻴﻜﻭﻥ ﺜﺎﺒﺘﺎﹰ ) ﺘﻜﺎﻤﻼﹰ ﻟﻠﺤﺭﻜﺔ(‪ ،‬ﻭﻫﻲ ﻨﺘﻴﺠﺔ ﻤﻌﺭﻭﻓﺔ ﺴﺎﺒﻘﺎﹰ ﻤﻥ‬
‫ﻤﻴﻜﺎﻨﻴﻙ ﻨﻴﻭﺘﻥ‪.‬‬
‫ﻴﺒﺩﻭ ﺃﻨﻪ ﻤﻥ ﺍﻟﻤﻤﻜﻥ ﻭﻀﻊ ﺍﻟﻤﻌﺎﺩﻻﺕ ﺍﻟﻘﺎﻨﻭﻨﻴﺔ ﺒﺼﻴﻐﺔ ﺃﺨﺭﻯ ﻋﻠﻰ ﻀﻭﺀ‬
‫ﺃﻗﻭﺍﺱ ﺒﻭﺍﺼﻭﻥ‪ ،‬ﻭﻟﻬﺫﺍ ﻨﺤﺴﺏ ‪ H, p k ‬ﻭ ‪ H, q k ‬ﻓﻨﺠﺩ‪:‬‬
‫‪ H p k H p k  ‬‬
‫‪H, p k    ‬‬ ‫‪‬‬ ‫‪    p j  kj  p k‬‬
‫‪j ‬‬
‫‪p j q j q j p j  ‬‬ ‫‪j‬‬
‫‪‬‬
‫)‪(3.48‬‬
‫‪‬‬ ‫‪q k H q k ‬‬
‫‪H, q k     H‬‬ ‫‪‬‬
‫‪‬‬
‫‪‬‬ ‫‪‬‬
‫‪   q j  kj  q k‬‬
‫‪j ‬‬ ‫‪‬‬‫‪p‬‬ ‫‪j‬‬ ‫‪‬‬‫‪q‬‬ ‫‪j‬‬ ‫‪‬‬‫‪q‬‬ ‫‪j‬‬ ‫‪‬‬‫‪p‬‬ ‫‪j ‬‬ ‫‪j‬‬
‫‪‬‬
‫ﻭﻫﻲ ﺍﻟﺼﻴﻐﺔ ﺍﻟﻤﻁﻠﻭﺒﺔ‪.‬‬
‫ﻨﻌﻤﻡ ﺘﻌﺭﻴﻑ ﺃﻗﻭﺍﺱ ﺒﻭﺍﺼﻭﻥ )‪ (3.43‬ﻋﻠﻰ ﺃﻱ ﺩﺍﻟﺘﻴﻥ ‪ f1‬و ‪ f 2‬ﻟﻠﻤﺘﺤﻭﻻﺕ‬
‫ﺍﻟﻘﺎﻨﻭﻨﻴﺔ ‪ p j‬ﻭ ‪ q j‬ﺒﺎﻟﻌﻼﻗﺔ‪:‬‬
‫‪ f1 f 2 f1 f 2 ‬‬
‫‪f1, f 2    ‬‬ ‫‪‬‬ ‫‪‬‬
‫‪‬‬
‫)‪(3.49‬‬
‫‪j ‬‬
‫‪‬‬‫‪p‬‬ ‫‪j‬‬ ‫‪‬‬‫‪q‬‬ ‫‪j‬‬ ‫‪‬‬‫‪q‬‬ ‫‪j‬‬ ‫‪‬‬‫‪p‬‬ ‫‪j‬‬

‫ﻭﻤﻥ ﺍﻟﺴﻬل ﺍﻟﺘﺄﻜﺩ ﻤﺒﺎﺸﺭﺓﹰ ﻤﻤﺎﻴﻠﻲ‪:‬‬


‫‪ -1‬ﺃﻗﻭﺍﺱ ﺒﻭﺍﺼﻭﻥ ﻻﺘﺒﺎﺩﻟﻴﺔ‪:‬‬
‫‪f1 , f 2   f 2 , f1   f1 , f 2 ‬‬ ‫)‪(3.50‬‬
‫‪ -2‬ﺘﺤﻘﻕ ﺨﺎﺼﺔ ﺍﻟﺠﻤﻊ‪:‬‬
‫‪f1  f 2 , f 3   f1 , f 3   f 2 , f 3 ‬‬ ‫)‪(3.51‬‬
‫‪ -3‬ﺘﺤﻘﻕ ﺍﻟﺨﺎﺼﺔ ﺍﻟﺘﺎﻟﻴﺔ‪:‬‬
‫‪f1f 2 , f 3   f1 f 2 , f 3   f 2 f1 , f 3 ‬‬ ‫)‪(3.52‬‬
‫‪ -4‬ﺘﺤﻘﻕ ﻤﻁﺎﺒﻘﺔ ﺠﺎﻜﻭﺒﻲ ﺍﻟﺘﺎﻟﻴﺔ‪:‬‬
‫‪f1 , f 2 , f 3   f 2 , f 3 , f1   f 3 , f1 , f 2   0‬‬ ‫)‪(3.53‬‬
‫‪ -5‬ﻨﺤﺴﺏ ﻤﺸﺘﻕ ﺃﻗﻭﺍﺱ ﺒﻭﺍﺼﻭﻥ ﺒﺎﻟﻨﺴﺒﺔ ﻟﻠﺯﻤﻥ ﺒﺎﻟﻌﻼﻗﺔ‪:‬‬
‫‪d‬‬ ‫‪ df‬‬ ‫‪  df ‬‬
‫‪dt‬‬
‫‪‬‬‫‪f1, f 2    1 , f 2   f1 , 2 ‬‬ ‫)‪(3.54‬‬
‫‪ dt‬‬ ‫‪ ‬‬ ‫‪dt ‬‬

‫‪- 88 -‬‬

‫)‪Create PDF files without this message by purchasing novaPDF printer (http://www.novapdf.com‬‬
‫ﻤﻥ ﺍﻟﻤﻔﻴﺩ‪ ،‬ﻗﺒل ﺃﻥ ﻨﻨﻬﻲ ﻫﺫﻩ ﺍﻟﻔﻘﺭﺓ ﺘﻌﻤﻴﻡ ﺍﻟﺘﻌﺭﻴﻑ )‪ (3.43‬ﻟﻴﺸﻤل ﻤﻴﻜﺎﻨﻴﻙ‬
‫‪‬‬
‫ﺍﻟﻜﻡ‪ ،‬ﺤﻴﺙ ﻨﺴﺘﺒﺩل ﺍﻟﻤﻘﺎﺩﻴﺭ ﺍﻟﻔﻴﺯﻴﺎﺌﻴﺔ ‪ f‬ﺒﻤﺅﺜﺭﺍﺕ ﻨﺭﻤﺯ ﻟﻬﺎ ‪ f‬ﻟﻴﺱ ﻟﻬﺎ ﻤﻌﻨﻰ‬
‫ﻓﻴﺯﻴﺎﺌﻴﺎﹰ ﺇﻻ ﺇﺫﺍ ﺃﺜﺭﺕ ﻋﻠﻰ ﻤﺎ ﺒﻌﺩﻫﺎ‪ .‬ﻭﻓﻲ ﻫﺫﻩ ﺍﻟﺤﺎﻟﺔ ﻨﻌﺭﻑ ﺃﻗﻭﺍﺱ ﺒﻭﺍﺼﻭﻥ ﻓﻲ‬
‫ﻤﻴﻜﺎﻨﻴﻙ ﺍﻟﻜﻡ ﺒﺎﻟﻌﻼﻗﺔ‪:‬‬
‫‪  ‬‬ ‫‪ ‬‬ ‫‪ ‬‬
‫‪f1 , f 2 qu  i f1 , f 2   i  f1 f 2  f 2 f1 ‬‬ ‫)‪(3.55‬‬
‫‪‬‬ ‫‪‬‬ ‫‪‬‬ ‫‪‬‬
‫‪ ‬‬ ‫‪‬‬
‫ﻴﺴﻤﻰ ﺍﻟﻤﻘﺩﺍﺭ ‪ f1 , f 2 ‬ﻤﺒﺩل ﺍﻟﻤﺅﺜﺭﻴﻥ‪ .‬ﻀﻤﻥ ﻫﺫﺍ ﺍﻻﻁﺎﺭ ﺘﻜﺘﺏ ﻤﻌﺎﺩﻟﺔ‬
‫‪‬‬ ‫‪‬‬
‫ﻫﺎﻴﻨﺯﺒﺭﻍ ﻓﻲ ﻤﻴﻜﺎﻨﻴﻙ ﺍﻟﻜﻡ ﺍﻟﺘﻲ ﺘﻘﺎﺒل ﺘﻤﺎﻤﺎﹰ ﻤﻌﺎﺩﻻﺕ ﺍﻟﺤﺭﻜﺔ ﻓﻲ ﺍﻟﻤﻴﻜﺎﻨﻴﻙ‬
‫ﺍﻟﺘﻘﻠﻴﺩﻱ ﺒﺎﻟﺼﻴﻐﺔ ﺍﻟﺘﺎﻟﻴﺔ‪:‬‬
‫‪‬‬ ‫‪‬‬
‫‪d p i    d x i   ‬‬
‫‪ H , p ,‬‬ ‫‪ H , x ‬‬ ‫)‪(3.56a‬‬
‫‪dt  ‬‬ ‫‪‬‬ ‫‪dt‬‬ ‫‪‬‬ ‫‪‬‬
‫‪‬‬ ‫‪‬‬ ‫‪‬‬
‫ﺤﻴﺙ ‪ p ، H‬ﻭ ‪ x‬ﻫﻲ ﻤﺅﺜﺭﺍﺕ ﻫﺎﻤﻠﺘﻭﻥ ﻭﺍﻻﻨﺩﻓﺎﻉ ﻭﺍﻻﺤﺩﺍﺜﻲ ‪ x‬ﻋﻠﻰ‬
‫ﺍﻟﺘﺭﺘﻴﺏ‪ ،‬ﻤﻊ ﺍﻟﻌﻠﻡ ﺃﻨﻪ ﻴﻤﻜﻥ ﻜﺘﺎﺒﺔ ﻤﻌﺎﺩﻟﺔ ﺤﺭﻜﺔ ﺃﻱ ﻤﺅﺜﺭ ﺒﺎﻟﻁﺭﻴﻘﺔ ﻨﻔﺴﻬﺎ ﺤﻴﺙ‬
‫ﻨﺠﺩ‪:‬‬
‫‪‬‬
‫‪d L i    i      ‬‬
‫‪ H, L  H L L H ‬‬ ‫)‪(3.56b‬‬
‫‪dt  ‬‬ ‫‪ ‬‬ ‫‪‬‬
‫‪‬‬
‫ﻭﻨﻘﻭل‪ ،‬ﻜﻤﺎ ﻓﻲ ﺍﻟﻤﻴﻜﺎﻨﻴﻙ ﺍﻟﺘﻘﻠﻴﺩﻱ‪ ،‬ﺃﻥ ﺍﻟﻤﻘﺩﺍﺭ ﺍﻟﻔﻴﺯﻴﺎﺌﻲ ﺍﻟﻤﻘﺎﺒل ﻟﻠﻤﺅﺜﺭ ‪L‬‬
‫ﻴﻜﻭﻥ ﻤﺼﺎﻨﺎﹰ ) ﺘﻜﺎﻤﻼﹰ ﻟﻠﺤﺭﻜﺔ( ﺇﺫﺍ ﺍﻨﻌﺩﻤﺕ ﺃﻗﻭﺍﺱ ﺒﻭﺍﺼﻭﻥ ﺍﻟﻜﻭﺍﻨﺘﻴﺔ ﺍﻟﻤﻭﺍﻓﻘﺔ ﻟﻪ‪.‬‬
‫‪ - 22‬ﻣﻌﺎﺩﻟﺔ ﻫﺎﻣﻠﺘﻮﻥ ﺟﺎﻛﻮﺑﻲ‪Hamilton – Jakobi Equation :‬‬
‫ﺴﻨﺭﻯ ﺍﻵﻥ ﻓﻲ ﻫﺫﻩ ﺍﻟﻔﻘﺭﺓ ﻤﻌﺎﺩﻟﺔ ﺤﺭﻜﺔ ﺠﺩﻴﺩﺓ ﺘﻀﺎﻑ ﺇﻟﻰ ﺍﻟﻤﻌﺎﺩﻻﺕ ﺍﻟﺘﻲ‬
‫ﺭﺃﻴﻨﺎﻫﺎ ﺴﺎﺒﻘﺎﹰ ﻓﻲ ﺍﻟﻤﻴﻜﺎﻨﻴﻙ ﺍﻟﺘﺤﻠﻴﻠﻲ‪ .‬ﻟﺫﻟﻙ ﻨﺩﺭﺱ ﺠﻤﻠﺔ ﻤﺎﺩﻴﺔ ﻤﺤﺎﻓﻅﺔ )ﻴﺘﺤﻘﻕ‬
‫ﺇﺤﺩﺍﺜﻴﺎﺘﻬﺎ‬ ‫ﻨﻘﻁﺔ‬ ‫‪N‬‬ ‫ﻤﻥ‬ ‫ﻤﺅﻟﻔﺔ‬ ‫ﺍﻟﻁﺎﻗﺔ(‬ ‫ﻤﺼﻭﻨﻴﺔ‬ ‫ﻗﺎﻨﻭﻥ‬ ‫ﻓﻴﻬﺎ‬
‫‪ (i  1,...., N) x i , y i , z i‬ﻭﺒﺎﻟﺘﺎﻟﻲ ﻓﻬﻲ ﻤﻭﺼﻭﻓﺔ ﺒـ ‪ 3N‬ﺇﺤﺩﺍﺜﻲ‪ .‬ﻟﻨﺸﻜل ﻓﻀﺎﺀ‪‬‬
‫ﺍﻗﻠﻴﺩﻴﺎﹰ ﺃﺒﻌﺎﺩﻩ ‪ (E 3N 1 )3N  1‬ﻤﺅﻟﻔﺎﹰ ﻤﻥ ﺇﺤﺩﺍﺜﻴﺎﺕ ﺍﻟﻨﻘﻁ ﺍﻟﺴﺎﺒﻘﺔ ﻭﺍﻟﺯﻤﻥ‪ .‬ﺇﻥ‬

‫‪- 89 -‬‬

‫)‪Create PDF files without this message by purchasing novaPDF printer (http://www.novapdf.com‬‬
‫ﻭﺍﺤﺩﺓ‬ ‫ﺒﻨﻘﻁﺔ‬ ‫ﻴﺘﻌﻴﻥ‬ ‫‪t‬‬ ‫ﺍﻟﻠﺤﻅﺔ‬ ‫ﻓﻲ‬ ‫ﻜﻜل‬ ‫ﺍﻟﺠﻤﻠﺔ‬ ‫ﻭﻀﻊ‬
‫) ‪ M(x1 , y1 , z1,...., x N , y N , z N , t‬ﻓﻲ ﻫﺫﺍ ﺍﻟﻔﻀﺎﺀ‪ ،‬ﺃﻤﺎ ﻭﻀﻊ ﺍﻟﺠﻤﻠﺔ ﻓﻲ ﻟﺤﻅﺔ‬
‫ﺃﺨﺭﻯ ‪ t‬ﻓﻴﺘﻌﻴﻥ ﺒﺎﻟﻨﻘﻁﺔ ) ‪ M (x1 , y1 , z1 ,...., x N , y N , z N , t‬ﻭﺘﺭﺴﻡ ﺍﻟﻨﻘﻁﺔ ‪M‬‬
‫'‬ ‫'‬ ‫'‬ ‫'‬ ‫'‬ ‫'‬ ‫'‬ ‫'‬ ‫'‬

‫ﺒﻴﻥ ﺍﻟﻠﺤﻅﺘﻴﻥ ‪ t‬ﻭ ‪ t‬ﻤﺴﺎﺭﺍﹰ '‪ MM‬ﺘﻤﺜل ﻜل ﻨﻘﻁﺔ ﻤﻨﻪ ﻭﻀﻌﺎﹰ ﻤﻴﻜﺎﻨﻴﻜﻴﺎﹰ ﻟﻠﺠﻤﻠﺔ‪.‬‬
‫'‬

‫ﻟﻨﺸﻜل ﺍﻵﻥ ﻓﻀﺎﺀ ﺁﺨﺭ ‪ E n 1‬ﻤﺅﻟﻔﺎﹰ ﻤﻥ ﺍﻹﺤﺩﺍﺜﻴﺎﺕ ﺍﻟﻤﻌﻤﻤﺔ‬


‫‪ ( j  1, 2,..., n) q j‬ﻭﺍﻟﺯﻤﻥ ‪ .t‬ﺇﻥ ﻭﻀﻊ ﺍﻟﺠﻤﻠﺔ ﻓﻲ ﺍﻟﻠﺤﻅﺔ ‪ t‬ﻓﻲ ﻫﺫﺍ ﺍﻟﻔﻀﺎﺀ‬
‫ﻴﺘﻌﻴﻥ ﺒﻨﻘﻁﺔ ﻭﺍﺤﺩﺓ ) ‪ m(q1 , q 2 ,..., q n , t‬ﺘﻘﺎﺒل ﺍﻟﻨﻘﻁﺔ ‪ M‬ﻓﻲ ﺍﻟﻔﻀﺎﺀ ‪E 3N 1‬‬
‫ﺃﻤﺎ ﻭﻀﻊ ﺍﻟﺠﻤﻠﺔ ﻓﻲ ﺍﻟﻠﺤﻅﺔ ‪ t‬ﻓﻴﺘﻌﻴﻥ ﺒﻨﻘﻁﺔ ﺜﺎﻨﻴﺔ '‪ m‬ﻓﻲ ﺍﻟﻔﻀﺎﺀ ‪ E n 1‬ﻤﻘﺎﺒﻠﺔ‬
‫ﻟﻠﻨﻘﻁﺔ '‪ M‬ﻓﻲ ﺍﻟﻔﻀﺎﺀ ‪ ، E 3N 1‬ﻭﻋﻨﺩﻤﺎ ﺘﺘﺤﺭﻙ ‪ M‬ﻓﻲ ﺍﻟﻔﻀﺎﺀ ‪ E 3N 1‬ﺘﺘﺤﺭﻙ‬
‫‪ m‬ﻓﻲ ﺍﻟﻔﻀﺎﺀ ‪ E n 1‬ﺒﺤﻴﺙ ﺘﺭﺴﻡ ﻤﺴﺎﺭﺍﹰ '‪ ، mm‬ﻤﻘﺎﺒﻼﹰ ﻟﻠﻤﺴﺎﺭ '‪ ، MM‬ﺘﻘﺎﺒل ﻜل‬
‫ﻨﻘﻁﺔ ﻓﻴﻪ ﻭﻀﻌﺎﹰ ﻤﻴﻜﺎﻨﻴﻜﻴﺎﹰ ﻟﻠﺠﻤﻠﺔ ﻓﻲ ﺍﻟﻔﻀﺎﺀ ‪. E n 1‬‬
‫ﻨﻌﺭﻑ ﺍﻵﻥ ﺘﺎﺒﻊ ﺍﻟﻔﻌل ‪ (Action Function) S‬ﺒﺎﻟﻌﻼﻗﺔ‪:‬‬
‫‪‬‬
‫‪S   L (q j , q j , t )dt‬‬ ‫)‪(3.57‬‬
‫ﻭﻟﺘﻜﻥ ﺍﻟﻨﻘﻁﺔ ‪ A j‬ﺍﻟﻤﻤﺜﻠﺔ ﻟﻭﻀﻊ ﺍﻟﻤﺠﻤﻭﻋﺔ ﺍﻟﻤﺎﺩﻴﺔ ﻓﻲ ﺍﻟﻠﺤﻅﺔ ‪ t 1‬ﻓﻲ‬
‫ﺍﻟﻔﻀﺎﺀ ‪ . E n 1‬ﻴﻤﻜﻥ ﺍﻨﻁﻼﻗﺎﹰ ﻤﻥ ﺍﻟﻨﻘﻁﺔ ‪ A j‬ﺭﺴﻡ ﻤﺠﻤﻭﻋﺔ ﻻﻨﻬﺎﺌﻴﺔ ﻤﻥ‬
‫ﺍﻟﻤﺴﺎﺭﺍﺕ ﺍﻻﻓﺘﺭﺍﻀﻴﺔ )ﺭﺴﻤﺕ ﻓﻲ ﺒﻌﺩﻴﻥ ﻓﻘﻁ( ﺸﻜل )‪ (3.1‬ﻭﻟﻜﻥ ﻭﺍﺤﺩﺍﹰ ﻤﻨﻬﺎ‬
‫ﻓﻘﻁ )ﺍﻟﺨﻁ ﺍﻟﻤﺘﺼل ﻋﻠﻰ ﺍﻟﺸﻜل( ﻴﻤﺜل ﺍﻟﺤﺭﻜﺔ ﺍﻟﺤﻘﻴﻘﻴﺔ ﺍﻟﺘﻲ ﺘﺤﺩﺙ ﻓﻌﻼﹰ ﺘﺤﺕ‬
‫ﺘﺄﺜﻴﺭ ﺍﻟﻘﻭﻯ ﺍﻟﻤﻁﺒﻘﺔ ﻋﻠﻰ ﺍﻟﺠﻤﻠﺔ ﺍﻟﻤﺩﺭﻭﺴﺔ‪.‬‬

‫‪- 90 -‬‬

‫)‪Create PDF files without this message by purchasing novaPDF printer (http://www.novapdf.com‬‬
(3.1) ‫ﺍﻟﺸﻜﻞ‬

:‫ ﻓﻨﺠﺩ‬t ‫ ﻭﻟﺤﻅﺔ ﺃﺨﺭﻯ ﺍﺨﺘﻴﺎﺭﻴﺔ‬t 1 ‫ ﺒﻴﻥ ﺍﻟﻠﺤﻅﺘﻴﻥ‬S ‫ﻨﻔﺎﻀل ﺍﻵﻥ ﺍﻟﺘﺎﺒﻊ‬


n t 
 L L  
S     q  q dt (3.58)
j 1 t 
q j j q  j 
1 j 
:‫ﻓﺈﺫﺍ ﻋﻠﻤﻨﺎ ﺃﻥ‬
 d d
q j   q j  q j (3.59)
dt dt
:‫( ﺒﺎﻟﺼﻴﻐﺔ‬3.58) ‫ﻓﻴﻤﻜﻥ ﻭﻀﻊ ﺍﻟﺤﺩ ﺍﻟﺜﺎﻨﻲ ﻓﻲ‬
n t n t
L d L d L
   dt j   q j tt 1    q j dt  dt (3.60)
q 
j 1 t 1 q j j 1 q j t1 q j

‫( ﻨﺠﺩ ) ﺒﻌﺩ ﻤﻼﺤﻅﺔ ﺍﻨﻌﺩﺍﻡ ﺍﻟﺤﺩ ﺍﻷﻭل ﻤﻥ ﺍﻟﻁﺭﻑ‬3.58) ‫ﻭﺒﺎﻟﺘﺒﺩﻴل ﻓﻲ‬


( t 1 ‫ ﻓﻲ ﺍﻟﻠﺤﻅﺔ‬A ‫ ﻷﻥ ﻜل ﺍﻟﻤﺴﺎﺭﺍﺕ ﺘﺒﺩﺃ ﻤﻥ‬t  t 1 ‫( ﻋﻨﺩﻤﺎ‬3.60) ‫ﺍﻟﺜﺎﻨﻲ ﻓﻲ‬
:‫ ﻤﺎﻴﻠﻲ‬t  t 1 ‫ ﻋﻨﺩﻤﺎ‬q j  0 ‫ﻭﺒﺎﻟﺘﺎﻟﻲ‬
n t  n
S    L  d L q .dt  L
  q j dt q   j  q  q j (3.61)
j 1 t 1  j  j 1 j

- 91 -

Create PDF files without this message by purchasing novaPDF printer (http://www.novapdf.com)
‫ﻭﺒﻤﺎ ﺃﻥ ﺍﻟﺠﻤﻠﺔ ﺍﻟﻤﺎﺩﻴﺔ ﺍﻟﻤﺩﺭﻭﺴﺔ ﻫﻲ ﺠﻤﻠﺔ ﻤﺤﺎﻓﻅﺔ ﻭﺒﺎﻟﺘﺎﻟﻲ ﺘﺤﻘﻕ‬
‫ﻤﻌﺎﺩﻻﺕ ﻻﻏﺭﺍﻨﺞ )‪ (3.1‬ﻓﻴﻤﻜﻥ ﻜﺘﺎﺒﺔ )‪ (3.61‬ﻜﻤﺎﻴﻠﻲ‪:‬‬
‫‪S   p jq j‬‬ ‫)‪(3.62‬‬
‫‪j‬‬

‫ﻭﻤﻨﻪ ﻨﺴﺘﻨﺘﺞ ﺍﻟﻤﺸﺘﻕ ﺍﻟﺠﺯﺌﻲ ﻟﺘﺎﺒﻊ ﺍﻟﻔﻌل ‪ S‬ﺒﺎﻟﻨﺴﺒﺔ ﻟﻺﺤﺩﺍﺜﻴﺎﺕ ﺍﻟﻤﻌﻤﻤﺔ‪:‬‬


‫‪S‬‬ ‫‪L‬‬ ‫‪‬‬
‫‪‬‬ ‫‪ Pj  P  grad S‬‬ ‫)‪(3.63‬‬
‫‪q j q ‬‬
‫‪j‬‬

‫ﺃﻤﺎ ﺍﻟﻤﺸﺘﻕ ﺍﻟﻜﻠﻲ ﻓﻬﻭ ﻤﻌﺭﻭﻑ ﻤﻥ ﺘﻌﺭﻴﻑ ﺘﺎﺒﻊ ﺍﻟﻔﻌل )‪:S (3.57‬‬
‫‪dS‬‬
‫‪L‬‬ ‫)‪(3.64‬‬
‫‪dt‬‬
‫ﻭﻤﻥ ﺠﻬﺔ ﺜﺎﻨﻴﺔ ﻨﺤﺼل ﺒﺤﺴﺎﺏ ﺍﻟﻤﺸﺘﻕ ﺍﻟﻜﻠﻲ ﻟﺘﺎﺒﻊ ﺍﻟﻔﻌل ﺒﺎﻋﺘﺒﺎﺭﻩ ﺘﺎﺒﻌ ﺎﹰ‬
‫ﻟﻺﺤﺩﺍﺜﻴﺎﺕ ﺍﻟﻤﻌﻤﻤﺔ ﺍﻟﺘﻲ ﺘﺘﺒﻊ ﺒﺩﻭﺭﻫﺎ ﺍﻟﺯﻤﻥ‪:‬‬
‫‪dS‬‬ ‫‪S  S‬‬ ‫‪S‬‬
‫‪‬‬ ‫‪qj ‬‬ ‫‪  p j q j ‬‬ ‫)‪(3.65‬‬
‫‪dt‬‬ ‫‪q j‬‬ ‫‪t‬‬ ‫‪t‬‬

‫ﻭﻁﺒﻘﺎﹰ ﻟﺘﻌﺭﻴﻑ ﺘﺎﺒﻊ ﻫﺎﻤﻠﺘﻭﻥ ﻭﺍﻟﻌﻼﻗﺔ )‪ (3.64‬ﻨﻀﻊ )‪ (3.65‬ﺒﺎﻟﺸﻜل‪:‬‬


‫‪S‬‬
‫‪L   p jq j  ‬‬
‫‪t‬‬
‫ﻭﻤﻨﻪ ﻨﺤﺼل ﻋﻠﻰ ﻤﻌﺎﺩﻟﺔ ﻫﺎﻤﻠﺘﻭﻥ – ﺠﺎﻜﻭﺒﻲ ﺍﻟﺘﺎﻟﻴﺔ‪:‬‬
‫‪S‬‬
‫‪H 0‬‬ ‫)‪(3.66‬‬
‫‪t‬‬
‫ﻭﻟﻠﺤﺼﻭل ﻋﻠﻰ ﻤﻌﺎﺩﻟﺔ ﻫﺎﻤﻠﺘﻭﻥ – ﺠﺎﻜﻭﺒﻲ ﺍﻟﻨﻅﺎﻤﻴﺔ ﻴﺠﺏ ﺘﺒﺩﻴل ‪p j‬‬

‫ﺍﻟﻤﻭﺠﻭﺩﺓ ﻓﻲ ﺘﺎﺒﻊ ﻫﺎﻤﻠﺘﻭﻥ ﺒﻘﻴﻤﺘﻬﺎ ﺒﺩﻻﻟﺔ ﺍﻟﻤﺸﺘﻕ ﺍﻟﺠﺯﺌﻲ ﻟﺘﺎﺒﻊ ﺍﻟﻔﻌل ‪ S‬ﺒﺎﻟﻨﺴﺒﺔ‬
‫ﻟﻺﺤﺩﺍﺜﻴﺎﺕ ﺍﻟﻤﻌﻤﻤﺔ )‪ ،(3.63‬ﻭﻫﻜﺫﺍ ﻨﺤﺼل ﻋﻠﻰ ﺍﻟﻤﻌﺎﺩﻟﺔ‪:‬‬
‫‪S‬‬ ‫‪‬‬ ‫‪S S‬‬ ‫‪S ‬‬
‫; ‪ H q1 , q 2 ,..., q n‬‬ ‫‪,‬‬ ‫‪,...,‬‬ ‫‪,t 0‬‬ ‫)‪(3.67‬‬
‫‪t‬‬ ‫‪‬‬ ‫‪q1 q 2‬‬ ‫‪q n ‬‬
‫‪2‬‬
‫‪ S ‬‬
‫ﺍﻟﻨﺎﺘﺠﺔ‬ ‫‪p 2j  ‬‬ ‫‪‬‬ ‫ﻭﻫﻲ ﻤﻌﺎﺩﻟﺔ ﺘﻔﺎﻀﻠﻴﺔ ﺠﺯﺌﻴﺔ ﻻﺨﻁﻴﺔ ﻷﻨﻬﺎ ﺘﺤﻭﻱ‬
‫‪ q1 ‬‬
‫ﻋﻥ ﺘﺎﺒﻊ ﻫﺎﻤﻠﺘﻭﻥ‪.‬‬

‫‪- 92 -‬‬

‫)‪Create PDF files without this message by purchasing novaPDF printer (http://www.novapdf.com‬‬
‫ﻴﻤﻜﻥ ﻤﻥ ﺍﻟﻤﻌﺎﺩﻟﺘﻴﻥ )‪ (3.65‬ﺍﺴﺘﻨﺘﺎﺝ ﺘﻌﺭﻴﻑ ﺁﺨﺭ ﻟﺘﺎﺒﻊ ﺍﻟﻔﻌل ‪ S‬ﺤﻴﺙ ﻨﺠﺩ‬
‫ﺒﻘﻴﻤﺘﻬﺎ ﻤﻥ )‪ ،(3.66‬ﺃﻥ‪:‬‬ ‫ﻤﻨﻬﻤﺎ ﺒﻌﺩ ﺘﺒﺩﻴل‬
‫‪S‬‬
‫‪t‬‬
‫‪n‬‬ ‫‪dq j‬‬
‫‪dS‬‬
‫‪ H   p j‬‬ ‫)‪(3.68‬‬
‫‪dt‬‬ ‫‪j 1‬‬
‫‪dt‬‬

‫ﻭﻤﻨﻪ ﻨﺠﺩ ﺒﻌﺩ ﻀﺭﺏ ﺍﻟﻁﺭﻓﻴﻥ ﺒـ ‪ dt‬ﻭﺍﻻﺴﺘﻜﻤﺎل‪ ،‬ﺘﻌﺭﻴﻔﺎﹰ ﺁﺨﺭ ﻟـ ‪S‬‬


‫ﺒﺎﻟﻌﻼﻗﺔ ﺍﻟﺘﺎﻟﻴﺔ‪:‬‬
‫‪S    p j dq j   Hdt‬‬ ‫)‪(3.69‬‬
‫‪j‬‬

‫ﻣﺜﺎﻝ‪:‬‬
‫ﺍﻜﺘﺏ ﻤﻌﺎﺩﻟﺔ ﻫﺎﻤﻠﺘﻭﻥ – ﺠﺎﻜﻭﺒﻲ ﻟﺠﺴﻴﻡ ﻴﺘﺤﺭﻙ ﺒﺘﺄﺜﻴﺭ ﻗﻭﺓ ﺘﺸﺘﻕ ﻤﻥ‬
‫ﻜﻤﻭﻥ ) ‪. V (x, y, z , t‬‬
‫ﺍﳊﻞ ‪:‬‬
‫ﻤﻥ ﺍﻟﻀﺭﻭﺭﻱ ﺃﻭﻻﹰ ﺤﺴﺎﺏ ﺩﺍﻟﺔ ﻫﺎﻤﻠﺘﻭﻥ‪:‬‬
‫‪1‬‬ ‫‪2‬‬ ‫‪2‬‬ ‫‪2‬‬
‫‪H  T V ‬‬ ‫) ‪m(x  y  z )  V (x , y, z, t‬‬
‫‪2‬‬
‫‪1‬‬
‫‪‬‬ ‫) ‪(p 2x  p 2y  p 2z )  V (x, y, z, t‬‬
‫‪2m‬‬
‫ﻭﻻﺒﺩ ﻟﻜﺘﺎﺒﺔ ﺍﻟﻤﻌﺎﺩﻟﺔ ﺍﻟﻤﻁﻠﻭﺒﺔ ﻤﻥ ﺘﺒﺩﻴل ‪ P j‬ﺒﻘﻴﻤﺘﻬﺎ ﺒﺩﻻﻟﺔ ﺍﻟﻤﺸﺘﻘﺎﺕ‬
‫ﺍﻟﺠﺯﺌﻴﺔ ﻟﺘﺎﺒﻊ ﺍﻟﻔﻌل ﺍﻋﺘﻤﺎﺩﺍﹰ ﻋﻠﻰ )‪ (3.63‬ﺤﻴﺙ ﻨﺠﺩ‪:‬‬
‫‪S‬‬ ‫‪S‬‬ ‫‪S‬‬
‫‪px ‬‬ ‫‪, py ‬‬ ‫‪, pz ‬‬
‫‪x‬‬ ‫‪y‬‬ ‫‪z‬‬
‫ﻭﻋﻨﺩﺌﺫ‪ ‬ﻨﻀﻊ ﺍﻟﻤﻌﺎﺩﻟﺔ ﺍﻟﻤﻁﻠﻭﺒﺔ ﺒﺎﻟﺼﻴﻐﺔ ﺍﻟﺘﺎﻟﻴﺔ‪:‬‬
‫‪2‬‬
‫‪S 1  S ‬‬
‫‪2‬‬ ‫‪2‬‬
‫‪ S ‬‬ ‫‪ S  ‬‬
‫‪‬‬ ‫‪         V (x , y, z, t )  0‬‬
‫‪t 2m  x ‬‬ ‫‪ y ‬‬ ‫‪ z  ‬‬
‫‪‬‬ ‫‪‬‬
‫ﻭﺇﺫﺍ ﺍﺴﺘﺨﺩﻤﻨﺎ ﺍﻹﺤﺩﺍﺜﻴﺎﺕ ﺍﻟﻜﺭﻭﻴﺔ ﻨﺤﺼل ﻋﻠﻰ ﺍﻟﻤﻌﺎﺩﻟﺔ‪:‬‬
‫‪2‬‬ ‫‪2‬‬ ‫‪2‬‬
‫‪S 1  S ‬‬ ‫‪1  S ‬‬ ‫‪1‬‬ ‫‪ S  ‬‬
‫‪‬‬ ‫‪     ‬‬ ‫‪    V (r, , , t )  0‬‬
‫‪t 2m  r ‬‬ ‫‪2   ‬‬ ‫‪2‬‬ ‫‪2    ‬‬
‫‪‬‬ ‫‪r‬‬ ‫‪r Sin    ‬‬

‫‪- 93 -‬‬

‫)‪Create PDF files without this message by purchasing novaPDF printer (http://www.novapdf.com‬‬
‫ﻭﺃﺨﻴﺭﺍﹰ ﺇﺫﺍ ﺍﺴﺘﺨﺩﻤﻨﺎ ﺍﻹﺤﺩﺍﺜﻴﺎﺕ ﺍﻻﺴﻁﻭﺍﻨﻴﺔ‪:‬‬
‫‪2‬‬ ‫‪2‬‬ ‫‪2‬‬
‫‪S 1  S ‬‬ ‫‪1  S   S  ‬‬
‫‪‬‬ ‫‪          H (, , z , t )  0‬‬
‫‪t 2m     2     z  ‬‬

‫‪ - 23‬ﻛﻴﻒ ﻧﺴﺘﻔﻴﺪ ﻣﻦ ﻣﻌﺎﺩﻟﺔ ﻫﺎﻣﻠﺘﻮﻥ – ﺟﺎﻛﻮﺑﻲ ﰲ ﺣﻞ ﻣﺴﺎﺋﻞ‬


‫ﺍﳌﻴﻜﺎﻧﻴﻚ؟‬
‫ﺘﻌﺘﺒﺭ ﻤﺴﺄﻟﺔ ﻤﻴﻜﺎﻨﻴﻜﻴﺔ ﻤﺎ ﻤﻨﺘﻬﻴﺔ ﻋﻨﺩﻤﺎ ﻨﺤﺼل ﻋﻠﻰ ﻗﺎﻨﻭﻥ ﺤﺭﻜﺘﻬﺎ‪ ،‬ﻭﻴﺘﻡ‬
‫ﺫﻟﻙ ﺒﺤل ﺍﻟﻤﻌﺎﺩﻻﺕ ﺍﻟﺘﻔﺎﻀﻠﻴﺔ ﻟﻬﺫﻩ ﺍﻟﺤﺭﻜﺔ‪.‬‬
‫ﻭﺴﻨﺭﻯ ﻓﻴﻤﺎﻴﻠﻲ ﺃﻥ ﻤﻌﺎﺩﻟﺔ ﻫﺎﻤﻠﺘﻭﻥ‪-‬ﺠﺎﻜﻭﺒﻲ ﻭﺜﻴﻘﺔ ﺍﻟﺼﻠﺔ ﺒﺎﻟﻤﻌﺎﺩﻻﺕ‬
‫ﺍﻟﻘﺎﻨﻭﻨﻴﺔ )‪ (3.10‬ﺍﻟﺘﻲ ﺭﺃﻴﻨﺎﻫﺎ ﺴﺎﺒﻘﺎﹰ‪ ،‬ﻭﺃﻥ ﻨﻅﺭﻴﺔ ﺠﺎﻜﻭﺒﻲ ﺘﻨﺹ ﻋﻠﻰ ﺃﻥ ﺤل‬
‫ﺍﻷﻭﻟﻰ ﻴﻜﺎﻓﻰﺀ ﺘﻤﺎﻤﺎﹰ ﺤل ﺍﻟﺜﺎﻨﻴﺔ‪ .‬ﻭﻟﻜﻥ ﻗﺒل ﺃﻥ ﻨﻔﺼل ﻓﻲ ﻫﺫﺍ ﺍﻟﻤﻭﻀﻭﻉ ﺴﻨﻌﻁﻲ‬
‫ﻓﻜﺭﺓ ﻋﺎﻤﺔ ﻋﻥ ﺤل ﺍﻟﻤﻌﺎﺩﻻﺕ ﺍﻟﺘﻔﺎﻀﻠﻴﺔ ﺍﻟﺠﺯﺌﻴﺔ ﻤﻥ ﺍﻟﻨﻭﻉ )‪.(3.67‬‬
‫ﺇﻥ ﻟﻜل ﻤﻥ ﺍﻟﻤﻌﺎﺩﻻﺕ ﺍﻟﻤﺫﻜﻭﺭﺓ‪ ،‬ﺒﺼﻭﺭﺓ ﻋﺎﻤﺔ‪ ،‬ﻨﻭﻋﻴﻥ ﻤﻥ ﺍﻟﺤﻠﻭل؛‬
‫ﻴﺴﻤﻰ ﺍﻷﻭل ﺤﻼﹰ ﻋﺎﻤﺎﹰ ﻭﻴﻜﻭﻥ ﺘﺎﺒﻌﺎﹰ ﺍﺨﺘﻴﺎﺭﻴﺎﹰ ﻟﻠﻤﺘﺤﻭﻻﺕ ﺍﻟﻤﺴﺘﻘﻠﺔ ﺃﻤﺎ ﺍﻟﺜﺎﻨﻲ‬
‫ﻓﻴﺴﻤﻰ ﺤﻼﹰ ﺘﺎﻤ ﺎﹰ )ﺘﻜﺎﻤﻼﹰ ﺘﺎﻤﺎﹰ( ﻴﺘﺒﻊ ﺍﻟﻤﺘﺤﻭﻻﺕ ﺍﻟﻤﺴﺘﻘﻠﺔ ﻭﺜﻭﺍﺒﺕ ﺍﻟﻤﺴﺄﻟﺔ‪ .‬ﻏﻴﺭ ﺃﻨﻨﺎ‬
‫ﻓﻲ ﺍﻟﻤﻴﻜﺎﻨﻴﻙ ﻻﻨﻬﺘﻡ ﺒﺎﻟﺤل ﺍﻷﻭل‪.‬‬
‫ﺘﺤﻭﻱ ﺍﻟﻤﻌﺎﺩﻟﺔ )‪ (3.67‬ﻋﻠﻰ ‪ n  1‬ﻤﺘﺤﻭﻻﹰ ﻫﻲ ﺍﻹﺤﺩﺍﺜﻴﺎﺕ ‪ q j‬ﻭﺍﻟﺯﻤﻥ ‪t‬‬
‫ﻭﺒﺎﻟﺘﺎﻟﻲ ﺴﻨﺠﺩ ﻋﻨﺩ ﺍﺴﺘﻜﻤﺎﻟﻬﺎ ‪ n  1‬ﺜﺎﺒﺘﺄً‪ .‬ﻭﺒﻤﺎ ﺃﻥ ‪ S‬ﻴﺩﺨل ﻓﻲ ﺍﻟﻤﻌﺎﺩﻟﺔ ﺍﻟﻤﺫﻜﻭﺭﺓ‬
‫ﺒﺸﻜل ﻤﺸﺘﻕ ﻓﻴﻤﻜﻥ ﺍﻟﺘﺄﻜﻴﺩ ﺃﻨﻪ ﺇﺫﺍ ﻭﺠﺩ ﺤل ﻤﻥ ﺍﻟﻨﻭﻉ ) ‪ S(q1 , q 2 ,..., q n , t‬ﻓﻼﺒﺩ‬
‫ﺃﻥ ﻴﻭﺠﺩ ﺤل ﺁﺨﺭ ﻫﻭ ‪ ، S  C‬ﻭﻟﻜﻥ ﺍﻟﺜﺎﺒﺕ ‪ C‬ﻻﻴﻠﻌﺏ ﺩﻭﺭﺍﹰ ﻫﺎﻤﺎﹰ ﻓﻲ ﺍﻟﻤﻴﻜﺎﻨﻴﻙ‪،‬‬
‫ﺸﺄﻨﻪ ﺸﺄﻥ ﺍﻟﺜﺎﺒﺕ ﻓﻲ ﺘﺎﺒﻊ ﺍﻟﺠﻬﺩ‪ ،‬ﻭﻟﺫﻟﻙ ﻨﻬﻤﻠﻪ‪ .‬ﻭﻫﻜﺫﺍ ﻨﻀﻊ ﺍﻟﺤل ﺍﻟﺘﺎﻡ ﻟﻠﻤﻌﺎﺩﻟﺔ‬
‫)‪ (3.67‬ﻜﻤﺎﻴﻠﻲ‪:‬‬
‫)‪S  S(q1 , q 2 ,..., q n , 1 ,  2 ,...,  n , t ) (3.70‬‬
‫ﺤﻴﺙ‪:‬‬
‫‪ :  j‬ﺜﻭﺍﺒﺕ ﺍﻟﺘﻜﺎﻤل ﺍﻟﺘﻲ ﻋﺩﺩﻫﺎ ‪.n‬‬

‫‪- 94 -‬‬

‫)‪Create PDF files without this message by purchasing novaPDF printer (http://www.novapdf.com‬‬
‫ﻟﻨﺒﺤﺙ ﺍﻵﻥ ﻋﻥ ﺤل ﺍﻟﻤﻌﺎﺩﻻﺕ ﺍﻟﻘﺎﻨﻭﻨﻴﺔ )‪ (3.10‬ﺍﻋﺘﻤﺎﺩﺍﹰ ﻋﻠﻰ ﻤﺎﺴﺒﻕ‬
‫ﻭﻟﻬﺫﺍ ﻨﺤﺴﺏ ﺍﻟﻤﺸﺘﻘﺎﺕ ﺍﻟﺠﺯﺌﻴﺔ ﺒﺎﻟﻨﺴﺒﺔ ﻟﻠﺜﻭﺍﺒﺕ ‪  j‬ﻤﻥ )‪ (3.70‬ﻭﻨﺴﺎﻭﻴﻬﺎ ﺒﺜﺎﺒﺕ‬
‫ﺁﺨﺭ ‪  j‬ﻁﺒﻘﺎﹰ ﻟﻠﻤﻌﺎﺩﻻﺕ‪:‬‬
‫‪S‬‬
‫)‪  j ( j  1, 2, ..., n) (3.71‬‬
‫‪ j‬‬

‫ﻭﻫﻲ ﻋﺒﺎﺭﺓ ﻋﻥ ‪ n‬ﻤﻌﺎﺩﻟﺔ ﺠﺒﺭﻴﺔ ﻴﻤﻜﻥ ﺃﻥ ﻨﺤﺴﺏ ﻤﻨﻬﺎ ﻜل ﺍﻟﻤﺘﺤﻭﻻﺕ ‪q j‬‬

‫ﺒﺩﻻﻟﺔ ﺍﻟﺜﻭﺍﺒﺕ ﻭﺍﻟﺯﻤﻥ )ﻗﻭﺍﻨﻴﻥ ﺍﻟﺤﺭﻜﺔ(‪:‬‬


‫)‪q j  q j (1 ,  2 ,...,  n , 1,  2 ,...,  n , t ) (3.72‬‬

‫ﻭﻟﻠﺤﺼﻭل ﻋﻠﻰ ﺍﻻﻨﺩﻓﺎﻋﺎﺕ ﺍﻟﻤﻌﻤﻤﺔ ‪ p j‬ﻨﺴﺘﻔﻴﺩ ﻤﻥ )‪ (3.63‬ﻓﻨﺠﺩ ﺃﻥ‬


‫ﺍﻟﻁﺭﻑ ﺍﻷﻴﺴﺭ ﻴﺘﺒﻊ ﺍﻹﺤﺩﺍﺜﻴﺎﺕ ﺍﻟﻤﻌﻤﻤﺔ ‪ q j‬ﻭﺍﻟﺜﻭﺍﺒﺕ ‪ .  j‬ﻓﺈﺫﺍ ﺒﺩﻟﻨﺎ ‪ q j‬ﺒﻘﻴﻤﺘﻬﺎ‬
‫ﻤﻥ )‪ (3.72‬ﻨﺤﺼل ﺃﺨﻴﺭ ﺍﹰ ﻋﻠﻰ ‪ p j‬ﺍﻟﺘﺎﻟﻴﺔ‪:‬‬
‫)‪p j  p j (1 ,  2 ,...,  n , 1,  2 ,...,  n , t ) (3.73‬‬

‫ﻭﻫﻨﺎ ﻨﺘﺴﺎﺀل‪ :‬ﻤﺎﻫﻲ ﺍﻟﻌﻼﻗﺔ ﺒﻴﻥ ﺍﻟﺤﻠﻭل )‪ (3.72‬ﻭ )‪ (3.73‬ﻭﺤﻠﻭل‬


‫ﺍﻟﻤﻌﺎﺩﻻﺕ ﺍﻟﻘﺎﻨﻭﻨﻴﺔ )‪(3.10‬؟ ﻟﻘﺩ ﺃﻜﺩﻨﺎ ﻓﻲ ﺒﺩﺍﻴﺔ ﺍﻟﻔﻘﺭﺓ ﺍﻻﺭﺘﺒﺎﻁ ﺍﻟﻭﺜﻴﻕ ﺒﻴﻨﻬﻤﺎ‪ .‬ﺇﻥ‬
‫ﺠﻭﺍﺏ ﺍﻟﺴﺅﺍل ﺍﻟﺴﺎﺒﻕ ﻴﻜﻤﻥ ﻓﻲ ﻨﺹ ﻨﻅﺭﻴﺔ ﺠﺎﻜﻭﺒﻲ ﺍﻟﺘﺎﻟﻲ‪ :‬ﺇﻥ ﺍﻟﻤﻌﺎﺩﻻﺕ‬
‫)‪ (3.72‬ﻭ )‪ (3.73‬ﻫﻲ ﺒﺎﻟﻔﻌل ﺘﻜﺎﻤﻼﺕ ﺍﻟﻤﻌﺎﺩﻻﺕ ﺍﻟﻘﺎﻨﻭﻨﻴﺔ )‪ .(3.10‬ﻭﻟﺒﺭﻫﺎﻥ‬
‫ﺫﻟﻙ ﻨﺤﺴﺏ ﺍﻟﻤﺸﺘﻕ ﺍﻟﻜﻠﻲ ﺒﺎﻟﻨﺴﺒﺔ ﻟﻠﺯﻤﻥ ﻟﻠﻤﻌﺎﺩﻻﺕ )‪ (3.71‬ﻤﻊ ﺍﻟﻌﻠﻡ ﺃﻥ ‪ S‬ﺘﺘﺒﻊ‬
‫‪ q j‬ﺍﻟﺘﻲ ﺘﺘﺒﻊ ﺒﺩﻭﺭﻫﺎ ﺍﻟﺯﻤﻥ ﻭﻫﻜﺫﺍ ﻨﺠﺩ‪:‬‬
‫‪2‬‬ ‫‪n‬‬ ‫‪2‬‬
‫‪ S‬‬ ‫‪ S‬‬ ‫‪‬‬
‫‪‬‬ ‫)‪q k  0 (3.74‬‬
‫‪ jt k 1 jq k‬‬

‫ﻭﻤﻥ ﺠﻬﺔ ﺜﺎﻨﻴﺔ ﺇﺫﺍ ﺒﺩﻟﻨﺎ ‪ S‬ﺒﻘﻴﻤﺘﻬﺎ ﻤﻥ )‪ (3.70‬ﻓﻲ )‪ (3.67‬ﺜﻡ ﺍﺸﺘﻘﻴﻨﺎ‬


‫ﺍﻟﻤﻌﺎﺩﻟﺔ ﺍﻟﻨﺎﺘﺠﺔ ﺒﺎﻟﻨﺴﺒﺔ ﻟـ ‪  j‬ﻓﺈﻨﻨﺎ ﻨﺤﺼل ﻋﻠﻰ ﺍﻟﻤﻌﺎﺩﻻﺕ‪:‬‬
‫‪ 2S‬‬ ‫‪H p k‬‬ ‫‪ 2S‬‬ ‫‪n‬‬
‫‪ 2S H‬‬
‫‪‬‬ ‫‪‬‬ ‫‪‬‬ ‫)‪ 0 (3.75‬‬
‫‪t j k p k  j t j k 1 j q k p k‬‬

‫‪- 95 -‬‬

‫)‪Create PDF files without this message by purchasing novaPDF printer (http://www.novapdf.com‬‬
‫ﺘﺘﻁﺎﺒﻕ ﻤﺠﻤﻭﻋﺘﻲ ﺍﻟﻤﻌﺎﺩﻻﺕ )‪ (3.74) ، (3,75‬ﻭﺒﺎﻟﺘﺎﻟﻲ ﻨﺠﺩ ﺒﺎﻟﻤﻘﺎﺭﻨﺔ‬
‫ﺒﻴﻨﻬﻤﺎ ﺃﻥ‪:‬‬
‫‪H‬‬
‫‪q k ‬‬ ‫)‪(k  1, 2, ..., n ) (3.76a‬‬
‫‪p k‬‬
‫ﻫﻜﺫﺍ ﻨﺤﺼل ﻤﻥ ﺠﺩﻴﺩ ﻋﻠﻰ ﺍﻟﻤﺠﻤﻭﻋﺔ ﺍﻷﻭﻟﻰ ﻤﻥ ﻤﻌﺎﺩﻻﺕ ﻫﺎﻤﻠﺘﻭﻥ‪.‬‬
‫ﻭﻟﻠﺤﺼﻭل ﻋﻠﻰ ﺍﻟﻤﺠﻤﻭﻋﺔ ﺍﻟﺜﺎﻨﻴﺔ ﻨﺤﺴﺏ ﺍﻟﻤﺸﺘﻕ ﺍﻟﻜﻠﻲ ﺒﺎﻟﻨﺴﺒﺔ ﻟﻠﺯﻤﻥ ﻤﻥ‬
‫ﺍﻟﻤﻌﺎﺩﻟﺔ )‪ (3.63‬ﻓﻨﺠﺩ‪:‬‬
‫‪ 2S‬‬ ‫‪n‬‬
‫‪ 2S ‬‬
‫‪‬‬ ‫)‪q k  p j (3.77‬‬
‫‪q j .t k 1q jq k‬‬

‫ﻭﻤﻥ ﺠﻬﺔ ﺜﺎﻨﻴﺔ ﻨﺠﺩ ﺒﺎﺸﺘﻘﺎﻕ ﺍﻟﻤﻌﺎﺩﻟﺔ ﺍﻟﻌﺎﻤﺔ )‪ (3.67‬ﺠﺯﺌﻴﺎﹰ ﺒﺎﻟﻨﺴﺒﺔ ﺇﻟﻰ‬
‫ﻭﻫﻲ ﺒﺩﻭﺭﻫﺎ ﺘﺎﺒﻊ‬ ‫‪) q j‬ﻤﻊ ﺍﻟﻌﻠﻡ ﺃﻥ ‪ q j‬ﻤﻭﺠﻭﺩﺓ ﻀﻤﻥ ‪ p j‬ﺍﻟﺘﻲ ﺘﺴﺎﻭﻱ‬
‫‪S‬‬
‫‪q j‬‬

‫ﺼﺭﻴﺢ ﻟـ ‪ ( q j‬ﻤﺎﻴﻠﻲ‪:‬‬
‫‪ 2S‬‬ ‫‪n‬‬
‫‪H p k H‬‬
‫‪‬‬ ‫‪‬‬ ‫‪0‬‬
‫‪tq j k 1p k q j q j‬‬

‫ﻭﻤﻨﻪ‪:‬‬
‫‪ 2S‬‬ ‫‪n‬‬ ‫‪p‬‬ ‫‪H‬‬
‫‪  q k k  ‬‬ ‫)‪(3.78‬‬
‫‪tq j k 1 q j‬‬ ‫‪q j‬‬

‫ﻭﺒﻤﻘﺎﺭﻨﺔ)‪ (3.77‬ﻤﻊ )‪ (3.78‬ﻨﺤﺼل ﻋﻠﻰ ﺍﻟﻤﺠﻤﻭﻋﺔ ﺍﻟﺜﺎﻨﻴﺔ ﻤﻥ ﻤﻌﺎﺩﻻﺕ‬


‫ﻫﺎﻤﻠﺘﻭﻥ ﺍﻟﺘﺎﻟﻴﺔ‪:‬‬
‫‪H‬‬
‫)‪ p k (3.76b‬‬
‫‪q k‬‬
‫ﻭﺍﻟﺠﺩﻴﺭ ﺒﺎﻟﺫﻜﺭ ﺃﻥ ﻋﻜﺱ ﻨﻅﺭﻴﺔ ﺠﺎﻜﻭﺒﻲ ﺼﺤﻴﺢ ﺒﻤﻌﻨﻰ ﺃﻨﻪ ﻴﻤﻜﻥ‬
‫ﺍﻟﺤﺼﻭل ﻋﻠﻰ ﺍﻟﺘﻜﺎﻤل )‪ (3.70‬ﻤﻥ ﺍﻟﻤﻌﺎﺩﻻﺕ ﺍﻟﻘﺎﻨﻭﻨﻴﺔ )‪.(3.10‬‬
‫ﻨﻠﺨﺹ ﻤﺎﺴﺒﻕ ﻓﻨﻘﻭل‪ :‬ﻴﺘﻡ ﺤل ﻤﺴﺎﺌل ﺍﻟﻤﻴﻜﺎﻨﻴﻙ ﻁﺒﻘﺎﹰ ﻟﻨﻅﺭﻴﺔ ﻫﺎﻤﻠﺘﻭﻥ‪-‬‬
‫ﺠﺎﻜﻭﺒﻲ ﺒﺎﻟﻁﺭﻴﻘﺔ ﺍﻟﺘﺎﻟﻴﺔ‪:‬‬

‫‪- 96 -‬‬

‫)‪Create PDF files without this message by purchasing novaPDF printer (http://www.novapdf.com‬‬
‫ﺒﻌﺩ ﺃﻥ ﻨﺨﺘﺎﺭ ﺍﻟﻤﺘﺤﻭﻻﺕ ﺍﻟﻘﺎﻨﻭﻨﻴﺔ ‪ p j‬ﻭ ‪ q j‬ﺍﻟﻼﺯﻤﺔ ﻟﻭﺼﻑ ﺍﻟﺠﻤﻠﺔ‬
‫ﺍﻟﻤﺤﺎﻓﻅﺔ‪ ،‬ﺘﺸﻜل ﺘﺎﺒﻊ ﻫﺎﻤﻠﺘﻭﻥ ﻟﻬﺎ ﻭﻨﺒﺩل ﻓﻴﻪ ﺍﻻﻨﺩﻓﺎﻋﺎﺕ ﺍﻟﻤﻌﻤﻤﺔ ‪ p j‬ﺒﺎﻟﻤﺸﺘﻘﺎﺕ‬
‫ﺍﻟﺠﺯﺌﻴﺔ ﻟﺘﺎﺒﻊ ﺍﻟﻔﻌل ‪ S‬ﺒﺎﻟﻨﺴﺒﺔ ﻟـ ‪ q j‬ﻓﻨﺤﺼل ﺒﺎﻟﻨﺘﻴﺠﺔ ﻋﻠﻰ ﻤﻌﺎﺩﻻﺕ ﻫﺎﻤﻠﺘﻭﻥ –‬
‫ﺠﺎﻜﻭﺒﻲ ﺍﻟﺘﻲ ﻨﺒﺤﺙ ﻋﻥ ﺤﻠﻬﺎ )ﺘﻜﺎﻤﻠﻬﺎ( ﻤﻥ ﺍﻟﻨﻭﻉ )‪ (3.70‬ﺜﻡ ﻨﺸﺘﻕ ﻫﺫﺍ ﺍﻟﺤل‬
‫ﺒﺎﻟﻨﺴﺒﺔ ﻟﻠﺜﻭﺍﺒﺕ ‪  j‬ﻭﻨﺴﺎﻭﻱ ﻫﺫﻩ ﺍﻟﻤﺸﺘﻘﺎﺕ ﺒﺜﻭﺍﺒﺕ ﺠﺩﻴﺩﺓ ‪  j‬ﻓﻨﺤﺼل ﻋﻠﻰ‬
‫ﻤﺠﻤﻭﻋﺔ ﻤﻌﺎﺩﻻﺕ ﺠﺩﻴﺩﺓ‪ ،‬ﻋﺩﺩﻫﺎ ‪ ،n‬ﺒﺎﻟﻨﺴﺒﺔ ﺇﻟﻰ ‪ ، q j‬ﺒﺤﻠﻬﺎ ﻨﺠﺩ ‪ q j‬ﻤﺒﺎﺸﺭﺓﹰ‬
‫ﺒﺩﻻﻟﺔ ﺍﻟﺯﻤﻥ ﻭﺍﻟﺜﻭﺍﺒﺕ ‪  j‬ﻭ ‪)  j‬ﻗﻭﺍﻨﻴﻥ ﺍﻟﺤﺭﻜﺔ( ﻭﻟﻜﻨﻨﺎ ﺤﺼﻠﻨﺎ ﻋﻠﻴﻬﺎ ﻓﻲ ﻫﺫﻩ‬
‫ﺍﻟﻤﺭﺓ ﺍﻋﺘﻤﺎﺩﺍﹰ ﻋﻠﻰ ﻤﻌﺎﺩﻟﺔ ﻫﺎﻤﻠﺘﻭﻥ – ﺠﺎﻜﻭﺒﻲ‪.‬‬
‫‪ - 24‬ﻃﺮﻳﻘﺔ ﺍﻳﻨﺸﺘﲔ ﻟﻠﺤﺼﻮﻝ ﻋﻠﻰ ﻣﻌﺎﺩﻟﺔ ﻫﺎﻣﻠﺘﻮﻥ – ﺟﺎﻛﻮﺑﻲ‪:‬‬
‫ﻗﺒل ﺃﻥ ﻨﻨﺘﻘل ﺇﻟﻰ ﺍﻟﻔﻘﺭﺍﺕ ﺍﻷﺨﻴﺭﺓ ﻓﻲ ﻫﺫﺍ ﺍﻟﻔﺼل ﻤﻥ ﺍﻟﻤﻔﻴﺩ ﺸﺭﺡ ﻁﺭﻴﻘﺔ‬
‫ﺃﺨﺭﻯ ﻤﻨﺴﻭﺒﺔ ﺇﻟﻰ ﺍﻨﻴﺸﺘﻴﻥ ﻻﺴﺘﻨﺘﺎﺝ ﻤﻌﺎﺩﻟﺔ ﻫﺎﻤﻠﺘﻭﻥ‪ -‬ﺠﺎﻜﻭﺒﻲ ﻭﻫﻲ ﺍﻟﺘﺎﻟﻴﺔ‪:‬‬
‫ﻓﺭﺽ ﺍﻴﻨﺸﺘﻴﻥ ﺃﻥ ﺍﻟﺤﺭﻜﺔ ﺘﺤﺩﺙ ﻓﻲ ﻓﻀﺎﺀ ﺍﻗﻠﻴﺩﻱ ‪ E n‬ﺫﻱ ‪ n‬ﺒﻌﺩﺍﹰ‬
‫ﻋﻨﺎﺼﺭﻩ ﻫﻲ ﺍﻹﺤﺩﺍﺜﻴﺎﺕ ﺍﻟﻤﻌﻤﻤﺔ ‪ q j‬ﺒﺤﻴﺙ ﺘﺘﺤﻘﻕ ﻤﻌﺎﺩﻻﺕ ﻫﺎﻤﻠﺘﻭﻥ )‪(3.10‬‬
‫ﺩﻭﻤﺎﹰ ﻭﺃﻥ ﺍﻻﻨﺩﻓﺎﻋﺎﺕ ﺍﻟﻤﻌﻤﻤﺔ ‪ p j‬ﻫﻲ ﺘﻭﺍﺒﻊ ﻟـ ‪ q j‬ﻭﺘﺎﺒﻊ ﻫﺎﻤﻠﺘﻭﻥ ‪ H‬ﻭﺘﺸﻜل‬
‫ﺤﻘﻼﹰ ﻋﻨﺎﺼﺭﻩ ‪ p j‬ﻤﻌﺭﻓﺎﹰ ﻓﻲ ﻜل ﺍﻟﻔﻀﺎﺀ ‪ . E n‬ﻓﺈﺫﺍ ﺤﺴﺒﻨﺎ ﺍﻟﻤﺸﺘﻕ ﺍﻟﻜﻠﻲ ﻟـ ‪p j‬‬

‫ﺒﺩﻻﻟﺔ ﺍﻟﺯﻤﻥ ﻨﺠﺩ‪:‬‬


‫‪dp j‬‬ ‫‪p j‬‬ ‫‪n‬‬ ‫‪p j q j‬‬
‫‪‬‬ ‫‪‬‬ ‫)‪(3.79‬‬
‫‪dt‬‬ ‫‪t‬‬ ‫‪q t‬‬
‫‪i 1 i‬‬
‫ﻭﻋﻨﺩﺌﺫ‪ ‬ﻨﻀﻊ ﻤﻌﺎﺩﻟﺔ ﻫﺎﻤﻠﺘﻭﻥ ﺍﻟﺜﺎﻨﻴﺔ ﺒﺎﻟﺸﻜل‪:‬‬
‫‪P j‬‬ ‫‪n P dq‬‬
‫‪j‬‬ ‫‪i  H  0‬‬
‫‪‬‬ ‫‪‬‬
‫‪t i 1 q i dt q j‬‬
‫)‪(3.80‬‬

‫‪‬‬
‫ﻴﺸﺘﻕ ﻤﻥ ﻜﻤﻭﻥ‬ ‫‪p‬‬ ‫ﻓﺈﺫﺍ ﻓﺭﻀﻨﺎ ﺍﻵﻥ ﺃﻥ ﺤﻘل ﺍﻻﻨﺩﻓﺎﻋﺎﺕ‬
‫‪‬‬
‫) ‪ S(q1 , q 2 ,..., q n‬ﺒﺤﻴﺙ ﻴﻜﻭﻥ ‪ p  S‬ﻭﺒﺎﻟﺘﺎﻟﻲ ‪ rot p  0‬ﻭﻤﻨﻪ‪:‬‬
‫‪p j‬‬ ‫‪p i‬‬ ‫‪S‬‬
‫‪‬‬ ‫‪, pj ‬‬ ‫)‪(3.81‬‬
‫‪q i‬‬ ‫‪q j‬‬ ‫‪q j‬‬

‫‪- 97 -‬‬

‫)‪Create PDF files without this message by purchasing novaPDF printer (http://www.novapdf.com‬‬
‫ﺒﻘﻴﻤﺘﻬﺎ ﻤﻥ ﻤﻌﺎﺩﻻﺕ‬ ‫ﻭﺒﺎﻟﺘﻌﻭﻴﺽ ﻓﻲ )‪ (3.80‬ﻨﺠﺩ ﺒﻌﺩ ﺘﺒﺩﻴل‬
‫‪dq‬‬
‫‪dt‬‬
‫ﻫﺎﻤﻠﺘﻭﻥ‪:‬‬
‫‪p j‬‬ ‫‪n‬‬ ‫‪p i H H‬‬
‫‪‬‬ ‫‪‬‬ ‫‪0‬‬
‫‪t‬‬ ‫‪i 1‬‬
‫‪‬‬‫‪q‬‬ ‫‪j‬‬ ‫‪‬‬‫‪p‬‬ ‫‪i‬‬ ‫‪‬‬‫‪q‬‬ ‫‪j‬‬
‫‪‬‬
‫‪2‬‬
‫ﻭﺃﻥ ﺍﻟﺤﺩﻴﻥ ﺍﻟﺒﺎﻗﻴﻴﻥ ﻫﻤﺎ ﺍﻟﻤﺸﺘﻕ ﺍﻟﺠﺯﺌﻲ‬ ‫ﻨﻼﺤﻅ ﺃﻥ ﺍﻟﺤﺩ ﺍﻷﻭل ﻫﻭ‬
‫‪ s‬‬
‫‪q jt‬‬

‫ﻟﺩﺍﻟﺔ ﻫﺎﻤﻠﺘﻭﻥ ﺒﺎﻟﻨﺴﺒﺔ ﺇﻟﻰ ‪ q j‬ﻭﺒﺎﻟﺘﺎﻟﻲ ﻴﻤﻜﻥ ﻭﻀﻊ ﺍﻟﻌﻼﻗﺔ ﺍﻟﺴﺎﺒﻘﺔ ﻜﻤﺎﻴﻠﻲ‪:‬‬
‫ﺃﻭ‪:‬‬
‫‪  S‬‬ ‫‪‬‬
‫‪‬‬ ‫‪ H  0‬‬
‫‪q j  t‬‬ ‫‪‬‬
‫ﻭﻤﻨﻪ‪:‬‬
‫‪S‬‬
‫)‪ H  0 (3.82‬‬
‫‪t‬‬
‫ﻭﻫﻲ ﻤﻌﺎﺩﻟﺔ ﻫﺎﻤﻠﺘﻭﻥ – ﺠﺎﻜﻭﺒﻲ‪.‬‬
‫ﻴﻤﻜﻥ ﺃﻴﻀﺎﹰ ﺍﻟﺤﺼﻭل ﻋﻠﻰ ﺍﻟﻤﻌﺎﺩﻟﺔ )‪ (3.71‬ﺒﺎﺸﺘﻘﺎﻕ )‪ (3.82‬ﺠﺯﺌﻴ ﺎﹰ‬
‫ﺒﺎﻟﻨﺴﺒﺔ ﻟـ ‪)  j‬ﺒﻌﺩ ﺍﻴﺠﺎﺩ ﺍﻟﺘﻜﺎﻤل ﺍﻟﺘﺎﻡ ﻟﻬﺎ( ﺤﻴﺙ ﻨﺠﺩ‪:‬‬
‫‪ 2S‬‬ ‫‪H p i‬‬ ‫‪ 2S‬‬ ‫‪n dq‬‬
‫‪i‬‬ ‫‪ 2S‬‬
‫‪‬‬ ‫‪‬‬ ‫‪‬‬ ‫‪‬‬
‫‪t j‬‬ ‫‪i‬‬
‫‪p i  j t j i 1 dt t j‬‬
‫‪n dq‬‬
‫‪‬‬ ‫‪  S ‬‬
‫‪   i‬‬ ‫‪‬‬ ‫)‪ 0 (3.83‬‬
‫‪ t i 1 dt q i   j ‬‬

‫ﻓﺈﺫﺍ ﻻﺤﻅﻨﺎ ﺃﻥ ﻤﺎﺒﻴﻥ ﺍﻟﻤﻌﺘﺭﻀﺘﻴﻥ ﻫﻭ ﻤﺅﺜﺭ ﺍﻟﻤﺸﺘﻕ ﺍﻟﻜﻠﻲ ﺒﺎﻟﻨﺴﺒﺔ ﻟﻠﺯﻤﻥ‪،‬‬


‫ﻨﺘﺄﻜﺩ ﺃﻨﻪ ﻟﻜﻲ ﺘﺘﺤﻘﻕ ﺍﻟﻌﻼﻗﺔ ﺍﻟﺴﺎﺒﻘﺔ ﻴﺠﺏ ﺃﻥ ﻴﻜﻭﻥ‪:‬‬
‫‪S‬‬
‫‪j‬‬
‫‪ j‬‬

‫ﻭﻫﻲ ﺍﻟﻤﻌﺎﺩﻟﺔ )‪ (3.71‬ﺒﺎﻟﻀﺒﻁ‪.‬‬

‫‪- 98 -‬‬

‫)‪Create PDF files without this message by purchasing novaPDF printer (http://www.novapdf.com‬‬
‫‪ - 25‬ﻣﻌﺎﺩﻟﺔ ﻫﺎﻣﻠﺘﻮﻥ – ﺟﺎﻛﻮﺑﻲ ﰲ ﺍﳌﻴﻜﺎﻧﻴﻚ ﺍﳌﻮﺟﻲ ﻭﺍﻟﻌﻼﻗﺔ‬
‫ﺑﲔ ﺍﳌﻴﻜﺎﻧﻴﻚ ﺍﳌﻮﺟﻲ ﻭﺍﳌﻴﻜﺎﻧﻴﻚ ﺍﻟﺘﻘﻠﻴﺪﻱ‪:‬‬
‫ﻟﻨﻜﺘﺏ ﻫﺫﻩ ﺍﻟﻤﻌﺎﺩﻟﺔ ﺃﻭﻻﹰ ﻟﻠﺠﻤل ﺍﻟﻤﺎﺩﻴﺔ ﺍﻟﺘﻲ ﻴﺘﺤﻘﻕ ﻓﻴﻬﺎ ﻗﺎﻨﻭﻥ ﻤﺼﻭﻨﻴﺔ‬
‫ﺍﻟﻁﺎﻗﺔ ﻭﻟﻬﺫﺍ ﻨﺄﺨﺫ ﺘﺎﺒﻊ ﻓﻌل ﻤﺼﻐﺭ )ﺘﺎﺒﻊ ﻓﻌل ﻻﻏﺭﺍﻨﺞ( ﻴﻌﺭﻑ ﺒﺎﻟﻌﻼﻗﺔ‪:‬‬
‫‪S  S(t )   Ldt   (T  V )dt   (2T  E)dt  W  Et‬‬

‫ﺤﻴﺙ ‪ W‬ﻫﻭ ﻤﺎﻴﺴﻤﻰ ﺘﺎﺒﻊ ﻓﻌل ﻻﻏﺭﺍﻨﺞ ﺃﻭ ﺘﺎﺒﻊ ﺍﻟﻔﻌل ﺍﻷﺼﻐﺭ ﻭﻫﻭ‬
‫ﻻﻴﺘﻌﻠﻕ ﺒﺎﻟﺯﻤﻥ ﺒﺼﻭﺭﺓ ﺼﺭﻴﺤﺔ ﻭﻟﺒﺭﻫﺎﻥ ﺫﻟﻙ ﻨﻼﺤﻅ ﺃﻥ‪:‬‬
‫‪ ‬‬ ‫‪ ‬‬ ‫‪ ‬‬
‫‪dW  2Tdt  mx x dt  my y dt  mz z dt ‬‬
‫)‪p x dx  p y dy  p z dz (3.85‬‬

‫ﻭﻤﻥ ﺠﻬﺔ ﺜﺎﻨﻴﺔ‪ ،‬ﺇﺫﺍ ﻓﺎﻀﻠﻨﺎ ‪ W‬ﺒﺎﻋﺘﺒﺎﺭﻩ ﺘﺎﺒﻌﺎﹰ ﻟﻺﺤﺩﺍﺜﻴﺎﺕ ﻭﺍﻟﺯﻤﻥ ﻨﺠﺩ‪:‬‬
‫‪W‬‬ ‫‪W‬‬ ‫‪W‬‬ ‫‪W‬‬
‫‪dW ‬‬ ‫‪dx ‬‬ ‫‪dy ‬‬ ‫‪dz ‬‬ ‫‪dt‬‬ ‫)‪(3.86‬‬
‫‪x‬‬ ‫‪y‬‬ ‫‪z‬‬ ‫‪t‬‬
‫ﻭﺒﺎﻟﺘﺎﻟﻲ ﻻﻴﺤﻭﻱ‬ ‫ﻭﺒﻤﻘﺎﺭﻨﺔ )‪ (3.85‬ﻤﻊ )‪ (3.86‬ﻨﺴﺘﻨﺘﺞ ﺃﻥ ‪ 0‬‬
‫‪W‬‬
‫‪t‬‬
‫ﺍﻟﺘﺎﺒﻊ ‪ W‬ﺍﻟﺯﻤﻥ ﺒﺸﻜل ﺼﺭﻴﺢ‪ ،‬ﻭﻴﻤﻜﻥ ﻋﻨﺩﺌﺫ‪ ‬ﻤﻥ ﺍﻟﻤﻌﺎﺩﻟﺔ )‪ (3.85‬ﺍﻟﺤﺼﻭل‬
‫ﻋﻠﻰ ﺍﻟﻌﻼﻗﺔ‪:‬‬
‫‪‬‬
‫‪p  grad W  W‬‬ ‫)‪(3.87‬‬
‫ﻭﻋﻨﺩﺌﺫ‪ ‬ﺇﺫﺍ ﺒﺩﻟﻨﺎ ‪ p‬ﺒﻘﻴﻤﺘﻬﺎ ﻓﻲ ﺍﻟﻤﻌﺎﺩﻟﺔ ‪ T  V  E  0‬ﻨﺠﺩ ﺍﻟﻌﻼﻗﺔ‪:‬‬
‫‪1‬‬
‫‪grad W 2  V  E  0‬‬ ‫)‪(3.88‬‬
‫‪2m‬‬
‫)ﻫﺫﺍ ﻤﻊ ﺍﻟﻌﻠﻡ ﺃﻥ ‪ grad S  grad W‬ﻷﻥ ‪ E‬ﺜﺎﺒﺘﺔ(‪ .‬ﻭﻴﻤﻜﻥ ﺍﻟﺤﺼﻭل ﻋﻠﻰ‬
‫ﺼﻴﻐﺔ ﺃﺨﺭﻯ ﻟﻤﻌﺎﺩﻟﺔ ﻫﺎﻤﻠﺘﻭﻥ – ﺠﺎﻜﻭﺒﻲ ﺇﺫﺍ ﻻﺤﻅﻨﺎ ﻤﻥ )‪ (3.84‬ﺃﻥ ‪ E‬‬
‫‪S‬‬
‫‪t‬‬
‫ﻭﺒﺎﻟﺘﺎﻟﻲ ﻨﻀﻊ )‪ (3.88‬ﻜﻤﺎﻴﻠﻲ‪:‬‬
‫‪1‬‬
‫‪grad S(t )2  V  S  0‬‬ ‫)‪(3.89‬‬
‫‪2m‬‬ ‫‪t‬‬
‫ﻭﻫﻲ ﺍﻟﺼﻴﻐﺔ ﺍﻟﻤﻁﻠﻭﺒﺔ‪.‬‬

‫‪- 99 -‬‬

‫)‪Create PDF files without this message by purchasing novaPDF printer (http://www.novapdf.com‬‬
‫‪,V  0‬‬ ‫ﻓﻲ ﺍﻟﺤﺎﻟﺔ ﺍﻟﺨﺎﺼﺔ ﻋﻨﺩﻤﺎ ﻴﻜﻭﻥ ﺍﻟﺠﺴﻴﻡ ﺤﺭﺍﹰ ) ‪, p  Const.‬‬
‫‪.( E  Const.‬‬
‫ﻴﺄﺨﺫ ﺘﺎﺒﻊ ﺍﻟﻔﻌل ‪ S‬ﺍﻟﺼﻴﻐﺔ ﺍﻟﺘﺎﻟﻴﺔ‪:‬‬
‫‪ ‬‬
‫‪‬‬
‫)‪ p . r  Et S(t )  W  Et   p x dx  p y dy  p z dz  Et (3.90‬‬ ‫‪‬‬
‫ﻭﺴﻨﺒﺤﺙ ﺍﻵﻥ‪ ،‬ﻋﻠﻰ ﻀﻭﺀ ﻤﺎﺘﻘﺩﻡ‪ ،‬ﻋﻥ ﺍﻟﻤﻌﺎﺩﻟﺔ ﺍﻟﺘﻲ ﻴﺤﻘﻘﻬﺎ ﺘﺎﺒﻊ ﺍﻟﻔﻌل ‪S‬‬
‫ﻓﻲ ﺍﻟﻤﻴﻜﺎﻨﻴﻙ ﺍﻟﻤﻭﺠﻲ ﻭﻟﻬﺫﺍ ﻨﻨﻁﻠﻕ ﻤﻥ ﺍﻟﻤﻌﺎﺩﻟﺔ ﺍﻟﻤﻭﺠﻴﺔ ﺍﻟﻤﺴﺘﻘﺭﺓ‪ ،‬ﺍﻟﺘﻲ ﻴﺤﻘﻘﻬﺎ‬
‫ﺍﻟﺘﺎﺒﻊ ﺍﻟﻤﻭﺠﻲ ﻭﺍﻟﺘﻲ ﺘﺼﻑ ﺠﺴﻴﻤﺎﹰ ﺤﺭﺍﹰ ﻭﻫﻲ ﺍﻟﺘﺎﻟﻴﺔ‪:‬‬
‫‪2‬‬ ‫‪2‬‬
‫‪ k  0‬‬ ‫)‪(3.91‬‬
‫‪‬‬
‫‪ ‬ﻫﻭ ﻤﺅﺜﺭ ﻻﺒﻼﺱ ﻭ ‪ k‬ﻫﻭ ﺍﻟﻤﺘﺠﻪ ﺍﻟﻤﻭﺠﻲ ﺍﻟﻤﺭﺘﺒﻁ ﻤﻊ‬ ‫ﻤﻊ ﺍﻟﻌﻠﻡ ﺃﻥ‬
‫‪2‬‬

‫ﻁﺎﻗﺔ ﺍﻟﺠﺴﻴﻡ ﻭﺍﻨﺩﻓﺎﻋﻪ ﺒﺎﻟﻌﻼﻗﺘﻴﻥ‪:‬‬


‫‪‬‬ ‫‪‬‬ ‫‪p 2  2k 2‬‬
‫‪p  k , E‬‬ ‫‪‬‬
‫‪2m‬‬ ‫‪2m‬‬
‫‪. (  1,05  10‬‬ ‫ﺤﻴﺙ ‪ ‬ﺜﺎﺒﺕ ﺒﻼﻨﻙ ﻤﻘﺴﻭﻤﺎﹰ ﻋﻠﻰ ‪J.Sec)2‬‬
‫‪34‬‬

‫ﺇﻥ ﺤل ﺍﻟﻤﻌﺎﺩﻟﺔ )‪ (3.91‬ﻫﻭ ﺍﻟﺘﺎﻟﻲ‪:‬‬


‫‪1‬‬ ‫‪‬‬ ‫‪‬‬
‫‪‬‬ ‫) ‪(p . r‬‬ ‫‪ ‬‬

‫‪ ( r )  Ae ‬‬ ‫) ‪ Ae i( k . r‬‬ ‫)‪(3.92a‬‬


‫ﺃﻤﺎ ﻋﻨﺩﻤﺎ ﻨﺄﺨﺫ ﺍﻟﻤﻌﺎﺩﻟﺔ ﺍﻟﻤﻭﺠﺒﺔ ﻏﻴﺭ ﺍﻟﻤﺴﺘﻘﺭﺓ ﺒﺩﻻﹰ ﻤﻥ )‪ (3.91‬ﻓﻴﺠﺏ‬
‫ﺩﻤﺞ ﺍﻟﻘﺴﻡ ﺍﻟﺯﻤﻨﻲ ﺒﺎﻟﻘﺴﻡ ﺍﻻﺤﺩﺍﺜﻲ ﻭﺒﺎﻟﺘﺎﻟﻲ ﻨﻀﻊ )‪:(3.92a‬‬
‫‪1  ‬‬
‫‪‬‬ ‫)‪( p . r  ET‬‬
‫‪ ( r , t )  Ae ‬‬ ‫)‪(3.92b‬‬
‫ﻨﻌﺒﺭ ﻋﻥ ﺍﻷﺱ ﺒﺩﻻﻟﺔ ﺘﺎﺒﻊ ﺍﻟﻔﻌل )‪ S (3.90‬ﺤﻴﺙ ﻨﺠﺩ‪:‬‬
‫‪1‬‬
‫‪‬‬ ‫) ‪S( t‬‬
‫‪‬‬
‫‪ ( r , t )  Ae‬‬ ‫)‪(3.92c‬‬
‫ﻭﻴﻤﻜﻥ ﺍﻵﻥ ﺍﻟﺤﺼﻭل ﻋﻠﻰ ﺼﻴﻐﺔ ﻟﻤﻌﺎﺩﻟﺔ ﻫﺎﻤﻠﺘﻭﻥ‪-‬ﺠﺎﻜﻭﺒﻲ ﻓﻲ ﺍﻟﻤﻴﻜﺎﻨﻴﻙ‬
‫ﺍﻟﻤﻭﺠﻲ ﺇﺫﺍ ﺍﻋﺘﻤﺩﻨﺎ ﻋﻠﻰ ﻤﻌﺎﺩﻟﺔ ﺸﺭﻭﺩﻴﻨﻐﺭ ﺍﻟﺘﻲ ﺘﺄﺨﺫ ﺍﻟﺨﻭﺍﺹ ﺍﻟﺠﺴﻴﻤﻴﺔ‬
‫ﻭﺍﻟﻤﻭﺠﻴﺔ ﺒﻌﻴﻥ ﺍﻻﻋﺘﺒﺎﺭ‪ ،‬ﻭﻫﻲ ﺍﻟﺘﺎﻟﻴﺔ‪:‬‬

‫‪- 100 -‬‬

‫)‪Create PDF files without this message by purchasing novaPDF printer (http://www.novapdf.com‬‬
‫‪ p2‬‬ ‫‪‬‬
‫‪(H  E)  ‬‬ ‫‪ V  E   0‬‬ ‫)‪(3.93‬‬
‫‪ 2m‬‬ ‫‪‬‬
‫‪‬‬ ‫‪‬‬
‫ﻭﺴﻨﻔﺭﺽ ﺍﻵﻥ ﺃﻥ ﺍﻟﺩﺍﻟﺔ ﺍﻟﻤﻭﺠﻴﺔ )‪ (3.92c‬ﺘﺤﻘﻕ ﻫﺫﻩ ﺍﻟﻤﻌﺎﺩﻟﺔ‪ ،‬ﻭﻨﺤﺴﺏ‬
‫ﺤﺩﻭﺩﻫﺎ ﺤﺩﺍﹰ ﺤﺩﺍﹰ ﺒﻌﺩ ﺍﻟﻌﻠﻡ ﺃﻥ ‪ p‬ﺘﻌﺘﺒﺭ ﻤﺅﺜﺭ ﺍﹰ ﻓﻲ ﻤﻴﻜﺎﻨﻴﻙ ﺍﻟﻜﻡ ﻴﻌﺒﺭ ﻋﻨﻪ‬
‫‪‬‬
‫ﺒﺎﻟﺼﻴﻐﺔ‪ p  i :‬ﻭﻴﻜﻭﻥ‪:‬‬
‫‪2‬‬ ‫‪‬‬ ‫‪‬‬ ‫‪‬‬ ‫‪        ‬‬
‫‪P   p p   p(i )  p  i i‬‬ ‫‪ j‬‬ ‫‪k‬‬ ‫‪‬‬
‫‪‬‬
‫‪‬‬ ‫‪‬‬‫‪x‬‬ ‫‪‬‬‫‪y‬‬ ‫‪‬‬‫‪z‬‬
‫‪‬‬ ‫‪‬‬ ‫‪‬‬
‫ﻭﺒﺎﻟﺤﺴﺎﺏ ﻨﺠﺩ ﺃﺨﻴﺭﺍﹰ‪:‬‬
‫‪p 2    2  2   i. 2S  (grad S) 2 ‬‬
‫ﻭﺒﺎﻟﺘﻌﻭﻴﺽ ﻓﻲ )‪ (3.93‬ﻭﺤﺫﻑ ‪ ‬ﻤﻥ ﺍﻟﻁﺭﻓﻴﻥ ﻨﺤﺼل ﻋﻠﻰ ﻤﻌﺎﺩﻟﺔ‬
‫ﻫﺎﻤﻠﺘﻭﻥ‪-‬ﺠﺎﻜﻭﺒﻲ‪ ،‬ﻓﻲ ﺍﻟﻤﻴﻜﺎﻨﻴﻙ ﺍﻟﻤﻭﺠﻲ‪ ،‬ﺍﻟﺘﺎﻟﻴﺔ‪:‬‬
‫‪2‬‬
‫‪1‬‬ ‫‪2‬‬ ‫‪ S‬‬
‫‪(grad S)  V  E  i‬‬ ‫‪0‬‬ ‫)‪(3.94‬‬
‫‪2m‬‬ ‫‪2m‬‬
‫ﺘﻌﺘﺒﺭ ﺍﻟﻤﻌﺎﺩﻟﺔ )‪ (3.94‬ﺼﻴﻐﺔ ﺜﺎﻨﻴﺔ ﻟﻤﻌﺎﺩﻟﺔ ﺸﺭﻭﺩﻴﻨﻐﺭ ﺍﻟﻤﻌﺭﻭﻓﺔ ﻓﻲ‬
‫ﻤﻴﻜﺎﻨﻴﻙ ﺍﻟﻜﻡ‪ ،‬ﻭﺇﺫﺍ ﺤﺫﻓﻨﺎ ﺍﻟﺤﺩ ﺍﻷﺨﻴﺭ ﻨﺠﺩ ﻤﻌﺎﺩﻟﺔ ﻫﺎﻤﻠﺘﻭﻥ – ﺠﺎﻜﻭﺒﻲ ﻨﻔﺴﻬﺎ‬
‫ﻭﺒﺎﻟﺘﺎﻟﻲ ﻓﺎﻟﻤﻌﺎﺩﻟﺔ )‪ (3.94‬ﻫﻲ ﺘﻌﻤﻴﻡ ﻟﻠﻤﻌﺎﺩﻟﺔ ﺍﻟﻤﺫﻜﻭﺭﺓ‪ .‬ﻭﺍﻟﺴﺅﺍل ﺍﻵﻥ‪ :‬ﻤﺘﻰ‬
‫ﻴﻤﻜﻥ ﺤﺫﻑ ﺍﻟﺤﺩ ﺍﻷﺨﻴﺭ ﻭﻤﺎﻫﻭ ﺍﻟﻤﻌﻨﻰ ﺍﻟﻔﻴﺯﻴﺎﺌﻲ ﻟﻬﺫﺍ ﺍﻟﺤﺫﻑ؟‬
‫ﻨﻼﺤﻅ ﻤﺒﺎﺸﺭﺓﹰ ﻤﻥ )‪ (3.94‬ﺃﻨﻪ ﻴﻤﻜﻥ ﺇﻫﻤﺎل ﺍﻟﺤﺩ ﺍﻷﺨﻴﺭ ﺍﻟﺤﺎﻭﻱ ﻋﻠﻰ‬
‫ﺍﻟﺘﺄﺜﻴﺭﺍﺕ ﺍﻟﻤﻭﺠﻴﺔ )ﺒﺴﺒﺏ ﻭﺠﻭﺩ ‪ ‬ﻓﻴﻪ( ﻋﻨﺩﻤﺎ ﺘﺘﺤﻘﻕ ﺍﻟﻌﻼﻗﺔ‪:‬‬
‫‪2‬‬ ‫‪2‬‬
‫‪(grad S)   /  S /‬‬ ‫)‪(3.95‬‬
‫ﻴﺴﻤﻰ ﻫﺫﺍ ﺍﻟﺘﻘﺭﻴﺏ‪ ،‬ﺍﻟﺫﻱ ﻴﺴﻤﺢ ﺒﺈﻫﻤﺎل ﺍﻟﺤﺩﻭﺩ ﺍﻟﺘﻲ ﺘﺤﻭﻱ ‪ ‬ﻓﻲ ﺃﻴﺔ‬
‫ﻤﻌﺎﺩﻟﺔ ﻤﻭﺠﻴﺔ‪ ،‬ﺍﻟﺘﻘﺭﻴﺏ ﺸﺒﻪ ﺍﻟﻜﻼﺴﻴﻜﻲ‪ ،‬ﻭﻋﻨﺩﺌﺫ‪ ‬ﻨﺤﺼل ﻋﻠﻰ ﺍﻟﻤﻌﺎﺩﻟﺔ ﺍﻟﻜﻼﺴﻴﻜﻴﺔ‬
‫ﺍﻟﻤﻘﺎﺒﻠﺔ‪ .‬ﻭﺒﻌﺎﺭﺓ ﺃﺨﺭﻯ ﻴﻤﻜﻥ ﺍﻋﺘﺒﺎﺭ ﺍﻟﻤﻴﻜﺎﻨﻴﻙ ﺍﻟﺘﻘﻠﻴﺩﻱ ﺤﺎﻟﺔ ﺨﺎﺼﺔ ﻤﻥ ﺍﻟﻤﻴﻜﺎﻨﻴﻙ‬
‫ﺍﻟﻤﻭﺠﻲ ﻴﻁﺒﻕ ﻋﻨﺩﻤﺎ ﻨﻬﻤل ﺍﻟﺘﺄﺜﻴﺭﺍﺕ )ﺍﻟﺤﺩﻭﺩ( ﺍﻟﺘﻲ ﺘﺤﻭﻱ ﺜﺎﺒﺕ ﺒﻼﻨﻙ ‪. ‬‬
‫ﻟﻨﻌﺩ ﺇﻟﻰ )‪ (3.95‬ﻭﻨﻜﺘﺏ ‪  S‬ﻜﻤﺎﻴﻠﻲ‪:‬‬
‫‪2‬‬

‫‪ 2S  .S  div grad S  div p‬‬

‫‪- 101 -‬‬

‫)‪Create PDF files without this message by purchasing novaPDF printer (http://www.novapdf.com‬‬
‫ﻭﺒﺎﻟﺘﺎﻟﻲ ﻴﺅﻭل ﺍﻟﺘﻘﺭﻴﺏ )‪ (3.95‬ﺇﻟﻰ‪:‬‬
‫‪‬‬
‫‪div p  1‬‬ ‫)‪(3.96‬‬
‫‪P2‬‬
‫ﻭﻋﻨﺩﻤﺎ ﺘﺤﺩﺙ ﺍﻟﺤﺭﻜﺔ ﻓﻲ ﺒﻌﺩ ﻭﺍﺤﺩ ﻨﺠﺩ‪:‬‬
‫‪‬‬
‫‪d ‬‬
‫‪ dp‬‬ ‫‪p d‬‬
‫‪‬‬ ‫‪‬‬ ‫‪ 1‬‬ ‫)‪(3.97‬‬
‫‪p 2 dx‬‬ ‫‪dx‬‬ ‫‪dx‬‬

‫ﺃﻱ ﺃﻥ ﺸﺭﻁ ﺘﻁﺒﻴﻕ ﺍﻟﺘﻘﺭﻴﺏ ﺍﻟﺴﺎﺒﻕ ﻫﻭ ﺃﻥ ﻴﻜﻭﻥ ﻁﻭل ﻤﻭﺠﺔ ﺩﻭﺒﺭﻱ‬


‫‪ 1‬‬
‫‪  ‬ﺜﺎﺒﺘﺎﹰ ﺃﻭ ﻴﺘﻐﻴﺭ ﺘﻐﻴﺭﺍﹰ ﻁﻔﻴﻔﺎﹰ ﺠﺩﺍﹰ ﻤﻊ ﺍﻟﻤﺴﺎﻓﺔ‪.‬‬ ‫‪‬‬
‫‪p k‬‬

‫ﻓﻲ ﺍﻟﺤﺎﻟﺔ ﺍﻟﺨﺎﺼﺔ ﻋﻨﺩﻤﺎ ﻴﺘﺤﺭﻙ ﺍﻟﺠﺴﻴﻡ ﻀﻤﻥ ﺍﻟﺠﻬﺩ )‪ V( x‬ﻴﻜﻭﻥ‪:‬‬


‫‪p  2mE  V (x )‬‬ ‫)‪(3.98‬‬

‫ﻓﺈﺫﺍ ﺤﺴﺒﻨﺎ ﺍﻟﺘﻘﺭﻴﺏ ﺍﻟﺴﺎﺒﻕ ﻓﻲ ﻫﺫﻩ ﺍﻟﺤﺎﻟﺔ ﻓﺈﻨﻨﺎ ﻨﺠﺩ‪:‬‬


‫‪V‬‬
‫‪2m‬‬
‫‪ dp‬‬ ‫‪‬‬ ‫‪x‬‬ ‫‪mF‬‬
‫‪2 dx‬‬
‫‪‬‬
‫‪2 2 2mE  V ( x ) ‬‬
‫‪‬‬ ‫)‪(3.99‬‬
‫‪p‬‬ ‫‪p‬‬ ‫‪P3‬‬

‫ﺤﻴﺙ ﻭﻀﻌﻨﺎ‪:‬‬
‫‪V‬‬
‫‪F  Fx  (grad V) x  ‬‬ ‫)‪(3.100‬‬
‫‪x‬‬

‫ﻓﺎﻟﺘﻘﺭﻴﺏ ﺸﺒﻪ ﺍﻟﻜﻼﺴﻴﻜﻲ ﻻﻴﻁﺒﻕ ﻋﻨﺩﻤﺎ ﻴﻜﻭﻥ ﺍﻻﻨﺩﻓﺎﻉ ‪ P‬ﺼﻐﻴﺭﺍﹰ ﺠﺩﺍﹰ‬


‫ﻭﻴﺤﺩﺙ ﻫﺫﺍ ﺒﺼﻭﺭﺓ ﺨﺎﺼﺔ ﻋﻨﺩﻤﺎ ﺘ ﺘﺴﺎﻭﻱ ﻁﺎﻗﺔ ﺍﻟﺠﺴﻴﻡ ﺍﻟﻜﻠﻴﺔ ﻤﻊ ﺍﻟﺠﻬﺩ ﺍﻟﺫﻱ‬
‫ﻴﺘﺤﺭﻙ ﻓﻴﻪ )‪ (E  V , p  0‬ﻭﻴﺠﺏ ﺍﻻﻨﺘﺒﺎﻩ ﺇﻟﻰ ﻭﺠﻭﺩ ‪ ‬ﻓﻲ ﺒﺴﻁ ﺍﻟﻜﺴﺭ ﻤﻤﺎ‬
‫ﻴﺠﻌل ﺍﻟﻜﺴﺭ ﺼﻐﻴﺭﺍﹰ ﺠﺩﺍﹰ ﻭﺒﺎﻟﺘﺎﻟﻲ ﻓﺈﻥ ﺸﺭﻭﻁ ﻫﺫﺍ ﺍﻟﺘﻘﺭﻴﺏ ﻻﺘﺘﺤﻘﻕ ﺇﻻ ﺇﺫﺍ ﺍﻨﺘﻬﻰ‬
‫ﺍﻻﻨﺩﻓﺎﻉ ﺇﻟﻰ ﺍﻟﺼﻔﺭ‪ .‬ﻭﻋﻨﺩﻤﺎ ﻴﺤﺩﺙ ﺫﻟﻙ )‪ (p  0‬ﻻﻴﻤﻜﻥ ﺇﻫﻤﺎل ﺍﻟﺤﺩ ﺍﻟﺤﺎﻭﻱ‬
‫ﻋﻠﻰ ﺍﻟﺘﺄﺜﻴﺭﺍﺕ ﺍﻟﻤﻭﺠﻴﺔ ﻭﻻﺒﺩ ﻤﻥ ﺘﻁﺒﻴﻕ ﺍﻟﻤﻴﻜﺎﻨﻴﻙ ﺍﻟﻤﻭﺠﻲ ﺃﻭ ﺍﻟﻤﻴﻜﺎﻨﻴﻙ ﺍﻟﻜﻤﻭﻤﻲ‬
‫)ﻤﻴﻜﺎﻨﻴﻙ ﺍﻟﻜﻡ(‪.‬‬

‫‪- 102 -‬‬

‫)‪Create PDF files without this message by purchasing novaPDF printer (http://www.novapdf.com‬‬
‫‪ - 26‬ﺗﺎﺑﻊ ﺍﻟﺘﻮﺯﻉ ﺍﻻﺣﺼﺎﺋﻲ‪:‬‬
‫ﻤﻥ ﺍﻟﻤﻔﻴﺩ ﺃﻥ ﻨﺨﺘﻡ ﺍﻟﻔﺼل ﺒﺈﻟﻘﺎﺀ ﻨﻅﺭﺓ ﻋﻠﻰ ﻁﺭﻴﻘﺔ ﺩﺭﺍﺴﺔ ﺠﻤﻠﺔ ﻤﺎﺩﻴﺔ‬
‫ﻤﺅﻟﻔﺔ ﻤﻥ ﻋﺩﺩ ﻫﺎﺌل ﻤﻥ ﺍﻟﺠﺴﻴﻤﺎﺕ ﻜﻤﺠﻤﻭﻋﺔ ﺠﺯﻴﺌﺎﺕ ﻏﺎﺯ ﻤﻭﺠﻭﺩﺓ ﻓﻲ ﺤﻴﺯ‬
‫ﻤﻌﻴﻥ‪ ،‬ﺃﻭ ﺠﺯﻴﺌﺎﺕ ﻤﺎﺌﻊ ﻤﺘﺤﺭﻙ ﺃﻭ ﻤﺠﻤﻭﻋﺔ ﻤﻥ ﺍﻟﺠﺴﻴﻤﺎﺕ ﺍﻟﻤﺠﻬﺭﻴﺔ ﺫﺍﺕ ﺩﺭﺠﺔ‬
‫ﺤﺭﺍﺭﺓ ﻋﺎﻟﻴﺔ )ﺒﻼﺯﻤﺎ(‪ ...‬ﺍﻟﺦ‪ .‬ﻜﻴﻑ ﻨﺘﻌﺎﻤل "ﻤﻴﻜﺎﻨﻴﻜﻴﺎﹰ" ﻤﻊ ﻫﺫﻩ ﺍﻟﺠﻤل؟ ﻜﻴﻑ‬
‫ﻨﺤﺴﺏ ﻁﺎﻗﺘﻬﺎ ﻭﻨﺩﺭﺱ ﺨﻭﺍﺼﻬﺎ ﺍﻟﻤﻴﻜﺎﻨﻴﻜﻴﺔ ﺍﻟﻌﺎﻤﺔ؟‬
‫ﻟﻘﺩ ﺩﺭﺴﻨﺎ ﻓﻴﻤﺎ ﺴﺒﻕ ﺠﻤﻼﹰ ﻤﺎﺩﻴﺔ ﺘﺤﻭﻱ ﻋﺩﺩ ﺍﹰ ﻤﺤﺩﻭﺩﺍﹰ ﻤﻥ ﺍﻟﻨﻘﻁ ﺍﻟﻤﺎﺩﻴﺔ‬
‫)ﺍﻟﺠﺴﻴﻤﺎﺕ( ﺃﻭ‪ ،‬ﺒﻌﺒﺎﺭﺓ ﺃﺨﺭﻯ‪ ،‬ﺫﺍﺕ ﻋﺩﺩ ﻤﺤﺩﻭﺩ ﻤﻥ ﺍﻹﺤﺩﺍﺜﻴﺎﺕ ﺍﻟﻤﻌﻤﻤﺔ ‪q j‬‬

‫ﻭﺍﻻﻨﺩﻓﺎﻋﺎﺕ ﺍﻟﻤﻌﻤﻤﺔ ‪ p j‬ﻭﻻﺸﻙ ﺃﻥ ﻋﺩﺩ ﺍﻟﻤﻌﺎﺩﻻﺕ ﺍﻟﺘﻔﺎﻀﻠﻴﺔ ﺍﻟﻼﺯﻤﺔ ﻟﻠﺤﺼﻭل‬


‫ﻋﻠﻰ ﻗﺎﻨﻭﻥ ﺍﻟﺤﺭﻜﺔ ﻴﺼﺒﺢ ﻜﺒﻴﺭ ﺍﹰ ﺠﺩ ﺍﹰ ﻭﺘﺯﺩﺍﺩ ﺼﻌﻭﺒﺔ ﺍﻟﺤﺼﻭل ﻋﻠﻰ ﻫﺫﻩ‬
‫ﺍﻟﻤﻌﺎﺩﻻﺕ ﻋﻨﺩﻤﺎ ﻴﺯﺩﺍﺩ ﻋﺩﺩ ﺍﻟﻨﻘﻁ ﺒﺸﻜل ﻫﺎﺌل‪ .‬ﻓﻤﻥ ﻤﻨﺎ ﻤﺴﺘﻌﺩ ﻟﺤل ﻤﻠﻴﻭﻥ‬
‫ﻤﻌﺎﺩﻟﺔ ﺘﻔﺎﻀﻠﻴﺔ ﻤﺜﻼﹰ؟ ﻭﻤﻥ ﺴﻴﺠﺩ ﻋﻨﺩﻩ ﺍﻟﻭﻗﺕ ﻭﺍﻟﻘﺭﻁﺎﺴﻴﺔ ﺍﻟﻜﺎﻓﻴﻴﻥ ﻻﻨﺠﺎﺯ ﻫﺫﻩ‬
‫ﺍﻟﻤﺴﺄﻟﺔ ﺤﺘﻰ ﺒﻤﺴﺎﻋﺩﺓ ﺍﻟﺤﺎﺴﻭﺏ‪ .‬ﺇﻥ ﺍﻟﺘﻔﻜﻴﺭ ﻓﻲ ﺤل ﻫﺫﻩ ﺍﻟﻤﺴﺄﻟﺔ ﻤﻴﻜﺎﻨﻴﻜﻴ ﺎﹰ‬
‫ﻀﺭﺏ ﻤﻥ ﺍﻟﺨﻴﺎل‪ .‬ﻟﺫﺍ ﻜﺎﻥ ﻻﺒﺩ ﻤﻥ ﺍﻟﺒﺤﺙ ﻋﻥ ﻁﺭﻕ ﺃﺨﺭﻯ ﻏﻴﺭ ﺘﻘﻠﻴﺩﻴﺔ‪.‬‬
‫ﻭﻟﺤﺴﻥ ﺍﻟﺤﻅ ﺘﺒﺭﺯ ﻫﻨﺎ ﺤﻘﻴﻘﺔ ﻫﺎﻤﺔ ﻭﻫﻲ ﺃﻥ ﺯﻴﺎﺩﺓ ﻋﺩﺩ ﺍﻟﺠﺴﻴﻤﺎﺕ ﻴﺅﺩﻱ ﺇﻟﻰ‬
‫ﺒﺭﻭﺯ ﻗﺎﻨﻭﻨﻴﺔ ﺨﺎﺼﺔ )ﻭﻟﻨﻘل ﻗﺎﻨﻭﻨﻴﺔ ﺇﺤﺼﺎﺌﻴﺔ( ﺒﺤﻴﺙ ﻴﻤﻜﻥ ﺤل ﺍﻟﻤﺸﻜﻠﺔ ﺒﻁﺭﻕ‬
‫ﺇﺤﺼﺎﺌﻴﺔ ﺘﺨﺘﻠﻑ ﺘﻤﺎﻤ ﺎﹰ ﻋﻥ ﺍﻟﻁﺭﻕ ﺍﻟﺴﺎﺒﻘﺔ ﺍﻟﺘﻲ ﺍﺘﺒﻌﻨﺎﻫﺎ ﺴﺎﺒﻘﺎﹰ‪ .‬ﻭﻤﻥ ﺍﻟﻀﺭﻭﺭﻱ‬
‫ﻟﺤل ﺍﻟﻤﺸﻜﻠﺔ ﺒﺎﻟﻁﺭﻕ ﺍﻟﺠﺩﻴﺩﺓ ﺍﻟﺘﻌﺭﻑ ﻋﻠﻰ ﺒﻌﺽ ﺍﻟﻤﺼﻁﻠﺤﺎﺕ‪.‬‬
‫ﻟﻨﻌﺭﻑ ﺃﻭﻻﹰ ﺍﻟﻔﺭﺍﻍ ﺍﻟﻁﻭﺭﻱ‪ ،‬ﻓﻨﻔﺭﺽ ﻋﺩﺩﺍﹰ ﻤﻥ ﺍﻟﻤﺤﺎﻭﺭ ﺍﻻﺤﺩﺍﺜﻴﺔ ‪q j‬‬

‫ﻭﻋﺩﺩﺍﹰ ﺁﺨﺭ ﻤﻥ ﺍﻟﻤﺤﺎﻭﺭ ‪ p j‬ﺒﺤﻴﺙ ﻨﺤﺼل ﻋﻠﻰ ﻓﺭﺍﻍ ﺍﻗﻠﻴﺩﻱ ﺫﻱ ‪ 2n‬ﺒﻌﺩ ﺍﹰ‬
‫) ‪ (E 2n‬ﻴﺴﻤﻰ ﺍﻟﻔﺭﺍﻍ ﺍﻟﻁﻭﺭﻱ‪ .‬ﺇﻥ ﻭﻀﻊ ﺍﻟﺠﻤﻠﺔ ﺍﻟﻤﺎﺩﻴﺔ ﺍﻟﻤﺩﺭﻭﺴﺔ ﺍﻟﻤﺅﻟﻔﺔ ﻤﻥ‬
‫‪ N‬ﺠﺴﻴﻡ‪ ،‬ﻓﻲ ﻟﺤﻅﺔ ﻤﺎ ‪ t‬ﻴﺘﻌﻴﻥ ﺒﻤﺠﻤﻭﻋﺔ ﻤﻥ ﺍﻹﺤﺩﺍﺜﻴﺎﺕ ﺍﻟﻤﻌﻤﻤﺔ ‪q j‬‬

‫ﻭﺍﻻﻨﺩﻓﺎﻋﺎﺕ ‪ ( j  1, 2, ..., n )p j‬ﺃﻱ ﺒﻨﻘﻁﺔ ﻭﺤﻴﺩﺓ ‪ M‬ﻓﻲ ﻫﺫﺍ ﺍﻟﻔﺭﺍﻍ ‪E 2 n‬‬

‫ﺇﺤﺩﺍﺜﻴﺎﺘﻬﺎ ) ‪ ، (p1, p 2 ,..., p n , q1 , q 2 ,..., q n‬ﺃﻤﺎ ﻭﻀﻊ ﺍﻟﺠﻤﻠﺔ ﻓﻲ ﻟﺤﻅﺔ ﺜﺎﻨﻴﺔ‬


‫) ‪ ( t  t‬ﻓﻴﺘﻌﻴﻥ ﺒﻨﻘﻁﺔ ﺃﺨﺭﻯ ﻤﻘﺎﺒﻠﺔ ‪ 'M‬ﻭﺃﻥ ﺃﻱ ﻭﻀﻊ ﻟﻠﺠﻤﻠﺔ ﻴﺘﻤﺜل ﺒﻨﻘﻁﺔ‬

‫‪- 103 -‬‬

‫)‪Create PDF files without this message by purchasing novaPDF printer (http://www.novapdf.com‬‬
‫ﻭﺤﻴﺩﺓ ﻓﻲ ﻫﺫﺍ ﺍﻟﻔﺭﺍﻍ ﺍﻟﻁﻭﺭﻱ‪ .‬ﻭﻋﻨﺩﻤﺎ ﻴﺘﻐﻴﺭ ﻭﻀﻊ ﺍﻟﺠﻤﻠﺔ ﺨﻼل ﺍﻟﺯﻤﻥ ﻓﺈﻥ‬
‫ﺍﻟﻨﻘﻁﺔ ‪ M‬ﻨﺭﺴﻡ ﻤﺴﺎﺭ ﺍﹰ ‪ ‬ﺘﻘﺎﺒل ﻜل ﻨﻘﻁﺔ ﻓﻴﻪ ﺤﺎﻟﺔ ﻤﻌﻴﻨﺔ ﻤﻥ ﺤﺎﻻﺕ ﺍﻟﺠﻤﻠﺔ‪.‬‬
‫ﻟﻨﺩﺭﺱ ﺍﻵﻥ ﺠﻤﻠﺔ ﻤﺎﺩﻴﺔ ﻤﻌﺯﻭﻟﺔ ﻭﻨﻘﺴﻤﻬﺎ ﺇﻟﻰ ﺠﻤل ﺠﺯﺌﻴﺔ ﺼﻐﻴﺭﺓ ﻻﻴﻤﻜﻥ‬
‫ﺃﻥ ﺘﻜﻭﻥ ﻤﻌﺯﻭﻟﺔ ﻓﻬﻲ ﺩﺍﺌﻤﺔ ﺍﻟﺘﻔﺎﻋل ﻓﻴﻤﺎ ﺒﻴﻨﻬﺎ ﻭﺒﺎﻟﺘﺎﻟﻲ ﻓﺈﻥ ﺍﻟﺤﺎﻟﺔ ﺍﻟﻤﻴﻜﺎﻨﻴﻜﻴﺔ‬
‫ﻟﻠﺠﻤﻠﺔ ﺍﻟﺠﺯﺌﻴﺔ ﻴﺘﻐﻴﺭ ﻓﻲ ﻜل ﻟﺤﻅﺔ‪ .‬ﻭﻴﻤﻜﻥ ﺩﺭﺍﺴﺔ ﺍﻟﺠﻤﻠﺔ ﻜﻜل ﺇﺫﺍ ﺩﺭﺴﻨﺎ ﻜﻼﹰ‬
‫ﻤﻥ ﺠﻤﻠﻬﺎ ﺍﻟﺠﺯﺌﻴﺔ ﻋﻠﻰ ﺤﺩﺓ‪ .‬ﻏﻴﺭ ﺃﻨﻪ ﻤﻥ ﺍﻟﺼﻌﺏ ﺍﻟﻘﻴﺎﻡ ﺒﻬﺫﻩ ﺍﻟﺩﺭﺍﺴﺔ ﻤﻥ ﻭﺠﻬﺔ‬
‫ﻨﻅﺭ ﻤﻴﻜﺎﻨﻴﻜﻴﺔ ﻷﻥ ﺫﻟﻙ ﻴﺘﻁﻠﺏ ﻜﺘﺎﺒﺔ ﻤﻌﺎﺩﻻﺕ ﺍﻟﺤﺭﻜﺔ ﻟﻜل ﻤﻨﻬﺎ ﻭﻫﺫﺍ ﺼﻌﺏ ﺠﺩ ﺍﹰ‬
‫ﻜﻤﺎ ﺃﺴﻠﻔﻨﺎ ﺴﺎﺒﻘﺎﹰ‪ .‬ﻭﻟﻜﻥ ﻫﺫﻩ ﺍﻟﺼﻌﻭﺒﺔ ﻫﻨﺎ ﺘﺘﻀﻤﻥ ﻓﻲ ﻁﻴﺎﺘﻬﺎ ﺒﻌﺽ ﺍﻟﺴﻬﻭﻟﺔ )!(‪.‬‬
‫ﺇﺫ ﺃﻨﻪ ﺒﺤﻜﻡ ﺍﻟﺘﻔﺎﻋل ﺍﻟﻤﻌﻘﺩ ﺍﻟﻌﺸﻭﺍﺌﻲ ﺒﻴﻥ ﺍﻟﺠﻤﻠﺔ ﺍﻟﺠﺯﺌﻴﺔ ﺍﻟﻤﻭﺼﻭﻓﺔ ﻓﻲ ﺍﻟﻔﺭﺍﻍ‬
‫ﺍﻟﻁﻭﺭﻱ ﺒﺎﻹﺤﺩﺍﺜﻴﺎﺕ ‪ q j , p j‬ﻭﻤﺎﻴﺤﻴﻁ ﺒﻬﺎ ﻓﻼﺒﺩ ﻟﻬﺎ ﻤﻥ ﺍﻟﻤﺭﻭﺭ ﺒﺠﻤﻴﻊ ﺤﺎﻻﺘﻬﺎ‬
‫ﺍﻟﻤﻴﻜﺎﻨﻴﻜﻴﺔ ﺍﻟﻤﻤﻜﻨﺔ ﺨﻼل ﻓﺘﺭﺓ ﺯﻤﻨﻴﺔ ﻜﺎﻓﻴﺔ‪ .‬ﻭﺒﻌﺒﺎﺭﺓ ﺃﺨﺭﻯ ﺇﺫﺍ ﺃﺨﺫﻨﺎ ﺤﺠﻤ ﺎﹰ‬
‫ﻋﻨﺼﺭﻴﺎﹰ ‪ pq‬ﻤﻥ ﺍﻟﻔﺭﺍﻍ ﺍﻟﻁﻭﺭﻱ ﺒﺠﻭﺍﺭ ﺍﻟﻨﻘﻁﺔ ) ‪ . M(p j , q j‬ﺍﻟﻤﻭﺍﻓﻘﺔ ﻟﻠﺤﺎﻟﺔ‬
‫ﺍﻟﻤﻴﻜﺎﻨﻴﻜﻴﺔ ﻟﻠﺠﻤﻠﺔ ﺍﻟﺠﺯﺌﻴﺔ ﻓﻲ ﺍﻟﻠﺤﻅﺔ ‪ ،t‬ﻓﻼﺒﺩ ﺃﻥ ﻴﻤﺭ ﺍﻟﻤﺴﺎﺭ ﺍﻟﻁﻭﺭﻱ ﻟﻬﺎ ﻋﺩﺩ ﺍﹰ‬
‫ﻜﺒﻴﺭ ﺍﹰ ﻤﻥ ﺍﻟﻤﺭﺍﺕ ﻓﻲ ﻋﻨﺼﺭ ﺍﻟﺤﺠﻡ ﺍﻟﺴﺎﺒﻕ ﺨﻼل ﺯﻤﻥ ﻜﺎﻑ‪ ،‬ﻟﻨﺭﻤﺯ ﺒـ ‪t‬‬
‫ﻟﻠﺯﻤﻥ ﺍﻟﺫﻱ ﺘﺘﻭﺍﺠﺩ ﻓﻴﻪ ﺍﻟﺠﻤﻠﺔ ﺍﻟﺠﺯﺌﻴﺔ ﻓﻲ ﻋﻨﺼﺭ ﺍﻟﺤﺠﻡ ‪ pq‬ﺨﻼل ﻜل‬
‫ﻋﻨﺩﻤﺎ ‪ T  ‬ﺒﻤﺜﺎﺒﺔ ﺍﺤﺘﻤﺎل‬ ‫ﺍﻟﺯﻤﻥ ‪ .T‬ﻤﻥ ﺍﻟﻭﺍﻀﺢ ﺃﻨﻪ ﻴﻤﻜﻥ ﺍﻋﺘﺒﺎﺭ‬
‫‪t‬‬
‫‪T‬‬
‫ﻭﺠﻭﺩ ﺍﻟﺠﻤﻠﺔ ﺍﻟﺠﺯﺌﻴﺔ ﻓﻲ ﻋﻨﺼﺭ ﺍﻟﺤﺠﻡ ‪ . Pq‬ﻟﻨﺭﻤﺯ ﻟﻬﺫﺍ ﺍﻻﺤﺘﻤﺎل ﺒﺎﻟﺭﻤﺯ‬
‫‪ W‬ﻓﻴﻜﻭﻥ‪:‬‬
‫‪t‬‬
‫‪W‬‬ ‫‪lim‬‬ ‫)‪(3.101‬‬
‫‪T‬‬
‫‪T‬‬
‫ﺃﻤﺎ ﺍﺤﺘﻤﺎل )ﻋﻨﺼﺭ ﺍﻻﺤﺘﻤﺎل( ﺃﻥ ﻴﻜﻭﻥ ﻟﻠﻤﺠﻤﻭﻋﺔ ﺍﻟﺠﺯﺌﻴﺔ ﺍﻨﺩﻓﺎﻋﺎﹰ‬
‫ﻭﺇﺤﺩﺍﺜﻴﺎﺕ ﻤﺤﺼﻭﺭﻴﻥ ﺒﻴﻥ ‪ q j  dq j , p j  dp j‬ﻓﻴﻤﻜﻥ ﺘﻌﺭﻴﻔﻪ ﺒﺎﻟﺸﻜل‪:‬‬
‫‪dW  (p1 , p 2 ,..., p n , q1 , q 2 ,..., q n )dp1 ,..., dp n , dq1,..., dq n‬‬
‫)‪ ( p, q)dp.dq (3.102‬‬

‫‪- 104 -‬‬

‫)‪Create PDF files without this message by purchasing novaPDF printer (http://www.novapdf.com‬‬
‫ﺤﻴﺙ ﻴﻤﺜل ﺍﻟﺘﺎﺒﻊ ) ‪ (p1 ,.., p n , q1 ,..., q n‬ﻜﺜﺎﻓﺔ ﺍﻻﺤﺘﻤﺎل ﻓﻲ ﺍﻟﻔﺭﺍﻍ‬
‫ﺍﻟﻁﻭﺭﻱ ﻭﻴﺴﻤﻰ ﺘﺎﺒﻊ ﺍﻟﺘﻭﺯﻉ ﺍﻻﺤﺼﺎﺌﻲ ﺃﻭ ﺘﺎﺒﻊ ﺍﻟ ﺘﺤﺎﺹ )ﺃﻭ‪ ،‬ﺃﺤﻴﺎﻨﺎﹰ‪ ،‬ﺘﺎﺒﻊ‬
‫ﺍﻟﺘﻭﺯﻉ(‪ ،‬ﻭﺤﻴﺙ ﺭﻤﺯﻨﺎ ﺒﻌﻨﺼﺭ ﺍﻟﺤﺠﻡ ﻓﻲ ﺍﻟﻔﺭﺍﻍ ﺍﻟﻁﻭﺭﻱ ﺒﺎﻟﺭﻤﺯ ‪dpdq‬‬
‫ﻭﻭﻀﻌﻨﺎ ) ‪ (p, q‬ﺒﺩﻻﹰ ﻤﻥ ) ‪ (p1 ,..., p n , q1,..., q n‬ﺇﻥ ﺍﺤﺘﻤﺎل ﺃﻥ ﺘﻤﺭ ﺍﻟﺠﻤﻠﺔ‬
‫ﺍﻟﺠﺯﺌﻴﺔ ﺒﺠﻤﻴﻊ ﺃﻭﻀﺎﻋﻬﺎ ﺍﻟﻤﻤﻜﻨﺔ ﻴﺴﺎﻭﻱ ﺍﻟﻭﺍﺤﺩ ﺃﻱ ﺃﻥ‪:‬‬
‫‪ (p, q)dp.dq  1‬‬ ‫)‪(3.103‬‬
‫ﺤﻴﺙ ﻴﺅﺨﺫ ﺍﻟﺘﻜﺎﻤل ﺍﻟﺴﺎﺒﻕ ﺒﺤﻴﺙ ﻴﺸﻤل ﺠﻤﻴﻊ ﺍﻟﻘﻴﻡ ﺍﻟﻤﻤﻜﻨﺔ ﻟـ ‪ p‬ﻭ ‪.q‬‬
‫ﻭﻴﻌﺒﺭ ﻋﻤﺎ ﻴﺴﻤﻰ ﺸﺭﻁ ﺘﻨﻅﻴﻡ ﺍﻟﺘﺎﺒﻊ ‪.‬‬
‫ﺘﻌﺘﺒﺭ ﻤﻌﺭﻓﺔ ﺘﺎﺒﻊ ﺍﻟﺘﻭﺯﻉ ) ‪ (p, q‬ﺃﺴﺎﺴﻴﺔ ﺠﺩ ﺍﹰ ﻟﺤل ﺍﻟﻤﺸﻜﻠﺔ‪ ،‬ﻷﻨﻨﺎ ﻨﺴﺘﻁﻴﻊ‬
‫ﺒﻭﺴﺎﻁﺘﻪ ﺤﺴﺎﺏ ﻤﺘﻭﺴﻁ ﺃﻱ ﻗﻴﻤﺔ ﻓﻴﺯﻴﺎﺌﻴﺔ )‪ f (p, q‬ﻤﺘﻌﻠﻘﺔ ﺒﺎﻟﺠﻤﻠﺔ ﻤﻬﻤﺎ ﻜﺎﻨﺕ‬
‫ﻫﺫﻩ ﺍﻟﻘﻴﻤﺔ‪ ،‬ﺒﺎﻟﻌﻼﻗﺔ‪:‬‬
‫‪f (p, q)   f (p, q).(p, q )dp.dq‬‬ ‫)‪(3.104‬‬
‫ﻫﺫﺍ ﻭﻗﺩ ﺘﻜﻭﻥ ‪ f‬ﻫﻲ ﺍﻟﻁﺎﻗﺔ ﺃﻭ ﻜﻤﻴﺔ ﺍﻟﺤﺭﻜﺔ ﺃﻭ ﺍﻟﻌﺯﻡ ﺍﻟﺤﺭﻜﻲ ﺃﻭ ﺃﻱ ﻜﻤﻴﺔ‬
‫ﺘﺸﺘﻕ ﻤﻨﻬﺎ‪.‬‬
‫‪ - 27‬ﻧﻈﺮﻳﺔ ﻟﻴﻮﻓﻴﻞ‪:‬‬
‫ﻟﻨﺄﺨﺫ ﺠﻤﻠﺔ ﺠﺯﺌﻴﺔ ﻭﻟﻨﺭﺼﺩ ﺍﻷﻭﻀﺎﻉ ﺍﻟﺘﻲ ﺘﺸﻐﻠﻬﺎ ﻓﻲ ﺍﻟﻔﺭﺍﻍ ﺍﻟﻁﻭﺭﻱ‬
‫ﺨﻼل ﻓﺘﺭﺓ ﺯﻤﻨﻴﺔ ﻜﺒﻴﺭﺓ ﺠﺩ ﺍﹰ‪ .‬ﻭﻟﻨﺭﻤﺯ ﻟﻠﻨﻘﻁ ﺍﻟﻤﻘﺎﺒﻠﺔ ﺍﻟﻤﻤﺜﻠﺔ ﻷﻭﻀﺎﻉ ﻫﺫﻩ ﺍﻟﺠﻤﻠﺔ‬
‫ﺍﻟﺠﺯﺌﻴﺔ ﺒﺎﻟﺭﻤﺯ ‪ A1, A 2 ,....‬ﻤﻭﺍﻓﻘﺔ ﻟﻠﺤﻅﺎﺕ ‪ t 1, t 2 ,...‬ﺃﻥ ﺘﻭﺯﻉ ﺍﻟﻨﻘﻁ ‪ Ai‬ﻓﻲ‬
‫ﺍﻟﻔﺭﺍﻍ ﺍﻟﻁﻭﺭﻱ ﻴﺘﻨﺎﺴﺏ ﻤ ﻊ ﻗﻴﻤﺔ ﺍﻟﺘﺎﺒﻊ ‪ .‬ﻜﻴﻑ ﺘﺼﺒﺢ ﺍﻟﺼﻭﺭﺓ ﻋﻨﺩﻤﺎ ﻻﻨﺄﺨﺫ‬
‫ﺠﻤﻠﺔ ﺠﺯﺌﻴﺔ ﻓﻘﻁ ﻭﺇﻨﻤﺎ ﺘﺄﺨﺫ ﻜل ﺍﻟﺠﻤﻠﺔ ﺍﻟﺘﻲ ﻟﺩﻴﻨﺎ ﺨﻼل ﻟﺤﻅﺔ ﺯﻤﻨﻴﺔ ﻤﻌﻴﻨﺔ؟‬
‫ﻴﻤﻜﻥ ﺍﻟﺘﺄﻜﻴﺩ ﺃﻥ ﺍﻟﺠﻤﻠﺔ ﻜﻜل ﺘﺸﻐل ﺍﻟﻨﻘﻁ ﺍﻟﺴﺎﺒﻘﺔ ‪ A1, A 2 ,....‬ﻓﻲ ﺃﻱ ﻟﺤﻅﺔ ‪.t‬‬
‫ﻟﻨﺒﺤﺙ ﻋﻥ ﺃﻭﻀﺎﻉ ﺍﻟﻨﻘﻁ ‪ A1, A 2 ,....‬ﺨﻼل ﺯﻤﻥ ﻗﺼﻴﺭ ‪ ، t‬ﻭﻴﻤﻜﻥ ﻓﻴﻪ ﺇﻫﻤﺎل‬
‫ﺘﻔﺎﻋل ﺍﻟﺠﻤل ﺍﻟﺠﺯﺌﻴﺔ ﺒﻌﻀﻬﺎ ﻤﻊ ﺒﻌﺽ‪ ،‬ﻭﺒﺎﻟﺘﺎﻟﻲ ﻓﺈﻨﻬﺎ ﺴﺘﺘﺤﺭﻙ ﻁﺒﻘﺎﹰ ﻟﻘﻭﺍﻨﻴﻥ‬
‫ﺍﻟﻤﻴﻜﺎﻨﻴﻙ‪ ،‬ﻭﻫﻲ ﺴﺘﺘﻭﺯﻉ ﻓﻲ ﺍﻟﻔﺭﺍﻍ ﺃﺜﻨﺎﺀ ﻫﺫﻩ ﺍﻟﺤﺭﻜﺔ ﺒﺸﻜل ﻤﺸﺎﺒﻪ ﺘﻤﺎﻤ ﺎﹰ ﻟﻠﺤﻅﺔ‬
‫‪ ،t‬ﻭﻤﺘﻭﺍﻓﻕ ﻤﻊ ﻗﻴﻡ ﺘﺎﺒﻊ ﺍﻟﺘﻭﺯﻉ ‪ ‬ﻓﻲ ﻜل ﻨﻘﻁﺔ‪ .‬ﻭﻫﻜﺫﺍ ﻴﻤﻜﻥ ﺍﻋﺘﺒﺎﺭ ﻫﺫﻩ ﺍﻟﺤﺭﻜﺔ‬

‫‪- 105 -‬‬

‫)‪Create PDF files without this message by purchasing novaPDF printer (http://www.novapdf.com‬‬
‫ﻜﺄﻨﻬﺎ ﺤﺭﻜﺔ "ﻏﺎﺯ" ﻤﺎ ﻓﻲ ﺍﻟﻔﺭﺍﻍ ﺍﻟﻁﻭﺭﻱ ﺘﻠﻌﺏ ﻓﻴﻬﺎ ﺍﻟﻨﻘﻁ ‪ A1, A 2 ,....‬ﺩﻭﺭ‬
‫ﺠﺯﻴﺌﺎﺕ ﺍﻟﻐﺎﺯ‪ ،‬ﺜﻡ ﺘﻁﺒﻕ ﻤﻌﺎﺩﻟﺔ ﺍﻻﺴﺘﻤﺭﺍﺭ ﺍﻟﻤﻌﺭﻭﻓﺔ ﺍﻟﺘﺎﻟﻴﺔ ﻋﻠﻴﻪ‪ ،‬ﻭﻫﻲ‪:‬‬
‫‪‬‬ ‫‪‬‬
‫‪ div(v)  0‬‬ ‫)‪(3.105‬‬
‫‪t‬‬
‫‪‬‬
‫ﺤﻴﺙ ‪ ‬ﻫﻲ ﺍﻟﻜﺜﺎﻓﺔ ﻭ ‪ v‬ﺍﻟﺴﺭﻋﺔ‪ .‬ﻭﻓﻲ ﺤﺎﻟﺔ ﺍﻟﺠﺭﻴﺎﻥ ﺍﻟﻤﺴﺘﻘﺭ ﻴﻜﻭﻥ‬
‫ﻭﻋﻨﺩﺌﺫ‪ ‬ﺘﻭﻀﻊ ﺍﻟﻌﻼﻗﺔ ﺍﻟﺴﺎﺒﻘﺔ ﺒﺎﻟﺸﻜل‪:‬‬
‫‪‬‬
‫‪0‬‬
‫‪t‬‬
‫‪‬‬
‫‪div ( v )  0‬‬ ‫)‪(3.106‬‬
‫‪‬‬
‫ﻟﻨﺤﺴﺏ ﺘﻔﺭﻕ ﺍﻟﻤﺘﺠﻪ ‪  v‬ﻓﻲ ﻓﺭﺍﻍ ﺫﻱ ‪ 2n‬ﺒﻌﺩﺍﹰ‪ ،‬ﻭﻫﻭ ﻴﺴﺎﻭﻱ ﻁﺒﻘﺎﹰ‬
‫ﻟﺘﻌﺭﻴﻑ ﺍﻟﺘﻔﺭﻕ‪:‬‬
‫‪‬‬ ‫‪2n‬‬
‫‪‬‬
‫‪div (. v )  ‬‬ ‫) ‪(v j‬‬ ‫)‪(3.107‬‬
‫‪j 1‬‬
‫‪‬‬‫‪x‬‬ ‫‪j‬‬

‫ﺤﻴﺙ ‪ v j‬ﻫﻲ ﻤﺭﻜﺒﺎﺕ ﺍﻟﺴﺭﻋﺔ ﻓﻲ ﺍﻟﻔﺭﺍﻍ ﺍﻟﻁﻭﺭﻱ ﻭﻫﻲ ﺘﺴﺎﻭﻱ ‪ P j‬ﺃﻭ‬


‫‪ q j‬ﺘﺒﻌﺎﹰ ﻟﻠﻤﺤﻭﺭ ﺍﻟﺘﻲ ﻨﺴﻘﻁ ﻋﻠﻴﻪ‪ .‬ﻭﻫﻜﺫﺍ ﻨﺭﻯ ﺃﻥ )‪ (3.107‬ﺘﻭﻀﻊ ﺒﺎﻟﺸﻜل‪:‬‬
‫‪n‬‬ ‫‪ ‬‬ ‫‪‬‬ ‫‪‬‬
‫‪‬‬ ‫‪‬‬ ‫‪(q j ) ‬‬ ‫‪(Pj ) ‬‬
‫‪j1‬‬ ‫‪ q j‬‬ ‫‪P j‬‬ ‫‪‬‬
‫‪      n  q j p j ‬‬
‫‪ ‬‬ ‫‪p  ‬‬
‫‪ q j j P j j    q j p j ‬‬
‫‪q ‬‬ ‫‪‬‬ ‫)‪(3.108‬‬
‫‪‬‬ ‫‪ j 1 ‬‬ ‫‪‬‬
‫ﻭﻤﻥ ﻤﻌﺎﺩﻻﺕ ﻫﺎﻤﻠﺘﻭﻥ )‪ (3.10‬ﻨﺠﺩ‪:‬‬
‫‪q j‬‬ ‫‪2H‬‬ ‫‪p j‬‬ ‫‪2H‬‬
‫‪‬‬ ‫‪,‬‬ ‫‪‬‬ ‫)‪(3.109‬‬
‫‪q j  j q j p j‬‬ ‫‪q j p j‬‬

‫ﻭﺒﺘﻌﻭﻴﺽ )‪ (3.109‬ﻓﻲ )‪ (3.108‬ﻨﺤﺼل ﻋﻠﻰ ﺍﻟﻌﻼﻗﺔ ﺍﻟﻬﺎﻤﺔ ﺍﻟﺘﺎﻟﻴﺔ‪:‬‬


‫‪d‬‬ ‫‪     ‬‬
‫‪ ‬‬ ‫‪q ‬‬ ‫‪p 0‬‬ ‫)‪(3.110‬‬
‫‪dt‬‬ ‫‪ q j j p j j ‬‬
‫‪‬‬ ‫‪‬‬
‫ﻭﻫﻲ ﺍﻟﺘﻌﺒﻴﺭ ﺍﻟﺭﻴﺎﻀﻲ ﻋﻤﺎ ﻴﺴﻤﻰ ﻨﻅﺭﻴﺔ ﻟﻴﻭﻓﻴل‪ .‬ﻭﺘﻌﻨﻲ ﺃﻥ ﺘﺎﺒﻊ ﺍﻟﺘﻭﺯﻉ‬
‫ﺍﻹﺤﺼﺎﺌﻲ ﻴﺒﻘﻰ ﺜﺎﺒﺘﺎﹰ ﻋﻠﻰ ﺍﻟﻤﺴﺎﺭﺍﺕ ﺍﻟﻁﻭﺭﻴﺔ ﻟﻠﺠﻤل ﺍﻟﺠﺯﺌﻴﺔ‪ .‬ﻭﻴﺠﺏ ﺍﻟﺘﻨﻭﻴﻪ ﺃﻨﻨﺎ‬

‫‪- 106 -‬‬

‫)‪Create PDF files without this message by purchasing novaPDF printer (http://www.novapdf.com‬‬
‫ﺤﺼﻠﻨﺎ ﻋﻠﻰ ﻫﺫﻩ ﺍﻟﻨﺘﻴﺠﺔ ﺒﻔﺭﺽ ﺃﻥ ﺍﻟﺠﻤل ﺍﻟﺠﺯﺌﻴﺔ ﺍﻟﻤﺨﺘﻠﻔﺔ‪ ،‬ﺍﻟﺘﻲ ﺘﺘﺄﻟﻑ ﻤﻨﻬ ﺎ‬
‫ﺍﻟﺠﻤﻠﺔ ﺍﻟﻤﺩﺭﻭﺴﺔ‪ ،‬ﻻﺘﺘﻔﺎﻋل ﺒﻤﺎ ﺒﻴﻨﻬﺎ ﺨﻼل ﺍﻟﺯﻤﻥ ﺍﻟﺫﻱ ﻴﻤﻜﻥ ﻓﻴﻪ ﺍﻋﺘﺒﺎﺭ ﺘﺎﺒﻊ‬
‫ﺍﻟﺘﻭﺯﻉ ﺍﻹﺤﺼﺎﺌﻲ ) ‪ (p, q‬ﺜﺎﺒﺘﺎﹰ‪.‬‬

‫‪- 107 -‬‬

‫)‪Create PDF files without this message by purchasing novaPDF printer (http://www.novapdf.com‬‬
‫ﲤﺎﺭﻳﻦ ﻭﻣﺴﺎﺋﻞ‬

:‫ ﺒﺭﻫﻥ ﺼﺤﺔ ﺍﻟﻌﻼﻗﺔ‬f (p, q) ‫ ﻟﻴﻜﻥ ﺍﻟﺘﺎﺒﻊ ﺍﻟﺴﻠﻤﻲ‬- 1


f , L z   0

. oz ‫ ﻋﻠﻰ ﺍﻟﻤﺤﻭﺭ‬L ‫ ﻫﻭ ﻤﺴﻘﻁ ﺍﻟﻌﺯﻡ ﺍﻟﺤﺭﻜﻲ‬L z ‫ﺤﻴﺙ‬
: ‫ﺍﳊﻞ‬
‫ﻭﺍﻻﻨﺩﻓﺎﻋﺎﺕ ﺍﻟﻤﻌﻤﻤﺔ‬ ‫ﺘﺎﺒﻌﺎﹰ ﻟﻺﺤﺩﺍﺜﻴﺎﺕ ﺍﻟﻤﻌﻤﻤﺔ‬ f ‫ﻨﻔﺭﺽ ﺃﻥ‬
 ‫ ﺒﺯﺍﻭﻴﺔ ﻋﻨﺼﺭﻴﺔ‬oz ‫ ﻭﻨﻘﻭﻡ ﺒﺩﻭﺭﺍﻥ ﺤﻭل ﺍﻟﻤﺤﻭﺭ‬f (x, y, z, p x , p y , p z )

:‫ ﺇﻟﻰ ﺍﻟﺼﻴﻐﺔ‬f ‫ﺘﺘﻐﻴﺭ ﻓﻴﻪ ﺍﻻﻨﺩﻓﺎﻋﺎﺕ ﻭﺍﻹﺤﺩﺍﺜﻴﺎﺕ ﺒﺤﻴﺙ ﻴﺘﺤﻭل ﺍﻟﺘﺎﺒﻊ‬


:‫ ﻓﻨﺠﺩ‬f ‫ ﺜﻡ ﻨﻨﺸﺭ‬f (x  x, y  y, z, p x  p x , p y  p y , p z )
f (x  x, y  y, z, p x  p x , p y  p y , p z )  f ( x, y, z, p x , p y , p z ) 
f f f f
x  y  p x  p y (1)
x y p x p y

:‫ﺤﻴﺙ‬
d d
x   y, y  x, p x  m x  m x 
dt dt
d
m ( y)  p y  ; p y  p x  (2)
dt
:‫ﺃﻱ ﺃﻥ‬
 f f f f 
f    y x py  p y   0 (3)
 x y p x Py 
 
:‫ﻓﺈﺫﺍ ﻋﻠﻤﻨﺎ ﺃﻥ‬
L z L z
L z  xp y  yp x   py ,   y.... (4)
x Px
:‫( ﻜﻤﺎﻴﻠﻲ‬3) ‫ﻓﻴﻤﻜﻥ ﻭﻀﻊ‬

- 108 -

Create PDF files without this message by purchasing novaPDF printer (http://www.novapdf.com)
 f L z f L z f L z f L z 
f       
 p x x p y y x p x y p y 
 
. L z , f   0  L z , f   0 :‫ﻭﻤﻨﻪ‬
.‫ﻭﻫﻭ ﺍﻟﻤﻁﻠﻭﺏ‬

:‫ ﺒﺭﻫﻥ ﺃﻥ ﺍﻟﻤﻌﺎﺩﻻﺕ ﺍﻟﺘﺎﻟﻴﺔ‬- 2


H H
q i  , p i  
p i q i
.‫ﺘﻜﺎﻓﻰﺀ ﻤﻌﺎﺩﻻﺕ ﻻﻏﺭﺍﻨﺞ‬
: ‫ﺍﳊﻞ‬
:H ‫ﻟﺩﻴﻨﺎ ﺤﺴﺏ ﺘﻌﺭﻴﻑ‬
H   p jq j  L
n  
H q j L L q j L
 p j   
q i i 1 q j q i q j q i q i

:‫ﻓﻨﺠﺩ‬ ‫ﻭﻤﻥ ﻤﻌﺎﺩﻻﺕ ﻻﻏﺭﺍﻨﺞ ﻨﺤﺴﺏ‬


L
q i
L d L d
  p i  p i

q i dt q i dt

:‫ﻓﻨﺠﺩ‬ ‫ﻭﺒﺎﻟﻁﺭﻴﻘﺔ ﻨﻔﺴﻬﺎ ﻨﺤﺴﺏ‬


H
p i

H  q j L q j

 qi   p j   q i
p i  pi 
i q j p i

.‫ﻭﻫﻭ ﺍﻟﻤﻁﻠﻭﺏ‬

‫ ﺃﻭﺠﺩ ﻤﻌﺎﺩﻟﺔ ﺤﺭﻜﺔ ﺠﺴﻴﻡ ﺇﺫﺍ ﻋﻠﻤﺕ ﺃﻥ ﺘﺎﺒﻊ ﻫﺎﻤﻠﺘﻭﻥ ﻟﻪ )ﺸﻌﺎﻉ‬- 3


:(‫ﺍﻟﻀﻭﺀ‬

- 109 -

Create PDF files without this message by purchasing novaPDF printer (http://www.novapdf.com)
‫‪‬‬
‫‪ ‬‬ ‫‪c / p/‬‬
‫‪H( p , r ) ‬‬ ‫)‪(1‬‬
‫‪ ‬‬
‫) ‪n( p , r‬‬
‫ﺤﻴﺙ ‪ c‬ﻫﻲ ﺴﺭﻋﺔ ﺍﻟﻀﻭﺀ ﻭ ‪ n‬ﻗﺭﻴﻨﺔ ﺍﻨﻜﺴﺎﺭ ﺍﻟﻭﺴﻁ ﺍﻟﺫﻱ ﻴﺘﺤﺭﻙ ﻓﻴﻪ‬
‫ﺍﻟﺠﺴﻴﻡ‪.‬‬
‫ﻣﻼﺣﻈﺔ‪:‬‬
‫ﺇﻥ ﺘﺎﺒﻊ ﻫﺎﻤﻠﺘﻭﻥ ﻟﺠﺴﻴﻡ ﻨﺴﺒﻭﻱ ﻫﻭ ﺍﻟﺘﺎﻟﻲ‪:‬‬
‫)‪H  p 2 c 2  m 2 c 4  V(q‬‬
‫ﻭﻓﻲ ﺤﺎﻟﺔ ﺍﻨﻌﺩﺍﻡ ﺍﻟﻜﻤﻭﻥ ﻭﺍﻨﻌﺩﺍﻡ ﺍﻟﻜﺘﻠﺔ )ﻓﻭﺘﻭﻥ( ﻭﻓﺭﺽ ﺃﻥ ﺸﻌﺎﻉ ﺍﻟﻀﻭﺀ‬
‫‪ ‬‬
‫ﻴﻨﺘﺸﺭ ﻓﻲ ﺍﻟﺨﻼﺀ ﻴﻜﻭﻥ ‪ n( p , r )  1‬ﻭﻴﺘﺤﻭل ﺘﺎﺒﻊ ﻫﺎﻤﻠﺘﻭﻥ ﺇﻟﻰ ﺍﻟﺸﻜل‬
‫ﺍﻟﻤﻌﺭﻭﻑ‪:‬‬
‫‪H  pc‬‬
‫ﺍﳊﻞ ‪:‬‬
‫ﻴﻤﻜﻥ ﻭﻀﻊ ﺘﺎﺒﻊ ﻫﺎﻤﻠﺘﻭﻥ )‪ (1‬ﺒﺎﻟﺼﻴﻐﺔ‪:‬‬
‫‪‬‬
‫) ‪H  c p 2x  p 2y  p 2z / n( p , r‬‬

‫ﻭﻴﻜﻭﻥ‪:‬‬
‫‪H‬‬ ‫‪cp x‬‬ ‫‪c / p / n‬‬
‫‪ x ‬‬ ‫‪‬‬ ‫‪....‬‬
‫‪‬‬
‫‪p x‬‬ ‫‪n 2 p x‬‬
‫‪n / p/‬‬
‫ﻨﻌﻤﻡ ﻫﺫﻩ ﺍﻟﻌﻼﻗﺔ ﻓﻲ ﺍﻟﻔﺭﺍﻍ ﻓﻨﺠﺩ‪:‬‬
‫‪‬‬
‫‪H‬‬ ‫‪‬‬‫‪cp‬‬ ‫‪c / p / n‬‬
‫‪ r ‬‬ ‫‪‬‬ ‫)‪(2‬‬
‫‪‬‬ ‫‪n/p/‬‬ ‫‪n2 ‬‬
‫‪p‬‬ ‫‪p‬‬
‫ﻨﺸﺘﻕ ﺒﺎﻟﻨﺴﺒﺔ ﻟـ ‪ x‬ﻭ ‪ y‬ﻭ ‪ z‬ﻓﻨﺠﺩ ﺒﺴﻬﻭﻟﺔ ﻤﻌﺎﺩﻟﺔ ﻫﺎﻤﻠﺘﻭﻥ ﺍﻟﺜﺎﻨﻴﺔ ﺍﻟﺘﺎﻟﻴﺔ‪:‬‬
‫‪‬‬ ‫‪cp n‬‬
‫‪p ‬‬ ‫)‪(3‬‬
‫‪n2‬‬ ‫‪‬‬
‫‪r‬‬

‫‪- 110 -‬‬

‫)‪Create PDF files without this message by purchasing novaPDF printer (http://www.novapdf.com‬‬
‫‪‬‬
‫ﻭﺇﺫﺍ ﻓﺭﻀﻨﺎ ﺃﻥ ﺍﻟﻀﻭﺀ ﻴﻨﺘﺸﺭ ﻓﻲ ﺍﻟﺨﻼﺀ ﻋﻠﻰ ﻤﺤﻭﺭ ‪ ‬ﻤﺘﺠﻪ ﻭﺍﺤﺩﺘﻪ ‪u‬‬
‫‪‬‬
‫‪p ‬‬
‫ﻭﻋﻨﺩﺌﺫ‪ ‬ﺘﺠﺩ ﺃﻥ ﺴﺭﻋﺔ ﺍﻟﻀﻭﺀ )ﺍﻟﻔﻭﺘﻭﻥ( ﻫﻲ‬ ‫ﻴﻜﻭﻥ ‪ n  1‬ﻭ ‪ u‬‬
‫‪p‬‬
‫‪. r'  v  C‬‬

‫‪ - 4‬ﺍﻜﺘﺏ ﻤﻌﺎﺩﻻﺕ ﻫﺎﻤﻠﺘﻭﻥ ﻟﺠﺴﻴﻤﻴﻥ ﻴﺘﻔﺎﻋﻼﻥ ﻓﻴﻤﺎ ﺒﻴﻨﻬﻤﺎ ﺒﻭﺍﺴﻁﺔ ﺠﻬﺩ‬


‫)ﻜﻤﻭﻥ( ﻤﺭﻜﺯﻱ ) ‪ V(r‬ﺒﺈﻫﻤﺎل ﺘﺄﺜﻴﺭ ﺍﻟﻘﻭﻯ ﺍﻟﺨﺎﺭﺠﻴﺔ‪.‬‬

‫‪ - 5‬ﺘﻌﻠﻕ ﻋﺎﺭﻀﺔ ﺜﻘﻴﻠﺔ ﻜﺘﻠﺘﻬﺎ ‪ m‬ﻓﻲ ﻨﻘﻁﺔ ﺜﺎﺒﺘﺔ ﻋﻥ ﻁﺭﻴﻕ ﻨﺎﺒﺽ ﻗﺴﺎﻭﺘﻪ‬
‫‪ k‬ﺸﻜل )‪ (3.2‬ﺒﺤﻴﺙ ﻴﻤﻜﻨﻬﺎ ﺍﻟﺘﺤﺭﻙ ﻓﻲ ﻤﺴﺘﻭﹴ ﺸﺎﻗﻭﻟﻲ ﻓﻘﻁ ﻭﻴﺘﺤﺭﻙ ﺍﻟﻨﺎﺒﺽ‬
‫ﻋﻠﻰ ﺍﻟﻤﺤﻭﺭ ﺍﻟﺸﺎﻗﻭﻟﻲ ﻓﻘﻁ‪ .‬ﺍﻜﺘﺏ ﻤﻌﺎﺩﻻﺕ ﻫﺎﻤﻠﺘﻭﻥ ﻟﻬﺫﻩ ﺍﻟﺠﻤﻠﺔ‪.‬‬

‫ﺍﻟﺸﻜﻞ )‪(3.2‬‬

‫‪ - 6‬ﺃﻭﺠﺩ ﻤﻌﺎﺩﻻﺕ ﻫﺎﻤﻠﺘﻭﻥ ﻟﻠﺩﻭﺍﻤﺔ )ﺭﺍﺠﻊ ﺍﻟﻔﺼل ﺍﻟﺴﺎﺩﺱ(‪.‬‬

‫‪ - 7‬ﻴﺩﻭﺭ ﺴﻠﻙ ﺩﺍﺌﺭﻱ ﻤﺭﻜﺯﻩ ‪ O‬ﻓﻲ ﻤﺴﺘﻭﻴﻪ ﺍﻷﻓﻘﻲ ﺤﻭل ﻨﻘﻁﺔ ﺜﺎﺒﺘﺔ ‪A‬‬
‫ﻤﻥ ﻤﺤﻴﻁﻪ ﺒﻴﻨﻤﺎ ﺘﺘﺤﺭﻙ ﺤﻠﻘﺔ ‪ p‬ﻋﻠﻰ ﻫﺫﺍ ﺍﻟﺴﻠﻙ ﺒﺩﻭﻥ ﺍﺤﺘﻜﺎﻙ‪.‬‬
‫ﺃﻭﺠﺩ ﻗﺎﻨﻭﻥ ﺤﺭﻜﺔ ﺍﻟﺤﻠﻘﺔ ﺍﻋﺘﻤﺎﺩﹰﺍ ﻋﻠﻰ ﻤﻌﺎﺩﻻﺕ ﻫﺎﻤﻠﺘﻭﻥ‪.‬‬

‫‪- 111 -‬‬

‫)‪Create PDF files without this message by purchasing novaPDF printer (http://www.novapdf.com‬‬
‫‪ - 8‬ﻴﺩﻭﺭ ﺴﻠﻙ ﺩﺍﺌﺭ ﻗﻁﺭﻩ ‪ a‬ﻭﻜﺘﻠﺘﻪ ‪ m2‬ﺤﻭل ﻤﺤﻭﺭ ﻴﻨﻁﺒﻕ ﻋﻠﻰ ﻗﻁﺭﻩ‬
‫ﺍﻟﺸﺎﻗﻭﻟﻲ ﺍﻟﺜﺎﺒﺕ ﺒﻴﻨﻤﺎ ﺘﺘﺤﺭﻙ ﻋﻠﻴﻪ ﺩﻭﻥ ﺍﺤﺘﻜﺎﻙ ﺤﻠﻘﺔ ‪ p‬ﻜﺘﻠﺘﻬﺎ ‪ m‬ﺒﺭﻫﻥ ﺃﻥ ﺘﺎﺒﻊ‬
‫ﻫﺎﻤﻠﺘﻭﻥ ﻟﻠﺠﻤﻠﺔ ﻫﻭ ﺍﻟﺘﺎﻟﻲ‪:‬‬
‫‪‬‬ ‫‪1‬‬‫‪2‬‬ ‫‪p 2 ‬‬
‫‪H‬‬ ‫‪‬‬ ‫‪p ‬‬ ‫‪  mga cos ‬‬
‫‪2  ‬‬ ‫‪2 ‬‬
‫‪2ma ‬‬ ‫‪1  sin  ‬‬
‫‪‬‬
‫ﺤﻴﺙ ‪ ‬ﻫﻲ ﺍﻟﺯﺍﻭﻴﺔ ﺍﻟﺘﻲ ﻴﺼﻨﻌﻬﺎ ‪ Op‬ﻤﻊ ‪ ‬ﻭ ‪ ‬ﺯﺍﻭﻴﺔ ﺩﻭﺭﺍﻥ ﻤﺴﺘﻭﻱ‬
‫ﺍﻟﺴﻠﻙ‪.‬‬

‫‪ - 9‬ﺍﻭﺠﺩ ﻗﺎﻨﻭﻥ ﺤﺭﻜﺔ ﺠﺴﻴﻡ ﺇﺫﺍ ﺃﻋﻁﻲ ﺘﺎﺒﻊ ﻫﺎﻤﻠﺘﻭﻥ ﻟﻪ ﺒﺎﻟﻌﻼﻗﺔ‪:‬‬


‫‪p 2  20 x 2‬‬ ‫‪ p 2 2 2 ‬‬
‫‪H ( x, p) ‬‬ ‫‪‬‬ ‫‪ ‬‬ ‫‪ 0x ‬‬
‫‪2‬‬ ‫‪2‬‬ ‫‪ 2‬‬ ‫‪2‬‬ ‫‪‬‬
‫‪‬‬ ‫‪‬‬

‫‪ - 10‬ﺍﺤﺴﺏ ﺘﺎﺒﻊ ﻫﺎﻤﻠﺘﻭﻥ ﻟﺠﺴﻴﻡ ﺇﺫﺍ ﻋﻠﻤﺕ ﺃﻥ ﺘﺎﺒﻊ ﻻﻏﺭﺍﻨﺞ ﻟﻪ ﻫﻭ‪:‬‬


‫‪2‬‬ ‫‪2 2‬‬
‫‪x‬‬ ‫‪ x‬‬ ‫‪3‬‬ ‫‪2‬‬
‫‪L‬‬ ‫‪‬‬ ‫‪ x   xx‬‬
‫‪2‬‬ ‫‪2‬‬

‫‪ - 11‬ﺍﺤﺴﺏ ﺃﻗﻭﺍﺱ ﺒﻭﺍﺼﻭﻥ ﺍﻟﺘﺎﻟﻴﺔ‪:‬‬


‫‪L i , L j ‬‬ ‫‪,‬‬ ‫‪L i , p j ‬‬ ‫‪,‬‬ ‫ﺃ‪Li , x j  -‬‬
‫‪   ‬‬ ‫‪  ‬‬ ‫‪   ‬‬
‫‪ a p , b r  ,‬‬ ‫‪a L , b r  ,‬‬ ‫ﺏ ‪a L , b p   -‬‬
‫‪ ‬‬ ‫‪‬‬ ‫‪‬‬ ‫‪‬‬ ‫‪‬‬ ‫‪ ‬‬
‫‪  ‬‬ ‫‪ n ‬‬ ‫‪‬‬ ‫‪2‬‬
‫‪ L, r . p ‬‬ ‫‪,‬‬ ‫‪ p,r  ,‬‬ ‫ﺝ‪ p , (ar)  -‬‬
‫‪‬‬ ‫‪‬‬ ‫‪‬‬ ‫‪‬‬ ‫‪‬‬ ‫‪‬‬
‫ﺤﻴﺙ ‪ b,a‬ﺜﺎﺒﺘﺎﻥ ﻭ ‪ L i , p i , x i‬ﻫﻲ ﺍﻟﻤﺭﻜﺒﺎﺕ ﺍﻟﺩﻴﻜﺎﺭﺘﻴﺔ ﻟﻜل ﻤﻥ‬
‫‪  ‬‬
‫‪ L , p , r‬ﻋﻠﻰ ﺍﻟﺘﺭﺘﻴﺏ‪.‬‬

‫‪‬‬ ‫‪‬‬
‫‪ - 12‬ﺍﺤﺴﺏ ﺃﻗﻭﺍﺱ ﺒﻭﺍﺼﻭﻥ ‪ A i , A j‬ﺤﻴﺙ‪:‬‬

‫‪- 112 -‬‬

‫)‪Create PDF files without this message by purchasing novaPDF printer (http://www.novapdf.com‬‬
‫‪1 2‬‬ ‫‪1‬‬
‫‪A1 ‬‬ ‫) ‪(x  p 2x  y 2  p 2y ), A 2  (xy  p x p y‬‬
‫‪4‬‬ ‫‪2‬‬
‫) ‪A 4  x 2  y 2  p 2x  p 2y , A 3  xp y  ypx‬‬

‫‪‬‬ ‫‪‬‬ ‫‪ ‬‬


‫‪ - 14‬ﺒﺭﻫﻥ ﺃﻥ ‪ L z , f   f  k‬ﺤﻴﺙ ‪ f‬ﻫﻭ ﺘﺎﺒﻊ ﺸﻌﺎﻋﻲ ﻭ ‪ k‬ﻤﺘﺠﻪ‬
‫‪‬‬ ‫‪‬‬
‫ﺍﻟﻭﺍﺤﺩﺓ ﻋﻠﻰ ‪.oz‬‬
‫ﺘﻭﺠﻴﻪ‪ :‬ﺍﺴﺘﻔﺩ ﻤﻥ ﺍﻟﻤﺴﺄﻟﺔ ﺍﻟﻤﺤﻠﻭﻟﺔ )‪.(1‬‬

‫‪ ‬‬
‫‪ - 15‬ﺍﺤﺴﺏ ﺃﻗﻭﺍﺱ ﺒﻭﺍﺼﻭﻥ ‪  f , a L ‬ﺤﻴﺙ ‪ f‬ﺃﻱ ﺘﺎﺒﻊ ﻟﻺﺤﺩﺍﺜﻴﺎﺕ‬
‫‪‬‬ ‫‪‬‬
‫‪‬‬
‫ﻭﺍﻻﻨﺩﻓﺎﻉ ‪ a‬ﺜﺎﺒﺕ‪ L ،‬ﺍﻟﻌﺯﻡ ﺍﻟﺤﺭﻜﻲ‪.‬‬

‫‪ - 16‬ﺍﺤﺴﺏ ﺃﻗﻭﺍﺱ ﺒﻭﺍﺼﻭﻥ ﻟﻤﺭﻜﺒﺎﺕ ﺍﻟﺴﺭﻋﺔ ﻟﺠﺴﻴﻡ ﻴﺘﺤﺭﻙ ﻓﻲ ﺤﻘل‬


‫‪‬‬
‫ﻤﻐﻨﺎﻁﻴﺴﻲ ‪. B‬‬

‫‪ - 17‬ﺃﻭﺠﺩ ﻤﺴﺎﺭ ﻭﻗﺎﻨﻭﻥ ﺤﺭﻜﺔ ﺠﺴﻴﻡ ﻴﺘﺤﺭﻙ ﻓﻲ ﻜل ﻤﻥ ﺍﻟﺤﻘﻠﻴﻥ ﺍﻟﺘﺎﻟﻴﻴﻥ‪:‬‬


‫‪m12 2 m 22 2‬‬
‫)‪a‬‬ ‫)‪V ( x )  Fx , b‬‬ ‫‪V ( x , y) ‬‬ ‫‪x ‬‬ ‫‪y‬‬
‫‪2‬‬ ‫‪2‬‬
‫ﻭﺫﻟﻙ ﺍﻋﺘﻤﺎﺩﺍﹰ ﻋﻠﻰ ﻤﻌﺎﺩﻟﺔ ﻫﺎﻤﻠﺘﻭﻥ‪-‬ﺠﺎﻜﻭﺒﻲ‪...‬‬
‫‪ -18‬ﺒﺭﻫﻥ ﺼﺢ ﺍﻟﻤﻌﺎﺩﻟﺔ )‪.(3.94‬‬

‫‪- 113 -‬‬

‫)‪Create PDF files without this message by purchasing novaPDF printer (http://www.novapdf.com‬‬
- 114 -

Create PDF files without this message by purchasing novaPDF printer (http://www.novapdf.com)
‫‪ ‬‬

‫ﻣﺒﺎﺩﻯء ﺍﻟﺘﻐﲑﺍﺕ‬
‫‪Variation Principles‬‬

‫ﺍﺴﺘﻁﺎﻉ ﻤﻴﻜﺎﻨﻴﻙ ﻨﻴﻭﺘﻥ ﺍﻟﻤﺭﺘﻜﺯ ﻋﻠﻰ ﺍﻟﻤﺒﺎﺩﻯﺀ ﺍﻟﺜﻼﺜﺔ ﺍﻟﻤﻌﺭﻭﻓﺔ‪ ،‬ﺼﻴﺎﻏﺔ‬


‫ﻗﻭﺍﻨﻴﻥ ﺍﻟﻤﻴﻜﺎﻨﻴﻙ ﺒﻁﺭﻴﻘﺔ ﺠﻴﺩﺓ‪ .‬ﺃﻤﺎ ﺍﻟﻤﻴﻜﺎﻨﻴﻙ ﺍﻟﺘﺤﻠﻴﻠﻲ‪ ،‬ﺍﻟﻤﺭﺘﻜﺯ ﻋﻠﻰ ﺍﻟﺘﺤﻠﻴل‬
‫ﺍﻟﺭﻴﺎﻀﻲ‪ ،‬ﻭﺍﻟﺫﻱ ﺭﺃﻴﻨﺎ ﺒﻌﻀﺎﹰ ﻤﻥ ﺇﻨﺠﺎﺯﺍﺘﻪ ﻓﻲ ﺍﻟﻔﺼﻭل ﺍﻟﺴﺎﺒﻘﺔ‪ ،‬ﻓﻘﺩ ﻗﺩﻡ ﻟﻨﺎ‬
‫ﻤﻌﺎﺩﻻﺕ ﺤﺭﻜﺔ ﺠﺩﻴﺩﺓ‪ ،‬ﻭﻟﻜﻥ ﻫﺫﺍ ﻟﻴﺱ ﻜل ﺸﻲﺀ ﻭﺴﻨﺭﻯ ﻓﻲ ﻫﺫﺍ ﺍﻟﻔﺼل ﻋﺩﺓ‬
‫ﻁﺭﻕ ﺘﺅﺩﻱ ﺇﻟﻰ ﻤﻌﺎﺩﻻﺕ ﺍﻟﺤﺭﻜﺔ ﺘﺴﺘﻨﺩ ﺠﻤﻴﻌﻬﺎ ﺇﻟﻰ ﻤﺎﻴﺴﻤﻰ ﻤﺒﺎﺩﻯﺀ ﺍﻟﺘﻐﻴﺭﺍﺕ‬
‫ﺍﻟﺘﻲ ﺴﻨﺸﺭﺡ ﺃﺴﺴﻬﺎ ﺍﻟﻌﺎﻤﺔ ﻓﻴﻤﺎﻴﻠﻲ‪.‬‬
‫ﻴﺒﺩﻭ ﺃﻥ ﺍﻟﺤﺭﻜﺔ ﺍﻟﺤﻘﻴﻘﻴﺔ ﺍﻟﺘﻲ ﺘﺤﺩﺙ ﻓﻌﻼﹰ ﺘﺤﺕ ﺘﺄﺜﻴﺭ ﺍﻟﻘﻭﻯ ﺍﻟﻤﻁﺒﻘﺔ ﻋﻠﻰ‬
‫ﺠﻤﻠﺔ ﻤﺎ ﻻﺒﺩ ﺃﻥ ﺘﺤﻘﻕ ﺒﻌﺽ ﺍﻟﺸﺭﻭﻁ ﺍﻟﺘﻲ ﻻﺘﺤﻘﻘﻬﺎ ﺤﺭﻜﺎﺕ ﺍﻓﺘﺭﺍﻀﻴﺔ ﺃﺨﺭﻯ‬
‫ﻻﻴﻤﻜﻥ ﺃﻥ ﺘﺤﺩﺙ ﻭﻻﺘﺤﻘﻕ ﺸﺭﻭﻁ ﺍﻟﻤﺴﺄﻟﺔ ﺍﻟﻤﻴﻜﺎﻨﻴﻜﻴﺔ‪ .‬ﻭﻫﺫﺍ ﻫﻭ ﺍﻷﺴﺎﺱ ﺍﻟﺫﻱ‬
‫ﻴﺭﺘﻜﺯ ﻋﻠﻴﻪ ﻤﺒﺩﺃ ﺍﻟﺘﻐﻴﺭﺍﺕ ﺍﻟﺫﻱ ﺴﻴﻘﻭﺩﻨﺎ ﺇﻟﻰ ﺍﻟﻤﻌﺎﺩﻻﺕ ﻨﻔﺴﻬﺎ ﺍﻟﺘﻲ ﺤﺼﻠﻨﺎ ﻋﻠﻴﻬﺎ‬
‫ﺴﺎﺒﻘﺎﹰ ﺒﻁﺭﻕ ﺃﺨﺭﻯ‪ .‬ﻭﻟﻥ ﻨﺤﺼل ﻤﺒﺎﺸﺭﺓﹰ ﻋﻠﻰ ﻗﻭﺍﻨﻴﻥ ﺍﻟﺤﺭﻜﺔ ﻭﻟﻜﻨﻨﺎ ﺴﻨﻀﻊ‬
‫ﺍﻹﻁﺎﺭ ﺍﻟﻌﺎﻡ ﺍﻟﺫﻱ ﻴﻤﻜﻥ ﺃﻥ ﺘﺤﺩﺙ ﻀﻤﻨﻪ ﺃﻴﺔ ﺤﺭﻜﺔ‪.‬‬
‫‪ -28‬ﻣﺒﺪﺃ ﺍﻟﻔﻌﻞ ﺍﻷﺻﻐﺮ )ﻣﺒﺪﺃ ﻫﺎﻣﻠﺘﻮﻥ – ﺍﻭﺳﱰﻏﺮﺍﺩﺳﻜﻲ(‪:‬‬
‫ﺇﺤﺩﺍﺜﻴﺎﺘﻬﺎ‬ ‫‪Mi‬‬ ‫ﻨﻘﻁﺔ‬ ‫‪N‬‬ ‫ﻤﻥ‬ ‫ﻤﺅﻟﻔﺔ‬ ‫ﻤﺎﺩﻴﺔ‬ ‫ﺠﻤﻠﺔ‬ ‫ﻟﺘﻜﻥ‬
‫) ‪ (i  1,..., N) ( x i , y i .z i‬ﻤﻭﺼﻭﻓﺔ ﻓﻲ ﻓﻀﺎﺀ ﺍﻹﺤﺩﺍﺜﻴﺎﺕ ﺍﻟﻤﻌﻤﻤﺔ ﺒـ ‪ n‬ﺍﺤﺩﺍﺜﻲ‬
‫ﻤﻌﻌﻤﻡ ) ‪ ( j  1, 2, ..., n‬ﻭﻟﻨﻔﺭﺽ ﺃﻥ ﺘﺎﺒﻊ ﻻﻏﺭﺍﻨﺞ ﻟﻬﺎ ﻫﻭ ) ‪ L(q j , q j , t‬ﻭﻟﻨﻌﺩ‬
‫‪‬‬

‫ﺇﻟﻰ ﺍﻟﻔﻀﺎﺀﻴﻥ ﺍﻹﻗﻠﻴﺩﻴﻴﻥ ‪ E 3N 1‬ﻭ ‪ E n 1‬ﺍﻟﻤﺸﻜﻠﻴﻥ ﻓﻲ ﺍﻟﻔﺼل ﺍﻟﺴﺎﺒﻕ ﺜﻡ ﻨﺒﺤﺙ‬

‫‪- 115 -‬‬

‫)‪Create PDF files without this message by purchasing novaPDF printer (http://www.novapdf.com‬‬
‫ﻋﻥ ﻤﻌﺎﺩﻻﺕ ﺍﻟﺤﺭﻜﺔ ﺍﻨﻁﻼﻗﺎﹰ ﻤﻤﺎ ﻴﺴﻤﻰ ﻤﺒﺩﺃ ﺍﻟﻔﻌل ﺍﻷﺼﻐﺭ ﻭﻟﻬﺫﺍ ﻨﺄﺨﺫ ﺍﻟﺩﺍﻟﺔ ﻓﻲ‬
‫ﺍﻟﻔﻀﺎﺀ ‪ E n 1‬ﺍﻟﺘﺎﻟﻴﺔ‪:‬‬
‫~‬
‫)‪q j (t )  q j (t )  q j (t )  q j ( t )   j ( t ) (4.1‬‬

‫ﺤﻴﺙ ﻴﻤﺜل ﺍﻟﻤﻘﺩﺍﺭ ) ‪ q j (t‬ﺘﻐﻴﺭﺍﹰ ﻁﻔﻴﻔﺎﹰ ﻟﻠﺩﺍﻟﺔ ) ‪ q j ( t‬ﻋﺒﺭ ﻋﻨﻪ ﺒﻭﺍﺴﻁﺔ‬


‫ﺍﻟﺩﺍﻟﺔ ) ‪  j ( t‬ﺍﻟﻘﺎﺒﻠﺔ ﻟﻼﺸﺘﻘﺎﻕ‪ ،‬ﻭﺍﻟﺘﻲ ﺘﺒﻴﻥ ﻤﻘﺩﺍﺭ ﻫﺫﺍ ﺍﻟﺘﻐﻴﺭ ﻭﻗﺩ ﺘﻡ ﻀﺭﺏ ﺍﻟﺘﺎﺒﻊ‬
‫) ‪ q j ( t‬ﺒﻭﺴﻴﻁ ﺼﻐﻴﺭ ‪ ‬ﻴﻨﺘﻬﻲ ﺇﻟﻰ ﺍﻟﺼﻔﺭ‪.‬‬
‫ﻴﻤﺜل ﺍﻟﺸﻜل )‪ (4.1‬ﺍﻟﻤﻌﺎﺩﻟﺔ )‪ ،(4.1‬ﺤﻴﺙ ﻴﻌﺒﺭ ﺍﻟﺨﻁ ﺍﻟﻤﺘﺼل ﻓﻴﻪ ﻋﻥ‬
‫ﻗﺎﻨﻭﻥ ﺍﻟﺤﺭﻜﺔ ﺍﻟﺤﻘﻴﻘﻴﺔ ﺍﻟﺘﻲ ﺘﺤﺩﺙ ﻓﻌﻼﹰ ﺘﺤﺕ ﺘﺄﺜﻴﺭ ﺍﻟﻘﻭﻯ ﺍﻟﻤﻁﺒﻘﺔ ﺒﻴﻨﻤﺎ ﻴﻌﺒﺭ‬
‫ﺍﻟﺨﻁ ﺍﻟﻤﺘﻘﻁﻊ ﻋﻥ ﺍﻟﺩﺍﻟﺔ ) ‪ . ~q j ( t‬ﺃﻤﺎ ﺍﻟﻔﺭﻕ ﺍﻟﺸﺎﻗﻭﻟﻲ ﺒﻴﻨﻬﻤﺎ ﻓﻴﻤﺜل ) ‪ . q j (t‬ﻤﻊ‬
‫ﺍﻟﻌﻠﻡ ﺃﻥ ﺍﻟﻨﻘﻁﺔ ‪ A j‬ﺘﻤﺜل ﻭﻀﻊ ﺍﻟﺠﻤﻠﺔ ﺍﻟﻤﺎﺩﻴﺔ ﻓﻲ ﺍﻟﻠﺤﻅﺔ ‪ t 1‬ﻭ ‪ B j‬ﺘﻤﺜل ﻫﺫﺍ‬
‫ﺍﻟﻭﻀﻊ ﻓﻲ ﺍﻟﻠﺤﻅﺔ ‪ t 2‬ﻓﻲ ﺍﻟﻔﻀﺎﺀ ‪ ، E n 1‬ﻭﻗﺩ ﺍﻗﺘﺼﺭﻨﺎ ﻋﻠﻰ ﺭﺴﻡ ﺍﻟﻤﺴﺎﺭ‬
‫‪ A j B j‬ﻓﻲ ﻓﻀﺎﺀ ﺇﻗﻠﻴﺩﻱ ‪ (n  1)E 2‬ﻴﺤﻭﻱ ﺍﻹﺤﺩﺍﺜﻲ ﺍﻟﻤﻌﻤﻡ ‪ q j‬ﻭﺍﻟﺯﻤﻥ ‪ t‬ﻓﻘﻁ‪.‬‬

‫ﺍﻟﺸﻜل )‪(4.1‬‬
‫ﻟﺘﻜﻥ ﺍﻟﻨﻘﻁﺘﺎﻥ ‪ A‬ﻭ ‪ B‬ﻤﻥ ﺍﻟﻔﻀﺎﺀ ‪ E n 1‬ﺍﻟﻤﻤﺜﻠﺘﻴﻥ ﻟﻭﻀﻊ ﺍﻟﺠﻤﻠﺔ ﻓﻲ‬
‫ﺍﻟﻠﺤﻅﺘﻴﻥ ‪ t 1‬ﻭ ‪ t 2‬ﻋﻠﻰ ﺍﻟﺘﺭﺘﻴﺏ‪ .‬ﻴﻤﻜﻥ ﺭﺴﻡ ﻋﺩﺩ ﻻﻨﻬﺎﺌﻲ ﻤﻥ ﺍﻟﻤﺴﺎﺭﺍﺕ ﺒﻴﻥ ‪A‬‬

‫ﻭ ‪ .B‬ﻭﺍﻟﺴﺅﺍل ﺍﻵﻥ ﻤﺎﻫﻭ ﺍﻟﻤﺴﺎﺭ ﺍﻟﻤﻘﺎﺒل ﻟﻠﺤﺭﻜﺔ ﺍﻟﺤﻘﻴﻘﻴﺔ ﺍﻟﺘﻲ ﺘﺤﺩﺙ ﻓﻌﻼﹰ ﻓﻲ‬
‫ﺍﻟﻔﻀﺎﺀ ‪ E 3N 1‬؟‬

‫‪- 116 -‬‬

‫)‪Create PDF files without this message by purchasing novaPDF printer (http://www.novapdf.com‬‬
‫ﻴﺠﺏ ﻋﻠﻰ ﻫﺫﺍ ﺍﻟﺴﺅﺍل ﻤﺒﺩﺃ ﺍﻟﻔﻌل ﺍﻷﺼﻐﺭ ﺍﻟﺫﻱ ﻴﻨﺹ ﻋﻠﻰ ﻤﺎﻴﻠﻲ‪:‬‬
‫ﺇﻥ ﺍﻟﻤﺴﺎﺭ ﺒﻴﻥ ‪ A‬ﻭ ‪ B‬ﺍﻟﺫﻱ ﻴﻭﺍﻓﻕ ﺍﻟﺤﺭﻜﺔ ﺍﻟﺤﻘﻴﻘﻴﺔ ﻓﻲ ﺍﻟﻔﻀﺎﺀ ‪E 3N 1‬‬
‫ﻫﻭ ﺫﻟﻙ ﺍﻟﻤﺴﺎﺭ ﺍﻟﻤﻘﺎﺒل ﻟﻨﻬﺎﻴﺔ ﺼﻐﺭﻯ ﻟﺘﺎﺒﻊ ﺍﻟﻔﻌل ‪ ،S‬ﺍﻟﻤﻌﺭﻑ ﻓﻲ ﺍﻟﻔﺼل‬
‫ﺍﻟﺴﺎﺒﻕ‪ ،‬ﺒﺎﻟﻌﻼﻗﺔ‪:‬‬
‫‪t2‬‬
‫*‬
‫)‪S   L (q j , q j , t )dt (4.2‬‬
‫‪t1‬‬

‫ﻭﻟﻜﻲ ﻴﻜﻭﻥ ﻟﻠﺘﺎﺒﻊ )ﺍﻟﺩﺍﻟﺔ( ‪ S‬ﻨﻬﺎﻴﺔ ﺼﻐﺭﻯ ﻴﺠﺏ ﺃﻥ ﻴﻨﻌﺩﻡ ﺘﻔﺎﻀﻠﻬﺎ‬


‫)‪ (S  0‬ﻋﻠﻰ ﺃﻥ ﻴﺅﺨﺫ ﻫﺫﺍ ﺍﻟﺘﻔﺎﻀل ﺒﺜﺒﺎﺕ ﺍﻟﺯﻤﻥ ﻭﺒﺤﻴﺙ ﻴﺘﺤﻘﻕ ﺍﻟﺸﺭﻁﺎﻥ‬
‫ﺍﻟﺘﺎﻟﻴﺎﻥ‪:‬‬
‫‪q j‬‬ ‫‪ q j‬‬ ‫‪0‬‬ ‫)‪(4.3‬‬
‫‪t  t1‬‬ ‫‪tt 2‬‬

‫ﻟﻨﻔﺎﻀل ﺍﻵﻥ ﺍﻟﺩﺍﻟﺔ ‪ S‬ﻀﻤﻥ ﺍﻟﺸﺭﻭﻁ ﺍﻟﻤﺫﻜﻭﺭﺓ ﺜﻡ ﻨﻌﺩﻡ ‪ S‬ﻭﺴﻨﺠﺩ ﺃﻨﻨﺎ‬


‫ﻨﺤﺼل ﺒﺎﻟﻔﻌل ﻋﻠﻰ ﻤﻌﺎﺩﻻﺕ ﻻﻏﺭﺍﻨﺞ ﺍﻟﺘﻲ ﺭﺃﻴﻨﺎﻫﺎ ﻓﻲ ﺍﻟﻔﺼل ﺍﻟﺜﺎﻨﻲ‪.‬‬
‫‪t2‬‬ ‫‪‬‬ ‫‪t‬‬ ‫‪n‬‬ ‫‪‬‬
‫‪‬‬ ‫‪‬‬ ‫‪L‬‬ ‫‪L  ‬‬
‫)‪(4.4‬‬ ‫‪S   ‬‬ ‫‪  ‬‬
‫‪L(q j , q j , t )dt‬‬ ‫‪q j ‬‬ ‫‪q j dt  0‬‬
‫‪ q‬‬ ‫‪‬‬ ‫‪‬‬
‫‪t1‬‬ ‫‪t 1 j 1 ‬‬ ‫‪j‬‬ ‫‪q j‬‬
‫‪‬‬
‫ﻓﺈﺫﺍ ﻋﻠﻤﻨﺎ ﺃﻥ ‪ q j  q j‬ﻭﺤﺴﺒﻨﺎ ﺍﻟﺘﻜﺎﻤل ﺍﻟﺜﺎﻨﻲ ﺒﺎﻟﺘﺠﺯﺌﺔ ﻨﺠﺩ‪:‬‬
‫‪‬‬ ‫‪d‬‬
‫‪dt‬‬
‫‪t2‬‬
‫‪t2 n‬‬ ‫‪t2‬‬ ‫‪‬‬ ‫‪‬‬
‫‪L d‬‬ ‫‪L‬‬ ‫‪d  L ‬‬
‫‪(4.5)  ‬‬ ‫‪q j  ‬‬ ‫‪q j‬‬ ‫‪  ‬‬ ‫‪q dt‬‬
‫‪j 1 q‬‬
‫‪ dt‬‬
‫‪q‬‬
‫‪‬‬ ‫‪dt  q   j‬‬
‫‪t1‬‬ ‫‪j‬‬ ‫‪j‬‬ ‫‪t1‬‬ ‫‪ j‬‬
‫‪t1‬‬

‫ﻴﻨﻌﺩﻡ ﺍﻟﺤﺩ ﺍﻷﻭل ﻁﺒﻘﺎﹰ ﻟـ )‪ (4.3‬ﻓﺈﺫﺍ ﺩﻤﺠﻨﺎ ﺍﻟﺤﺩ ﺍﻟﺒﺎﻗﻲ ﻓﻲ ﺍﻟﻤﻌﺎﺩﻟﺔ‬


‫)‪ (4.4‬ﻨﺤﺼل ﻋﻠﻰ ﺍﻟﻤﻌﺎﺩﻟﺔ‪:‬‬
‫‪n t 2 ‬‬ ‫‪‬‬
‫‪L d L ‬‬
‫‪S    ‬‬ ‫‪‬‬ ‫‪q .dt  0‬‬ ‫)‪(4.6‬‬
‫‪‬‬‫‪q‬‬ ‫‪dt‬‬ ‫‪ j‬‬
‫‪j 1 t ‬‬ ‫‪j‬‬ ‫‪q j ‬‬
‫‪1‬‬ ‫‪‬‬

‫* ‪ -‬ﳜﺘﻠﻒ ﺣﺪ ﺍﻟﺘﻜﺎﻣﻞ ﺍﻷﻋﻠﻰ ﻫﻨﺎ ﻋﻤﺎ ﺭﺃﻳﻨﺎﻩ ﰲ ﺍﻟﻔﺼﻞ ﺍﻟﺴﺎﺑﻖ ﻋﻨﺪ ﺍﳊﺼﻮﻝ ﻋﻠﻰ ﻣﻌﺎﺩﻟﺔ ﻫﺎﻣﻠﺘﻮﻥ – ﺟﺎﻛﻮﰊ ﺣﻴﺚ ﺗﺮﻛﻨﺎﻩ ﻣﻔﺘﻮﺣﺎﹰ ‪،t‬‬
‫ﺃﻣﺎ ﻫﻨﺎ ﻓﻘﺪ ﻭﺿﻌﻨﺎﻩ ‪ t 2‬ﻭﻟﻴﺲ ‪ t‬ﻛﻲ ﻧﺴﺘﻔﻴﺪ ﻣﻦ ﺍﻟﺸﺮﻁ )‪ ،(4.3‬ﳍﺬﺍ ﺍﻗﺘﻀﻰ ﺍﻟﺘﻨﻮﻳﻪ‪.‬‬

‫‪- 117 -‬‬

‫)‪Create PDF files without this message by purchasing novaPDF printer (http://www.novapdf.com‬‬
‫ﻭﻟﻜﻥ ﻴﻨﻌﺩﻡ ‪ S‬ﻤﻬﻤﺎ ﻜﺎﻨﺕ ‪ q j‬ﻴﺠﺏ ﺃﻥ ﺘﺘﺤﻘﻕ ﺍﻟﻌﻼﻗﺔ‪:‬‬
‫‪t 2‬‬ ‫‪‬‬
‫‪ d L L ‬‬
‫‪  dt   q dt  0‬‬ ‫)‪(4.7‬‬
‫‪t 1‬‬ ‫‪q j‬‬ ‫‪j‬‬
‫‪‬‬
‫ﻭﻟﻜﻲ ﻴﺘﺤﻘﻕ ﺫﻟﻙ ﻤﻥ ﺃﺠل ﺃﻱ ﻟﺤﻅﺘﻴﻥ ﺍﺨﺘﻴﺎﺭﻴﺘﻴﻥ ‪ t 1‬ﻭ ‪ t 2‬ﻤﻬﻤﺎ ﻜﺎﻥ‬
‫ﺍﻟﻔﺎﺭﻕ ﺒﻴﻨﻬﻤﺎ ) ‪ (t  t 2  t 1‬ﻴﺠﺏ ﺃﻥ ﻴﻨﻌﺩﻡ ﺍﻟﻤﺴﺘﻜﻤل‪ .‬ﻭﻫﺫﺍ ﻴﺅﺩﻱ ﺒﺩﻭﺭﻩ ﺇﻟﻰ‬
‫ﻤﻌﺎﺩﻻﺕ ﻻﻏﺭﺍﻨﺞ ﺍﻟﺘﺎﻟﻴﺔ‪:‬‬
‫‪d L L‬‬
‫‪‬‬ ‫‪0‬‬ ‫)‪(4.8‬‬
‫‪dt q  q j‬‬
‫‪j‬‬

‫ﻟﻨﺘﺴﺎﺀل ﺍﻵﻥ ﻫل ﻴﻭﺠﺩ ﺘﺎﺒﻊ ﻻﻏﺭﺍﻨﺞ ﻭﺍﺤﺩ ﻴﺤﻘﻕ ﺍﻟﻤﻌﺎﺩﻻﺕ )‪(4.8‬؟‬


‫ﻟﻺﺠﺎﺒﺔ ﻋﻥ ﻫﺫﺍ ﺍﻟﺴﺅﺍل ﻨﺄﺨﺫ ﺘﺎﺒﻌﺎﹰ ﺁﺨﺭ ‪ 'L‬ﻤﺤﻘﻘﺎﹰ ﻟﻠﻌﻼﻗﺔ‪:‬‬
‫‪d‬‬
‫‪L'  L ‬‬ ‫) ‪f (q1 , q 2 ,..., q n , t‬‬ ‫)‪(4.9‬‬
‫‪dt‬‬
‫ﻭﻟﻨﺤﺴﺏ ﺘﺎﺒﻊ ﺍﻟﻔﻌل ﺍﻟﻤﻘﺎﺒل ﻟـ ‪ 'L‬ﻓﻨﺠﺩ‪:‬‬
‫‪t2‬‬ ‫‪t2‬‬
‫‪S' ‬‬ ‫) ‪ L' dt   Ldt  f (t B , q jB )  f (t A , q jA‬‬ ‫)‪(4.10‬‬
‫‪t1‬‬ ‫‪t1‬‬

‫ﺃﻱ ﺃﻥ‪:‬‬
‫) ‪S'  S  f (t B , q jB )  f ( t A , q jA‬‬ ‫)‪(4.11‬‬
‫ﻭﺒﺎﻟﻤﻔﺎﻀﻠﺔ ﻨﺠﺩ‪:‬‬
‫) ‪n  f ( t , q‬‬ ‫) ‪f (t A , q jA‬‬ ‫‪‬‬
‫)‪(4.12‬‬
‫‪B jB‬‬
‫‪S'  S   ‬‬ ‫‪q jB ‬‬ ‫‪q jA ‬‬
‫‪j 1‬‬
‫‪q jB‬‬ ‫‪q jA‬‬ ‫‪‬‬
‫ﻭﺍﻟﺤﺩﺍﻥ ﺍﻷﺨﻴﺭﺍﻥ ﻤﻌﺩﻭﻤﺎﻥ ﻁﺒﻘﺎﹰ ﻟـ )‪ (4.3‬ﺃﻱ ﺃﻥ '‪ S  S‬ﻭﺒﺎﻟﺘﺎﻟﻲ‬
‫ﻴﻜﻭﻥ‪:‬‬
‫'‪d L' L‬‬
‫‪‬‬ ‫‪0‬‬ ‫) ‪( j  1, 2, ..., n‬‬ ‫)‪(4.13‬‬
‫‪dt q  q j‬‬
‫‪j‬‬

‫‪- 118 -‬‬

‫)‪Create PDF files without this message by purchasing novaPDF printer (http://www.novapdf.com‬‬
‫ﺃﻱ ﺃﻨﻪ ﻴﻭﺠﺩ ﻋﺩﺩ ﻻﻨﻬﺎﺌﻲ ﻤﻥ ﺘﻭﺍﺒﻊ ﻻﻏﺭﺍﻨﺞ ﺘﺤﻘﻕ ﺍﻟﻤﻌﺎﺩﻻﺕ )‪(4.13‬‬
‫ﻭﺃﻥ ﻫﺫﻩ ﺍﻟﻤﻌﺎﺩﻻﺕ ﺘﺒﻘﻰ ﺼﺤﻴﺤﺔ ﻋﻨﺩ ﺇﻀﺎﻓﺔ ﺃﻱ ﻤﺸﺘﻕ ﻜﻠﻲ ﻟﺘﺎﺒﻊ ﻤﺎ ﻟﻺﺤﺩﺍﺜﻴﺎﺕ‬
‫ﺍﻟﻤﻌﻤﻤﺔ ﻭﺍﻟﺯﻤﻥ‪.‬‬
‫ﻤﻥ ﺍﻟﻤﻔﻴﺩ‪ ،‬ﻓﻲ ﻨﻬﺎﻴﺔ ﻫﺫﻩ ﺍﻟﻔﻘﺭﺓ‪ ،‬ﺃﻥ ﺘﺴﺘﻨﺘﺞ ﻤﺒﺩﺃ ﺍﻟﻔﻌل ﺍﻷﺼﻐﺭ ﻤﻥ ﺍﻟﻤﺒﺩﺃ‬
‫ﺍﻟﺩﻴﻨﺎﻤﻴﻜﻲ )ﺍﻟﺘﺤﺭﻴﻜﻲ( ﻟﻼﻨﺘﻘﺎﻻﺕ ﺍﻻﻓﺘﺭﺍﻀﻴﺔ ﺍﻟﺫﻱ ﻴﻁﻠﻕ ﻋﻠﻴﻪ ﺃﺤﻴﺎﻨﺎﹰ ﻤﺒﺩﺃ‬
‫ﺩﺍﻟﻤﺒﻴﺭ‪-‬ﻻﻏﺭﺍﻨﺞ )ﺍﻟﻔﺼل ﺍﻟﺜﺎﻨﻲ(‪ .‬ﻟﺩﻴﻨﺎ ﻁﺒﻘﺎﹰ ﻟﻠﻤﺒﺩﺃ ﺍﻟﻤﺫﻜﻭﺭ‪:‬‬
‫‪‬‬
‫‪N‬‬
‫‪‬‬ ‫‪‬‬
‫‪  n‬‬ ‫‪N‬‬
‫‪‬‬ ‫‪‬‬
‫‪  ri‬‬
‫‪‬‬ ‫‪F‬‬
‫‪ 1‬‬ ‫‪‬‬ ‫‪m‬‬ ‫‪i‬‬ ‫‪W‬‬ ‫‪i ‬‬‫‪.‬‬‫‪‬‬ ‫‪r‬‬‫‪i‬‬ ‫‪‬‬ ‫‪‬‬ ‫‪‬‬‫‪q‬‬ ‫‪j   Fi  m i Wi .‬‬ ‫)‪ 0 (4.14‬‬
‫‪i 1 ‬‬ ‫‪‬‬ ‫‪j1‬‬ ‫‪i 1 ‬‬ ‫‪ q j‬‬
‫ﻭﻤﻨﻪ ﻁﺒﻘﺎﹰ ﻟﻤﺎ ﺭﺃﻴﻨﺎﻩ ﻓﻲ ﺍﻟﻔﺼل ﺍﻟﺜﺎﻨﻲ ﻨﺠﺩ‪:‬‬
‫‪n‬‬ ‫‪‬‬ ‫‪T d T ‬‬
‫‪ q  Q‬‬ ‫‪j‬‬ ‫‪j‬‬ ‫‪ Q 'j ‬‬ ‫‪‬‬
‫‪q j dt q j ‬‬
‫‪0‬‬ ‫)‪(4.15a‬‬
‫‪j1‬‬ ‫‪‬‬
‫ﻭﻫﺫﻩ ﺍﻟﻤﻌﺎﺩﻟﺔ ﺼﺤﻴﺤﺔ ﻓﻲ ﻜل ﻟﺤﻅﺔ ﻭﺒﺎﻟﺘﺎﻟﻲ ﻴﻤﻜﻥ ﺍﺴﺘﻜﻤﺎﻟﻬﺎ ﺒﺎﻟﺯﻤﻥ‬
‫ﺒﻔﺭﺽ ﺍﻨﻌﺩﺍﻡ ﺍﻟﻘﻭﻯ ﺍﻟﻤﻌﻤﻤﺔ ﻏﻴﺭ ﺍﻟﻤﺸﺘﻕ ﻤﻥ ﻜﻤﻭﻥ )‪: (Q j  0‬‬
‫'‬

‫‪t2 n‬‬
‫‪ L d L ‬‬
‫‪  j  q j  dt q j  dt‬‬
‫‪‬‬‫‪q‬‬ ‫)‪(4.15b‬‬
‫‪t 1 j1‬‬ ‫‪‬‬ ‫‪‬‬
‫ﻨﻼﺤﻅ ﻋﻨﺩﺌﺫ ﺃﻥ ﺍﻟﻁﺭﻑ ﺍﻷﻴﺴﺭ ﻤﻥ )‪ (4.15a‬ﻫﻭ ﺘﻔﺎﻀل ﺍﻟﺘﺎﺒﻊ ‪ L‬ﻭﻫﻜﺫﺍ‬
‫ﻨﺠﺩ ﺃﺨﻴﺭﺍﹰ ﻤﺒﺩﺃ ﺍﻟﻔﻌل ﺍﻷﺼﻐﺭ ﻨﻔﺴﻪ ﺍﻟﺘﺎﻟﻲ‪:‬‬
‫‪S   L.dt  0‬‬ ‫)‪(4.16‬‬

‫‪ - 29‬ﺍﺳﺘﻨﺘﺎﺝ ﻣﻌﺎﺩﻻﺕ ﻫﺎﻣﻠﺘﻮﻥ ﻣﻦ ﻣﺒﺪﺃ ﺍﻟﻔﻌﻞ ﺍﻷﺻﻐﺮ‪:‬‬


‫ﻟﺩﻴﻨﺎ ﻁﺒﻘﺎﹰ ﻟﻠﻤﺒﺩﺃ ﺍﻟﻤﺫﻜﻭﺭ‪:‬‬
‫‪t2‬‬ ‫‪t2 n‬‬
‫)‪(4.17‬‬
‫‪‬‬ ‫‪‬‬
‫‪ L(q j , q j , t )dt  ‬‬
‫‪‬‬ ‫‪‬‬
‫‪(P jq j  H )dt‬‬
‫‪t1‬‬ ‫‪t 1 i 1‬‬

‫ﻭﺒﺈﺠﺭﺍﺀ ﺍﻟﺘﻔﺎﻀل ﻨﺠﺩ‪:‬‬


‫‪t2 n‬‬ ‫‪ n‬‬ ‫‪n‬‬ ‫‪‬‬
‫‪‬‬ ‫‪‬‬ ‫‪ P j‬‬
‫‪‬‬
‫)‪  P jq j  q j P j    H P j   H q j dt  0 (4.18‬‬
‫‪‬‬
‫‪t 1  j 1‬‬ ‫‪ j 1‬‬ ‫‪j 1 q j‬‬ ‫‪‬‬

‫‪- 119 -‬‬

‫)‪Create PDF files without this message by purchasing novaPDF printer (http://www.novapdf.com‬‬
‫ﻓﺈﺫﺍ ﻋﻠﻤﻨﺎ ﺃﻥ‪:‬‬
‫‪t2‬‬ ‫‪t‬‬ ‫‪t2‬‬
‫‪d‬‬ ‫‪2‬‬ ‫‪dPj‬‬
‫‪‬‬
‫‪ P j .q j .dt   Pj‬‬ ‫‪q j .dt  Pj q j‬‬ ‫‪  q j‬‬ ‫)‪dt (4.19‬‬
‫‪t1‬‬
‫‪dt‬‬ ‫‪t1‬‬ ‫‪t1‬‬
‫‪dt‬‬

‫ﻭﺃﻥ ﺍﻟﺤﺩ ﺍﻷﻭل ﻤﻥ ﺍﻟﻁﺭﻑ ﺍﻟﺜﺎﻨﻲ ﻴﺴﺎﻭﻱ ﺍﻟﺼﻔﺭ ﻓﺈﻨﻨﺎ ﻨﺠﺩ ﺒﺎﻟﺘﺒﺩﻴل ﻓﻲ‬
‫)‪:(4.18‬‬
‫‪t2 n ‬‬
‫‪‬‬ ‫‪H ‬‬ ‫‪  H  ‬‬
‫)‪(4.20‬‬
‫‪‬‬
‫‪   q j  q‬‬ ‫‪‬‬
‫‪P j   P j ‬‬
‫‪‬‬ ‫‪‬‬‫‪q‬‬
‫‪q dt  0‬‬
‫‪ j‬‬
‫‪t 1 j 1 ‬‬ ‫‪j‬‬ ‫‪‬‬ ‫‪j‬‬ ‫‪‬‬
‫ﻭﻟﻜﻲ ﺘﺘﺤﻘﻕ ﻫﺫﻩ ﺍﻟﻤﻌﺎﺩﻟﺔ ﻓﻲ ﻜل ﻟﺤﻅﺔ ﻴﺠﺏ ﺃﻥ ﻴﻨﻌﺩﻡ ﺍﻟﻤﺴﺘﻜﻤل‪ ،‬ﻭﻤﻨﻪ‪:‬‬
‫‪n‬‬ ‫‪  H ‬‬ ‫‪‬‬ ‫‪‬‬ ‫‪‬‬
‫‪q ‬‬ ‫‪P   P   H q   0‬‬ ‫)‪(4.21‬‬
‫‪  j P  j  j q  j ‬‬
‫‪‬‬
‫‪j 1‬‬ ‫‪j‬‬ ‫‪‬‬ ‫‪j‬‬ ‫‪‬‬
‫ﻭﻟﻜﻲ ﺘﺘﺤﻘﻕ )‪ (4.21‬ﻤﻬﻤﺎ ﻜﺎﻥ ﺍﻟﻤﺘﺤﻭﻻﻥ ﺍﻟﻤﺴﺘﻘﻼﻥ ‪ P j‬و ‪ q j‬ﻴﺠﺏ ﺃﻥ‬
‫ﻴﻜﻭﻥ‪:‬‬
‫‪H‬‬ ‫‪H‬‬
‫)‪(4.22‬‬
‫‪‬‬ ‫‪‬‬
‫‪ qj ,‬‬ ‫) ‪ P j , ( j  1, 2,..., n‬‬
‫‪P j‬‬ ‫‪q j‬‬

‫ﻭﻫﻲ ﺍﻟﻤﻌﺎﺩﻻﺕ ﺍﻟﻘﺎﻨﻭﻨﻴﺔ )ﻤﻌﺎﺩﻻﺕ ﻫﺎﻤﻠﺘﻭﻥ( ﻨﻔﺴﻬﺎ‪.‬‬


‫‪ - 30‬ﺍﻟﺘﻔﺎﺿﻞ ﺍﻻﻓﱰﺍﺿﻲ )ﺍﳌﺘﺴﺎﻭﻱ ﺍﻟﺰﻣﻦ ( ﻭﺍﻟﺘﻔﺎﺿﻞ ﺍﻟﻜﻠﻲ‬
‫ﻭﺍﻟﻌﻼﻗﺔ ﺑﻴﻨﻬﻤﺎ‪:‬‬
‫ﻟﻘﺩ ﺘﻌﺭﻓﻨﺎ ﻓﻲ ﻫﺫﺍ ﺍﻟﻔﺼل ﻋﻠﻰ ﻤﺒﺩﺃ ﻫﺎﻡ ﻤﻥ ﻤﺒﺎﺩﻯﺀ ﺍﻟﺘﻐﻴﺭﺍﺕ ﻭﻟﻜﻥ ﻻﺒﺩ‬
‫ﻟﻨﺎ ﻜﻲ ﻨﺘﺎﺒﻊ ﺩﺭﺍﺴﺔ ﺍﻟﻤﺒﺎﺩﻯﺀ ﺍﻟﺒﺎﻗﻴﺔ ﻤﻥ ﺸﺭﺡ ﺃﻨﻭﺍﻉ ﺃﺨﺭﻯ ﻤﻥ ﺍﻟﺘﻔﺎﻀﻼﺕ ﻏﻴﺭ‬
‫ﺘﻠﻙ ﺍﻟﺘﻲ ﺍﺴﺘﺨﺩﻤﻨﺎﻫﺎ ﻓﻲ ﺍﻟﻤﺒﺩﺃ ﺍﻟﺴﺎﺒﻕ‪.‬‬
‫ﻟﺘﻜﻥ ﺍﻟﺩﺍﻟﺔ‪:‬‬
‫) ‪q  f (t‬‬ ‫)‪(4.23‬‬
‫ﺇﻥ ﺘﻔﺎﻀﻠﻬﺎ ﺍﻟﻜﻠﻲ ﻫﻭ‪:‬‬
‫‪dq  f ' ( t )dt‬‬ ‫)‪(4.24‬‬

‫‪- 120 -‬‬

‫)‪Create PDF files without this message by purchasing novaPDF printer (http://www.novapdf.com‬‬
‫ﻭﻫﻭ ﺘﻔﺎﻀل ﺍﻟﺘﺎﺒﻊ ‪ q‬ﻋﻨﺩﻤﺎ ﻴﺘﻐﻴﺭ ﺍﻟﻤﺘﺤﻭل )ﺍﻟﺯﻤﻥ(‪ .‬ﻭﻤﻥ ﺍﻟﻭﺍﻀﺢ ﺃﻥ ﻫﺫﺍ‬
‫ﺍﻟﺘﻔﺎﻀل ﻴﻘﺎﺒل ﺍﻟﺤﺭﻜﺔ ﺍﻟﺘﻲ ﺘﺤﺩﺙ ﻓﻌﻼﹰ ﻭﺍﻟﺘﻲ ﻗﺎﻨﻭﻨﻬﺎ )‪ (4.23‬ﻭﺃﻥ‬
‫‪‬‬ ‫‪dq‬‬
‫‪q ‬‬
‫‪dt‬‬
‫ﻭﻴﻤﺜل ﻅل ﺍﻟﺯﺍﻭﻴﺔ ﺍﻟﺘﻲ ﻴﺼﻨﻌﻬﺎ ﺍﻟﻤﻤﺎﺱ ﻤﻊ ﺍﻟﻤﺤﻭﺭ ﺍﻷﻓﻘﻲ )ﻤﺤﻭﺭ ﺍﻟﺯﻤﻥ ﻜﻤﺎ‬
‫ﻓﻲ ﺍﻟﺸﻜل ‪ .(4.2‬ﺃﻤﺎ ﺍﻟﺘﻔﺎﻀل ﺍﻻﻓﺘﺭﺍﻀﻲ ﺍﻭ ﺍﻟﻤﺘﺴﺎﻭﻱ ﺍﻟﺯﻤﻥ ﻓﻴﺤﺩﺙ ﺜﺒﺎﺕ‬
‫ﺍﻟﻤﺘﺤﻭل ﻭﻫﻭ ﻤﺎ ﻋﺒﺭﻨﺎ ﻋﻨﻪ ﺒﺎﻟﻌﻼﻗﺔ )‪ (4.1‬ﺃﻱ ﺃﻥ‪:‬‬
‫) ‪( t ) q  q~  q  ( t‬‬ ‫)‪(4.25‬‬
‫ﻭﻫﻭ ﺍﻟﺘﻐﻴﺭ ﺍﻟﺫﻱ ﺍﺴﺘﺨﺩﻤﻨﺎﻩ ﻓﻲ ﺍﻟﻔﻘﺭﺘﻴﻥ ﺍﻟﺴﺎﺒﻘﺘﻴﻥ‪.‬‬

‫ﺍﻟﺸﻜل )‪(4.2‬‬
‫ﻟﻨﺤﺴﺏ ﻤﺸﺘﻕ ‪ q‬ﺒﺎﻟﻨﺴﺒﺔ ﻟﻠﺯﻤﻥ ﻓﻨﺠﺩ‪:‬‬
‫‪d‬‬
‫)‪(4.26‬‬
‫‪‬‬ ‫‪‬‬
‫~ ‪q ‬‬
‫‪q q‬‬
‫‪dt‬‬
‫ﺴﻨﻌﺭﺽ ﺍﻵﻥ ﺘﻐﻴﺭﺍﹰ ﺁﺨﺭ ﺍﹰ ﻟﻠﺩﺍﻟﺔ ‪ q‬ﻴﺘﻐﻴﺭ ﻓﻴﻪ ﺍﻟﻤﺘﺤﻭل ‪ t‬ﻟﻴﺼﺒﺢ ‪ ~t‬ﻁﺒﻘﺎﹰ‬
‫ﻟﻠﻌﻼﻗﺔ‪:‬‬
‫)‪(4.27‬‬
‫~‬ ‫~‬
‫) ‪t  t  (t ), t  t  t   (t‬‬
‫ﺤﻴﺙ ) ‪  (t‬ﻫﻲ ﺩﺍﻟﺔ ﻗﺎﺒﻠﺔ ﻟﻼﺸﺘﻘﺎﻕ‪ ،‬ﻭﻋﻨﺩﺌ ﺫ‪ ‬ﻨﺠﺩ ﻁﺒﻘﺎﹰ ﻟﻠﺸﻜل )‪ (4.3‬ﺃﻥ‬
‫ﺘﻐﻴﺭ ‪ q‬ﺍﻟﻤﻘﺎﺒل ﻟـ ‪) t‬ﻭﺍﻟﺫﻱ ﻴﺴﻤﻰ ﺍﻟﺘﻐﻴﺭ ﺍﻟﻜﻠﻲ( ﻫﻭ‪:‬‬
‫)‪(4.28‬‬
‫‪‬‬
‫‪q  q  q t‬‬

‫‪- 121 -‬‬

‫)‪Create PDF files without this message by purchasing novaPDF printer (http://www.novapdf.com‬‬
‫ﺍﻟﺸﻜل )‪(4.3‬‬

‫ﻭﻫﺫﺍ ﺍﻟﺘﻐﻴﺭ‪ ،‬ﻜﻤﺎ ﻨﺭﻯ‪ ،‬ﻤﺅﻟﻑ ﻤﻥ ﻗﺴﻤﻴﻥ‪ :‬ﺘﻐﻴﺭ ﻤﺘﺴﺎﻭﻱ ﺍﻟﺯﻤﻥ ‪q‬‬

‫ﻭﺘﻐﻴﺭ ﻨﺎﺘﺞ ﻋﻥ ﺘﻐﻴﺭ ﺍﻟﻤﺘﺤﻭل ﻫﻭ ‪ . q t‬ﺇﻥ ﻤﺸﺘﻕ ‪ q‬ﺒﺎﻟﻨﺴﺒﺔ ﻟﻠﺯﻤﻥ ﻫﻭ‪:‬‬


‫‪‬‬

‫‪d‬‬ ‫‪dt‬‬
‫‪q  q   q  t  q ‬‬ ‫)‪(4.29‬‬
‫‪dt‬‬ ‫‪dt‬‬
‫ﻓﺈﺫﺍ ﺤﺴﺒﻨﺎ ‪ q‬ﻤﻥ )‪ (4.28‬ﻓﺈﻨﻨﺎ ﻨﺠﺩ‪:‬‬
‫‪‬‬

‫)‪(4.30‬‬
‫‪‬‬ ‫‪‬‬ ‫‪‬‬
‫‪q  q  q t‬‬

‫ﺃﻱ ﻋﻨﺩﻤﺎ ﻴﺤﺩﺙ ‪t‬‬ ‫ﺇﻻ ﺇﺫﺍ ﻜﺎﻥ ‪ 0‬‬ ‫ﻭﻫﻲ ﻻﺘﺴﺎﻭﻱ ‪q‬‬
‫‪dt‬‬ ‫‪d‬‬
‫‪dt‬‬ ‫‪dt‬‬
‫ﺍﻟﻤﻌﺭﻑ ﻓﻲ )‪ (4.27‬ﺒﺜﺒﺎﺕ ﺍﻟﺯﻤﻥ‪.‬‬
‫‪ - 31‬ﻣﺒﺪﺃ ﻣﻮﺑﲑﺗﻮﻱ – ﻻﻏﺮﺍﻧﺞ )ﻣﺒﺪﺃ ﺃﻭﻟﺮ – ﻻﻏﺮﺍﻧﺞ(‪:‬‬
‫ﻴﺨﺘﻠﻑ ﺍﻟﻤﺅﻟﻔﻭﻥ ﻋﻠﻰ ﺘﺴﻤﻴﺔ ﻫﺫﺍ ﺍﻟﻤﺒﺩﺃ ﻷﻥ ﻤﻭﺒﻴﺭﺘﻭﻱ ﺼﺎﻏﻪ ﻋﺎﻡ‬
‫)‪ (1744‬ﻭﺴﻤﺎﻩ ﻤﺒﺩﺃ ﺍﻟﻔﻌل ﺍﻷﺼﻐﺭ ﻭﻁﺒﻘﻪ ﻋﻠﻰ ﻨﻅﺭﻴﺔ ﺍﻻﺼﻁﺩﺍﻡ ﻭﻋﻠﻰ ﻤﺴﺎﺌل‬
‫ﺃﺨﺭﻯ ﻭﻟﻜﻥ ﺃﻭﻟﺭ ﺒﺭﻫﻨﻪ‪ ،‬ﺃﻤﺎ ﻻﻏﺭﺍﻨﺞ ﻓﻘﺩ ﻋﻤﻤﻪ ﻋﻠﻰ ﻜﺜﻴﺭ ﻤﻥ ﻤﺴﺎﺌل ﺘﺤﺭﻴﻙ‬
‫ﺍﻟﻤﺠﻤﻭﻋﺎﺕ ﺍﻟﻤﺎﺩﻴﺔ‪.‬‬

‫‪- 122 -‬‬

‫)‪Create PDF files without this message by purchasing novaPDF printer (http://www.novapdf.com‬‬
‫ﻟﻘﺩ ﻜﻨﺎ ﻋﺭﻓﻨﺎ ﻓﻲ ﺍﻟﻔﺼل ﺍﻟﺴﺎﺒﻕ ﺘﺎﺒﻊ ﻓﻌل ﻻﻏﺭﺍﻨﺞ ﺃﻭ ﺘﺎﺒﻊ ﺍﻟﻔﻌل ﺍﻷﺼﻐﺭ‬
‫‪ W‬ﺍﻟﺫﻱ ﺴﻨﺴﺘﺨﺩﻤﻪ ﺍﻵﻥ‪ ،‬ﻭﻟﻜﻨﻨﺎ ﺴﻨﺒﺩﺃ ﻤﻥ ﺘﺎﺒﻊ ﺍﻟﻔﻌل )‪ (4.2‬ﺍﻟﻤﻌﺭﻑ ﺴﺎﺒﻘﺎﹰ‬
‫ﻭﻨﻀﻌﻪ ﺒﺎﻟﺼﻴﻐﺔ ﺍﻟﺘﺎﻟﻴﺔ‪:‬‬
‫‪t1‬‬ ‫‪t2‬‬
‫‪S   L.dt ‬‬ ‫‪ (T  V)dt‬‬
‫'‪t‬‬ ‫‪t1‬‬

‫ﻭﺴﻨﻔﺭﺽ ﺃﻥ ﺍﻟﺠﻤﻠﺔ ﺍﻟﻤﺩﺭﻭﺴﺔ ﺘﺤﻘﻕ ﻗﺎﻨﻭﻥ ﻤﺼﻭﻨﻴﺔ ﺍﻟﻁﺎﻗﺔ‬


‫‪ T  V  E  Const.‬ﻭﺒﺎﻟﺘﺎﻟﻲ ﻴﻜﻭﻥ‪:‬‬
‫‪t2‬‬ ‫‪t2‬‬
‫‪S‬‬ ‫) ‪ (T  E  T)dt   2Tdt  E(t 2  t1‬‬ ‫)‪(4.31‬‬
‫‪t1‬‬ ‫‪t1‬‬

‫ﻭﻁﺒﻘﺎﹰ ﻟـ )‪ (3.84‬ﻴﻜﻭﻥ‪:‬‬
‫‪t2‬‬ ‫‪t2 N‬‬
‫)‪(4.32‬‬
‫‪2‬‬
‫‪W   2Tdt ‬‬ ‫‪  mi v i dt‬‬
‫‪t1‬‬ ‫‪t1 i 1‬‬

‫ﻭﻤﻥ )‪ (4.31‬ﻭ )‪ (4.32‬ﻨﺠﺩ‪:‬‬


‫) ‪W  S  E( t 2  t 1‬‬ ‫)‪(4.33‬‬
‫ﻴﻨﺹ ﻤﺒﺩﺃ ﻤﻭﺒﻴﺭﺘﻭﻱ – ﻻﻏﺭﺍﻨﺞ ﻋﻠﻰ ﻤﺎﻴﻠﻲ‪:‬‬
‫ﺘﺤﺩﺙ ﺍﻟﺤﺭﻜﺔ ﺍﻟﺤﻘﻴﻘﻴﺔ ﻟﺠﻤﻠﺔ ﻤﺎﺩﻴﺔ ﺨﺎﻀﻌﺔ ﻻﺭﺘﺒﺎﻁﺎﺕ ﻫﻭﻟﻭﻨﻭﻤﻴﺔ‬
‫ﻭﻤﺴﺘﻘﺭﺓ ﺒﻴﻥ ﺒﺩﺍﻴﺔ ‪) A‬ﻤﻭﺍﻓﻘﺔ ﻟﻭﻀﻊ ﺍﻟﺠﻤﻠﺔ ﻓﻲ ﺍﻟﻔﻀﺎﺀ ‪ E n 1‬ﻓﻲ ﺍﻟﻠﺤﻅﺔ ‪( t 1‬‬
‫ﻭﻨﻬﺎﻴﺔ ‪) B‬ﻤﻭﺍﻓﻘﺔ ﻟﻭﻀﻊ ﺍﻟﺠﻤﻠﺔ ﻓﻲ ﺍﻟﻔﻀﺎﺀ ‪ E n 1‬ﻓﻲ ﺍﻟﻠﺤﻅﺔ ‪ ( t 2‬ﺒﺤﻴﺙ ﻴﻨﻌﺩﻡ‬
‫ﺍﻟﺘﻔﺎﻀل ﺍﻟﻜﻠﻲ ﻟﺘﺎﺒﻊ ﺍﻟﻔﻌل ﺍﻷﺼﻐﺭ ‪ W‬ﺃﻱ‪:‬‬
‫‪t2‬‬
‫‪W    2Tdt  0‬‬ ‫)‪(4.34‬‬
‫‪t1‬‬

‫ﻤﻊ ﺍﻟﻌﻠﻡ ﺃﻥ ﻫﺫﺍ ﺍﻟﺘﻔﺎﻀل ﻫﻭ ﺘﻔﺎﻀل ﻜﻠﻲ ﻭﻴﺅﺨﺫ ﺒﺜﺒﺎﺕ ﺍﻟﻁﺎﻗﺔ ‪ E‬ﺃﻱ‪:‬‬
‫‪E   (T  V )  0‬‬ ‫)‪(4.35‬‬
‫ﻭﻤﻥ ﺍﻟﻀﺭﻭﺭﻱ ﺍﻟﺘﺄﻜﻴﺩ ﺃﻥ ﺍﻟﺘﻔﺎﻀل )‪ (4.34‬ﻴﺨﺘﻠﻑ ﺠﺫﺭﻴﺎﹰ ﻋﻥ ﺍﻟﺘﻔﺎﻀل‬
‫ﻓﻲ ﺍﻟﻤﺒﺩﺃ ﺍﻟﺴﺎﺒﻕ ﻓﻬﻭ ﻟﻥ ﻴﻜﻭﻥ ﻤﺘﺴﺎﻭﻱ ﺍﻟﺯﻤﻥ )‪ ‬ﻭﻟﻴﺱ ‪ (‬ﻭﻻﺒﺩ ﺃﻥ ﻴﺤﻘﻕ‬

‫‪- 123 -‬‬

‫)‪Create PDF files without this message by purchasing novaPDF printer (http://www.novapdf.com‬‬
‫ﺍﻟﻌﻼﻗﺔ )‪ .(4.35‬ﺇﻥ ‪ V‬ﺘﺎﺒﻊ ﻟﻠﻤﻜﺎﻥ؛ ﻓﻬﻭ ﻴﺘﻐﻴﺭ ﻤﻥ ﻨﻘﻁﺔ ﻷﺨﺭﻯ ﻭﺒﺎﻟﺘﺎﻟﻲ ﻓﺈﻥ ‪T‬‬
‫ﺘﺘﻐﻴﺭ ﺃﻴﻀﺎﹰ ﺒﺤﻴﺙ ﻴﺒﻘﻰ ﻤﺠﻤﻭﻋﻬﻤﺎ ﺜﺎﺒﺘﺎﹰ‪ .‬ﻭﺃﻥ ﺍﻟﻤﺴﺎﺭ ﺍﻟﺤﻘﻴﻘﻲ ﺍﻟﺫﻱ ﻴﻭﺍﻓﻕ‬
‫ﺍﻟﺤﺭﻜﺔ ﺍﻟﺘﻲ ﺘﺤﺩﺙ ﻓﻌﻼﹰ ﻟﻥ ﻴﺘﻘﺎﻁﻊ ﻤﻊ ﺃﻱ ﻤﺴﺎﺭ ﺍﻓﺘﺭﺍﻀﻲ ﺁﺨﺭ ﺒﻴﻥ ﺍﻟﻨﻘﻁﺘﻴﻥ ‪A‬‬
‫ﻭ ‪ B‬ﻷﻨﻬﻤﺎ ﻟﻭ ﺘﻘﺎﻁﻌﺎ ﻷﻤﻜﻥ ﺭﺴﻡ ﻤﻤﺎﺱ ﻭﺍﺤﺩ ﻟﻤﻨﺤﻨﻴﻴﻥ ﻤﻥ ﻨﻘﻁﺔ ﻭﺍﺤﺩﺓ‪.‬‬
‫ﻭﻟﺒﺭﻫﺎﻥ ﺫﻟﻙ ﻨﺩﺭﺱ ﺤﺭﻜﺔ ﺫﺍﺕ ﺒﻌﺩ ﻭﺍﺤﺩ ﻁﺎﻗﺘﻬﺎ ﺍﻟﻜﻠﻴﺔ ‪:E‬‬
‫‪1‬‬
‫)‪(4.36‬‬
‫‪2‬‬
‫‪E‬‬ ‫)‪a (q)q  V (q‬‬
‫‪2‬‬
‫ﻭﻤﻨﻪ ﻨﺤﺼل ﻋﻠﻰ ﻤﻴل ﺍﻟﻤﻤﺎﺱ ﻟﻠﻤﻨﺤﻨﻲ ) ‪: q (t‬‬
‫‪dq‬‬ ‫‪2‬‬
‫)‪(4.37‬‬
‫‪‬‬
‫‪q ‬‬ ‫‪‬‬ ‫‪E  V(q )‬‬
‫‪dt‬‬ ‫)‪a ( q‬‬
‫ﻭﺒﻤﺎ ﺃﻥ )‪ V(q‬ﻫﻭ ﺘﺎﺒﻊ ﻭﺤﻴﺩ ﺍﻟﺘﻌﻴﻴﻥ )ﻴﺄﺨﺫ ﻗﻴﻤﺔ ﻭﺍﺤﺩﺓ ﻓﻲ ﻜل ﻨﻘﻁﺔ(‬
‫ﻨﺴﺘﻁﻴﻊ ﺍﻟﺘﺄﻜﻴﺩ ﺃﻨﻪ ﺇﺫﺍ ﺘﻘﺎﻁﻊ ﺍﻟﻤﻨﺤﻨﻲ ﺍﻟﻤﻤﺜل ﻟﻠﺤﺭﻜﺔ ﺍﻟﺤﻘﻴﻘﻴﺔ ﻤﻊ ﺃﻱ ﻤﻨﺤﻨﻲ ﺁﺨﺭ‬
‫ﺃﻴﻀﺎﹰ‪ .‬ﻭﻫﺫﺍ‬ ‫ﻓﻲ ﻨﻘﻁﺔ ﻤﺎ ‪ M‬ﻓﺈﻥ ‪ V‬ﺒﺄﺨﺫ ﺍﻟﻘﻴﻤﺔ ﻨﻔﺴﻬﺎ ﻟﻠﻤﻨﺤﻨﻴﻴﻥ ﻭﺒﺎﻟﺘﺎﻟﻲ‬
‫‪dq‬‬
‫‪dt‬‬
‫ﻴﻌﻨﻲ ﺭﺴﻡ ﻤﻤﺎﺱ ﻭﺍﺤﺩ ﻟﻤﻨﺤﻨﻴﻴﻥ ﻤﺨﺘﻠﻔﻴﻥ )ﺍﻟﺤﻘﻴﻘﻲ ﻭﺍﻻﻓﺘﺭﺍﻀﻲ( ﻤﻥ ﺍﻟﻨﻘﻁﺔ ‪M‬‬
‫ﻨﻔﺴﻬﺎ‪ ،‬ﻭﻫﺫﺍ ﻏﻴﺭ ﻤﻌﻘﻭل ﻭﺒﺎﻟﺘﺎﻟﻲ ﻟﻥ ﻴﺘﻘﺎﻁﻌﺎ‪.‬‬
‫ﻟﻨﺤﺴﺏ ﺍﻵﻥ ‪ W‬ﺍﻋﺘﻤﺎﺩﺍﹰ ﻋﻠﻰ ﺍﻟﻌﻼﻗﺔ )‪.(4.33‬‬
‫‪W  S  Et 2  Et 1  S  Et 2‬‬ ‫)‪(4.38‬‬
‫ﻷﻥ ﻜل ﺍﻟﻤﺴﺎﺭﺍﺕ ﺘﻨﻁﻠﻕ ﻤﻥ ‪ A‬ﻓﻲ ﺍﻟﻠﺤﻅﺔ ﻨﻔﺴﻬﺎ )‪ (t1  0‬ﻭﻟﻜﻨﻬﺎ ﻟﻥ‬
‫ﺘﺼل ﻜﻠﻬﺎ ﺇﻟﻰ ‪ B‬ﻓﻲ ﺍﻟﻠﺤﻅﺔ ‪ t 2‬ﻷﻥ ﺍﻟﺘﻔﺎﻀل ﻏﻴﺭ ﻤﺘﺴﺎﻭﻱ ﺍﻟﺯﻤﻥ؛ ﻓﻜل ﻤﺴﺎﺭ‬
‫ﺍﻓﺘﺭﺍﻀﻲ ﻟﻪ ﺘﻔﺎﻀﻠﻪ ﺍﻟﺨﺎﺹ‪ ،‬ﻭﻁﺒﻘﺎﹰ ﻟـ )‪ (4.28‬ﻨﺠﺩ‪:‬‬
‫‪‬‬
‫‪S  S  S t 2  S  Lt 2  S  (T  V )t 2‬‬
‫‪ S  (T  E  T)t 2‬‬ ‫)‪(4.39‬‬
‫ﻭﺒﺎﻟﺘﺒﺩﻴل ﻓﻲ )‪ (4.38‬ﻨﺠﺩ‪:‬‬
‫‪W  S  2Tt 2‬‬ ‫)‪(4.40‬‬
‫ﻤﻥ ﺍﻟﺴﻬل ﺤﺴﺎﺏ ‪ S‬ﺍﻋﺘﻤﺎﺩﺍﹰ ﻋﻠﻰ ﺘﻌﺭﻴﻔﻬﺎ )‪ (4.2‬ﺤﻴﺙ ﻨﺠﺩ‪:‬‬

‫‪- 124 -‬‬

‫)‪Create PDF files without this message by purchasing novaPDF printer (http://www.novapdf.com‬‬
t2 n t2 n
 T V  T 
S      q dt 
   q j .dt (4.41)
 q j q j  j
t 1 j 1  t 1 j 1 q j

:‫ﻨﺤﺴﺏ ﺍﻟﺘﻜﺎﻤل ﺍﻟﺜﺎﻨﻲ ﺒﺎﻟﺘﺠﺯﺌﺔ‬


t2 t2 t2 n
n
T  T d T
 
q j .dt  q
q j j
   q j dt 
dt
t1 q j j 1 t1 t 1 j 1 q j

‫ ﺍﻟﻤﻭﺠﻭﺩﺓ ﻓﻲ ﺍﻟﺤﺩ ﺍﻷﻭل ﻤﻥ ﺍﻟﻁﺭﻑ ﺍﻟﺜﺎﻨﻲ ﺒﻘﻴﻤﺘﻬﺎ ﻤﻥ‬q j ‫ﺜﻡ ﻨﺒﺩل‬


:‫( ﻓﻨﺠﺩ‬4.28)
t2 n n t
T  T t2 T  t 2 2 n d T
  q q j .dt    q j |t 1   q j t |t 1    q j dt  dt
t 1 j 1 j j 1 q j q j t 1 j 1 q j

(4.42)
‫ ﺃﻤﺎ ﺍﻟﺤﺩ‬،‫ﻭﺍﻟﺤﺩ ﺍﻷﻭل ﻤﻥ ﺍﻟﻁﺭﻑ ﺍﻟﺜﺎﻨﻲ ﻴﺴﺎﻭﻱ ﺍﻟﺼﻔﺭ ﻁﺒﻘﺎﹰ ﻟﻨﺹ ﺍﻟﻤﺒﺩﺃ‬
t 1 ‫ ﻓﻲ ﺍﻟﻠﺤﻅﺔ ﻨﻔﺴﻬﺎ‬A ‫ )ﻜل ﺍﻟﻤﺴﺎﺭﺍﺕ ﺘﺒﺩﺃ ﻤﻥ‬2Tt 2 ‫ﺍﻟﺫﻱ ﻴﻠﻴﻪ ﻓﻴﺴﺎﻭﻱ‬
‫ ﺇﺫﺍ‬‫ ( ﻭﻋﻨﺩﺌﺫ‬t 2 ‫ ﻓﻲ ﺍﻟﻠﺤﻅﺔ‬B ‫ ﻭﻟﻜﻨﻬﺎ ﻟﻥ ﺘﺼل ﻜﻠﻬﺎ ﺇﻟﻰ‬2Tt 1  0 ‫ﻭﺒﺎﻟﺘﺎﻟﻲ‬
:‫ ﺍﻟﺘﺎﻟﻴﺔ‬S ‫( ﻨﺤﺼل ﻋﻠﻰ‬4.41) ‫ﺒﺩﻟﻨﺎ ﻓﻲ‬
t2 n  
d T T V 
S        q .dt  2Tt 2 (4.43)
i 1
 dt  q
q j q j  j
t1  j 
:‫( ﻓﻨﺠﺩ‬4.40) ‫ ﺒﻘﻴﻤﺘﻬﺎ ﺍﻟﺴﺎﺒﻘﺔ ﻓﻲ‬S ‫ﻨﺒﺩل ﺃﺨﻴﺭﺍﹰ‬
t2 n  
d t T V 
W        q .dt  0 (4.44)
 dt  q q  j
t1 j 1 q j j j 
:‫ ﻴﺠﺏ ﺃﻥ ﺘﺘﺤﻘﻕ ﺍﻟﻤﻌﺎﺩﻟﺔ‬t ‫ ﻭ‬q j ‫ﻭﻟﻜﻲ ﺘﺘﺤﻘﻕ ﻫﺫﻩ ﺍﻟﻌﻼﻗﺔ ﻤﻬﻤﺎ ﻜﺎﻨﺕ‬
d T T V
  , (i  1, 2,...., n ) (4.45)
dt q  q j q j
j

.‫ﻭﻫﻲ ﻤﻌﺎﺩﻻﺕ ﻻﻏﺭﺍﻨﺞ ﻤﺎ ﻴﺅﻜﺩ ﺼﺤﺔ ﺍﻟﻤﺒﺩﺃ‬

- 125 -

Create PDF files without this message by purchasing novaPDF printer (http://www.novapdf.com)
‫‪ - 32‬ﺍﻟﺼﻴﻎ ﺍﳌﺨﺘﻠﻔﺔ ﻟﻠﻤﺒﺪﺃ ﺍﻟﺴﺎﺑﻖ‪:‬‬
‫ﻴﻌﺒﺭ ﻋﻥ ﺍﻟﻤﺒﺩﺃ ﺍﻟﺴﺎﺒﻕ ﺒﺼﻴﻎ ﻤﺨﺘﻠﻔﺔ؛ ﻓﻘﺩ ﻭﻀﻊ ﻤﻭﺒﺭﺘﻭﻱ ﺍﻟﻁﺎﻗﺔ ﺍﻟﺤﺭﻜﻴﺔ‬
‫ﺒﺸﻜل ﺁﺨﺭ ﻭﻟﺫﻟﻙ ﺼﺎﻍ ﺍﻟﻤﺒﺩﺃ ﺭﻴﺎﻀﻴﺎﹰ ﻜﻤﺎﻴﻠﻲ‪:‬‬
‫‪t2‬‬ ‫‪t2‬‬ ‫‪t2‬‬ ‫‪‬‬ ‫‪‬‬
‫‪W   2Tdt ‬‬ ‫‪2‬‬
‫‪  m i v i dt ‬‬ ‫‪  m i v i . v i dt‬‬ ‫)‪(4.46‬‬
‫‪t1‬‬ ‫‪t1‬‬ ‫‪i‬‬ ‫‪t1‬‬ ‫‪i‬‬

‫‪t2 n‬‬
‫‪ ‬‬
‫‪‬‬ ‫‪  m i vi . d i‬‬
‫‪t 1 i 1‬‬

‫ﺤﻴﺙ ‪ / di /‬ﻫﻭ ﻁﻭل ﺍﻟﻘﻭﺱ‪ .‬ﻴﺤﺴﺏ ﺍﻟﺘﻜﺎﻤل ﺍﻟﺴﺎﺒﻕ ﺒﻴﻥ ﺍﻟﻨﻘﻁﺘﻴﻥ ‪ A‬ﻭ‬
‫‪ B‬ﺍﻟﻤﻘﺎﺒﻠﺘﻴﻥ ﻟﻭﻀﻊ ﺍﻟﺠﻤﻠﺔ ﺍﻟﻤﺩﺭﻭﺴﺔ ﻓﻲ ﺍﻟﻠﺤﻅﺘﻴﻥ ‪ t 1‬ﻭ ‪ t 2‬ﻓﻲ ﺍﻟﻔﻀﺎﺀ‬
‫‪. E 3N 1‬‬
‫ﻋﺒﺭ ﺠﺎﻜﻭﺒﻲ ﻋﻥ ﺍﻟﺘﻜﺎﻤل ﺍﻟﺴﺎﺒﻕ ﺒﻭﺍﺴﻁﺔ ﺘﻜﺎﻤل ﺍﻟﻁﺎﻗﺔ ﻜﻤﺎﻴﻠﻲ‪:‬‬
‫‪2‬‬
‫‪N‬‬ ‫‪ d‬‬ ‫‪N‬‬ ‫‪‬‬
‫‪  12‬‬ ‫)‪(4.47‬‬
‫‪2‬‬ ‫‪2‬‬
‫‪2T   m i v   m i  i‬‬
‫‪i‬‬ ‫‪‬‬ ‫‪ m .d‬‬ ‫‪i‬‬ ‫‪i‬‬
‫‪i 1‬‬ ‫‪i 1‬‬ ‫‪ dt‬‬ ‫‪‬‬ ‫‪dt‬‬ ‫‪i‬‬

‫ﻨﻌﺒﺭ ﻋﻥ ﺍﻟﻁﺎﻗﺔ ﺍﻟﺤﺭﻜﻴﺔ ﺒﺩﻻﻟﺔ ‪ E‬ﻭ ‪ V‬ﺜﻡ ﻨﺤﺴﺏ ﺍﻟﺯﻤﻥ ﻤﻥ )‪(4.47‬‬


‫ﻓﻨﺠﺩ‪:‬‬
‫‪N‬‬ ‫‪2‬‬
‫‪ m i d i‬‬
‫‪dt ‬‬ ‫‪i‬‬ ‫)‪(4.48‬‬
‫)‪2(E  V‬‬
‫ﺜﻡ ﻨﺒﺩل ‪ dt‬ﺒﻘﻴﻤﺘﻬﺎ ﻫﺫﻩ ﻓﻲ )‪ (4.46‬ﻓﻨﺠﺩ‪:‬‬
‫‪1‬‬
‫‪ m d 2  2‬‬
‫‪t2‬‬ ‫‪B‬‬ ‫‪ i i ‬‬ ‫‪B‬‬
‫‪2‬‬
‫‪W   2Tdt   2(E  V ) i‬‬ ‫) ‪   2( E  V‬‬ ‫‪ m i di‬‬
‫‪ 2( E  V ) ‬‬ ‫‪i‬‬
‫‪t1‬‬ ‫‪A‬‬ ‫‪A‬‬
‫‪‬‬ ‫‪‬‬
‫‪‬‬ ‫‪‬‬
‫)‪(4.49‬‬
‫ﺃﻤﺎ ﺇﺫﺍ ﻋﺒﺭﻨﺎ ﻋﻥ ﺍﻟﻁﺎﻗﺔ ﺍﻟﺤﺭﻜﻴﺔ ﺒﺩﻻﻟﺔ ﺍﻟﺴﺭﻉ ﺍﻟﻤﻌﻤﻤﺔ )ﺭﺍﺠﻊ ﺍﻟﻔﺼل‬
‫ﺍﻟﺜﺎﻨﻲ( ﻁﺒﻘﺎﹰ ﻟﻠﻌﻼﻗﺔ‪:‬‬

‫‪- 126 -‬‬

‫)‪Create PDF files without this message by purchasing novaPDF printer (http://www.novapdf.com‬‬
‫‪1‬‬ ‫‪1‬‬
‫)‪(4.50‬‬
‫‪ ‬‬
‫‪T‬‬
‫‪2‬‬
‫‪‬‬ ‫‪a ijq i q j ‬‬
‫‪2‬‬ ‫‪ a ijdq i dq j‬‬
‫‪2dt‬‬
‫ﻓﺈﻨﻨﺎ ﻨﺠﺩ ﻋﻭﻀﺎﹰ ﻋﻥ )‪) (4.49‬ﺒﻌﺩ ﺍﻟﺘﻌﺒﻴﺭ ﻋﻥ ﺍﻟﻜﻤﻭﻥ ﺒﺩﻻﻟﺔ ﺍﻹﺤﺩﺍﺜﻴﺎﺕ‬
‫ﺍﻟﻤﻌﻤﻤﺔ ‪ ( q j‬ﺍﻟﻤﻌﺎﺩﻟﺔ ﺍﻟﺘﺎﻟﻴﺔ‪:‬‬
‫‪B‬‬
‫‪W‬‬ ‫‪ 2E  V(q1 , q 2 ,.., q n  a ijdq i dq j‬‬ ‫)‪(4.51‬‬
‫‪A‬‬ ‫‪ij‬‬

‫ﻭﻓﻲ ﺍﻟﺤﺎﻟﺔ ﺍﻟﺨﺎﺼﺔ ﻋﻨﺩﻤﺎ ﻴﻨﻌﺩﻡ ﺍﻟﻜﻤﻭﻥ )ﺍﻟﺠﻤﻠﺔ ﺍﻟﻤﺩﺭﻭﺴﺔ ﺤﺭﺓ( ﻴﻤﻜﻥ‬
‫ﺍﺨﺭﺍﺝ ‪ E‬ﺨﺎﺭﺝ ﺍﻟﺘﻜﺎﻤل ﺜﻡ ﺇﻫﻤﺎﻟﻬﺎ ﻷﻥ ﺍﻟﻤﺒﺩﺃ ﻴﻨﺹ ﻋﻠﻰ ﺃﻥ ‪ . W  0‬ﻭﻫﻜﺫﺍ‬
‫ﻨﺤﺼل ﻋﻠﻰ ﺍﻟﺼﻴﻐﺔ ﺍﻟﺘﺎﻟﻴﺔ ﻟـ ‪:W‬‬
‫‪B‬‬ ‫‪N‬‬ ‫‪B‬‬ ‫‪n‬‬
‫)‪(4.52‬‬
‫‪2‬‬
‫‪W‬‬ ‫‪‬‬ ‫‪ m i d i‬‬ ‫‪‬‬ ‫‪  a ijdq i dq j‬‬
‫‪A i 1‬‬ ‫‪A i, j 1‬‬

‫ﻟﻜﻥ ﺍﻟﻤﻘﺩﺍﺭ ﺍﻟﻤﺠﺫﻭﺭ ﻟﻴﺱ ﺇﻻ ﻤﺭﺒﻊ ﻁﻭل ﻗﻭﺱ ﻓﻲ ﻓﻀﺎﺀ ﺫﻱ ‪ n‬ﺒﻌﺩﺍﹰ ﺃﻱ‬
‫‪ d‬ﻭﻤﻨﻪ ﻨﺠﺩ‪:‬‬
‫‪2‬‬

‫‪B‬‬
‫‪W   d‬‬ ‫)‪(4.53‬‬
‫‪A‬‬
‫ﻭﻫﻜﺫﺍ ﺘﺅﻭل ﺤﺭﻜﺔ ﺠﻤﻠﺔ ﻤﺎﺩﻴﺔ ﻫﻭﻟﻭﻨﻭﻤﻴﺔ ﻏﻴﺭ ﺨﺎﻀﻌﺔ ﻷﻱ ﺍﺭﺘﺒﺎﻁﺎﺕ ﺃﻭ‬
‫ﻗﻭﻯ )ﺤﺭﻜﺔ ﻋﻁﺎﻟﻴﺔ( ﺇﻟﻰ ﻤﺴﺄﻟﺔ ﺨﻁﻭﻁ ﺠﻴﻭﺩﻴﺯﻴﺔ )ﺭﺍﺠﻊ ﺍﻟﻔﻘﺭﺓ ﺍﻷﺨﻴﺭﺓ ﻤﻥ ﻫﺫﺍ‬
‫ﺍﻟﻔﺼل( ﻷﻥ ﺍﻟﺤﺭﻜﺔ ﺍﻟﺤﻘﻴﻘﻴﺔ ﺘﺤﻘﻕ ﺍﻟﻤﺒﺩﺃ )‪ .(4.34‬ﻭﺒﺎﻟﺘﺎﻟﻲ ﻴﻤﻜﻥ ﻨﺹ ﻤﺒﺩﺃ‬
‫ﻤﻭﻴﺭﺘﻭﻱ‪ -‬ﺠﺎﻜﻭﺒﻲ ﻓﻲ ﻫﺫﻩ ﺍﻟﺤﺎﻟﺔ ﻜﻤﺎﻴﻠﻲ‪:‬‬
‫ﺘﺤﺩﺙ ﺍﻟﺤﺭﻜﺔ ﺍﻟﻌﻁﺎﻟﻴﺔ ﻟﺠﻤﻠﺔ ﻤﺎﺩﻴﺔ ﺒﺤﻴﺙ ﺘﺘﺤﺭﻙ ﻨﻘﻁﻬﺎ ﻋﻠﻰ ﺨﻁﻭﻁ‬
‫ﺠﻴﻭﺩﻴﺯﻴﺔ )ﺃﻗﺼﺭ ﻁﺭﻴﻕ( ﻓﻲ ﻓﻀﺎﺀ ﺫﻱ ‪ n‬ﺒﻌﺩﺍﹰ‪.‬‬
‫ﻭﺍﻟﺠﺩﻴﺭ ﺒﺎﻟﺫﻜﺭ ﺃﻨﻪ ﻴﻤﻜﻥ ﺭﺩ ﺍﻟﺤﺭﻜﺔ ﺇﻟﻰ ﺤﺭﻜﺔ ﻋﻠﻰ ﺨﻁﻭﻁ ﺠﻴﻭﺩﻴﺯﻴﺔ‬
‫ﺤﺘﻰ ﻟﻭ ﻟﻡ ﻴﻨﻌﺩﻡ ﺍﻟﻜﻤﻭﻥ ﻭﻟﻜﻥ ﺒﺸﺭﻁ ﺃﻥ ﻻﻴﺘﺒﻊ ﺍﻟﺯﻤﻥ ﺒﺸﻜل ﺼﺭﻴﺢ‪.‬‬
‫ﻓﻲ ﻫﺫﻩ ﺍﻟﺤﺎﻟﺔ ﻴﻜﻭﻥ‪:‬‬
‫‪n‬‬ ‫‪n‬‬
‫‪2(E  V)  a ijdq i dq j   b ijdqi dq j‬‬ ‫)‪(4.54‬‬
‫‪i 1‬‬ ‫‪i 1‬‬

‫‪- 127 -‬‬

‫)‪Create PDF files without this message by purchasing novaPDF printer (http://www.novapdf.com‬‬
‫ﺤﻴﺙ‪:‬‬
‫‪b ij  2(E  V )a ij‬‬ ‫)‪(4.55‬‬
‫ﻭﺒﺎﻟﺘﺎﻟﻲ ﻴﻤﻜﻥ ﻭﻀﻊ ﺘﺎﺒﻊ ﻓﻌل ﻻﻏﺭﺍﻨﺞ ﻜﻤﺎﻴﻠﻲ‪:‬‬
‫‪B n‬‬ ‫‪‬‬
‫‪W     b ijdq i dq j ‬‬ ‫)‪(4.56‬‬
‫‪‬‬
‫‪A i, j 1‬‬
‫‪‬‬
‫‪‬‬
‫ﻭﻤﻨﻪ ﻨﺴﺘﻨﺘﺞ ﺃﻨﻪ ﻴﻤﻜﻥ ﺩﺭﺍﺴﺔ ﺤﺭﻜﺔ ﺠﻤﻠﺔ ﻤﺎﺩﻴﺔ ﻫﻭﻟﻭﻨﻭﻤﻴﺔ ﻜﺄﻨﻬﺎ ﺤﺭﻜﺔ‬
‫ﻋﻠﻰ ﺨﻁﻭﻁ ﺠﻴﻭﺩﻴﺯﻴﺔ ﻓﻲ ﻓﺭﺍﻍ "ﺭﻴﻤﺎﻨﻲ" ﻴﻌﺭﻑ ﺒﺎﻟﻌﻼﻗﺔ‪:‬‬
‫‪n‬‬
‫)‪(4.57‬‬
‫‪2‬‬
‫‪d ‬‬ ‫‪ b ijdq i dq j‬‬
‫‪i, j 1‬‬

‫ﻟﻌل ﻤﻥ ﺍﻟﻤﻔﻴﺩ ﻓﻲ ﻨﻬﺎﻴﺔ ﻫﺫﻩ ﺍﻟﻔﻘﺭﺓ ﺍﻟﺘﺄﻜﻴﺩ ﻋﻠﻰ ﺍﻟﺘﺸﺎﺒﻪ ﺍﻟﺤﺎﺼل ﺒﻴﻥ ﻫﺫﺍ‬
‫ﺍﻟﻤﺒﺩﺃ ﻭﻤﺒﺩﺃ "ﻓﺭﻤﺎ" ﻓﻲ ﺍﻟﻀﻭﺀ ﺍﻟﻬﻨﺩﺴﻲ ﺍﻟﺫﻱ ﻴﻨﺹ ﻋﻠﻰ ﺃﻥ ﺍﻟﺸﻌﺎﻉ ﺍﻟﻀﻭﺌﻲ‬
‫ﻴﺴﻠﻙ ﺍﻟﻁﺭﻴﻕ ﺍﻟﺫﻱ ﻴﺴﺘﻐﺭﻕ ﺃﻗﺼﺭ ﺯﻤﻥ ﻤﻤﻜﻥ ﺒﻴﻥ ﻨﻘﻁﺘﻴﻥ ‪ A‬ﻭ‪) B‬ﻨﻬﺎﻴﺔ‬
‫ﺼﻐﺭﻯ( ﺃﻱ ﺃﻥ‪:‬‬
‫‪B‬‬ ‫‪B‬‬
‫‪d 1‬‬
‫‪t AB   dt ‬‬ ‫‪‬‬ ‫‪  n (x , y, z )d‬‬ ‫)‪(4.58‬‬
‫‪v‬‬ ‫‪C‬‬
‫‪A‬‬ ‫‪A‬‬
‫ﺤﻴﺙ ‪ n‬ﻗﺭﻴﻨﺔ ﺍﻨﻜﺴﺎﺭ ﺍﻟﻭﺴﻁ ﺍﻟﺫﻱ ﻴﻨﺘﺸﺭ ﻓﻴﻪ ﺍﻟﺸﻌﺎﻉ ﺍﻟﻀﻭﺌﻲ‪ C ،‬ﺴﺭﻋﺔ‬
‫ﺍﻟﻀﻭﺀ ﻓﻲ ﺍﻟﺨﻼﺀ‪.‬‬
‫ﻟﻨﺄﺨﺫ‪ ،‬ﻋﻠﻰ ﺴﺒﻴل ﺍﻟﻤﺜﺎل‪ ،‬ﺠﺴﻴﻤﺎﹰ ﻭﺍﺤﺩﺍﹰ ﻴﺘﺤﺭﻙ ﻓﻲ ﻓﺭﺍﻍ ﺜﻼﺜﻲ‪ .‬ﺇﻥ ﻤﺭﺒ ﻊ‬
‫ﻫﻭ‪:‬‬ ‫‪dt‬‬ ‫ﺯﻤﻥ‬ ‫ﺨﻼل‬ ‫ﺍﻟﺠﺴﻴﻡ‬ ‫ﻫﺫﺍ‬ ‫ﻴﺠﺘﺎﺯﻩ‬ ‫ﺍﻟﺫﻱ‬ ‫ﺍﻟﻘﻭﺱ‬ ‫ﻁﻭل‬
‫‪ d  dx  dy  dz‬ﻭﺒﺎﻟﺘﺎﻟﻲ ﻓﺘﺎﺒﻊ ﺍﻟﻔﻌل ﺍﻷﺼﻐﺭ ﻟﻪ ﻫﻭ‪:‬‬
‫‪2‬‬ ‫‪2‬‬ ‫‪2‬‬ ‫‪2‬‬

‫‪B‬‬ ‫‪B‬‬
‫)‪2m(E  V)d (4.59‬‬
‫‪2‬‬ ‫‪2‬‬ ‫‪2‬‬
‫‪W‬‬ ‫‪‬‬ ‫‪2m(E  V) dx  dy  dz ‬‬ ‫‪‬‬
‫‪A‬‬ ‫‪A‬‬
‫ﻭﺒﺎﻟﻤﻘﺎﺭﻨﺔ ﻤﻊ )‪ (4.58‬ﻨﺤﺼل ﻋﻠﻰ ﻗﺭﻴﻨﺔ ﺍﻨﻜﺴﺎﺭ ﺍﻟﻭﺴﻁ ﺒﺩﻻﻟﺔ ﺍﻟﻁﺎﻗﺔ‬
‫ﻭﺍﻟﻜﻤﻭﻥ‪:‬‬
‫) ‪n  C1 E  V( x, y, z‬‬ ‫)‪(4.60‬‬

‫‪- 128 -‬‬

‫)‪Create PDF files without this message by purchasing novaPDF printer (http://www.novapdf.com‬‬
‫‪1‬‬
‫‪‬‬
‫‪.M‬‬ ‫‪2 L1T‬‬ ‫ﺤﻴﺙ ‪ C1‬ﺜﺎﺒﺕ ﺃﺒﻌﺎﺩﻩ‬
‫ﻭﻫﻜﺫﺍ ﻨﺭﻯ ﺃﻥ ﺤﺭﻜﺔ ﻤﺎ ﻓﻲ ﺍﻟﻤﻴﻜﺎﻨﻴﻙ ﺍﻟﺘﻘﻠﻴﺩﻱ ﻓﻲ ﺤﻘل ﻤﺴﺘﻘﺭ ﺘﻘﺎﺒل‬
‫ﻤﺴﺄﻟﺔ ﻓﻲ ﺍﻟﻀﻭﺀ ﺍﻟﻬﻨﺩﺴﻲ‪ ،‬ﻭﺍﻟﻌﻜﺱ ﺼﺤﻴﺢ‪.‬‬
‫‪ -33‬ﻣﺒﺪﺃ ﺍﻟﻘﺴﺮ ﺍﻷﺻﻐﺮ )ﻣﺒﺪﺃ ﻏﻮﺹ(‪:‬‬
‫ﺘﺘﺠﻠﻰ ﺃﻫﻤﻴﺔ ﻫﺫﺍ ﺍﻟﻤﺒﺩﺃ ﻓﻲ ﺇﻤﻜﺎﻨﻴﺔ ﺍﻟﺤﺼﻭل ‪،‬ﺍﻨﻁﻼﻗﺎﹰ ﻤﻨﻪ‪ ،‬ﻋﻠﻰ ﻤﻌﺎﺩﻟﺔ‬
‫ﺍﻟﺤﺭﻜﺔ ﻟﺠﻤﻠﺔ ﻤﺎﺩﻴﺔ ﻓﻲ ﺍﻟﺤﺎﻟﺔ ﺍﻟﻌﺎﻤﺔ ﺴﻭﺍﺀ‪ ‬ﻜﺎﻨﺕ ﺍﻻﺭﺘﺒﺎﻁﺎﺕ ﻫﻭﻟﻭﺘﻭﻤﻴﺔ ﺃﻭ‬
‫ﻻﻫﻭﻟﻭﻨﻭﻤﻴﺔ‪.‬‬
‫ﻟﻔﻬﻡ ﻤﺒﺩﺃ ﻏﻭﺹ ﻻﺒﺩ ﺃﻭﻻﹰ ﻤﻥ ﺘﻌﺭﻴﻑ ﺍﻟﻘﺴﺭ )‪ (Compulsion‬ﺍﻟﺫﻱ ﻴﻌﺭﻑ‬
‫ﻜﻤﺎﻴﻠﻲ‪:‬‬
‫ﺍﻟﻘﺴﺭ ﻫﻭ ﺍﻨﺤﺭﺍﻑ ﺍﻟﺠﻤﻠﺔ ﺍﻟﻤﺎﺩﻴﺔ ﺍﻟﻤﺘﺤﺭﻜﺔ ﺘﺤﺕ ﺘﺄﺜﻴﺭ ﻗﻭﻯ ﻓﻌﺎﻟﺔ ‪Fi‬‬

‫ﻭﺍﻟﺨﺎﻀﻌﺔ ﻻﺭﺘﺒﺎﻁﺎﺕ ﻫﻭﻟﻭﻨﻭﻤﻴﺔ ﻭﻻﻫﻭﻟﻭﻨﻭﻤﻴﺔ ﻋﻥ ﺤﺭﻜﺘﻬﺎ ﺍﻟﻁﺒﻴﻌﻴﺔ ﺍﻟﺘﻲ ﻜﺎﻥ‬


‫ﻴﻤﻜﻥ ﺃﻥ ﺘﺤﺩﺙ ﻟﻬﺎ ﺘﺤﺕ ﺘﺄﺜﻴﺭ ﺍﻟﻘﻭﻯ ﺍﻟﻔﻌﺎﻟﺔ ﻓﻘﻁ‪ ،‬ﺍﻋﺘﺒﺎﺭﺍﹰ ﻤﻥ ﻟﺤﻅﺔ ﻤﻌﻴﻨﺔ‪ ،‬ﻓﻴﻤﺎ‬
‫ﻟﻭﺤﺭﺭﻨﺎ ﺍﻟﺠﻤﻠﺔ ﻤﻥ ﺍﻻﺭﺘﺒﺎﻁﺎﺕ ﻓﻲ ﺘﻠﻙ ﺍﻟﻠﺤﻅﺔ ﺒﺎﻟﺫﺍﺕ‪.‬‬
‫ﻟﺘﻭﻀﻴﺢ ﻤﻔﻬﻭﻡ ﺍﻟﻘﺴﺭ ﻨﻔﺭﺽ ﺠﺴﻴﻤﺎﹰ ﻤﻥ ﺍﻟﺠﻤﻠﺔ ﻜﺘﻠﺘﻪ ‪ mi‬ﻤﻭﺠﻭﺩﺍﹰ ﻓﻲ‬
‫ﺍﻟﻨﻘﻁﺔ ) ‪ M i ( x i , y i , z i‬ﻓﻲ ﺍﻟﻠﺤﻅﺔ ‪ t‬ﺸﻜل )‪ .(4.4‬ﺇﺫﺍ ﺘﺤﺭﺭ ﺍﻟﺠﺴﻴﻡ ﻤﻥ ﺍﻟﻘﻭﻯ‬
‫ﺍﻟﻔﻌﺎﻟﺔ ﻭﺍﻻﺭﺘﺒﺎﻁﺎﺕ ﻓﺈﻨﻪ ﺴﻴﻨﺘﻘل ﺨﻼل ﺯﻤﻥ ﺼﻐﻴﺭ ‪ ‬ﺇﻟﻰ ﺍﻟﻨﻘﻁﺔ ‪ Ai‬ﺒﺤﻴﺙ‬
‫ﻴﻜﻭﻥ )ﺍﻨﻅﺭ ﺍﻟﺸﻜل(‪:‬‬

‫ﺍﻟﺸﻜل )‪(4.4‬‬

‫‪- 129 -‬‬

‫)‪Create PDF files without this message by purchasing novaPDF printer (http://www.novapdf.com‬‬
‫‪‬‬ ‫‪‬‬
‫‪M i Ai  v i ‬‬ ‫)‪(4.61‬‬
‫‪‬‬
‫ﺃﻤﺎ ﺇﺫﺍ ﺘﺤﺭﻙ ﺍﻟﺠﺴﻴﻡ ﺘﺤﺕ ﺘﺄﺜﻴﺭ ﺍﻟﻘﻭﻯ ﺍﻟﻔﻌﺎﻟﺔ ‪ Fi‬ﺩﻭﻥ ﺃﻥ ﻴﺨﻀﻊ ﻷﻱ‬
‫ﺍﺭﺘﺒﺎﻁﺎﺕ ﻓﺴﻴﻨﺘﻘل ﺇﻟﻰ ﺍﻟﻨﻘﻁﺔ ‪ Bi‬ﺒﺤﻴﺙ ﻴﻜﻭﻥ‪:‬‬
‫‪‬‬
‫‪‬‬
‫‪‬‬ ‫‪‬‬ ‫‪F 2‬‬
‫‪M i Bi  v i   i ‬‬ ‫)‪(4.62‬‬
‫‪m‬‬ ‫‪i‬‬
‫‪‬‬
‫ﻭﺃﺨﻴﺭﺍﹰ ﺇﺫﺍ ﺘﺤﺭﻙ ﺍﻟﺠﺴﻴﻡ ﺘﺤﺕ ﺘﺄﺜﻴﺭ ﺍﻟﻘﻭﻯ ﺍﻟﻔﻌﺎﻟﺔ ‪ Fi‬ﻭﺍﻻﺭﺘﺒﺎﻁﺎﺕ‬
‫ﻓﺴﻴﻨﺘﻘل ﺇﻟﻰ ﺍﻟﻨﻘﻁﺔ ‪ Bi‬ﺒﺤﻴﺙ ﻴﻜﻭﻥ‪:‬‬
‫‪‬‬‫‪‬‬ ‫‪‬‬ ‫‪1  2‬‬
‫‪M i C i  vi   w i ‬‬ ‫)‪(4.63‬‬
‫‪2‬‬
‫‪‬‬
‫‪‬‬ ‫‪‬‬
‫ﺤﻴﺙ ‪ Wi  ri‬ﻫﻭ ﺍﻟﺘﺴﺎﺭﻉ ﺍﻟﺫﻱ ﻴﻜﺘﺴﺒﻪ ﺍﻟﺠﺴﻴﻡ ﻨﺘﻴﺠﺔ ﻟﺘﺄﺜﻴﺭ ﺍﻟﻘﻭﻯ‬
‫‪‬‬ ‫‪‬‬
‫ﺍﻟﻔﻌﺎﻟﺔ ‪ Fi‬ﻭﻗﻭﻯ ﺍﻻﺭﺘﺒﺎﻁ ‪ . R i‬ﻭﻋﻨﺩﺌﺫ‪ ‬ﻴﻌﻁﻰ ﺍﻻﻨﺤﺭﺍﻑ ﺍﻟﻤﻌﺭﻑ ﺴﺎﺒﻘﺎﹰ‪،‬‬
‫‪‬‬
‫ﺒﺎﻟﻤﺘﺠﻪ ‪ Bi Ci‬ﺍﻟﺘﺎﻟﻲ‪:‬‬
‫‪ ‬‬ ‫‪‬‬
‫‪‬‬ ‫‪‬‬
‫‪‬‬ ‫‪‬‬
‫‪‬‬ ‫‪1    Fi‬‬ ‫‪‬‬
‫‪Bi C i  M i C i  M i Bi   ri ‬‬ ‫‪‬‬ ‫)‪(4.64‬‬
‫‪2‬‬ ‫‪mi‬‬ ‫‪‬‬
‫‪‬‬ ‫‪‬‬
‫‪‬‬ ‫‪‬‬
‫ﺍﺴﺘﺨﺩﻡ ﻏﻭﺹ ﻤﻘﺩﺍﺭﺍﹰ ﻴﺘﻨﺎﺴﺏ ﻤﻊ ﺍﻻﻨﺤﺭﺍﻑ ﺍﻟﺴﺎﺒﻕ؛ ﻓﻌﺭﻑ ﺍﻟﻘﺴﺭ ﺍﻟﺫﻱ‬
‫ﻴﺘﻌﺭﺽ ﻟﻪ ﺍﻟﺠﺴﻴﻡ ‪ M i‬ﺒﺎﻟﻌﻼﻗﺔ‪:‬‬
‫‪2‬‬
‫‪  ‬‬
‫‪1    Fi ‬‬
‫‪ZWi  m i ri ‬‬ ‫)‪(4.65‬‬
‫‪2 ‬‬ ‫‪mi ‬‬
‫‪‬‬ ‫‪‬‬
‫‪‬‬ ‫‪‬‬
‫ﺃﻤﺎ ﺍﻟﻘﺴﺭ ﺍﻟﺫﻱ ﺘﺘﻌﺭﺽ ﻟﻪ ﺠﻤﻠﺔ ﺍﻟﺠﺴﻴﻤﺎﺕ ﻓﻬﻭ‪:‬‬
‫‪‬‬ ‫‪2‬‬
‫‪N‬‬ ‫‪‬‬ ‫‪‬‬
‫‪1   Fi‬‬
‫‪ZWi   m i  ri ‬‬
‫‪‬‬
‫‪‬‬ ‫)‪(4.66‬‬
‫‪i 1 2‬‬ ‫‪‬‬ ‫‪mi‬‬
‫‪‬‬ ‫‪‬‬

‫‪- 130 -‬‬

‫)‪Create PDF files without this message by purchasing novaPDF printer (http://www.novapdf.com‬‬
‫ﺃﻭ ﺒﺩﻻﻟﺔ ﺍﻹﺤﺩﺍﺜﻴﺎﺕ‪:‬‬
‫‪‬‬ ‫‪2‬‬ ‫‪2‬‬ ‫‪‬‬
‫‪1  N  X i‬‬ ‫‪‬‬ ‫‪‬‬ ‫‪y ‬‬ ‫‪‬‬ ‫‪z ‬‬
‫)‪   y    i    z   i  (4.67‬‬
‫‪ZW ‬‬ ‫‪x  m‬‬
‫‪2  i 1 i‬‬ ‫‪‬‬ ‫‪ i‬‬ ‫‪m i ‬‬ ‫‪ i‬‬ ‫‪m i ‬‬
‫‪‬‬ ‫‪i‬‬ ‫‪‬‬ ‫‪‬‬ ‫‪‬‬ ‫‪‬‬
‫ﻴﻨﺹ ﻤﺒﺩﺃ ﻏﻭﺹ ﻋﻠﻰ ﻤﺎﻴﻠﻲ‪:‬‬
‫ﺘﺤﺩﺙ ﺤﺭﻜﺔ ﺠﻤﻠﺔ ﻤﺎﺩﻴﺔ ﺨﺎﻀﻌﺔ ﻻﺭﺘﺒﺎﻁﺎﺕ ﻫﻭﻟﻭﻨﻭﻤﻴﺔ ﻭﻻﻫﻭﻟﻭﻨﻭﻤﻴﺔ‬
‫ﺒﺤﻴﺙ ﻴﻜﻭﻥ ﺍﻟﻘﺴﺭ ﺍﻟﻤﻌﺭﻑ ﺒﺎﻟﻌﻼﻗﺔ )‪ (4.66‬ﺃﺼﻐﺭﻴﺎﹰ‪:‬‬
‫‪ZW  0‬‬ ‫)‪(4.68‬‬
‫ﻫﺫﺍ ﻤﻊ ﺍﻟﻌﻠﻡ ﺃﻥ ﺍﻟﺘﻔﺎﻀل ﺍﻟﺴﺎﺒﻕ ﻴﺠﺏ ﺃﻥ ﻴﺸﻤل ﺍﻟﺘﺴﺎﺭﻉ ﻓﻘﻁ‪ ،‬ﻭﻟﻬﺫﺍ‬
‫ﻴﺴﻤﻰ ﺍﻟﺘﻔﺎﻀل ﺍﻟﻐﻭﺼﻲ )ﻨﺴﺒﺔ ﺇﻟﻰ ﺍﻟﻌﺎﻟﻡ ﻏﻭﺹ( ﺃﻱ ﺃﻨﻪ ﻴﺤﺴﺏ ﻁﺒﻘﺎﹰ ﻟﻠﻌﻼﻗﺎﺕ‪:‬‬
‫‪‬‬
‫‪‬‬ ‫‪‬‬ ‫‪‬‬ ‫‪‬‬
‫‪ ri   v i  0 ,  Fi  0 ,  ri  0‬‬ ‫)‪(4.69‬‬
‫ﻭﻫﻜﺫﺍ ﻴﺼﺎﻍ ﺍﻟﻤﺒﺩﺃ ﺭﻴﺎﻀﻴﺎﹰ ﺒﺎﻟﺸﻜل‪:‬‬
‫‪‬‬ ‫‪‬‬
‫‪N ‬‬ ‫‪  ‬‬
‫‪ZW    Fi  m i ri . ri  0‬‬ ‫)‪(4.70a‬‬
‫‪i 1‬‬ ‫‪‬‬
‫‪‬‬
‫‪‬‬ ‫‪‬‬
‫ﻭﻤﻨﻪ ﻨﺤﺼل ﻋﻠﻰ ﺍﻟﺼﻴﻐﺔ ﺍﻟﺭﻴﺎﻀﻴﺔ ﻟﻠﻤﺒﺩﺃ ﺒﺎﻹﺤﺩﺍﺜﻴﺎﺕ ﺍﻟﺩﻴﻜﺎﺭﺘﻴﺔ‪:‬‬
‫‪N  ‬‬
‫‪    ‬‬ ‫‪   ‬‬ ‫‪   ‬‬
‫‪‬‬ ‫‪ X i  m i x i x i   Yi  m i y i y i   Zi  m i z i z i   0‬‬
‫‪i 1 ‬‬ ‫‪‬‬ ‫‪‬‬ ‫‪‬‬ ‫‪‬‬ ‫‪‬‬ ‫‪‬‬
‫)‪(4.70b‬‬
‫ﻣﺜﺎﻝ‪:‬‬
‫ﺍﺤﺴﺏ ﺘﺴﺎﺭﻉ ﺠﺴﻴﻡ ﺜﻘﻴل ﻴﻤﻜﻨﻪ ﺍﻟﺘﺤﺭﻙ ﺩﻭﻥ ﺴﺭﻋﺔ ﺍﺒﺘﺩﺍﺌﻴﺔ ﻋﻠﻰ ﻤﺴﺘ ﻭﹴ‬
‫ﻴﻤﻴل ﻋﻠﻰ ﺍﻷﻓﻕ ﺒﺯﺍﻭﻴﺔ ‪.‬‬

‫‪- 131 -‬‬

‫)‪Create PDF files without this message by purchasing novaPDF printer (http://www.novapdf.com‬‬
‫ﺍﻟﺸﻜل )‪(4.5‬‬

‫ﻤﻥ ﺍﻟﻭﺍﻀﺢ )ﺸﻜل ‪ (4.5‬ﺃﻥ ﺍﻟﺤﺭﻜﺎﺕ ﺍﻻﻓﺘﺭﺍﻀﻴﺔ ﻟﻠﺠﺴﻴﻡ ﻴﻤﻜﻥ ﺃﻥ ﺘﻜﻭﻥ‬


‫ﺒﺎﻻﺘﺠﺎﻫﺎﺕ ‪ C1, C 2 , C 3 ,...‬ﺃﻤﺎ ﺇﺫﺍ ﺘﺤﺭﺭ ﺍﻟﺠﺴﻴﻡ ﻤﻥ ﺍﻟﺴﻁﺢ ﻓﻴﻤﻜﻥ ﺃﻥ ﻴﺴﻘﻁ‬
‫ﺘﺤﺕ ﺘﺄﺜﻴﺭ ﺜﻘﻠﻪ ﺇﻟﻰ ‪ .B‬ﺇﻥ ﻤﻌﺎﺩﻻﺕ ﺍﻟﺤﺭﻜﺔ ﺍﻟﻤﻤﻜﻨﺔ ﻫﻲ‪:‬‬

‫‪‬‬ ‫‪1  2‬‬ ‫‪ 1  2  1  2‬‬


‫‪MC1 ‬‬ ‫‪Wi  , MC 2  Wi  .... MB  g t‬‬
‫‪2‬‬ ‫‪2‬‬ ‫‪2‬‬
‫‪ ‬‬ ‫‪‬‬
‫ﻭﻴﺒﺩﻭ ﻤﻥ ﺍﻟﺸﻜل ﺃﻥ ﺃﺼﻐﺭ ﻗﺴﺭ ﻫﻭ ‪ MC1  MB  BC1‬ﻭﺒﺎﻟﺘﺎﻟﻲ‬
‫ﺴﻴﺘﺤﺭﻙ ﺍﻟﺠﺴﻴﻡ ﺒﺎﺘﺠﺎﻩ ‪ C1‬ﺤﻴﺙ ﻴﻜﻭﻥ‪:‬‬
‫‪1 2‬‬
‫‪/ MC1 /  MB sin  ‬‬ ‫‪gt sin ‬‬
‫‪2‬‬
‫ﺃﻱ ﺃﻥ ﺘﺴﺎﺭﻉ ﺍﻟﺠﺴﻴﻡ ﻋﻠﻰ ﺍﻟﺴﻁﺢ ﺍﻟﻤﺎﺌل ﻫﻭ‪:‬‬
‫‪W  W1  g sin ‬‬
‫ﻭﻤﻨﻪ ﺍﻟﻨﺘﻴﺠﺔ ﺍﻟﺘﺎﻟﻴﺔ‪:‬‬

‫ﻴﺘﺤﺭﻙ ﺠﺴﻴﻡ ﺜﻘﻴل ﻤﻭﻀﻭﻉ ﻋﻠﻰ ﺴﻁﺢ ﻤﺎﺌل ﺒﺯﺍﻭﻴﺔ ‪ ‬ﺇﺫﺍ ﺘﺭﻙ ﺩﻭﻥ‬
‫ﺴﺭﻋﺔ ﺍﺒﺘﺩﺍﺌﻴﺔ‪ ،‬ﻋﻠﻰ ﺃﻗﺭﺏ ﻤﻨﺤﺩﺭ ﺒﺘﺴﺎﺭﻉ ‪. W1  g sin ‬‬

‫‪- 132 -‬‬

‫)‪Create PDF files without this message by purchasing novaPDF printer (http://www.novapdf.com‬‬
‫ﺍﳊﺼﻮﻝ ﻋﻠﻰ ﻣﻌﺎﺩﻻﺕ ﺍﳊﺮﻛﺔ ﺍﻋﺘﻤﺎﺩﺍ ً ﻋﻠﻰ ﻣﺒﺪﺃ ﻏﻮﺹ‪:‬‬
‫‪- 34‬‬
‫ﻟﺘﻜﻥ ﺠﻤﻠﺔ ﻤﺎﺩﻴﺔ ﻤﺅﻟﻔﺔ ﻤﻥ ‪ N‬ﺠﺴﻴﻡ ﺇﺤﺩﺍﺜﻴﺎﺘﻬﺎ ) ‪ ( x i , y i , z i‬ﻭﻟﻨﺒﺤﺙ ﻋﻥ‬
‫ﻤﻌﺎﺩﻟﺔ ﺤﺭﻜﺘﻬﺎ‪.‬‬
‫ﺇﺫﺍ ﻜﺎﻨﺕ ﺍﻟﺠﻤﻠﺔ ﻁﻠﻴﻘﺔ )ﻻﺘﺅﺜﺭ ﻋﻠﻴﻬﺎ ﺃﻱ ﻗﻴﻭﺩ( ﺘﻜﻭﻥ ﺠﻤﻴﻊ ﺍﻨﺘﻘﺎﻻﺘﻬﺎ‬
‫ﺍﻻﻓﺘﺭﺍﻀﻴﺔ ﻤﺴﺘﻘﻠﺔ ﻭﺒﺎﻟﺘﺎﻟﻲ ﻨﺤﺼل ﻋﻠﻰ ﻤﻌﺎﺩﻻﺕ ﺍﻟﺤﺭﻜﺔ ﻤﺒﺎﺸﺭﺓﹰ ﻁﺒﻘﺎﹰ ﻟـ‬
‫)‪ (b4.70‬ﻭﻫﻲ‪:‬‬
‫‪‬‬ ‫‪‬‬
‫) ‪Fi  m i Wi (i  1, 2,..., N‬‬ ‫)‪(4.71‬‬
‫ﻓﻲ ﺍﻟﺤﺎﻟﺔ ﺍﻟﻌﺎﻤﺔ ﻋﻨﺩﻤﺎ ﺘﺘﺄﺜﺭ ﺍﻟﺠﻤﻠﺔ ﺒـ ‪ k‬ﺍﺭﺘﺒﺎﻁ ﻫﻭﻟﻭﻨﻭﻤﻲ ﻴﻌﺒﺭ ﻋﻨﻬﺎ‬
‫ﺒﺎﻟﻤﻌﺎﺩﻻﺕ‪:‬‬
‫)‪f  (...., x i , y i , z i ,...t )  0 , (  1, 2, ..., k‬‬ ‫)‪(4.72‬‬
‫ﻭ ‪ ‬ﺍﺭﺘﺒﺎﻁﺎﹰ ﻻﻫﻭﻟﻭﻨﻭﻤﻴﺎﹰ ﻴﻌﺒﺭ ﻋﻨﻬﺎ ﺒﺎﻟﻤﻌﺎﺩﻻﺕ‪:‬‬
‫‪‬‬ ‫‪‬‬
‫)‪  ....x i , y i , z i ,...., x i , y i , z i ,..., t  0, (  1, 2,..., ) (4.73‬‬

‫ﻭﻋﻨﺩﺌﺫ‪ ‬ﻨﺸﺘﻕ )‪ (4.72‬ﺍﺸﺘﻘﺎﻗﺎﹰ ﻜﻠﻴﺎﹰ ﺒﺎﻟﻨﺴﺒﺔ ﻟﻠﺯﻤﻥ ﻓﻨﺠﺩ‪:‬‬


‫‪N‬‬ ‫‪f‬‬ ‫‪f   f   ‬‬
‫)‪z i   x i , y i , z i , x i , z i , z i , t  (4.74‬‬
‫‪‬‬ ‫‪  ‬‬
‫‪ x x i‬‬ ‫‪‬‬
‫‪y i‬‬
‫‪yi ‬‬
‫‪z i‬‬ ‫‪‬‬ ‫‪‬‬
‫‪i 1‬‬ ‫‪i‬‬
‫ﻤﻊ ﺍﻟﻌﻠﻡ ﺃﻥ ‪ ‬ﻻﻴﺤﻭﻱ ﺍﻟﻤﺸﺘﻘﺎﺕ ﻤﻥ ﺍﻟﻤﺭﺘﺒﺔ ﺍﻟﺜﺎﻨﻴﺔ ﺒﺎﻟﻨﺴﺒﺔ ﻟﻺﺤﺩﺍﺜﻴﺎﺕ‪.‬‬
‫ﻨﺸﺘﻕ )‪ (4.73‬ﺒﺎﻟﻁﺭﻴﻘﺔ ﻨﻔﺴﻬﺎ ﻓﻨﺠﺩ‪:‬‬
‫‪N‬‬
‫‪‬‬ ‫‪ ‬‬ ‫‪‬‬
‫‪ x‬‬ ‫‪fx i  ‬‬ ‫‪y i  ‬‬ ‫)‪z i    (x i , y i , z i ,..., x i , y i , z i ,...t )  0 (4.75‬‬
‫‪i 1‬‬ ‫‪i‬‬ ‫‪y i‬‬ ‫‪z i‬‬
‫ﺤﻴﺙ ﻻﻴﺤﻭﻱ ﺍﻟﺘﺎﺒﻊ ‪ ‬ﺃﻱ ﻤﺸﺘﻘﺎﺕ ﻤﻥ ﺍﻟﻤﺭﺘﺒﺔ ﺍﻟﺜﺎﻨﻴﺔ ﺸﺄﻨﻪ ﻓﻲ ﺫﻟﻙ ﺸﺄﻥ‬
‫ﺍﻟﺘﺎﺒﻊ ‪ ،‬ﺃﻱ ﺃ ﻥ ﻜﻼﹰ ﻤﻨﻬﻤﺎ ﻻﻴﺤﻭﻱ ﺍﻟﺘﺴﺎﺭﻋﺎﺕ‪.‬‬
‫ﻨﻔﺎﻀل ﻜﻼﹰ ﻤﻥ )‪ (4.74‬ﻭ )‪ (4.75‬ﺘﻔﺎﻀﻼﹰ ﻏﻭﺼﻴﺎﹰ ﻓﻨﺠﺩ‪:‬‬
‫‪N‬‬
‫‪ f ‬‬ ‫‪f   f   ‬‬
‫‪  x‬‬ ‫‪x i  ‬‬ ‫‪y i ‬‬ ‫‪z i   0‬‬ ‫)‪(4.76‬‬
‫‪ i‬‬
‫‪i 1‬‬ ‫‪y i‬‬ ‫‪z i‬‬ ‫‪‬‬
‫‪N‬‬
‫‪ ‬‬ ‫‪‬‬ ‫‪   ‬‬
‫‪‬‬ ‫‪‬‬ ‫‪x i  ‬‬ ‫‪y i  ‬‬ ‫‪z i   0‬‬ ‫)‪(4.77‬‬
‫‪i 1  x i‬‬ ‫‪y i‬‬ ‫‪z i‬‬ ‫‪‬‬

‫‪- 133 -‬‬

‫)‪Create PDF files without this message by purchasing novaPDF printer (http://www.novapdf.com‬‬
‫ﺜﻡ ﻨﻀﺭﺏ ﻜﻼﹰ ﻤﻥ )‪ (4.76‬ﻭ )‪ (4.77‬ﺒﺎﻟﻤﻀﺎﺭﻴﺏ ‪  ‬ﻭ ‪ ‬ﻋﻠﻰ‬
‫ﺍﻟﺘﺭﺘﻴﺏ )ﻤﻀﺎﺭﻴﺏ ﻻﻏﺭﺍﻨﺞ( ﻭﻨﺠﻤﻌﻬﻤﺎ ﻤﻊ )‪ (4.74‬ﻓﻨﺠﺩ ﺍﻟﻤﻌﺎﺩﻟﺔ ﺍﻟﺘﺎﻟﻴﺔ‪:‬‬
‫‪N‬‬ ‫‪‬‬ ‫‪k‬‬
‫‪f ‬‬ ‫‪‬‬ ‫‪   ‬‬
‫‪‬‬ ‫‪‬‬‫‪‬‬‫‪‬‬ ‫‪X‬‬ ‫‪i‬‬ ‫‪‬‬ ‫‪m‬‬ ‫‪x‬‬
‫‪i i‬‬
‫‪‬‬
‫‪‬‬ ‫‪‬‬ ‫‪‬‬ ‫‪‬‬ ‫‪‬‬ ‫‪‬‬ ‫‪  x x i ‬‬
‫‪x i  1‬‬
‫‪‬‬
‫‪i 1 ‬‬‫‪‬‬ ‫‪ 1‬‬ ‫‪i ‬‬

‫‪‬‬ ‫‪k‬‬ ‫‪‬‬ ‫‪   ‬‬


‫‪f ‬‬
‫‪‬‬ ‫‪‬‬
‫‪‬‬
‫‪ Yi  m i y i    ‬‬ ‫‪  ‬‬
‫‪y i  1‬‬ ‫‪y i ‬‬
‫‪y i ‬‬
‫‪‬‬ ‫‪ 1‬‬

‫‪‬‬ ‫‪i‬‬
‫‪f ‬‬ ‫‪‬‬ ‫‪    ‬‬
‫‪‬‬ ‫‪‬‬ ‫)‪z i   0 (4.78‬‬
‫‪‬‬
‫‪Z‬‬
‫‪ i‬‬ ‫‪‬‬ ‫‪m‬‬ ‫‪z‬‬
‫‪i i‬‬ ‫‪‬‬ ‫‪‬‬ ‫‪‬‬ ‫‪‬‬ ‫‪‬‬ ‫‪‬‬
‫‪z i  1‬‬
‫‪‬‬
‫‪z i ‬‬ ‫‪‬‬
‫‪‬‬ ‫‪ 1‬‬

‫ﻭﻫﻲ ﻋﺒﺎﺭﺓ ﻋﻥ ‪ 3N‬ﻤﻌﺘﺭﻀﺔ ﺘﺤﻭﻱ ‪ 3N  k  ‬ﻤﺠﻬﻭﻻﹰ ﻫﻲ ﺇﺤﺩﺍﺜﻴﺎﺕ‬


‫ﺠﺴﻴﻤﺎﺕ ﺍﻟﺠﻤﻠﺔ ) ‪ ( x i , y i , z i‬ﻭﻋﺩﺩﺍﹰ ﻤﻥ ﻤﻀﺎﺭﻴﺏ ﻻﻏﺭﺍﻨﺞ ‪  ‬ﻭﻋﺩﺩﺍﹰ ﺁﺨﺭ ‪‬‬

‫ﻤﻥ ﺍﻟﻤﻀﺎﺭﻴﺏ ‪ . ‬ﻨﻌﻴﻥ ﻤﻥ )‪ (k  ‬ﻤﻌﺘﺭﻀﺔ ﻤﻨﻬﺎ‪ ،‬ﺒﻤﺴﺎﻭﺍﺘﻬﺎ ﺍﻟﺼﻔﺭ‪،‬‬


‫ﺍﻟﻤﺠﺎﻫﻴل ‪  ‬ﻭ ‪ . ‬ﻭﻴﺒﻘﻰ ﻋﻨﺩﺌ ﺫ‪ 3N  (k  ) ‬ﻤﻌﺘﺭﻀﺔ ﻴﻤﻜﻥ ﺃﻥ ﺘﺴﺎﻭﻱ‬
‫ﺍﻟﺼﻔﺭ ﺒﺴﺒﺏ ﺍﺴﺘﻘﻼﻟﻴﺔ ﺍﻻﻨﺘﻘﺎﻻﺕ ﺍﻟﺒﺎﻗﻴﺔ ﻓﻲ ﻓﻀﺎﺀ ﺍﻟﺘﺴﺎﺭﻋﺎﺕ‪.‬‬
‫ﻭﻫﻜﺫﺍ ﺘﻜﻭﻥ ﻟﺩﻴﻨﺎ ‪ 3N  (k  )‬ﻤﻌﺘﺭﻀﺔ ﺒﺎﻹﻀﺎﻓﺔ )‪ (k  ‬ﺃﺨﺭﻯ‬
‫‪ 3N  (k  )  (k   )  3N‬ﻤﻌﺘﺭﻀﺔ ﺘﺴﺎﻭﻱ ﺍﻟﺼﻔﺭ‪.‬‬ ‫ﻭﻴﻜﻭﻥ ﺍﻟﻤﺠﻤﻭﻉ‬
‫ﻨﻀﻴﻑ ﻟﻬﺎ ﻋﺩﺩﺍﹰ ‪ k‬ﻤﻥ ﻤﻌﺎﺩﻻﺕ ﺍﻻﺭﺘﺒﺎﻁ ﺍﻟﻬﻭﻟﻭﻨﻭﻤﻴﺔ ﻭﻋﺩﺩﺍﹰ ﺁﺨﺭ ‪ ‬ﻤﻥ‬
‫ﺍﻻﺭﺘﺒﺎﻁﺎﺕ ﺍﻟﻼﻫﻭﻟﻭﻨﻭﻤﻴﺔ ﻓﻨﺤﺼل ﺃﺨﻴﺭﺍﹰ ﻋﻠﻰ ‪ 3N  k  ‬ﻤﻌﺎﺩﻟﺔ ﻜﺎﻓﻴﺔ ﻟﺤﺴﺎﺏ‬
‫‪ 3N  k  ‬ﻤﺠﻬﻭﻻﹰ ﻫﻲ )‪ ( k  ‬ﻤﻥ ﻤﻀﺎﺭﻴﺏ ﻻﻏﺭﺍﻨﺞ ‪  ‬ﻭ ‪ ‬ﻭ ‪N3‬‬

‫ﺍﺤﺩﺍﺜﻲ ﺍﻟﺠﺴﻴﻤﺎﺕ ﺍﻟﺒﺎﻗﻴﺔ ﻭﻫﻲ ﺍﻟﻤﻌﺎﺩﻻﺕ ﻨﻔﺴﻬﺎ ﺍﻟﺘﻲ ﺭﺃﻴﻨﺎﻫﺎ ﻓﻲ ﺍﻟﻔﺼل ﺍﻷﻭل‬
‫)ﻤﻌﺎﺩﻻﺕ ﻻﻏﺭﺍﻨﺞ ﺍﻟﻨﻭﻉ ﺍﻷﻭل(‪.‬‬
‫‪ - 35‬ﻣﺒﺪﺃ ﺍﻟﺘﻘﻮﺱ ﺍﻷﺻﻐﺮ )ﻣﺒﺪﺃ ﻫﺮﺗﺰ(‪:‬‬
‫ﻟﺘﻜﻥ ﺠﻤﻠﺔ ﻤﺎﺩﻴﺔ ﻤﺅﻟﻔﺔ ﻤﻥ ‪ N‬ﺠﺴﻴﻡ ﺨﺎﻀﻌﺔ ﻻﺭﺘﺒﺎﻁﺎﺕ ﻫﻭﻟﻭﻨﻭﻤﻴﺔ ﻓﻘﻁ‬
‫ﻭﻤﺎ ﻤﻥ ﻗﻭﻯ ﻓﻌﺎﻟﺔ ﺘﺅﺜﺭ ﻓﻴﻬﺎ ﻭﻟﻬﺫﺍ ﺴﺘﻜﻭﻥ ﻁﺎﻗﺘﻬﺎ ﺍﻟﻜﻠﻴﺔ ﺜﺎﺒﺘﺔ‪ ،‬ﻭﻟﻤﺎ ﻜﺎﻥ ﺘﺎﺒﻊ‬
‫ﺍﻟﺠﻬﺩ ﺜﺎﺒﺘﺎﹰ ﺃﻴﻀﺎﹰ ﻓﻲ ﻫﺫﻩ ﺍﻟﺤﺎﻟﺔ ﻓﺎﻟﻁﺎﻗﺔ ﺍﻟﺤﺭﻜﻴﺔ ﺘﺒﻘﻰ ﺜﺎﺒﺘﺔ ﻭﺘﻌﻁﻰ ﺒﺎﻟﻌﻼﻗﺔ‪:‬‬
‫‪1‬‬
‫)‪(4.79‬‬
‫‪2‬‬
‫‪T‬‬ ‫‪‬‬
‫‪2 i‬‬
‫‪m i .v i‬‬

‫‪- 134 -‬‬

‫)‪Create PDF files without this message by purchasing novaPDF printer (http://www.novapdf.com‬‬
‫‪‬‬ ‫‪‬‬
‫ﻟﻨﻔﺭﺽ ﺇﺤﺩﺍﺜﻴﺎﺕ ﺠﺩﻴﺩﺓ ) ‪  i (i , i ,  i‬ﺒﺩﻻﹰ ﻤﻥ ) ‪ ri ( x i , y i , z i‬ﻁﺒﻘﺎﹰ‬
‫ﻟﻠﻌﻼﻗﺔ‪:‬‬
‫‪‬‬ ‫‪mi ‬‬ ‫‪mi‬‬ ‫‪mi‬‬ ‫‪mi‬‬
‫‪i ‬‬ ‫‪ri   i ‬‬ ‫‪x i i ‬‬ ‫‪yi , i ‬‬ ‫)‪z i (4.80‬‬
‫‪m‬‬ ‫‪m‬‬ ‫‪m‬‬ ‫‪m‬‬
‫ﺤﻴﺙ ‪ m‬ﻫﻲ ﺍﻟﻜﺘﻠﺔ ﺍﻟﻜﻠﻴﺔ ﻟﺠﻤﻠﺔ ﺍﻟﺠﺴﻴﻤﺎﺕ ) ‪ (m   mi‬ﻭﻟﻨﺸﻜل ﻓﻀﺎﺀﺍﹰ‬
‫ﻟ ﻠﺠﺴﻴﻤﺎﺕ‬ ‫ﺍﻟﺠﺩﻴﺩﺓ‬ ‫ﺍﻹﺤﺩﺍﺜﻴﺎﺕ‬ ‫ﻫﻲ‬ ‫ﻋﻨﺎﺼﺭﻩ‬ ‫‪E 3N‬‬ ‫ﺍﻗﻠﻴﺩﻴﺎﹰ‬
‫) ‪ . (1 , 1 , 1.... N , N ,  N‬ﺍﻥ ﻭﻀﻊ ﺍﻟﺠﻤﻠﺔ ﺍﻟﻤﺎﺩﻴﺔ ﻓﻲ ﻫﺫﺍ ﺍﻟﻔﻀﺎﺀ ﻴﺘﺤﺩﺩ‬
‫ﺒﻨﻘﻁﺔ ﻭﺍﺤﺩﺓ ‪ M‬ﻭﺇﺫﺍ ﺘﺤﺭﻜﺕ ﺠﻤﻠﺔ ﺍﻟﺠﺴﻴﻤﺎﺕ ﻓﺈﻥ ‪ M‬ﺘﺭﺴﻡ ﻤﺴﺎﺭﺍﹰ ﻓﻲ ﺍﻟﻔﻀﺎﺀ‬
‫‪ E 3N‬ﺘﻤﺜل ﻜل ﻨﻘﻁﺔ ﻓﻴﻪ ﻭﻀﻊ ﺍﻟﺠﺴﻴﻤﺎﺕ ﻓﻲ ﻟﺤﻅﺔ ﻤﺎ‪ .‬ﺇﻥ ﻁﻭل ﻗﻭﺱ ﺍﻟﻤﺴﺎﺭ‬
‫ﻓﻲ ﻫﺫﺍ ﺍﻟﻔﻀﺎﺀ ﻴﺴﺎﻭﻱ‪:‬‬
‫‪N‬‬ ‫‪N‬‬
‫)‪(4.81‬‬
‫‪2‬‬ ‫‪2‬‬ ‫‪2‬‬ ‫‪2‬‬ ‫‪2‬‬
‫‪d    d i‬‬ ‫‪   di  d i  d i ‬‬
‫‪i 1‬‬ ‫‪i 1‬‬
‫‪‬‬ ‫‪‬‬

‫ﻭﺒﺘﻘﺴﻴﻡ ﺍﻟﻁﺭﻓﻴﻥ ﻋﻠﻰ ‪ d‬ﻨﺠﺩ‪:‬‬


‫‪2‬‬

‫‪N‬‬ ‫‪2‬‬
‫‪d i‬‬
‫‪‬‬ ‫‪2‬‬
‫‪1‬‬ ‫)‪(4.82‬‬
‫‪i 1‬‬ ‫‪d‬‬
‫ﻭﺒﺎﻻﺸﺘﻘﺎﻕ ﺒﺎﻟﻨﺴﺒﺔ ﺇﻟﻰ ‪:‬‬
‫‪‬‬ ‫‪‬‬
‫‪N‬‬ ‫‪2‬‬
‫‪d i d i‬‬
‫‪2‬‬ ‫‪.‬‬
‫‪2‬‬
‫‪0‬‬ ‫)‪(4.83‬‬
‫‪i 1‬‬
‫‪d‬‬ ‫‪d‬‬
‫‪‬‬
‫‪2‬‬ ‫‪‬‬
‫‪d i‬‬ ‫‪d i‬‬
‫ﻴﺴﺎﻭﻱ ﺍﻟﺼﻔﺭ ﻭﺒﺎﻟﺘﺎﻟﻲ‬ ‫‪2‬‬
‫ﻭ‬ ‫ﺃﻱ ﺃﻥ ﺍﻟﺠﺩﺍﺀ ﺍﻟﻌﺩﺩﻱ ﻟﻠﻤﺘﺠﻬﻴﻥ‬
‫‪d‬‬ ‫‪d‬‬
‫‪‬‬
‫‪d 2‬‬
‫ﻫﻭ‬ ‫ﻫﻭ ﻤﺘﺠﻪ ﺍﻟﻤﻤﺎﺱ ﻓﺈﻥ‬ ‫ﻓﻬﻤﺎ ﻤﺘﻌﺎﻤﺩﺍﻥ ﻓﻲ ﺍﻟﻔﻀﺎﺀ ‪ E 3N‬ﻭﺒﻤﺎ ﺃﻥ‬
‫‪d‬‬
‫‪2‬‬
‫‪d‬‬ ‫‪d‬‬

‫‪ K ‬ﺤﻴﺙ ‪R‬‬ ‫ﻤﺘﺠﻪ ﺍﻟﻨﺎﻅﻡ ﺍﻷﺴﺎﺴﻲ ﻤﻊ ﺍﻟﻌﻠﻡ ﺃﻥ ﻁﻭﻴﻠﺘﻪ ﺘﺴﺎﻭﻱ ﺍﻟﺘﻘﻭﺱ‬


‫‪1‬‬
‫‪R‬‬
‫ﻨﺼﻑ ﻗﻁﺭ ﺍﻟﺘﻘﻭﺱ ﺃﻱ ﺃﻥ‪:‬‬

‫‪- 135 -‬‬

‫)‪Create PDF files without this message by purchasing novaPDF printer (http://www.novapdf.com‬‬
‫‪‬‬ ‫‪2‬‬ ‫‪2‬‬ ‫‪2‬‬ ‫‪2‬‬
‫‪N  d 2 ‬‬
‫‪ d 2‬‬ ‫‪‬‬ ‫‪ d 2‬‬ ‫‪‬‬ ‫‪ 2‬‬ ‫‪‬‬
‫‪1‬‬ ‫‪‬‬ ‫‪ N  d i‬‬
‫‪‬‬ ‫‪i‬‬ ‫‪‬‬ ‫‪‬‬ ‫‪‬‬ ‫‪‬‬
‫)‪(4.84‬‬
‫‪2‬‬ ‫‪i‬‬ ‫‪i‬‬
‫‪K ‬‬ ‫‪  ‬‬ ‫‪‬‬ ‫‪‬‬ ‫‪   2‬‬ ‫‪‬‬ ‫‪    2‬‬ ‫‪‬‬
‫‪2‬‬ ‫‪‬‬ ‫‪2‬‬ ‫‪‬‬ ‫‪‬‬ ‫‪2‬‬
‫‪R‬‬ ‫‪i 1  d ‬‬ ‫‪ d‬‬ ‫‪‬‬ ‫‪ d‬‬ ‫‪‬‬ ‫‪ i 1 d‬‬ ‫‪‬‬
‫‪‬‬ ‫‪‬‬
‫ﺼﺎﻍ ﻫﺭﺘﺯ ﻤﺒﺩﺃﻩ ﺍﻋﺘﻤﺎﺩﺍﹰ ﻋﻠﻰ ﺍﻟﻌﻼﻗﺔ ﺍﻟﺴﺎﺒﻘﺔ ﻭﺒﺸﻜل ﻤﺸﺎﺒﻪ ﻟﻤﺒﺩﺃ ﻨﻴﻭﺘﻥ‬
‫ﺍﻷﻭل ﺤﻴﺙ ﺃﻜﺩ ﺃﻥ ﺍﻟﺠﻤﻠﺔ ﺍﻟﻤﺎﺩﻴﺔ ﺍﻟﺘﻲ ﻁﺎﻗﺘﻬﺎ ﺍﻟﺤﺭﻜﻴﺔ ﺜﺎﺒﺘﺔ ﺘﺤﺎﻓﻅ ﻋﻠﻰ ﻭﻀﻌﻬﺎ‬
‫‪‬‬
‫ﺍﻟﺴﻜﻭﻨﻲ ﺃﻭ ﺃﻨﻬﺎ ﺘﺘﺤﺭﻙ ﺒﺤﺭﻜﺔ ﻤﺴﺘﻘﻴﻤﺔ ﻤﻨﺘﻅﻤﺔ ﻨﺭﻤﺯ ﻟﺴﺭﻋﺘﻬﺎ ‪ v‬ﻓﻲ ﺍﻟﻔﻀﺎﺀ‬
‫‪ E 3N‬ﻭﺒﺎﻟﺘﺎﻟﻲ ﻴﻜﻭﻥ ﺘﻘﻭﺱ ﻤﺴﺎﺭﻫﺎ ﺃﺼﻐﺭﻴ ﺎﹰ )ﻨﺼﻑ ﻗﻁﺭ ﺍﻟﺘﻘﻭﺱ ﻓﻲ ﻫﺫﻩ ﺍﻟﺤﺎﻟﺔ‬
‫ﻓﻲ ﺍﻟﻔﻀﺎﺀ ﺍﻟﻤﺫﻜﻭﺭ ﻴﺴﺎﻭﻱ ﺍﻟﻼﻨﻬﺎﻴﺔ ﻓﺎﻟﺘﻘﻭﺱ ﺃﺼﻐﺭﻱ ﻭﻴﺴﺎﻭﻱ ﺍﻟﺼﻔﺭ(‪.‬‬
‫ﻭﺍﻟﺠﺩﻴﺭ ﺒﺎﻟﺫﻜﺭ ﺃﻥ ﻋﺒﺎﺭﺓ ﺍﻟﻁﺎﻗﺔ ﺍﻟﺤﺭﻜﻴﺔ )‪ (4.79‬ﺘﺘﻀﻤﻥ ﺼﺤﺔ ﺍﻟﻤﺒﺩﺃ‬
‫ﻷﻨﻬﺎ ﺘﺘﻀﻤﻥ ﺜﺒﺎﺕ ﻫﺫﻩ ﺍﻟﻁﺎﻗﺔ ﻭﻟﻠﺘﺄﻜﺩ ﻤﻥ ﺫﻟﻙ ﻨﻘﺴﻡ ﻁﺭﻓﻲ )‪ (4.81‬ﻋﻠﻰ‬
‫‪2‬‬
‫‪dt‬‬
‫ﻓﻨﺠﺩ‪:‬‬
‫‪2‬‬ ‫‪N‬‬ ‫‪2‬‬
‫‪ d ‬‬ ‫‪d i‬‬
‫)‪(4.85‬‬
‫‪2‬‬
‫‪  v  2‬‬
‫‪ dt ‬‬ ‫‪i 1 dt‬‬
‫ﻭﻤﻥ ﺠﻬﺔ ﺜﺎﻨﻴﺔ ﻟﺩﻴﻨﺎ ﻤﻥ )‪:(4.79‬‬
‫‪2‬‬
‫‪mi 2‬‬ ‫‪ d 2‬‬ ‫‪‬‬
‫‪2T‬‬
‫)‪(4.86‬‬
‫‪‬‬ ‫‪‬‬ ‫‪2‬‬
‫‪‬‬ ‫‪vi    i‬‬ ‫‪  v  Const.‬‬
‫‪m‬‬ ‫‪m‬‬ ‫‪ dt‬‬ ‫‪‬‬
‫‪‬‬ ‫‪‬‬
‫ﺃﻱ ﺃﻥ ﻤﺭﺒﻊ ﺴﺭﻋﺔ ﺍﻟﻨﻘﻁﺔ ‪ ،M‬ﺍﻟﻤﻤﺜﻠﺔ ﻟﻭﻀﻊ ﺍﻟﺠﻤﻠﺔ ﻓﻲ ﺍﻟﻠﺤﻅﺔ ‪،t‬‬
‫ﻴﺴﺎﻭﻱ ﻀﻌﻑ ﺍﻟﻁﺎﻗﺔ ﺍﻟﺤﺭﻜﻴﺔ ﻤﻘﺴﻭﻤﺎﹰ ﻋﻠﻰ ﺍﻟﻜﺘﻠﺔ ﺍﻟﻜﻠﻴﺔ ﻟﻠﺠﻤﻠﺔ‪.‬‬
‫ﻤﺎﻫﻲ ﺍﻟﻌﻼﻗﺔ ﺒﻴﻥ ﻤﺒﺩﺃ ﻏﻭﺹ ﻭﻤﺒﺩﺃ ﻫﺭﺘﺯ؟‬
‫ﻴﻭﻀﻊ ﺍﻟﻘﺴﺭ ﻓﻲ ﺤﺎﻟﺔ ﺍﻨﻌﺩﺍﻡ ﺍﻟﻘﻭﻯ ﺍﻟﻔﻌﺎﻟﺔ ﻜﻤﺎﻴﻠﻲ‪:‬‬
‫‪1 N‬‬
‫)‪(4.87‬‬
‫‪2‬‬
‫‪ZW ‬‬ ‫‪‬‬
‫‪2 i 1‬‬
‫‪m i ri‬‬

‫ﻭﻤﻨﻪ‪:‬‬
‫‪2‬‬
‫‪ 2‬‬
‫‪ZW 1  d  i ‬‬
‫‪ ‬‬ ‫‪‬‬ ‫)‪(4.88‬‬
‫‪m‬‬ ‫‪2  dt 2 ‬‬
‫‪‬‬ ‫‪‬‬

‫‪- 136 -‬‬

‫)‪Create PDF files without this message by purchasing novaPDF printer (http://www.novapdf.com‬‬
‫‪4‬‬
‫‪ v   ‬ﻓﻨﺠﺩ‪ :‬ﻁﺒﻘﺎﹰ ﻟـ )‪:(4.84‬‬ ‫ﻨﻘﺴﻡ ﻁﺭﻓﻲ ﺍﻟﻌﻼﻗﺔ )‪ (4.88‬ﻋﻠﻰ‬
‫‪d‬‬
‫‪4‬‬
‫‪ dt ‬‬
‫‪2‬‬
‫‪ d 2 ‬‬
‫‪‬‬ ‫‪i‬‬
‫‪‬‬ ‫‪‬‬ ‫‪2‬‬
‫‪dt‬‬ ‫‪ 2‬‬ ‫‪‬‬
‫‪ZW 1 N ‬‬ ‫‪1  d i‬‬ ‫‪1 2‬‬
‫‪ ‬‬ ‫‪‬‬ ‫‪ ‬‬
‫‪‬‬
‫‪  2 K‬‬ ‫)‪(4.89‬‬
‫‪4‬‬ ‫‪2‬‬ ‫‪4‬‬ ‫‪2 i 1 d 2‬‬
‫‪mv‬‬ ‫‪i 1 ‬‬ ‫‪d‬‬‫‪‬‬ ‫‪‬‬ ‫‪‬‬ ‫‪‬‬
‫‪ ‬‬
‫‪ dt ‬‬
‫ﺃﻱ ﺃﻥ‪:‬‬
‫‪1‬‬
‫‪ZW ‬‬ ‫‪mv 4 K 2‬‬ ‫)‪(4.90‬‬
‫‪2‬‬
‫ﻭﻟﻤﺎ ﻜﺎﻨﺕ ‪ v‬ﺜﺎﺒﺘﺔ ﻓﺈﻨﻨﺎ ﻨﺠﺩ ﺒﺘﻔﺎﻀل ﻁﺭﻓﻲ )‪ (4.90‬ﺘﻔﺎﻀﻼﹰ ﻏﻭﺼﻴﺎﹰ‪:‬‬
‫‪4‬‬

‫‪1‬‬
‫)‪(4.91‬‬
‫‪4‬‬ ‫‪2‬‬
‫‪ZW ‬‬ ‫‪mv K  0‬‬
‫‪2‬‬
‫ﻭﺒﺎﻟﺘﺎﻟﻲ ﻴﻜﻭﻥ ﻤﺒﺩﺃ ﻫﺭﺘﺯ ﺤﺎﻟﺔ ﺨﺎﺼﺔ ﻤﻥ ﻤﺒﺩﺃ ﻏﻭﺹ‪.‬‬
‫‪ - 36‬ﻣﺒﺪﺃ ﺟﻮﺭﺩﻥ‪:‬‬
‫ﻴﻨﺹ ﻤﺒﺩﺃ ﺠﻭﺭﺩﻥ ﻋﻠﻰ ﻤﺎﻴﻠﻲ‪:‬‬
‫ﺘﺠﺭﻱ ﺤﺭﻜﺔ ﺠﻤﻠﺔ ﻤﺎﺩﻴﺔ ﺒﺤﻴﺙ ﻴﻜﻭﻥ ﻤﺠﻤﻭﻉ ﺍﻷﻋﻤﺎل ﺍﻻﻓﺘﺭﺍﻀﻴﺔ‬
‫‪‬‬ ‫‪‬‬ ‫‪‬‬ ‫‪‬‬
‫ﺍﻟﺠﺯﺌﻴﺔ ﻟﻠﻘﻭﺓ ﺍﻟﻀﺎﺌﻌﺔ ‪  Fi  m i Wi ‬ﻓﻲ ﻓﻀﺎﺀ ﺍﻟﺴﺭﻉ ‪ ، v i‬ﻴﺴﺎﻭﻱ ﺍﻟﺼﻔﺭ‪،‬‬
‫‪‬‬ ‫‪‬‬
‫ﻭﺒﺎﻟﺘﺎﻟﻲ ﻴﺼﺎﻍ ﺍﻟﻤﺒﺩﺃ ﺭﻴﺎﻀﻴﺎﹰ ﺒﺎﻟﻤﻌﺎﺩﻟﺔ‪:‬‬
‫‪N‬‬ ‫‪‬‬ ‫‪ ‬‬
‫‪  i 1 . v i  0‬‬
‫‪‬‬ ‫‪F‬‬ ‫‪‬‬ ‫‪m‬‬ ‫‪Wi‬‬ ‫)‪(4.92‬‬
‫‪i 1 ‬‬ ‫‪‬‬
‫ﻤﻊ ﺍﻟﻌﻠﻡ ﺃﻥ ﺍﻟﺘﻔﺎﻀل ﻁﺒﻘﺎﹰ ﻟﻬﺫﺍ ﺍﻟﻤﺒﺩﺃ ﻴﺤﻘﻕ ﺍﻟﻌﻼﻗﺎﺕ‪:‬‬
‫‪‬‬ ‫‪‬‬
‫‪‬‬ ‫‪‬‬ ‫‪‬‬ ‫‪‬‬ ‫‪‬‬ ‫‪‬‬
‫‪ri   ri   Wi   Fi  0,  ri   v i  0‬‬ ‫)‪(4.93‬‬
‫ﻟﻨﺘﺴﺎﺀل ﻓﻲ ﻨﻬﺎﻴﺔ ﻫﺫﺍ ﺍﻟﻔﺼل‪ :‬ﻫل ﻴﻭﺠﺩ ﻗﺎﺴﻡ ﻤﺸﺘﺭﻙ ﺒﻴﻥ ﺍﻟﻤﺒﺎﺩﻯﺀ ﺍﻟﺘﻲ‬
‫ﺭﺃﻴﻨﺎﻫﺎ ﻓﻲ ﺍﻟﻤﻴﻜﺎﻨﻴﻙ ﺍﻟﺘﺤﻠﻴﻠﻲ؟‬
‫ﻤﻥ ﺍﻟﺴﻬل ﺃﻥ ﻨﻼﺤﻅ ﺃﻨﻪ ﻴﻤﻜﻥ ﺍﻟﺤﺼﻭل ﻋﻠﻰ ﻤﺒﺩﺃ ﺍﻻﻨﺘﻘﺎﻻﺕ ﺍﻻﻓﺘﺭﺍﻀﻴﺔ‬
‫)ﻤﻌﺎﺩﻟﺔ ﺍﻟﻤﺒﻴﺭ – ﻻﻏﺭﺍﻨﺞ( ﻭﻤﺒﺩﺃ ﺠﻭﺭﺩﻥ ﻭﻤﺒﺩﺃ ﻏﻭﺹ ﺒﺎﺴﺘﺨﺩﺍﻡ ﻁﺭﻴﻘﺔ ﺘﻔﺎﻀل‬

‫‪- 137 -‬‬

‫)‪Create PDF files without this message by purchasing novaPDF printer (http://www.novapdf.com‬‬
‫ﻟﻜل ﻤﻨﻬﺎ؛ ﻓﻲ ﺍﻷﻭل؛ ﺍﻟﺫﻱ ﺘﻌﺭﻓﻨﺎ ﻋﻠﻴﻪ ﻓﻲ ﺍﻟﻔﺼل ﺍﻟﺜﺎﻨﻲ‪ ،‬ﻴﺘﻡ ﺍﻟﺘﻔﺎﻀل ﻁﺒﻘﺎﹰ‬
‫ﻟﻠﺸﺭﻭﻁ‪:‬‬
‫‪‬‬ ‫‪‬‬ ‫‪ ‬‬ ‫‪‬‬ ‫‪‬‬
‫‪ Fi   Wi   ri   v i  0 ,  ri  0‬‬ ‫)‪(4.94‬‬
‫ﻭﻋﻨﺩﺌﺫ‪ ‬ﺇﺫﺍ ﻓﺎﻀﻠﻨﺎ ﺍﻟﻤﻌﺎﺩﻟﺔ ﺍﻟﺘﺎﻟﻴﺔ )ﻁﺒﻘﺎﹰ ﻟﻠﺸﺭﻭﻁ ‪:(4.94‬‬
‫‪N‬‬ ‫‪‬‬ ‫‪ ‬‬
‫‪  i i i . ri  0‬‬
‫‪‬‬ ‫‪F‬‬ ‫‪‬‬ ‫‪m‬‬ ‫‪W‬‬ ‫)‪(4.95‬‬
‫‪i 1 ‬‬ ‫‪‬‬
‫ﻨﺤﺼل ﻋﻠﻰ ﻤﺒﺩﺃ ﺍﻻﻨﺘﻘﺎﻻﺕ ﺍﻻﻓﺘﺭﺍﻀﻴﺔ ﺍﻟﺘﺎﻟﻲ‪:‬‬
‫‪N‬‬ ‫‪‬‬ ‫‪  ‬‬
‫‪  i i i . ri  0‬‬
‫‪‬‬ ‫‪F‬‬ ‫‪‬‬ ‫‪m‬‬ ‫‪W‬‬ ‫)‪(4.96‬‬
‫‪i 1‬‬ ‫‪‬‬
‫ﺃﻤﺎ ﺇﺫﺍ ﻓﺎﻀﻠﻨﺎ ﺍﻟﻤﻌﺎﺩﻟﺔ‪:‬‬
‫‪N‬‬ ‫‪‬‬ ‫‪  ‬‬
‫‪  i i i . v i  0‬‬
‫‪‬‬ ‫‪F‬‬ ‫‪‬‬ ‫‪m‬‬ ‫‪W‬‬ ‫)‪(4.97‬‬
‫‪i 1‬‬ ‫‪‬‬
‫ﻀﻤﻥ ﺍﻟﺸﺭﻭﻁ )‪ (4.93‬ﻨﺤﺼل ﻋﻠﻰ ﻤﺒﺩﺃ ﺠﻭﺭﺩﻥ‪.‬‬
‫ﻭﺃﺨﻴﺭﺍﹰ ﺇﺫﺍ ﻓﺎﻀﻠﻨﺎ ﺍﻟﻤﻌﺎﺩﻟﺔ‪:‬‬
‫‪‬‬
‫‪N‬‬ ‫‪‬‬ ‫‪‬‬ ‫‪‬‬ ‫‪‬‬
‫‪  Fi  m i Wi . ri  0‬‬ ‫)‪(4.98‬‬
‫‪i 1‬‬ ‫‪‬‬
‫ﻀﻤﻥ ﺍﻟﺸﺭﻭﻁ‪:‬‬
‫‪‬‬ ‫‪‬‬
‫‪‬‬ ‫‪‬‬ ‫‪‬‬ ‫‪‬‬
‫‪ Fi   ri   ri  0 ,  ri  0‬‬ ‫)‪(4.99‬‬
‫ﻓﺈﻨﻨﺎ ﻨﺤﺼل ﻋﻠﻰ ﻤﺒﺩﺃ ﻏﻭﺹ ﺍﻟﺘﺎﻟﻲ‪:‬‬
‫‪‬‬
‫‪N‬‬ ‫‪‬‬ ‫‪‬‬ ‫‪‬‬ ‫‪‬‬
‫‪  Fi  m i Wi . ri‬‬ ‫‪0‬‬
‫‪i 1‬‬ ‫‪‬‬

‫ﻭﻨﻼﺤﻅ ﺃﻥ ﺍﻟﻘﻭﺓ ﺍﻟﻀﺎﺌﻌﺔ ‪ Fi  m i .Wi‬ﻫﻲ ﺍﻟﻌﺎﻤل ﺍﻟﻤﺸﺘﺭﻙ ﺒﻴﻥ ﺍﻟﻤﺒﺎﺩﻯﺀ‬


‫ﺍﻟﺜﻼﺜﺔ؛ ﻓﻲ ﺍﻷﻭل ﻴﻨﻌﺩﻡ ﻋﻤﻠﻬﺎ ﻓﻲ ﻓﻀﺎﺀ ﺍﻹﺤﺩﺍﺜﻴﺎﺕ ﻭﻓﻲ ﺍﻟﺜﺎﻨﻲ ﻴﻨﻌﺩﻡ ﻋﻤﻠﻬﺎ ﻓﻲ‬
‫ﻓﻀﺎﺀ ﺍﻟﺴﺭﻉ ﻭﻓﻲ ﺍﻟﺜﺎﻟﺙ ﻴﻨﻌﺩﻡ ﻋﻤﻠﻬﺎ ﻓﻲ ﻓﻀﺎﺀ ﺍﻟﺘﺴﺎﺭﻋﺎﺕ‪.‬‬

‫‪- 138 -‬‬

‫)‪Create PDF files without this message by purchasing novaPDF printer (http://www.novapdf.com‬‬
‫‪ - 37‬ﺍﳋﻄﻮﻁ ﺍﳉﻴﻮﺩﻳﺰﻳﺔ‪:‬‬
‫ﺃ‪ -‬ﺘﺭﺘﺒﻁ ﺍﻟﺨﻁﻭﻁ ﺍﻟﺠﻴﻭﺩﻴﺯﻴﺔ ﺍﺭﺘﺒﺎﻁﺎﹰ ﻭﺜﻴﻘﺎﹰ ﺒﻤﺒﺩﺃ ﺍﻟﺘﻐﻴﺭﺍﺕ ﻟﺫﻟﻙ ﻜﺎﻥ ﻤﻥ‬
‫ﺍﻟﻀﺭﻭﺭﻱ ﺍﻟﺘﻌﺭﻑ ﻋﻠﻴﻬﺎ ﻓﻲ ﻫﺫﺍ ﺍﻟﻔﺼل ﻻﺴﻴﻤﺎ ﺇﺫﺍ ﻋﻠﻤﻨﺎ ﺃﻥ ﻟﻬﺎ ﺘﻁﺒﻴﻘﺎﺕ ﻜﺜﻴﺭﺓ‬
‫ﻓﻲ ﺍﻟﻌﻠﻭﻡ ﺍﻟﺠﻐﺭﺍﻓﻴﺔ ﻭﺍﻟﻬﻨﺩﺴﻴﺔ‪.‬‬
‫ﻤﻥ ﺍﻟﻤﻌﻠﻭﻡ ﺃﻨﻪ ﻴﻤﻜﻥ ﺭﺴﻡ ﻋﺩﺩ ﻻﻨﻬﺎﺌﻲ ﻤﻥ ﺍﻟﻤﻨﺤﻨﻴﺎﺕ ﺘﺼل ﺒﻴﻥ ﻨﻘﻁﺘﻴﻥ‬
‫‪ A‬ﻭ ‪ B‬ﻤﻥ ﺴﻁﺢ ﻓﺭﺍﻏﻲ ) ‪ . ( x, y, z , t‬ﻭﺴﻨﺒﺤﺙ ﻓﻲ ﻫﺫﻩ ﺍﻟﻔﻘﺭﺓ ﻋﻥ ﺃﻗﺼﺭ‬
‫ﺍﻟﻨﻘﻁﺘﻴﻥ‬ ‫ﺃﻥ‬ ‫ﺒﻔﺭﺽ‬ ‫ﺃﻭﻻﹰ‬ ‫ﺍﻟﻤﺴﺄﻟﺔ‬ ‫ﻫﺫﻩ‬ ‫ﺴﻨﺤل‬ ‫ﻭﻟﻜﻨﻨﺎ‬ ‫ﺍﻟﻁﺭﻕ‪،‬‬
‫) ‪ B( x 2 , y 2 ) , A(x1 , y1‬ﻭﺍﻗﻌﺘﺎﻥ ﻓﻲ ﻤﺴﺘﻭﻱ ‪) xoy‬ﺸﻜل ‪ (4.6‬ﻭﺴﻨﺒﺤﺙ ﻋﻥ‬
‫ﻤﻌﺎﺩﻟﺔ ﺍﻟﻤﻨﺤﻨﻲ ) ‪ y  f (x‬ﺒﺤﻴﺙ ﻴﻜﻭﻥ ﺍﻟﺘﻜﺎﻤل ﺍﻟﻤﻨﺤﻨﻲ ﺍﻟﺘﺎﻟﻲ‪:‬‬

‫ﺍﻟﺸﻜل )‪(4.6‬‬
‫‪B‬‬
‫‪J   f ( y, y' , x)dx‬‬ ‫)‪(4.101‬‬
‫‪A‬‬

‫‪ y' ‬ﻭﻫﺫﺍ ﻴﻜﺎﻓﻰﺀ ﻟﻠﺸﺭﻁ ‪. J  0‬‬ ‫ﺃﺼﻐﺭﻴﺎﹰ ﻋﻠﻤﺎﹰ ﺃﻥ‬


‫‪dy‬‬
‫‪dx‬‬
‫ﻟﺤل ﻫﺫﻩ ﺍﻟﻤﺴﺄﻟﺔ ﻨﻤﻴﺯ ﺍﻟﻤﻨﺤﻨﻴﺎﺕ ﺒﻴﻥ ‪ A‬ﻭ ‪ B‬ﺒﻭﺴﻴﻁ ‪ ،‬ﻜﻤﺎ ﻓﻌﻠﻨﺎ ﻋﻨﺩ‬
‫ﺍﺴﺘﻨﺘﺎﺝ ﻤﺒﺩﺃ ﺍﻟﻔﻌل ﺍﻷﺼﻐﺭ‪ ،‬ﻁﺒﻘﺎﹰ ﻟﻠﻤﻌﺎﺩﻟﺔ‪:‬‬
‫) ‪y( x, )  y( x,0)  ( x‬‬ ‫)‪(4.102‬‬

‫‪- 139 -‬‬

‫)‪Create PDF files without this message by purchasing novaPDF printer (http://www.novapdf.com‬‬
‫ﺤﻴﺙ ‪   0‬ﻴﻭﺍﻓﻕ ﺍﻟﻤﻨﺤﻨﻲ ﺍﻟﻤﻁﻠﻭﺏ ﺃﻤﺎ ) ‪ (x‬ﻓﻬﻲ ﺩﺍﻟﺔ ﺘﻀﻤﻥ ﺍﻟﺘﻘﺎﺀ‬
‫ﺠﻤﻴﻊ ﺍﻟﻤﻨﺤﻨﻴﺎﺕ ﻓﻲ ‪ A‬ﻭ ‪ B‬ﻷﻨﻬﺎ ﺘﻨﻌﺩﻡ ﻓﻴﻬﻤﺎ‪ .‬ﻭﻋﻨﺩﺌﺫ‪ ‬ﻨﻀﻊ ﺍﻟﺘﻜﺎﻤل )‪(4.101‬‬
‫ﺒﺎﻟﺼﻴﻐﺔ ﺍﻟﺘﺎﻟﻴﺔ‪:‬‬
‫‪B‬‬
‫‪J()   f ( y, x,  ), y' ( x, ), x dx‬‬ ‫)‪(4.103‬‬
‫‪A‬‬
‫ﺜﻡ ﻨﻔﺎﻀل ﺒﺎﻟﻨﺴﺒﺔ ﺇﻟﻰ ‪ ‬ﻭﻨﻀﻊ )‪: J(‬‬
‫‪B‬‬
‫‪ f y‬‬ ‫‪f y' ‬‬
‫‪J()   ‬‬ ‫‪ ‬‬ ‫‪ dx  0‬‬ ‫)‪(4.104‬‬
‫‪‬‬ ‫‪y ‬‬ ‫‪y'  ‬‬
‫‪A‬‬
‫ﻨﺴﺘﻜﻤل ﺍﻟﺤﺩ ﺍﻟﺜﺎﻨﻲ ﺒﺎﻟﺘﺠﺯﺌﺔ ﻓﻨﺠﺩ‪:‬‬
‫‪B‬‬ ‫‪B‬‬ ‫‪B‬‬
‫‪f‬‬ ‫‪d f‬‬ ‫‪f d y‬‬ ‫‪f d‬‬
‫)‪ y' dx ydx (4.105‬‬
‫‪B‬‬
‫‪‬‬ ‫‪y‬‬ ‫‪  y‬‬ ‫‪dx‬‬ ‫‪ y' dx  dx ‬‬
‫'‪y‬‬ ‫‪A‬‬ ‫'‪dx y‬‬
‫‪A‬‬ ‫‪A‬‬ ‫‪A‬‬
‫ﻨﺒﺩل ﻓﻲ )‪ ،(4.104‬ﺒﻌﺩ ﻤﻼﺤﻅﺔ ﺍﻨﻌﺩﺍﻡ ﺍﻟﺤﺩ ﺍﻷﻭل‪ ،‬ﻓﻨﺠﺩ‪:‬‬
‫‪B‬‬
‫‪ f‬‬ ‫‪d f‬‬ ‫‪‬‬
‫‪J()    y ‬‬ ‫‪y dx  0‬‬
‫‪y‬‬ ‫‪dx y' ‬‬
‫‪A‬‬
‫ﻭﻟﻜﻲ ﻴﻨﻌﺩﻡ ﻫﺫﺍ ﺍﻟﺘﻜﺎﻤل ﺩﻭﻤﺎﹰ ﻴﺠﺏ ﺃﻥ ﻴﻨﻌﺩﻡ ﺍﻟﻤﺴﺘﻜﻤل‪:‬‬
‫‪ f‬‬ ‫‪d f ‬‬
‫‪ ‬‬ ‫‪y  0‬‬
‫‪‬‬ ‫‪‬‬‫‪y‬‬ ‫‪dx‬‬ ‫‪‬‬‫‪y‬‬ ‫'‬ ‫‪‬‬
‫ﻭﻟﻜﻲ ﻴﺘﺤﻘﻕ ﺫﻟﻙ ﻤﻬﻤﺎ ﻜﺎﻥ ‪ y‬ﻴﺠﺏ ﺃﻥ ﺘﺘﺤﻘﻕ ﺍﻟﻤﻌﺎﺩﻟﺔ‪:‬‬
‫‪d f f‬‬
‫‪‬‬ ‫‪0‬‬ ‫)‪(4.106‬‬
‫‪dx y' y‬‬
‫ﻭﻫﻲ ﻤﻌﺎﺩﻟﺔ ﺘﻔﺎﻀﻠﻴﺔ ﻤﻥ ﺍﻟﻤﺭﺘﺒﺔ ﺍﻷﻭﻟﻰ ﻴﺠﺏ ﺃﻥ ﺘﺤﻘﻘﻬﺎ ﺍﻟﺩﺍﻟﺔ ‪ f‬ﺍﻟﺘﻲ‬
‫ﺘﻤﺜل ﺍﻟﻤﻨﺤﻨﻲ ﺍﻟﻤﻁﻠﻭﺏ )ﺍﻟﺠﻴﻭﺩﻴﺯﻱ(‪ ،‬ﻭﺃﻥ ﺤل ﻫﺫﻩ ﺍﻟﻤﻌﺎﺩﻟﺔ ﻴﻘﻭﺩ ﺇ ﻟﻰ ﺤﺴﺎﺏ‬
‫ﻫﺫﻩ ﺍﻟﺩﺍﻟﺔ‪.‬‬
‫ﻏﻨﻲ ﻋﻥ ﺍﻟﺒﻴﺎﻥ ﺃﻥ ﻤﺒﺩﺃ ﺍﻟﻔﻌل ﺍﻷﺼﻐﺭ ﺍﻟﺫﻱ ﺭﺃﻴﻨﺎﻩ ﻓﻲ ﺒﺩﺍﻴﺔ ﻫﺫﻩ ﺍﻟﻔﺼل‬
‫ﻴﺩﺨل ﻀﻤﻥ ﺍﻁﺎﺭ ﺤﺴﺎﺏ ﺍﻟﻨﻬﺎﻴﺎﺕ ﻟﺩﺍﻟﺔ ﺍﻟﻔﻌل ‪ ،S‬ﻭﻴﻤﻜﻥ ﺍﻟﺤﺼﻭل ﻋﻠﻰ ﻤﻌﺎﺩﻻﺕ‬
‫ﻻﻏﺭﺍﻨﺞ ﻤﻥ ﻫﻨﺎ ﻤﺒﺎﺸﺭﺓﹰ ﺇﺫﺍ ﺍﺴﺘﺒﺩﻟﻨﺎ ﺍﻟﺩﺍﻟﺔ )‪ f (y, y' , x‬ﺒﺩﺍﻟﺔ ﻻﻏﺭﺍﻨﺞ‬
‫) ‪ L(q, q , t‬ﻭﺍﺴﺘﻜﻤﻠﻨﺎ ﺒﺎﻟﺯﻤﻥ ﻁﺒﻘﺎﹰ ﻟﻠﺸﺭﻭﻁ ﺍﻟﺴﺎﺒﻘﺔ‪.‬‬
‫‪‬‬

‫‪- 140 -‬‬

‫)‪Create PDF files without this message by purchasing novaPDF printer (http://www.novapdf.com‬‬
‫ﺏ ‪ -‬ﻤﻥ ﺍﻟﻤﻌﻠﻭﻡ ﻓﻲ ﺍﻟﻬﻨﺩﺴﺔ ﺍﻟﻤﺴﺘﻭﻴﺔ ﺃﻥ ﺃﻗﺼﺭ ﺒﻌﺩ ﺒﻴﻥ ﻨﻘﻁﺘﻴﻥ ﻫﻭ ﺍﻟﺨﻁ‬
‫ﺍﻟﻤﺴﺘﻘﻴﻡ ﺍﻟﻭﺍﺼل ﺒﻴﻨﻬﻤﺎ‪ .‬ﻓﻬل ﻴﻤﻜﻥ ﺍﻟﺘ ﺄﻜﺩ ﻤﻥ ﺫﻟﻙ ﻋﻠﻰ ﻀﻭﺀ ﻫﺫﻩ ﺍﻟﻔﻘﺭﺓ؟‬
‫ﺇﻥ ﻁﻭل ﻗﻭﺱ ﺍﻟﻤﻨﺤﻨﻲ ﺍﻟﻤﺭﺴﻭﻡ ﻋﻠﻰ ﻤﺴﺘﻭﻱ ﻫﻭ‪:‬‬
‫‪2‬‬ ‫‪2‬‬ ‫‪2‬‬ ‫‪2‬‬
‫‪d  dx  dy  (1  y '2 )dx‬‬
‫ﻭﻤﻨﻪ‪:‬‬
‫‪d  (1  y '2 )dx‬‬ ‫)‪(4.107‬‬
‫ﺃﻱ ﺃﻥ‪:‬‬
‫‪   1  y ' 2 dx‬‬

‫ﻓﺈﺫﺍ ﻗﺎﺭﻨﺎ )‪ (4.107‬ﻤﻊ )‪ (4.101‬ﻨﺠﺩ ﺃﻥ ﺍﻟﺩﺍﻟﺔ ‪ f‬ﺍﻟﺘﻲ ﻴﺠﺏ ﺃﻥ ﺘﺤﻘﻕ‬


‫ﺍﻟﻤﻌﺎﺩﻟﺔ )‪ (4.106‬ﻫﻲ‪:‬‬
‫‪'2‬‬
‫‪f  1 y‬‬
‫ﻨﺸﻜل ﺍﻟﻤﻌﺎﺩﻟﺔ ﺍﻟﺘﻲ ﺘﺤﻘﻘﻬﺎ ‪ f‬ﻓﻨﺠﺩ ﺃﻭﻻﹰ‪:‬‬
‫‪f‬‬ ‫‪f‬‬ ‫'‪y‬‬
‫‪0 ,‬‬ ‫‪‬‬
‫‪y‬‬ ‫'‪y‬‬ ‫‪'2‬‬
‫‪1 y‬‬
‫ﻭﻤﻨﻪ ﺍﻟﻤﻌﺎﺩﻟﺔ ﺍﻟﻤﻁﻠﻭﺒﺔ‪:‬‬
‫‪d‬‬ ‫'‪y‬‬ ‫'‪y‬‬
‫‪0‬‬ ‫‪C‬‬ ‫)‪(4.108‬‬
‫‪dx 1  y‬‬ ‫'‬ ‫‪2‬‬
‫‪1  y '2‬‬
‫ﻭﻤﻥ ﻫﺫﻩ ﺍﻟﻤﻌﺎﺩﻟﺔ ﻨﺤﺴﺏ ‪ 'y‬ﻓﻨﺠﺩ‪:‬‬
‫‪dy‬‬ ‫‪C‬‬
‫‪y' ‬‬ ‫‪‬‬ ‫‪a‬‬
‫‪dx‬‬ ‫‪2‬‬
‫‪1 C‬‬
‫ﻭﻤﻨﻪ ﻨﺤﺼل ﻋﻠﻰ ﻤﻌﺎﺩﻟﺔ ﺍﻟﻤﻨﺤﻨﻲ‪:‬‬
‫‪y  ax  b‬‬ ‫)‪(4.109‬‬
‫ﻭﻫﻲ ﻤﻌﺎﺩﻟﺔ ﻤﺴﺘﻘﻴﻡ ﻭﺒﺎﻟﺘﺎﻟﻲ ﻓﺎﻟﻤﺴﺘﻘﻴﻡ ﺍﻟﻭﺍﺼل ﺒﻴﻥ ﻨﻘﻁﺘﻴﻥ ‪ A‬ﻭ ‪ B‬ﻤﻥ‬
‫ﻤﺴﺘﻭﻱ ﻫﻭ ﺃﻗﺼﺭ ﻁﺭﻴﻕ ﺒﻴﻨﻬﻤﺎ‪ ،‬ﻭﻫﻲ ﺒﺩﻴﻬﺔ ﻤﻌﺭﻭﻓﺔ ﻓﻲ ﺍﻟﻬﻨﺩﺴﺔ ﺍﻟﻤﺴﺘﻭﻴﺔ؛ ﺃﻱ‬
‫ﺃﻥ ﺍﻟﻤﺴﺘﻘﻴﻡ ﻫﻭ ﺨﻁ ﺠﻴﻭﺩﻴﺯﻱ ﻓﻲ ﺍﻟﻤﺴﺘﻭﻱ‪ .‬ﻭﺍﻟﺠﺩﻴﺭ ﺒﺎﻟﺫﻜﺭ ﺃﻥ ﺍﻟﺜﺎﺒﺘﻴﻥ ‪ a‬ﻭ ‪b‬‬
‫ﻴﺘﻌﻴﻨﺎﻥ ﻤﻥ ﻭﺠﻭﺏ ﻤﺭﻭﺭ ﺍﻟﻤﺴﺘﻘﻴﻡ ﻓﻲ ﺍﻟﻨﻘﻁﺘﻴﻥ ﺍﻟﻤﻌﻠﻭﻤﺘﻴﻥ ) ‪ A(x1 , y1‬ﻭ‬

‫‪- 141 -‬‬

‫)‪Create PDF files without this message by purchasing novaPDF printer (http://www.novapdf.com‬‬
‫) ‪ . B(x 2 , y 2‬ﻭﺒﺎﻟﻁﺭﻴﻘﺔ ﻨﻔﺴﻬﺎ ﻨﺒﺭﻫﻥ ﺃﻥ ﺍﻟﺨﻁﻭﻁ ﺍﻟﺠﻴﻭﺩﻴﺯﻴﺔ ﻋﻠﻰ ﻜﺭﺓ ﻫﻲ‬
‫ﺍﻟﺩﻭﺍﺌﺭ ﺍﻟﻌﻅﻤﻰ ﺍﻟﻤﺭﺴﻭﻤﺔ ﻋﻠﻰ ﻫﺫﻩ ﺍﻟﻜﺭﺓ‪.‬‬
‫ﺝ ‪ -‬ﺍﻟﺴﻠﻴﺴﻠﺔ‪:‬‬
‫ﺴﻨﺩﺭﺱ ﻓﻲ ﻫﺫﺍ ﺍﻟﻨﺒﺩ ﻤﺜﺎ ﻻﹰ ﻫﺎﻤﺎﹰ ﻋﻠﻰ ﺍﻟﺨﻁﻭﻁ ﺍﻟﺠﻴﻭﺩﻴﺯﻴﺔ ﻭﺍﻟﺴﻁﻭﺡ‬
‫ﺍﻟﺠﻴﻭﺩﻴﺯﻴﺔ ﻴﻬﺘﻡ ﺒﻪ ﺼﺎﻨﻌﻭ ﺍﻟﺴﻔﻥ ﻭﺍﻟﻁﺎﺌﺭﺍﺕ ﻭﺍﻟﺼﻭﺍﺭﻴﺦ ﻭﻜل ﻤﺎﻴﺘﺤﺭﻙ ﻓﻲ‬
‫ﻭﺴﻁ ﻤﻘﺎﻭﻡ؛ ﻓﻨﺒﺤﺙ ﻋﻥ ﺃﺼﻐﺭ ﻤﺴﺎﺤﺔ ﻟﺴﻁﺢ ﺩﻭﺭﺍﻨﻲ ﻨﺎﺘﺞ ﻋﻥ ﺩﻭﺭﺍﻥ ﻤﻨﺤﻨﻲ‬
‫ﺤﻭل ﻤﺤﻭﺭ ﻤﺎ‪ .‬ﻏﻨﻲ ﻋﻥ ﺍﻟﺒﻴﺎﻥ ﺃﻥ ﻗﻭﺓ ﺍﻟﻤﻘﺎﻭﻤﺔ ﺘﺘﻨﺎﺴﺏ ﻁﺭﺩﺍﹰ ﻤﻊ ﻤﺴﺎﺤﺔ‬
‫ﺍﻟﺴﻁﺢ ﻭﺃﻥ ﺃﺼﻐﺭ ﻤﻘﺎﻭﻤﺔ ﺘﻘﺎﺒل ﺃﺼﻐﺭ ﻤﺴﺎﺤﺔ‪.‬‬
‫ﻨﺤﺴﺏ ﺃﻭﻻﹰ ﻁﻭل ﻗﻭﺱ ﺍﻟﻤﻨﺤﻨﻲ ﺒﺎﻻﺴﺘﻔﺎﺩﺓ ﻤﻥ )‪ (4.107‬ﺜﻡ ﻨﺤﺴﺏ ﻤﺴﺎﺤﺔ‬
‫ﺍﻟﺴﻁﺢ ﺍﻟﻨﺎﺘﺞ ﻋﻥ ﻋﻨﺼﺭ ﺍﻟﻘﻭﺱ ‪) d‬ﺸﻜل ‪ (4.7‬ﻓﻨﺠﺩ‪:‬‬

‫ﺍﻟﺸﻜل )‪(4.7‬‬

‫‪ds  2xd  2x 1  y '2 dx‬‬


‫ﺃﻤﺎ ﺍﻟﻤﺴﺎﺤﺔ ﺍﻟﻨﺎﺘﺠﺔ ﻋﻥ ﺩﻭﺭﺍﻥ ﺍﻟﻘﻁﻌﺔ ‪ AB‬ﻤﻥ ﺍﻟﻤ ﻨﺤﻨﻲ ﻓﺘﺴﺎﻭﻱ‪:‬‬
‫‪B‬‬
‫)‪(4.110‬‬
‫‪'2‬‬
‫‪S   2x 1  y dx‬‬
‫‪A‬‬
‫ﺃﻱ ﺃﻨﻪ ﻁﺒﻘﺎﹰ ﻟـ )‪ (4.101‬ﺘﻜﻭﻥ ﺍﻟﺩﺍﻟﺔ ‪ f‬ﻫﻲ ﺍﻟﺘﺎﻟﻴﺔ‪:‬‬
‫)‪(4.111‬‬
‫‪'2‬‬
‫‪f (y, y' )  x 1  y‬‬

‫‪- 142 -‬‬

‫)‪Create PDF files without this message by purchasing novaPDF printer (http://www.novapdf.com‬‬
‫ﺘﺸﻜل ﺍﻟﻤﻌﺎﺩﻟﺔ )‪ (4.106‬ﻭﻟﻬﺫﺍ ﻨﺸﺘﻕ ‪:f‬‬
‫‪f‬‬ ‫‪f‬‬ ‫'‪xy‬‬
‫‪0,‬‬ ‫‪‬‬
‫‪y‬‬ ‫'‪y‬‬ ‫‪'2‬‬
‫‪1 y‬‬
‫ﻭﻫﻜﺫﺍ ﻨﺤﺼل ﻋﻠﻰ ﺍﻟﻤﻌﺎﺩﻟﺔ‪:‬‬
‫'‪xy‬‬
‫‪a‬‬
‫‪'2‬‬
‫‪1 y‬‬
‫ﻭﻤﻨﻪ‪:‬‬
‫‪dy‬‬ ‫‪a‬‬
‫‪y' ‬‬ ‫‪‬‬
‫‪dx‬‬ ‫‪2‬‬ ‫‪2‬‬
‫‪x a‬‬
‫ﻭﺒﺎﻻﺴﺘﻜﻤﺎل ﻨﺠﺩ‪:‬‬
‫‪dx‬‬ ‫‪x‬‬
‫‪y  a‬‬ ‫‪ a arc ch‬‬ ‫‪b‬‬
‫‪2‬‬ ‫‪2‬‬ ‫‪a‬‬
‫‪x a‬‬
‫ﻭﻤﻨﻪ ﻤﻌﺎﺩﻟﺔ ﺍﻟﻤﻨﺤﻨﻲ ﺍﻟﻤﻁﻠﻭﺏ‪:‬‬
‫‪yb‬‬
‫‪x  a ch‬‬ ‫)‪(4.112‬‬
‫‪a‬‬
‫ﻭﻫﻲ ﻤﻌﺎﺩﻟﺔ ﻤﻨﺤﻨﻲ ﻴﺴﻤﻰ ﺍﻟﺴﻠﻴﺴﻠﺔ )‪.(catenary‬‬
‫ﻴﺘﻌﻴﻥ ﺍﻟﺜﺎﺒﺘﺎﻥ ‪ a‬ﻭ ‪ b‬ﻤﻥ ﻭﺠﻭﺏ ﺘﺤﻘﻴﻕ ﺍﻟﻤﻌﺎﺩﻟﺔ )‪ (4.112‬ﻹﺤﺩﺍﺜﻴﺎﺕ‬
‫ﺍﻟﻨﻘﻁﺘﻴﻥ ‪ A‬ﻭ ‪ B‬ﻭﻫﺫﺍ ﻴﻌﻁﻲ ﻤﻌﺎﺩﻟﺘﻴﻥ ﻟﺤﺴﺎﺒﻬﻤﺎ‪.‬‬
‫ﺍﻟﱪﺍﺷﻴﺴﺘﻮﺷﺮﻭﻥ ﺃﻭ ﺍﳌﻨﺤﻨﻲ ﺍﻷﻗﺼﺮ ﺯﻣﻨﺎ ً )ﺍﻷﻗﺰﻥ( )‪(Brachistochron‬‬
‫ﺩ‪-‬‬
‫ﻤﺎﻫﻭ ﺍﻟﻤﻨﺤﻨﻲ ﺍﻟﺫﻱ ﺇﺫﺍ ﺴﻘﻁﺕ ﻋﻠﻴﻪ ﻨﻘﻁﺔ ﻤﺎﺩﻴﺔ ﺘﺤﺕ ﺘﺄﺜﻴﺭ ﺜﻘﻠﻬﺎ ﻓﻘﻁ‬

‫)ﺤﻠﻘﺔ ﺜﻘﻴﻠﺔ ﻤﺜﻼﹰ( ﻓﺈﻨﻬﺎ ﺘﺴﺘﻐﺭﻕ ﺃﻗل ﺯﻤﻥ ﻤﻤﻜﻥ ﻟﻘﻁﻊ ﻗﻭﺱ ‪ AB‬ﻤﻨﻪ؟ ﻟﻘﺩ ﺤل‬
‫ﺒﺭﻨﻭﻟﻲ ﻫﺫﻩ ﺍﻟﻤﺴﺄﻟﺔ ﻭﺃﻁﻠﻕ ﻋﻠﻰ ﺍﻟﻤﻨﺤﻨﻲ ﺍﺴﻡ ﺒﺭﺍﺸﻴﺴﺘﻭﺸﺭﻭﻥ ﻭﻴﺒﺩﻭ ﺃﻥ ﺍﻟﺘﺴﻤﻴﺔ‬
‫ﻤﺸﺘﻘﺔ ﻤﻥ ﻜﻠﻤﺘﻴﻥ ﻓﻲ ﺍﻟﻠﻐﺔ ﺍﻟﻴﻭﻨﺎﻨﻴﺔ ﻫﻤﺎ "ﺒﺭﺍﺨﻴﺴﺘﻭﺱ" ﺍﻟﺘﻲ ﺘﻌﻨﻲ ﺍﻷﻗﺼﺭ‬
‫)ﺍﻷﻗل( ﻭ"ﺨﺭﻭﻨﻭﺱ" ﺍﻟﺘﻲ ﺘﻌﻨﻲ ﺯﻤﻥ ﻭﺒﺎﻟﺘﺎﻟﻲ ﻴﻤﻜﻥ ﺃﻥ ﻨﺴﻤﻲ ﺍﻟﻤﻨﺤﻨﻲ ﺍﻟﻤﺫﻜﻭﺭ‬
‫ﺒﺎﻟﻌﺭﺒﻴﺔ "ﺍﻟﻤﻨﺤﻨﻲ ﺍﻷﻗﺼﺭ ﺯﻤﻨﺎﹰ" ﻭﻗﺩ ﺭﺃﻴﺕ ﺃﻥ ﺃﻨﺤﺕ ﻤﻥ ﻫﺫﻩ ﺍﻟﻜﻠﻤﺎﺕ ﺍﻟﺜﻼﺙ‬
‫ﻜﻠﻤﺔ ﻭﺍﺤﺩﺓ ﻫﻲ "ﺍﻷﻗﺯﻥ" ﺍﻟﺫﻱ ﺴﻨﺒﺤﺙ ﻋﻥ ﻤﻌﺎﺩﻟﺘﻪ‪ .‬ﻭﻟﻜﻥ ﻗﺒل ﺫﻟﻙ ﺴﻨﺒﺭﻫﻥ ﺃﻥ‬

‫‪- 143 -‬‬

‫)‪Create PDF files without this message by purchasing novaPDF printer (http://www.novapdf.com‬‬
‫ﺍﻟﻤﻌﺎﺩﻟﺔ )‪ (4.106‬ﺘﻘﺒل ﺘﻜﺎﻤﻼﹰ ﺃﻭﻟﻴﺎﹰ ﻤﺒﺎﺸﺭ ﺍﹰ ﻋﻨﺩﻤﺎ ﻻﻴﺘﻌﻠﻕ ﺍﻟﺘﺎﺒﻊ ‪ f‬ﺒﺎﻟﻤﺘﺤﻭل ‪x‬‬
‫ﺒﺸﻜل ﺼﺭﻴﺢ‪ .‬ﻭﻟﻬﺫﺍ ﻨﻀﺭﺏ ﻁﺭﻓﻴﻬﺎ ﺒـ ‪ 'y‬ﻓﻨﺠﺩ‪:‬‬
‫‪d f‬‬ ‫‪f‬‬
‫'‪y‬‬ ‫‪ y'  0‬‬
‫'‪dx y‬‬ ‫‪y‬‬
‫"‪ y‬ﺇﻟﻰ ﻫﺫﻩ ﺍﻟﻤﻌﺎﺩﻟﺔ‪:‬‬ ‫ﺃﻭ ﺒﺎﻀﺎﻓﺔ ﻭﻁﺭﺡ‬
‫‪f‬‬
‫'‪y‬‬
‫‪d  f   f‬‬ ‫‪f ‬‬
‫‪‬‬ ‫"‪y'    y‬‬ ‫‪ y'   0‬‬ ‫)‪(4.113a‬‬
‫'‪dx  y'   y‬‬ ‫‪y ‬‬
‫ﻓﺈﺫﺍ ﻟﻡ ﻴﺘﺒﻊ ‪ f‬ﺍﻟﻤﺘﺤﻭل ‪ x‬ﺒﺸﻜل ﺼﺭﻴﺢ ﻓﻴﻤﻜﻥ ﺍﻟﺘﺄﻜﺩ ﺃﻥ ﺍﻟﻘﻭﺱ ﺍﻟﺜﺎﻨﻲ ﻫﻭ‬
‫ﺍﻟﻤﺸﺘﻕ ﺍﻟﻜﻠﻲ ﻟﻠﺩﺍﻟﺔ ‪ f y(x ), y' (x )‬ﺒﺎﻟﻨﺴﺒﺔ ﻟﻠﻤﺘﺤﻭل ‪ .x‬ﻭﻋﻨﺩﺌﺫ‪ ‬ﺇﺫﺍ ﺩﻤﺠﻨﺎ‬
‫ﺍﻟﻘﻭﺴﻴﻥ ﻤﻊ ﺒﻌﻀﻬﻤﺎ ﻨﺤﺼل ﻋﻠﻰ ﺍﻟﻌﻼﻗﺔ‪:‬‬
‫‪d  f‬‬ ‫‪‬‬
‫‪‬‬ ‫‪y'f   0‬‬ ‫)‪(4.113b‬‬
‫'‪dx  y‬‬ ‫‪‬‬
‫ﻭﻤﻨﻪ ﺍﻟﺘﻜﺎﻤل ﺍﻟﻤﻁﻠﻭﺏ‪:‬‬
‫‪f‬‬
‫‪y'f  Const.‬‬ ‫)‪(4.114‬‬
‫'‪y‬‬
‫ﻨﻨﺘﻘل ﺍﻵﻥ ﺇﻟﻰ ﻤﻌﺎﻟﺠﺔ ﻤﺴﺄﻟﺔ ﺍﻷﻗﺯﻥ‪ ،‬ﻓﻨﻔﺭﺽ ﻤﻨﺤﻨﻴﺎﹰ ‪ ‬ﻨﺴﻘﻁ ﻋﻠﻴﻪ ﻨﻘﻁﺔ‬
‫ﺜﻘﻴﻠﺔ ‪ M‬ﺩﻭﻥ ﺍﺤﺘﻜﺎﻙ )ﺸﻜل ‪.(4.8‬‬

‫ﺍﻟﺸﻜل )‪(4.8‬‬
‫ﺇﻥ ﺍﻟﺯﻤﻥ ﺍﻟﻼﺯﻡ ﻟﻘﻁﻊ ﻗﻭﺱ ﻋﻨﺼﺭﻱ ‪ d‬ﻫﻭ‪:‬‬

‫‪- 144 -‬‬

‫)‪Create PDF files without this message by purchasing novaPDF printer (http://www.novapdf.com‬‬
d 1  y '2
dt   dx
v 2gy
:AB ‫ﻭﺍﻟﺯﻤﻥ ﺍﻟﻼﺯﻡ ﻟﻘﻁﻊ ﻗﻭﺱ‬
d 1  y '2
t  dx
v 2gy

:‫ ﻜﻤﺎﻴﻠﻲ‬f ‫ﻭﺒﺎﻟﺘﺎﻟﻲ ﻴﻤﻜﻥ ﺃﻥ ﻨﺄﺨﺫ ﺍﻟﺩﺍﻟﺔ‬


'2
1 y
f ( y, y' ) 
y
‫( ﺍﻟﺫﻱ‬4.114) ‫ ﻭﻫﻜﺫﺍ ﻴﻤﻜﻨﻨﺎ ﺍﻻﺴﺘﻔﺎﺩﺓ ﻤﻥ ﺍﻟﺘﻜﺎﻤل‬،x ‫ﻭﻫﻲ ﻻﺘﺤﻭﻱ‬
:‫ﻴﺴﺎﻭﻱ‬
'2 '2
f y 1 y
y ' f    Const.
y' '2 y
y(1  y )
:‫ﻭﻤﻨﻪ‬
1
 a  y(1  y '2 )  C
'2
y(1  y )
:‫' ﻤﻥ ﻫﺫﻩ ﺍﻟﻌﻼﻗﺔ ﻨﺠﺩ‬y ‫ﻭﺒﺤﺴﺎﺏ‬
dy C y
y'   (4.115)
dx y
:‫ﻟﺤل ﻫﺫﻩ ﺍﻟﻤﻌﺎﺩﻟﺔ ﺍﻟﺘﻔﺎﻀﻠﻴﺔ ﻨﻐﻴﺭ ﺍﻟﻤﺘﺤﻭل ﻓﻨﻔﺭﺽ‬
 C
y  C sin 2  (1  cos )
2 2
:(4.115) ‫ ﻨﺠﺩ ﺒﺎﻟﺘﺒﺩﻴل ﻓﻲ‬‫ﻭﻋﻨﺩﺌﺫ‬
dy 
 ctg
dx 2
:‫ﻭﻤﻨﻪ‬

- 145 -

Create PDF files without this message by purchasing novaPDF printer (http://www.novapdf.com)
‫‪‬‬ ‫‪‬‬
‫‪C sin‬‬ ‫‪cos d‬‬
‫‪dy‬‬ ‫‪2‬‬ ‫‪2‬‬ ‫‪‬‬
‫‪dx ‬‬ ‫‪‬‬ ‫‪ C sin 2 d‬‬
‫‪‬‬ ‫‪‬‬ ‫‪2‬‬
‫‪ctg‬‬ ‫‪ctg‬‬
‫‪2‬‬ ‫‪2‬‬
‫ﻭﺒﺎﻻﺴﺘﻜﻤﺎل ﻨﺠﺩ‪:‬‬
‫‪C‬‬ ‫‪C‬‬
‫‪x‬‬ ‫‪‬‬ ‫‪(1  cos )d  (  sin )  C1‬‬
‫‪2‬‬ ‫‪2‬‬
‫ﻭﺒﺎﺨﺘﻴﺎﺭ ﺸﺭﻭﻁ ﺍﺒﺘﺩﺍﺌﻴﺔ ﻤﻨﺎﺴﺒﺔ ﺘﺠﻌل ‪ C1  0‬ﻨﺠﺩ ﺃﺨﻴﺭﺍﹰ ﻤﻌﺎﺩﻟﺔ ﺍﻷﻗﺯﻥ‬
‫ﺍﻟﻭﺴﻴﻁﻴﺔ ﺍﻟﺘﺎﻟﻴﺔ‪:‬‬
‫‪C‬‬ ‫‪C‬‬
‫‪x‬‬ ‫)‪(  sin ), y  (1  cos ‬‬ ‫)‪(4.116‬‬
‫‪2‬‬ ‫‪2‬‬
‫ﻭﻫﻲ ﻤﻌﺎﺩﻟﺔ ﺴﻴﻜﻠﻭﺌﻴﺩ‪ ،‬ﻭﻫﻭ‪ ،‬ﻜﻤﺎ ﻨﻌﻠﻡ‪ ،‬ﺍﻟﻤﺤل ﺍﻟﻬﻨﺩﺴﻲ ﻟﻨﻘﻁﺔ ﻤﻥ ﺩﺍﺌﺭﺓ‬
‫ﺘﺘﺩﺤﺭﺝ ﻋﻠﻰ ﺍﻟﻤﺴﺘﻭﻱ ﺍﻷﻓﻘﻲ‪.‬‬
‫ﻨﻌﻤﻡ ﺍﻟﻤﺴﺄﻟﺔ ﺍﻟﺴﺎﺒﻘﺔ؛ ﻓﻨﺒﺤﺙ ﻋﻥ ﺍﻟﻤﻨﺤﻨﻲ ﺍﻟﻔﺭﺍﻏﻲ ﺍﻟﺫﻱ ﺍﺫﺍ ﺴﻘﻁ ﻋﻠﻴﻪ‬
‫‪‬‬
‫ﺠﺴﻴﻡ ﺩﻭﻥ ﺍﺤﺘﻜﺎﻙ ﺘﺤﺕ ﺘﺄﺜﻴﺭ ﻗﻭﺓ ‪ F‬ﺘﺸﺘﻕ ﻤﻥ ﻜﻤﻭﻥ ) ‪ V (x, y, z‬ﻓﺈﻨﻬﺎ‬
‫ﺘﺴﺘﻐﺭﻕ ﺃﻗل ﺯﻤﻥ ﻤﻤﻜﻥ‪.‬‬
‫ﻤﻥ ﺍﻟﺴﻬل ﺤﺴﺎﺏ ﺍﻟﺴﺭﻋﺔ ﻓﻲ ﻫﺫﻩ ﺍﻟﺤﺎﻟﺔ ﻤﻥ ﻗﺎﻨﻭﻥ ﻤﺼﻭﻨﻴﺔ ﺍﻟﻁﺎﻗﺔ ﺤﻴﺙ‬
‫ﻨﺠﺩ‪:‬‬
‫‪2‬‬
‫‪v‬‬ ‫‪E  V(x, y, z)‬‬
‫‪m‬‬
‫ﺇﻥ ﻁﻭل ﻗﻭﺱ ﻋﻨﺼﺭﻱ ﻓﻲ ﻫﺫﻩ ﺍﻟﺤﺎﻟﺔ ﻫﻭ‪:‬‬
‫‪2‬‬ ‫‪2‬‬ ‫‪2‬‬
‫‪d  dx  dy  dz  1  y 'x2  z 'x2 dx‬‬
‫ﺃﻤﺎ ﺍﻟﺯﻤﻥ ﺍﻟﻼﺯﻡ ﻟﻘﻁﻊ ﺍﻟﻘﻭﺱ ‪ AB‬ﻓﻬﻭ‪:‬‬
‫‪d‬‬ ‫‪1  y 'x2  z 'x2‬‬
‫‪t‬‬ ‫‪‬‬ ‫‪dx‬‬ ‫)‪(4.117‬‬
‫‪v‬‬ ‫‪2‬‬
‫‪E  V(x, y, z)‬‬
‫‪m‬‬
‫ﻭﺒﺎﻟﺘﺎﻟﻲ ﻴﻤﻜﻥ ﺃﻥ ﻨﺄﺨﺫ ﺍﻟﺩﺍﻟﺔ ‪ f‬ﺒﺎﻟﺼﻴﻐﺔ ﺍﻟﺘﺎﻟﻴﺔ‪:‬‬
‫‪'2‬‬ ‫‪'2‬‬
‫‪1 yx  zx‬‬
‫‪f (y, y' , z, z ' , x ) ‬‬ ‫)‪(4.118‬‬
‫‪E  V(x , y, z)‬‬

‫‪- 146 -‬‬

‫)‪Create PDF files without this message by purchasing novaPDF printer (http://www.novapdf.com‬‬
‫ﻭﻹﻜﻤﺎل ﺤل ﻫﺫﻩ ﺍﻟﻤﺴﺄﻟﺔ ﻻﺒﺩ ﻤﻥ ﻤﻌﺭﻓﺔ ﺩﺍﻟﺔ ﺍﻟﺠﻬﺩ ‪ V‬ﺍﻟﺘﻲ ﺘﺸﺘﻕ ﻤﻨﻬﺎ‬
‫‪‬‬
‫ﺍﻟﻘﻭﺓ ‪ F‬ﺍﻟﻤﺅﺜﺭﺓ ﻋﻠﻰ ﺍﻟﺠﺴﻴﻡ‪.‬‬
‫ﻭﻫﻜﺫﺍ ﺘﺅﻭل ﺍﻟﻤﺴﺄﻟﺔ ﻟﺤﺴﺎﺏ ﺍﻟﻘﻴﻤﺔ ﺍﻟﺼﻐﺭﻯ ﻟﻠﺘﻜﺎﻤل )‪ (4.117‬ﻭﺴﻨﺤﺼل‬
‫ﻋﻨﺩﺌﺫ‪ ‬ﻋﻠﻰ ﻤﻌﺎﺩﻟﺘﻴﻥ ﻟﺤﺴﺎﺏ ﺍﻟﺩﺍﻟﺘﻴﻥ )‪ y( x‬ﻭ ) ‪ z (x‬ﺸﺒﻴﻬﺘﻴﻥ ﺒﻤﻌﺎﺩﻻﺕ ﻻﻏﺭﺍﻨﺞ‬
‫ﺃﻭ ﺍﻟﻤﻌﺎﺩﻟﺔ )‪ (4.106‬ﻭﻟﻜﻥ ﺍﻟﻤﺘﺤﻭل ﻫﻨﺎ ﻫﻭ ‪ x‬ﺒﺩﻻﹰ ﻤﻥ ‪.t‬‬

‫‪- 147 -‬‬

‫)‪Create PDF files without this message by purchasing novaPDF printer (http://www.novapdf.com‬‬
- 148 -

Create PDF files without this message by purchasing novaPDF printer (http://www.novapdf.com)
‫‪‬‬
‫‪‬‬

‫ﺣﺮﻛﺔ ﺟﺴﻴﻢ ﻣﺸﺤﻮﻥ ﰲ ﺣﻘﻞ ﻛﻬﺮﻃﻴﺴﻲ‬


‫ﻣﺪﺧﻞ ﺇﱃ ﻧﻈﺮﻳﺔ ﺍﻻﺿﻄﺮﺍﺏ ﺍﻟﺘﻘﻠﻴﺪﻳﺔ‬

‫ﺘﻌﺘﺒﺭ ﺩﺭﺍﺴﺔ ﺍﻟﺸﺤﻨﺎﺕ ﺍﻟﻜﻬﺭﺒﺎﺌﻴﺔ ﻭﺍﻟﺤﻘﻭل ﺍﻟﻜﻬﺭﻁﻴﺴﻴﺔ ﺍﻟﻨﺎﺘﺠﺔ ﻋﻨﻬﺎ ﻤﻥ‬


‫ﻤﻭﺍﻀﻴﻊ ﺍﻟﺘﺤﺭﻴﻙ ﺍﻟﻜﻬﺭﺒﺎﺌﻲ )‪ (Electrodynamics‬ﻟﻜﻥ ﺤﺭﻜﺎﺕ ﺍﻟﺠﺴﻴﻤﺎﺕ ﻀﻤﻥ‬
‫ﺍﻟﺤﻘﻭل ﺍﻟﺴﺎﺒﻘﺔ ﻫﻲ ﻤﻭﺍﻀﻴﻊ ﺘﺨﺹ ﺍﻟﻤﻴﻜﺎﻨﻴﻙ‪ ،‬ﻭﻟﺫﻟﻙ ﺨﺼﺼﻨﺎ ﺍﻟﻔﺼل ﺍﻟﺨﺎﻤﺱ‬
‫ﻟﺩﺭﺍﺴﺔ ﻫﺫﻩ ﺍﻟﺤﺭﻜﺎﺕ ﻜﺘﻁﺒﻴﻕ ﻋﻠﻰ ﻗﻭﺍﻨﻴﻥ ﺍﻟﻤﻴﻜﺎﻨﻴﻙ ﺍﻟﺘﺤﻠﻴﻠﻲ‪.‬‬
‫‪ - 38‬ﺗﺎﺑﻊ ﻻﻏﺮﺍﻧﺞ ﳉﺴﻴﻢ ﻳﺘﺤﺮﻙ ﰲ ﺣﻘﻞ ﻛﻬﺮﻃﻴﺴﻲ‪:‬‬
‫ﻤﻥ ﺍﻟﻤﻌﻠﻭﻡ ﺃﻥ ﺘﺎﺒﻊ ﻻﻏﺭﺍﻨﺞ ) ‪ L(q j , q j , t‬ﻟﻠﺠﻤل ﺍﻟﻤﺤﺎﻓﻅﺔ ﻴﺤﻘﻕ‬
‫‪‬‬

‫ﺍﻟﺨﺎﺼﺘﻴﻥ ﺍﻟﺘﺎﻟﻴﺘﻴﻥ‪:‬‬
‫‪L‬‬ ‫‪L‬‬ ‫‪V‬‬
‫‪ Pj ,‬‬ ‫‪‬‬ ‫‪ Fj‬‬ ‫)‪(5.1‬‬
‫‪‬‬ ‫‪q j‬‬ ‫‪q j‬‬
‫‪q j‬‬

‫ﺤﻴﺙ ‪ q j ، q j‬ﻫﻤﺎ ﺍﻻﺤﺩﺍﺜﻴﺎﺕ ﺍﻟﻤﻌﻤﻤﺔ ﻭﺍﻟﺴﺭﻉ ﺍﻟﻤﻌﻤﻤﺔ ﻋﻠﻰ ﺍﻟﺘﺭﺘﻴﺏ‪.‬‬


‫‪‬‬

‫ﻭﺴﻨﺒﺤﺙ‪،‬ﻋﻠﻰ ﻀﻭﺀ ﺫﻟﻙ‪،‬ﻋﻥ ﺘﺎﺒﻊ ﻻﻏﺭﺍﻨﺞ ﻟﺠﺴﻴﻡ ﺸﺤﻨﺘﻪ ‪ q‬ﻴﺘﺤﺭﻙ‬


‫‪‬‬
‫ﺒﺴﺭﻋﺔ ‪ v‬ﻓﻲ ﺤﻘل ﻜﻬﺭﻁﻴﺴﻲ ﻭﺘﺅﺜﺭ ﻋﻠﻴﻪ ﻗﻭﺓﻟﻭﺭﻨﺘﺯ ﺍﻟﺘﺎﻟﻴﺔ)ﻓﻲ ﺍﻟﺠﻤﻠﺔ ﺍﻟﺴﻐﺜﻴﺔ(‪:‬‬
‫‪‬‬ ‫‪ 1   ‬‬
‫‪F  q  E  ( v  B )‬‬ ‫)‪(5.2‬‬
‫‪‬‬ ‫‪c‬‬ ‫‪‬‬
‫‪‬‬ ‫‪‬‬
‫ﺤﻴﺙ ‪ E‬ﻫﻭ ﺍﻟﺤﻘل ﺍﻟﻜﻬﺭﺒﺎﺌﻲ ﻭ ‪ B‬ﻫﻭ ﺍﻟﺤﻘل ﺍﻟﻤﻐﻨﺎﻁﻴﺴﻲ‪ ،‬ﻭﻜﻼ ﺍﻟﺤﻘﻠﻴﻥ‬
‫ﻴﺸﺘﻘﺎﻥ ﻤﻥ ﻜﻤﻭﻨﻴﻥ ﻁﺒﻘﺎﹰ ﻟﻠﻌﻼﻗﺘﻴﻥ‪:‬‬
‫‪‬‬
‫‪‬‬ ‫‪1 A ‬‬
‫‪E  grad  ‬‬ ‫‪, B  rot A‬‬ ‫)‪(5.3‬‬
‫‪c t‬‬

‫‪- 149 -‬‬

‫)‪Create PDF files without this message by purchasing novaPDF printer (http://www.novapdf.com‬‬
‫ﻓﻬل ﻴﺤﻘﻕ ﺘﺎﺒﻊ ﻻﻏﺭﺍﻨﺞ ﺍﻟﺘﺎﻟﻲ‪:‬‬
‫‪v2‬‬ ‫‪q ‬‬
‫‪L  mc 2 1 ‬‬ ‫‪ q ‬‬ ‫) ‪(A . v‬‬ ‫)‪(5.4a‬‬
‫‪c2‬‬ ‫‪c‬‬
‫ﺍﻟﺨﺎﺼﺘﻴﻥ )‪(5.3‬؟‬
‫ﻟﻺﺠﺎﺒﺔ ﻋﻥ ﻫﺫﺍ ﺍﻟﺴﺅﺍل ﻨﺸﺘﻕ ‪ L‬ﺒﻌﺩ ﻭﻀﻌﻪ ﺒﺎﻟﺼﻴﻐﺔ‪:‬‬
‫‪x 2  y2  z 2‬‬ ‫‪q‬‬
‫‪L  mc 2 1 ‬‬ ‫‪ q(x, y, z ) ‬‬ ‫) ‪(A x x   A y y   A z z ‬‬
‫‪c2‬‬ ‫‪c‬‬

‫)‪(5.4b‬‬
‫ﻨﺸﺘﻕ ﺃﻭﻻﹰ ﺒﺎﻟﻨﺴﺒﺔ ﺇﻟﻰ ‪ x‬ﻓﻨﺠﺩ‪:‬‬
‫‪‬‬

‫‪L‬‬ ‫‪mx ‬‬ ‫‪q‬‬


‫‪‬‬ ‫‪‬‬ ‫‪Ax‬‬ ‫)‪(5.5‬‬
‫‪x ‬‬ ‫‪v‬‬ ‫‪2‬‬ ‫‪c‬‬
‫‪1‬‬
‫‪c2‬‬
‫ﻫﻭ ﻤﺘﺠﻪ‬ ‫‪ .‬ﻓﺈﺫﺍ ﺍﻋﺘﺒﺭﻨﺎ ﺃﻥ‬ ‫ﻭﺒﺎﻟﻁﺭﻴﻘﺔ ﻨﻔﺴﻬﺎ ﻨﺤﺴﺏ‬
‫‪L‬‬ ‫‪L‬‬ ‫‪L‬‬
‫‪,‬‬
‫‪‬‬ ‫‪‬‬ ‫‪‬‬
‫‪v‬‬ ‫‪z‬‬ ‫‪y‬‬

‫ﻓﺈﻨﻨﺎ ﻨﺠﺩ ﻜﺘﻌﻤﻴﻡ ﻟـ )‪:(5.5‬‬ ‫ﻤﺭﻜﺒﺎﺘﻪ‬


‫‪L‬‬ ‫‪L‬‬ ‫‪L‬‬
‫‪,‬‬ ‫‪,‬‬
‫‪‬‬ ‫‪‬‬ ‫‪‬‬
‫‪z‬‬ ‫‪y‬‬ ‫‪x‬‬
‫‪‬‬
‫‪L‬‬ ‫‪mv‬‬ ‫‪q  q ‬‬
‫‪‬‬ ‫‪‬‬ ‫)‪A  p  A  P (5.6a‬‬
‫‪‬‬ ‫‪c‬‬ ‫‪c‬‬
‫‪v‬‬ ‫‪v2‬‬
‫‪1‬‬
‫‪c2‬‬
‫ﻭﻤﻨﻪ‪:‬‬
‫‪‬‬
‫‪mv‬‬ ‫‪‬‬ ‫‪q‬‬
‫‪ P‬‬ ‫)‪A (5.6b‬‬
‫‪v2‬‬ ‫‪c‬‬
‫‪1‬‬
‫‪c2‬‬
‫‪‬‬ ‫‪‬‬ ‫‪q‬‬
‫ﺤﻴﺙ ‪ P  p  A‬ﻫﻭ ﻤﺎﻴﺴﻤﻰ ﺍﻻﻨﺩﻓﺎﻉ ﺍﻟﻤﻌﻤﻡ ﺘﻤﻴﻴﺯﺍﹰ ﻟﻪ ﻋﻥ ﺍﻻﻨﺩﻓﺎﻉ‬
‫‪c‬‬
‫‪‬‬
‫ﺍﻟﻌﺎﺩﻱ ‪ P‬ﻓﻲ ﺤﺎﻟﺔ ﻏﻴﺎﺏ ﺍﻟﺤﻘل ﺍﻟﻜﻬﺭﻁﻴﺴﻲ‪ .‬ﻭﻫﻜﺫﺍ ﺘﺘﺤﻘﻕ ﺍﻟﺨﺎﺼﺔ ﺍﻷﻭﻟﻰ ﻤﻥ‬
‫)‪.(5.1‬‬

‫‪- 150 -‬‬

‫)‪Create PDF files without this message by purchasing novaPDF printer (http://www.novapdf.com‬‬
‫ﻟﻨﺤﺴﺏ ﻤﺸﺘﻕ ‪ L‬ﺒﺎﻟﻨﺴﺒﺔ ﻟﻼﺤﺩﺍﺜﻴﺎﺕ ﺍﻟﻤﻌﻤﻤﺔ ‪ x , y, z‬ﻭﻟﻬﺫﺍ ﻨﻌﺘﺒﺭ ﺃﻥ‬
‫ﻓﻨﺠﺩ ﺒﺴﻬﻭﻟﺔ‪:‬‬ ‫‪ ‬ﻫﻭ ﻤﺘﺠﻪ ﻤﺭﻜﺒﺎﺘﻪ‬
‫‪L L L‬‬ ‫‪L‬‬
‫‪, ,‬‬
‫‪x y z‬‬
‫‪dr‬‬
‫‪L‬‬ ‫‪q‬‬ ‫‪ ‬‬
‫‪ q grad  ‬‬ ‫) ‪grad ( A . v‬‬ ‫)‪(5.7‬‬
‫‪‬‬ ‫‪c‬‬
‫‪r‬‬
‫ﻭﻁﺒﻘﺎﹰ ﻟﻘﻭﺍﻋﺩ ﺍﻟﺘﺤﻠﻴل ﺍﻟﺸﻌﺎﻋﻲ ﻴﻜﻭﻥ‪:‬‬
‫‪‬‬ ‫‪ ‬‬ ‫‪ ‬‬ ‫‪ ‬‬ ‫‪ ‬‬
‫)‪(A) v  (v ) A  A rot v  v  rot A grad ( A . v )   (A . v )  (5.8‬‬
‫ﻭﺒﻤﺎ ﺃﻥ ‪   grad‬ﺘﺅﺜﺭ ﻓﻲ ﻓﻀﺎﺀ ﺍﻻﺤﺩﺍﺜﻴﺎﺕ ﻭﻻﺘﺅﺜﺭ ﻓﻲ ﻓﻀﺎﺀ ﺍﻟﺴﺭﻉ‬
‫ﻓﻴﻤﻜﻥ ﻭﻀﻊ ﺍﻟﻌﻼﻗﺔ ﺍﻟﺴﺎﺒﻘﺔ ﻜﻤﺎﻴﻠﻲ‪:‬‬
‫‪L‬‬ ‫‪‬‬ ‫‪‬‬ ‫‪  ‬‬
‫‪ q grad   ( v . grad ) A  v  B‬‬ ‫)‪(5.9‬‬
‫‪‬‬
‫‪r‬‬
‫ﻓﺈﺫﺍ ﻋﻠﻤﻨﺎ ﺃﻥ )ﺭﺍﺠﻊ ﻤﻭﺍﻀﻴﻊ ﺍﻟﺘﺤﻠﻴل ﺍﻟﺸﻌﺎﻋﻲ(‪:‬‬
‫‪‬‬ ‫‪‬‬ ‫‪‬‬ ‫‪‬‬ ‫‪‬‬ ‫‪‬‬
‫‪d A  A  A x  A y  A z  A ‬‬ ‫‪‬‬
‫‪‬‬ ‫‪‬‬ ‫‪‬‬ ‫‪‬‬ ‫‪‬‬ ‫)‪ ( v .grad ) A (5.10‬‬
‫‪dt‬‬ ‫‪t‬‬ ‫‪dx t y t‬‬ ‫‪z t‬‬ ‫‪t‬‬
‫‪‬‬
‫ﻭﺒﺩﻟﻨﺎ ‪ ( v .grad )A‬ﺒﻘﻴﻤﺘﻬﺎ ﻓﻲ )‪ (5.9‬ﻨﺤﺼل ﻋﻠﻰ ﺍﻟﻤﺸﺘﻕ ﺍﻟﻤﻁﻠﻭﺏ‪:‬‬
‫‪  ‬‬
‫‪L‬‬ ‫‪q  dA A  q  ‬‬
‫‪‬‬
‫‪ q grad   ‬‬ ‫‪‬‬ ‫‪  ( v B) ‬‬ ‫)‪(5.11‬‬
‫‪c  dt‬‬ ‫‪t  c‬‬
‫‪r‬‬ ‫‪‬‬ ‫‪‬‬
‫‪‬‬ ‫‪‬‬
‫‪ 1    q d A  q d A‬‬
‫‪ q  E  ( v  B ) ‬‬ ‫‪ F‬‬
‫‪‬‬ ‫‪c‬‬ ‫‪ c dt‬‬ ‫‪c dt‬‬
‫ﺇﻥ ﻭﺠﻭﺩ ﺍﻟﺤﺩ ﺍﻷﺨﻴﺭ ﻻﻴﺴﺒﺏ ﺃﻱ ﻤﺸﻜﻠﺔ ﻷﻥ ﺘﺎﺒﻊ ﻻﻏﺭﺍﻨﺞ ﻴﻌﻁﻰ‬
‫ﺒﺎﻟﺘﻘﺭﻴﺏ ﺇﻟﻰ ﺍﻟﻤﺸﺘﻕ ﺍﻟﻜﻠﻲ ﻷﻱ ﺘﺎﺒﻊ ﻟﻠﺯﻤﻥ )ﺭﺍﺠﻊ ﺍﻟﻔﺼل ﺍﻟﺭﺍﺒﻊ(‪.‬‬
‫ﻭﻫﻜﺫﺍ ﻨﺭﻯ ﺃﻥ ﻤﺸﺘﻕ ﺘﺎﺒﻊ ﻻﻏﺭﺍﻨﺞ ﺍﻟﻤﻌﺭﻑ ﺒﺎﻟﻌﻼﻗﺔ ) ‪ (5.4a‬ﻴﺤﻘﻕ‬
‫ﺍﻟﺨﺎﺼﺘﻴﻥ )‪ (5.1‬ﻭﺒﺎﻟﺘﺎﻟﻲ ﻴﻤﻜﻥ ﻋﺩﻩ ﺍﻟﺘﺎﺒﻊ ﺍﻟﻤﻁﻠﻭﺏ‪.‬‬
‫ﻜﻴﻑ ﻴﺼﺒﺢ ﻫﺫﺍ ﺍﻟﺘﺎﺒﻊ ﻓﻲ ﺍﻟﺘﻘﺭﻴﺏ ﺍﻟﻜﻼﺴﻴﻜﻲ ﻋﻨﺩﻤﺎ ‪ v  c‬؟ ﻟﻼﺠﺎﺒﺔ ﻋﻥ‬
‫ﻫﺫﺍ ﺍﻟﺴﺅﺍل ﻨﻨﺸﺭ ‪ L‬ﺒﻘﻭﻯ ‪  ‬ﻓﻨﺠﺩ‪:‬‬
‫‪v‬‬
‫‪c‬‬

‫‪- 151 -‬‬

‫)‪Create PDF files without this message by purchasing novaPDF printer (http://www.novapdf.com‬‬
1
2 2
 v q     mv 2 q  
L  mc 2 1     A . v   q   q   A . v  (5.12)
 c 2  c 2 c
  
.‫ ﺍﻟﺫﻱ ﻻﻴﺅﺜﺭ ﻋﻠﻰ ﺼﻴﻐﺔ ﻤﻌﺎﺩﻻﺕ ﻻﻏﺭﺍﻨﺞ‬ mC ‫ﻭﺫﻟﻙ ﺒﺎﻫﻤﺎل ﺍﻟﺜﺎﺒﺕ‬
2

:‫ ﺗﺎﺑﻊ ﻫﺎﻣﻠﺘﻮﻥ ﳉﺴﻴﻢ ﻳﺘﺤﺮﻙ ﰲ ﺣﻘﻞ ﻛﻬﺮﻃﻴﺴﻲ‬- 39


:‫ﻤﻥ ﺍﻟﻤﻌﻠﻭﻡ )ﺭﺍﺠﻊ ﺍﻟﻔﺼل ﺍﻟﺜﺎﻟﺙ( ﺃﻥ ﺩﺍﻟﺔ ﻫﺎﻤﻠﺘﻭﻥ ﺘﻌﻁﻰ ﺒﺎﻟﻌﻼﻗﺔ‬
 
H   P jq j  L(q j , q j , t )

:‫ ﻭﺒﺎﻟﺘﺎﻟﻲ ﻴﻜﻭﻥ‬v ‫ﻭﻓﻲ ﺤﺎﻟﺘﻨﺎ ﻫﺫﻩ ﻟﺩﻴﻨﺎ ﺠﺴﻴﻡ ﻭﺍﺤﺩ ﺴﺭﻋﺘﻪ‬
L  mv q  v2 q 
H vL  (  A ). v  mc 2 1   q  ( A . v )

v2 c c2 c
v 1
c2
mc 2
(5.13a)   q
2
v
1
c2
‫ ﺇﻟﻰ ﺸﻜﻠﻪ ﺍﻟﻨﻅﺎﻤﻲ ﻜﻲ ﻴﻤﻜﻥ ﺍﻻﺴﺘﻔﺎﺩﺓ ﻤﻨﻪ ﻓﻲ ﺤل‬H ‫ﻭﻻﺒﺩ ﻤﻥ ﺭﺩ‬
:‫( ﻓﻨﺠﺩ‬5.13b ) ‫( ﻭ‬5.6b ) ‫ﺍﻟﻤﺴﺎﺌل ﻭﻟﻬﺫﺍ ﻨﺭﺒﻊ ﺍﻟﻌﻼﻗﺘﻴﻥ‬
m 2c4 m2 v2 q 2
(H  q) 2  ,  (P  A) (5.14)
v2 v2 c
1 1
c2 c2
:‫ﻨﻀﻊ ﺍﻟﺜﺎﻨﻴﺔ ﺒﺎﻟﺸﻜل‬
m2 v2 m2 v2  m2c2  m 2v 2
 m 2 c 2  m 2 c2   m 2c 2 (5.15)
2 2
v v
1 1
c2 c2
:‫( ﻨﺠﺩ‬5.15) ‫( ﻭ‬5.14) ‫ﻭﻤﻥ‬
2
m2c2  q 
2 2
 m c  P  A
2  c 
v
1
c2

- 152 -

Create PDF files without this message by purchasing novaPDF printer (http://www.novapdf.com)
‫ﺃﻱ ﺃﻨﻪ‪ ،‬ﺒﻌﺩ ﺍﻻﺴﺘﻔﺎﺩﺓ ﻤﻥ ﺍﻟﻌﻼﻗﺔ ﺍﻷﻭﻟﻰ ﻤﻥ )‪ (5.14‬ﻴﻤﻜﻥ ﻭﻀﻊ ﺍﻟﻌﻼﻗﺔ‬
‫ﺍﻷﺨﻴﺭﺓ ﻜﻤﺎﻴﻠﻲ‪:‬‬
‫‪2‬‬
‫‪(H  q) 2‬‬ ‫‪‬‬ ‫‪q ‬‬
‫‪ m2c2   P  A ‬‬
‫‪c2‬‬ ‫‪‬‬ ‫‪c ‬‬
‫ﻭﻤﻨﻪ ﻨﺤﺼل ﻋﻠﻰ ﺘﺎﺒﻊ ﻫﺎﻤﻠﺘﻭﻥ ﺒﺸﻜﻠﻪ ﺍﻟﻨﻅﺎﻤﻲ ﺍﻟﺘﺎﻟﻲ‪:‬‬
‫‪2‬‬
‫‪‬‬ ‫‪q ‬‬
‫‪(5.13b) H  m c   P  A  c 2  q‬‬
‫‪2 4‬‬
‫‪‬‬ ‫‪c ‬‬
‫ﻓﻲ ﺍﻟﺘﻘﺭﻴﺏ ﺍﻟﻜﻼﺴﻴﻜﻲ ﺤﻴﺙ ﺘﻜﻭﻥ ﺴﺭﻋﺔ ﺍﻟﺠﺴﻴﻡ ﺃﻗل ﺒﻜﺜﻴﺭ ﻤﻥ ﺴﺭﻋﺔ‬
‫ﺍﻟﻀﻭﺀ ﻴﻤﻜﻥ ﻨﺸﺭ ﺍﻟﺠﺫﺭ ﺤﻴﺙ ﻨﺠﺩ‪:‬‬
‫‪2‬‬
‫‪‬‬ ‫‪q ‬‬
‫‪ P  A‬‬
‫‪c ‬‬ ‫‪‬‬ ‫‪1 ‬‬ ‫‪q ‬‬
‫‪H  mc 2 1  ‬‬ ‫‪ q  mc 2 1 ‬‬ ‫‪ p  A   q‬‬
‫‪2 2‬‬ ‫‪2 2‬‬ ‫‪c ‬‬
‫‪m c‬‬ ‫‪ 2m c‬‬
‫‪2‬‬
‫‪‬‬ ‫‪q ‬‬
‫‪P  A‬‬
‫‪c ‬‬
‫‪ mc 2  ‬‬ ‫)‪ q (5.16‬‬
‫‪2m‬‬
‫ﻭﻴﻤﻜﻥ ﺇﻫﻤﺎل ﺍﻟﺤﺩ ﺍﻷﻭل ﺍﻟﺜﺎﺒﺕ ﻋﻨﺩﻤﺎ ﺘﺸﻜل ﻤﻌﺎﺩﻻﺕ ﻫﺎﻤﻠﺘﻭﻥ‪.‬‬
‫‪ - 40‬ﻣﻌﺎﺩﻻﺕ ﺍﳊﺮﻛﺔ ﰲ ﺍﻟﻨﻈﺮﻳﺔ ﺍﻟﻨﺴﺒﻴﺔ‪:‬‬
‫ﺭﺃﻴﻨﺎ ﺃﻨﻪ ﻤﻥ ﺍﻟﻤﻔﻴﺩ‪ ،‬ﻗﺒل ﺩﺭﺍﺴﺔ ﺤﺭﻜﺔ ﺠﺴﻴﻡ ﻓﻲ ﺤﻘل ﻜﻬﺭﻁﻴﺴﻲ‪ ،‬ﺃﻥ‬
‫ﻨﺫﻜﺭ ﺒﻤﻌﺎﺩﻻﺕ ﺍﻟﺤﺭﻜﺔ ﺍﻟﺘﻲ ﺘﻭﻀﻊ ﻓﻲ ﺍﻟﻔﻀﺎﺀ ﺍﻟﺭﺒﺎﻋﻲ ﻜﻤﺎﻴﻠﻲ‪:‬‬
‫‪dP‬‬ ‫‪d‬‬
‫‪‬‬ ‫)‪mu   F (  1, 2, 3, 4‬‬ ‫)‪(5.17‬‬
‫‪dt‬‬ ‫‪dt 0‬‬
‫ﻤﻊ ﺍﻟﻌﻠﻡ ﺃﻥ ‪ m‬ﻫﻲ ﻜﺘﻠﺔ ﺍﻟﺴﻜﻭﻥ ﻟﻠﺠﺴﻴﻡ ﻭ ‪ dt 0‬ﻫﻲ ﻋﻨﺼﺭ ﺍﻟﺯﻤﻥ‬
‫ﺍﻟﺨﺎﺹ‪ ،‬ﺃﻤﺎ ‪ u ‬ﻓﻬﻲ ﺍﻟﺴﺭﻋﺔ ﻓﻲ ﺍﻟﻔﻀﺎﺀ ﺍﻟﺭﺒﺎﻋﻲ )‪ ( x, y, z, ‬ﻭﻤﺭﻜﺒﺎﺘﻬﺎ ﻫﻲ‪:‬‬
‫‪vx‬‬ ‫‪vy‬‬ ‫‪vz‬‬ ‫‪ic‬‬
‫‪ux ‬‬ ‫‪, uy ‬‬ ‫‪, uz ‬‬ ‫‪, u ‬‬ ‫)‪(5.18‬‬
‫‪2‬‬ ‫‪2‬‬ ‫‪2‬‬ ‫‪2‬‬
‫‪v‬‬ ‫‪v‬‬ ‫‪v‬‬ ‫‪v‬‬
‫‪1‬‬ ‫‪1‬‬ ‫‪1‬‬ ‫‪1‬‬
‫‪2‬‬ ‫‪2‬‬ ‫‪2‬‬
‫‪c‬‬ ‫‪c‬‬ ‫‪c‬‬ ‫‪c2‬‬
‫‪‬‬
‫ﻭ ‪ v‬ﻫﻲ ﺴﺭﻋﺔ ﺍﻟﺠﺴﻴﻡ ﻓﻲ ﺍﻟﻔﻀﺎﺀ ﺍﻟﺜﻼﺜﻲ‪.‬‬

‫‪- 153 -‬‬

‫)‪Create PDF files without this message by purchasing novaPDF printer (http://www.novapdf.com‬‬
‫ﻭﻋﻨﺩﺌﺫ‪ ‬ﺘﻭﻀﻊ ﻤﻌﺎﺩﻻﺕ ﺍﻟﺤﺭﻜﺔ )‪ (5.17‬ﻜﻤﺎﻴﻠﻲ‪) :‬ﻋﻠﻰ ﺍﻟﻤﺤﻭﺭ ‪.(ox‬‬
‫‪dPx‬‬ ‫‪1‬‬ ‫‪d‬‬ ‫‪mv x‬‬
‫)‪(5.19a‬‬ ‫‪‬‬ ‫‪ Fx‬‬
‫‪dt 0‬‬ ‫‪v 2 dt‬‬ ‫‪v2‬‬
‫‪1‬‬ ‫‪1‬‬
‫‪c2‬‬ ‫‪c2‬‬
‫ﺤﻴﺙ ‪ Fx‬ﻫﻲ ﻤﺭﻜﺒﺔ ﺍﻟﻘﻭﺓ ﻋﻠﻰ ‪ ox‬ﻓﻲ ﺍﻟﻔﻀﺎﺀ ﺍﻟﺭﺒﺎﻋﻲ‪.‬‬
‫ﻟﻜﻥ ﻨﻨﺘﻘل ﺇﻟﻰ ﺍﻟﺘﻘﺭﻴﺏ ﺍﻟﻜﻼﺴﻴﻜﻲ ﻨﻀﻊ )‪ (a5.19‬ﺒﺎﻟﺼﻴﻐﺔ ﺍﻟﺘﺎﻟﻴﺔ‪:‬‬
‫‪d‬‬ ‫‪mv x‬‬ ‫‪v2‬‬
‫‪ Fx 1 ‬‬ ‫)‪(5.19b‬‬
‫‪dt‬‬ ‫‪v‬‬ ‫‪2‬‬ ‫‪c2‬‬
‫‪1‬‬
‫‪c2‬‬
‫ﻓﺈﺫﺍ ﻓﺭﻀﻨﺎ‪:‬‬
‫‪v2‬‬
‫‪Fx  Fx 1 ‬‬
‫‪c2‬‬
‫ﻴﻤﻜﻥ ﺃﻥ ﺘﺘﺤﻭل ‪ Fx‬ﺇﻟﻰ ‪ Fx‬ﻋﻨﺩﻤﺎ ‪ v  c‬ﻭﻋﻨﺩﺌﺫ‪ ‬ﻨﻀﻊ )‪.(b5.19‬‬
‫‪d‬‬ ‫‪mv x‬‬
‫)‪ Fx (5.19c‬‬
‫‪dt‬‬ ‫‪v‬‬ ‫‪2‬‬
‫‪1‬‬
‫‪c2‬‬
‫ﻭﺘﺘﺤﻭل ﻋﻨﺩﺌﺫ‪ (5.19c ) ‬ﺇﻟﻰ ﻤﺴﻘﻁ ﻤﻌﺎﺩﻟﺔ ﺍﻟﺤﺭﻜﺔ ﻋﻠﻰ ﺍﻟﻤﺤﻭﺭ ‪ ox‬ﻓﻲ ﺍﻟﻤﻴﻜﺎﻨﻴﻙ‬
‫ﺍﻟﺘﻘﻠﻴﺩﻱ‪.‬‬
‫ﻭﻜﺫﻟﻙ ﻨﺠﺩ ﺒﺎﻟﺘﻌﻤﻴﻡ ﻋﻠﻰ ﺍﻟﻤﺤﺎﻭﺭ ﺍﻟﺒﺎﻗﻴﺔ‪.‬‬
‫‪d‬‬ ‫‪mv y‬‬ ‫‪d‬‬ ‫‪mvz‬‬ ‫‪d‬‬ ‫‪icm‬‬ ‫‪v2‬‬
‫‪ Fy ,‬‬ ‫‪ Fz ,‬‬ ‫‪ F 1 ‬‬ ‫)‪(5.20‬‬
‫‪dt‬‬ ‫‪v‬‬ ‫‪2‬‬ ‫‪dt‬‬ ‫‪v‬‬ ‫‪2‬‬ ‫‪dt‬‬ ‫‪v‬‬ ‫‪2‬‬ ‫‪c2‬‬
‫‪1‬‬ ‫‪1‬‬ ‫‪1‬‬
‫‪2‬‬ ‫‪2‬‬
‫‪c‬‬ ‫‪c‬‬ ‫‪c2‬‬
‫ﻭﻻﻴﻭﺠﺩ ﻓﻲ ﺍﻟﻔﻀﺎﺀ ﺍﻟﺜﻼﺜﻲ ﻤﺎﻴﻘﺎﺒل ﺍﻟﻤﺭﻜﺒﺔ ﺍﻟﺯﻤﻨﻴﺔ ﻷﻨﻬﺎ ﻋﻘﺩﻴﺔ‪ ،‬ﻟﻜﻨﻬﺎ‬
‫ﺘﺭﺘﺒﻁ ﺒﺎﻻﺴﺘﻁﺎﻋﺔ ﻭﻟﺒﺭﻫﺎﻥ ﺫﻟﻙ ﻨﻀﺭﺏ ﻤﻌﺎﺩﻻﺕ ﺍﻟﺤﺭﻜﺔ )‪ ( 5.18‬ﺒـ ‪u ‬‬
‫ﻭﻨﺠﻤﻊ ﺒـ ‪ ‬ﻓﻨﺠﺩ‪:‬‬
‫‪dmu‬‬
‫‪ u‬‬ ‫‪dt 0‬‬
‫)‪  F .u   Fx u x  Fy u y  Fz u z  F u   0 (5.21a‬‬
‫‪‬‬ ‫‪‬‬

‫‪- 154 -‬‬

‫)‪Create PDF files without this message by purchasing novaPDF printer (http://www.novapdf.com‬‬
‫ﻷﻥ ﺍﻟﺴﺭﻋﺔ ﻭﺍﻟﺘﺴﺎﺭﻉ )ﻭﺒﺎﻟﺘﺎﻟﻲ ﺍﻟﺴﺭﻋﺔ ﻭﺍﻟﻘﻭﺓ( ﻤﺘﻌﺎﻤﺩﺍﻥ ﻓﻲ ﺍﻟﻔﺭﺍﻍ‬
‫ﺍﻟﺭﺒﺎﻋﻲ‪ ،‬ﻜﻤﺎ ﻨﻌﻠﻡ‪ .‬ﻭﻋﻨﺩﺌﺫ‪ ‬ﻨﺠﺩ ﻤﻥ ﺍﻟﻤﻌﺎﺩﻟﺔ ﺍﻟﺴﺎﺒﻘﺔ ﺃﻥ‪:‬‬
‫‪Fx u x‬‬ ‫‪Fy v y‬‬ ‫‪Fz v z‬‬ ‫‪ic‬‬
‫‪‬‬ ‫‪‬‬ ‫‪ F‬‬ ‫)‪ 0 (5.21b‬‬
‫‪v2‬‬ ‫‪v2‬‬ ‫‪v2‬‬ ‫‪v‬‬ ‫‪2‬‬
‫‪1‬‬ ‫‪1‬‬ ‫‪1‬‬ ‫‪1‬‬
‫‪2‬‬ ‫‪2‬‬ ‫‪2‬‬
‫‪c‬‬ ‫‪c‬‬ ‫‪c‬‬ ‫‪c2‬‬
‫‪ ‬‬
‫ﺍﻥ ﺍﻟﺤﺩﻭﺩ ﺍﻟﺜﻼﺜﺔ ﺍﻷﻭﻟﻰ ﻫﻲ ﺍﻟﺠﺩﺍﺀ ﺍﻟﻌﺩﺩﻱ ‪ F . v‬ﻭﺒﺎﻟﺘﺎﻟﻲ ﻴﻜﻭﻥ‪:‬‬
‫‪ ‬‬
‫) ‪i (F . v‬‬
‫‪F ‬‬ ‫)‪(5.22‬‬
‫‪c‬‬ ‫‪v2‬‬
‫‪1‬‬
‫‪c2‬‬
‫)‪(5.20‬‬ ‫ﻓﺈﺫﺍ ﺒﺩﻟﻨﺎ ‪ F‬ﺒﻘﻴﻤﺘﻬﺎ ﻤﻥ )‪ (5.22‬ﻓﻲ ﺍﻟﻤﻌﺎﺩﻟﺔ ﺍﻷﺨﻴﺭﺓ ﻤﻥ‬
‫ﻨﺤﺼل ﻋﻠﻰ ﺍﻟﻌﻼﻗﺔ‪:‬‬
‫‪d‬‬ ‫‪mc 2‬‬ ‫‪    dA‬‬
‫‪  F . v  ‬‬ ‫)‪(5.23‬‬
‫‪dt‬‬ ‫‪v2 ‬‬ ‫‪ dt‬‬
‫‪1‬‬
‫‪c2‬‬
‫ﻫﻭ ﺍﻟﻌﻤل ﺍﻟﻤﻨﺠﺯ ﻓﻲ ﻭﺍﺤﺩﺓ ﺍﻟﺯﻤﻥ )ﺍﻻﺴﺘﻁﺎﻋﺔ(‪.‬‬ ‫ﺤﻴﺙ‬
‫‪dA‬‬
‫‪dt‬‬
‫ﻭﻤﻥ ﺍﻟﻤﻌﻠﻭﻡ ﻓﻲ ﺍﻟﻤﻴﻜﺎﻨﻴﻙ ﺍﻟﺘﻘﻠﻴﺩﻱ ﺃﻥ ﻤﺸﺘﻕ ﺍﻟﻁﺎﻗﺔ ﺍﻟﺤﺭﻜﻴﺔ‪ ،‬ﺍﻟﺫﻱ‬
‫ﺴﻨﺭﻤﺯ ﻟﻪ ‪ ،E‬ﻴﺴﺎﻭﻱ ﺍﻻﺴﺘﻁﺎﻋﺔ ﻭﺒﺎﻟﺘﺎﻟﻲ ﻤﻥ ﺍﻟﻁﺒﻴﻌﻲ ﺃﻥ ﻨﻔﺭﺽ‪:‬‬
‫‪mc 2‬‬
‫‪E  E kin ‬‬ ‫‪ m( v) c 2‬‬ ‫)‪(5.24‬‬
‫‪2‬‬
‫‪v‬‬
‫‪1‬‬
‫‪c2‬‬
‫ﻭﻫﻲ ﻤﻌﺎﺩﻟﺔ ﺇﻴﻨﺸﺘﺎﻴﻥ ﺍﻟﻤﻌﺭﻭﻓﺔ ﻟﺤﺴﺎﺏ ﻁﺎﻗﺔ ﺠﺴﻴﻡ ﻜﺘﻠﺘﻪ ﺍﻟﺴﻜﻭﻨﻴﺔ ‪m‬‬
‫‪‬‬
‫ﻭﻴﺘﺤﺭﻙ ﺒﺴﺭﻋﺔ ‪. v‬‬
‫ﻤﻥ ﺍﻟﻤﻔﻴﺩ ﻓﻲ ﻨﻬﺎﻴﺔ ﻫﺫﻩ ﺍﻟﻔﻘﺭﺓ ﺃﻥ ﻨﺴﺘﻨﺘﺞ ﺍﻟﻌﻼﻗﺔ )‪ (5.24‬ﺒﻴﻥ ﺍﻟﻁﺎﻗﺔ )‪(E‬‬
‫‪‬‬
‫ﻭﻜﻤﻴﺔ ﺍﻟﺤﺭﻜﺔ ‪ P‬ﻭﻟﻬﺫﺍ ﻨﺠﺩ ﻤﻥ ﺍﻟﻤﻌﺎﺩﻻﺕ ﺍﻟﺜﻼﺙ ﺍﻷﻭﻟﻰ ﻤﻥ )‪ (5.18‬ﺒﻌﺩ‬

‫‪- 155 -‬‬

‫)‪Create PDF files without this message by purchasing novaPDF printer (http://www.novapdf.com‬‬
‫‪‬‬
‫ﻀﺭﺏ ﺍﻟﻁﺭﻓﻴﻥ ﺒﻜﺘﻠﺔ ﺍﻟﺴﻜﻭﻥ ‪ m‬ﻨﺤﺼل ﻋﻠﻰ ﻜﻤﻴﺔ ﺍﻟﺤﺭﻜﺔ ‪ P‬ﻓﻲ ﺍﻟﻔﺭﺍﻍ‬
‫ﺍﻟﺜﻼﺜﻲ‪:‬‬
‫‪‬‬
‫‪‬‬ ‫‪mv‬‬
‫‪P ‬‬ ‫)‪(5.25‬‬
‫‪2‬‬
‫‪v‬‬
‫‪1‬‬
‫‪c2‬‬
‫‪v2‬‬
‫ﻤﻥ )‪ (5.24‬ﻭ )‪ (5.25‬ﻨﺠﺩ ﺃﺨﻴﺭﺍﹰ ﺍﻟﻌﻼﻗﺔ ﺍﻟﻬﺎﻤﺔ‬ ‫‪1‬‬ ‫ﻭﺒﺤﺫﻑ‬
‫‪c2‬‬
‫ﺍﻟﺘﺎﻟﻴﺔ‪:‬‬
‫‪‬‬ ‫‪E ‬‬
‫‪P‬‬ ‫)‪v (5.26‬‬
‫‪c2‬‬
‫‪ - 41‬ﺣﺮﻛﺔ ﺟﺴﻴﻢ ﻣﺸﺤﻮﻥ ﰲ ﺣﻘﻞ ﻛﻬﺮﺑﺎﺋﻲ‪:‬‬
‫ﻤﻥ ﺍﻟﺴﻬل ﺍﻵﻥ‪ ،‬ﻋﻠﻰ ﻀﻭﺀ ﺍﻟﻔﻘﺭﺓ ﺍﻟﺴﺎﺒﻘﺔ‪ ،‬ﺃﻥ ﻨﻜﺘﺏ ﻤﻌﺎﺩﻟﺔ ﺤﺭﻜﺔ ﺍﻟﺠﺴﻴﻡ‬
‫‪‬‬
‫ﺒﻔﺭﺽ ﺃﻥ ﺍﻟﺤﻘل ‪ ‬ﻴﺘﺠﻪ ﺒﺎﺘﺠﺎﻩ ﺍﻟﻤﺤﻭﺭ ‪ ox‬ﺤﻴﺙ ﻨﺠﺩ ﻁﺒﻘﺎﹰ ﻟـ )‪ (5.20‬ﻭ‬
‫‪v2‬‬
‫(‪:‬‬ ‫)‪) (5.23‬ﺒﺎﻫﻤﺎل‬
‫‪c2‬‬
‫‪dPx dmv x‬‬ ‫‪dPy‬‬ ‫‪dP‬‬ ‫‪d‬‬ ‫‪mc 2‬‬ ‫‪dE  ‬‬
‫‪‬‬ ‫‪ Fx ,‬‬ ‫‪ Fy , z  Fz ,‬‬ ‫‪‬‬ ‫)‪ F . v (5.27‬‬
‫‪dt‬‬ ‫‪dt‬‬ ‫‪dt‬‬ ‫‪dt‬‬ ‫‪dt‬‬ ‫‪v2‬‬ ‫‪dt‬‬
‫‪1‬‬
‫‪c2‬‬
‫ﻤﻊ ﺍﻟﻌﻠﻡ ﺃﻥ ‪ F  q ‬ﻭﺒﺎﻟﺘﺎﻟﻲ ﻴﻜﻭﻥ‪ Fx  q x , Fy  Fz  0 :‬ﻭﻤﻥ‬
‫‪‬‬ ‫‪‬‬

‫ﺍﻷﺨﻴﺭﺓ ﻨﺠﺩ‪:‬‬
‫‪dE  ‬‬ ‫‪dx‬‬
‫‪ F . v  Fx v x  Fy v y  Fz v z  q x‬‬ ‫)‪(5.28‬‬
‫‪dt‬‬ ‫‪dt‬‬
‫ﻻﻴﺠﺎﺩ ﻗﺎﻨﻭﻥ ﺍﻟﺤﺭﻜﺔ ﻭﻤﻌﺎﺩﻟﺔ ﺍﻟﻤﺴﺎﺭ ﻨﺴﺘﻜﻤل ﺍﻟﻤﻌﺎﺩﻻﺕ ﺍﻟﺴﺎﺒﻘﺔ ﻁﺒﻘﺎﹰ‬
‫ﻟﻠﺸﺭﻭﻁ ﺍﻻﺒﺘﺩﺍﺌﻴﺔ ﺍﻟﺘﺎﻟﻴﺔ‪:‬‬
‫)‪t  0, x 0  y 0  z 0  0, Px  Pz  0, Py  P0  0 (5.29‬‬

‫‪- 156 -‬‬

‫)‪Create PDF files without this message by purchasing novaPDF printer (http://www.novapdf.com‬‬
‫‪‬‬ ‫‪‬‬
‫ﺃﻱ ﺃﻥ ﺍﻟﺤﻘل ‪ ‬ﻴﺘﻌﺎﻤﺩ ﻤﻊ ﺍﺘﺠﺎﻩ ﺍﻻﻨﺩﻓﺎﻉ ﺍﻹﺒﺘﺩﺍﺌﻲ ‪ P0‬ﻓﻲ ﺒﺩﺍﻴﺔ ﺍﻟﺤﺭﻜﺔ‪.‬‬
‫ﻓﺈﺫﺍ ﺤﺴﺒﻨﺎ ‪ E 0‬ﻀﻤﻥ ﻫﺫﻩ ﺍﻟﺸﺭﻭﻁ ﻨﺠﺩ‪:‬‬
‫‪E 0  P02 c 2  m 2 c 4‬‬ ‫)‪(5.30‬‬

‫ﻭﻟﻠﺤﺼﻭل ﻋﻠﻰ ﺍﻻﻨﺩﻓﺎﻉ ‪ P‬ﻨﺴﺘﻜﻤل ﺍﻟﻤﻌﺎﺩﻻﺕ )‪ (5.27‬ﻓﻨﺠﺩ‪:‬‬


‫‪‬‬

‫‪Px  q x t , Py  P0 , Pz  0‬‬ ‫)‪(5.31‬‬


‫ﻭﺒﺎﻟﺘﺎﻟﻲ ﺘﺤﺩﺙ ﺍﻟﺤﺭﻜﺔ ﻓﻲ ﺍﻟﻤﺴﺘﻭﻱ ‪.xoy‬‬
‫ﺃﻤﺎ ﺍﻟﻁﺎﻗﺔ ﻓﺘﺤﺴﺏ ﻤﻥ )‪ (5.28‬ﺒﺎﺴﺘﻜﻤﺎﻟﻬﺎ ﺒﺎﻟﺯﻤﻥ‪:‬‬
‫‪E  q x x  E 0 ‬‬ ‫‪Px2  Py2 c 2  mc4 ‬‬
‫)‪ c 2q 2  2x  P02 c 2  m 2 c 4  q 2c 2  2x t 2  E 20 (5.32‬‬
‫ﻭﻤﻨﻪ ﻨﺤﺼل ﻋﻠﻰ ﻗﺎﻨﻭﻥ ﺍﻟﺤﺭﻜﺔ ﺍﻟﺘﺎﻟﻲ‪:‬‬
‫‪1  2 2 2 2‬‬
‫‪x‬‬ ‫‪‬‬ ‫‪q c  x t  E 20  E 0 ‬‬ ‫)‪(5.33‬‬
‫‪q x ‬‬ ‫‪‬‬
‫ﻭﻴﺴﻬل ﺍﻟﺤﺼﻭل ﻋﻠﻰ ﻤﻌﺎﺩﻟﺔ ﺍﻟﻤﺴﺎﺭ ﺇﺫﺍ ﺤﺫﻓﻨﺎ ﺍﻟﺯﻤﻥ ﻤﻥ )‪ (5.31‬ﻭ‬
‫)‪ (5.33‬ﻭﻟﻬﺫﺍ ﻨﻜﺘﺏ ﺃﻭﻻﹰ‪:‬‬
‫‪Px‬‬ ‫‪mv x‬‬ ‫‪mv y‬‬ ‫‪v‬‬ ‫‪dx q x t‬‬
‫‪‬‬ ‫‪‬‬ ‫‪ x ‬‬ ‫‪‬‬ ‫)‪(5.34‬‬
‫‪Py‬‬ ‫‪v2‬‬ ‫‪v2‬‬ ‫‪v y dy‬‬ ‫‪P0‬‬
‫‪1‬‬ ‫‪1‬‬
‫‪c2‬‬ ‫‪c2‬‬
‫ﻨﺒﺩل ‪ t‬ﺒﻘﻴﻤﺘﻬﺎ ﻤﻥ )‪ (5.34‬ﻓﻲ )‪ (5.33‬ﻓﻨﺠﺩ ﺒﻌﺩ ﺘﺭﺒﻴﻌﻬﺎ‪:‬‬
‫‪2‬‬
‫‪2‬‬ ‫‪‬‬ ‫‪dx ‬‬
‫‪q x x  E 0 ‬‬ ‫‪  cP0‬‬ ‫‪  E 02‬‬
‫‪‬‬ ‫‪dy ‬‬
‫ﻭﻤﻨﻪ‪:‬‬
‫‪y‬‬ ‫‪dx‬‬ ‫‪1‬‬ ‫‪E‬‬
‫‪‬‬ ‫‪‬‬ ‫‪arc.ch‬‬
‫‪cP0 ‬‬ ‫‪q x x  E 0 2  E 02‬‬ ‫‪q x‬‬ ‫‪E0‬‬

‫ﺃﻭ ﺒﺸﻜﻠﻬﺎ ﺍﻟﻨﻬﺎﺌﻲ‪:‬‬


‫‪E0‬‬ ‫‪ q x y  q x x‬‬ ‫‪q‬‬
‫‪x‬‬ ‫‪ ch‬‬ ‫‪ 1‬‬ ‫‪ 1  ch x y ‬‬ ‫)‪(5.35‬‬
‫‪q x‬‬ ‫‪‬‬ ‫‪cP0‬‬ ‫‪ E‬‬ ‫‪cP0‬‬

‫‪- 157 -‬‬

‫)‪Create PDF files without this message by purchasing novaPDF printer (http://www.novapdf.com‬‬
‫‪q x‬‬
‫ﺼﻐﻴﺭ ﺍﹰ )ﺘﻘﺭﻴﺏ ﻜﻼﺴﻴﻜﻲ ﻷﻥ ‪c‬‬ ‫ﻭﻓﻲ ﺍﻟﺤﺎﻟﺔ ﺍﻟﺨﺎﺼﺔ ﻋﻨﺩﻤﺎ ‪y‬‬
‫‪cP0‬‬
‫ﻤﻭﺠﻭﺩﺓ ﻓﻲ ﻤﻘﺎﻡ ﺍﻟﻜﺴﺭ( ﻴﻤﻜﻥ ﻨﺸﺭ ﺠﻴﺏ ﺍﻟﺘﻤﺎﻡ ﺍﻟﻘﻁﻌﻲ ﻓﻨﺠﺩ ﺒﻌﺩ ﺍﺴﺘﺒﺩﺍل ‪E 0‬‬
‫ﺒـ ‪ mc2‬ﻭ ‪ P0‬ﺒـ ‪ mv02‬ﺃﻥ‪:‬‬
‫‪E0‬‬ ‫‪ q 2 2 xy 2‬‬ ‫‪ q ‬‬
‫‪x‬‬ ‫‪1 ‬‬
‫‪‬‬
‫‪x‬‬ ‫‪ 1 ‬‬
‫‪‬‬
‫‪x y2‬‬ ‫)‪(5.36‬‬
‫‪q x‬‬ ‫‪2 2‬‬ ‫‪2‬‬
‫‪‬‬ ‫‪c py‬‬ ‫‪ mv 0‬‬
‫ﻭﻫﻲ ﻤﻌﺎﺩﻟﺔ ﻗﻁﻊ ﻤﻜﺎﻓﻰﺀ‪ ،‬ﻭﻫﺫﻩ ﻨﺘﻴﺠﺔ ﻁﺒ ﻴﻌﻴﺔ ﻭﻤﻨﻁﻘﻴﺔ ﻷﻥ ﻤﺴﺎﺭ ﺃﻱ‬
‫ﺠﺴﻴﻡ ﻓﻲ ﺤﻘل ﻤﺘﺠﺎﻨﺱ ﻭﻤﺴﺘﻘﺭ )ﺤﻘل ﺍﻟﺜﻘﺎﻟﺔ ﻤﺜﻼﹰ( ﻫﻭ ﺒﺎﻟﻔﻌل ﻗﻁﻊ ﻤﻜﺎﻓﻰﺀ‪.‬‬
‫‪‬‬
‫ﻜﻴﻑ ﺘﺘﻐﻴﺭ ﺍﻟﻨﺘﺎﺌﺞ ﺍﻟﺴﺎﺒﻘﺔ ﻋﻨﺩﻤﺎ ﻴﻜﻭﻥ ﺍﻟﺤﻘل ‪ ‬ﻤﺴﺘﻘﺭ ﺍﹰ )ﻻﻴﺘﺒﻊ ﺍﻟﺯﻤﻥ(‬
‫‪‬‬ ‫‪‬‬
‫ﻭﻟﻜﻨﻪ ﻴﺘﺒﻊ ﺍﻹﺤﺩﺍﺜﻴﺎﺕ؟ ﻭﻟﻨﻔﺭﺽ ﺃﻥ ) ‪    (x‬ﻭﺃﻨﻪ ﻴﺸﺘﻕ ﻤﻥ ﻜﻤﻭﻥ )‪. ( x‬‬
‫ﻭﻋﻨﺩﺌﺫ‪ ‬ﻨﺠﺩ ﻤﻥ ﺍﻟﻤﻌﺎﺩﻟﺔ ﺍﻷﺨﻴﺭﺓ ﻤﻥ )‪:(5.27‬‬
‫‪d dx‬‬ ‫‪d dE  ‬‬ ‫‪dx‬‬ ‫‪dx‬‬
‫‪ q‬‬ ‫‪.‬‬ ‫‪ q‬‬ ‫‪ F . v  q x‬‬ ‫‪ q grad ‬‬ ‫)‪(5.37‬‬
‫‪dx dt‬‬ ‫‪dt dt‬‬ ‫‪dt‬‬ ‫‪dt‬‬
‫ﻭﺒﺎﻻﺴﺘﻜﻤﺎل ﻨﺠﺩ‪:‬‬
‫)‪E  q  Const.  E 0  q 0 (5.38‬‬
‫‪‬‬ ‫‪‬‬
‫‪‬‬ ‫‪‬‬
‫‪‬‬ ‫‪1‬‬ ‫‪‬‬
‫‪E  E  E 0  mc 2 ‬‬ ‫)‪ 1  q(   0 )  qV (5.39‬‬
‫‪‬‬ ‫‪v2‬‬ ‫‪‬‬
‫‪ 1  2‬‬ ‫‪‬‬
‫‪‬‬ ‫‪c‬‬ ‫‪‬‬
‫ﺤﻴﺙ ‪ V‬ﻓﺭﻕ ﺍﻟﻜﻤﻭﻥ ﺒﻴﻥ ﻭﻀﻌﻴﻥ ﺘﻐﻴﺭﺕ ﺒﻴﻨﻬﻤﺎ ﺍﻟﻁﺎﻗﺔ ﻤﻥ ‪ E 0‬ﺇﻟﻰ ‪.E‬‬
‫ﻟﻨﺤﺴﺏ ﺍﻟﺴﺭﻋﺔ ﺍﻟﺘﻲ ﻴﻜﺘﺴﺒﻬﺎ ﺍﻟﺠﺴﻴﻡ ﻤﻥ ﺍﻟﻌﻼﻗﺔ ﺍﻟﺴﺎﺒﻘﺔ )‪ (5.39‬ﻓﻨﺠﺩ‬
‫ﺃﺨﻴﺭﺍﹰ‪:‬‬
‫‪‬‬ ‫‪q 2V2‬‬ ‫‪‬‬
‫‪mc 4  2qV ‬‬ ‫‪‬‬
‫‪q 2 V 2  2qmc 2 V‬‬ ‫‪‬‬ ‫‪mc 2‬‬ ‫‪‬‬
‫‪vc‬‬ ‫‪‬‬ ‫‪‬‬ ‫‪‬‬
‫‪(qV  mc 2 ) 2‬‬ ‫‪2 4‬‬ ‫‪qV ‬‬
‫‪2‬‬
‫‪m c 1 ‬‬ ‫‪‬‬
‫‪ mc 2 ‬‬

‫‪- 158 -‬‬

‫)‪Create PDF files without this message by purchasing novaPDF printer (http://www.novapdf.com‬‬
‫‪qV‬‬
‫‪1‬‬
‫‪‬‬
‫‪2qV‬‬ ‫‪2mc 2‬‬ ‫)‪(5.40‬‬
‫‪m ‬‬ ‫‪2‬‬
‫‪qV ‬‬
‫‪1 ‬‬ ‫‪‬‬
‫‪ mc 2 ‬‬
‫ﻭﻟﻜﻲ ﻨﻭﻀﺢ ﺍﻟﻤﻌﻨﻰ ﺍﻟﻔﻴﺯﻴﺎﺌﻲ ﻟﻠﻌﻼﻗﺔ ﺍﻟﺴﺎﺒﻘﺔ‪ ،‬ﺍﻟﺘﻲ ﺘﻌﺘﺒﺭ ﺃﺴﺎﺱ‬
‫ﺍﻟﻤﺴﺭﻋﺎﺕ ﺍﻟﺨﻁﻴﺔ ﻨﻤﻴﺯ ﺤﺎﻟﺘﻴﻥ‪:‬‬
‫‪ -1‬ﺍﻟﻁﺎﻗﺔ ﺍﻟﻤﻜﺘﺴﺒﺔ ‪ E  qV‬ﺃﺼﻐﺭ ﺒﻜﺜﻴﺭ ﻤﻥ ‪ mc2‬ﻭﻋﻨﺩﺌﺫ‪ ‬ﻨﺠﺩ ﻤﻥ‬
‫)‪ (5.40‬ﺃﻥ‪:‬‬
‫‪1‬‬
‫)‪(5.41‬‬
‫‪2‬‬
‫‪mv  qV‬‬
‫‪2‬‬
‫ﻭﻫﻲ ﺍﻟﻌﻼﻗﺔ ﺍﻟﻜﻼﺴﻴﻜﻴﺔ ﺍﻟﻤﻌﺭﻭﻓﺔ‪.‬‬
‫ﺏ‪ -‬ﺍﻟﻁﺎﻗﺔ ﺍﻟﻤﻜﺘﺴﺒﺔ ﺃﻜﺒﺭ ﺒﻜﺜﻴﺭ ﻤﻥ ‪ (qV  mc2 )mc2‬ﻭﻋﻨﺩﺌﺫ‪ ‬ﻨﺠﺩ ﻤﻥ‬
‫)‪:(5.40‬‬
‫‪qV‬‬
‫‪2qV 2mc 2‬‬ ‫‪2qV mc 2‬‬
‫‪v‬‬ ‫‪‬‬ ‫‪c‬‬ ‫)‪(5.42‬‬
‫‪m q2 V2‬‬ ‫‪m 2qV‬‬
‫‪m 2 c4‬‬
‫ﺃﻱ ﺃﻥ ﺴﺭﻋﺔ ﺍﻟﺠﺴﻴﻡ ﺫﻱ ﺍﻟﺸﺤﻨﺔ ‪ q‬ﺍﻟﻤﺘﺤﺭﻙ ﻓﻲ ﺤﻘل ﻜﻬﺭﺒﺎﺌﻲ ) ‪ ( x‬‬
‫ﺘﺯﺩﺍﺩ ﺯﻴﺎﺩﺓ ﻜﺒﻴﺭﺓ‪.‬‬
‫‪‬‬
‫‪ - 42‬ﺣﺮﻛﺔ ﺟﺴﻴﻢ ﰲ ﺣﻘﻞ ﻣﻐﻨﺎﻃﻴﺴﻲ ‪: B‬‬
‫‪‬‬
‫ﻨﻔﺭﺽ ﺃﻥ ﺍﻟﺤﻘل ‪ B‬ﻤ ﺘﺠﺎﻨﺱ ﻭﻴﺘﺠﻪ ﺒﺎﺘﺠﺎﻩ ‪ oz‬ﻭﻨﻜﺘﺏ ﻤﻌﺎﺩﻻﺕ ﺍﻟﺤﺭﻜﺔ‬
‫ﺍﻋﺘﻤﺎﺩﺍﹰ ﻋﻠﻰ )‪ (5.2‬ﻭﻋﻠﻰ )‪ (5.20‬ﻓﻨﺠﺩ‪:‬‬
‫‪d‬‬ ‫‪mv x‬‬ ‫‪dPx q‬‬ ‫‪q‬‬
‫‪‬‬ ‫)‪ (v  B) x  v y B (5.43‬‬
‫‪dt‬‬ ‫‪v2‬‬ ‫‪dt‬‬ ‫‪c‬‬ ‫‪c‬‬
‫‪1‬‬
‫‪c2‬‬
‫‪dPy‬‬ ‫‪q‬‬ ‫‪dPz‬‬ ‫‪dE q     ‬‬
‫‪  v x B,‬‬ ‫‪ 0,‬‬ ‫‪‬‬ ‫)‪v  B . v  0 (5.44‬‬
‫‪dt‬‬ ‫‪c‬‬ ‫‪dt‬‬ ‫‪dt c ‬‬ ‫‪‬‬

‫‪- 159 -‬‬

‫)‪Create PDF files without this message by purchasing novaPDF printer (http://www.novapdf.com‬‬
‫ﻨﻼﺤﻅ ﻤﻥ ﺍﻟﻤﻌﺎﺩﻟﺔ ﺍﻷﺨﻴﺭﺓ ﺃﻥ ﺍﻟﻁﺎﻗﺔ ‪ E‬ﺜﺎﺒﺘﺔ ) ‪ (E  E 0‬ﻭﺃﻥ‬
‫‪ . v z  (v z ) 0‬ﻭﺇﺫﺍ ﺍﻨﻌﺩﻤﺕ ﻤﺭﻜﺒﺔ ﺍﻟﺴﺭﻋﺔ ﻋﻠﻰ ‪ oz‬ﻓﻲ ﺒﺩﺍﻴﺔ ﺍﻟﺤﺭﻜﺔ ﺘﻜﻭﻥ‬
‫ﺍﻟﺤﺭﻜﺔ ﻤﺴﺘﻭﻴﺔ ﻭﺘﺤﺩﺙ ﻓﻲ ﺍﻟﻤﺴﺘﻭﻱ ‪.xoy‬‬
‫ﺒﺎﻻﺴﺘﻔﺎﺩﺓ ﻤﻥ ﺍﻟﻌﻼﻗﺔ )‪ (5.26‬ﺒﻴﻥ ﺍﻟﻁﺎﻗﺔ ﻭﺍﻟﺴﺭﻋﺔ ﻴﻤﻜﻥ ﺃﻥ ﻨﻜﺘﺏ‪:‬‬
‫‪dPx‬‬ ‫‪d E0‬‬ ‫‪q‬‬
‫‪‬‬ ‫)‪v x  v y B (5.45‬‬
‫‪dt‬‬ ‫‪dt c 2‬‬ ‫‪c‬‬
‫ﻭﻤﻨﻪ‪:‬‬
‫‪1 dv x qcB‬‬
‫‪‬‬ ‫)‪(5.46‬‬
‫‪v y dt‬‬ ‫‪E0‬‬

‫ﻭﻜﺫﻟﻙ ﻨﺠﺩ ﺒﺎﻟﻁﺭﻴﻘﺔ ﻨﻔﺴﻬﺎ‪:‬‬


‫‪1 dv y‬‬ ‫‪qcB‬‬
‫‪‬‬ ‫)‪(5.47‬‬
‫‪v x dt‬‬ ‫‪E0‬‬
‫ﻭﺒﺤل ﺍﻟﻤﻌﺎﺩﻟﺘﻴﻥ ﺍﻟﺴﺎﺒﻘﺘﻴﻥ ﺤﻼﹰ ﻤﺸﺘﺭﻜﺎﹰ ﻨﺤﺼل ﻋﻠﻰ ﺍﻟﻤﻌﺎﺩﻟﺘﻴﻥ‪:‬‬
‫‪d2vx‬‬ ‫‪q 2c 2B2‬‬ ‫‪d 2v y‬‬ ‫‪q 2c 2B2‬‬
‫‪‬‬ ‫‪vx  0 ,‬‬ ‫‪‬‬ ‫)‪v y  0 (5.48‬‬
‫‪dt 2‬‬ ‫‪E 20‬‬ ‫‪dt 2‬‬ ‫‪E 20‬‬
‫ﻭﻤﻨﻪ ﻨﺤﺼل ﻋﻠﻰ ﻗﺎﻨﻭﻥ ﺍﻟﺴﺭﻋﺔ ﺍﻟﺘﺎﻟﻲ‪:‬‬
‫)‪v x  a cos(t  ), v y  a sin(t  ) (5.49‬‬

‫‪  ‬ﻫﻭ ﺘﻭﺍﺘﺭ ﺍﻟﺤﺭﻜﺔ‪.‬‬ ‫ﺤﻴﺙ‬


‫‪qcB‬‬
‫‪E0‬‬
‫ﻭﻤﻨﻪ )‪ (5.49‬ﻨﺤﺴﺏ ﺍﻟﺴﺭﻋﺔ ﻓﻨﺠﺩ‪:‬‬
‫)‪v  v x  v y  a  v xy  Const. (5.50‬‬
‫‪2‬‬ ‫‪2‬‬ ‫‪2‬‬ ‫‪2‬‬ ‫‪2‬‬

‫ﻭﻹﻴﺠﺎﺩ ﻗﺎﻨﻭﻥ ﺍﻟﺤﺭﻜﺔ ﻨﺴﺘﻜﻤل ﺍﻟﻤﻌﺎﺩﻟﺘﻴﻥ )‪ (5.49‬ﻓﻨﺠﺩ ﺃﺨﻴﺭ ﺍﹰ ﺒﻌﺩ ﺤﺴﺎﺏ‬


‫ﺍﻟﺜﻭﺍﺒﺕ‪:‬‬
‫‪(v xy ) 0‬‬ ‫‪(v xy ) 0‬‬
‫‪x  x0 ‬‬ ‫‪sin(t  ), y  y 0 ‬‬ ‫)‪cos(t  ‬‬ ‫)‪(5.51‬‬
‫‪‬‬ ‫‪‬‬
‫ﻭﻴﻜﻭﻥ ﺍﻟﻤﺴﺎﺭ ﺩﺍﺌﺭﺓ ﻴﺘﺤﺭﻙ ﺍﻟﺠﺴﻴﻡ ﻋﻠﻴﻬﺎ ﺒﺴﺭﻋﺔ ﺯﺍﻭﻴﺔ ‪ ‬ﻤﺭﻜﺯﻫﺎ ﻴﻘﻊ‬
‫ﻓﻲ ﺍﻟﻨﻘﻁﺔ ) ‪ M 0 (x 0 , y 0‬ﻭﻤﻌﺎﺩﻟﺘﻬﺎ‪:‬‬

‫‪- 160 -‬‬

‫)‪Create PDF files without this message by purchasing novaPDF printer (http://www.novapdf.com‬‬
‫‪2‬‬
‫‪ v xy‬‬ ‫‪‬‬
‫)‪(5.52‬‬
‫‪2‬‬ ‫‪2‬‬
‫‪(x  x 0 )  (y  y0 )  ‬‬ ‫‪‬‬
‫‪ ‬‬ ‫‪‬‬
‫‪‬‬ ‫‪‬‬
‫ﻭﻓﻲ ﺍﻟﺤﺎﻟﺔ ﺍﻟﺨﺎﺼﺔ ﻋﻨﺩﻤﺎ ﺘﻜﻭﻥ ‪ E‬ﺼﻐﻴﺭﺓ ‪ E  mc2‬ﻴﺼﺒﺢ ﺍﻟﺘﻭﺍﺘﺭ‪:‬‬
‫‪qcB qB‬‬
‫‪‬‬ ‫‪‬‬ ‫)‪(5.53‬‬
‫‪E 0 mC‬‬
‫ﺘﻌﺘﺒﺭ ﺍﻟﻌﻼﻗﺎﺕ ﺍﻟﺴﺎﺒﻘﺔ ﺃﺴﺎﺴﻴﺔ ﻓﻲ ﺍﻟﺤﺴﺎﺒﺎﺕ ﺍﻟﻤﺘﻌﻠﻘﺔ ﺒﻤﺴﺭﻋﺎﺕ ﺍﻟﺠﺴﻴﻤﺎﺕ‬
‫ﺍﻟﺩﺍﺌﺭﻴﺔ )ﺍﻟﺴﻴﻜﻠﻭﺘﺭﻭﻥ( ﻭﻴﻼﺤﻅ ﺃﻥ ﺍﻟﺘﻭﺍﺘﺭ ﻴﺒﻘﻰ ﺜﺎﺒﺘﺎﹰ ﻭﻻﻴﺘﻌﻠﻕ ﺒﻁﺎﻗﺔ ﺍﻟﺠﺴﻴﻡ‬
‫ﻭﻻﺒﺴﺭﻋﺘﻪ‪ ،‬ﻓﺎﻟﺠﺴﻴﻡ ﻴﺴﺘﻐﺭﻕ ﺍﻟﺯﻤﻥ ﻨﻔﺴﻪ ﻓﻲ ﻜل ﺩﻭﺭﺓ ﻤﻥ ﺩﻭﺭﺍﻨﻪ ﺤﺘﻰ ﻋﻨﺩﻤﺎ‬
‫ﺘﺯﺩﺍﺩ ﺴﺭﻋﺘﻪ‪.‬‬
‫ﻭﻤﻥ ﺍﻟﻭﺍﻀﺢ ﺃﻥ ﺍﻟﺴﺭﻋﺔ ﺘﺒﻘﻰ ﺜﺎﺒﺘﺔ ﻁﺒﻘﺎﹰ ﻟـ )‪ ،(5.50‬ﻭﻟﻜﻥ ﻴﻤﻜﻥ ﺘﺴﺭﻴﻊ‬
‫ﺍﻟﺠﺴﻴﻡ ﺒﺘﻤﺭﻴﺭﻩ ﻀﻤﻥ ﺘﺠﻭﻴﻔﻴﻥ ﻤﺘﻤﺎﺜﻠﻴﻥ )ﺸﻜل ‪ D1 (5.1‬ﻭ ‪ D 2‬ﻤﻔﺭﻏﻴﻥ ﻤﻥ‬
‫ﺍﻟﻬﻭﺍﺀ ﻜل ﻤﻨﻬﻤﺎ ﻴﺸﻜل ﻨﺼﻑ ﻗﺭﺹ ﻴﻭﺠﺩ ﻀﻤﻨﻬﻤﺎ ﺤﻘل ﻜﻬﺭﺒﺎﺌﻲ ﻴﺘﻐﻴﺭ‬
‫ﺒﺎﻟﺘﻨﺎﻭﺏ ﺒﺤﻴﺙ ﺘﺯﺩﺍﺩ ﺴﺭﻋﺔ ﺍﻟﺠﺴﻴﻡ ﻋﻨﺩ ﺩﺨﻭﻟﻪ ﻓﻲ ﻜل ﻤﻥ ﺍﻟﺘﺠﻭﻴﻔﻴﻥ‪ .‬ﻭﺘﻜﺭﺭ‬
‫ﻫﺫﻩ ﺍﻟﻌﻤﻠﻴﺔ ﺤﺘﻰ ﻟﺤﻅﺔ ﺨﺭﻭﺝ ﺍﻟﺠﺴﻴﻡ ﻤﻥ ﺍﻟﻤﺴﺭﻉ ﻭﻟﻜﻥ ﻋﻨﺩﻤﺎ ﺘﺯﺩﺍﺩ ﺍﻟﺴﺭﻋﺔ‬
‫‪  ‬ﺍﻟﺘﻲ‬ ‫ﺯﻴﺎﺩﺓ ﻜﺒﻴﺭﺓ ﻟﻥ ﻴﻜﻭﻥ ﺍﻟﺘﻭﺍﺘﺭ ‪ ‬ﺜﺎﺒﺘﺄً ﻭﻻﺒﺩ ﻤﻥ ﺘﻁﺒﻴﻕ ﺍﻟﻌﻼﻗﺔ‬
‫‪qcB‬‬
‫‪E0‬‬
‫ﻻﺘﻬﻤل ﺍﻟﺘﺄﺜﻴﺭﺍﺕ ﺍﻟﻨﺴﺒﻭﻴﺔ ﻋﻠﻰ ﺍﻟﻜﺘﻠﺔ ﺍﻟﻤﻭﺠﻭﺩﺓ ﻓﻲ ﺍﻟﻤﻘﺎﻡ‪ .‬ﻭﻴﺴﻤﻰ ﺍﻟﻤﺴﺭﻉ ﺍﻟﺫﻱ‬
‫ﻴﻌﻤل ﻋﻠﻰ ﻫﺫﺍ ﺍﻷﺴﺎﺱ ﺍﻟﻤﺴﺭﻉ ﺍﻟﻨﺴﺒﻭﻱ ﺃﻭ ﺍﻟﺴﻴﻨﻜﺭﻭﺘﺭﻭﻥ )‪(Synchrotron‬‬
‫ﻭﻻﻨﻭﺩ ﺍﻟﺘﻔﺼﻴل ﻓﻲ ﺫﻟﻙ ﻷﻨﻨﺎ ﺴﻨﺨﺭﺝ ﻤﻥ ﻤﻭﺍﻀﻴﻊ ﺍﻟﻤﻴﻜﺎﻨﻴﻙ ﺍﻟﺘﺤﻠﻴﻠﻲ ﻭﻨﺩﺨل ﻓﻲ‬
‫ﻤﻭﺍﻀﻴﻊ ﺍﻟﻔﻴﺯﻴﺎﺀ ﺍﻟﻨﻭﻭﻴﺔ‪.‬‬

‫‪- 161 -‬‬

‫)‪Create PDF files without this message by purchasing novaPDF printer (http://www.novapdf.com‬‬
‫ﺍﻟﺸﻜل )‪(5.1‬‬
‫‪ - 43‬ﻣﺪﺧﻞ ﺇﱃ ﻧﻈﺮﻳﺔ ﺍﻻﺿﻄﺮﺍﺏ ﺍﻟﻜﻼﺳﻴﻜﻴﺔ‪:‬‬
‫ﺘﺸﻜل ﺍﻷﺠﺴﺎﻡ ﺍﻟﻔﻀﺎﺌﻴﺔ ﺍﻵﺘﻴﺔ ﻤﻥ ﺍﻟﻜﻭﻥ ﺨﻁﺭﺍﹰ ﻜﺒﻴﺭﺍﹰ ﻋﻠﻰ ﺍﻷﺭﺽ ﻭﻋﻠﻰ‬
‫ﺍﻷﻗﻤﺎﺭ ﺍﻟﺼﻨﺎﻋﻴﺔ ﺍﻟﺘﻲ ﺘﺩﻭﺭ ﺤﻭﻟﻬﺎ‪ .‬ﻭﻴﻜﺜﺭ ﺍﻟﺤﺩﻴﺙ ﻋﻥ ﺫﻟﻙ ﻋﻨﺩﻤﺎ ﻴﻘﺘﺭﺏ ﻤﺫﻨﺏ‬
‫ﻤﺎ ﻤﻥ "ﻟﺅﻟﺅﺓ" ﺍﻟﻤﺠﻤﻭﻋﺔ ﺍﻟﺸﻤﺴﻴﺔ‪ ،‬ﻭﻫﻨﺎﻙ ﻨﻅﺭﻴﺔ ﻤﻔﺎﺩﻫﺎ ﺃﻥ ﺴﺒﺏ ﺍﻨﻘﺭﺍﺽ‬
‫ﺍﻟﺩﻴﻨﺎﺼﻭﺭﺍﺕ ﻫﻭ ﺍﺭﺘﻁﺎﻡ ﻨﻴﺯﻙ ﻫﺎﺌل ﺒﺎﻷﺭﺽ ﻤﻨﺫ ‪ 65‬ﻤﻠﻴﻭﻥ ﺴﻨﺔ‪ .‬ﻓﻬل ﻴﻤﻜﻥ‬
‫ﺘﺠﻨﺏ ﻜﺎﺭﺜﺔ ﻤﻥ ﻫﺫﺍ ﺍﻟﻨﻭﻉ ﺇﺫﺍ ﺃﻁﻠﻕ ﺼﺎﺭﻭﺥ ﻤﻥ ﺍﻷﺭﺽ ﻴﻤﻜﻥ ﺃﻥ ﻴﺤﺭﻑ‬
‫ﺍﻟﻨﻴﺯﻙ ﻋﻥ ﻤﺴﺎﺭﻩ ﻭﻴﺯﻴل ﺨﻁﺭ ﺍﺼﻁﺩﺍﻤﻪ ﺒﺎﻷﺭﺽ؟‬
‫ﺴﻨﻌﺎﻟﺞ ﻓﻲ ﻫﺫﻩ ﺍﻟﻔﻘﺭﺓ ﺍﻷﺨﻴﺭﺓ ﻤﻥ ﺍﻟﻔﺼل ﺍﻟﺨﺎﻤﺱ ﺍﻟﺘﻐﻴﺭ ﺍﻟﺤﺎﺼل ﻋﻠﻰ‬
‫‪‬‬
‫ﻤﺴﺎﺭ ﺠﺴﻴﻡ ﻴﺘﺤﺭﻙ ﺘﺤﺕ ﺘﺄﺜﻴﺭ ﻗﻭﺓ ﻤﻌﻠﻭﻤﺔ ‪ ، F‬ﻋﻨﺩﻤﺎ ﻴﺘﻌﺭﺽ ﺇﻟﻰ ﺍﻀﻁﺭﺍﺏ‬
‫‪‬‬
‫ﻨﺎﺘﺞ ﻋﻥ ﻗﻭﺓ )ﺘﺄﺜﻴﺭﻫﺎ ﺃﻗل ﺒﻜﺜﻴﺭ ﻤﻥ ﺘﺄﺜﻴﺭ ﺍﻟﻘﻭﺓ ‪ ( F‬ﻤﻥ ﻭﺠﻬﺔ ﻨﻅﺭ ﺍﻟﻤﻴﻜﺎﻨﻴﻙ‬
‫ﺍﻟﺘﻘﻠﻴﺩﻱ‪ ،‬ﻭﻫﺫﺍ ﻤﺎﻴﺴﻤﻰ ﻨﻅﺭﻴﺔ ﺍﻻﻀﻁﺭﺍﺏ ﺍﻟﻜﻼﺴﻴﻜﻴﺔ ﺘﻤﻴﻴﺯﺍﹰ ﻟﻬﺎ ﻋﻥ ﻨﻅﺭﻴﺔ‬
‫ﺍﻻﻀﻁﺭﺍﺏ ﺍﻟﻜﻤﻭﻤﻴﺔ ﺍﻟﺘﻲ ﺘﺩﺭﺱ ﺍﻟﻅﺎﻫﺭﺓ ﻨﻔﺴﻬﺎ ﻓﻲ ﻤﻴﻜﺎﻨﻴﻙ ﺍﻟﻜﻡ ﺤﻴﺙ ﺘﺴﻭﺩ‬
‫ﻗﻭﺍﻨﻴﻥ ﻤﺨﺘﻠﻔﺔ ﻓﻲ ﺍﻟﻌﺎﻟﻡ ﺍﻟﻤﺠﻬﺭﻱ‪.‬‬
‫ﻭﺴﻨﻘﺩﻡ ﺒﻌﺽ ﺍﻟﻤﻌﻠﻭﻤﺎﺕ ﺍﻟﻤﻔﻴﺩﺓ ﻓﻲ ﻫﺫﺍ ﺍﻟﻤﻭﻀﻭﻉ ﺒﻤﺎ ﻴﺴﻤﺢ ﺒﻪ ﺤﺠﻡ‬
‫ﺍﻟﻜﺘﺎﺏ ﺩﻭﻥ ﺍﻟﺘﻌﻤﻕ ﻓﻲ ﺫﻟﻙ‪.‬‬
‫ﻟﺘﻜﻥ ﺍﻟﺩﺍﻟﺔ‪:‬‬
‫‪f ( , ,  ,....)  0‬‬ ‫)‪(5.54‬‬

‫‪- 162 -‬‬

‫)‪Create PDF files without this message by purchasing novaPDF printer (http://www.novapdf.com‬‬
‫ﻭﻟﻨﻔﺭﺽ ﺃﻨﻪ ﺤﺩﺙ ﺘﻐﻴﺭ ﻁﻔﻴﻑ )ﺍﻀﻁﺭﺍﺏ( ﻋﻠﻰ ﻤﺘﺤﻭﻻﺕ ﻫﺫﻩ ﺍﻟﺩﺍﻟﺔ‬
‫ﺒﺤﻴﺙ ﺃﺼﺒﺤﺕ )‪ . f (  ,     ......‬ﻟﻨﻨﺸﺭ ﺍﻟﺩﺍﻟﺔ ﺍﻟﺠﺩﻴﺩﺓ ﺒﺠﻭﺍﺭ ﺍﻟﻨﻘﻁﺔ‬
‫)‪ ( 0 ,  0 ,  0 ,...‬ﺤﻴﺙ ﺒﺩﺃ ﺍﻻﻀﻁﺭﺍﺏ ﻓﻨﺠﺩ‪:‬‬
‫‪ f ‬‬ ‫‪ f ‬‬
‫‪f (  ,   ,...)  f 0          ...‬‬
‫‪   0‬‬ ‫‪   0‬‬
‫‪  2f ‬‬ ‫‪  2f ‬‬
‫)‪   ... (5.55‬‬
‫‪2‬‬
‫‪  2    ‬‬
‫‪   0‬‬ ‫‪   0‬‬
‫ﻭﻟﻨﻔﺭﺽ ﺃﻥ ﺍﻟﺩﺍﻟﺔ )‪ (5.54‬ﺘﻤﺜل ﻤﺴﺎﺭ ﻤﺘﺤﺭﻙ ﻓﻲ ﻭﻀﻊ ﻤﺴﺘﻘﺭ )ﺃﻭ ﺃﻱ‬
‫ﻤﻥ ﻋﻨﺎﺼﺭ ﻫﺫﺍ ﺍﻟﻤﺴﺎﺭ( ﻭﻟﻨﺩﺭﺱ ﻜﻴﻑ ﻴﺘﻐﻴﺭ ﺍﻟﻤﺴﺎﺭ ﻨﺘﻴﺠﺔ ﺘﺄﺜﻴﺭ ﻗﻭﻯ ﺨﺎﺭﺠﻴﺔ‬
‫‪‬‬
‫ﺘﺄﺜﻴﺭﻫﺎ ﺃﻗل ﺒﻜﺜﻴﺭ ﻤﻥ ﺘﺄﺜﻴﺭ ﺍﻟﻘﻭﺓ ‪ F‬ﺍﻟﺘﻲ ﺘﺴﺒﺏ ﺍﻟﻤﺴﺎﺭ ﺍﻟﻤﺫﻜﻭﺭ‪ ،‬ﺒﺎﺩﺌﻴﻥ ﺒﺎﻟﻤﺜﺎل‬
‫ﺍﻟﺘﺎﻟﻲ‪:‬‬
‫ﻴﻌﻁﻰ ﻨﺼﻑ ﻁﻭل ﺍﻟﻤﺤﻭﺭ ﺍﻟﻜﺒﻴﺭ ﻟﻤﺴﺎﺭ ﻗﻤﺭ ﺼﻨﺎﻋﻲ ﻴﺩﻭﺭ ﺤﻭل ﺍﻷﺭﺽ‬
‫ﺒﺎﻟﻌﻼﻗﺔ‪:‬‬
‫‪r‬‬
‫‪a‬‬ ‫)‪(5.56‬‬
‫‪2‬‬
‫‪2  rv‬‬
‫ﺤﻴﺙ ‪ ‬ﺜﺎﺒﺕ ﻏﻭﺹ‪ r ،‬ﺒﻌﺩ ﺍﻟﻘﻤﺭ ﻋﻥ ﻤﺭﻜﺯ ﺍﻷﺭﺽ‪ ،‬ﻭﻟﻨﻔﺭﺽ ﺤﺩﻭﺙ‬
‫ﺍﻀﻁﺭﺍﺏ ﻨﺘﻴﺠﺔ ﻋﻭﺍﻤل ﺨﺎﺭﺠﻴﺔ ﺃﺩﻯ ﺇﻟﻰ ﺘﻐﻴﺭ ﻓﻲ ﺴﺭﻋﺘﻪ ‪ ،v‬ﻓﻤﺎ ﻫﻭ ﺘﺄﺜﻴﺭ ﺫﻟﻙ‬
‫ﻋﻠﻰ ‪ a‬ﻭﻋﻠﻰ ﺩﻭﺭ ﺍﻟﻘﻤﺭ ‪T‬؟‬
‫‪:‬‬ ‫ﻟﻺﺠﺎﺒﺔ ﻋﻥ ﻫﺫﺍ ﺍﻟﺴﺅﺍل ﻨﺤﺴﺏ ﺍﻟﻤﺸﺘﻕ‬
‫‪a‬‬
‫‪v‬‬
‫‪2‬‬ ‫‪2‬‬
‫‪a‬‬ ‫‪2r v‬‬ ‫‪2a v‬‬
‫‪‬‬ ‫‪‬‬ ‫)‪(5.57‬‬
‫‪v (2  rv 2 ) 2‬‬ ‫‪‬‬

‫ﻭﻫﻜﺫﺍ ﻨﺭﻯ ﺃﻥ ﺘﻐﻴﺭﺍﹰ ﻁﻔﻴﻔﺎﹰ ﻓﻲ ﺍﻟﺴﺭﻋﺔ ﻴﺅﺩﻱ ﺇﻟﻰ ﺘﻐﻴﺭ ﻓﻲ ‪ a‬ﻴﺘﻨﺎﺴﺏ ﻤﻊ‬
‫‪.v‬‬
‫ﻭﻟﺤﺴﺎﺏ ﺍﻟﺘﺄﺜﻴﺭ ﻋﻠﻰ ﺩﻭﺭ ﺍﻟﻘﻤﺭ ﻨﻨﻁﻠﻕ ﻤﻥ ﺍﻟﻌﻼﻗﺔ ﺍﻟﻤﻌﺭﻭﻓﺔ‪:‬‬
‫‪3‬‬ ‫‪3‬‬
‫‪a‬‬ ‫‪a‬‬ ‫‪2 3 2‬‬
‫‪T  2‬‬ ‫‪ 2‬‬ ‫‪‬‬ ‫‪a‬‬
‫‪2‬‬ ‫‪‬‬ ‫‪1‬‬
‫‪r g‬‬ ‫‪ 2‬‬

‫‪- 163 -‬‬

‫)‪Create PDF files without this message by purchasing novaPDF printer (http://www.novapdf.com‬‬
‫ﻭﻤﻨﻪ‪:‬‬
‫‪5‬‬ ‫‪3‬‬ ‫‪2‬‬
‫‪‬‬ ‫‪3aTv T T a‬‬ ‫‪2 3 1 2 2a v‬‬
‫‪‬‬ ‫‪6a 2  2 v‬‬ ‫‪‬‬ ‫‪‬‬ ‫‪. ‬‬ ‫‪a .‬‬ ‫)‪(5.58‬‬
‫‪ v a v‬‬ ‫‪1 2‬‬ ‫‪‬‬
‫‪‬‬ ‫‪2‬‬

‫ﻟﻨﺤﺴﺏ ﺍﻟﺘﻐﻴﺭ ﺍﻟﻨﺴﺒﻲ ﻟﻜل ﻤﻥ ‪ a‬ﻭ ‪ T‬ﻓﻨﺠﺩ‪:‬‬


‫‪a‬‬ ‫‪T‬‬
‫‪2‬‬ ‫‪av‬‬ ‫‪3av‬‬
‫‪v ‬‬ ‫‪, v ‬‬ ‫)‪(5.59‬‬
‫‪a‬‬ ‫‪‬‬ ‫‪T‬‬ ‫‪‬‬
‫ﻭﻤﻨﻬﺎ‪:‬‬
‫‪ T ‬‬ ‫‪ a ‬‬
‫‪ ‬‬ ‫‪ ‬‬
‫‪ v ‬‬ ‫‪ v   3av  3‬‬ ‫)‪(5.60‬‬
‫‪T‬‬ ‫‪a‬‬ ‫‪2av 2‬‬
‫ﻓﺎﻟﺘﻐﻴﺭ ﺍﻟﺫﻱ ﻴﻁﺭﺃ ﻋﻠﻰ ﺍﻟﺩﻭﺭ ﺃﻜﺒﺭ ﺒﻤﺭﺓ ﻭﻨﺼﻑ ﻤﻥ ﺫﻟﻙ ﺍﻟﺫﻱ ﻴﻁﺭﺃ ﻋﻠﻰ‬
‫ﻨﺼﻑ ﻁﻭل ﺍﻟﻤﺤﻭﺭ ﺍﻟﻜﺒﻴﺭ ﻟﻠﻤﺴﺎﺭ‪.‬‬
‫ﻤﺜﺎل ﺁﺨﺭ‪ :‬ﻴﻁﻠﺏ ﺤﺴﺎﺏ ﺘﻐﻴﺭ ﻋﻨﺎﺼﺭ ﻤﺴﺎﺭ ﺩﺍﺌﺭﻱ ﻟﺠﺴﻴﻡ ﻴﺘﺤﺭﻙ ﻓﻲ‬
‫ﺤﻘل ﻤﺭﻜﺯﻱ ﺇﺫﺍ ﺘﻐﻴﺭﺕ ﺍﻟﺴﺭﻋﺔ ﺒﺸﻜل ﻁﻔﻴﻑ‪.‬‬
‫ﺴﻨﻨﻁﻠﻕ ﻫﺫﻩ ﺍﻟﻤﺭﺓ ﻤﻥ ﻤﻌﺎﺩﻻﺕ ﺍﻟﺤﺭﻜﺔ ﻨﻔﺴﻬﺎ‪.‬‬
‫ﻟﺩﻴﻨﺎ ﺍﻟﻤﻌﺎﺩﻟﺔ ﺍﻷﻭﻟﻰ‪:‬‬
‫‪‬‬
‫)‪(5.61‬‬
‫‪‬‬ ‫‪‬‬ ‫‪‬‬ ‫‪2‬‬
‫‪f (r, r ,  )  r‬‬ ‫‪ r‬‬ ‫‪‬‬ ‫‪0‬‬
‫‪2‬‬
‫‪r‬‬
‫ﻭﻁﺒﻘﺎﹰ ﻟﻠﻤﻌﺎﺩﻟﺔ )‪ (5.55‬ﻭﺒﺈﻫﻤﺎل ﺍﻟﺤﺩﻭﺩ ﻤﻥ ﺍﻟﻤﺭﺘﺒﺔ ﺍﻟﺜﺎﻨﻴﺔ ﺼﻐﺭﺍﹰ ﻨﺠﺩ‬
‫ﺒﺎﻟﺘﻔﺎﻀل‪:‬‬
‫‪‬‬ ‫‪‬‬
‫‪‬‬ ‫‪2‬‬ ‫‪‬‬ ‫‪‬‬ ‫‪  2 2 ‬‬ ‫‪ ‬‬
‫‪f  f 0  f ‬‬ ‫‪r0‬‬ ‫‪ r0 0‬‬ ‫‪‬‬ ‫‪ r‬‬ ‫‪  0 ‬‬ ‫‪‬‬ ‫‪r  2r0  0   0‬‬
‫‪2‬‬ ‫‪‬‬ ‫‪3‬‬
‫‪r0‬‬ ‫‪‬‬ ‫‪r0 ‬‬
‫)‪(5.62‬‬
‫ﻭﺒﻤﺎ ﺃﻥ ﺍﻟﻤﻌﺎﺩﻟﺔ )‪ (5.61‬ﺘﺘﺤﻘﻕ ﻓﻲ ﻜل ﻨﻘﻁﺔ ﻭﻤﻨﻬﺎ ) ‪ ، M 0 (r0 , 0‬ﺤﻴﺙ‬
‫ﺒﺩﺃ ﺍﻻﻀﻁﺭﺍﺏ ﻴﻜﻭﻥ‪:‬‬

‫‪- 164 -‬‬

‫)‪Create PDF files without this message by purchasing novaPDF printer (http://www.novapdf.com‬‬
‫‪‬‬
‫)‪(5.63‬‬
‫‪‬‬ ‫‪2‬‬
‫‪r0  r0 0 ‬‬ ‫‪0‬‬
‫‪2‬‬
‫‪r0‬‬

‫ﻭﻴﻤﻜﻥ ﺃﻥ ﻨﻀﻊ ‪ r0  0‬ﻷﻥ ﺍﻟﻤﺴﺎﺭ ﺩﺍﺌﺭﻱ ﻭﺒﺎﻟﺘﺎﻟﻲ ﻨﺤﺼل ﻤﻥ )‪(5.63‬‬


‫‪‬‬

‫ﻋﻠﻰ ﺍﻟﻌﻼﻗﺔ‪:‬‬
‫‪‬‬
‫)‪(5.64‬‬
‫‪2‬‬
‫‪r0  0 ‬‬
‫‪2‬‬
‫‪r0‬‬
‫ﻭﻤﻥ )‪ (5.62‬ﻭ )‪ (5.64‬ﻨﺠﺩ ﺍﻟﻤﻌﺎﺩﻟﺔ‪:‬‬
‫)‪ 3 0 r  2r0  0   0 (5.65‬‬
‫‪‬‬ ‫‪2‬‬ ‫‪‬‬ ‫‪‬‬
‫‪r‬‬
‫ﻭﻤﻥ ﻤﻌﺎﺩﻟﺔ ﺍﻟﺤﺭﻜﺔ ﺍﻟﺜﺎﻨﻴﺔ ﺍﻟﻤﻌﺒﺭﺓ ﻋﻥ ﻗﺎﻨﻭﻥ ﺍﻟﺴﻁﻭﺡ‪:‬‬
‫‪d 2 ‬‬
‫)‪(5.66‬‬
‫‪ ‬‬ ‫‪‬‬
‫‪,‬‬ ‫‪(r  )  0  2r   r  0‬‬
‫‪dt‬‬
‫ﻨﺤﺼل ﺒﺎﻟﺘﻔﺎﻀل ﻋﻠﻰ ﺍﻟﻌﻼﻗﺔ‪:‬‬
‫)‪ 0 (5.67‬‬
‫‪ ‬‬ ‫‪‬‬ ‫‪‬‬ ‫‪‬‬ ‫‪‬‬ ‫‪‬‬ ‫‪‬‬ ‫‪‬‬
‫‪2r0  0  r0  0  2 0 r  2r0    0 r  r0 ‬‬

‫ﻓﺈﺫﺍ ﻋﻠﻤﻨﺎ ﺃﻨﻪ ‪ r0  0‬ﻷﻥ ﺍﻟﻤﺴﺎﺭ ﺩﺍﺌﺭﻱ ﻭﻜﺫﻟﻙ ﻨﺠﺩ ﻤﻥ )‪ (5.66‬ﺍﻟﺘﻲ‬


‫‪‬‬

‫ﺘﺘﺤﻘﻕ ﻓﻲ ﻜل ﻨﻘﻁ ﺍﻟﻤﺴﺎﺭ ﻭﻤﻨﻬﺎ ) ‪ ، M(r0 , 0‬ﺃﻥ ‪  0  0‬ﺃﻴﻀﺎﹰ‪ ،‬ﻭﻋﻨﺩﺌ ﺫ‪‬‬


‫‪‬‬

‫ﻨﺤﺼل ﻤﻥ )‪ (5.67‬ﻋﻠﻰ ﻤﻌﺎﺩﻟﺔ ﺜﺎﻨﻴﺔ ﺘﻀﺎﻑ ﺇﻟﻰ )‪ (5.65‬ﻫﻲ ﺍﻟﺘﺎﻟﻴﺔ‪:‬‬


‫‪‬‬ ‫‪‬‬ ‫‪‬‬
‫‪(5.68) r0 ‬‬ ‫‪ 2 0 r  0‬‬
‫ﻨﺤل ﺍﻟﻤﻌﺎﺩﻟﺘﻴﻥ ﺍﻟﺘﻔﺎﻀﻠﻴﺘﻴﻥ )‪ (5.65‬ﻭ )‪ (5.68‬ﺤﻼﹰ ﻤﺸﺘﺭﻜ ﹰﺎ ﺒﺎﻋﺘﺒﺎﺭ ﺃﻥ‬
‫ﺍﻟﻤﺠﻬﻭﻟﻴﻥ ﻫﻤﺎ ‪ r‬ﻭ ‪ ‬ﻭﺃﻥ ﺍﻟﺸﺭﻭﻁ ﺍﻻﺒﺘﺩﺍﺌﻴﺔ ﻫﻲ‪:‬‬
‫‪v‬‬
‫)‪(5.69‬‬
‫‪‬‬ ‫‪‬‬
‫‪(r ) t  0  r(0)  0, r (0)  0,  (0) ‬‬
‫‪r‬‬
‫ﻤﻊ ﺍﻟﻌﻠﻡ ﺃﻥ‪:‬‬
‫‪dr d‬‬ ‫‪‬‬
‫‪r  r0  r  r  ‬‬ ‫‪ r  r ‬‬
‫‪‬‬
‫‪dt dt‬‬
‫‪‬‬
‫‪‬‬ ‫)‪(5.70‬‬
‫‪d‬‬ ‫‪‬‬ ‫‪d‬‬ ‫‪‬‬
‫‪   0      ‬‬ ‫‪     ‬‬
‫‪dt dt‬‬ ‫‪‬‬
‫ﻨﺴﺘﻜﻤل ﺍﻟﻤﻌﺎﺩﻟﺔ )‪ (5.68‬ﻤﺒﺎﺸﺭﺓ ﻓﻨﺠﺩ‪:‬‬

‫‪- 165 -‬‬

‫)‪Create PDF files without this message by purchasing novaPDF printer (http://www.novapdf.com‬‬
‫)‪(5.71‬‬
‫‪‬‬ ‫‪‬‬
‫‪r0   2 0 r  v‬‬

‫ﻭﺒﺘﺒﺩﻴل ‪ r0 ‬ﺒﻘﻴﻤﺘﻬﺎ ﻤﻥ )‪ (5.65‬ﻓﻲ )‪ (5.71‬ﻨﺠﺩ‪:‬‬


‫‪‬‬

‫)‪(5.72‬‬
‫‪‬‬ ‫‪2‬‬ ‫‪‬‬
‫‪r‬‬ ‫‪  0 r  2 0 v‬‬
‫ﻭﻫﻲ ﻤﻌﺎﺩﻟﺔ ﺘﻔﺎﻀﻠﻴﺔ ﻤﻥ ﺍﻟﻤﺭﺘﺒﺔ ﺍﻟﺜﺎﻨﻴﺔ ﺒﺎﻟﻨﺴﺒﺔ ﺇﻟﻰ ‪ r‬ﺒﺤﻠﻬﺎ ﻨﺠﺩ‪:‬‬
‫‪2 v‬‬
‫‪r ‬‬ ‫)‪(1  cos 0 t ) (5.73‬‬
‫‪0‬‬

‫ﻭﻨﻼﺤﻅ ﺃﻥ ﺍﻟﻘﻴﻤﺔ ﺍﻟﻌﻅﻤﻰ ﻟـ ‪ r‬ﺘﻘﺎﺒل ‪  0 t 1  ‬ﻭﻫﻲ ﺘﺴﺎﻭﻱ‬


‫‪4v‬‬ ‫‪‬‬
‫‪‬‬
‫‪0‬‬
‫ﻭﻫﻜﺫﺍ ﻴﺘﺤﻭل ﺍﻟﻤﺴﺎﺭ ﻤﻥ ﺩﺍﺌﺭﺓ ﺇﻟﻰ ﻗﻁﻊ ﻨﺎﻗﺹ )ﺸﻜل ‪ (5.2‬ﻨﺼﻑ ﻁﻭل‬
‫ﻤﺤﻭﺭﻩ ﺍﻟﻜﺒﻴﺭ ﻫﻭ‪:‬‬

‫ﺍﻟﺸﻜل )‪(5.2‬‬

‫‪rmax  rmin r  (r) t    r  (r) t  0‬‬ ‫‪2 v‬‬


‫‪R‬‬ ‫‪‬‬ ‫‪r ‬‬
‫‪2‬‬ ‫‪2‬‬ ‫‪0‬‬

‫‪- 166 -‬‬

‫)‪Create PDF files without this message by purchasing novaPDF printer (http://www.novapdf.com‬‬
‫ﻭﻫﺫﺍ‬ ‫ﺃﻱ ﺃﻥ ﺍﻟﺘﻐﻴﺭ ﺍﻟﺤﺎﺼل ﻋﻠﻰ ﻨﺼﻑ ﻗﻁﺭ ﺍﻟﻤﺴﺎﺭ ﺍﻟﺩﺍﺌﺭﻱ ﻫﻭ‬
2v

0
:‫( ﻭﻟﻠﺘﺄﻜﺩ ﻤﻥ ﺫﻟﻙ ﻨﻜﺘﺏ‬5.57) ‫ﻴﺘﻭﺍﻓﻕ ﻤﻊ ﺍﻟﻨﺘﻴﺠﺔ ﺍﻟﺘﻲ ﺤﺼﻠﻨﺎ ﻋﻠﻴﻬﺎ ﺴﺎﺒﻘﺎﹰ‬
2 3 
 a  2a v 0 2r 
a    v  v  0 0 v
 v  0  
:(5.64) ‫ ﺒﻘﻴﻤﺘﻬﺎ ﻤﻥ‬ ‫ﺃﻭ ﺒﻌﺩ ﺘﺒﺩﻴل‬
2 v
a  (5.74)

0
.‫ﻭﻫﻜﺫﺍ ﺘﺅﺩﻱ ﺍﻟﻁﺭﻴﻘﺘﺎﻥ ﺇﻟﻰ ﺍﻟﻨﺘﺎﺌﺞ ﻨﻔﺴﻬﺎ‬
(5.71) ‫( ﻓﻲ‬5.73) ‫ ﺒﻘﻴﻤﺘﻬﺎ ﻤﻥ‬r ‫ ( ﻨﺒﺩل‬ ‫ )ﺃﻱ‬ ‫ﻭﻟﺤﺴﺎﺏ ﺘﻐﻴﺭ‬
 

:‫ﻓﻨﺠﺩ‬
v
   (3  4 cos  0 t ) (5.75)
r0
:‫ ﺒﺎﻟﻌﻼﻗﺔ‬‫ﻭﻴﺤﺴﺏ ﺍﻟﺩﻭﺭ ﻋﻨﺩﺌﺫ‬
T0  T
    3v 
2 ( 0   )dt    0  T  T 
  r  0
0  0 
T0  T
4v
Cos  0 tdt (5.76)

  r0
0
:‫ﻓﻴﻤﻜﻥ ﺃﻥ ﻨﺠﺩ ﻤﻥ ﺍﻟﻌﻼﻗﺔ ﺍﻷﺨﻴﺭﺓ‬  2 ‫ﻓﺈﺫﺍ ﻋﻠﻤﻨﺎ ﺃﻥ‬

 0T0
3v 3v
T (5.77)

 T0   0T  0  T 
r0  0
r0  0
:‫( ﻓﻨﺠﺩ‬5.76) ‫ﻟﻨﺤﺴﺏ ﺍﻟﺘﻜﺎﻤل ﺍﻟﺜﺎﻨﻲ ﻤﻥ‬
T t 0  t
4 v  4v 
 r0 cos 0 tdt  r  sin 0 t 
0 0 0 0

 
4v
r0 0
sin  (T

0 0  T)   4v
r0 0
sin  0 t  vT (5.78)

- 167 -

Create PDF files without this message by purchasing novaPDF printer (http://www.novapdf.com)
‫ﻭﻫﻭ ﻤﻥ ﺍﻟﻤﺭﺘﺒﺔ ﺍﻟﺜﺎﻨﻴﺔ ﺼﻐﺭﺍﹰ ﺒﺎﻟﻨﺴﺒﺔ ﻟﻠﺘﻜﺎﻤل ﺍﻷﻭل ﻭﻟﺫﻟﻙ ﻨﻬﻤﻠﻪ‪.‬‬
‫ﻭﺒﺎﻟﺘﺎﻟﻲ ﻓﺎﻟﺘﻐﻴﺭ ﺍﻟﺤﺎﺼل ﻋﻠﻰ ﺍﻟﺩﻭﺭ ﻫﻭ ﺍﻟﺘﻐﻴﺭ ﺍﻟﻤﺤﺴﻭﺏ ﺒﺎﻟﻌﻼﻗﺔ )‪.(5.77‬‬
‫ﻭﻟﻠﺘﺄﻜﺩ ﻤﻥ ﺘﻁﺎﺒﻕ ﻫﺫﻩ ﺍﻟﻨﺘﻴﺠﺔ ﻤﻊ )‪ (5.58‬ﺍﻟﺘﻲ ﺘﻭﻀﻊ ﺒﺎﻟﺼﻴﻐﺔ‪:‬‬
‫‪‬‬
‫‪3aT0 v‬‬ ‫‪3r0 T0 r0  0‬‬ ‫‪3vT0‬‬
‫‪T ‬‬ ‫‪v ‬‬ ‫‪‬‬ ‫)‪(5.58b‬‬
‫‪‬‬ ‫‪3 2‬‬ ‫‪‬‬
‫‪r0  0‬‬ ‫‪r0  0‬‬
‫ﻭﻫﻲ ﺍﻟﻨﺘﻴﺠﺔ )‪ (5.77‬ﻨﻔﺴﻬﺎ‪.‬‬
‫ﻣﺜﺎﻝ ﳏﻠﻮﻝ‪:‬‬
‫ﻴﺘﻌﺭﺽ ﻨﻭﺍﺱ ﻤﺨﺭﻭﻁﻲ ﻁﻭﻟﻪ ‪ a‬ﻴﺼﻨﻊ ﺯﺍﻭﻴﺔ ‪   0‬ﻤﻊ ﺍﻟﺸﺎﻗﻭل‪ ،‬ﺇﻟﻰ‬
‫ﺍﻀﻁﺭﺍﺏ ﻴﺴﺒﺏ ﺍﻨﺤﺭﺍﻓﺎﹰ ﺒﻤﻘﺩﺍﺭ ‪. ‬‬
‫ﺍﻭﺠﺩ ﺍﻟﺘﻐﻴﺭ ﻓﻲ ﺍﻟﺘﻭﺍﺘﺭ ﺍﻟﻨﺎﺘﺞ ﻋﻥ ﻫﺫﺍ ﺍﻻﻀﻁﺭﺍﺏ‪ .‬ﺍﺩﺭﺱ ﺍﻟﺤﺎﻟﺔ ﺍﻟﺨﺎﺼﺔ‬
‫ﻋﻨﺩﻤﺎ ﺘﻜﻭﻥ ‪ ‬ﺼﻐﻴﺭﺓ ﺠﺩ ﺍﹰ )‪. (  1‬‬

‫ﺍﻟﺸﻜل )‪(5.3‬‬

‫‪- 168 -‬‬

‫)‪Create PDF files without this message by purchasing novaPDF printer (http://www.novapdf.com‬‬
‫ﺍﳊﻞ‪:‬‬
‫ﻨﻜﺘﺏ ﺘﺎﺒﻊ ﻻﻏﺭﺍﻨﺞ ﻟﻬﺫﺍ ﺍﻟﻨﻭﺍﺱ )ﺸﻜل ‪ (5.3‬ﺒﻔﺭﺽ ﺃﻥ ﺇﺤﺩﺍﺜﻴﺎﺘﻪ ﻫﻲ‬
‫)‪ M (r,   , ‬ﻓﻨﺠﺩ‪:‬‬
‫‪1‬‬
‫‪L  TV ‬‬ ‫‪m(r  2  r 2   2  r  sin 2  2 )  mgz‬‬
‫‪2‬‬
‫‪1‬‬
‫‪(5.79)  ma 2 (  2    2 sin 2 )  mga cos ‬‬
‫‪2‬‬
‫ﻭﻤﻨﻪ ﻨﺤﺼل ﻋﻠﻰ ﻤﻌﺎﺩﻟﺘﻲ ﺍﻟﺤﺭﻜﺔ ﺍﻟﺘﺎﻟﻴﺘﻴﻥ‪:‬‬

‫)‪f1 (   ,   , )  a    a 2 sin  cos   g sin   0 (5.80‬‬

‫)‪f (  , )   2 sin 2    02 sin 2  0 (5.81‬‬

‫ﻨﻔﺎﻀل )‪ (5.80‬ﻭ )‪ (5.81‬ﻁﺒﻘﺎﹰ ﻟـ )‪ (5.55‬ﻓﻨﺤﺼل ﻋﻠﻰ ﺍﻟﻤﻌﺎﺩﻟﺘﻴﻥ‪:‬‬


‫‪a   2a0 sin  0 cos  0 '  a 02 / cos 2   sin 2 )  g cos  0   0‬‬

‫)‪(5.82‬‬

‫)‪sin  0   2 0 cos  0   0 (5.83‬‬

‫ﻨﺤﺫﻑ ‪ ‬ﻤﻥ ﺍﻟﻤﻌﺎﺩﻟﺘﻴﻥ ﺍﻟﺴﺎﺒﻘﺘﻴﻥ ﻓﻨﺤﺼل ﻋﻠﻰ ﺍﻟﻤﻌﺎﺩﻟﺔ ﺍﻟﺘﺎﻟﻴﺔ‪:‬‬


‫‪‬‬

‫‪q‬‬
‫‪  ‬‬ ‫‪(4 cos  0  sin  0 tg  0 )  0‬‬ ‫)‪(5.84‬‬
‫‪a‬‬

‫ﻭﻫﻲ ﻤﻌﺎﺩﻟﺔ ﺘﻔﺎﻀﻠﻴﺔ ﻤﻥ ﺍﻟﻤﺭﺘﺒﺔ ﺍﻟﺜﺎﻨﻴﺔ ﺫﺍﺕ ﺃﻤﺜﺎل ﺜﺎﺒﺘﺔ ﺘﻤﺜل ﺤﺭﻜﺔ‬
‫ﺍﻫﺘﺯﺍﺯﻴﺔ ﺘﻭﺍﻓﻘﻴﺔ‪ ،‬ﻭﺒﺎﻟﺘﺎﻟﻲ ﻓﺎﻟﺘﻐﻴﺭ ﺍﻟﺤﺎﺼل ﻓﻲ ‪) ‬ﺃﻱ ‪ ( ‬ﻴﻜﻭﻥ ﺩﻭﺭﻴﺎﹰ ﺘﻭﺍﺘﺭﻩ‬
‫ﻴﻌﻁﻰ ﺒﺎﻟﻌﻼﻗﺔ‪:‬‬
‫‪1‬‬
‫‪g‬‬ ‫‪ 2‬‬
‫‪   (4 cos  0  sin  0 tg 0 ‬‬ ‫)‪(5.85‬‬
‫‪a‬‬ ‫‪‬‬

‫ﻭﻋﻨﺩﻤﺎ ‪   1‬ﻴﻤﻜﻥ ﻨﺸﺭ ﺍﻟﺠﺫﺭ ﺒﻌﺩ ﺘﺒﺩﻴل ‪ cos 0‬ﺒـ ‪ 1‬ﻭ ‪ sin  0‬ﺒـ‬
‫‪ 0‬ﻭﻜﺫﻟﻙ ‪ tg 0‬ﺒـ ‪ 0‬ﻭﻫﻜﺫﺍ ﻨﺠﺩ‪:‬‬

‫‪- 169 -‬‬

‫)‪Create PDF files without this message by purchasing novaPDF printer (http://www.novapdf.com‬‬
‫‪‬‬ ‫‪2‬‬ ‫‪‬‬
‫‪g‬‬ ‫‪g  0‬‬
‫)‪ (5.86‬‬
‫‪2‬‬ ‫‪‬‬
‫‪‬‬ ‫‪( 4   0 )  2 1 ‬‬
‫‪a‬‬ ‫‪a‬‬ ‫‪g‬‬
‫‪‬‬ ‫‪‬‬
‫ﺃﻤﺎ ﺍﻟﺩﻭﺭ ‪ T‬ﺍﻟﻤﻘﺎﺒل ﻟﻬﺫﺍ ﺍﻟﺘﻭﺍﺘﺭ ﻓﻴﺴﺎﻭﻱ‪:‬‬
‫‪1‬‬
‫‪‬‬ ‫‪2‬‬ ‫‪‬‬ ‫‪‬‬ ‫‪2‬‬ ‫‪‬‬
‫‪2 2 a   0‬‬ ‫‪a  0‬‬
‫)‪ (5.87‬‬
‫‪‬‬ ‫‪‬‬
‫‪T‬‬ ‫‪‬‬ ‫‪1‬‬ ‫‪  1 ‬‬
‫‪‬‬ ‫‪2 g ‬‬ ‫‪8‬‬ ‫‪‬‬ ‫‪g‬‬ ‫‪8‬‬
‫‪‬‬ ‫‪‬‬ ‫‪‬‬ ‫‪‬‬
‫ﻭﻫﻭ ﻤﺎﻴﻌﺎﺩل ﺘﻘﺭﻴﺒﺎﹰ ﻨﺼﻑ ﺍﻟﺩﻭﺭ ﻋﻨﺩﻤﺎ ﻻﻴﻭﺠﺩ ﺍﻀﻁﺭﺍﺏ‪.‬‬
‫ﻭﺍﻟﺯﻤﻥ ﺍﻟﻼﺯﻡ ﻟﺩﻭﺭﺘﻴﻥ ﻫﻭ‪:‬‬
‫‪‬‬ ‫‪2‬‬ ‫‪‬‬
‫‪4‬‬ ‫‪a  0‬‬
‫‪t1 ‬‬ ‫‪ 2 1 ‬‬
‫‪‬‬
‫‪‬‬ ‫)‪(5.88‬‬
‫‪‬‬ ‫‪g‬‬ ‫‪8‬‬
‫‪‬‬ ‫‪‬‬
‫ﻭﻟﺤﺴﺎﺏ ﺍﻟﺯﺍﻭﻴﺔ ‪ ‬ﺍﻟﺘﻲ ﻴﻘﻁﻌﻬﺎ ﺍﻟﻨﻭﺍﺱ ﺨﻼل ﻫﺫﻩ ﺍﻟﻔﺘﺭﺓ ﻨﻼﺤﻅ ﺃﻭﻻﹰ ﻤﻥ‬
‫)‪ (5.80‬ﻋﻨﺩﻤﺎ ‪)  0  0‬ﺍﻟﻤﻌﺎﺩﻟﺔ ﺼﺤﻴﺤﺔ ﻭﻤﺤﻘﻘﺔ ﻟﻠﻤﺴﺎﺭ ﺍﻟﻤﺴﺘﻘﺭ ﻋﻨﺩﻤﺎ‬
‫‪‬‬

‫ﻻﻴﻭﺠﺩ ﺍﻀﻁﺭﺍﺏ(‪:‬‬
‫‪‬‬ ‫‪2‬‬
‫‪g‬‬ ‫‪g  0 ‬‬
‫)‪(5.89‬‬
‫‪‬‬
‫‪0‬‬ ‫‪‬‬ ‫‪‬‬ ‫‪1‬‬
‫‪a Cos  0‬‬ ‫‪a ‬‬ ‫‪4 ‬‬
‫‪‬‬ ‫‪‬‬
‫ﻭﻋﻨﺩﺌﺫ‪ ‬ﻨﺠﺩ ﺃﻥ ﺍﻟﺯﺍﻭﻴﺔ ﺍﻟﻤﻘﻁﻭﻋﺔ ﺨﻼل ﺍﻟﺯﻤﻥ ‪ t 1‬ﻫﻲ‪:‬‬
‫‪‬‬ ‫‪2‬‬ ‫‪  2‬‬ ‫‪‬‬ ‫‪ 2‬‬ ‫‪‬‬
‫‪a  0‬‬
‫)‪ (5.90‬‬
‫‪‬‬ ‫‪‬‬ ‫‪0‬‬ ‫‪‬‬ ‫‪‬‬ ‫‪0‬‬ ‫‪‬‬
‫‪1 ‬‬ ‫‪ 0 t1‬‬ ‫‪ 2 1 ‬‬ ‫‪1  8‬‬ ‫‪  21  8‬‬
‫‪g‬‬ ‫‪4‬‬
‫‪‬‬ ‫‪‬‬ ‫‪‬‬ ‫‪‬‬ ‫‪‬‬
‫ﻭﺍﻟﺨﻼﺼﺔ ﺃﻨﻪ ﺒﻴﻨﻤﺎ ‪ ‬ﺘﻘﻁﻊ ﺍﻟﻤﺴﺎﺭ ﻜﻠﻪ ﺃﻱ ‪ 2‬ﺨﻼل ﺍﻟﺯﻤﻥ ‪ t 1‬ﺘﻜﻭﻥ‬
‫‪ ‬ﻗﺩ ﻤﺭﺕ ﺒﻘﻴﻤﺘﻬﺎ ﺍﻟﻌﻅﻤﻰ ﻭﺍﻟﺼﻐﺭﻯ ﻤﺭﺘﻴﻥ )ﺸﻜل ‪.(5.4‬‬

‫‪- 170 -‬‬

‫)‪Create PDF files without this message by purchasing novaPDF printer (http://www.novapdf.com‬‬
(5.4) ‫ﺍﻟﺸﻜل‬

- 171 -

Create PDF files without this message by purchasing novaPDF printer (http://www.novapdf.com)
- 172 -

Create PDF files without this message by purchasing novaPDF printer (http://www.novapdf.com)
‫‪‬‬
‫‪‬‬

‫ﺣﺮﻛﺔ ﺟﺴﻢ ﺻﻠﺐ ﰲ ﺍﻟﻔﻀﺎء‬


‫‪MOTION OF RIGID BODY IN SPACE‬‬

‫ﺘﻜﻭﻥ ﻓﻲ ﺍﻟﺤﺎﻟﺔ ﺍﻟﻌﺎﻤﺔ ﺤﺭﻜﺔ ﺍﻟﺠﺴﻡ ﺍﻟﺼﻠﺏ ﻓﻲ ﺍﻟﻔﻀﺎﺀ ﻤﻜﻭﻨﺔ ﻤﻥ‬


‫ﺤﺭﻜﺘﻴﻥ‪ ،‬ﺍﻷﻭﻟﻰ ﺤﺭﻜﺔ ﺍﻨﺴﺤﺎﺒﻴﺔ ﻟﻨﻘﻁﺔ ﻤﻥ ﺍﻟﺠﺴﻡ‪ ،‬ﺘﻌﺘﺒﺭ ﻫﺫﻩ ﺍﻟﻨﻘﻁﺔ ﻓﻲ ﻤﻌﻅﻡ‬
‫ﺍﻷﺤﻴﺎﻥ ﻨﻘﻁﺔ ﻤﺭﻜﺯ ﺜﻘل ﺍﻟﺠﺴﻡ ﺍﻟﺼﻠﺏ‪ ،‬ﻭﺍﻟﺤﺭﻜﺔ ﺍﻟﺜﺎﻨﻴﺔ‪ ،‬ﺩﻭﺭﺍﻨﻴﺔ ﺤﻭل ﻤﺤﻭﺭ‬
‫ﻤﺎﺭ ﻤﻥ ﻤﺭﻜﺯ ﺜﻘل ﺍﻟﺠﺴﻡ ﻋﺎﺩﺓ‪ ،‬ﻏﻴﺭ ﺃﻥ ﺍﺘﺠﺎﻩ ﺍﻟﻤﺤﻭﺭ ﻓﻲ ﻫﺫﻩ ﺍﻟﺤﺎﻟﺔ ﻤﺘﻐﻴﺭ ﻤﻊ‬
‫ﺍﻟﺯﻤﻥ‪ ،‬ﻭﻫﺫﺍ ﻫﻭ ﺍﻟﻔﺭﻕ ﺒﻴﻥ ﺍﻟﺤﺭﻜﺔ ﺍﻟﻤﺴﺘﻭﻴﺔ ﻟﻠﺠﺴﻡ ﺍﻟﺼﻠﺏ ﻭﺍﻟﺤﺭﻜﺔ ﺍﻟﻔﻀﺎﺌﻴﺔ‬
‫ﻟﻪ‪ ،‬ﻜﻤﺎ ﺃﻨﻪ ﻴﻜﻔﻲ ﺜﻼﺜﺔ ﻭﺴﻁﺎﺀ ﻟﺘﻌﻴﻴﻥ ﺤﺭﻜﺔ ﺠﺴﻡ ﺼﻠﺏ ﻴﺘﺤﺭﻙ ﺤﺭﻜﺔ ﻤﺴﺘﻭﻴﺔ‪،‬‬
‫ﺒﻴﻨﻤﺎ ﻟﺘﻌﻴﻴﻥ ﺤﺭﻜﺘﻪ ﺍﻟﻔﻀﺎﺌﻴﺔ ﻴﻠﺯﻡ ﺴﺘﺔ ﻭﺴﻁﺎﺀ‪ .‬ﺜﻼﺜﺔ ﻤﻨﻬﺎ ﻟﺘﻌﻴﻴﻥ ﺍﻟﺤﺭﻜﺔ‬
‫ﺍﻹﻨﺴﺤﺎﺒﻴﺔ ﻟﻤﺭﻜﺯ ﺜﻘل ﺍﻟﺠﺴﻡ ﻫﻲ ])‪ [x(c) ،y(c) ،z(c‬ﻭﺍﻟﺜﻼﺜﺔ ﺍﻷﺨﺭﻯ ﻟﺘﻌﻴﻴﻥ‬
‫ﺍﻟﺤﺭﻜﺔ ﺍﻟﺩﻭﺭﺍﻨﻴﺔ‪ ،‬ﻻﻥ ﺘﻌﻴﻴﻥ ﻤﺤﻭﺭ ﺍﻟﺩﻭﺭﺍﻥ ﻴﺘﻁﻠﺏ ﻤﺘﺤﻭﻟﻴﻥ ﻤﺴﺘﻘﻠﻴﻥ ﻭﺘﻌﻴﻴﻥ‬
‫ﻭﻀﻊ ﺍﻟﺠﺴﻡ ﺍﻟﺩﺍﺌﺭ ﺤﻭل ﺍﻟﻤﺤﻭﺭ ﻴﺘﻁﻠﺏ ﻤﺘﺤﻭﻻﹰ ﻤﺴﺘﻘﻼﹰ ﻭﺍﺤﺩ ﺍﹰ‪.‬‬
‫ﻤﻥ ﺍﻟﻤﻌﻠﻭﻡ ﺃﻥ ﻤﻌﺎﺩﻟﺔ ﺤﺭﻜﺔ ﻤﺭﻜﺯ ﺜﻘل ﺠﺴﻡ ﺼﻠﺏ ﺃﻭ ﻤﺠﻤﻭﻋﺔ ﻤﻴﻜﺎﻨﻴﻜﻴﺔ‬
‫ﺘﺨﻀﻊ ﻟﺘﺄﺜﻴﺭ ﻗﻭﻯ ﺨﺎﺭﺠﻴﺔ ﻫﻲ‪:‬‬
‫‪‬‬ ‫‪‬‬
‫‪(6.1) F  MrCM‬‬
‫‪‬‬
‫ﺤﻴﺙ ‪ rCM‬ﺘﺴﺎﺭﻉ ﻤﺭﻜﺯ ﺜﻘل ﺍﻟﺠﺴﻡ ﺍﻟﺼﻠﺏ ﺃﻭ ﺍﻟﻤﺠﻤﻭﻋﺔ ﺍﻟﻤﻴﻜﺎﻨﻴﻜﻴﺔ‬
‫‪‬‬
‫ﺍﻟﻤﺘﺤﺭﻜﺔ ﻨﺘﻴﺠﺔ ﺘﺄﺜﻴﺭ ﻗﻭﻯ ﺨﺎﺭﺠﻴﺔ ﻤﺤﺼﻠﺘﻬﺎ ‪ . F‬ﺃﻱ ﺃﻥ ﺍﻟﺤﺭﻜﺔ ﺍﻹﻨﺴﺤﺎﺒﻴﺔ‬
‫ﻟﺠﺴﻡ ﺼﻠﺏ ﺘﺩﺭﺱ ﻜﺩﺭﺍﺴﺔ ﺍﻟﺤﺭﻜﺔ ﺍﻹﻨﺴﺤﺎﺒﻴﺔ ﻟﻨﻘﻁﺔ ﻤﺎﺩﻴﺔ ﻭﺍﺤﺩﺓ‪ .‬ﻭﻗﺩ ﺃﻏﻨﻴﻨﺎ‬
‫ﻓﻲ ﻤﻘﺭﺭﺍﺕ ﻓﻴﺯﻴﺎﺌﻴﺔ ﺴﺎﺒﻘﺔ ﺩﺭﺍﺴﺔ ﺍﻟﺤﺭﻜﺔ ﺍﻹﻨﺴﺤﺎﺒﻴﺔ ﻟﻨﻘﻁﺔ ﻤﺎﺩﻴﺔ ﻓﻲ ﻤﺨﺘﻠﻑ‬

‫‪- 173 -‬‬

‫)‪Create PDF files without this message by purchasing novaPDF printer (http://www.novapdf.com‬‬
‫ﺍﻟﺠﻤل ﺍﻹﺤﺩﺍﺜﻴﺔ‪ .‬ﻭﺴﻨﺭﻜﺯ ﺍﻟﺩﺭﺍﺴﺔ ﻓﻲ ﻫﺫﺍ ﺍﻟﻔﺼل ﻋﻠﻰ ﺩﺭﺍﺴﺔ ﺍﻟﺤﺭﻜﺔ ﺍﻟﺩﻭﺭﺍﻨﻴﺔ‬
‫ﻟﺠﺴﻡ ﺼﻠﺏ ﺤﻭل ﻤﺤﻭﺭ ﻤﺘﻐﻴﺭ ﻤﺎﺭ ﻤﻥ ﻤﺭﻜﺯ ﺍﻟﻜﺘﻠﺔ‪.‬‬
‫‪- 44‬ﻋﺰﻭﻡ ﻭﻣﻀﺎﺭﻳﺐ ﺍﻟﻌﻄﺎﻟﺔ ‪Moments and products of Inertia‬‬
‫‪‬‬
‫ﻴﻤﻜﻥ ﺤﺴﺎﺏ ﺍﻟﻌﺯﻡ ﺍﻟﺯﺍﻭﻱ )ﺍﻟﻌﺯﻡ ﺍﻟﺤﺭﻜﻲ( ‪ L‬ﻟﺠﺴﻡ ﺼﻠﺏ ﺤﻭل ﻨﻘﻁﺔ ‪O‬‬
‫‪‬‬
‫ﺒﺩﻻﻟﺔ ﺍﻟﺴﺭﻋﺔ ﺍﻟﺯﺍﻭﻴﺔ ‪. ω‬‬
‫ﻴﺘﻌﻴﻥ ﻤﻭﻀﻊ ﻨﻘﻁﺔ ﻤﺎﺩﻴﺔ ﻜﺘﻠﺘﻬﺎ ‪ mi‬ﻤﻥ ﺍﻟﺠﺴﻡ ﺒﺎﻟﻨﺴﺒﺔ ﺇﻟﻰ ﺍﻟﻨﻘﻁﺔ ‪O‬‬
‫‪‬‬
‫ﺒﻤﺘﺠﻪ ﻤﻭﻀﻌﻬﺎ ‪ ، ri‬ﻭﻴﻌﻁﻰ ﻋﻨﺩﺌ ﺫ‪ ‬ﻤﺘﺠﻪ ﺴﺭﻋﺘﻬﺎ ﺒﺩﻻﻟﺔ ﺍﻟﺴﺭﻋﺔ ﺍﻟﺯﺍﻭﻴﺔ ﺒﺎﻟﻌﻼﻗﺔ‬
‫ﺍﻟﺘﺎﻟﻴﺔ‪:‬‬
‫‪  ‬‬
‫‪(6.2) v i  ωx ri‬‬
‫ﺃﻤﺎ ﺍﻟﻌﺯﻡ ﺍﻟﺯﺍﻭﻱ ﻟﻜل ﺍﻟﺠﺴﻡ ﺍﻟﺼﻠﺏ ﺤﻭل ﺍﻟﻨﻘﻁﺔ ‪ O‬ﻓﻴﺴﺎﻭﻱ‪:‬‬
‫‪‬‬ ‫‪ ‬‬ ‫‪‬‬ ‫‪ ‬‬
‫‪(6.3) L   m i  ri xv i    m i ri x ωxri ‬‬
‫‪i‬‬ ‫‪i‬‬

‫ﺒﺄﺨﺫ ﺍﻟﺠﻤﻊ ﻋﻠﻰ ﺠﻤﻴﻊ ﻨﻘﻁ ﺍﻟﺠﺴﻡ ﺍﻟﺼﻠﺏ‪ ،‬ﻭﺒﺎﺴﺘﻌﻤﺎل ﺨﻭﺍﺹ ﺍﻟﻀﺭﺏ‬
‫ﺍﻻﺘﺠﺎﻫﻲ ﻓﻲ ﺍﻟﻌﻼﻗﺔ ﺍﻟﺴﺎﺒﻘﺔ‪ ،‬ﻨﺤﺼل ﻋﻠﻰ‪:‬‬
‫‪‬‬ ‫‪‬‬ ‫‪ 2‬‬ ‫‪  ‬‬
‫‪(6.4) L  ω  m i ri‬‬
‫‪   m i ri ri .ω‬‬ ‫‪ ‬‬
‫‪i‬‬ ‫‪ i‬‬
‫‪‬‬
‫ﻤﻥ ﺍﻟﺠﺩﻴﺭ ﻤﻼﺤﻅﺘﻪ ﻟﻴﺱ ﺒﺎﻟﻀﺭﻭﺭﺓ ﺃﻥ ﻴﻜﻭﻥ ﻤﺘﺠﻪ ﺍﻟﻌﺯﻡ ﺍﻟﺯﺍﻭﻱ ‪L‬‬
‫‪‬‬
‫ﻤﻭﺍﺯﻴﺎﹰ ﻟﻤﺘﺠﻪ ﺍﻟﺴﺭﻋﺔ ﺍﻟﺯﺍﻭﻴﺔ ‪ . ω‬ﻴﻤﻜﻥ ﻜﺘﺎﺒﺔ ﻤﺴﻘﻁ ﺍﻟﻤﻌﺎﺩﻟﺔ )‪ ،(6.4‬ﺒﺩﻻﻟﺔ‬
‫ﺍﻹﺤﺩﺍﺜﻴﺎﺕ ﺍﻟﻜﺎﺭﺘﻴﺯﻴﺔ )ﺍﻟﺩﻴﻜﺎﺭﺘﻴﺔ(‪ ،‬ﺒﺎﻟﺼﻭﺭﺓ‪:‬‬

‫‪L x  ω x  m i  y 2  z 2   ω y  m i x i y i  ω z  m i x i z i‬‬
‫‪i‬‬ ‫‪ i‬‬ ‫‪i ‬‬ ‫‪i‬‬ ‫‪i‬‬
‫‪(6.5) L y  ω x  m i y i x i  ω y  m i  x 2  z 2   ω z  m i y i z i‬‬
‫‪i‬‬ ‫‪i‬‬ ‫‪ i‬‬ ‫‪i ‬‬ ‫‪i‬‬
‫‪L z  ω x  m i z i x i  ω y  m i z i y i  ω z  mi  x 2  y 2 ‬‬
‫‪i‬‬ ‫‪i‬‬ ‫‪i‬‬ ‫‪ i‬‬ ‫‪i ‬‬
‫ﻭﻟﺘﺒﺴﻴﻁ ﺍﻟﻤﻌﺎﺩﻻﺕ ﻓﻲ )‪ ،(6.5‬ﻨﻌﺭﻑ ﺍﻟﻜﻤﻴﺎﺕ ﺍﻟﺘﺎﻟﻴﺔ‪:‬‬

‫‪- 174 -‬‬

‫)‪Create PDF files without this message by purchasing novaPDF printer (http://www.novapdf.com‬‬
‫‪‬‬ ‫‪‬‬
‫‪I xx   m i y 2i  z 2i , I xy   mi x i y i , I xz    mi x i z i‬‬
‫‪i‬‬ ‫‪i‬‬ ‫‪i‬‬
‫‪‬‬ ‫‪‬‬
‫‪I yx   m i y i x i , I yy   m i x 2i  z i2 , I yz   m i yi z i‬‬ ‫)‪(6.6‬‬
‫‪i‬‬ ‫‪i‬‬ ‫‪i‬‬
‫‪‬‬
‫‪I zx    m i z i x i , I zy   m i z i y i , I zz   m i x i2  y i2‬‬ ‫‪‬‬
‫‪i‬‬ ‫‪i‬‬ ‫‪i‬‬

‫ﺒﺎﻟﺘﻌﻭﻴﺽ ﻤﻥ ﺍﻟﻤﻌﺎﺩﻻﺕ )‪ ، (6.6‬ﻓﻲ ﺍﻟﻤﻌﺎﺩﻻﺕ )‪ ،(6.5‬ﻨﺠﺩ ﺃﻥ ﻤﺭﻜﺒﺎﺕ‬


‫ﺍﻟﻌﺯﻡ ﺍﻟﺯﺍﻭﻱ ﺘﺼﺒﺢ ﺒﺎﻟﺼﻭﺭﺓ‪:‬‬
‫‪L x  I xx ωx  I xy ω y  I xz ωz‬‬
‫‪(6.7) L y  I yx ω x  I yyω y  I yz ωz‬‬
‫‪L z  I zx ω x  I zy ω y  I zz ω z‬‬
‫ﻭﻴﻤﻜﻥ ﺍﻟﺘﻌﺒﻴﺭ ﻋﻥ ﻤﺭﻜﺒﺎﺕ ﺍﻟﻌﺯﻡ ﺍﻟﺯﺍﻭﻱ ﻓﻲ ﺍﻟﻤﻌﺎﺩﻻﺕ )‪ ،(6.7‬ﺒﺼﻭﺭﺓ‬
‫ﻤﺨﺘﺼﺭﺓ ﻜﻤﺎ ﻴﻠﻲ‪:‬‬
‫‪(6.8) L j   I jk ω k‬‬
‫‪k‬‬

‫ﺤﻴﺙ ﺘﺄﺨﺫ ﺍﻟﻤﺘﺤﻭﻻﺕ ‪ j,k‬ﺍﻟﻘﻴﻡ ‪ ، x,y,z‬ﻭﺘﻌﺭﻑ ﺍﻟﻜﻤﻴﺎﺕ ‪ Ijj‬ﺒﻌﺯﻭﻡ‬


‫ﺘﺴﻤﻰ ﻤﻀﺎﺭﻴﺏ ﺍﻟﻌﻁﺎﻟﺔ‪ .‬ﻴﻤﻜﻥ ﺃﻥ‬ ‫ﺍﻟﻌﻁﺎﻟﺔ ﻭﺍﻟﻜﻤﻴﺎﺕ ﺍﻟﺴﺘﺔ ‪ Ijk‬ﺤﻴﺙ ‪j≠k‬‬
‫ﻨﻼﺤﻅ ﻤﻥ ﺍﻟﻤﻌﺎﺩﻻﺕ )‪ ،(6.6‬ﺃﻥ ﻤﻀﺎﺭﻴﺏ ﺍﻟﻌﻁﺎﻟﺔ ﻤﺘﻨﺎﻅﺭﺓ‪ ،‬ﺃﻱ ﺃﻥ‪:‬‬
‫‪(6.9) I jk  I kj‬‬
‫ﻨﺴﺘﻁﻴﻊ ﻓﻲ ﻫﺫﻩ ﺍﻟﺤﺎﻟﺔ ﻜﺘﺎﺒﺔ ﺍﻟﻜﻤﻴﺎﺕ ﺍﻟﺘﺴﻌﺔ ‪ Ijk‬ﻋﻠﻰ ﻫﻴﺌﺔ ﻤﺼﻔﻭﻓﺔ‬
‫ﺘﻨﺴﻭﺭﻴﺔ ‪ 3×3‬ﺒﺎﻟﺼﻭﺭﺓ‪:‬‬
‫‪ I xx‬‬ ‫‪I xy‬‬ ‫‪I xz ‬‬
‫‪‬‬ ‫‪‬‬
‫‪(6.10) II   I yx‬‬ ‫‪I yy‬‬ ‫‪I yz ‬‬
‫‪I‬‬ ‫‪Izy‬‬ ‫‪I zz ‬‬
‫‪ zx‬‬
‫ﺘﺴﻤﻰ ﻫﺫﻩ ﺍﻟﻤﺼﻔﻭﻓﺔ ﻤﺼﻔﻭﻓﺔ ﺍﻟﻌﻁﺎﻟﺔ‪.‬‬
‫ﻴﻤﻜﻥ ﺒﺎﻻﺴﺘﻔﺎﺩﺓ ﻤﻥ ﻤﺼﻔﻭﻓﺔ ﺍﻟﻌﻁﺎﻟﺔ ﺼﻴﺎﻏﺔ ﺍﻟﻌﻼﻗﺎﺕ ﻓﻲ ﺍﻟﻤﻌﺎﺩﻻﺕ‬
‫)‪ ،(6.8‬ﻋﻠﻰ ﻫﻴﺌﺔ ﻤﻌﺎﺩﻟﺔ ﻤﺘﺠﻬﺔ ﻭﺍﺤﺩﺓ ﺒﺎﻟﺸﻜل‪:‬‬
‫‪‬‬ ‫‪‬‬
‫‪(6.11) L  II.ω‬‬

‫‪- 175 -‬‬

‫)‪Create PDF files without this message by purchasing novaPDF printer (http://www.novapdf.com‬‬
‫ﻭﻨﺴﺘﻁﻴﻊ ﺒﺎﻻﺴﺘﻔﺎﺩﺓ ﻤﻥ ﺍﻟﺩﺭﺍﺴﺔ ﺍﻟﺴﺎﺒﻘﺔ ﺼﻴﺎﻏﺔ ﺍﻟﻤﻌﺎﺩﻟﺔ ﺍﻟﻌﺎﻤﺔ ﻟﺩﻭﺭﺍﻥ‬
‫ﺠﺴﻡ ﺼﻠﺏ ﺤﻭل ﻤﺭﻜﺯ ﻜﺘﻠﺘﻪ ‪ C‬ﺃﻭ ﺤﻭل ﻨﻘﻁﺔ ﻤﺜﺒﺘﺔ ﻓﻲ ﺍﻟﻔﻀﺎﺀ ﺒﺎﻟﻌﻼﻗﺔ‪:‬‬
‫‪dL j‬‬ ‫‪d‬‬
‫‪(6.12) M j ‬‬
‫‪dt‬‬
‫‪‬‬
‫‪dt‬‬
‫‪I jk .ωk ‬‬
‫‪k‬‬

‫ﺃﻭ ﺒﺼﻭﺭﺓ ﻤﻌﺎﺩﻟﺔ ﻤﺘﺠﻬﺔ ﻤﺨﺘﺼﺭﺓ ﻜﺎﻵﺘﻲ‪:‬‬


‫‪d‬‬ ‫‪‬‬ ‫‪‬‬
‫‪‬‬
‫‪(6.13) M  L ‬‬
‫‪II.ω‬‬
‫‪dt‬‬
‫‪‬‬
‫ﺤﻴﺙ ‪ M‬ﻤﺘﺠﻪ ﻋﺯﻡ ﺍﻟﻘﻭﺓ ﺍﻟﻤﺅﺜﺭﺓ ﻋﻠﻰ ﺍﻟﺠﺴﻡ‪.‬‬
‫ﻫﺫﺍ ﻭﻴﻤﻜﻥ ﻜﺘﺎﺒﺔ ﻋﻼﻗﺔ ﻁﺎﻗﺔ ﺤﺭﻜﺔ ﺠﺴﻡ ﺼﻠﺏ ﺒﺩﻻﻟﺔ ﻋﺯﻭﻡ ﻭﻤﻀﺎﺭﻴﺏ‬
‫ﺍﻟﻌﻁﺎﻟﺔ ﺒﺎﻟﻤﻌﺎﺩﻟﺔ ﺍﻟﺘﺎﻟﻴﺔ‪:‬‬
‫‪1‬‬ ‫‪ ‬‬ ‫‪1‬‬ ‫‪   ‬‬ ‫‪1 ‬‬ ‫‪   ‬‬
‫‪(6.14) T ‬‬ ‫‪‬‬ ‫‪m i .v i .v i   m i ωxri ‬‬
‫‪. ωxri   ω m i ri x ωxri ‬‬
‫‪2 i‬‬ ‫‪2 i‬‬ ‫‪2  i‬‬ ‫‪‬‬
‫ﻨﺤﺼل ﺒﺎﻟﺘﻌﻭﻴﺽ ﻤﻥ ﺍﻟﻤﻌﺎﺩﻟﺔ )‪ ،(6.3‬ﻭﺍﻟﻤﻌﺎﺩﻟﺔ )‪ ،(6.8‬ﻓﻲ ﺍﻟﻤﻌﺎﺩﻟﺔ‬
‫)‪ ،(6.14‬ﻋﻠﻰ ﻁﺎﻗﺔ ﺍﻟﺤﺭﻜﺔ ﺒﺎﻟﺸﻜل‪:‬‬
‫‪1 ‬‬ ‫‪1‬‬
‫‪(6.15) T  ω.L ‬‬
‫‪2‬‬
‫‪ I jk ω j .ω k‬‬
‫‪2 jk‬‬
‫ﻭﺘﻜﺘﺏ ﻋﻠﻰ ﻫﻴﺌﺔ ﻤﻌﺎﺩﻟﺔ ﻤﺘﺠﻬﺔ ﻜﻤﺎ ﻴﻠﻲ‪:‬‬

‫‪1‬‬ ‫‪‬‬
‫‪(6.16) T  ω.II.ω‬‬
‫‪2‬‬
‫‪- 45‬ﻣﻌﺎﺩﻻﺕ ﺣﺮﻛﺔ ﺟﺴﻢ ﺻﻠﺐ ﻃﻠﻴﻖ‬
‫‪Equations of Motion Free Rigid Body:‬‬
‫ﻴﻌﺭﻑ ﺍﻟﺠﺴﻡ ﺍﻟﺼﻠﺏ ﺍﻟﻁﻠﻴﻕ ﺒﺄﻨﻪ ﺍﻟﺠﺴﻡ ﺍﻟﺫﻱ ﻻ ﻴﺨﻀﻊ ﺃﺜﻨﺎﺀ ﺤﺭﻜﺘﻪ ﺇﻟﻰ‬
‫ﺃﻱ ﺍﺭﺘﺒﺎﻁﺎﺕ‪ ،‬ﻭﺒﺎﻟﺘﺎﻟﻲ ﻻ ﺘﺅﺜﺭ ﻋﻠﻰ ﻫﺫﺍ ﺍﻟﺠﺴﻡ ﺴﻭﻯ ﺍﻟﻘﻭﻯ ﺍﻟﻔﻌﺎﻟﺔ ﺍﻟﺘﻲ ﻨﻔﺭﺽ‬
‫‪‬‬
‫ﺃﻥ ﻟﻬﺎ ﻤﺤﺼﻠﺔ ﺘﺴﺎﻭﻱ ‪ ، F‬ﻜﻤﺎ ﺫﻜﺭﻨﺎ ﻓﻲ ﺍﻟﻔﻘﺭﺓ ﺍﻟﺴﺎﺒﻘﺔ ﻓﺎﻥ ﺍﻟﺤﺭﻜﺔ ﺍﻟﻌﺎﻤﺔ ﻟﺠﺴﻡ‬
‫ﺼﻠﺏ ﺘﺘﻌﻴﻥ ﺒﺴﺘﺔ ﻭﺴﻁﺎﺀ‪ .‬ﻹﻴﺠﺎﺩ ﻤﻌﺎﺩﻟﺔ ﺤﺭﻜﺔ ﺠﺴﻡ ﺼﻠﺏ ﻨﻁﺒﻕ ﻨﻅﺭﻴﺔ ﻜﻤﻴﺔ‬
‫ﺍﻟﺤﺭﻜﺔ ﻭﺍﻟﻌﺯﻡ ﺍﻟﺯﺍﻭﻱ‪ ،‬ﻓﻭﻓﻘﺎﹰ ﻟﻨﻅﺭﻴﺔ ﻜﻤﻴﺔ ﺍﻟﺤﺭﻜﺔ ﻴﻜﻭﻥ‪:‬‬
‫‪‬‬ ‫‪‬‬
‫‪(6.17) MrcM  Fxc , y c , z c , θ, φ, ψ‬‬

‫‪- 176 -‬‬

‫)‪Create PDF files without this message by purchasing novaPDF printer (http://www.novapdf.com‬‬
‫ﺒﺈﺴﻘﺎﻁ ﻫﺫﻩ ﺍﻟﻤﻌﺎﺩﻟﺔ ﻋﻠﻰ ﺜﻼﺜﺔ ﻤﺤﺎﻭﺭ ﺇﺤﺩﺍﺜﻴﺔ ﻤﺘﻌﺎﻤﺩﺓ ﺜﺎﺒﺘﺔ ‪ XYZ‬ﻨﺠﺩ‪:‬‬
‫‪(6.18) Mxc  Fx , Myc  Fy , Mzc  Fz‬‬
‫ﻫﺫﻩ ﺍﻟﻤﻌﺎﺩﻻﺕ ﻜﺎﻓﻴﺔ ﻟﺘﻌﻴﻴﻥ ﺤﺭﻜﺔ ﻤﺭﻜﺯ ﻜﺘﻠﺔ ﺍﻟﺠﺴﻡ ﺍﻟﻤﺘﺤﺭﻙ ﺒﺩﻻﻟﺔ‬
‫ﺍﻟﺯﻤﻥ‪ ،‬ﻭﻟﺘﻌﻴﻴﻥ ﺍﻟﺤﺭﻜﺔ ﺍﻟﺩﻭﺭﺍﻨﻴﺔ ﻟﻠﺠﺴﻡ ﺍﻟﺼﻠﺏ‪ ،‬ﻨﺤﺎﻭل ﺇﻴﺠﺎﺩ ﺜﻼﺙ ﻤﻌﺎﺩﻻﺕ‬
‫ﺃﺨﺭﻯ ﺘﺼﻑ ﺍﻟﺤﺭﻜﺔ ﺍﻟﺩﻭﺭﺍﻨﻴﺔ‪ ،‬ﻭﻫﺫﺍ ﻤﻤﻜﻥ ﻤﻥ ﺘﻁﺒﻴﻕ ﻨﻅﺭﻴﺔ ﺍﻟﻌﺯﻡ ﺍﻟﺤﺭﻜﻲ‬
‫‪‬‬
‫‪dL  ext‬‬
‫‪ M‬ﻋﺯﻭﻡ ﺍﻟﻘﻭﻯ ﺍﻟﺨﺎﺭﺠﻴﺔ ﺍﻟﻤﺅﺜﺭﺓ ﻋﻠﻰ ﺍﻟﺠﺴﻡ(‪.‬‬ ‫‪ext‬‬
‫‪) . M‬‬
‫‪dt‬‬
‫ﺒﻔﺭﺽ ﺃﻥ ﺠﻤﻠﺔ ﺇﺤﺩﺍﺜﻴﺎﺕ ‪ OXYZ‬ﻤﺜﺒﺘﺔ ﻭﺠﻤﻠﺔ ﺇﺤﺩﺍﺜﻴﺎﺕ ﺃﺨﺭﻯ ‪cxyz‬‬
‫ﻤﺘﻤﺎﺴﻜﺔ ﻤﻊ ﺍﻟﺠﺴﻡ ﺍﻟﺼﻠﺏ ﻜﻤﺎ ﻫﻭ ﻤﺒﻴﻥ ﻋﻠﻰ ﺍﻟﺸﻜل )‪.(6.1‬‬
‫‪‬‬
‫ﻴﻌﻁﻰ ﻤﺘﺠﻪ ﺍﻟﻌﺯﻡ ﺍﻟﺤﺭﻜﻲ ‪ ، L‬ﺒﺩﻻﻟﺔ ﻤﺭﻜﺒﺎﺘﻪ ﻋﻠﻰ ﻤﺤﺎﻭﺭ ﺍﻟﺠﻤﻠﺔ‬
‫ﺍﻹﺤﺩﺍﺜﻴﺔ ‪ ،xyz‬ﺒﺎﻟﻌﻼﻗﺔ‪:‬‬
‫‪‬‬ ‫‪‬‬ ‫‪‬‬ ‫‪‬‬
‫‪(6.19) L  L x i  L y j  L z k‬‬

‫ﺍﻟﺸﻜل)‪ :(6.1‬ﺍﻟﺤﺭﻜﺔ ﺍﻟﻌﺎﻤﺔ ﻟﻠﺠﺴﻡ ﺍﻟﺼﻠﺏ‬


‫‪‬‬
‫ﺒﺎﺸﺘﻘﺎﻕ ﺍﻟﻌﺯﻡ ﺍﻟﺤﺭﻜﻲ ‪ L‬ﺒﺎﻟﻨﺴﺒﺔ ﻟﻠﺯﻤﻥ ﻓﻲ ﺠﻤﻠﺔ ﺍﻹﺤﺩﺍﺜﻴﺎﺕ ﺍﻟﻤﺜﺒﺘﺔ‬
‫‪‬‬
‫‪ ،XYZ‬ﻨﺤﺼل ﻋﻠﻰ ‪ M‬ﻋﺯﻡ ﺍﻟﻘﻭﺓ ﺍﻟﻤﺅﺜﺭﺓ ﺒﺎﻟﻨﺴﺒﺔ ﺇﻟﻰ ﻫﺫﻩ ﺍﻟﺠﻤﻠﺔ‪ .‬ﻴﺠﺏ‬
‫‪  ‬‬ ‫‪‬‬
‫ﻤﻼﺤﻅﺔ ﺃﻨﻪ ﻟﻴﺱ ﺍﻟﻤﺘﺠﻪ ‪ L‬ﻭﺤﺩﻩ ﻫﻭ ﺍﻟﻤﺘﻐﻴﺭ ﺒل ﺘﺘﻐﻴﺭ ﻤﺘﺠﻬﺎﺕ ﺍﻟﻭﺤﺩﺓ ‪k , j , i‬‬

‫‪- 177 -‬‬

‫)‪Create PDF files without this message by purchasing novaPDF printer (http://www.novapdf.com‬‬
‫ ﺍﻟﺘﻲ ﺘﺩﻭﺭ )ﺘﺘﻐﻴﺭ ﻤﻊ ﺍﻟﺯﻤﻥ( ﺒﺎﻟﻨﺴﺒﺔ ﺇﻟﻰ‬z,y,x ‫ﻷﻨﻬﺎ ﻤﺤﻤﻭﻟﺔ ﻋﻠﻰ ﺍﻟﻤﺤﺎﻭﺭ‬
:‫ ﻟﺫﻟﻙ ﻨﺠﺩ ﺃﻥ‬،‫ﺍﻟﻤﺠﻤﻭﻋﺔ ﺍﻟﻤﺜﺒﺘﺔ‬
   
 dL dL x  dL y  dL z  di d j dk
M  i j k  Lx  Ly  Lz
dt F dt dt dt dt dt dt
(6.20)    
dL di dj dk
  Lx  Ly  Lz
dt M dt dt dt
‫ ﺘﺤﺕ ﺍﻟﺨﻁ ﺍﻟﻘﺎﺌﻡ ﻋﻠﻰ ﺃﻥ ﺍﻻﺸﺘﻘﺎﻕ‬،(6.20) ‫ ﻓﻲ ﺍﻟﻤﻌﺎﺩﻟﺔ‬M ‫ﻴﺩل ﺍﻟﺭﻤﺯ‬
‫ ﻋﻠﻰ ﺃﻥ ﺍﻻﺸﺘﻘﺎﻕ ﻴﺘﻡ‬F ‫ ﻭ ﻴﺩل ﺍﻟﺭﻤﺯ‬xyz ‫ﻴﺘﻡ ﺒﺎﻟﻨﺴﺒﺔ ﺇﻟﻰ ﺍﻟﺠﻤﻠﺔ ﺍﻟﻤﺘﺤﺭﻜﺔ‬
.XYZ ‫ﺒﺎﻟﻨﺴﺒﺔ ﺇﻟﻰ ﺍﻟﺠﻤﻠﺔ ﺍﻟﻤﺜﺒﺘﺔ‬
  
‫ ﺒﺎﻟﻨﺴﺒﺔ ﻟﻠﺯﻤﻥ ﻭﻨﻌﻭﺽ‬, j , i ‫ﻨﺤﺴﺏ ﺍﻵﻥ ﻤﺸﺘﻘﺎﺕ ﻤﺘﺠﻬﺎﺕ ﺍﻟﻭﺍﺤﺩﺓ‬
k

‫ ﻫﻲ ﻨﻬﺎﻴﺎﺕ‬A,B,C ‫ ﻤﻥ ﺃﺠل ﺫﻟﻙ ﻨﻔﺭﺽ ﺃﻥ ﺍﻟﻨﻘﻁ‬، (6.20) ‫ﻓﻲ ﺍﻟﻤﻌﺎﺩﻟﺔ‬


  
‫ ﻓﻨﺤﺼل ﻤﻥ‬،( 6.1) ‫ ﻋﻠﻰ ﺍﻟﺘﺭﺘﻴﺏ ﻜﻤﺎ ﻋﻠﻰ ﺍﻟﺸﻜل‬i , j , k ‫ﻤﺘﺠﻬﺎﺕ ﺍﻟﻭﺍﺤﺩﺓ‬
:‫ﻤﺸﺘﻘﺎﺕ ﻫﺫﻩ ﺍﻟﻤﺘﺠﻬﺎﺕ ﻋﻠﻰ‬
 
d i doA    
  vA   ω  oA  ω  i
dt dt
 
d j doB    
(6.21)   vB  ω  oB  ω  j
dt dt
 
dk doC    
  vC   ω  oC  ω  k
dt dt
:‫ﻨﺤﺼل ﻋﻠﻰ‬،(6.20) ‫ﺒﺎﻟﺘﻌﻭﻴﺽ ﻓﻲ ﺍﻟﻤﻌﺎﺩﻟﺔ‬
   
 dL dL x  dL y  dLz  di dj dk
M  i j k  Lx  Ly  Lz
dt F dt dt dt dt dt dt

dL      

dt M
   
 L x ω  i  L y ω  j  Lz ω  k  

dL      

dt M
    
 ω  L x i  ω  L y j  ω  Lz k 
 
dL     dL  

dt M

 ωx Lx i  L y j  Lz k 
dt M
 ωL

- 178 -

Create PDF files without this message by purchasing novaPDF printer (http://www.novapdf.com)
‫ﺃﻱ ﺃﻥ‪:‬‬
‫‪‬‬ ‫‪‬‬
‫‪ dL‬‬ ‫‪dL‬‬ ‫‪ ‬‬
‫‪M‬‬ ‫‪‬‬ ‫‪ ω L‬‬ ‫)‪(6.22‬‬
‫‪dt F dt M‬‬

‫ﻨﺴﺘﻨﺘﺞ ﺃﻥ ﻋﺯﻡ ﺍﻟﻘﻭﺓ ﺍﻟﻤﺅﺜﺭﺓ ﻋﻠﻰ ﺍﻟﺠﺴﻡ ﺒﺎﻟﻨﺴﺒﺔ ﻟﻤﺠﻤﻭﻋﺔ ﻤﺤﺎﻭﺭ ﻤﺜﺒﺘﺔ‬
‫‪‬‬
‫ﻴﺴﺎﻭﻱ ﺇﻟﻰ ﻋﺯﻡ ﻫﺫﻩ ﺍﻟﻘﻭﺓ ﺒﺎﻟﻨﺴﺒﺔ ﻟﻤﺠﻤﻭﻋﺔ ﻤﺤﺎﻭﺭ ﺘﺩﻭﺭ ﺒﺴﺭﻋﺔ ﺯﺍﻭﻴﺔ ‪ω‬‬
‫‪‬‬
‫ﺒﺎﻟﻨﺴﺒﺔ ﻟﻠﺠﻤﻠﺔ ﺍﻟﻤﺜﺒﺘﺔ ﻤﻀﺎﻓﺎ ﺇﻟﻴﻬﺎ ﺍﻟﻀﺭﺏ ﺍﻻﺘﺠﺎﻫﻲ ﻟﻤﺘﺠﻪ ﺍﻟﺴﺭﻋﺔ ﺍﻟﺯﺍﻭﻴﺔ ‪ω‬‬
‫‪‬‬
‫ﻟﺩﻭﺭﺍﻥ ﺍﻟﺠﻤﻠﺔ ﺍﻟﻤﺘﺤﺭﻜﺔ ﻓﻲ ﻤﺘﺠﻪ ﺍﻟﻌﺯﻡ ﺍﻟﺯﺍﻭﻱ ‪ L‬ﻟﻠﺠﺴﻡ ﺍﻟﻤﺘﺤﺭﻙ‪ .‬ﻨﺴﻘﻁ‬
‫ﺍﻟﻤﻌﺎﺩﻟﺔ )‪ ،(6.22‬ﻋﻠﻰ ﻤﺤﺎﻭﺭ ﺍﻟﺠﻤﻠﺔ ‪ cxyz‬ﺍﻟﻤﺘﺤﺭﻜﺔ ﻤﻊ ﺍﻟﺠﺴﻡ ﺍﻟﺼﻠﺏ‪ ،‬ﻨﺠﺩ‪:‬‬
‫‪dL x‬‬
‫‪Mx ‬‬ ‫‪ ω y L z  ω z L y ‬‬
‫‪dt‬‬
‫‪dL‬‬
‫‪(6.23) M y  y  ω z L x  ω x L z ‬‬
‫‪dt‬‬
‫‪dL z‬‬
‫‪Mz ‬‬ ‫‪ ω x L y  ω y L x ‬‬
‫‪dt‬‬
‫ﺤﻴﺙ ‪ Lx ،Ly ،Lz‬ﻋﺯﻭﻡ ﺍﻟﻌﻁﺎﻟﺔ ﺤﻭل ﺍﻟﻤﺤﺎﻭﺭ ‪ cx ،cy ،cz‬ﻋﻠﻰ‬
‫ﺍﻟﺘﺭﺘﻴﺏ‪.‬‬
‫‪ - 46‬ﺍﶈﺎﻭﺭ ﺍﻟﺮﺋﻴﺴﻴﺔ ﻭﻣﻌﺎﺩﻻﺕ ﺃﻭﻟﺮ‪:‬‬
‫‪Principal Axes and Euler's Equations:‬‬
‫ﺘﻌﺘﺒﺭ ﺍﻟﺤﺎﻟﺔ ﺍﻷﻜﺜﺭ ﺴﻬﻭﻟﺔ ﻫﻲ ﺍﻋﺘﺒﺎﺭ ﻤﺤﺎﻭﺭ ﺍﻟﺠﻤﻠﺔ ﺍﻟﻤﺘﺤﺭﻜﺔ ‪cxyz‬‬
‫ﻤﺤﺎﻭﺭ ﺭﺌﻴﺴﻴﺔ )ﻤﺤﺎﻭﺭ ﻋﻁﺎﻟﺔ ﺃﺴﺎﺴﻴﺔ( )‪ (principal axes‬ﻟﻠﺠﺴﻡ ﺍﻟﺼﻠﺏ ﻓﻲ‬
‫ﺍﻟﻌﻁﺎﻟﺔ ﺘﺴﺎﻭﻱ‬ ‫ﺍﻟﺭﺴﻡ ﺍﻟﺴﺎﺒﻕ‪ .‬ﻴﻤﻜﻥ ﻓﻲ ﻫﺫﻩ ﺍﻟﺤﺎﻟﺔ ﺍﻋﺘﺒﺎﺭ ﺃﻥ ﻤﻀﺎﺭﻴﺏ‬
‫ﺍﻟﺼﻔﺭ‪ ،‬ﺃﻱ ﺃﻥ‪:‬‬
‫‪ i  j‬ﻤﻥ ﺃﺠل ‪I ij  0‬‬ ‫)‪(6.24‬‬

‫ﺘﺴﻤﻰ ﺍﻟﻤﺤﺎﻭﺭ ﺍﻟﺘﻲ ﻤﻥ ﺃﺠﻠﻬﺎ ﺘﺘﺤﻘﻕ ﺍﻟﻤﻌﺎﺩﻟﺔ )‪ ،(6.24‬ﺒﻤﺤﺎﻭﺭ ﺍﻟﻌﻁﺎﻟﺔ‬


‫ﺍﻟﺭﺌﻴﺴﻴﺔ ﻟ ﻠﺠﺴﻡ ﺍﻟﺼﻠﺏ )ﻤﺤﺎﻭﺭ ﻤﺘﻌﺎﻤﺩﺓ ﻭﺘﻨﻌﺩﻡ ﻤﻥ ﺃﺠﻠﻬﺎ ﻤﻀﺎﺭﻴﺏ ﺍﻟﻌﻁﺎﻟﺔ (‪.‬‬
‫‪‬‬
‫ﻴﻤﻜﻥ ﻜﺘﺎﺒﺔ ﻤﺭﻜﺒﺎﺕ ﺍﻟﻌﺯﻡ ﺍﻟﺯﺍﻭﻱ ‪ ، L‬ﺍﻟﻤﻌﺎﺩﻻﺕ )‪ ،(6.7‬ﺒﺎﻟﻨﺴﺒﺔ ﻟﻤﺤﺎﻭﺭ ﺍﻟﻌﻁﺎﻟﺔ‬
‫ﺍﻟﺭﺌﻴﺴﻴﺔ ﺒﺎﻟﺼﻭﺭﺓ‪:‬‬

‫‪- 179 -‬‬

‫)‪Create PDF files without this message by purchasing novaPDF printer (http://www.novapdf.com‬‬
‫‪L1  I11ω1  I1ω1‬‬
‫‪L 2  I 22 ω 2  I 2 ω 2‬‬ ‫)‪(6.25‬‬
‫‪L 3  I 33ω 3  I 3 ω3‬‬
‫ﻴﻤﻜﻥ ﺍﻟﺤﺼﻭل ﻤﻥ ﺍﻟﻤﻌﺎﺩﻟﺔ )‪ ،(6.22‬ﻋﻠﻰ ﻤﻌﺎﺩﻻﺕ ﺍﻟﺤﺭﻜﺔ ﺍﻟﺩﻭﺭﺍﻨﻴﺔ‬
‫ﻟﻠﺠﺴﻡ ﺍﻟﺼﻠﺏ ﺒﺎﻟﻨﺴﺒﺔ ﻟﺠﻤﻠﺔ ﺇﺤﺩﺍﺜﻴﺎﺕ ﻤﺤﺎﻭﺭ ﺍﻟﻌﻁﺎﻟﺔ ﺍﻟﺭﺌﻴﺴﻴﺔ ﺍﻟﻤﺘﻤﺎﺴﻜﺔ ﻤﻊ‬
‫ﺍﻟﺠﺴﻡ ﺍﻟﺼﻠﺏ‪ ،‬ﻭﺫﻟﻙ ﺒﺎﻟﺼﻭﺭﺓ‪:‬‬
‫‪M1  I1ω‬‬‫‪ 1  I3  I2 ω3ω2‬‬
‫‪(6.26) M 2  I2ω 2  I1  I3 ω1ω3‬‬
‫‪M3  I3ω 3  I 2  I1 ω2ω1‬‬
‫‪‬‬
‫ﺘﺴﻤﻰ ﻫﺫﻩ ﺍﻟﻤﻌﺎﺩﻻﺕ ﺒﻤﻌﺎﺩﻻﺕ ﺃﻭﻟﺭ‪ ،‬ﻴﻌﻁﻲ ﺤﻠﻬﺎ ﻤﺘﺠﻪ ﺍﻟﺩﻭﺭﺍﻥ ‪ ‬ﻓﻲ‬
‫ﺃﻴﺔ ﻟﺤﻅﺔ ﺯﻤﻨﻴﺔ ‪ ،t‬ﻭﻴﺘﺒﻴﻥ ﻤﻥ ﻫﺫﻩ ﺍﻟﻤﻌﺎﺩﻻﺕ ﺃﻨﻪ ﻤﻥ ﺍﻟﺼﻌﺏ ﺇﻥ ﻟﻡ ﻴﻜﻥ ﻤﻥ‬
‫ﺍﻟﻤﺴﺘﺤﻴل ﺤﻠﻬﺎ ﺇﻻ ﻓﻲ ﺤﺎﻻﺕ ﺨﺎﺼﺔ‪ ،‬ﻭﻟﺸﺭﺡ ﺘﻁﺒﻴﻘﺎﺕ ﻤﻌﺎﺩﻻﺕ ﺃﻭﻟﺭ ﻨﺩﺭﺱ‬
‫ﺩﻭﺭﺍﻥ ﺠﺴﻡ ﺼﻠﺏ ﻋﻠﻰ ﺸﻜل ﻗﻀﻴﺏ ﻁﻭﻴل ﻓﻲ ﻨﻬﺎﻴﺘﻴﻪ ﻜﺘﻠﺘﻴﻥ ﻜ لٍ ﻤﻨﻬﻤﺎ ‪،m‬‬
‫ﻴﺩﻭﺭ ﺒﺴﺭﻋﺔ ﺯﺍﻭﻴﺔ ﺜﺎﺒﺘﺔ‪ ،‬ﺤﻴﺙ ﻨﻌﺘﺒﺭ ﺍﻟﻤﺤﺎﻭﺭ ﺍﻹﺤﺩﺍﺜﻴﺔ ﻜﻤﺎ ﻫﻲ ﻤﺒﻴﻨﺔ ﻋﻠﻰ‬
‫ﺍﻟﺸﻜل )‪.(6.2‬‬

‫‪  ‬‬
‫ﺍﻟﺸﻜل)‪ :(6.2‬ﺍﻟﻤﺤﺎﻭﺭ ﺍﻟﺭﺌﻴﺴﻴﺔ ‪ x1 , x 2 , x 3‬ﻟﻠﻜﺘﻠﺘﻴﻥ ﻓﻲ ﺍﻟﻘﻀﻴﺏ‬

‫‪- 180 -‬‬

‫)‪Create PDF files without this message by purchasing novaPDF printer (http://www.novapdf.com‬‬
‫‪‬‬
‫ﻴﻌﺘﺒﺭ ﺍﻟﻤﺤﻭﺭ ‪ z‬ﻋﻠﻰ ﺍﻤﺘﺩﺍﺩ ﺍﻟﻘﻀﻴﺏ ﻭﺍﻟﻤﺤﻭﺭ ‪ x‬ﻭﺍﻟﻤﺘﺠﻪ ‪ ‬ﻓﻲ ﻤﺴﺘﻭﻯ‬
‫‪‬‬
‫ﺍﻟﻘﻀﻴﺏ‪ ،‬ﻋﻨﺩﺌﺫ‪ ‬ﺘﻜﻭﻥ ﻤﺭﻜﺒﺎﺕ ﻤﺘﺠﻪ ﺍﻟﺴﺭﻋﺔ ﺍﻟﺯﺍﻭﻴﺔ ‪ ‬ﻫﻲ‪:‬‬
‫‪1   sin ‬‬
‫‪(6.27) ω 2  0‬‬
‫‪ω 3  ωcosθ‬‬
‫‪‬‬
‫ﺤﻴﺙ ‪ θ‬ﻫﻲ ﺍﻟﺯﺍﻭﻴﺔ ﺒﻴﻥ ﺍﻟﻤﺘﺠﻪ ‪ ω‬ﻭﺍﻟﻘﻀﻴﺏ )ﺍﻟﻤﺤﻭﺭ ‪ ،( z‬ﻨﺠﺩ ﺤﺴﺏ‬
‫ﻤﺒﺩﺃ ﻋﺯﻡ ﺍﻟﻌﻁﺎﻟﺔ ﺃﻥ‪:‬‬
‫‪2‬‬ ‫‪2‬‬
‫‪l‬‬ ‫‪l‬‬ ‫‪1‬‬
‫‪(6.28) I1  I 2  m   m   ml 2‬‬
‫‪ 2‬‬ ‫‪ 2‬‬ ‫‪2‬‬
‫ﺒﺎﻟﺘﻌﻭﻴﺽ ﻤﻥ ﺍﻟﻤﻌﺎﺩﻟﺘﻴﻥ )‪ (6.27‬ﻭ)‪ ،(6.28‬ﻓﻲ ﺍﻟﻤﻌﺎﺩﻻﺕ )‪،(6.26‬‬
‫ﻨﺤﺼل ﻋﻠﻰ ﻤﻌﺎﺩﻻﺕ ﺍﻟﺤﺭﻜﺔ ﺒﺎﻟﺸﻜل ﺍﻟﺘﺎﻟﻲ‪:‬‬
‫‪M1  0‬‬
‫‪1‬‬ ‫‪‬‬
‫‪(6.29) M 2   ml 2 ω 2sinθ cosθ‬‬
‫‪2‬‬ ‫‪‬‬
‫‪M3  0‬‬
‫‪ ، ω‬ﻭﺘﻡ ﺍﻟﺤﺼﻭل ﻋﻠﻰ ﻫﺫﻩ ﺍﻟﻨﺘﻴﺠﺔ ﻤﻥ ﻤﻌﺎﺩﻻﺕ ﺃﻭﻟﺭ‪.‬‬
‫ﺤﻴﺙ ﺃﻥ ‪  0‬‬
‫ﻭﻗﺩ ﺍﺴﺘﺨﺩﻤﻨﺎ ﻋﻨﺩ ﺍﻟﺤﺼﻭل ﻋﻠﻰ ﺍﻟﻤﻌﺎﺩﻻﺕ )‪ ،(6.26‬ﺍﻟﺨﺎﺼﺔ ﺍﻟﻘﻁﺭﻴﺔ‬
‫)‪ ،(6.24‬ﻟﺘﻨﺴﻭﺭ ﺍﻟﻌﻁﺎﻟﺔ ﻓﻲ ﺠﻤﻠﺔ ﺍﻟﻤﺤﺎﻭﺭ ﺍﻟﺭﺌﻴﺴﻴﺔ‪ ،‬ﻭﺴﻨﺤﺎﻭل ﺍﻵﻥ ﺘﺭﺴﻴﺦ ﻫﺫﻩ‬
‫ﺍﻟﺨﺎﺼﺔ‪ .‬ﺒﻔﺭﺽ ﺃﻨﻪ ﻴﻭﺠﺩ ﺍﺘﺠﺎﻩ ﻓﻲ ﻓﻀﺎﺀ ‪ ω‬ﺒﺤﻴﺙ ﻴﻜﻭﻥ ﻤﺘﺠﻪ ﺍﻟﻌﺯﻡ ﺍﻟﺯﺍﻭﻱ‬
‫‪‬‬ ‫‪‬‬
‫‪ L‬ﻭﻤﺘﺠﻪ ﺍﻟﺴﺭﻋﺔ ﺍﻟﺯﺍﻭﻴﺔ ‪ ‬ﻤﺘﻭﺍﺯﻴﺎﻥ‪ ،‬ﻴﻤﻜﻨﻨﺎ ﻋﻨﺩﺌﺫ‪ ‬ﺃﻥ ﻨﻜﺘﺏ ﻤﺘﺠﻪ ﺍﻟﻌﺯﻡ‬
‫ﺍﻟﺯﺍﻭﻱ ﺒﺎﻟﺼﻭﺭﺓ‪:‬‬
‫‪‬‬ ‫‪‬‬
‫‪(6.30) L  Iω‬‬
‫ﺇﺫﺍ ﺃﻤﻜﻥ ﺇﻴﺠﺎﺩ ﻤﺜل ﻫﺫﺍ ﺍﻻﺘﺠﺎﻩ‪ ،‬ﻓﺈﻨﻪ ﻴﻤﻜﻥ ﺘﻌﺭﻴﻑ ﺍﻟﻤﺤﻭﺭ ﺍﻟﺭﺌﻴﺴﻲ ﺤﻴﺙ‬
‫ﺘﻨﻌﺩﻡ ﻤﻀﺎﺭﻴﺏ ﺍﻟﻌﻁﺎﻟﺔ‪ ،‬ﻭﻴﻜﻭﻥ ﻋﺯﻡ ﺍﻟﻌﻁﺎﻟﺔ ﺍﻟﺭﺌﻴﺴﻲ ﻫﻭ ‪ .I‬ﻴﻤﻜﻥ ﺒﺸﻜل ﻋﺎﻡ‬
‫‪‬‬
‫ﻜﺘﺎﺒﺔ ﻤﺘﺠﻪ ﺍﻟﺴﺭﻋﺔ ﺍﻟﺯﺍﻭﻴﺔ ‪ ‬ﺒﺩﻻﻟﺔ ﻤﺭﻜﺒﺎﺘﻪ ﺒﺎﻟﺸﻜل ﺍﻟﺘﺎﻟﻲ‪:‬‬

‫‪‬‬
‫̂‪(6.31) ω  ω1x̂  ω 2 ŷ  ω 3 z‬‬
‫ﺤﻴﺙ ̂‪ x̂, ŷ, z‬ﻤﺘﺠﻬﺎﺕ ﻭﺍﺤﺩﺓ ﻋﻠﻰ ﺍﻟﻤﺤﺎﻭﺭ ‪ x,y,z‬ﻋﻠﻰ ﺍﻟﺘﺭﺘﻴﺏ‪.‬‬

‫‪- 181 -‬‬

‫)‪Create PDF files without this message by purchasing novaPDF printer (http://www.novapdf.com‬‬
‫ﻨﺠﺩ ﺒﺈﺴﻘﺎﻁ ﺍﻟﻤﻌﺎﺩﻟﺔ )‪ ،(6.30‬ﻋﻠﻰ ﻤﺤﺎﻭﺭ ﺠﻤﻠﺔ ﻋﻁﺎﻟﻴﺔ ﺃﻥ‪:‬‬
‫‪L1  Iω1‬‬
‫‪(6.32) L 2  Iω 2‬‬
‫‪L 3  Iω3‬‬
‫‪‬‬
‫ﻴﺠﺏ ﺃﻥ ﺘﻜﺎﻓﺊ ﻫﺫﻩ ﺍﻟﻤﺭﻜﺒﺎﺕ ﻤﺭﻜﺒﺎﺕ ﺍﻟﻌﺯﻡ ﺍﻟﺯﺍﻭﻱ ‪ L‬ﻓﻲ ﺍﻟﻤﻌﺎﺩﻟﺔ‬
‫)‪ ،(6.7‬ﺃﻱ ﺃﻥ‪:‬‬
‫‪L1  I11ω1  I12 ω 2  I13ω 3‬‬
‫‪(6.33) L 2  I 21ω1  I 22 ω 2  I 23 ω 3‬‬
‫‪L 3  I 31ω1  I 32 ω 2  I 33 ω3‬‬
‫ﻭﻨﺤﺼل ﻤﻥ ﺍﻟﻤﻁﺎﺒﻘﺔ ﺒﻴﻥ ﺍﻟﻤﻌﺎﺩﻟﺘﻴﻥ )‪ (6.32‬ﻭ)‪ ،(6.33‬ﻋﻠﻰ‪:‬‬
‫‪I11  I ω1  I12 ω 2  I13 ω3  0‬‬
‫‪(6.34) I 21ω1  I 22  Iω 2  I 23ω 3  0‬‬
‫‪I 31ω1  I 32 ω 2  I 33  I ω 3  0‬‬
‫ﻫﺫﺍ ﻭﻴﻤﻜﻥ ﻜﺘﺎﺒﺔ ﺍﻟﻤﻌﺎﺩﻻﺕ )‪،(6.34‬ﻋﻠﻰ ﻫﻴﺌﺔ ﻤﻌﺎﺩﻟﺔ ﻤﺨﺘﺼﺭﺓ ﻭﺍﺤﺩﺓ‬
‫ﺒﺎﻟﺼﻭﺭﺓ‪:‬‬
‫‪3‬‬
‫‪(6.35)  I ij ω j  Iω i‬‬ ‫‪i , j  x, y , z‬‬
‫‪j1‬‬

‫ﺃﻭ ﺒﻤﻌﺎﺩﻟﺔ ﻤﺘﺠﻬﺔ ﺒﺎﻟﺸﻜل‪:‬‬


‫‪‬‬ ‫‪‬‬
‫‪(6.36) IIω  Iω‬‬
‫ﺤﻴﺙ ﺘﻌﻁﻰ ﺍﻟﻤﺼﻔﻭﻓﺔ ‪ II‬ﺒﺎﻟﻌﻼﻗﺔ )‪ ،(6.10‬ﻭﺘﻤﺜل ﺍﻟﻤﻌﺎﺩﻻﺕ )‪،(6.34‬‬
‫‪‬‬
‫ﻋﻨﺩﻤﺎ ‪ ω  0‬ﺜﻼﺙ ﻤﻌﺎﺩﻻﺕ ﺠﺒﺭﻴﺔ ﻤﺘﺠﺎﻨﺴﺔ ﻓﻴﻬﺎ ﺜﻼﺜﺔ ﻤﺘﺤﻭﻻﺕ ‪، ω1 , ω 2 , ω 3‬‬
‫ﻭﻟﻜﻲ ﻴﻜﻭﻥ ﻟﻬﺎ ﺤل ﻻ ﻴﺴﺎﻭﻱ ﺍﻟﺼﻔﺭ‪ ،‬ﻴﺠﺏ ﺃﻥ ﻴﻜﻭﻥ ﻤﺤﺩﺩ ﺍﻷﻤﺜﺎل ﻓﻴﻬﺎ ﻤﺴﺎﻭﻴﺎﹰ‬
‫ﻟﻠﺼﻔﺭ‪ ،‬ﺃﻱ ﺃﻥ‪:‬‬
‫‪I11  I‬‬ ‫‪I12‬‬ ‫‪I13‬‬
‫‪(6.37) I 21‬‬ ‫‪I 22  I‬‬ ‫‪I 23  0‬‬
‫‪I 31‬‬ ‫‪I 32‬‬ ‫‪I 33  I‬‬
‫ﻨﺤﺼل ﻤﻥ ﺤل ﻫﺫﺍ ﺍﻟﻤﺤﺩﺩ ﻋﻠﻰ ﺍﻟﻤﻌﺎﺩﻟﺔ ﺍﻟﺘﺎﻟﻴﺔ‪:‬‬
‫‪(6.38) I3  aI 2  bI  c  0‬‬

‫‪- 182 -‬‬

‫)‪Create PDF files without this message by purchasing novaPDF printer (http://www.novapdf.com‬‬
‫ﻭﻫﻲ ﻤﻌﺎﺩﻟﺔ ﻤﻥ ﺍﻟﺩﺭﺠﺔ ﺍﻟﺜﺎﻟﺜﺔ ﺒﺎﻟﻨﺴﺒﺔ ﺇﻟﻰ ‪ I‬ﻭﺃﻥ ‪ a،b،c‬ﻫﻲ ﻤﻀﺎﺭﻴﺏ‬
‫ﻟﻌﻨﺎﺼﺭ ﺘﻨﺴﻭﺭ ﺍﻟﻌﻁﺎﻟﺔ ‪ . Iij‬ﻴﺘﺒﻴﻥ ﻤﻥ ﺍﻟﻤﻌﺎﺩﻟﺔ )‪ ،(6.38‬ﺃﻨﻪ ﺘﻭﺠﺩ ﺜﻼﺜﺔ ﺤﻠﻭل‬
‫ﺤﻘﻴﻘﻴﺔ ﻟـ ‪ I‬ﻫﻲ ﺠﺫﻭﺭ ﻫﺫﻩ ﺍﻟﻤﻌﺎﺩﻟﺔ‪ ،‬ﻟﺘﻜﻥ ‪. I3 , I 2 , I1‬‬
‫ﺒﺘﻌﻭﻴﺽ ﻜل ﻗﻴﻤﺔ ﻟـ ‪ I‬ﻓﻲ ﺍﻟﻤﻌﺎﺩﻻﺕ )‪ ،(6.34‬ﻨﺤﺼل ﻋﻠﻰ ﻗﻴﻡ‬
‫‪ ω1 , ω 2 , ω 3‬ﺒﺸﺭﻁ ﺃﻥ ﻨﺨﺘﺎﺭ ﺇﺤﺩﺍﻫﺎ ﺒﺸﻜل ﺍﺨﺘﻴﺎﺭﻱ‪ .‬ﺘﻭﺠﺩ ﺇﺫﻥ ﻤﺤﺎﻭﺭ ﺭﺌﻴﺴﻴﺔ‬
‫‪‬‬ ‫‪‬‬ ‫‪‬‬
‫‪ D3 , D 2 , D1‬ﺘﻜﻭﻥ ﻤﺘﺠﻬﺎﺕ ﺍﻟﺩﻭﺭﺍﻥ ﺤﻭﻟﻬﺎ ﻫﻲ‪ ، ω3 , ω2 , ω1 :‬ﻭﺘﻌﻁﻰ‬
‫ﺒﺎﻟﻤﻌﺎﺩﻻﺕ ﺍﻟﺘﺎﻟﻴﺔ‪:‬‬
‫‪‬‬ ‫‪‬‬ ‫‪‬‬ ‫‪‬‬
‫‪ω1  ω11 i  ω21 j  ω31k‬‬
‫‪‬‬ ‫‪‬‬ ‫‪‬‬ ‫‪‬‬
‫‪(6.39) ω2  ω21 i  ω22 j  ω23k‬‬
‫‪‬‬ ‫‪‬‬ ‫‪‬‬ ‫‪‬‬
‫‪ω3  ω31 i  ω32 j  ω33k‬‬
‫‪‬‬ ‫‪‬‬ ‫‪‬‬
‫ﺘﻜﻭﻥ ﺍﻟﻤﺤﺎﻭﺭ ‪ D3 , D 2 , D1‬ﺃﻭ ‪ ω3 , ω2 , ω1‬ﻤﺘﻌﺎﻤﺩﺓ ﻓﻴﻤﺎ ﺒﻴﻨﻬﺎ‪ .‬ﻭﺼﻔﺔ‬
‫ﺍﻟﺘﻌﺎﻤﺩ ﻨﺎﺘﺠﺔ ﻤﻥ ﻜﻭﻥ ﻫﺫﻩ ﺍﻟﻤﺘﺠﻬﺎﺕ ﺍﻟﺜﻼﺙ ﺤﻠﻭﻻﹰ ﻟﻤﺠﻤﻭﻋﺔ ﻭﺍﺤﺩﺓ ﻤﻥ‬
‫ﺍﻟﻤﻌﺎﺩﻻﺕ )‪.(6.34‬‬
‫ﺘﺨﺘﺎﺭ ﻓﻲ ﻤﻌﻅﻡ ﺍﻟﺘﻁﺒﻴﻘﺎﺕ ﻤﺤﺎﻭﺭ ﺍﻟﻌﻁﺎﻟﺔ ﺍﻟﺭﺌﻴﺴﻴﺔ ﻜﻤﺤﺎﻭﺭ ﺘﻨﺎﻅﺭ ﻟﻠﺠﺴﻡ‬
‫ﺍﻟﺼﻠﺏ‪ ،‬ﻓﺈﺫﺍ ﺘﺴﺎﻭﻯ ﺍﺜﻨﺎﻥ ﻤﻥ ﺍﻟﻌﺯﻭﻡ ﺍﻟﺭﺌﻴﺴﻴﺔ ﻓﺈﻥ ﺍﻟﺠﺴﻡ ﻴﺩﻭﺭ ﺒﺸﻜل ﻤﺘﻨﺎﻅﺭ‬
‫ﺤﻭل ﺍﻟﻤﺤﻭﺭ ﺍﻟﺜﺎﻟﺙ ﻓﻲ ﺍﻟﻤﺴﺘﻭﻯ‪.‬‬
‫ﻭﺇﺫﺍ ﺘﺴﺎﻭﺕ ﺍﻟﻌﺯﻭﻡ ﺍﻟﺭﺌﻴﺴﻴﺔ ﺍﻟﺜﻼﺜﺔ ﻓﺈ ﻥ ﺍﻟﺠﺴﻡ ﻴﻜﻭﻥ ﻤﺘﻤﺎﺜل ﺍﻟﺩﻭﺭﺍﻥ‬
‫ﺤﻭل ﺍﻟﻤﺤﺎﻭﺭ‪.‬‬
‫ﺘﻌﺘﺒﺭ ﺨﺎﺼﺔ ﺍﻟﺘﻨﺎﻅﺭ ﺫﺍﺕ ﺃﻫﻤﻴﺔ ﻜﺒﺭﻯ ﻓﻲ ﺍﻟﺤﺼﻭل ﻋﻠﻰ ﻤﻌﺎﺩﻻﺕ ﺤﺭﻜﺔ‬
‫ﺍﻟﺠﺴﻡ ﻜﻤﺎ ﺴﻨﺭﻯ ﻻﺤﻘﺎﹰ‪.‬‬
‫‪- 47‬ﺍﻷﺭﺽ ﻛﺪﻭﺍﻣﺔ ﺣﺮﺓ ﻣﺘﻨﺎﻇﺮﺓ‪:‬‬
‫‪The Earth as a Free Symmetric Top:‬‬
‫ﻟﻘﺩ ﻭﺠﺩﻨﺎ ﻋﻨﺩ ﺩﺭﺍﺴﺔ ﺍﻟﺤﺭﻜﺔ ﻓﻲ ﺍﻟﻤﺠﺎل ﺍﻟﻤﺭﻜﺯﻱ ﺃﻥ ﺤﺭﻜﺔ ﺍﻷﺭﺽ‬
‫ﺘﺘﻜﻭﻥ ﻤﻥ ﺤﺭﻜﺘﻴﻥ ﺭﺌﻴﺴﺘﻴﻥ‪ ،‬ﺍﻷﻭﻟﻰ ﺤﺭﻜﺔ ﻤﺭﻜﺯ ﺍﻷﺭﺽ ﺤﻭل ﺍﻟﺸﻤﺱ ﻭﺍﻟﺜﺎﻨﻴﺔ‬
‫ﺤﺭﻜﺘﻬﺎ ﺍﻟﺩﻭﺭﺍﻨﻴﺔ ﺤﻭل ﻤﺤﻭﺭ ﻴﻤﺭ ﻤﻥ ﻤﺭﻜﺯ ﻜﺘﻠﺘﻬﺎ‪ .‬ﺒﻔﺭﺽ ﺃﻥ ﻟﻸﺭﺽ ﺸﻜﻼﹰ‬
‫ﻜﺭﻭﻴﺎﹰ‪ .‬ﻭﺒﻔﺭﺽ ﺃﻥ ﺍﻟﻘﻭﺓ ﺍﻟﻤﺅﺜﺭﺓ ﻋﻠﻴﻬﺎ ﻫﻲ ﻗﻭﺓ ﺠﺫﺏ ﺍﻟﺸﻤﺱ ﻭﺍﻟﻘﻤﺭ‪ ،‬ﺇﻥ ﻫﺫﻩ‬

‫‪- 183 -‬‬

‫)‪Create PDF files without this message by purchasing novaPDF printer (http://www.novapdf.com‬‬
‫ﺍﻟﻘﻭﻯ ﺘﻤﺭ ﻤﻥ ﻤﺭﻜﺯ ﺍﻷﺭﺽ‪ ،‬ﻟﺫﻟﻙ ﻓﺎﻥ ﻋﺯﻭﻡ ﺍﻟﻘﻭﻯ ﺍﻟﺨﺎﺭﺠﻴﺔ ﺍﻟﻤﺅﺜﺭﺓ ﻋﻠﻰ‬
‫‪‬‬
‫ﺍﻷﺭﺽ ﻴﺴﺎﻭﻱ ﺍﻟﺼﻔﺭ‪ ،‬ﺃﻱ ﺃﻥ ‪ . M ext  0‬ﺒﻤﺎ ﺃﻨﻨﺎ ﺍﻓﺘﺭﻀﻨﺎ ﺃﻥ ﻟﻸﺭﺽ ﺸﻜﻼﹰ‬
‫ﻜﺭﻭﻴﺎﹰ ﻓﻬﻲ ﻤﺘﻨﺎﻅﺭﺓ ﺒﺎﻟﻨﺴﺒﺔ ﻟﻤﺤﻭﺭﻫﺎ‪ ،‬ﻭﺒﺎﻟﺘﺎﻟﻲ ﺘﺘﺴﺎﻭﻯ ﻋﺯﻭﻡ ﺍﻟﻌﻁﺎﻟﺔ ﺤﻭل‬
‫ﻤﺤﻭﺭﻴﻥ ﺭﺌﻴﺴﻴﻴﻥ ﻓﻲ ﺍﻟﻤﺴﺘﻭﻯ ﺍﻻﺴﺘﻭﺍﺌﻲ ﻟﻠﻜﺭﺓ‪ ،‬ﺃﻱ ﺃﻥ‪:‬‬
‫‪(6.40) I1  I2  I‬‬
‫ﺃﻤﺎ ﺍﻟﻤﺤﻭﺭ ﺍﻟﺭﺌﻴﺴﻲ ﺍﻟﺜﺎﻟﺙ ﻓﻴﻜﻭﻥ ﻋﻠﻰ ﺍﻤﺘﺩﺍﺩ ﺍﻟﻤﺤﻭﺭ ﺍﻟﻘﻁﺒﻲ ﺍﻟﺘﻨﺎﻅﺭﻱ‬
‫ﻟﻸﺭﺽ‪ .‬ﻴﻤﻜﻥ ﺇﻴﺠﺎﺩ ﻤﻌﺎﺩﻻﺕ ﺤﺭﻜﺔ ﺍﻷﺭﺽ ﻤﻥ ﻤﺠﻤﻭﻋﺔ ﻤﻌﺎﺩﻻﺕ ﺃﻭﻟﺭ‬
‫)‪ ،(6.26‬ﺤﻴﺙ ﻨﺤﺼل ﺒﻌﺩ ﺍﻟﺘﻌﻭﻴﺽ ﻋﻥ ﻋﺯﻡ ﺍﻟﻘﻭﻯ ﺍﻟﺨﺎﺭﺠﻴﺔ ﺒﺎﻟﺼﻔﺭ ﻋﻠﻰ‪:‬‬
‫‪I3  I‬‬
‫‪ω 1 ‬‬ ‫‪ω 2ω3  0‬‬
‫‪I‬‬
‫‪I I‬‬
‫‪(6.41) ω 2  3 ω1ω 3  0‬‬
‫‪I‬‬
‫‪ω3  0‬‬
‫ﻴﺩﻋﻰ ﺃﻱ ﺠﺴﻡ ﺼﻠﺏ ﺘﺤﻘﻕ ﺤﺭﻜﺘﻪ ﻤﺠﻤﻭﻋﺔ ﺍﻟﻤﻌﺎﺩﻻﺕ )‪ ،(6.41‬ﺩﻭﺍﻤﺔ‬
‫ﺤﺭﺓ ﻤﺘﻨﺎﻅﺭﺓ ﻤﺤﻭﺭﻴﺎﹰ‪ ،‬ﻭﻴﻤﻜﻥ ﺤل ﻫﺫﻩ ﺍﻟﻤﺠﻤﻭﻋﺔ ﺒﺩﻗﺔ‪ ،‬ﺤﻴﺙ ﻴﻨﺘﺞ ﻤﻥ ﺍﻟﻤﻌﺎﺩﻟﺔ‬
‫ﺍﻟﺜﺎﻟﺜﺔ ﻓﻲ ﺍﻟﻤﻌﺎﺩﻻﺕ )‪ ،(6.41‬ﺃﻥ ‪ ω3‬ﺘﺴﺎﻭﻱ ﻤﻘﺩﺍﺭﺍﹰ ﺜﺎﺒﺘﺎﹰ‪.‬‬
‫‪(6.42) ω 3  0  ω3 t   ω3 0  ω 3‬‬

‫ﻴﻌﻁﻰ ﺤل ﺍﻟﻤﻌﺎﺩﻻﺕ )‪ ،(6.41‬ﺒﺎﻟﺼﻭﺭﺓ‪:‬‬


‫‪ω1 t   acosΩt  α ‬‬
‫)‪(6.43‬‬
‫‪ω 2 t   asin Ωt  α ‬‬
‫ﺤﻴﺙ‪:‬‬
‫‪ I3  I ‬‬
‫‪(6.44) Ω  ω3 ‬‬ ‫‪‬‬
‫‪ I ‬‬
‫ﻭﺘﻌﻁﻰ ﻗﻴﻤﺔ ﺍﻟﺴﺭﻋﺔ ﺍﻟﺯﺍﻭﻴﺔ ﺒﺎﻟﻌﻼﻗﺔ‪:‬‬
‫‪(6.45) ω  ω12  ω 22  ω32  a 2  ω23‬‬
‫ﻴﺘﻀﺢ ﻤﻥ ﺍﻟﻤﻌﺎﺩﻟﺘﻴﻥ ﻓﻲ )‪ ،(6.43‬ﺃﻥ ﺍﻟﻤﺭﻜﺒﺘﻴﻥ ‪ ω1‬ﻭ ‪ ω2‬ﺘﺭﺴﻤﺎﻥ ﺩﺍﺌﺭﺓ‬
‫ﺨﺎﺭﺠﻴﺔ ﻨﺼﻑ ﻗﻁﺭﻫﺎ ‪ .a‬ﺒﻴﻨﻤﺎ ‪ ω3‬ﻭ ‪ ω‬ﺘﺒﻘﻴﺎﻥ ﺜﺎﺒﺘﺘﻴﻥ ﺒﺎﻟﻨﺴﺒﺔ ﺇﻟﻰ ﻤﺭﺍﻗﺏ ﻋﻠﻰ‬

‫‪- 184 -‬‬

‫)‪Create PDF files without this message by purchasing novaPDF printer (http://www.novapdf.com‬‬
‫ﺍﻷﺭﺽ ﻴﻨﻅﺭ ﺇﻟﻰ ﻤﺘﺠﻪ ﺍﻟﺴﺭﻋﺔ ﺍﻟﺯﺍﻭﻴﺔ ﺍﻟﺫﻱ ﻴﺩﻭﺭ ﺤﻭل ﻤﺤﻭﺭ ﺍﻟﺘﻨﺎﻅﺭ ﺒﺴﺭﻋﺔ‬
‫ﺯﺍﻭﻴﺔ ‪ Ω‬ﻜﻤﺎ ﻴﺘﻀﺢ ﻤﻥ ﺍﻟﺸﻜل )‪.(6.3‬‬
‫‪‬‬
‫ﺃﻤﺎ ﺩﻭﺭ ﺤﺭﻜﺔ ﺍﻟﻤﺒﺎﺩﺭﺓ ﻟﻠﻤﺘﺠﻪ ‪ ω‬ﺤﻭل ﻤﺤﻭﺭ ﺘﻨﺎﻅﺭ ﺍﻷﺭﺽ ﻓﻴﺴﺎﻭﻱ‪:‬‬
‫‪2π  I  2π‬‬
‫‪(6.46) τ ‬‬ ‫‪‬‬ ‫‪‬‬
‫‪Ω  I3  I  ω3‬‬

‫ﺍﻟﺸﻜل )‪ :(6.3‬ﺩﻭﺭﺍﻥ ﺍﻷﺭﺽ ﺤﻭل ﻤﺤﻭﺭ ﺘﻨﺎﻅﺭﻫﺎ‬

‫ﻭﻴﻜﻭﻥ ﺒﺎﻟﻨﺴﺒﺔ ﻟﻸﺭﺽ ‪ ،  1day‬ﻭﻴﺘﻌﻴﻥ ﺩﻭﺭ ﺤﺭﻜﺔ ﺍﻟﻤﺒﺎﺩﺭﺓ ﺒﺎﻷﻴﺎﻡ‬


‫‪2π‬‬
‫‪ω3‬‬
‫ﻤﻥ ﺤﺴﺎﺏ ﻨﺴﺒﺔ ﻋﺯﻡ ﺍﻟﻌﻁﺎﻟﺔ‪ .‬ﻓﺈﺫﺍ ﺍﻋﺘﺒﺭﺕ ﺍﻷﺭﺽ ﺸﺒﻪ ﻜﺭﻭﻴﺔ ﻭﺫﺍﺕ ﻜﺜﺎﻓﺔ‬
‫ﻤﻨﺘﻅﻤﺔ‪ ،‬ﻓﺈﻥ ﻗﻴﻤﺔ ﺍﻟﻨﺴﺒﺔ ﺘﺤﺴﺏ ﻤﻥ ﻗﻴﺎﺱ ﻨﺼﻑ ﻗﻁﺭ ﺍﻷﺭﺽ‪ ،‬ﻭﻴﻜﻭﻥ‪:‬‬
‫‪I‬‬
‫)‪(6.47‬‬ ‫‪ 300‬‬
‫‪I3  I‬‬
‫ﺒﺎﻟﺭﻏﻡ ﻤﻥ ﺃﻥ ﻜﺜﺎﻓﺔ ﺍﻷﺭﺽ ﺘﺯﺩﺍﺩ ﺍﺒﺘﺩﺍﺀ‪ ‬ﻤﻥ ﺍﻟﻤﺭﻜﺯ‪ ،‬ﻓﺎﻨﻪ ﻻ ﻴﻤﻜﻥ ﺘﻘﺩﻴﺭ‬
‫ﺘﻐﻴﺭ ﻨﺴﺒﺔ ﻋﺯﻡ ﺍﻟﻌﻁﺎﻟﺔ ﻓﻲ ﻫﺫﻩ ﺍﻟﺤﺎﻟﺔ ﻤﻥ ﺤﺎﻟﺔ ﺍﻟﺸﻜل ﺍﻟﻤﻨﺘﻅﻡ ﺍﻟﻜﺜﺎﻓﺔ‪ .‬ﻭﻫﻜﺫﺍ‬
‫ﻓﺈﻥ ﺍﻟﺩﻭﺭ ﺍﻟﻤﺘﻭﻗﻊ ﻟﺤﺭﻜﺔ ﺍﻟﻤﺒﺎﺩﺭﺓ ﻴﻜﻭﻥ ﺤﻭﺍﻟﻲ ‪ 300‬ﻴﻭﻡ‪.‬‬
‫ﻭﻴﻤﻜﻥ ﺘﻌﻴﻴﻥ ﺍﺘﺠﺎﻩ ﻤﺤﻭﺭ ﺩﻭﺭﺍﻥ ﺍﻷﺭﺽ )ﺍﺘﺠﺎﻩ ﺍﻟﻤﺘﺠﻪ ‪ ( ω‬ﺒﺎﻟﺘﺠﺭﺒﺔ‪،‬‬
‫ﻭﻗﺩ ﻭﺠﺩ ﺒﺎﻟﻤﺭﺍﻗﺒﺔ ﺃﻥ ﺩﻭﺭ ﺤﺭﻜﺔ ﻤﺒﺎﺩﺭﺓ ﺍﻷﺭﺽ ﺤﻭل ﻤﺤﻭﺭ ﺘﻨﺎﻅﺭﻫﺎ ﺤﻭﺍﻟﻲ‬

‫‪- 185 -‬‬

‫)‪Create PDF files without this message by purchasing novaPDF printer (http://www.novapdf.com‬‬
‫‪ 440‬ﻴﻭﻤﺎﹰ‪ ،‬ﻭﺃﻥ ﺍﻟﺯﺍﻭﻴﺔ ﺒﻴﻥ ﺍﻟﻤﺘﺠﻪ ‪ ω‬ﻭﻤﺤﻭﺭ ﺘﻨﺎﻅﺭ ﺍﻷﺭﺽ ﺼﻐﻴﺭﺓ ﺠﺩﺍﹰ‪،‬‬
‫ﺒﺤﻴﺙ ﻻ ﻴﺘﺤﺭﻙ ﺍﻟﻤﺘﺠﻪ ‪ ω‬ﻋﻨﺩ ﺍﻟﻘﻁﺏ ﺍﻟﺸﻤﺎﻟﻲ ﺒﻌﻴﺩﺍﹰ ﻋﻥ ﻤﺤﻭﺭ ﺍﻟﺘﻨﺎﻅﺭ ﺃﻜﺜﺭ‬
‫ﻤﻥ ‪ 10‬ﻤﺘﺭ‪ ،‬ﻭﺘﻜﻭﻥ ﺤﺭﻜﺔ ﺍﻟﻤﺘﺠﻪ ‪ ω‬ﻋﺎﺩﺓ ﻏﻴﺭ ﻤﻨﺘﻅﻤﺔ ﻭﺫﻟﻙ ﺒﺴﺒﺏ ﺍﻟﺯﻻﺯل‬
‫ﻭﺍﻟﺘﻐﻴﺭﺍﺕ ﺍﻟﻤﻭﺴﻤﻴﺔ ﺍﻟﺘﻲ ﺘﺤﺩﺙ‪.‬‬
‫ﻭﺒﻴﻥ ﺍﻟﻘﻴﻤﺔ‬ ‫ﻭﻴﻌﻠل ﺍﻟﺘﻨﺎﻗﺽ ﺒﻴﻥ ﺍﻟﻘﻴﻤﺔ ﺍﻟﻤﺘﻭﻗﻌﺔ ﻟﻠﺩﻭﺭ ﺒـ ‪ 300‬ﻴﻭﻡ‬
‫ﺍﻟﻤﺤﺴﻭﺒﺔ ﺒﺎﻟﻤﺭﺍﻗﺒﺔ ﺤﻭﺍﻟﻲ ‪ 440‬ﻴﻭﻡ ﺇﻟﻰ ﺃﻥ ﻜﺜﺎﻓﺔ ﺍﻷﺭﺽ ﻏﻴﺭ ﻤﺘﺠﺎﻨﺴﺔ‪ ،‬ﻓﻬﻲ‬
‫ﻟﻴﺴﺕ ﻜﻠﻬﺎ ﺼﻠﺒﺔ ﺒل ﺘﺸﻐل ﺍﻟﺴﻭﺍﺌل ﺠﺯﺀ ﻜﺒﻴﺭ ﺍﹰ ﻤﻥ ﻜﺘﻠﺘﻬﺎ‪.‬‬
‫‪- 48‬ﺣﺮﻛﺔ ﺟﺴﻢ ﺻﻠﺐ ﻣﺘﻨﺎﻇﺮ – ﳐﺮﻭﻁ ﺍﳉﺴﻢ ﻭﳐﺮﻭﻁ ﺍﻟﻔﻀﺎء‪:‬‬
‫‪Motion of symmetric Rigid Body-Body Cone and space Cone:‬‬
‫ﻟﻘﺩ ﺘﻡ ﻓﻲ ﺍﻟﻔﻘﺭﺓ ﺍﻟﺴﺎﺒﻘﺔ ﻭﺼﻑ ﺤﺭﻜﺔ ﺍﻷﺭﺽ ﻜﺩﻭﺍﻤﺔ ﺤﺭﺓ ﻤﺘﻨﺎﻅﺭﺓ ﻤﻥ‬
‫ﻭﺠﻬﺔ ﻨﻅﺭ ﻤﺭﺍﻗﺏ ﻤﻭﺠﻭﺩ ﻓﻲ ﺠﻤﻠﺔ ﺩﻭﺍﺭﺓ‪ .‬ﻭﺴﻨﺭﻜﺯ ﺍﻟﺩﺭﺍﺴﺔ ﻓﻲ ﻫﺫﻩ ﺍﻟﻔﻘﺭﺓ‬
‫ﻋﻠﻰ ﺤﺭﻜﺔ ﺠﺴﻡ ﺼﻠﺏ ﻤﺘﻨﺎﻅﺭ ﺒﺎﻟﻨﺴﺒﺔ ﻟﻤﺤﻭﺭ ﻤﻨﻪ‪ ،‬ﻨﺩﺭﺱ ﻜﻤﺜﺎل ﻋﻠﻰ ﺫﻟﻙ‬
‫ﺤﺭﻜﺔ ﺩﻭﺍﻤﺔ ﺤﺭﺓ ﻤﺘﻨﺎﻅﺭﺓ ﻜﻤﺎ ﺘﻅﻬﺭ ﻤﻥ ﻗﺒل ﻤﺭﺍﻗﺏ ﺨﺎﺭﺠﻲ ﻤﻭﺠﻭﺩ ﻓﻲ ﺠﻤﻠﺔ‬
‫ﻋﻁﺎﻟﻴﺔ‪.‬‬
‫‪‬‬
‫ﻴﻌﻁﻰ ﻓﻲ ﺤﺎﻟﺔ ﺍﻟﺩﻭﺍﻤﺔ ﺍﻟﻤﺘﻨﺎﻅﺭﺓ ﻤﺘﺠﻪ ﺍﻟﻌﺯﻡ ﺍﻟﺯﺍﻭﻱ ‪ L‬ﻭﻤﺘﺠﻪ ﺍﻟﺴﺭﻋﺔ‬
‫‪‬‬
‫ﺍﻟﺯﺍﻭﻴﺔ ‪ ) ω‬ﻤﺘﺠﻪ ﺍﻟﺩﻭﺭﺍﻥ ( ﺒﺩﻻﻟﺔ ﺍﻟﻤﺭﻜﺒﺎﺕ ﻓﻲ ﺠﻤﻠﺔ ﺍﻟﻤﺤﺎﻭﺭ ﺍﻷﺴﺎﺴﻴﺔ‬
‫) ‪ (x,y,z‬ﻓﻲ ﺍﻟﺩﻭﺍﻤﺔ ﺒﺎﻟﺼﻭﺭﺓ‪:‬‬
‫‪‬‬
‫̂‪L  Iω 1 xˆ  ω 2 yˆ   I 3ω 3 z‬‬
‫‪(6.48) ‬‬
‫‪‬‬ ‫‪‬‬
‫ˆ‪ω  ω 1xˆ  ω 2 yˆ  ω 3 z‬‬

‫ﺤﻴﺙ ̂‪ x̂, ŷ, z‬ﻤﺘﺠﻬﺎﺕ ﻭﺍﺤﺩﺓ ﻋﻠﻰ ﻤﺤﺎﻭﺭ ﺍﻟﺠﻤﻠﺔ ﺍﻟﻌﻁﺎﻟﻴﺔ ) ‪،(x,y,z‬‬
‫‪‬‬
‫ﻭﻴﻌﺘﺒﺭ ﺍﻟﻤﺤﻭﺭ ‪ z‬ﻤﺤﻭﺭ ﺘﻨﺎﻅﺭ ﺍﻟﺩﻭﺍﻤﺔ ﻭ ̂‪ L‬ﻤﺘﺠﻪ ﺍﻟﻭﺍﺤﺩﺓ ﻋﻠﻰ ﺍﻟﻤﺘﺠﻪ ‪. L‬‬
‫ﻨﺤﺴﺏ ̂‪ ω1x̂  ω 2 y‬ﻤﻥ ﺍﻟﻤﻌﺎﺩﻟﺔ ﺍﻷﻭﻟﻰ ﻓﻲ )‪ ،(6.48‬ﻭﻨﻌﻭﺽ ﻓﻲ ﺍﻟﻤﻌﺎﺩﻟﺔ‬
‫‪‬‬ ‫‪‬‬
‫ﺍﻟﺜﺎﻨﻴﺔ‪ ،‬ﻨﺤﺼل ﻋﻠﻰ ﻤﺘﺠﻪ ﺍﻟﺴﺭﻋﺔ ﺍﻟﺯﺍﻭﻴﺔ ‪ ω‬ﺒﺩﻻﻟﺔ ﺍﻟﻤﺘﺠﻬﻴﻥ ‪ L‬ﻭ ̂‪: z‬‬
‫‪‬‬ ‫‪L‬‬
‫‪(6.49) ω ‬‬ ‫̂‪L̂  z‬‬
‫‪I‬‬
‫ﺤﻴﺙ‪:‬‬

‫‪- 186 -‬‬

‫)‪Create PDF files without this message by purchasing novaPDF printer (http://www.novapdf.com‬‬
‫‪I  I‬‬
‫‪ 3‬‬ ‫‪ω 3‬‬
‫‪ I ‬‬
‫‪ ‬‬
‫ﻴﺘﺒﻴﻥ ﻤﻥ ﺍﻟﻤﻌﺎﺩﻟﺔ )‪ ،(6.49‬ﺃﻥ ﺍﻟﻌﻼﻗﺔ ﺒﻴﻥ ﺍﻟﻤﺘﺠﻬﺎﺕ ‪ ẑ, L, ω‬ﻫﻲ ﻋﻼﻗﺔ‬
‫ﺨﻁﻴﺔ‪ ،‬ﻭﺒﺎﻟﺘﺎﻟﻲ ﻓﻬﺫﻩ ﺍﻟﻤﺘﺠﻬﺎﺕ ﻴﺠﺏ ﺃﻥ ﺘﻘﻊ ﻓﻲ ﻤﺴﺘﻭﻯ ﻭﺍﺤﺩ‪ ،‬ﻭﻴﻨﺘﺞ ﻜﺫﻟﻙ ﻤﻥ‬
‫‪‬‬
‫ﺍﻨﻌﺩﺍﻡ ﻋﺯﻭﻡ ﺍﻟﻘﻭﻯ ﺍﻟﺨﺎﺭﺠﻴﺔ ﻋﻠﻰ ﺍﻟﺩﻭﺍﻤﺔ ﺃﻥ ﺍﻟﻤﺘﺠﻪ ‪ L‬ﺜﺎﺒﺕ ﻓﻲ ﺠﻤﻠﺔ‬
‫‪‬‬
‫ﺍﻹﺤﺩﺍﺜﻴﺎﺕ ﺍﻟﻌﻁﺎﻟﻴﺔ‪ ،‬ﻭﺒﺎﻟﺘﺎﻟﻲ ﻓﺈﻥ ﻤﺴﺘﻭﻯ ﺍﻟﻤﺘﺠﻬﻴﻥ ̂‪ ω, z‬ﻴﺩﻭﺭ )ﺤﺭﻜﺔ ﻤﺒﺎﺩﺭﺓ(‬
‫‪‬‬
‫ﺤﻭل ﺍﻟﻤﺤﻭﺭ ‪ . L‬ﻴﻤﻜﻥ ﺤل ﺍﻟﻤﻌﺎﺩﻟﺔ )‪ ،(6.49‬ﺒﺎﻟﻨﺴﺒﺔ ﻟﺤﺭﻜﺔ ﺍﻟﺩﻭﺍﻤﺔ ﻜﻤﺎ‬
‫ﻴﺸﺎﻫﺩﻫﺎ ﻤﺭﺍﻗﺏ ﻓﻲ ﺠﻤﻠﺔ ﺇﺤﺩﺍﺜﻴﺎﺕ ﻋﻁﺎﻟﻴﺔ ﻭﺫﻟﻙ ﺒﺈﻴﺠﺎﺩ ﻤﺭﻜﺒﺘﻴﻥ ﻟﻠﺴﺭﻋﺔ‬
‫‪‬‬
‫ﺍﻟﺯﺍﻭﻴﺔ ﻫﻤﺎ ‪ ωL‬ﻋﻠﻰ ﺍﻤﺘﺩﺍﺩ ﺍﻟﻤﺘﺠﻪ ‪ L‬ﻭ ‪ ω3‬ﻋﻠﻰ ﺍﻤﺘﺩﺍﺩ ﺍﻟﻤﺘﺠﻪ ̂‪ z‬ﻏﻴﺭ‬
‫ﺍﻟﻤﺘﻌﺎﻤﺩﻴﻥ ﻭﺫﻟﻙ ﻜﻤﺎ ﻴﻠﻲ‪:‬‬
‫‪L‬‬
‫‪ωL ‬‬
‫)‪(6.50‬‬ ‫‪I‬‬
‫‪ω 3  ‬‬
‫ﺒﻤﺎ ﺃﻥ ̂‪ z‬ﻤﺘﺠﻪ ﻤﺜﺒﺕ ﻓﻲ ﺍﻟﺠﺴﻡ )ﻴﺩﻭﺭ ﻤﻊ ﺍﻟﺠﺴﻡ(‪ .‬ﻨﺤﺼل ﻤﻥ ﺍﻟﻤﻌﺎﺩﻟﺔ‬
‫)‪ ،(6.49‬ﻭﻤﻥ ﺼﺤﺔ ﺍﻟﻌﻼﻗﺎﺕ ﺍﻟﺘﺎﻟﻴﺔ‪:‬‬
‫‪dx̂ ‬‬ ‫‪dŷ ‬‬ ‫‪dẑ ‬‬
‫‪ ω  x̂ ,‬‬ ‫‪ ω  ŷ ,‬‬ ‫̂‪ ω  z‬‬
‫‪dt‬‬ ‫‪dt‬‬ ‫‪dt‬‬
‫ﻋﻠﻰ ﺃﻥ‪:‬‬
‫‪dẑ  ‬‬
‫)‪(6.51‬‬
‫‪dt‬‬
‫‪ ‬‬
‫̂‪ ẑ  ω  ẑ  ωL L̂  z‬‬

‫ﺘﺒﻴﻥ ﻫﺫﻩ ﺍﻟﻤﻌﺎﺩﻟﺔ ﺃﻥ ﻤﺤﻭﺭ ﺍﻟﺘﻨﺎﻅﺭ ̂‪ z‬ﻴﺩﻭﺭ )ﺤﺭﻜﺔ ﻤﺒﺎﺩﺭﺓ( ﺒﺴﺭﻋﺔ‬


‫‪‬‬
‫ﺯﺍﻭﻴﺔ ﺜﺎﺒﺘﺔ ̂‪ ω L L‬ﺤﻭل ﻤﺤﻭﺭ ﺍﻟﻌﻁﺎﻟﺔ ﺍﻟﺜﺎﺒﺕ ‪ . L‬ﻗﺩ ﻴﻁﺭﺡ ﺍﻟﺴﺅﺍل ﺍﻟﺘﺎﻟﻲ‪:‬‬
‫ﻜﻴﻑ ﺘﺒﺩﻭ ﺤﺭﻜﺔ ﺍﻟﺩﻭﺍﻤﺔ ﻟﻤﺭﺍﻗﺏ ﻤﻭﺠﻭﺩ ﻋﻠﻰ ﺴﻁﺤﻬﺎ ﻭﻴﺩﻭﺭ ﻤﻌﻬﺎ ﺒﺴﺭﻋﺔ‬
‫ﺯﺍﻭﻴﺔ ̂‪ ω L L‬؟ ﺘﻜﻭﻥ ﺍﻟﺴﺭﻋﺔ ﺍﻟﺯﺍﻭﻴﺔ ‪  ‬ﻟﻠﺩﻭﺍﻤﺔ ﻜﻤﺎ ﺘﺒﺩﻭ ﻤﻥ ﻗﺒل ﻤﺭﺍﻗﺏ‬
‫ﻤﻭﺠﻭﺩ ﻓﻲ ﻨﻅﺎﻡ ﻴﺩﻭﺭ ﺒﺴﺭﻋﺔ ﺯﺍﻭﻴﺔ ̂‪ ω L L‬ﻤﺴﺎﻭﻴﺔ ﺇﻟﻰ‪:‬‬
‫‪‬‬ ‫‪‬‬
‫̂‪ω  ω  ωL L̂  Ωz‬‬
‫ﺃﻱ ﺃﻥ ﺤﺭﻜﺔ ﺍﻟﺩﻭﺍﻤﺔ ﻜﻤﺎ ﺘﺒﺩﻭ ﻤﻥ ﺠﻤﻠﺔ ﻤﺜﺒﺘﺔ ﻓﻴﻬﺎ ﻫﻲ ﺤﺭﻜﺔ ﺩﻭﺭﺍﻨﻴﺔ‬
‫ﺤﻭل ﻤﺤﻭﺭ ﺍﻟﺘﻨﺎﻅﺭ ‪ z‬ﺒﻤﻌﺩل ﺴﺭﻋﺔ ﺯﺍﻭﻴﺔ ‪ ،-Ω‬ﻭﻫﻭ ﻤﻌﺩل ﺩﻭﺭﺍﻥ ﺍﻟﺩﻭﺍﻤﺔ‬

‫‪- 187 -‬‬

‫)‪Create PDF files without this message by purchasing novaPDF printer (http://www.novapdf.com‬‬
‫ﺒﺎﻟﻨﺴﺒﺔ ﺇﻟﻰ ‪) ω‬ﻤﺘﺠﻪ ﻤﺜﺒﺕ ﻓﻲ ﺠﻤﻠﺔ ﺍﻟﻤﺒﺎﺩﺭﺓ(‪ ،‬ﻨﺴﺘﻨﺘﺞ ﺃﻥ ‪ +Ω‬ﻤﻌﺩل ﺴﺭﻋﺔ‬
‫‪‬‬
‫ﺩﻭﺭﺍﻥ ﺍﻟﻤﺘﺠﻪ ‪ ω‬ﺒﺎﻟﻨﺴﺒﺔ ﺇﻟﻰ ﺍﻟﺠﺴﻡ‪ ،‬ﻭﻗﻴﻤﺘﻬﺎ ﺘﻌﻁﻰ ﻤﻥ ﺍﻟﻤﻌﺎﺩﻟﺔ )‪،(6.44‬‬
‫ﺍﻟﻤﺴﺘﻨﺘﺠﺔ ﻤﻥ ﻤﻌﺎﺩﻻﺕ ﺃﻭﻟﺭ‪.‬‬
‫‪‬‬ ‫‪‬‬
‫ﻴﺼﻨﻊ ﻤﺘﺠﻪ ﺘﻨﺎﻅﺭ ﺍﻟﺩﻭﺍﻤﺔ ̂‪ z‬ﺃﺜﻨﺎﺀ ﺩﻭﺭﺍﻨﻬﺎ ﻤﻊ ﺍﻟﻤﺘﺠﻪ ‪ L‬ﻭﻤﻊ ﺍﻟﻤﺘﺠﻪ ‪ω‬‬
‫ﺯﻭﺍﻴﺎ ﺘﺒﻘﻰ ﺜﺎﺒﺘﺔ‪ ،‬ﻜﻤﺎ ﻴﺘﻀﺢ ﻤﻥ ﺍﻟﻤﻌﺎﺩﻟﺘﻴﻥ )‪ ،(6.51) ،(6.49‬ﺃﻭ ﻤﻥ ﻗﺎﻨﻭﻥ‬
‫ﺤﻔﻅ ﺍﻟﻁﺎﻗﺔ ﻭﺍﻟﻌﺯﻡ ﺍﻟﺯﺍﻭﻱ‪ .‬ﺒﻤﺎ ﺃﻥ ﻋﺯﻡ ﺍﻟﻘﻭﻯ ﺍﻟﺨﺎﺭﺠﻴﺔ ﻴﺴﺎﻭﻱ ﺼﻔﺭﺍﹰ ﻜﻤﺎ‬
‫‪‬‬
‫ﺫﻜﺭﻨﺎ ﺴﺎﺒﻘﺎﹰ‪ ،‬ﻓﺈﻥ ﻜلٍ ﻤﻥ ﺍﻟﻌﺯﻡ ﺍﻟﺯﺍﻭﻱ ‪ L‬ﻭﻁﺎﻗﺔ ﺍﻟﺤﺭﻜﺔ ﺍﻟﺯﺍﻭﻴﺔ ‪ T‬ﻴﻜﻭﻥ ﺜﺎﺒﺘﺎﹰ‪.‬‬
‫‪‬‬
‫ﻴﻤﻜﻨﻨﺎ ﺒﺎﻻﺴﺘﻔﺎﺩﺓ ﻤﻥ ﺍﻟﻤﻌﺎﺩﻟﺔ )‪ ،(6.48‬ﺃﻥ ﻨﻜﺘﺏ ‪ L‬ﺒﺎﻟﺸﻜل ﺍﻟﺘﺎﻟﻲ‪:‬‬
‫‪‬‬
‫̂‪(6.52) L  Iωn n̂  I3ω3z‬‬
‫ﺤﻴﺙ‪:‬‬
‫̂‪(6.53) ω n n̂  ω1x̂  ω2 y‬‬
‫ﺒﻤﺎ ﺃﻥ ̂‪ ẑ , n‬ﻤﺘﻌﺎﻤﺩﺍﻥ‪ ،‬ﻓﺈﻨﻨﺎ ﻨﺴﺘﻁﻴﻊ ﺃﻥ ﻨﺤﺼل ﺒﺩﻻﻟﺔ ﺍﻟﻤﺭﻜﺒﺘﻴﻥ ‪ ωn‬ﻭ‬
‫‪ ω3‬ﻋﻠﻰ‪:‬‬
‫‪L2  I 2ω2n  I23ω23‬‬
‫)‪(6.54‬‬ ‫‪ ‬‬
‫‪2T  L.ω  Iω2n  I3ω32‬‬
‫ﺤﻴﺙ ‪ T‬ﻁﺎﻗﺔ ﺍﻟﺤﺭﻜﺔ ﺍﻟﺩﻭﺭﺍﻨﻴﺔ‪.‬‬
‫ﻴﻨﺘﺞ ﻤﻥ ﻜﻭﻥ ﻜلٍ ﻤﻥ ‪ L2‬ﻭ ‪ T‬ﻴﺴﺎﻭﻱ ﻤﻘﺩﺍﺭﺍﹰ ﺜﺎﺒﺘﺎﹰ‪ ،‬ﺃﻥ ‪ ω n , ω3‬ﻤﻘﺩﺍﺭﻴﻥ‬
‫ﺜﺎﺒﺘﻴﻥ‪ ،‬ﻭﻨﺠﺩ ﻤﻥ ﺍﻟﻤﻌﺎﺩﻟﺔ )‪ ،(6.48‬ﺃﻥ ﻗﻴﻤﺔ ﺍﻟﺴﺭﻋﺔ ﺍﻟﺯﺍﻭﻴﺔ ﺘﺴﺎﻭﻱ ﺇﻟﻰ‪:‬‬
‫‪ω  ω2n  ω23‬‬
‫‪‬‬
‫ﻨﺴﺘﻨﺘﺞ ﺃﻥ ﻤﺘﺠﻪ ﺍﻟﺩﻭﺭﺍﻥ ‪ ω‬ﻴﺭﺴﻡ ﻤﺨﺭﻭﻁﺎﹰ ﺩﻭﺭﺍﻨﻴﺎﹰ ﺤﻭل ﺍﻟﻤﺤﻭﺭ ‪oz‬‬
‫ﺒﺴﺭﻋﺔ ﺯﺍﻭﻴﺔ ﺜﺎﺒﺘﺔ ‪ ،Ω‬ﻭﻫﺫﺍ ﺍﻟﻤﺨﺭﻭﻁ ﺜﺎﺒﺕ ﻓﻲ ﺠﻤﻠﺔ ﺍﻹﺤﺩﺍﺜﻴﺎﺕ ﺍﻟﻤﺜﺒﺘﺔ ﻓﻲ‬
‫ﺍﻟﺠﺴﻡ‪ ،‬ﻟﺫﻟﻙ ﻴﺴﻤﻰ ﻤﺨﺭﻭﻁ ﺍﻟﺠﺴﻡ ﻜﻤﺎ ﻫﻭ ﻤﺒﻴﻥ ﻋﻠﻰ ﺍﻟﺸﻜل )‪ ،(6.5‬ﻭﻗﺩ ﻭﺠﺩﻨﺎ‬
‫‪‬‬
‫ﺴﺎﺒﻘﺎﹰ ﺃﻥ ﺍﻟﻌﺯﻡ ﺍﻟﺯﺍﻭﻱ ‪ L‬ﺜﺎﺒﺕ ﻓﻲ ﺍﻟﻔﻀﺎﺀ ﻭﺃﻥ ﻨﺎﺘﺞ ﺍﻟﻀﺭﺏ ﺍﻟﻘﻴﺎﺴﻲ )ﺍﻟﻌﺩﺩﻱ(‬
‫‪‬‬ ‫‪‬‬ ‫‪ ‬‬
‫ﻟﻠﻤﺘﺠﻬﻴﻥ ‪ ω.L‬ﺜﺎﺒﺕ ﺃﻴﻀﺎ‪ ،‬ﻫﺫﺍ ﻴﻌﻨﻲ ﺃﻥ ﻤﺴﻘﻁ ﺍﻟﻤﺘﺠﻪ ‪ ω‬ﻋﻠﻰ ﺍﻟﻤﺘﺠﻪ ‪ L‬ﺜﺎﺒﺕ‬
‫‪‬‬ ‫‪‬‬
‫ﺩﻭﻤﺄً‪ ،‬ﻭﻟﻤﺎ ﻜﺎﻨﺕ ﻗﻴﻤﺔ ﻤﺘﺠﻪ ﺍﻟﺴﺭﻋﺔ ﺍﻟﺯﺍﻭﻴﺔ ‪ ω‬ﺜﺎﺒﺘﺔ‪ ،‬ﻓﺈﻨﻨﺎ ﻨﺴﺘﻨﺘﺞ ﺃﻥ ﺍﻟﻤﺘﺠﻪ ‪ω‬‬

‫‪- 188 -‬‬

‫)‪Create PDF files without this message by purchasing novaPDF printer (http://www.novapdf.com‬‬
‫‪‬‬
‫ﻴﺭﺴﻡ ﻤﺨﺭﻭﻁﺎﹰ ﺜﺎﺒﺘﺎﹰ ﻓﻲ ﺍﻟﻔﻀﺎﺀ ﻤﺤﻭﺭﻩ ‪ . L‬ﻴﺴﻤﻰ ﺍﻟﻤﺨﺭﻭﻁ ﺍﻟﺜﺎﺒﺕ ﻓﻲ ﺍﻟﻔﻀﺎﺀ‬
‫)ﻤﺨﺭﻭﻁ ﺍﻟﻔﻀﺎﺀ(‪.‬‬

‫ﺍﻟﺸﻜل)‪ (6.4‬ﻤﺭﻜﺒﺎﺕ ﺍﻟﺴﺭﻋﺔ ﺍﻟﺯﺍﻭﻴﺔ ﻭﺍﻟﻌﺯﻡ ﺍﻟﺯﺍﻭﻱ ﻋﻠﻰ ﻤﺤﻭﺭ ﺍﻟﺘﻨﺎﻅﺭ ̂‪ z‬ﻟﻠﺩﻭﺍﻤﺔ ﻭﻋﻠﻰ‬
‫‪ ‬‬
‫ﺍﻟﻤﺤﻭﺭ ̂‪ n‬ﺍﻟﻌﻤﻭﺩ ﻋﻠﻰ ̂‪ z‬ﻓﻲ ﻤﺴﺘﻭﻯ ﺍﻟﻤﺘﺠﻬﻴﻥ ‪L,‬‬

‫ﻨﺴﺘﻨﺘﺞ ﻤﻥ ﻜل ﻤﺎ ﺘﻘﺩﻡ ﺍﻟﻨﺘﻴﺠﺔ ﺍﻟﺘﺎﻟﻴﺔ‪ :‬ﻴﺘﺩﺤﺭﺝ ﻤﺨﺭﻭﻁ ﺍﻟﺠﺴﻡ ﺍﻟﺫﻱ‬


‫‪‬‬
‫ﻴﺭﺴﻤﻪ ﺍﻟﻤﺘﺠﻪ ‪ ω‬ﻓﻲ ﺍﻟﺠﻤﻠﺔ ﺍﻹﺤﺩﺍﺜﻴﺔ ﺍﻟﻤﺘﻤﺎﺴﻜﺔ ﻤﻊ ﺍﻟﺠﺴﻡ ﺃﺜﻨﺎﺀ ﺍﻟﺤﺭﻜﺔ ﻋﻠﻰ‬
‫‪‬‬ ‫‪‬‬
‫ﻤﺨﺭﻭﻁ ﺍﻟﻔﻀﺎﺀ ﺍﻟﺫﻱ ﻴﺭﺴﻤﻪ ﺍﻟﻤﺘﺠﻪ ‪ ω‬ﺤﻭل ﺍﻟﻤﺘﺠﻪ ‪ ، L‬ﻭﺒﻌﺒﺎﺭﺓ ﺃﺨﺭﻯ‪ ،‬ﻴﻤﻜﻥ‬
‫ﺍﻟﻘﻭل ﺃﻥ ﺍﻟﺤﺭﻜﺔ ﺘﺘﻌﻴﻥ ﺒﺩﺤﺭﺠﺔ ﻤﺨﺭﻭﻁ ﺍﻟﺠﺴﻡ ﻋﻠﻰ ﻤﺨﺭﻭﻁ ﺍﻟﻔﻀﺎﺀ‪ ،‬ﻟﺫﻟﻙ‬
‫ﻴﺴﻤﻰ ﻤﺨﺭﻭﻁ ﺍﻟﺠﺴﻡ ﺒﺎﻟﻤﺘﺩﺤﺭﺝ ﻭﻤﺨﺭﻭﻁ ﺍﻟﻔﻀﺎﺀ ﺒﺎﻟﻘﺎﻋﺩﺓ‪ ،‬ﻭﻟﻤﻌﺭﻓﺔ ﻭﻀﻊ‬
‫ﺍﻟﻤﺨﺭﻭﻁﻴﻥ ﺍﻟﺠﺴﻤﻲ ﻭﺍﻟﻔﻀﺎﺌﻲ ﺒﺎﻟﻨﺴﺒﺔ ﻟﺒﻌﻀﻬﻤﺎ‪ ،‬ﻨﺭﻤﺯ ﻟﻠﺯﺍﻭﻴﺔ ﺒﻴﻥ ﺍﻟﻤﺤﻭﺭ ̂‪z‬‬
‫‪‬‬ ‫‪‬‬
‫ﻭﺍﻟﻤﺘﺠﻪ ‪ L‬ﺒﺎﻟﺭﻤﺯ ‪ θ‬ﻭﻟﻠﺯﺍﻭﻴﺔ ﺒﻴﻥ ̂‪ z‬ﻭ ‪ ω‬ﺒﺎﻟﺭﻤﺯ ‪ ، α‬ﻜﻤﺎ ﻫﻭ ﻤﻭﻀﺢ ﻋﻠﻰ‬
‫ﺍﻟﺸﻜل )‪ ،(6.4‬ﻭﺘﺘﻌﻴﻥ ﻫﺎﺘﺎﻥ ﺍﻟﺯﺍﻭﻴﺘﺎﻥ ﺒﺎﻟﻌﻼﻗﺘﻴﻥ ﺍﻟﺘﺎﻟﻴﺘﻴﻥ‪:‬‬
‫‪ωn‬‬
‫‪tanα ‬‬
‫‪ω3‬‬
‫)‪(6.55‬‬
‫‪Ln Iω n‬‬
‫‪tanθ ‬‬ ‫‪‬‬
‫‪L3 I3ω3‬‬

‫‪- 189 -‬‬

‫)‪Create PDF files without this message by purchasing novaPDF printer (http://www.novapdf.com‬‬
‫ﻨﺤﺼل ﻤﻥ ﻨﺴﺏ ﺍﻟﻤﻌﺎﺩﻟﺔ ﺍﻟﺜﺎﻨﻴﺔ ﺇﻟﻰ ﺍﻷﻭﻟﻰ ﻋﻠﻰ‪:‬‬
‫‪tanθ I‬‬
‫)‪(6.56‬‬ ‫‪‬‬
‫‪tanα I3‬‬
‫ﻓﺈﺫﺍ ﺇﺫﺍ ﻜﺎﻥ ‪ I3  I‬ﻓﺈﻥ ﺍﻟﺯﺍﻭﻴﺔ ‪ α‬ﺘﻜﻭﻥ ﺃﺼﻐﺭ ﻤﻥ ﺍﻟﺯﺍﻭﻴﺔ ‪ ،θ‬ﺘﻭﺍﻓﻕ ﻫﺫﻩ‬
‫ﺍﻟﺤﺎﻟﺔ ﺤﺎﻟﺔ ﻗﻤﺔ ﻤﺘﻁﺎﻭﻟﺔ‪ ،‬ﻭﻴﻘﻊ ﻓﻲ ﻫﺫﻩ ﺍﻟﺤﺎﻟﺔ ﻤﺨﺭﻭﻁ ﺍﻟﻔﻀﺎﺀ ﺨﺎﺭﺝ ﻤﺨﺭﻭﻁ‬
‫ﺍﻟﺠﺴﻡ‪ ،‬ﻭﻴﻘﺎل ﻓﻲ ﻫﺫﻩ ﺍﻟﺤﺎﻟﺔ ﺃﻥ ﻤﺨﺭﻭﻁ ﺍﻟﺠﺴﻡ ﻴﺘﺩﺤﺭﺝ ﻤﻥ ﺍﻟﺨﺎﺭﺝ ﻋﻠﻰ‬
‫ﻤﺨﺭﻭﻁ ﺍﻟﻔﻀﺎﺀ ﺍﻟﺸﻜل )‪.(6.5a‬‬
‫ﺃﻤﺎ ﺇﺫﺍ ﻜﺎﻥ ‪ I3  I‬ﻓﺈﻥ ﺍﻟﺯﺍﻭﻴﺔ ‪ α‬ﺘﻜﻭﻥ ﺃﻜﺒﺭ ﻤﻥ ﺍﻟﺯﺍﻭﻴﺔ ‪ ، θ‬ﺘﻭﺍﻓﻕ ﻫﺫﻩ‬
‫ﺍﻟﺤﺎﻟﺔ ﺤﺎﻟﺔ ﻗﻤﺔ ﻤﻔﻠﻁﺤﺔ‪ ،‬ﻭﻴﻘﻊ ﻓﻲ ﻫﺫﻩ ﺍﻟﺤﺎﻟﺔ ﻤﺨﺭﻭﻁ ﺍﻟﻔﻀﺎﺀ ﺩﺍﺨل ﻤﺨﺭﻭﻁ‬
‫ﺍﻟﺠﺴﻡ‪ ،‬ﻭﻴﻘﺎل ﻋﻨﺩﺌﺫ‪ ‬ﺃﻥ ﻤﺨﺭﻭﻁ ﺍﻟﺠﺴﻡ ﻴﺘﺩﺤﺭﺝ ﻤﻥ ﺍﻟﺩﺍﺨل ﻋﻠﻰ ﻤﺨﺭﻭﻁ ﺍﻟﻔﻀﺎﺀ‬
‫ﺍﻟﺸﻜل )‪،(6.5b‬‬

‫‪a‬‬ ‫‪b‬‬
‫اﻟﺸﻜﻞ )‪ :(6.5‬ﻣﺨﺮوط اﻟﻔﻀﺎء وﻣﺨﺮوط اﻟﺠﺴﻢ‬
‫‪ - 49‬ﺍﳊﺮﻛﺔ ﺍﳌﻐﺰﻟﻴﺔ ﻟﺪﻭﺍﻣﺔ ﺛﻘﻴﻠﺔ ﻣﺘﻨﺎﻇﺮﺓ‪:‬‬
‫‪The Heavy symmetric Top:‬‬
‫ﺩﺭﺴﻨﺎ ﺇﻟﻰ ﺍﻵﻥ ﺍﻟﺤﺭﻜﺔ ﺍﻟﺩﻭﺭﺍﻨﻴﺔ ﻟﺠﺴﻡ ﺼﻠﺏ ﻓﻲ ﺍﻟﻔﺭﺍﻍ ﺒﺩﻻﻟﺔ ﻤﺭﻜﺒﺎﺕ‬
‫‪‬‬
‫ﻤﺘﺠﻪ ﺍﻟﺩﻭﺭﺍﻥ ‪ ω‬ﻋﻠﻰ ﻤﺤﺎﻭﺭ ﺠﻤﻠﺔ ﺇﺤﺩﺍﺜﻴﺎﺕ ﻤﺘﻤﺎﺴﻜﺔ ﻤﻊ ﺍﻟﺠﺴﻡ ﺍﻟﺩﺍﺌﺭ‪ .‬ﻏﻴﺭ‬

‫‪- 190 -‬‬

‫)‪Create PDF files without this message by purchasing novaPDF printer (http://www.novapdf.com‬‬
‫ﺃﻥ ﻫﺫﻩ ﺍﻟﺩﺭﺍﺴﺔ ﻻ ﺘﺼﻑ ﺍﻟﺤﺭﻜﺔ ﺍﻟﺩﻭﺭﺍﻨﻴﺔ ﺒﺸﻜلٍ ﺠﻴﺩ ﻭﺴﻠﺱ‪ ،‬ﻟﺫﻟﻙ ﻨﺴﺘﺨﺩﻡ‬
‫ﻟﺘﺤﻠﻴل ﺍﻟﺤﺭﻜﺔ ﺍﻟﺩﻭﺭﺍﻨﻴﺔ ﻟﺩﻭﺍﻤﺔ ﻤﺜﻼﹰ ﺜﻼﺜﺔ ﺇﺤﺩﺍﺜﻴﺎﺕ ﺯﺍﻭﻴﺔ ﻤﻨﺎﺴﺒﺔ ﻫﻲ‬
‫‪ ،  , θ , ψ‬ﺘﺴﻤﻰ ﺯﻭﺍﻴﺎ ﺃﻭﻟﺭ ﻭﻫﺫﻩ ﺍﻟﺯﻭﺍﻴﺎ ﻤﺒﻴﻨﺔ ﻋﻠﻰ ﺍﻟﺸﻜل )‪.(6.6‬‬
‫ﻨﻌﺘﺒﺭ ﺃﻥ ﺠﻤﻠﺔ ﺍﻹﺤﺩﺍﺜﻴﺎﺕ ‪ ، x1y1z1‬ﺠﻤﻠﺔ ﻋﻁﺎﻟﻴﺔ )ﻤﺜﺒﺘﺔ( ﻤﺒﺩﺅﻫﺎ ﻓﻲ‬
‫ﻨﻘﻁﺔ ﺘﻤﺎﺱ ﺍﻟﺩﻭﺍﻤﺔ ﻤﻊ ﺍﻟﺴﻁﺢ ﺍﻟﺫﻱ ﺘﺘﺤﺭﻙ ﻋﻠﻴﻪ‪ ،‬ﻨﺤﺼل ﻋﻠﻰ ﺠﻤﻠﺔ ﺍﻹﺤﺩﺍﺜﻴﺎﺕ‬
‫‪ xyz‬ﻤﻥ ﺩﻭﺭﺍﻥ ﺍﻟﺩﻭﺍﻤﺔ ﺒﺯﺍﻭﻴﺔ ‪ ‬ﺤﻭل ﺍﻟﻤﺤﻭﺭ ‪ z 1‬ﻤﺘﺒﻭﻋﺎﹰ ﺒﺩﻭﺭﺍﻥ ﺒﺯﺍﻭﻴﺔ ‪θ‬‬
‫ﺤﻭل ﺍﻟﻤﺤﻭﺭ‪.x‬‬

‫ﺍﻟﺸﻜل )‪ :(6.6‬ﺯﻭﺍﻴﺎ ﺃﻭﻟﺭ‬

‫ﻭﻴﺴﻤﻰ ﺍﻟﻤﺤﻭﺭ ‪ x‬ﺨﻁ ﺍﻟﻌﻘﺩ‪ .‬ﻭﻴﺘﻡ ﺍﻟﺤﺼﻭل ﻋﻠﻰ ﻨﻅﺎﻡ ﺍﻹﺤﺩﺍﺜﻴﺎﺕ '‪xy z‬‬
‫ﻤﻥ ﺍﻟﺩﻭﺭﺍﻥ ﺒﺯﺍﻭﻴﺔ ‪ ψ‬ﺤﻭل ﺍﻟﻤﺤﻭﺭ ‪ .z‬ﺘﻭﻀﺢ ﺍﻟﺯﻭﺍﻴﺎ ‪  , θ , ψ‬ﻨﻅﺎﻡ‬
‫ﺍﻹﺤﺩﺍﺜﻴﺎﺕ '‪ xy z‬ﺒﺎﻟﻨﺴﺒﺔ ﺇﻟﻰ ﻨﻅﺎﻡ ﺍﻹﺤﺩﺍﺜﻴﺎﺕ ‪. x1y1z1‬‬
‫ﺘﻌﺘﺒﺭ ﺩﺭﺍﺴﺔ ﺤﺭﻜﺔ ﺍﻟﺩﻭﺍﻤﺔ )ﺍﻟﺒﻠﺒل( ﺍﻟﺘﻲ ﻴﺴﺘﺨﺩﻤﻬﺎ ﺍﻷﻁﻔﺎل ﻓﻲ ﺃﻟﻌﺎﺒﻬﻡ‬
‫ﺃﺤﺩ ﺃﻫﻡ ﺘﻁﺒﻴﻘﺎﺕ ﺩﺭﺍﺴﺔ ﺤﺭﻜﺔ ﺠﺴﻡ ﺼﻠﺏ ﻓﻲ ﺍﻟﻔﻀﺎﺀ ﻟﻤﺎ ﻟﺫﻟﻙ ﻤﻥ ﺃﻫﻤﻴﺔ ﻓﻲ‬
‫ﻓﻴﺯﻴﺎﺀ ﺍﻟﻔﻠﻙ ﻭﺍﻟﻔﻴﺯﻴﺎﺀ ﺍﻟﻨﻭﻭﻴﺔ‪ ،‬ﻭﺴﻨﻨﺎﻗﺵ ﻓﻲ ﻫﺫﻩ ﺍﻟﻔﻘﺭﺓ ﺤﺭﻜﺔ ﺍﻟﺩﻭﺍﻤﺔ ﺍﻟﻤﺘﻨﺎﻅﺭﺓ‬
‫ﻓﻲ ﻤﺠﺎل ﺍﻟﺠﺎﺫﺒﻴﺔ ﺍﻷﺭﻀﻴﺔ ﻜﺤﺎﻟﺔ ﺨﺎﺼﺔ‪.‬‬

‫‪- 191 -‬‬

‫)‪Create PDF files without this message by purchasing novaPDF printer (http://www.novapdf.com‬‬
‫ﻨﻌﺘﺒﺭ ﻨﻘﻁﺔ ﻤﻥ ﺍﻟﺩﻭﺍﻤﺔ ﻋﻠﻰ ﺘﻤﺎﺱ ﻤﻊ ﺍﻟﺴﻁﺢ ﺍﻟﺫﻱ ﺘﺩﻭﺭ ﻋﻠﻴﻪ‪،‬‬
‫ﻭﺴﻨﺴﺘﺨﺩﻡ ﻓﻲ ﺘﺤﻠﻴل ﺩﺭﺍﺴﺔ ﺍﻟﺤﺭﻜﺔ ﺍﻟﺩﻭﺭﺍﻨﻴﺔ ﻜﻤﺎ ﺫﻜﺭﻨﺎ ﺯﻭﺍﻴﺎ ﺃﻭﻟﺭ ‪ , θ , ψ‬‬
‫ﻜﺈﺤﺩﺍﺜﻴﺎﺕ ﺯﺍﻭﻴﺔ ﻟﺘﻌﻴﻴﻥ ﺍﻟﺤﺭﻜﺔ ﻜﻤﺎ ﻫﻭ ﻤﺒﻴﻥ ﻋﻠﻰ ﺍﻟﺸﻜل )‪.(6.6‬‬
‫ﻗﺒل ﺩﺭﺍﺴﺔ ﺤﺭﻜﺔ ﺍﻟﺩﻭﺍﻤﺔ ﺭﻴﺎﻀﻴﺎﹰ ﻨﻠﻘﻲ ﻨﻅﺭﺓ ﻋﻠﻰ ﻫﺫﻩ ﺍﻟﻅﺎﻫﺭﺓ ﺍﻟﻔﻴﺯﻴﺎﺌﻴﺔ‪.‬‬
‫ﺘﺒﻴﻥ ﺩﺭﺍﺴﺔ ﺍﻟﺤﺭﻜﺔ ﺍﻟﻌﺎﻤﺔ‪ ،‬ﺃﻥ ﺍﻟﺩﻭﺍﻤﺔ ﺘﺩﻭﺭ ﺤﻭل ﻤﺤﻭﺭﻫﺎ ﺒﺴﺭﻋﺔ ﺯﺍﻭﻴﺔ ﻜﺒﻴﺭﺓ‬
‫ﻴﻁﻠﻕ ﻋﻠﻴﻬﺎ ﺃﺴﻡ ﺤﺭﻜﺔ ﺍﻟﺒﺭﻡ ﺃﻭ ﺍﻻﻟﺘﻔﺎﻑ )‪ .(spin‬ﺒﺎﻹﻀﺎﻓﺔ ﺇﻟﻰ ﺫﻟﻙ ﻓﺎﻥ ﻤﺤﻭﺭ‬
‫ﺘﻨﺎﻅﺭﻫﺎ ﻴﺩﻭﺭ ﺒﺒﻁﺀ ﺤﻭل ﺍﻟﻤﺤﻭﺭ ﺍﻟﺭﺃﺴﻲ ‪ ، z 1‬ﻭﺘﺴﻤﻰ ﻫﺫﻩ ﺍﻟﺤﺭﻜﺔ ﺤﺭﻜﺔ‬
‫ﻤﺒﺎﺩﺭﺓ )‪ .(precession‬ﻋﻼﻭﺓ ﻋﻠﻰ ﺫﻟﻙ ﻓﺎﻥ ﻤﺤﻭﺭ ﺘﻨﺎﻅﺭ ﺍﻟﺩﻭﺍﻤﺔ ‪ z‬ﻴﺒﺘﻌﺩ‬
‫ﻭﻴﻘﺘﺭﺏ ﻤﻥ ﺍﻟﻤﺤﻭﺭ ‪ ، z 1‬ﻭﺘﺴﻤﻰ ﻫﺫﻩ ﺍﻟﺤﺭﻜﺔ ﺒﺤﺭﻜﺔ ﺍﻟﺘﺭﻨﺢ )‪،(nutation‬‬
‫ﻭﺍﻟﺴﺅﺍل ﺍﻟﻤﺴﺘﻐﺭﺏ ﻓﻲ ﺤﺭﻜﺔ ﺍﻟﺩﻭﺍﻤﺔ‪ ،‬ﻫﻭ ﻟﻤﺎﺫﺍ ﻻ ﺘﺴﻘﻁ ﺍﻟﺩﻭﺍﻤﺔ ﻋﻠﻰ ﺍﻷﺭﺽ‬
‫ﺒﺘﺄﺜﻴﺭ ﻗﻭﺓ ﺜﻘﻠﻬﺎ؟ ﻟﻺﺠﺎﺒﺔ ﻋﻥ ﻫﺫﺍ ﺍﻟﺴﺅﺍل ﻻ ﺒﺩ ﻤﻥ ﺩﺭﺍﺴﺔ ﺍﻟﺤﺭﻜﺔ ﺭﻴﺎﻀﻴﺎﹰ‪ ،‬ﻭﻤﻥ‬
‫ﺜﻡ ﺘﻔﺴﻴﺭ ﺍﻟﻨﺘﺎﺌﺞ ﺍﻟﺭﻴﺎﻀﻴﺔ ﻤﻥ ﺍﻟﻨﺎﺤﻴﺔ ﺍﻟﻔﻴﺯﻴﺎﺌﻴﺔ‪.‬‬
‫ﻴﻌﻁﻰ ﻤﺘﺠﻪ ﺍﻟﺴﺭﻋﺔ ﺍﻟﺯﺍﻭﻴﺔ ﻟﺩﻭﺭﺍﻥ ﺍﻟﺩﻭﺍﻤﺔ ﺒﺩﻻﻟﺔ ﺯﻭﺍﻴﺎ ﺃﻭﻟﺭ ﺒﺎﻟﻌﻼﻗﺔ‬
‫ﺍﻟﺘﺎﻟﻴﺔ‪:‬‬

‫‪‬‬
‫̂‪ω   ẑ1  θ x̂  ψ z‬‬ ‫)‪(6.57‬‬
‫ﺤﻴﺙ ̂‪ ẑ1, ẑ, ŷ, x‬ﻤﺘﺠﻬﺎﺕ ﻭﺍﺤﺩﺓ ﻋﻠﻰ ﺍﻟﻤﺤﺎﻭﺭ )‪ (z1,z, y, x‬ﻋﻠﻰ ﺍﻟﺘﺭﺘﻴﺏ‪.‬‬
‫ﻨﺠﺩ ﻤﻥ ﺍﻟﺸﻜل )‪ ،(6.6‬ﺃﻥ‪:‬‬
‫̂‪ẑ 1  cosθ ẑ  sinθ y‬‬ ‫)‪(6.58‬‬
‫ﺒﺎﻟﺘﻌﻭﻴﺽ ﻓﻲ ﺍﻟﻤﻌﺎﺩﻟﺔ )‪ ،(6.57‬ﻨﺤﺼل ﻋﻠﻰ‪:‬‬
‫‪‬‬
‫‪‬‬
‫̂‪ω  θ x̂   sinθ ŷ  ψ   cosθ z‬‬ ‫‪‬‬ ‫)‪(6.59‬‬

‫‪- 192 -‬‬

‫)‪Create PDF files without this message by purchasing novaPDF printer (http://www.novapdf.com‬‬
‫ﺍﻟﺸﻜل )‪ :(6.7‬ﺍﻟﺤﺭﻜﺔ ﺍﻟﺩﻭﺭﺍﻨﻴﺔ ﻟﺩﻭﺍﻤﺔ‬

‫ﺍﻟﻌﻁﺎﻟﺔ‬ ‫ﻴﻨﺘﺞ ﻤﻥ ﺘﻨﺎﻅﺭ ﺍﻟﺩﻭﺍﻤﺔ ﺒﺎﻟﻨﺴﺒﺔ ﻟﻠﻤﺤﻭﺭ ‪ z‬ﺃﻥ ﻋﺯﻭﻡ‬


‫‪ I xx  I yy  I‬ﺘﻜﻭﻥ ﻤﺘﺴﺎﻭﻴﺔ ﺒﺎﻟﻨﺴﺒﺔ ﺇﻟﻰ ﺃﻱ ﻤﺠﻤﻭﻋﺔ ﻤﺤﺎﻭﺭ ﺇﺤﺩﺍﺜﻴﺔ ﻤﺘﻌﺎﻤﺩﺓ‬
‫ﻓﻲ ﺍﻟﻤﺴﺘﻭﻯ ‪ ،xy‬ﻭﺒﺎﻟﺘﺎﻟﻲ ﻓﺎﻥ ﺍﻟﻌﺯﻡ ﺍﻟﺯﺍﻭﻱ ﻟﻠﺩﻭﺍﻤﺔ ﺤﻭل ﻨﻘﻁﺔ ﺍﻷﺼل ﻴﻌﻁﻰ‬
‫ﺒﺎﻟﻌﻼﻗﺔ ﺍﻟﺘﺎﻟﻴﺔ‪:‬‬

‫‪‬‬
‫‪‬‬
‫‪L  I θ x̂  I  sinθ ŷ  I 3 ψ‬‬ ‫‪‬‬
‫̂‪   cosθ z‬‬ ‫)‪(6.60‬‬

‫‪- 193 -‬‬

‫)‪Create PDF files without this message by purchasing novaPDF printer (http://www.novapdf.com‬‬
‫‪‬‬ ‫‪‬‬
‫ﺤﻴﺙ ‪ ، I zz  I 3‬ﺒﺎﻻﺴﺘﻔﺎﺩﺓ ﻤﻥ ﻋﺒﺎﺭﺓ ﺍﻟﺴﺭﻋﺔ ﺍﻟﺯﺍﻭﻴﺔ ‪ ω‬ﻭﺍﻟﻌﺯﻡ ﺍﻟﺯﺍﻭﻱ ‪، L‬‬
‫ﻨﺠﺩ ﺃﻥ ﻁﺎﻗﺔ ﺍﻟﺤﺭﻜﺔ ﺍﻟﺩﻭﺭﺍﻨﻴﺔ ﻟﻠﺩﻭﺍﻤﺔ ﺒﺩﻻﻟﺔ ﺯﻭﺍﻴﺎ ﺃﻭﻟﺭ ﺘﺴﺎﻭﻱ ﺇﻟﻰ‪:‬‬
‫‪1   1   2 2 2  1‬‬
‫‪T‬‬
‫‪2‬‬ ‫‪2 ‬‬
‫‪‬‬
‫‪ω. L  I θ   sin θ   I3 ψ   cosθ‬‬ ‫‪‬‬‫‪2‬‬
‫)‪(6.61‬‬
‫‪ 2‬‬
‫ﻭﺘﻌﻁﻰ ﻁﺎﻗﺔ ﺠﻬﺩ ﺍﻟﺠﺎﺫﺒﻴﺔ ﺍﻷﺭﻀﻴﺔ )ﺍﻟﻁﺎﻗﺔ ﺍﻟﻜﺎﻤﻨﺔ( ﺒﺎﻟﻌﻼﻗﺔ‪:‬‬
‫‪(6.62) V  Mg l cosθ‬‬
‫ﺤﻴﺙ ‪ l‬ﺍﻟﻤﺴﺎﻓﺔ ﻤﻥ ﻨﻘﻁﺔ ﺍﻷﺼل ﺇﻟﻰ ﻤﺭﻜﺯ ﺜﻘل ﺍﻟﺩﻭﺍﻤﺔ‪ ،‬ﻜﻤﺎ ﻫﻭ ﻤﻭﻀﺢ‬
‫ﻋﻠﻰ ﺍﻟﺸﻜل )‪.(6.7‬‬
‫ﺘﻌﻁﻰ ﻋﻨﺩﺌﺫ‪ ‬ﺩﺍﻟﺔ ﻻﻏﺭﺍﻨﺞ ‪ L‬ﺒﺩﻻﻟﺔ ﺍﻟﻤﺘﺤﻭﻻﺕ ‪  , ,,‬ﻭﻻ ﺘﻌﺘﻤﺩ‬
‫ﻋﻠﻰ ‪ ، , ‬ﺒﺎﻟﺼﻭﺭﺓ‪:‬‬
‫‪1   2 ‬‬ ‫‪‬‬
‫‪2  1‬‬
‫‪2 ‬‬ ‫‪ 2‬‬
‫‪‬‬
‫‪L  T  v  I θ   sin θ   I 3 ψ   cosθ‬‬ ‫‪‬‬ ‫)‪(6.63‬‬
‫‪‬‬ ‫‪‬‬
‫‪ Mg cosθ‬‬
‫ﺒﻤﺎ ﺃﻥ ﺩﺍﻟﺔ ﻻﻏﺭﺍﻨﺞ ﻻ ﺘﻌﺘﻤﺩ ﻋﻠﻰ ‪ ‬و ‪ ،‬ﺘﻭﺠﺩ ﺇﺫﻥ ﻤﺭﻜﺒﺘﺎﻥ ﻤﺤﻔﻭﻅﺘﺎﻥ‬
‫ﻟﻠﺩﻓﻊ ﺍﻟﻤﻌﻤﻡ )ﺭﺍﺠﻊ ﺍﻟﻔﺼل ﺍﻟﺜﺎﻨﻲ(‪ ،‬ﺘﻜﻭﻥ ﺍﻟﻤﺭﻜﺒﺔ ﺍﻷﻭﻟﻰ ﻤﺴﺎﻭﻴﺔ ‪:‬‬
‫‪L‬‬
‫‪(6.64) p  ‬‬
‫‪‬‬
‫‪ I3 ‬‬ ‫‪‬‬ ‫‪‬‬
‫‪   cos   I3 3‬‬

‫ﺘﻜﻭﻥ ﺒﺎﻟﺘﺎﻟﻲ ﺍﻟﺴﺭﻋﺔ ﺍﻟﺯﺍﻭﻴﺔ ‪ ω3‬ﺜﺎﺒﺘﺔ ﺒﺎﺘﺠﺎﻩ ﻤﺤﻭﺭ ﺍﻟﺘﻨﺎﻅﺭ ‪ ، z‬ﻭﺘﻌﻁﻰ‬


‫ﺍﻟﻤﺭﻜﺒﺔ ﺍﻟﺜﺎﻨﻴﺔ ﺍﻟﻤﺤﻔﻭﻅﺔ ﻟﻠﺩﻓﻊ ﺍﻟﻤﻌﻤﻡ ﻤﻥ ﺍﻟﻌﻼﻗﺔ‪:‬‬

‫‪L‬‬
‫‪(6.65) p ‬‬ ‫‪ I sin 2   I 3 3 cos‬‬
‫‪‬‬
‫ﺒﻤﺎ ﺃﻥ ﺍﻟﺯﺍﻭﻴﺔ ‪ ‬ﺘﻌﺘﺒﺭ ﺘﻜﺎﻤل ﻟﻠﺤﺭﻜﺔ )ﺜﺎﺒﺕ ﻟﻠﺤﺭﻜﺔ(‪ ،‬ﻴﻨﺘﺞ ﺃﻥ ‪، p Φ  0‬‬
‫ﺍﻷﻤﺭ ﺍﻟﺫﻱ ﻴﺅﺩﻱ ﺇﻟﻰ ﺍﻟﻤﻌﺎﺩﻟﺔ ﺍﻟﺘﺎﻟﻴﺔ‪:‬‬
‫‪(6.66) Isinθ  θ I3ω3  2I ‬‬
‫‪ p θ ‬ﻋﻠﻰ ﻤﻌﺎﺩﻟﺔ ﺍﻟﺤﺭﻜﺔ ﺍﻟﺘﺎﻟﻴﺔ‪:‬‬ ‫ﻭﻨﺤﺼل ﻤﻥ ﻤﻌﺎﺩﻟﺔ ﻻﻏﺭﺍﻨﺞ‬
‫‪L‬‬
‫‪θ‬‬
‫‪‬‬
‫‪(6.67) Iθ  Mgl  I3ω3  I 2cosθ sinθ‬‬
‫‪‬‬ ‫‪‬‬

‫‪- 194 -‬‬

‫)‪Create PDF files without this message by purchasing novaPDF printer (http://www.novapdf.com‬‬
‫ﻴﻭﺍﻓﻕ ﺍﻟﺩﻭﺭﺍﻥ ﺒﺯﺍﻭﻴﺔ ‪ ‬ﺤﺭﻜﺔ ﺍﻟﻤﺒﺎﺩﺭﺓ ﺤﻭل ﺍﻟﻤﺤﻭﺭ ‪ ، z 1‬ﻭﺘﻭﺍﻓﻕ‬
‫ﺘﻐﻴﺭﺍﺕ ﺍﻟﺯﺍﻭﻴﺔ ‪ θ‬ﺤﺭﻜﺔ ﺍﻟﺘﺭﻨﺢ‪ .‬ﻨﺴﺘﺨﺩﻡ ﻤﻌﺎﺩﻟﺔ ﺍﻟﺤﺭﻜﺔ ﺍﻷﺨﻴﺭﺓ ﻟﺘﺤﺩﻴﺩ ﺍﻟﺸﺭﻭﻁ‬
‫ﺍﻟﺘﻲ ﻤﻥ ﺃﺠﻠﻬﺎ ﺘﺘﺤﺭﻙ ﺍﻟﺩﻭﺍﻤﺔ ﺤﺭﻜﺔ ﻤﺒﺎﺩﺭﺓ ﺼﺭﻓﺔ‪ ،‬ﺤﻴﺙ ﺘﻜﻭﻥ ﺍﻟﺯﺍﻭﻴﺔ ‪ θ‬ﺜﺎﺒﺘﺔ‬
‫ﻓﻲ ﺤﺭﻜﺔ ﺍﻟﻤﺒﺎﺩﺭﺓ ﺍﻟﺼﺭﻓﺔ‪ ،‬ﻭﻴﻨﺘﺞ ﻤﻥ ﺫﻟﻙ ﺃﻥ ﺍﻟﻁﺭﻑ ﺍﻷﻴﻤﻥ ﻓﻲ ﺍﻟﻤﻌﺎﺩﻟﺔ‬
‫)‪ ،(6.66‬ﻴﺴﺎﻭﻱ ﺼﻔﺭﺍﹰ‪ .‬ﻨﺠﺩ ﻤﻥ ﺤل ﺍﻟﻤﻌﺎﺩﻟﺔ )‪ .(6.67‬ﻤﻥ ﺃﺠل ‪ ‬ﺒﺩﻻﻟﺔ‬
‫ﺍﻟﺯﺍﻭﻴﺔ ‪ θ‬ﺃﻥ‪:‬‬
‫‪I33 ‬‬ ‫‪4MglIcosθ ‬‬
‫‪(6.68)  ‬‬ ‫‪1‬‬ ‫‪‬‬ ‫‪1‬‬ ‫‪‬‬
‫‪2I cos ‬‬ ‫‪I32ω32‬‬ ‫‪‬‬
‫‪‬‬
‫ﻟﻜﻲ ﻴﻜﻭﻥ ﻟﻬﺫﺍ ﺍﻟﺤل ﻤﻌﻨﻰ ﻓﻴﺯﻴﺎﺌﻲ ﻴﺠﺏ ﺃﻥ ﻴﻜﻭﻥ ﺍﻟﻤﻘﺩﺍﺭ ﺘﺤﺕ ﺍﻟﺠﺫﺭ‬
‫ﻤﻭﺠﺏ‪ ،‬ﺒﻤﺎ ﺃﻥ ‪ cos   0‬ﺃﺜﻨﺎﺀ ﺤﺭﻜﺔ ﺍﻟﺩﻭﺍﻤﺔ ﻋﻠﻰ ﺍﻟﻁﺎﻭﻟﺔ‪ ،‬ﻓﺎﻥ ﺴﺭﻋﺔ‬
‫ﺍﻟﺒﺭﻡ ‪ ، 3‬ﻴﺠﺏ ﺃﻥ ﺘﺤﻘﻕ ﺍﻟﻤﺘﺭﺍﺠﺤﺔ‪:‬‬
‫‪4Mgl Icosθ‬‬
‫‪ω3 ‬‬ ‫)‪(6.69‬‬
‫‪I32‬‬
‫ﺇﺫﺍ ﻜﺎﻨﺕ ﺴﺭﻋﺔ ﺍﻟﺒﺭﻡ ﺃﻗل ﻤﻥ ﻫﺫﻩ ﺍﻟﻘﻴﻤﺔ ﺍﻟﺼﻐﺭﻯ‪ ،‬ﻓﺈﻥ ﺤﺭﻜﺔ‬
‫ﺍﻟﺩﻭﺍﻤﺔ ﺘﻜﻭﻥ ﺤﺭﻜﺔ ﻤﺒﺎﺩﺭﺓ ﺼﺭﻓﺔ ﻓﻘﻁ‪ ،‬ﻭﻋﻨﺩﻤﺎ ﺘﻜﻭﻥ ﺴﺭﻋﺔ ﺍﻟﺒﺭﻡ ﺃﻜﺒﺭ ﺒﻜﺜﻴﺭ‬
‫ﻤﻥ ﺍﻟﻘﻴﻤﺔ ﺍﻟﺩﻨﻴﺎ ﻟـ ‪ ،  3‬ﻴﻤﻜﻥ ﻓﻲ ﻫﺫﻩ ﺍﻟﺤﺎﻟﺔ ﺒﺘﻘﺭﻴﺏ ﻤﻘﺒﻭل ﻨﺸﺭ ﺍﻟﺠﺫﺭ‬
‫ﺍﻟﺘﺭﺒﻴﻌﻲ ﻓﻲ ﺍﻟﻤﻌﺎﺩﻟﺔ )‪ ،(6.68‬ﻭﺍﻻﻜﺘﻔﺎﺀ ﺒﺎﻟﺤﺩﻴﻥ ﺍﻷﻭﻟﻴﻥ‪ ،‬ﻨﺤﺼل ﻋﻠﻰ ﻗﻴﻤﺘﻴﻥ‬
‫ﺘﻘﺭﻴﺒﻴﺘﻴﻥ ﻟﻠﺠﺫﺭ ‪ ‬ﻴﻭﺍﻓﻘﺎﻥ ﺍﻟﺤﺎﻟﺘﻴﻥ ﺍﻟﺘﺎﻟﻴﺘﻴﻥ‪:‬‬
‫ﺤﺭﻜﺔ ﻤﺒﺎﺩﺭﺓ ﺒﻁﻴﺌﺔ‪:‬‬
‫‪Mgl‬‬
‫‪ ‬‬ ‫‪ ωP‬‬ ‫)‪(6.70‬‬
‫‪I3ω3‬‬
‫ﺤﺭﻜﺔ ﻤﺒﺎﺩﺭﺓ ﺴﺭﻴﻌﺔ‪:‬‬
‫‪Iω‬‬
‫‪  3 3‬‬ ‫)‪(6.71‬‬
‫‪Icosθ‬‬
‫ﺘﺤﻘﻕ ﺤﺭﻜﺔ ﺍﻟﻤﺒﺎﺩﺭﺓ ﺍﻟﺒﻁﻴﺌﺔ ﻓﻲ ﺍﻟﺤل ﺍﻷﻭل ﺍﻟﺸﺭﻁﻴﻥ‪   ω 3 :‬ﻭﻴﻜﻭﻥ‬
‫‪‬‬
‫ﻤﺘﺠﻪ ﺍﻟﻌﺯﻡ ﺍﻟﺯﺍﻭﻱ ‪ L‬ﺘﻘﺭﻴﺒﺎﹰ ﻋﻠﻰ ﺍﻤﺘﺩﺍﺩ ﺍﻟﻤﺤﻭﺭ ‪ ،z‬ﻭﻴﺤﻘﻕ ﻓﻲ ﺤﺭﻜﺔ ﺍﻟﻤﺒﺎﺩﺭﺓ‬
‫‪‬‬
‫ﺍﻟﺴﺭﻴﻌﺔ ﺍﻟﺤل ﺍﻟﺜﺎﻨﻲ ﺍﻟﺸﺭﻁﻴﻥ‪   ω 3 :‬ﻭﻴﻜﻭﻥ ﻤﺘﺠﻪ ﺍﻟﻌﺯﻡ ﺍﻟﺯﺍﻭﻱ ‪ L‬ﺘﻘﺭﻴﺒﺎﹰ‬

‫‪- 195 -‬‬

‫)‪Create PDF files without this message by purchasing novaPDF printer (http://www.novapdf.com‬‬
‫ﻋﻠﻰ ﺍﻤﺘﺩﺍﺩ ﺍﻟﻤﺘﺠﻪ ‪ ،z‬ﻭﺘﺘﺤﻘﻕ ﺃﻴﻀﺎﹰ ﻓﻲ ﻫﺫﻩ ﺍﻟﺤﺎﻟﺔ ﺍﻟﻌﻼﻗﺔ ‪ Lcosθ  I 3ω 3‬ﻭ‬
‫‪ ،  ‬ﻭﺘﺴﺎﻭﻱ ﻫﺫﻩ ﺍﻟﻘﻴﻤﺔ ﺍﻟﺘﺭﺩﺩ ﺍﻟﺯﺍﻭﻱ ‪  L‬ﻟﺤﺭﻜﺔ ﺩﻭﺍﻤﺔ ﺤﺭﺓ ﺍﻟﻌﻼﻗﺔ‬
‫‪L‬‬
‫‪I‬‬
‫)‪ ، (6.50‬ﻭﻴﻜﻭﻥ ﺍﻟﺤل ﻓﻲ ﺤﺭﻜﺔ ﺍﻟﻤﺒﺎﺩﺭﺓ ﺍﻟﺴﺭﻴﻌﺔ ﺤﻭل ﻤﺤﻭﺭ ﻋﻤﻭﺩﻱ ﻤﺴﺘﻘل‬
‫ﻋﻥ ﻗﻭﺓ ﺍﻟﺠﺎﺫﺒﻴﺔ ﻓﻲ ﺍﻟﺤﺎﻟﺔ ﺍﻟﺤﺩﻴﺔ ‪. limω 3  ω 3 min‬‬
‫ﻋﻨﺩﻤﺎ ﺘﻜﻭﻥ ﺤﺭﻜﺔ ﺒﺭﻡ ﺍﻟﺩﻭﺍﻤﺔ ﺴﺭﻴﻌﺔ ﻓﺎﻥ ﻜ لٍ ﻤﻥ ﺤﺭﻜﺘﻲ ﺍﻟﻤﺒﺎﺩﺭﺓ‬
‫ﻭﺍﻟﺘﺭﻨﺢ ﺘﻜﻭﻥ ﺒﻁﻴﺌﺔ‪ ،‬ﻭﻴﻤﻜﻥ ﺇﻴﺠﺎﺩ ﺍﻟﺤل ﺍﻟﺘﻘﺭﻴﺒﻲ ﻟﻠﺤﺭﻜﺔ ﻓﻲ ﻫﺫﻩ ﺍﻟﺤﺎﻟﺔ ﺒﺈﻫﻤﺎل‬
‫ﺍﻟﺤﺩ ﺍﻟﺘﺭﺒﻴﻌﻲ ﻟﻠﻤﺸﺘﻕ ﻟﻠﺯﺍﻭﻴﺘﻴﻥ ‪ ‬ﻭ ‪ ‬ﻓﻲ ﺍﻟﻤﻌﺎﺩﻟﺘﻴﻥ ﺍﻟﺘﻔﺎﻀﻠﻴﺘﻴﻥ )‪،(6.66‬‬
‫)‪ ،(6.67‬ﻓﻨﺤﺼل ﻋﻠﻰ‪:‬‬
‫‪I ω‬‬
‫‪ sinθ  3 3 θ‬‬
‫‪I‬‬
‫)‪(6.72‬‬
‫‪θ   Mgl  I 3ω 3   sinθ‬‬
‫‪ I‬‬ ‫‪I‬‬ ‫‪‬‬
‫ﺒﻔﺭﺽ ﺃﻥ‪:‬‬
‫‪I 3 ω3‬‬
‫‪(6.73) ω L ‬‬
‫‪I‬‬
‫ﻴﻤﻜﻥ ﻜﺘﺎﺒﺔ ﺍﻟﻤﻌﺎﺩﻟﺘﻴﻥ ﺍﻟﺴﺎﺒﻘﺘﻴﻥ ﺒﺎﻟﺸﻜل‪:‬‬
‫‪ sinθ  ω L θ‬‬ ‫)‪(6.74‬‬
‫‪θ  ω ω   sinθ‬‬ ‫)‪(6.75‬‬
‫‪L‬‬ ‫‪P‬‬

‫ﻨﻔﺎﻀل ﺍﻟﻤﻌﺎﺩﻟﺔ )‪ ،(6.74‬ﺒﺎﻟﻨﺴﺒﺔ ﻟﻠﺯﻤﻥ ﻭﻨﻌﻭﺽ ﻓﻲ ﺍﻟﻤﻌﺎﺩﻟﺔ )‪،(6.75‬‬


‫ﻨﺤﺼل ﻋﻠﻰ ﺍﻟﻤﻌﺎﺩﻟﺔ ﺍﻵﺘﻴﺔ‪:‬‬
‫‪d 2‬‬
‫‪(6.76) 2  ω2L  ω 2L ω P‬‬
‫‪dt‬‬
‫ﺤﻴﺙ ﺃﻫﻤﻠﻨﺎ ﺍﻟﺤﺩ ‪ ‬ﻟﺼﻐﺭﻩ‪ ،‬ﻭﻴﻌﻁﻰ ﺤل ﺍﻟﻤﻌﺎﺩﻟﺔ ﺍﻟﺘﻔﺎﻀﻠﻴﺔ )‪،(6.76‬‬
‫‪ ‬‬
‫ﺒﺎﻟﻨﺴﺒﺔ ﺇﻟﻰ ‪ ‬ﺒﺎﻟﻌﻼﻗﺔ ﺍﻟﺘﺎﻟﻴﺔ‪:‬‬
‫‪(6.77) t   ω P  acosω L t  α ‬‬
‫ﺒﺄﺨﺫ ﺍﻟﺸﺭﻭﻁ ﺍﻻﺒﺘﺩﺍﺌﻴﺔ ﺍﻟﺘﺎﻟﻴﺔ‪:‬‬
‫‪   0 , θ  0 , θ  θ 0‬ﻋﻨﺩﻤﺎ ‪ ،t=0‬ﻨﺠﺩ ﺃﻥ ‪ ، 0  0‬ﻭ ﻴﺼﺒﺢ ﺍﻟﺤل ﻓﻲ‬
‫)‪ ،(6.76‬ﺒﺎﻟﺸﻜل ﺍﻟﺘﺎﻟﻲ‪:‬‬

‫‪- 196 -‬‬

‫)‪Create PDF files without this message by purchasing novaPDF printer (http://www.novapdf.com‬‬
‫‪(6.78) t   ω P  ω P  ω 0  cosω L t‬‬
‫ﻨﻜﺎﻤل ﻫﺫﻩ ﺍﻟﻤﻌﺎﺩﻟﺔ ﺒﺎﻟﻨﺴﺒﺔ ﻟﻠﺯﻤﻥ‪ ،‬ﻨﺤﺼل ﻋﻠﻰ ﻤﻌﺎﺩﻟﺔ ﺍﻟﺤﺭﻜﺔ ﻤﻥ ﺃﺠل‬
‫ﺍﻟﺯﺍﻭﻴﺔ ‪  t ‬ﺒﺎﻟﺼﻭﺭﺓ‪:‬‬
‫‪ ω P  ω0‬‬ ‫‪‬‬
‫‪(6.79)  t   ω P t  ‬‬ ‫‪ sinω L t‬‬
‫‪ ωL‬‬ ‫‪‬‬
‫ﻭﻹﻴﺠﺎﺩ ﺍﻟﺤل ﺒﺎﻟﻨﺴﺒﺔ ﻟﻠﺯﺍﻭﻴﺔ ‪ ،θ‬ﻨﻌﻭﺽ ﻤﻥ ﺍﻟﻤﻌﺎﺩﻟﺔ )‪ ،(6.78‬ﻓﻲ‬
‫ﺍﻟﻤﻌﺎﺩﻟﺔ )‪ ،(6.74‬ﻓﻨﺤﺼل ﻋﻠﻰ‪:‬‬
‫‪(6.80) θ  ωP  ω0  sinL t sinθ‬‬

‫ﺒﻤﺎ ﺃﻥ ‪ ω P‬و ‪ ω 0‬ﻜﻤﻴﺎﺕ ﺼﻐﻴﺭﺓ‪ ،‬ﻴﻤﻜﻥ ﺒﺘﻘﺭﻴﺏ ﻤﻘﺒﻭل ﺃﻥ ﻨﻀﻊ‬


‫‪ sinθ  sinθ0‬ﻓﻲ ﺍﻟﻁﺭﻑ ﺍﻷﻴﻤﻥ ﻤﻥ ﺍﻟﻤﻌﺎﺩﻟﺔ )‪ ،(6.80‬ﻭ ﺒﺈﺠﺭﺍﺀ ﺍﻟﺘﻜﺎﻤل‪،‬‬
‫ﻨﺤﺼل ﻋﻠﻰ ﺍﻟﺤل ﺒﺎﻟﻨﺴﺒﺔ ﻟﻠﺯﺍﻭﻴﺔ ‪ ،θ‬ﺒﺎﻟﺸﻜل ﺍﻟﺘﺎﻟﻲ‪:‬‬
‫‪ ω  ω0‬‬ ‫‪‬‬
‫‪θ t   θ 0   P‬‬ ‫‪sinθ 0 1  cosω L t ‬‬ ‫)‪(6.81‬‬
‫‪ ωL‬‬ ‫‪‬‬
‫ﺘﻜﻤ‪‬ل ﻫﺫﻩ ﺍﻟﻤﻌﺎﺩﻟﺔ ﺍﻟﺤل ﺍﻷﺴﺎﺴﻲ ﻟﻤﻌﺎﺩﻟﺔ ﺍﻟﺤﺭﻜﺔ ﻓﻲ ﺤﺎﻟﺔ ﺤﺭﻜﺔ ﻤﺒﺎﺩﺭﺓ‬
‫ﺒﻁﻴﺌﺔ ﻭ ﺘﺭﻨﺢ ﺼﻐﻴﺭ‪.‬‬
‫ﻴ‪‬ﻅﻬﹺﺭ ﺤل ﺍﻟﻤﻌﺎﺩﻟﺔ )‪ ،(6.75‬ﻟﺤﺭﻜﺔ ﺘﺭﻨﺢ ﺍﻟﺩﻭﺍﻤﺔ‪ ،‬ﺃﻥ ﻗﻴﻤﺔ ﺍﻟﺯﺍﻭﻴﺔ ‪θ t ‬‬

‫ﺇﺸﺎﺭﺓ‬ ‫ﻭﺘﺤﺩﺩ‬ ‫‪. θ 0  2‬‬


‫‪ω‬‬ ‫‪p‬‬ ‫‪ ω0 ‬‬ ‫‪‬‬
‫‪sinθ 0 , θ 0‬‬ ‫ﺍﻟﺤﺩﻴﻥ‪:‬‬ ‫ﺒﻴﻥ‬
‫‪‬‬ ‫‪ω p ‬‬
‫ﺍﻟﻤﻘﺩﺍﺭ ‪ ،  p  0 ‬ﺍﻟﻘﻴﻤﺔ ﺍﻟﻌﻅﻤﻰ ﻭﺍﻟﻘﻴﻤﺔ ﺍﻟﺼﻐﺭﻯ ﻟﻠﺯﺍﻭﻴﺔ ‪ .θ‬ﻭﺘﺘﻀﻤﻥ ﺤﺭﻜﺔ‬
‫ﺍﻟﻤﺒﺎﺩﺭﺓ ﻓﻲ ﺍﻟﻤﻌﺎﺩﻟﺔ )‪ ،(6.79‬ﻤﺠﻤﻭﻉ ﺤﺭﻜﺘﻴﻥ‪ ،‬ﺤﺭﻜﺔ ﺠﻴﺒﻴﻪ ﺘﺭﺍﻓﻘﻬﺎ ﺤﺭﻜﺔ‬
‫ﺘﺭﻨﺢ ﻤﺭﻜﺒﺔ ﻋﻠﻰ ﻤﺒﺎﺩﺭﺓ ﻤﺴﺘﻘﺭﺓ‪ .‬ﻋﻨﺩﻤﺎ ﻴﻜﻭﻥ ﺍﻟﺘﺭﺩﺩ ﺍﻻﺒﺘﺩﺍﺌﻲ ﻟﺤﺭﻜﺔ ﺍﻟﻤﺒﺎﺩﺭﺓ‬
‫‪ ω 0‬ﻤﺴﺎﻭﻴﺎﹰ ﺇﻟﻰ ‪ ، ω P‬ﻓﺈﻥ ﺤﺭﻜﺔ ﺍﻟﺩﻭﺍﻤﺔ ﺘﻜﻭﻥ ﺤﺭﻜﺔ ﻤﺒﺎﺩﺭﺓ ﻤﺴﺘﻘﺭﺓ ﺒﺩﻭﻥ‬
‫ﺘﺭﻨﺢ‪ .‬ﻴﻭﻀﺢ ﺍﻟﺸﻜل )‪ ،(6.8‬ﻤﻨﺤﻨﻴﺎﺕ ﺘﺭﻨﺢ ﺍﻟﺩﻭﺍﻤﺔ ﺒﻭﺍﺴﻁﺔ ﻤﺤﺎﻭﺭ ﺘﻨﺎﻅﺭﻫﺎ ﻤﻥ‬
‫ﺃﺠل ﻗﻴﻡ ﺍﺒﺘﺩﺍﺌﻴﺔ ﻤﺨﺘﻠﻔﺔ ﻟـ ‪ ω 0‬ﻤﻘﺎﺒﻠﺔ ﻟﻘﻴﻤﺔ ﻭﺍﺤﺩﺓ ﻟﻠﺯﺍﻭﻴﺔ ‪.θ‬‬
‫ﻴﻌﺘﺒﺭ ‪  L‬ﻓﻲ ﺍﻟﻤﻌﺎﺩﻟﺔ )‪ ،(6.81‬ﺘﺭﺩﺩ ﺘﺭﻨﺢ ﺍﻟﺩﻭﺍﻤﺔ‪ .‬ﻴﺘﻀﺢ ﻤﻥ ﺍﻟﻤﻌﺎﺩﻟﺘﻴﻥ‬
‫ﺇﻟﻰ ﺍﺯﺩﻴﺎﺩ ﺘﺭﺩﺩ‬ ‫)‪ (6.70‬ﻭ)‪ ،(6.73‬ﺃﻥ ﺍﺯﺩﻴﺎﺩ ﺘﺭﺩﺩ ﺍﻟﺒﺭﻡ ‪ ω 3‬ﻴﺅﺩﻱ‬

‫‪- 197 -‬‬

‫)‪Create PDF files without this message by purchasing novaPDF printer (http://www.novapdf.com‬‬
‫ﺍﻟﺘﺭﻨﺢ ‪ ،  L‬ﺒﻴﻨﻤﺎ ﻴﻨﻘﺹ ﺘﺭﺩﺩ ﺍﻟﻤﺒﺎﺩﺭﺓ ‪ . ω P‬ﻏﻴﺭ ﺃﻥ ﺴﻌﺔ ﺍﻟﺘﺭﻨﺢ ﺘﺘﻨﺎﺴﺏ ﻋﻜﺴﻴ ﺎﹰ‬
‫ﻤﻊ ﺘﺭﺩﺩ ﺍﻟﺘﺭﻨﺢ ‪ ،  L‬ﻟﺫﻟﻙ ﻓﺄﻥ ﺘﺭﻨﺢ ﺍﻟﺩﻭﺍﻤﺔ ﻻ ﻴﻜﻭﻥ ﻤﻠﺤﻭﻅﺎﹰ ﻋﻨﺩﻤﺎ ﺘﺘﺤﺭﻙ‬
‫ﺒﺴﺭﻋﺔ ﺒﺭﻡ ﻜﺒﻴﺭﺓ‪ .‬ﻋﻨﺩﻤﺎ ﻴﻜﻭﻥ ﺒﺭﻡ )ﻏﺯل( ﺍﻟﺩﻭﺍﻤﺔ ﺴﺭﻴﻌﺎﹰ ﻋﻠﻰ ﺴﻁﺢ ﻤﺠﻭﻑ‬
‫ﻓﺈﻨﻨﺎ ﻨﺴﻤﻊ ﻨﻐﻤﺔ ﺃﺯﻴﺯ )ﻁﻨﻴﻥ( ﺒﺘﺭﺩﺩ ﻤﻁﺎﺒﻕ ﻟﺤﺭﻜﺔ ﺍﻟﺘﺭﻨﺢ‪.‬‬

‫ﺍﻟﺸﻜل )‪ :(6.8‬ﻤﻨﺤﻨﻴﺎﺕ ﺍﻟﺘﺭﻨﺢ ﻤﻥ ﺃﺠل ﻗﻴﻡ ﺍﺒﺘﺩﺍﺌﻴﺔ ﻤﺨﺘﻠﻔﺔ ﻟـ ‪ ω 0‬ﻭﻗﻴﻤﺔ ﻭﺍﺤﺩﺓ ﻟـ ‪θ‬‬

‫ﻴﻤﻜﻥ ﺸﺭﺡ ﻅﺎﻫﺭﺓ ﺘﺭﻨﺢ ﺍﻟﺩﻭﺍﻤﺔ ﺍﻟﺘﻲ ﻋﺭﻀﺕ ﻓﻲ ﺍﻟﻤﻌﺎﺩﻻﺕ ﺍﻟﺴﺎﺒﻘﺔ‬


‫‪‬‬
‫ﺒﻁﺭﻴﻘﺔ ﺒﺴﻴﻁﺔ‪ .‬ﻴﻜﻭﻥ ﻤﺘﺠﻪ ﺍﻟﻌﺯﻡ ﺍﻟﺯﺍﻭﻱ ‪ L‬ﻟﻠﺩﻭﺍﻤﺔ ﻋﻠﻰ ﺍﻤﺘﺩﺍﺩ ﻤﺤﻭﺭ‬

‫‪- 198 -‬‬

‫)‪Create PDF files without this message by purchasing novaPDF printer (http://www.novapdf.com‬‬
‫ﺘﻨﺎﻅﺭﻫﺎ ‪ z‬ﻓﻲ ﺤﺎﻟﺔ ﺤﺭﻜﺔ ﺍﻟﺒﺭﻡ ﺍﻟﺴﺭﻴﻌﺔ‪ ،‬ﻭﻨﺤﺼل ﻤﻥ ﺍﻟﻤﻌﺎﺩﻟﺔ )‪ ،(6.62‬ﻋﻠﻰ‬
‫ﻋﺯﻡ ﻗﻭﺓ ﺍﻟﺠﺎﺫﺒﻴﺔ ﺒﺎﻟﺸﻜل ﺍﻟﺘﺎﻟﻲ‪:‬‬
‫‪‬‬
‫‪dV‬‬
‫‪(6.82) M  ‬‬
‫̂‪θ̂  Mglsinθx‬‬
‫‪dθ‬‬
‫‪‬‬
‫‪ ،z1‬ﻭﻴﺴﺒﺏ ﺩﻭﺭﺍﻥ ﺍﻟﺩﻭﺍﻤﺔ‬ ‫ﻭﻴﻜﻭﻥ ﺍﻟﻌﺯﻡ ‪ M‬ﻋﻤﻭﺩﻴﺎﹰ ﻋﻠﻰ ﺍﻟﻤﺤﻭﺭ‬
‫)ﻤﺒﺎﺩﺭﺓ( ﺤﻭل ﺍﺘﺠﺎﻩ ﻋﻤﻭﺩﻱ‪ .‬ﻴﻤﻜﻥ ﺤﺴﺎﺏ ﺍﻟﺴﺭﻋﺔ ﺍﻟﺯﺍﻭﻴﺔ ﻟﺤﺭﻜﺔ ﺍﻟﻤﺒﺎﺩﺭﺓ‬
‫‪‬‬
‫‪ ω p  ωp ẑ1‬ﻤﻥ ﺍﻟﻤﻁﺎﺒﻘﺔ ﺒﻴﻥ ﺍﻟﻤﻌﺎﺩﻟﺘﻴﻥ ﺍﻟﺘﺎﻟﻴﺘﻴﻥ‪:‬‬
‫‪‬‬
‫‪dL ‬‬
‫̂‪ M  Mglsinθ x‬‬
‫‪dt‬‬
‫‪‬‬
‫‪dL  ‬‬
‫‪ ωp xL  ωp Lẑ1xẑ ‬‬
‫‪dt‬‬
‫ﺤﻴﺙ ‪ ẑ1  ẑ   x̂sinθ‬ﻭ ‪ ، L  I3ω 3‬ﻓﺈﻨﻨﺎ ﻨﺤﺼل ﻋﻠﻰ ﺍﻟﺘﺭﺩﺩ ﺍﻟﺯﺍﻭﻱ‬
‫ﺍﻟﻤﺘﻭﻗﻊ ﻟﺤﺭﻜﺔ ﻤﺒﺎﺩﺭﺓ ﻤﺴﺘﻘﺭﺓ ﺤﻭل ﺍﻟﻤﺤﻭﺭ ﺍﻟﻌﻤﻭﺩﻱ ﺍﻟﻤﺜﺒﺕ ‪ z1‬ﺒﺎﻟﺸﻜل‪:‬‬
‫‪mgl‬‬
‫‪(6.83) ω p ‬‬
‫‪I 3ω 3‬‬
‫ﻋﻨﺩﻤﺎ ﺘﻜﻭﻥ ‪ ω 3‬ﻜﺒﻴﺭﺓ ﺠﺩﺍﹰ‪ ،‬ﻓﺎﻥ ﻤﻌﺩل ﺍﻟﺘﺭﺩﺩ ﺍﻟﺯﺍﻭﻱ ‪ ω P‬ﻟﺤﺭﻜﺔ‬
‫ﺍﻟﻤﺒﺎﺩﺭﺓ ﻴﻜﻭﻥ ﺼﻐﻴﺭﺍﹰ‪ ،‬ﻭﻴﻜﻭﻥ ﺒﺎﻟﺘﺎﻟﻲ ﻤﺤﻭﺭ ﺘﻨﺎﻅﺭ ﺍﻟﺩﻭﺍﻤﺔ ﺜﺎﺒﺕ ﺘﻘﺭﻴﺒﺎﹰ‪ .‬ﺇﺫﺍ‬
‫ﺃﻋﻁﻴﺕ ﺍﻟﺩﻭﺍﻤﺔ ﺩﻓﻌﺔ ﺯﻫﻴﺩﺓ‪ ،‬ﻓﺈﻨﻬﺎ ﺘﺤﺼل ﺁﻨﻴﺎﹰ ﻋﻠﻰ ﻋﺯﻡ ﻜﻤﻴﺔ ﺤﺭﻜﺔ ﺯﺍﻭﻴﺔ‬
‫‪‬‬
‫ﺼﻐﻴﺭﺓ ‪ Δ L‬ﻋﻤﻭﺩﻴﺔ ﻋﻠﻰ ﺍﻟﻤﺤﻭﺭ ‪ ،z‬ﻭﺘﻜﻭﻥ ﻤﺤﺼﻠﺔ ﻋﺯﻡ ﻜﻤﻴﺔ ﺍﻟﺤﺭﻜﺔ ﺍﻟﺯﺍﻭﻴﺔ‬
‫‪‬‬ ‫‪‬‬ ‫‪‬‬
‫ﺍﻟﻜﻠﻴﺔ ﻤﺴﺎﻭﻴﺔ ﺇﻟﻰ ‪ ، L  L  ΔL‬ﻭﻴﺨﺘﻠﻑ ﺍﺘﺠﺎﻩ ﻫﺫﻩ ﺍﻟﻤﺤﺼﻠﺔ ﻋﻥ ﺍﺘﺠﺎﻩ ﻤﺤﻭﺭ‬
‫~‬

‫ﺍﻟﺘﻨﺎﻅﺭ ‪ Z‬ﺒﺸﻜل ﺯﻫﻴﺩ ﺠﺩﺍﹰ‪ .‬ﺘﺸﺒﻪ ﺍﻟﺤﺭﻜﺔ ﺍﻟﻨﺎﺘﺠﺔ ﺤﺭﻜﺔ ﺩﻭﺍﻤﺔ ﺤﺭﺓ ﻴﺩﻭﺭ ﻤﺤﻭﺭ‬
‫‪‬‬
‫ﺘﻨﺎﻅﺭﻫﺎ ﺤﻭل ﺍﻟﻤﺤﻭﺭ ‪ L‬ﻓﻲ ﺩﺍﺌﺭﺓ ﺼﻐﻴﺭﺓ‪ .‬ﻴﻤﻜﻥ ﺇﻴﺠﺎﺩ ﺍﻟﺘﺭﺩﺩ ﺍﻟﺯﺍﻭﻱ ﻟﻬﺫﻩ‬
‫~‬

‫ﺍﻟﺤﺭﻜﺔ ﺍﻟﺩﺍﺌﺭﻴﺔ ﻤﻥ ﺍﻟﻤﻌﺎﺩﻟﺔ )‪ ،(6.60‬ﻟﻴﻜﻭﻥ ﻤﺴﺎﻭﻴﺎﹰ‪:‬‬


‫‪‬‬
‫~‬
‫‪L Iω‬‬
‫‪ωL   3 3‬‬ ‫)‪(6.84‬‬
‫‪I‬‬ ‫‪I‬‬
‫ﺘﻜﻭﻥ ﺍﻟﺤﺭﻜﺔ ﺍﻟﺘﺎﻤﺔ ﻟﻤﺤﻭﺭ ﺍﻟﺘﻨﺎﻅﺭ ﺘﺭﺍﻜﺏ ﻟﻬﺫﻩ ﺍﻟﺤﺭﻜﺔ ﺍﻟﺩﺍﺌﺭﻴﺔ ﺍﻟﺴﺭﻴﻌﺔ‬
‫‪‬‬
‫ﻟﻠﺩﻭﺍﻤﺔ ﺤﻭل ﺍﻟﻤﺘﺠﻪ ‪ L‬ﻓﻲ ﻤﺒﺎﺩﺭﺓ ﺒﻁﻴﺌﺔ ﻟﻠﻤﺘﺠﻪ ̂‪ L‬ﺤﻭل ﺍﻻﺘﺠﺎﻩ ﺍﻟﻌﻤﻭﺩﻱ‪.‬‬
‫~‬

‫‪- 199 -‬‬

‫)‪Create PDF files without this message by purchasing novaPDF printer (http://www.novapdf.com‬‬
‫‪ - 50‬ﺍﳉﲑﻭﺳﻜﻮﺏ‪The gyroscope :‬‬
‫ﻴﻁﻠﻕ ﺍﺴﻡ ﺍﻟﺠﻴﺭﻭﺴﻜﻭﺏ ﻋﻠﻰ ﺃﻱ ﺠﺴﻡ ﺼﻠﺏ ﻴﺩﻭﺭ ﺤﻭل ﻤﺤﻭﺭﻩ‬
‫ﺍﻟﺘﻨﺎﻅﺭﻱ ﺒﺴﺭﻋﺔ ﺃﻜﺒﺭ ﺒﻜﺜﻴﺭ ﻤﻥ ﺴﺭﻋﺔ ﺩﻭﺭﺍﻥ ﺍﻟﻤﺤﻭﺭ ﻨﻔﺴﻪ ﺤﻭل ﺃﻱ ﻤﺤﻭﺭ‬
‫ﺁﺨﺭ‪ ،‬ﻭﻴﻌﺘﺒﺭ ﺃﻭﻀﺢ ﻤﺜﺎل ﻋﻠﻰ ﺍﻟﺠﻴﺭﻭﺴﻜﻭﺏ ﺍﻵﻟﺔ ﺫﺍﺕ ﺍﻟﻤﻔﺼل ﺍﻟﻤﺘﺤﺭﻙ‬
‫ﺍﻟﻤﻭﻀﺢ ﺭﺴﻤﻬﺎ ﺍﻟﺘﺨﻁﻴﻁﻲ ﻋﻠﻰ ﺍﻟﺸﻜل )‪ ،(6.9‬ﻭﺘﻜﻭﻥ ﺁﻟﻴﺔ ﻋﻤﻠﻬﺎ ﺒﺎﻟﺸﻜل‬
‫ﺍﻟﺘﺎﻟﻲ‪ :‬ﺇﺫﺍ ﺃﺩﻴﺭ ﺍﻟﻘﺭﺹ ﺍﻟﺩﺍﺨﻠﻲ ﺒﺴﺭﻋﺔ ﻜﺒﻴﺭﺓ ﺤﻭل ﻤﺤﻭﺭﻩ ‪ ،A1A2‬ﻓﺎﻥ‬
‫ﺍﻹﻁـــﺎﺭ ﻴﺩﻭﺭ ﺤﻭل ﺍﻟﻤﺤﻭﺭ ‪ B1B2‬ﺍﻟﻤﻭﺠﻭﺩ ﻓﻲ ﺍﻹﻁﺎﺭ ﺍﻟﻌﻤﻭﺩﻱ ﺍﻟﺫﻱ‬
‫ﻴﻤﻜﻨﻪ ﺒﺩﻭﺭﻩ ﺃﻥ ﻴﺩﻭﺭ ﺤﻭل ﺍﻟﻤﺤﻭﺭ ‪.G1G2‬‬

‫ﺍﻟﺸﻜل )‪ :(6.9‬ﺍﻟﺠﻴﺭﻭﺴﻜﻭﺏ‬

‫ﺘﺩل ﻫﺫﻩ ﺍﻟﺤﺭﻜﺔ ﻋﻠﻰ ﺃﻥ ﻟﻺﻁﺎﺭ ﺍﻟﺩﺍﺨﻠﻲ ﺜﻼﺙ ﺩﺭﺠﺎﺕ ﺤﺭﻴﺔ ﻭﻟﻺﻁﺎﺭ‬
‫ﺍﻟﻭﺴﻁﻲ ﺩﺭﺠﺘﻲ ﺤﺭﻴﺔ ﻭﻟﻠﺨﺎﺭﺠﻲ ﺩﺭﺠﺔ ﺤﺭﻴﺔ ﻭﺍﺤﺩﺓ‪ .‬ﻏﻴﺭ ﺃﻥ ﺍﻟﺠﻴﺭﻭﺴﻜﻭﺏ‬
‫ﻤﺼﻤﻡ ﺒﺤﻴﺙ ﻴﻅل ﺍﺘﺠﺎﻩ ﻤﺤﻭﺭ ﺍﻟﺩﻭﺭﺍﻥ ﺒﺼﻔﺔ ﻋﺎﻤﺔ ﻤﺜﺒﺘﺎﹰ ﺤﺘﻰ ﻋﻨﺩﻤﺎ ﻴﻜﻭﻥ‬
‫ﺍﻹﻁﺎﺭ ﺍﻟﺨﺎﺭﺠﻲ ﺍﻟﻤﺘﺼل ﺒﺎﻟﺠﺴﻡ ﺤﺭ ﺍﻟﺤﺭﻜﺔ ﻓﻲ ﺍﻟﻔﺭﺍﻍ‪ ،‬ﻓﻌﻨﺩﻤﺎ ﻴﺩﻭﺭ ﺍﻟﻘﺭﺹ‬
‫‪‬‬
‫ﺤﻭل ﻤﺤﻭﺭﻩ ﺒﺴﺭﻋﺔ ﻜﺒﻴﺭﺓ ﻴﻜﻭﻥ ﺍﻟﻌﺯﻡ ﺍﻟﺯﺍﻭﻱ ‪ L‬ﻜﺒﻴﺭﺍﹰ ﻭﻤﺤﻤﻭﻻﹰ ﻋﻠﻰ ﺍﻟﻤﺤﻭﺭ‬
‫‪ .A1A2‬ﻴﻌﺘﺒﺭ ﺍﻟﺠﻴﺭﻭﺴﻜﻭﺏ ﻤﺜﺎﻻﹰ ﻟﺠﺴﻡ ﺼﻠﺏ ﻴﺘﺤﺭﻙ ﺤﺭﻜﺔ ﺩﻭﺭﺍﻨﻴﺔ )ﻤﻐﺯﻟﻴﺔ(‪،‬‬
‫ﻭﺘﻜﻭﻥ ﺇﺤﺩﻯ ﻨﻘﻁﻪ )ﻋﺎﺩﺓ ﻤﺭﻜﺯ ﻜﺘﻠﺘﻪ( ﻤﺜﺒﺘﺔ ﻋﻠﻰ ﻤﺤﻭﺭ ﺘﻨﺎﻅﺭﻩ‪ ،‬ﻭﻴﻜﻭﻥ ﺒﺎﻟﺘﺎﻟﻲ‬
‫‪‬‬
‫ﻋﺯﻡ ﻗﻭﺓ ﺍﻟﺜﻘل ‪ M‬ﻤﻌﺩﻭﻤﺎﹰ‪ ،‬ﻭﻴﻨﺘﺞ ﺃﻥ ﺍﻟﻌﺯﻡ ﺍﻟﺯﺍﻭﻱ ﻴﻜﻭﻥ ﺜﺎﺒﺘﺎﹰ ﻤﻘﺩﺍﺭ ﺍﹰ ﻭﺍﺘﺠﺎﻫﺎﹰ‪،‬‬
‫ﻭﻴﺒﻘﻰ ﺍﺘﺠﺎﻩ ﺍﻟﻤﺤﻭﺭ ‪ A1A2‬ﺜﺎﺒﺘﺎﹰ ﺩﻭﻤﺎﹰ ﻤﻬﻤﺎ ﻜﺎﻨﺕ ﺤﺭﻜﺔ ﺍﻟﻭﺴﻁ ﺍﻟﻤﻭﺠﻭﺩ ﻓﻴﻪ‪.‬‬

‫‪- 200 -‬‬

‫)‪Create PDF files without this message by purchasing novaPDF printer (http://www.novapdf.com‬‬
‫ﻓﻲ‬ ‫ﻓﻲ ﺍﻟﻔﺭﺍﻍ‬ ‫ﻴﺴﺘﻔﺎﺩ ﻤﻥ ﺨﺎﺼﺔ ﺜﺒﺎﺕ ﺍﺘﺠﺎﻩ ﻤﺤﻭﺭ ﺍﻟﺠﻴﺭﻭﺴﻜﻭﺏ‬
‫ﺘﻁﺒﻴﻘﺎﺕ ﻋﺩﻴﺩﺓ‪ ،‬ﻭﺫﻟﻙ ﻓﻲ ﺍﻷﺤﻭﺍل ﺍﻟﺘﻲ ﺘﺴﺘﻠﺯﻡ ﺍﻻﺤﺘﻔﺎﻅ ﺒﺎﻻﺘﺠﺎﻩ )ﺃﻭ ﺇﺘﺒﺎﻉ ﻫﺩﻑ‬
‫ﻤﻌﻴﻥ(‪ ،‬ﻤﺜل ﻤﻼﺤﺔ ﻭﺇﺭﺸﺎﺩ ﺍﻟﺴﻔﻥ ﻭﺍﻟﻁﺎﺌﺭﺍﺕ ﻭﺍﻟﻐﻭﺍﺼﺎﺕ‪ ،‬ﻭﺍﻟﺘﺤﻜﻡ ﻓﻲ ﺤﺭﻜﺔ‬
‫ﺍﻟﻘﺫﺍﺌﻑ ‪.........‬‬
‫‪- 51‬ﺗﻄﺒﻴﻘﺎﺕ‪:‬‬
‫ﺃ ‪-‬ﺍﳉﲑﻭﺳﻜﻮﺏ ﺍﳊﺮ‪ :‬ﻟﻴﻜﻥ ﺍﻟﺠﻴﺭﻭﺴﻜﻭﺏ ﺍﻟﺫﻱ ﻴﺘﺤﺭﻙ ﺘﺤﺕ ﺘﺄﺜﻴﺭ ﻗﻭﺓ‬
‫‪‬‬
‫‪ ext‬‬
‫‪ M‬ﻭﻴﻨﺘﺞ ﻤﻥ ﺫﻟﻙ ﺃﻥ‪:‬‬ ‫ﻭﺯﻨﻪ ﻓﻘﻁ ﺍﻟﺸﻜل )‪ ،(6.10‬ﻭﺒﺎﻟﺘﺎﻟﻲ ﻓﺈﻥ‬
‫‪dL‬‬
‫‪‬‬ ‫‪0‬‬
‫‪dt‬‬
‫‪‬‬ ‫‪‬‬
‫‪ L  L0‬ﻭﻤﻥ ﺜﻡ ‪   0‬ﺃﻱ ﺃﻥ ﺍﻟﺠﻴﺭﻭﺴﻜﻭﺏ ﻴﺩﻭﺭ ﺒﺴﺭﻋﺔ ﺯﺍﻭﻴﺔ ﺜﺎﺒﺘﺔ ﻤﻘﺩﺍﺭﺍﹰ‬
‫ﻭﺍﺘﺠﺎﻫﺎﹰ‪ ،‬ﻭﻴﺴﺘﻔﺎﺩ ﻤﻥ ﺫﻟﻙ ﻓﻲ ﺼﻨﻊ ﻤﺎ ﻴﺴﻤﻰ ﺒﺎﻟﺒﻭﺼﻠﺔ ﺍﻟﺠﻴﺭﻭﺴﻜﻭﺒﻴﺔ ﺤﻴﺙ‬
‫ﻴﻭﺠﻪ ﻤﺤﻭﺭ ﺍﻟﺠﻴﺭﻭﺴﻜﻭﺏ ﺒﺎﺘﺠﺎﻩ ﻤﻌﻴﻥ )ﺒﺎﺘﺠﺎﻩ ﺍﻟﺸﻤﺎل ﻤﺜﻼﹰ( ﻴﺒﻘﻰ ﻤﺤﺎﻓﻅﺎﹰ ﻋﻠﻴﻪ‬
‫ﻁﺎﻟﻤﺎ ﺒﻘﻴﺕ ﻤﺤﺼﻠﺔ ﺍﻟﻘﻭﻯ ﺍﻟﺨﺎﺭﺠﻴﺔ ﺍﻟﻤﺅﺜﺭﺓ ﻋﻠﻴﻪ ﻤﺴﺎﻭﻴﺔ ﻟﻠﺼﻔﺭ‪.‬‬

‫ﺍﻟﺸﻜل )‪ (6.10‬ﺠﻴﺭﻭﺴﻜﻭﺏ ﺤﺭ‬


‫ﺏ ‪-‬ﺣﺮﻛﺔ ﳏﻮﺭ ﺍﳉﲑﻭﺳﻜﻮﺏ ﻋﻨﺪ ﺗﺄﺛﲑ ﻗﻮﺓ ﻋﻠﻴﻪ‪:‬‬
‫ﺒﻔﺭﺽ ﺃﻨﻨﺎ ﻁﺒﻘﻨﺎ ﻋﻠﻰ ﺍﻟﺠﻴﺭﻭﺴﻜﻭﺏ ﺍﻟﻤﻭﻀﺢ ﻋﻠﻰ ﺍﻟﺸﻜل )‪ ،(6.11‬ﻗﻭﺓ‬
‫‪‬‬
‫ﺃﻓﻘﻴﺔ ‪ ، F‬ﻭﻟﻨﺩﺭﺱ ﺤﺭﻜﺘﻪ‪ .‬ﻴﻨﺤﺭﻑ ﻓﻲ ﻫﺫﻩ ﺍﻟﺤﺎﻟﺔ ﻤﺤﻭﺭ ﺍﻟﺠﻴﺭﻭﺴﻜﻭﺏ ﺒﺎﺘﺠﺎﻩ‬
‫‪‬‬
‫ﺘﺄﺜﻴﺭﻫﺎ ﺇﺫﺍ ﻜﺎﻥ ﺴﺎﻜﻨﺎﹰ‪ ،‬ﻭﻟﻜﻥ ﻜﻴﻑ ﻴﺘﺤﺭﻙ ﺘﺤﺕ ﺘﺄﺜﻴﺭ ﺍﻟﻘﻭﺓ ‪ F‬ﺇﺫﺍ ﻜﺎﻥ ﻴﺩﻭﺭ‬
‫‪‬‬
‫ﺒﺴﺭﻋﺔ ﺯﺍﻭﻴﺔ ‪ 1‬ﻗﺒل ﺘﺄﺜﻴﺭ ﺍﻟﻘﻭﺓ؟‬

‫‪- 201 -‬‬

‫)‪Create PDF files without this message by purchasing novaPDF printer (http://www.novapdf.com‬‬
‫ﺍﻟﺸﻜل )‪(6.11‬‬
‫ﺘﺄﺜﻴﺭ ﻗﻭﺓ ﻋﻠﻰ ﻤﺤﻭﺭ ﺠﻴﺭﺴﻜﻭﺏ‬
‫‪‬‬
‫ﻟﻺﺠﺎﺒﺔ ﻋﻥ ﻫﺫﺍ ﺍﻟﺴﺅﺍل ﻨﺤﺴﺏ ﻋﺯﻡ ﺍﻟﻘﻭﺓ ‪ F‬ﺍﻟﺫﻱ ﺴﻴﻜﻭﻥ ﻤﺘﻌﺎﻤﺩﺍﹰ ﻤﻊ‬
‫‪‬‬
‫ﺍﻟﻘﻭﺓ ‪ F‬ﻭﻴﺘﺠﻪ ﻤﻥ ﺍﻟﻨﻘﻁﺔ ‪ 0‬ﺇﻟﻰ ﺍﻟﻨﺎﻅﺭ ﺤﺴﺏ ﻗﺎﻋﺩﺓ ﺍﻟﺒﺯﺍل ﺍﻟﻤﻌﺭﻭﻓﺔ‪.‬‬
‫‪ext‬‬
‫‪ ‬‬
‫ﻓﺈﻥ ﻤﺤﻭﺭ ﺍﻟﺠﻴﺭﻭﺴﻜﻭﺏ ﺴﻴﺘﺤﺭﻙ ﺒﺎﻻﺘﺠﺎﻩ ﻨﻔﺴﻪ ﺃﻱ‬ ‫‪uM‬‬ ‫ﺒﻔﺭﺽ ﺃﻥ‬
‫ﻜﻤﺎ ﻫﻭ ﻤﺒﻴﻥ ﻋﻠﻰ ﺍﻟﺸﻜل )‪ ،(6.11‬ﻭﺒﺎﻟﺘﺎﻟﻲ ﻓﺈﻥ ﺘﺄﺜﻴﺭ ﺍﻟﻘﻭﺓ ﺍﻷﻓﻘﻴﺔ ﻻ ﻴﺅﺩﻱ ﺇﻟﻰ‬
‫ﺤﺭﻜﺔ ﺍﻟﺠﻴﺭﻭﺴﻜﻭﺏ ﺒﺎﺘﺠﺎﻫﻬﺎ‪.‬‬
‫ﻭﺍﻟﺠﺩﻴﺭ ﺒﺎﻟﺫﻜﺭ ﺃﻨﻪ ﻋﻨﺩﻤﺎ ﺘﺅﺜﺭ ﻋﻠﻰ ﺍﻟﺠﻴﺭﻭﺴﻜﻭﺏ ﻗﻭﺓ ﻴﺩﻭﻡ ﺘﺄﺜﻴﺭﻫ ﺎ ﻗﻠﻴﻼﹰ‬
‫ﻤﻊ ﺍﻟﺯﻤﻥ ﻓﺈﻥ ﻤﺤﻭﺭ ﺍﻟﺠﻴﺭﻭﺴﻜﻭﺏ ﻻ ﻴﻐﻴﺭ ﺘﻘﺭﻴﺒﺎﹰ ﻤﻥ ﺩﻭﺭﺍﻨﻪ ﻭﻟﻜﻨﻪ ﻴﺘﺭﻨﺢ ﻗﻠﻴﻼﹰ‬
‫)ﺤﺭﻜﺔ ﺘﺭﻨﺤﻴﺔ( ﺒﺤﻴﺙ ﺘﺘﻐﻴﺭ ﺍﻟﺯﺍﻭﻴﺔ ‪ θ‬ﺤﻭل ﻭﻀﻌﻬﺎ ﺍﻷﻭﻟﻲ ﻗﺒل ﺘﺄﺜﻴﺭ ﺍﻟﻘﻭﺓ‪.‬‬
‫‪‬‬
‫ﻭﻟﺤﺴﺎﺏ ﺍﻟﺴﺭﻋﺔ ﺍﻟﺯﺍﻭﻴﺔ ﻟﻤﺤﻭﺭ ﺍﻟﺩﻭﺭﺍﻥ ﻋﻨﺩ ﺘﺄﺜﻴﺭ ﺍﻟﻘﻭﺓ ‪ F‬ﻋﻠﻴﻪ ﻨﺴﺘﺨﺩﻡ‬
‫ﺍﻟﻌﻼﻗﺔ ﺍﻟﺘﻲ ﺘﻌﻁﻲ ﺴﺭﻋﺔ ﺩﻭﺭﺍﻥ ﺠﺴﻡ ﺼﻠﺏ ﺤﻭل ﻤﺤﻭﺭ ﺜﺎﺒﺕ‪ ،‬ﺤﻴﺙ ﻨﺠﺩ‪:‬‬
‫‪ ‬‬ ‫‪  ‬‬
‫‪v  ω 2  L  u  M ext‬‬ ‫)‪(6.85‬‬
‫ﻭﻤﻨﻪ‪:‬‬
‫‪‬‬
‫‪M ext‬‬
‫‪ω‬‬ ‫)‪(6.86‬‬
‫‪L sinθ‬‬
‫‪‬‬ ‫‪‬‬
‫ﺤﻴﺙ ‪ θ‬ﺍﻟﺯﺍﻭﻴﺔ ﺒﻴﻥ ‪ L‬ﻭ ‪.  2‬‬

‫‪- 202 -‬‬

‫)‪Create PDF files without this message by purchasing novaPDF printer (http://www.novapdf.com‬‬
‫ﺟـ ‪ -‬ﻜﺎﻨﺕ ﺍﻟﻘﻭﺓ ﺍﻟﻤﺅﺜﺭﺓ ﻋﻠﻰ ﻤﺤﻭﺭ ﺍﻟﺠﻴﺭﻭﺴﻜﻭﺏ ﻓﻲ ﺍﻟﻤﺜﺎﻟﻴﻥ ﺍﻟﺴﺎﺒﻘﻴﻥ‬
‫ﻤﻌﻠﻭﻤﺔ‪ ،‬ﻭﻗﺩ ﺤﺴﺒﻨﺎ ﺴﺭﻋﺔ ﺩﻭﺭﺍﻥ ﺍﻟﻤﺤﻭﺭ ﺍﻟﻨﺎﺘﺠﺔ ﻤﻥ ﺘﺄﺜﻴﺭ ﻫﺫﻩ ﺍﻟﻘﻭﺓ‪ ،‬ﻭﻟﻜﻥ ﻫل‬
‫ﻴﻤﻜﻥ ﻤﻌﺭﻓﺔ ﻗﻭﺓ ﺘﺅﺜﺭ ﻋﻠﻰ ﺍﻟﻤﺤﻭﺭ ﻭﺘﺴﺒﺏ ﺩﻭﺭﺍﻨﺎﹰ ﻤﺎ ﻤﻌﻠﻭﻤﺎﹰ؟ )ﺍﻟﻤﺴﺄﻟﺔ ﺍﻟﻌﻜﺴﻴﺔ(‬
‫ﻟﻴﻜﻥ ﺍﻟﺠﻴﺭﻭﺴﻜﻭﺏ ﺍﻟﺫﻱ ﻴﻤﻜﻨﻪ ﺍﻟﺩﻭﺭﺍﻥ ﺤﻭل ﻤﺤﻭﺭ ﺭﺃﺴﻲ )ﺸﺎﻗﻭﻟﻲ(‬
‫ﺒﺎﻹﻀﺎﻓﺔ ﺇﻟﻰ ﺩﻭﺭﺍﻨﻪ ﺤﻭل ﻤﺤﻭﺭ ‪ AB‬ﻜﻤﺎ ﻋﻠﻰ ﺍﻟﺸﻜل )‪ ،(6.12‬ﻭﻟﻨﺒﺤﺙ ﻋﻥ‬
‫‪‬‬ ‫‪‬‬
‫ﺍﻟﻘﻭﻯ ﺍﻟﺘﻲ ﺘﺴﺒﺏ ﺍﻟﺩﻭﺭﺍﻥ ﺒﺴﺭﻋﺔ ﺯﺍﻭﻴﺔ ‪ .  2‬ﻴﻌﻁﻰ ‪ M ext‬ﻋﺯﻡ ﺍﻟﻘﻭﻯ‬
‫ﺍﻟﺨﺎﺭﺠﻴﺔ ﺍﻟﻤﺅﺜﺭﺓ ﻋﻠﻰ ﻤﺤﻭﺭ ﺍﻟﺩﻭﺭﺍﻥ ﺒﺎﻟﻌﻼﻗﺔ )‪ ،(6.85‬ﻴﻨﺘﺞ ﻤﻥ ﺘﻠﻙ ﺍﻟﻌﻼﻗﺔ ﺃﻥ‬
‫‪‬‬
‫ﺒﺤﻴﺙ ﺘﻜﻭﻥ ﺍﻟﺜﻼﺜﻴﺔ ‪ 2 , L , M ext ‬‬ ‫‪‬‬ ‫‪‬‬
‫ﺍﻟﻤﺘﺠﻪ ‪ M ext‬ﻤﺘﻌﺎﻤﺩ ﻤﻊ ﻜلٍ ﻤﻥ ‪  2‬ﻭ ‪L‬‬
‫ﻤﻭﺠﺒﺔ ﺃﻱ ﻜﻤﺎ ﻫﻭ ﻤﺒﻴﻥ ﻋﻠﻰ ﺍﻟﺸﻜل )‪ ،(6.12‬ﻟﻜﻲ ﻴﻜﻭﻥ ﻫﺫﺍ ﺍﻟﻌﺯﻡ ﻤﻭﺠﻭﺩﺍﹰ‬
‫‪ ‬‬
‫ﻴﺠﺏ ﺃﻥ ﺘﺅﺜﺭ ﻋﻠﻰ ﻤﺤﻭﺭ ﺍﻟﺠﻴﺭﻭﺴﻜﻭﺏ ﻤﺯﺩﻭﺠﺔ ﻗﻭﻯ ‪ Q,Q‬ﺘﺅﺜﺭ ﻋﻠﻰ ﺤﻠﻘﺘﻲ‬
‫ﺘﺜﺒﻴﺘﻪ ‪ A‬ﻭ ‪ ،B‬ﻭﻭﻓﻘﺎﹰ ﻟﻘﺎﻨﻭﻥ ﻨﻴﻭﺘﻥ ﺍﻟﺜﺎﻟﺙ ﻴﻨﺒﻐﻲ ﺃﻥ ﻴﺅﺜﺭ ﻤﺤﻭﺭ ﺍﻟﺠﻴﺭﻭﺴﻜﻭﺏ‬
‫‪ ‬‬
‫ﻋﻠﻰ ﺤﻠﻘﺘﻲ ﺍﻟﺘﺜﺒﻴﺕ ‪ A‬ﻭ ‪ B‬ﺒﻤﺯﺩﻭﺠﺔ ﻤﻌﺎﻜﺴﺔ ‪ ، F , F ‬ﺘﺴﻤﻰ ﺍﻟﻤﺯﺩﻭﺠﺔ‬
‫‪‬‬
‫ﺍﻟﺠﻴﺭﻭﺴﻜﻭﺒﻴﺔ‪ ،‬ﻋﺯﻤﻬﺎ ﻴﺴﺎﻭﻱ ﻭﻴﻌﺎﻜﺱ ﺍﻟﻌﺯﻡ ‪ M ext‬ﻜﻤﺎ ﻫﻭ ﻤﻭﻀﺢ ﻋﻠﻰ‬
‫ﺍﻟﺸﻜل )‪ ،(6.12‬ﻭﻴﻜﻭﻥ‪:‬‬
‫‪‬‬ ‫‪‬‬ ‫‪ ‬‬ ‫‪‬‬ ‫‪‬‬
‫‪M g  M ext  L   2  I1   2‬‬ ‫)‪(6.87‬‬

‫ﺍﻟﺸﻜل )‪(6.12‬‬

‫‪- 203 -‬‬

‫)‪Create PDF files without this message by purchasing novaPDF printer (http://www.novapdf.com‬‬
‫ﻭﻤﻥ ﺍﻟﺠﺩﻴﺭ ﺒﺎﻟﺫﻜﺭ ﺃﻥ ﺍﻟﺒﻭﺍﺨﺭ ﻭﺍﻟﻁﺎﺌﺭﺍﺕ ﻭﺍﻟﺴﻴﺎﺭﺍﺕ ﺘﺘﻌﺭﺽ ﻟﻬﺫﻩ ﺍﻟﻘﻭﺓ‬
‫ﻋﻨﺩﻤﺎ ﺘﻐﻴﺭ ﻤﺴﺎﺭﻫﺎ‪ .‬ﺇﺫ ﺃﻥ ﻤﺤﺭﻙ ﺍﻟﺒﺎﺨﺭﺓ ﻋﻠﻰ ﺴﺒﻴل ﺍﻟﻤﺜﺎل ﻴﻠﻌﺏ ﺩﻭﺭ‬
‫ﺠﻴﺭﻭﺴﻜﻭﺏ ﻀﺨﻡ‪ ،‬ﻭﺇﺫﺍ ﻓﺭﻀﻨﺎ ﺃﻥ ﺍﻟﺒﻌﺩ ﺒﻴﻥ ﻨﻘﻁﺘﻲ ﺘﺜﺒﻴﺘﻪ ﻴﺴﺎﻭﻱ ‪ ، l‬ﻓﺈﻥ ﻋﺯﻡ‬
‫‪ ‬‬
‫ﺍﻟﻤﺯﺩﻭﺠﺔ ‪ F , F ‬ﺍﻟﻤﺅﺜﺭﺓ ﻋﻠﻰ ﺤﻠﻘﺘﻲ ﺘﺜﺒﻴﺕ ﻤﺤﻭﺭ ﺍﻟﻤﺤﺭﻙ ﻓﻲ ‪ A‬ﻭ ‪، B‬‬
‫‪‬‬ ‫‪‬‬ ‫‪‬‬
‫ﻴﻌﻁﻰ ﺒﺎﻟﻌﻼﻗﺔ )‪ ،(6.87‬ﻭﻴﻤﻜﻥ ﺤﺴﺎﺏ ‪ F‬ﺇﺫﺍ ﻓﺭﻀﻨﺎ ﺃﻥ‪   0 :‬ﻭﺫﻟﻙ ﻜﻤﺎ‬
‫ﻋﻠﻰ ﺍﻟﺸﻜل )‪ ،(6.13‬ﺤﻴﺙ ﻨﺠﺩ‪:‬‬
‫‪Iω0ω‬‬
‫‪lF  lF   IωO ω  F ‬‬ ‫)‪(6.88‬‬
‫‪l‬‬

‫ﺍﻟﺸﻜل )‪(6.13‬‬
‫ﻭﻗﺩ ﺘﺅﺩﻱ ﻫﺫﻩ ﺍﻟﻘﻭﺓ ﺇﺫﺍ ﻟﻡ ﺘﺅﺨﺫ ﺒﻌﻴﻥ ﺍﻻﻋﺘﺒﺎﺭ ﺇﻟﻰ ﺘﺸﻭﻩ ﺤﻠﻘﺎﺕ ﺘﺜﺒﻴﺕ‬
‫ﺍﻟﻤﺤﺭﻙ ﻭﺒﺎﻟﺘﺎﻟﻲ ﺇﻟﻰ ﺘﻬﺩﻴﺩ ﺴﻼﻤﺔ ﺍﻟﺒﺎﺨﺭﺓ ﻜﻜل‪ ،‬ﻭﻟﻜﻲ ﻴﻘﻠل ﺘﺄﺜﻴﺭﻫﺎ ﻴﻨﺒﻐﻲ ﺃﻥ‬
‫ﺘﻨﻌﻁﻑ ﺍﻟﺒﺎﺨﺭﺓ ﺒﺴﺭﻋﺔ ﺒﻁﻴﺌﺔ )‪ ω2‬ﺼﻐﻴﺭﺓ ( ﻭﺃﻥ ﻴﻜﻭﻥ ‪ l‬ﺍﻟﺒﻌﺩ ﺒ ﻴﻥ ﺤﻠﻘﺎﺕ‬
‫ﺘﺜﺒﻴﺕ ﺍﻟﻤﺤﺭﻙ ﻜﺒﻴﺭﺍﹰ‪ .‬ﻴﺒﺩﻭ ﻋﻨﺩﺌﺫ‪ ‬ﻤﻥ ﺍﻟﻤﻌﺎﺩﻟﺔ )‪ ،(6.88‬ﺃﻥ ‪ F‬ﺘﻜﻭﻥ ﺼﻐﻴﺭﺓ‪.‬‬
‫ﻭﻫﺫﺍ ﻤﺎ ﻴﺭﺍﻋﻰ ﺃﺜﻨﺎﺀ ﺒﻨﺎﺀ ﺍﻟﺒﻭﺍﺨﺭ ﻭﻏﻴﺭﻫﺎ ﻤﻥ ﻭﺴﺎﺌﻁ ﺍﻟﻨﻘل ﺍﻟﺘﻲ ﻴﻠﻌﺏ‬
‫ﻓﻴﻬﺎ ﺍﻟﻤﺤﺭﻙ ﺩﻭﺭ ﺍﻟﺠﻴﺭﻭﺴﻜﻭﺏ‪.‬‬
‫ﺩ ‪ -‬ﺗﻄﺒﻴﻘﺎﺕ ﺟﲑﻭﺳﻜﻮﺑﻴﺔ ﰲ ﺍﻟﺘﻘﻨﻴﺔ‪ :‬ﺇﻥ ﺍﻟﻤﻴﺯﺍﺕ ﺍﻟﺘﻲ ﻴﺘﻤﺘﻊ ﺒﻬﺎ‬
‫ﺍﻟﺠﻴﺭﻭﺴﻜﻭﺏ ﺘﺠﻌﻠﻪ ﻴﺴﺘﻌﻤل ﻋﻠﻰ ﻨﻁﺎﻕ ﻭﺍﺴﻊ ﻓﻲ ﺍﻟﺼﻨﺎﻋﺔ ﻭﺍﻟﺘﻘﻨﻴﺔ‪ ،‬ﻭﻗﺩ ﺫﻜﺭﻨﺎ‬

‫‪- 204 -‬‬

‫)‪Create PDF files without this message by purchasing novaPDF printer (http://www.novapdf.com‬‬
‫ﺴﺎﺒﻘﺎﹰ ﺍﻟﺒﻭﺼﻠﺔ ﺍﻟﺠﻴﺭﻭﺴﻜﻭﺒﻴﺔ ﺍﻟﻤﺒﻨﻴﺔ ﻋﻠﻰ ﺃﺴﺎﺱ ﺍﻟﺠﻴﺭﻭﺴﻜﻭﺏ ﺍﻟﺤﺭ ﺍﻟﺫﻱ ﻴﺒﻘﻰ‬
‫ﻤﺤﻭﺭ ﺩﻭﺭﺍﻨﻪ ﺜﺎﺒﺘﺎﹰ ﺇﺫﺍ ﺍﻨﻌﺩﻤﺕ ﺍﻟﻘﻭﻯ ﺍﻟﺨﺎﺭﺠﻴﺔ ﺍﻟﻤﺅﺜﺭﺓ ﻋﻠﻴﻪ‪.‬‬
‫ﻭﺘﻭﺠﺩ ﻤﺠﺎﻻﺕ ﻜﺜﻴﺭﺓ ﻟﺘﻁﺒﻴﻘﺎﺕ ﺍﻟﺠﻴﺭﻭﺴﻜﻭﺏ‪ ،‬ﻜﺎﻻﺴﺘﺨﺩﺍﻡ ﻓﻲ ﻋﻤﻠﻴﺎﺕ‬
‫ﺍﻟﻤﻼﺤﺔ ﺍﻟﺒﺤﺭﻴﺔ ﻭﻓﻲ ﺼﻨﺎﻋﺔ ﻤﺒﻴﻥ ﺍﻷﻓﻕ ﺍﻟﺼﻨﺎﻋﻲ ﻭﻤﺒﻴﻥ ﺍﻟﺩﻭﺭﺍﻥ‬
‫ﺍﻟﺠﻴﺭﻭﺴﻜﻭﺒﻲ ﻭﻤﻨﻬﺎ ﻤﺎ ﻴﺴﺘﺨﺩﻡ ﻓﻲ ﺍﻟﻤﻼﺤﺔ ﺍﻟﺠﻭﻴﺔ‪ ،‬ﻓﺎﻟﻁﺎﺌﺭﺍﺕ ﺍﻻﺘﻭﻤﺎﺘﻴﻜﻴﺔ ﺍﻟﺘﻲ‬
‫ﺘﻁﻴﺭ ﺒﺩﻭﻥ ﻁﻴﺎﺭ ﻭﺍﻟﻁﺎﺌﺭﺍﺕ ﺍﻟﻌﻤﻭﺩﻴﺔ ﺘﺤﺘﻭﻱ ﻋﻠﻰ ﺃﺠﻬﺯﺓ ﺠﻴﺭﻭﺴﻜﻭﺒﻴﺔ‪.‬‬
‫ﻭﺴﻨﺨﺹ ﻓﻲ ﻫﺫﻩ ﺍﻟﻔﻘﺭﺓ ﺠﻬﺎﺯﻴﻥ ﻓﻘﻁ‪:‬‬
‫‪-‬ﺍﻟﺠﻬﺎﺯ ﺍﻟﻤﻀﺎﺩ ﻟﻠﺘﺄﺭﺠﺢ‪ :‬ﻭﻫﻭ ﻋﺒﺎﺭﺓ ﻋﻥ ﺠﻴﺭﺴﻜﻭﺏ ﻀﺨﻡ ﻴﺩﻭﺭ ﺤﻭل‬
‫ﻤﺤﻭﺭ ﻋﻤﻭﺩﻱ ﺒﺴﺭﻋﺔ ﺯﺍﻭﻴﺔ ﻜﺒﻴﺭﺓ ‪ ω0‬ﻭﻫﻭ ﻤﻭﻀﻭﻉ ﻀﻤﻥ ﺇﻁﺎﺭ ﻴﻤﻜﻨﻪ‬
‫ﺍﻟﺩﻭﺭﺍﻥ ﺤﻭل ﻤﺤﻭﺭ ﺃﻓﻘﻲ‪ .‬ﻭﻴﻌﻤل ﻫﺫﺍ ﺍﻟﺠﻬﺎﺯ ﻜﻤﺎ ﻴﻠﻲ‪:‬‬
‫‪‬‬
‫ﻋﻨﺩﻤﺎ ﻴﺅﺜﺭ ﻋﺯﻡ ﻤﺎ ‪ M‬ﻋﻠﻰ ﺍﻟﺴﻔﻴﻨﺔ ﺒﻔﻌل ﺍﻷﻤﻭﺍﺝ ﺍﻟﺒﺤﺭﻴﺔ ﻤﺜﻼﹰ ﻴﺒﺩﺃ‬
‫ﺍﻟﻤﺤﺭﻙ ﺍﻟﻤﺠﻬﺯ ﺒﻨﻅﺎﻡ ﺨﺎﺹ‪ ،‬ﺒﺈﺩﺍﺭﺓ ﺍﻻﻁﺎﺭ ‪ D‬ﺒﺴﺭﻋﺔ ﺯﺍﻭﻴﺔ '‪ ω‬ﻤﻤﺎ ﻴﺴﺒﺏ‬
‫ﻀﻐﻁﺎﹰ ﺠﻴﺭﻭﺴﻜﻭﺒﻴﺎﹰ )‪ ('N,N‬ﻴﺅﺜﺭ ﻋﻠﻰ ﺤﻠﻘﺘﻲ ﺍﻟﺘﺜﺒﻴﺕ ‪ A‬ﻭ ‪ B‬ﻜﻤﺎ ﻋﻠﻰ ﺍﻟﺸﻜل‬
‫‪‬‬
‫)‪ ،(6.14‬ﺒﺤﻴﺙ ﻴﻌﺎﻜﺱ ﻋﺯﻡ ﻫﺫﺍ ﺍﻟﻀﻐﻁ‪ ،‬ﺍﻟﻌﺯﻡ ﺍﻟﺨﺎﺭﺠﻲ ‪ M‬ﺍﻟﻨﺎﺘﺞ ﻋﻥ‬
‫ﺍﻷﻤﻭﺍﺝ ﻭﻫﺫﺍ ﻴﺤﺩ ﻤﻥ ﺘﺄﺭﺠﺢ ﺍﻟﺴﻔﻴﻨﺔ‪.‬‬

‫ﺍﻟﺸﻜل )‪(6.14‬‬

‫‪- 205 -‬‬

‫)‪Create PDF files without this message by purchasing novaPDF printer (http://www.novapdf.com‬‬
‫‪-‬ﺠﻬﺎﺯ ﺍﻭﺒﺭﻯ ﻟﺘﺼﺤﻴﺢ ﺍﻟﻘﺫﺍﺌﻑ ﺍﻟﻤﻨﻁﻠﻘﺔ ﺇﻟﻰ ﺃﻫﺩﺍﻓﻬﺎ ﺤﻴﺙ ﻴﻭﺠﻪ ﻤﺤﻭﺭ‬
‫ﺠﻴﺭﻭﺴﻜﻭﺏ ﻴﺩﻭﺭ ﺒﺴﺭﻋﺔ ﻜﺒﻴﺭﺓ ﻨﺤﻭ ﺍﻟﻬﺩﻑ ﺍﻟﻤﺭﺍﺩ ﺘﺩﻤﻴﺭﻩ‪ ،‬ﻭﻋﻨﺩﻤﺎ ﻴﻨﺤﺭﻑ‬
‫ﺍﻟﻁﻭﺭﺒﻴﺩ ﻋﻥ ﻤﺴﺎﺭﻩ‪ ،‬ﺒﻔﻌل ﺃﻤﻭﺍﺝ ﺍﻟﺒﺤﺭ ﻤﺜﻼﹰ‪ ،‬ﺒﺯﺍﻭﻴﺔ ‪ θ‬ﺸﻜل )‪ ،(6.15‬ﻓﺈﻥ‬
‫ﻤﺤﻭﺭ ﺍﻟﺠﻴﺭﻭﺴﻜﻭﺏ ﻴﺒﻘﻰ ﺜﺎﺒﺘﺎﹰ ﺒﺴﺒﺏ ﺍﻨﻌﺩﺍﻡ ﺍﻟﻘﻭﻯ ﺍﻟﺨﺎﺭﺠﻴﺔ ﺍﻟﻤﺅﺜﺭﺓ ﻋﻠﻴﻪ‪،‬‬
‫ﻭﻋﻨﺩﺌﺫ‪ ‬ﻴﻐﻠﻕ ﺍﻟﻤﺤﻭﺭ ‪ A1A2‬ﺩﺍﺭﺓ ﻜﻬﺭﺒﺎﺌﻴﺔ ﺨﺎﺼﺔ ﻜﻤﺎ ﻫﻭ ﻤﺒﻴﻥ ﻋﻠﻰ ﺍﻟﺸﻜل‬
‫)‪ ،(6.16‬ﺘﺩﻭﺭ ﻤﺤﺭﻜﺎﹰ ﺨﺎﺼﺎﹰ‪ ،‬ﻤﺜﺒﺘﺎﹰ ﻓﻲ ﺠﺴﻡ ﺍﻟﻁﻭﺭﺒﻴﺩ‪ ،‬ﻴﻌﻤل ﻋﻠﻰ ﺘﺼﺤﻴﺢ‬
‫ﺍﻟﻤﺴﺎﺭ ﺒﺤﻴﺙ ﻴﺘﻡ ﺍﻟﺘﻭﺠﻪ ﻤﻥ ﺠﺩﻴﺩ ﺒﺎﺘﺠﺎﻩ ﺍﻟﻬﺩﻑ‪.‬‬

‫ﺍﻟﺸﻜل )‪(6.15‬‬

‫ﺍﻟﺸﻜل )‪ :(6.16‬ﺩﺍﺭﺓ ﺘﻭﺠﻴﻪ ﺍﻟﺠﻴﺭﻭﺴﻜﻭﺏ‬

‫‪- 206 -‬‬

‫)‪Create PDF files without this message by purchasing novaPDF printer (http://www.novapdf.com‬‬
Solved Examples ‫ﺃﻣﺜﻠﺔ ﳏﻠﻮﻟﺔ‬

‫ﺍﺴﺘﻨﺘﺞ ﻤﻌﺎﺩﻻﺕ ﻋﺯﻡ ﻜﻤﻴﺔ ﺍﻟﺤﺭﻜﺔ ﺍﻟﺯﺍﻭﻴﺔ ﺒﺩﻻﻟﺔ ﻋﺯﻭﻡ ﻭﻤﻀﺎﺭﺏ‬-1


.‫ﺍﻟﻌﻁﺎﻟﺔ ﻟﻤﺠﻤﻭﻋﺔ ﻤﻴﻜﺎﻨﻴﻜﻴﺔ‬
:‫ﺍﳊﻞ‬
:‫ﻴﻌﻁﻰ ﻋﺯﻡ ﻜﻤﻴﺔ ﺍﻟﺤﺭﻜﺔ ﺍﻟﺯﺍﻭﻴﺔ ﻟﻤﺠﻤﻭﻋﺔ ﻤﻴﻜﺎﻨﻴﻜﻴﺔ ﺒﺎﻟﻌﻼﻗﺔ ﺍﻟﺘﺎﻟﻴﺔ‬
 N     
L   m i  ri  vi    m i  ri  ω  ri 
i 1 i

:‫ﻨﺠﺩ ﺒﺎﺴﺘﺨﺩﺍﻡ ﺨﻭﺍﺹ ﺍﻟﻀﺭﺏ ﺍﻟﻤﺘﺠﻪ ﺃﻥ‬


      
ri  ω  r̂i   ωri . ri   ri ω. ri 
  
 
 ω x i  ω y j  ω z k x i2  y i2  z i2 
  
 
- x i i  y i j  z i k ω i x i  ω i y i  ω i z i 

  
 ω x y i2  z i2  ω y x i y i  ω z x i z i z i i 

  
 ω y x 2i  z 2i  ω x x i y i  ω z y i z i j 

  
 ω z x i2  y i2  ω x x i z i  ω y y i z i k 
i ‫ ﻭﺍﻟﺠﻤﻊ ﻋﻠﻰ‬i ‫ﺒﻀﺭﺏ ﻁﺭﻓﻲ ﻫﺫﻩ ﺍﻟﻌﻼﻗﺔ ﺒﻜﺘﻠﺔ ﺍﻟﺠﺴﻴﻡ ﺭﻗﻡ‬
  
:‫ ﻨﺠﺩ ﺃﻥ‬،‫ ﻋﻠﻰ ﺍﻟﺘﺭﺘﻴﺏ‬L x , L y , L z ‫ ﻤﻊ‬i , j , k ‫ﻭﻤﺴﺎﻭﺍﺓ ﻤﻌﺎﻤﻼﺕ‬
N   N   N 
 
Lx   mi y2i  zi2 ωx   mi xi yi ωy    mi xi z i ωz
i   i   i 
 Ixxωx  Ixyωy  Ixzωz

 N  N N

Ly   mi xi yi ωx  mi x2i  z2i ω   m y z ω
y i i i z
 i  i   i 
 Ixyωx  Iyyωy  Iyzωz

- 207 -

Create PDF files without this message by purchasing novaPDF printer (http://www.novapdf.com)
 N   N  N

Lz   mi xi zi ωx   mi yi zi ωy  mi x2i  y2i ω z
 i   i  i 
 Ixzωx  Iyzωy  Izzωz

‫ﻴﻤﻜﻥ ﺍﻟﺤﺼﻭل ﻋﻠﻰ ﺍﻟﻨﺘﺎﺌﺞ ﻨﻔﺴﻬﺎ ﺒﺎﻟﻨﺴﺒﺔ ﻟﻜﺘﻠﺔ ﻤﺘﺼﻠﺔ ﺍﻟﺘﻭﺯﻴﻊ ﻜﺜﺎﻓﺘﻬﺎ‬
:‫ ﻭﺫﻟﻙ ﺍﻨﻁﻼﻗﺎﹰ ﻤﻥ ﺍﻟﻌﻼﻗﺔ‬،‫ ﺒﺄﺨﺫ ﺍﻟﺘﻜﺎﻤل ﺒﺩﻻﹰ ﻤﻥ ﺍﻟﺠﻤﻊ‬، σ
     
L   σ r  vdτ    r  ω  r dτ
R R

‫ ﺇﺫﺍ ﻜﺎﻥ‬.‫ ﻓﻴﻪ ﻨﻘﻁﺔ ﻭﺍﺤﺩﺓ ﻤﺜﺒﺘﺔ‬ ‫ ﻴﺩﻭﺭ ﺠﺴﻡ ﺼﻠﺏ ﺒﺴﺭﻋﺔ ﺯﺍﻭﻴﺔ‬-2
‫ ﺍﺜﺒﺕ ﺃﻥ ﻁﺎﻗﺔ ﺍﻟﺤﺭﻜﺔ ﺍﻟﺩﻭﺭﺍﻨﻴﺔ ﻟﻠﺠﺴﻡ‬، L ‫ﻋﺯﻡ ﻜﻤﻴﺔ ﺍﻟﺤﺭﻜﺔ ﺍﻟﺯﺍﻭﻴﺔ ﻟﻠﺠﺴﻡ ﻫﻭ‬
:‫ﺘﻌﻁﻰ ﺒـ‬
1 
T ωL -‫أ‬
2
1
T
2

Ixx ω2x  I yyω2y  Izz ω2z  2Ixyωx ωy  2Ixzωxωz  2Iyzωyωz  -‫ب‬

:‫ﺍﳊﻞ‬
.‫ ﻨﻌﻠﻡ ﺃﻥ ﻁﺎﻗﺔ ﺍﻟﺤﺭﻜﺔ ﺘﻌﻁﻰ ﺒﺎﻟﻌﻼﻗﺔ‬-‫ﺃ‬
1 N
1 N
T 
2 i
mi v2i   mi ri .ri
2 i
 
1 N
    1 N   
. ω  ri    mi ω.ri  ω  ri 
  mi ω  ri 
2 i 2 i
1 N
   1 
 ω. m i ri  ω  ri   ω.L
2 i 2
-‫ﺏ‬
1   1      
T  ω .L 
2 2
 
ω x i  ω y j  ω zk . L x i  L y j  L zk 
1

2

ωxLx  ωyLy  ωzLz 
1

2
   
ω x I xx ω x  I xy ω y  I xz ω z  ω y I yx ω x  I yy ω y  I yz ω z 

ω z I zx ω z  I zy ω y  I zz ω z 

1
2

I xx ω 2x  I yy ω 2y  I zz ω 2z  2I xy ω x ω y  2I xz ω x ω z  2I yz ω y ω z 

- 208 -

Create PDF files without this message by purchasing novaPDF printer (http://www.novapdf.com)
I xy  I yx , I xz  I zx , I yz  I zy ‫ﺤﻴﺙ ﺍﺴﺘﺨﺩﻤﻨﺎ ﺨﺎﺼﺔ ﺍﻟﺘﻨﺎﻅﺭ‬
‫ ﺒﺎﻟﻨﺴﺒﺔ‬R ‫ﺍﻟﻌﻁﺎﻟﺔ ﻟﺠﺴﻡ ﺼﻠﺏ‬ ‫ ﺒﻔﺭﺽ ﺃﻥ ﻋﺯﻭﻡ ﻭﻤﻀﺎﺭﻴﺏ‬-3
‫ ﺍﺜﺒﺕ ﺃﻥ ﻋﺯﻭﻡ‬. I xx , I yy , I zz , I xy , I xz , I yz ‫ ﻫﻲ‬xyz ‫ﻟﻤﺠﻤﻭﻋﺔ ﺇﺤﺩﺍﺜﻴﺎﺕ‬
‫ ﻤﻊ ﺍﻟﻤﺤﺎﻭﺭ‬α , β , γ ‫ ﺍﻟﺫﻱ ﻴﺼﻨﻊ ﺯﻭﺍﻴﺎ‬OA ‫ﻋﻁﺎﻟﺔ ﺍﻟﺠﺴﻡ ﺍﻟﺼﻠﺏ ﺤﻭل ﻤﺤﻭﺭ‬
:‫ ﺘﻌﻁﻰ ﺒﺎﻟﻌﻼﻗﺔ‬، x،y،z
2 2 2
I  I xx cos α  I yycos β  I zz cos γ  2I xy cosα cosβ
 2I xz cosβ cosγ  2I yz cosβ cosγ

:‫ ﺑﺸﻜﻞ ﻋﺎم ﺑﺎﻟﻌﻼﻗﺔ‬A ‫ ﯾﻌﻄﻰ ﻣﺘﺠﮫ اﻟﻮﺣﺪة ﻓﻲ اﺗﺠﺎه اﻟﻤﺤﻮر‬:‫ﺍﳊﻞ‬


   
n  cosα i  cosβ j  cosγ k


‫ ﻴﻜﻭﻥ ﻋﺯﻡ ﺍﻟﻌﻁﺎﻟﺔ ﺤﻭل ﺍﻟﻤﺤﻭﺭ‬، mi ‫ ﻤﺘﺠﻪ ﻤﻭﻀﻊ ﺍﻟﻜﺘﻠﺔ‬ri ‫ﺇﺫﺍ ﻜﺎﻥ‬
 
d i  ri xn ‫ ﺤﻴﺙ‬mi di2 ‫ﻫﻭ‬OA
:‫ﻴﻨﺘﺞ ﺃﻥ‬
  
i j k
  
ri x n  xi yi zi  y i cos γ  z i cos β i
cos α cos β cos γ
 
 z i cos α  x i cos γ  j  x i cos β  y i cos α k
 2 2 2
 ri  n  yi cosγ  z i cosβ   z i cosα  x i cosγ 
2
 x i cosβ  y i cosα 
    
 yi2  z 2i cos2α  x 2i  z i2 cos2β  x i2  yi2 cos 2γ 
- 2x i yi cosα cosβ - 2x i z i cosα cosγ - 2yi z i cosβ cosγ

- 209 -

Create PDF files without this message by purchasing novaPDF printer (http://www.novapdf.com)
‫ﻭﻴﻌﻁﻰ ﺍﻟﻌﺯﻡ ﺍﻟﻜﻠﻲ ﻟﺠﻤﻴﻊ ﺍﻟﻜﺘل ‪ mi‬ﺒﺎﻟﻌﻼﻗﺔ‪:‬‬
‫‪‬‬
‫‪‬‬
‫‪I  mid2i  mi yi2  z2i‬‬ ‫‪cos α  m x‬‬
‫‪2‬‬
‫‪i‬‬
‫‪2‬‬
‫‪i‬‬ ‫‪‬‬ ‫‪‬‬
‫‪ z2i cos2β‬‬
‫‪i‬‬ ‫‪i‬‬ ‫‪‬‬ ‫‪‬‬ ‫‪i‬‬ ‫‪‬‬
‫‪‬‬ ‫‪‬‬ ‫‪‬‬ ‫‪‬‬
‫‪‬‬ ‫‪‬‬
‫‪ mi xi2  yi2 cos2γ  2 mi xi yi cosα cosβ‬‬
‫‪i‬‬ ‫‪‬‬ ‫‪ i‬‬ ‫‪‬‬
‫‪‬‬ ‫‪‬‬ ‫‪‬‬ ‫‪‬‬
‫‪ 2 mi xi zi cosα cosγ  2 mi yi zi cosβ cosγ‬‬
‫‪ i‬‬ ‫‪‬‬ ‫‪ i‬‬ ‫‪‬‬
‫‪ Ixxcos2α  Iyycos2β  Izzcos2γ‬‬
‫‪ 2Ixycosα cosβ  2Ixzcosα cosγ  2Iyzcosβc osγ‬‬

‫‪ -4‬ﺃﺩﺭﺱ ﺤﺭﻜﺔ ﺠﺴﻡ ﺼﻠﺏ ﻤﺘﻨﺎﻅﺭ ﺒﺎﻟﻨﺴﺒﺔ ﺇﻟﻰ ﻤﺤﻭﺭ‪ ،‬ﻓﻴﻪ ﻨﻘﻁﺔ ﻭﺍﺤﺩﺓ‬
‫ﻤﺜﺒﺘﺔ ﻋﻠﻰ ﺍﻟﻤﺤﻭﺭ‪ .‬ﻋﻠﻤﺎﹰ ﺒﺄﻨﻪ ﻻ ﺘﺅﺜﺭ ﻋﻠﻰ ﺍﻟﺠﺴﻡ ﺴﻭﻯ ﻗﻭﺓ ﺜﻘﻠﻪ ﻭﻗﻭﺓ ﺭﺩ ﺍﻟﻔﻌل‬
‫ﻋﻨﺩ ﻨﻘﻁﺔ ﺍﻟﺘﺜﺒﻴﺕ‪.‬‬
‫ﺍﳊﻞ‪:‬‬
‫ﻨﺨﺘﺎﺭ ﻤﺤﻭﺭ ﺍﻟﺘﻨﺎﻅﺭ ﻤﻨﻁﺒﻘﺎﹰ ﻋﻠﻰ ﺃﺤﺩ ﺍﻟﻤﺤﺎﻭﺭ ﺍﻟﺭﺌﻴﺴﻴﺔ‪ ،‬ﻭﻟﻴﻜﻥ ﺍﻟﻤﺤﻭﺭ‬
‫‪ ،z‬ﻋﻨﺩﺌﺫ‪ ‬ﻴﻜﻭﻥ ‪ ، I1  I 2‬ﻭﺘﺼﺒﺢ ﻤﻌﺎﺩﻻﺕ ﺃﻭﻟﺭ ﺒﺎﻟﺸﻜل ﺍﻟﺘﺎﻟﻲ‪:‬‬
‫‪ 1  I3  I1 ω 2ω 3  0‬‬
‫‪I1ω‬‬
‫‪ 2  I1  I 3 ω3ω1  0‬‬
‫‪(1) I1ω‬‬
‫‪I3ω 3  0‬‬
‫ﻨﺠﺩ ﻤﻥ ﺍﻟﻤﻌﺎﺩﻟﺔ ﺍﻷﺨﻴﺭﺓ ﺃﻥ‪ ، ω 3  const  A :‬ﻭﺘﺼﺒﺢ ﺍﻟﻤﻌﺎﺩﻟﺔ ﺍﻷﻭﻟﻰ‬
‫ﻭﺍﻟﺜﺎﻨﻴﺔ ﺒﻌﺩ ﻗﺴﻤﺘﻬﻤﺎ ﻋﻠﻰ ‪ I1‬ﺒﺎﻟﺼﻭﺭﺓ‪:‬‬
‫‪ω I ‬‬
‫‪ω 1   3 1  Aω 2  0‬‬
‫‪ I1 ‬‬
‫)‪(2‬‬
‫‪ω I ‬‬
‫‪ω 2   1 31 Aω1  0‬‬
‫‪ I1 ‬‬
‫ﺒﺄﺨﺫ ﺘﻔﺎﻀل ﺍﻟﻤﻌﺎﺩﻟﺔ ﺍﻟﺜﺎﻨﻴﺔ ﻓﻲ )‪ ،(2‬ﺒﺎﻟﻨﺴﺒﺔ ﻟﻠﺯﻤﻥ ‪ t‬ﻭﺍﻻﺴﺘﻔﺎﺩﺓ ﻤﻥ‬
‫ﺍﻟﻤﻌﺎﺩﻟﺔ ﺍﻷﻭﻟﻰ‪ ،‬ﻴﻜﻭﻥ‪:‬‬

‫‪- 210 -‬‬

‫)‪Create PDF files without this message by purchasing novaPDF printer (http://www.novapdf.com‬‬
‫‪2‬‬
‫‪I I ‬‬
‫‪ 2   3 1  A 2ω 2  0‬‬
‫‪(3) ω‬‬
‫‪ I1 ‬‬
‫ﺒﻔﺭﺽ ﺃﻥ‪:‬‬
‫)‪(4‬‬
‫‪I 3  I1‬‬
‫‪k‬‬ ‫‪A‬‬
‫‪I1‬‬
‫ﺘﺼﺒﺢ ﺍﻟﻤﻌﺎﺩﻟﺔ )‪ ،(3‬ﺒﺎﻟﺸﻜل‪:‬‬
‫‪ 2  k 2 ω2  0‬‬
‫‪ω‬‬ ‫)‪(5‬‬
‫ﺘﻤﺜل ﺍﻟﻤﻌﺎﺩﻟﺔ )‪ ،(5‬ﻤﻌﺎﺩﻟﺔ ﺘﻔﺎﻀﻠﻴﺔ ﻤﺘﺠﺎﻨﺴﺔ‪ ،‬ﻴﻌﻁﻰ ﺤﻠﻬﺎ ﺒﺎﻟﻌﻼﻗﺔ ﺍﻟﺘﺎﻟﻴﺔ‪:‬‬
‫‪ω 2  Bcoskt  Csinkt‬‬
‫ﺤﻴﺙ ‪ C, B‬ﺜﻭﺍﺒﺕ‪ ،‬ﻨﻌﺘﺒﺭ ﺸﺭﻭﻁ ﺍﺒﺘﺩﺍﺌﻴﺔ ﻫﻲ‪  2  0 :‬ﻋﻨﺩﻤﺎ ‪، t  0‬‬
‫ﻴﺼﺒﺢ ﺍﻟﺤل ﺍﻟﺴﺎﺒﻕ ﺒﺎﻟﺼﻭﺭﺓ‪:‬‬
‫‪(6) ω 2  Csinkt‬‬

‫ﻴﻨﺘﺞ ﻋﻨﺩﺌ ﺫ‪ ‬ﻤﻥ ﺍﻟﻤﻌﺎﺩﻟﺔ ﺍﻟﺜﺎﻨﻴﺔ ﻓﻲ )‪ ،(2‬ﺃﻥ‪:‬‬


‫‪(7) ω1  Ccoskt‬‬
‫ﻭﺘﻜﻭﻥ ﺒﺫﻟﻙ ﺍﻟﺴﺭﻋﺔ ﺍﻟﺯﺍﻭﻴﺔ ﻫﻲ‪:‬‬
‫‪‬‬ ‫‪‬‬ ‫‪‬‬ ‫‪‬‬
‫‪ω  ω1e1  ω 2 e 2  ω3e 3‬‬
‫‪‬‬ ‫‪‬‬ ‫)‪ (8‬‬
‫‪ Ccoskte1  Csinkt e 2  Ae 3‬‬
‫‪  ‬‬
‫‪ e3 , e2 , e1‬ﻤﺘﺠﻬﺎﺕ ﻭﺤﺩﺓ ﻋﻠﻰ ﺍﻟﻤﺤﺎﻭﺭ ‪ z, y, x‬ﻋﻠﻰ ﺍﻟﺘﺭﺘﻴﺏ‪.‬‬

‫‪- 211 -‬‬

‫)‪Create PDF files without this message by purchasing novaPDF printer (http://www.novapdf.com‬‬
‫ﻭﺘﻌﻁﻰ ﻗﻴﻤﺔ ﺍﻟﺴﺭﻋﺔ ﺍﻟﺯﺍﻭﻴﺔ ﺒﺎﻟﻌﻼﻗﺔ‪:‬‬
‫‪ω  C2  A 2‬‬
‫ﻴﻨﺘﺞ ﻤﻥ ﻫﺫﻩ ﺍﻟﻌﻼﻗﺔ ﺃﻥ ﺍﻟﺴﺭﻋﺔ ﺍﻟﺯﺍﻭﻴﺔ ﺜﺎﺒﺘﺔ ﻓﻲ ﺍﻟﻤﻘﺩﺍﺭ ﻭﺒﺎﻟﺘﺎﻟﻲ ﻓﺈﻥ‬
‫ﺍﻟﺠﺴﻡ ﻴﺘﺤﺭﻙ ﺤﺭﻜﺔ ﻤﺒﺎﺩﺭﺓ ﺤﻭل ﺍﻟﻤﺤﻭﺭ ‪ z‬ﺒﺘﺭﺩﺩ ﺯﺍﻭﻱ ﻫﻭ‪:‬‬
‫‪I I‬‬
‫)‪(9‬‬
‫‪k‬‬
‫‪f‬‬ ‫‪ 3 1A‬‬
‫‪2π‬‬ ‫‪2π‬‬

‫‪-5‬ﺍﺩﺭﺱ ﺤﺭﻜﺔ ﺩﻭﺭﺍﻥ ﺍﻷﺭﺽ ﺤﻭل ﻨﻔﺴﻬﺎ ﺒﺎﺴﺘﺨﺩﺍﻡ ﻤﺨﺭﻭﻁ ﺍﻟﻔﻀﺎﺀ‬


‫ﻭﻤﺨﺭﻭﻁ ﺍﻟﺠﺴﻡ‪.‬‬
‫ﺍﳊﻞ‪:‬‬
‫‪‬‬ ‫‪‬‬
‫ﻭﻋﺯﻡ ﻜﻤﻴﺔ ﺍﻟﺤﺭﻜﺔ ﺍﻟﺯﺍﻭﻴﺔ ‪L‬‬ ‫ﺫﻜﺭﻨﺎ ﺴﺎﺒﻘﺎﹰ ﺃﻥ ﺍﻟﺴﺭﻋﺔ ﺍﻟﺯﺍﻭﻴﺔ ‪‬‬
‫ﺘﻌﻁﻴﺎﻥ ﺒﺎﻟﻌﻼﻗﺘﻴﻥ ﺍﻟﺘﺎﻟﻴﺘﻴﻥ‪:‬‬
‫‪‬‬ ‫‪‬‬ ‫‪‬‬ ‫‪‬‬ ‫‪‬‬ ‫‪‬‬ ‫‪‬‬
‫‪ω  ω1e1  ω1e1  ω1e1  Ccoskt e1  sinkt e2   Ae 3‬‬
‫‪‬‬ ‫‪‬‬ ‫‪‬‬ ‫‪‬‬ ‫‪‬‬ ‫‪‬‬ ‫‪‬‬
‫‪L  I1ω1e1  I2ω2e 2  I3ω3e 3  I1Ccoskte1  sinkt e 2   I3Ae3‬‬
‫‪ ‬‬
‫ﺒﻔﺭﺽ ﺃﻥ ‪ θ‬ﻫﻲ ﺍﻟﺯﺍﻭﻴﺔ ﺒﻴﻥ ﺍﻟﻤﺘﺠﻬﻴﻥ ‪ e3‬و ‪ ، L‬ﻴﻜﻭﻥ ﻋﻨﺩﺌﺫ‪:‬‬
‫‪‬‬ ‫‪‬‬ ‫‪‬‬
‫‪ω 3  Ae3  ω 3e3‬‬
‫‪ ‬‬
‫ﻭﻨﺤﺼل ﻤﻥ ﻋﻤﻠﻴﺔ ﺍﻟﻀﺭﺏ ﺍﻟﻘﻴﺎﺴﻲ ﻟﻠﻤﺘﺠﻬﻴﻥ ‪ ω‬و ‪ L‬ﻋﻠﻰ‪:‬‬
‫‪   ‬‬
‫‪ω 3 .L  ω 3 . L cosθ  A I12C 2  I 32 A 2 cosθ  I3 A 2‬‬

‫ﻴﻨﺘﺞ ﺃﻥ‪:‬‬
‫‪I 3A‬‬
‫‪(1) cosθ ‬‬
‫‪I C 2  I 23 A 2‬‬
‫‪2‬‬
‫‪1‬‬
‫‪ ‬‬
‫ﻭﺒﻔﺭﺽ ﺃﻥ ‪ β‬ﻫﻲ ﺍﻟﺯﺍﻭﻴﺔ ﺒﻴﻥ ‪ ω 3‬و‪ ، ω‬ﻨﺠﺩ ﺃﻥ‪:‬‬
‫‪   ‬‬
‫‪ω 3 .ω  ω3 . ω cosβ  A C 2  A 2 cosβ  A 2‬‬
‫ﺃﻭ ﺒﺎﻟﺸﻜل‪:‬‬
‫‪A‬‬
‫‪(2) cosβ ‬‬
‫‪C2  A 2‬‬
‫ﻴﻨﺘﺞ ﻤﻥ ﺍﻟﻤﻌﺎﺩﻟﺘﻴﻥ )‪ (1‬ﻭ)‪ ،(2‬ﺃﻥ‪:‬‬

‫‪- 212 -‬‬

‫)‪Create PDF files without this message by purchasing novaPDF printer (http://www.novapdf.com‬‬
‫‪I1C‬‬ ‫‪C‬‬
‫‪(3) sinθ ‬‬ ‫‪, sinβ ‬‬
‫‪2‬‬ ‫‪2‬‬ ‫‪2‬‬ ‫‪2‬‬
‫‪I C I A‬‬
‫‪1‬‬ ‫‪3‬‬ ‫‪C  A2‬‬
‫‪2‬‬

‫ﻴﻜﻭﻥ ﺒﺫﻟﻙ‪:‬‬
‫‪I1C‬‬ ‫‪C‬‬
‫‪tanθ ‬‬ ‫‪, tanβ ‬‬ ‫)‪(4‬‬
‫‪I3 A‬‬ ‫‪A‬‬
‫ﺃﻭ ﺒﺎﻟﺸﻜل‪:‬‬
‫‪tanθ I1‬‬
‫‪‬‬ ‫)‪(5‬‬
‫‪tanβ I 3‬‬
‫ﺒﻔﺭﺽ ﺃﻥ ﺍﻷﺭﺽ ﺸﺒﻪ ﻜﺭﻭﻴﺔ‪ ،‬ﻴﻜﻭﻥ ‪ I1  I 3‬ﻭﻴﻨﺘﺞ ﺃﻥ‪ . θ  β :‬ﻴﻌﺘﺒﺭ‬
‫‪‬‬
‫ﺍﻟﻤﺨﺭﻭﻁ ﺍﻟﺫﻱ ﻤﺤﻭﺭﻩ ﻓﻲ ﺍﺘﺠﺎﻩ ﻤﺤﻭﺭ ﻜﻤﻴﺔ ﺍﻟﺤﺭﻜﺔ ﺍﻟﺯﺍﻭﻴﺔ ‪ L‬ﺜﺎﺒﺘﺎﹰ ﻓﻲ ﺍﻟﻔﺭﺍﻍ‪،‬‬
‫‪‬‬ ‫‪‬‬
‫ﻭﻴﺴﻤﻰ ﺍﻟﻤﺨﺭﻭﻁ ﺍﻟﻔﺭﺍﻏﻲ‪ ،‬ﺒﻴﻨﻤﺎ ﻴﺴﻤﻰ ﺍﻟﻤﺨﺭﻭﻁ ﺍﻟﺫﻱ ﻤﺤﻭﺭﻩ ‪ω 3  ω3 e3‬‬
‫ﺒﺎﻟﻤﺨﺭﻭﻁ ﺍﻟﺠﺴﻤﻲ ﻭﻴﻌﺘﺒﺭ ﻜﺄﻨﻪ ﻤﺜﺒﺕ ﻋﻠﻰ ﺍﻷﺭﺽ ﻜﻤﺎ ﻫﻭ ﻤﺒﻴﻥ ﻋﻠﻰ ﺍﻟﺸﻜل‬
‫ﺍﻟﻤﺭﺴﻭﻡ‪ .‬ﻴﺘﺩﺤﺭﺝ ﺍﻟﻤﺨﺭﻭﻁ ﺍﻟﺠﺴﻤﻲ ﻋﻠﻰ ﺍﻟﻤﺨﺭﻭﻁ ﺍﻟﻔﺭﺍﻏﻲ ﻭﻴﺤﺩﺩ ﺨﻁ‬
‫‪‬‬
‫ﺍﻟﺘﻼﻤﺱ ﻤﺘﺠﻪ ﺍﻟﺴﺭﻋﺔ ﺍﻟﺯﺍﻭﻴﺔ ‪. ‬‬
‫‪ ‬‬ ‫‪‬‬
‫ﻴﻤﻜﻥ ﺍﻟﺒﺭﻫﺎﻥ ﻋﻠﻰ ﺃﻥ ﺍﻟﻤﺘﺠﻬﺎﺕ ‪ ω , ω 3 , L‬ﺘﻘﻊ ﻓﻲ ﻤﺴﺘﻭﻯ ﻭﺍﺤﺩ ﻭﺫﻟﻙ‬
‫ﻜﻤﺎ ﻴﻠﻲ‪:‬‬

‫‪- 213 -‬‬

‫)‪Create PDF files without this message by purchasing novaPDF printer (http://www.novapdf.com‬‬
‫‪‬‬ ‫‪‬‬ ‫‪‬‬
‫‪e1‬‬ ‫‪e2‬‬ ‫‪e3‬‬
‫‪ ‬‬
‫‪ω3  ω ‬‬ ‫‪0‬‬ ‫‪0‬‬ ‫‪A‬‬
‫‪I1Ccoskt I1Csinkt‬‬ ‫‪I3A‬‬
‫‪ -AICsinkt  ACcoskt‬‬
‫ﻭﻨﺠﺩ ﻤﻥ ﺤﺎﺼل ﺍﻟﻀﺭﺏ ﺍﻟﻤﺨﺘﻠﻁ ﺍﻟﺘﺎﻟﻲ‪:‬‬
‫‪  ‬‬
‫‪L.ω3  ω ‬‬
‫‪‬‬ ‫‪‬‬ ‫‪‬‬ ‫‪‬‬ ‫‪‬‬
‫‪I1Ccoskte1  I1Csinke2  I3Ae3 .  AI1Csinke1  AI1Ccoske2 ‬‬
‫‪ I12AC2sinktcoskt  I12AC2sinktcoskt  0‬‬
‫‪  ‬‬
‫ﻴﻨﺘﺞ ﺃﻥ ﺍﻟﻤﺘﺠﻬﺎﺕ ‪ ω , ω 3 , L‬ﺘﻘﻊ ﻓﻲ ﻤﺴﺘﻭﻯ ﻭﺍﺤﺩ‪ .‬ﻴﺸﺎﻫﺩ ﻤﺭﺍﻗﺏ ﻓﻲ‬
‫‪‬‬
‫ﻨﻅﺎﻡ ﻤﺜﺒﺕ ﻓﻲ ﺍﻟﻔﻀﺎﺀ ﺃﻥ ﺍﻟﻤﺘﺠﻪ ‪ ω‬ﻴﺭﺴﻡ ﻤﺨﺭﻭﻁ ﺍﻟﻔﻀﺎﺀ‪ ،‬ﺒﻴﻨﻤﺎ ﻴﺭﻯ ﻤﺭﺍﻗﺏ‬
‫‪‬‬
‫ﻤﻭﺠﻭﺩ ﻓﻲ ﻨﻅﺎﻡ ﻤﺜﺒﺕ ﻋﻠﻰ ﺍﻟﺠﺴﻡ ﻭﻴﺘﺤﺭﻙ ﻤﻌﻪ ﺃﻥ ﺍﻟﻤﺘﺠﻪ ‪ ω‬ﻴﺭﺴﻡ ﻤﺨﺭﻭﻁ‬
‫ﺍﻟﻔﻀﺎﺀ‪.‬‬
‫ﻴﻜﻭﻥ ﻓﻲ ﺤﺎﻟﺔ ﺍﻷﺭﺽ ﺍﻟﻤﺨﺭﻭﻁ ﺍﻟﻔﻀﺎﺌﻲ ﺩﺍﺨل ﺍﻟﻤﺨﺭﻭﻁ ﺍﻟﺠﺴﻤﻲ ﻜﻤﺎ ﻫﻭ‬
‫ﻤﻭﻀﺢ ﻋﻠﻰ ﺍﻟﺸﻜل ﺍﻟﺴﺎﺒﻕ‪ ،‬ﻷﻥ ‪. I1  I 3‬‬
‫‪ -6‬ﺍﺴﺘﻨﺘﺞ ﻤﻌﺎﺩﻻﺕ ﺍﻟﺤﺭﻜﺔ ﺍﻟﻤﻐﺯﻟﻴﺔ ﻟﺩﻭﺍﻤﺔ ﻓﻴﻬﺎ ﻨﻘﻁﺔ ﻤﺜﺒﺘﺔ‪.‬‬
‫ﺍﳊﻞ‪ :‬ﺒﻔﺭﺽ ﺃﻥ ﺠﻤﻠﺔ ﺍﻹﺤﺩﺍﺜﻴﺎﺕ ‪ xyz‬ﻋﻁﺎﻟﻴﺔ‪ ،‬ﻤﺜﺒﺕ ﻓﻲ ﺍﻟﻨﻘﻁﺔ ‪،0‬‬
‫ﻭﻨﻌﺘﺒﺭ ﻤﺤﺎﻭﺭ ﺠﻤﻠﺔ ﺍﻹﺤﺩﺍﺜﻴﺎﺕ ‪ xyz‬ﻤﺤﺎﻭﺭ ﺭﺌﻴﺴﻴﺔ ﻟﻠﺩﻭﺍﻤﺔ‪ ،‬ﻭﻨﻘﻁﺔ ﺃﺼﻠﻬﺎ‬
‫ﻨﻘﻁﺔ ﺍﻟﺘﻤﺎﺱ ﻤﻊ ﺍﻷﺭﺽ ﻨﻔﺘﺭﻀﻬﺎ ﻨﻔﺱ ﺍﻟﻨﻘﻁﺔ ‪ .0‬ﺘﻘﻊ ﺍﻟﻤﺤﺎﻭﺭ ‪oy , oz  , oz‬‬
‫ﻓﻲ ﻤﺴﺘﻭﻯ ﻭﺍﺤﺩ‪ ،‬ﺘﻜﻭﻥ ﻋﻨﺩﺌﺫ‪ ‬ﺍﻟﻤﺤﺎﻭﺭ ‪ oy , ox , ox‬ﻭﺍﻗﻌﺔ ﻓﻲ ﻤﺴﺘﻭﻯ ﻭﺍﺤﺩ‬
‫ﻭﺫﻟﻙ ﻜﻤﺎ ﻫﻭ ﻤﺒﻴﻥ ﻋﻠﻰ ﺍﻟﺸﻜل ﺍﻟﺘﺎﻟﻲ‪:‬‬

‫‪- 214 -‬‬

‫)‪Create PDF files without this message by purchasing novaPDF printer (http://www.novapdf.com‬‬
‫ﺘﻌﻁﻰ ﺍﻟﺴﺭﻋﺔ ﺍﻟﺯﺍﻭﻴﺔ ﻟﺩﻭﺭﺍﻥ ﻤﺤﺎﻭﺭ ﺍﻟﻨﻅﺎﻡ ‪ xyz‬ﺒﺎﻟﻨﺴﺒﺔ ﻟﻤﺤﺎﻭﺭ ﺠﻤﻠﺔ‬
‫‪ xyz‬ﺒﺎﻟﻌﻼﻗﺔ ﺍﻟﺘﺎﻟﻴﺔ‪:‬‬
‫‪‬‬ ‫‪‬‬ ‫‪‬‬ ‫‪‬‬
‫‪(1) ω  ω1e1  ω 2 e 2  ω3e 3‬‬
‫ﻭﺘﻌﻁﻰ ﻜﻤﻴﺔ ﺍﻟﺤﺭﻜﺔ ﺍﻟﺯﺍﻭﻴﺔ ﺒﺎﻟﻌﻼﻗﺔ‪:‬‬
‫‪‬‬ ‫‪‬‬ ‫‪‬‬ ‫‪‬‬
‫‪‬‬
‫‪L  I1θ e1  I 2 e 2  I 3    cosθ e 3‬‬ ‫‪‬‬
‫ﺃﻭ ﺒﺎﻟﺸﻜل‪:‬‬
‫‪‬‬ ‫‪‬‬ ‫‪‬‬ ‫‪‬‬
‫‪L  I1θ e1  I 2 e 2  I 3   s  e 3‬‬ ‫)‪(2‬‬
‫ﺤﻴﺙ ﻓﺭﻀﻨﺎ ﺃﻥ‪.  cosθ  s :‬‬
‫ﺒﺎﺴﺘﺨﺩﺍﻡ ﻋﻼﻗﺔ ﺍﺸﺘﻘﺎﻕ ﻤﺘﺠﻪ ﻓﻲ ﺠﻤﻠﺔ ﻤﺘﺤﺭﻜ ﺔ ﺒﺎﻟﻨﺴﺒﺔ ﺇﻟﻰ ﺠﻤﻠﺔ ﻤﺜﺒﺘﺔ‪،‬‬
‫ﻨﺠﺩ ﺃﻥ‪:‬‬
‫‪‬‬ ‫‪‬‬
‫‪dL‬‬ ‫‪dL‬‬ ‫‪ ‬‬
‫‪‬‬ ‫‪ωL‬‬ ‫)‪(3‬‬
‫‪dt F dt M‬‬

‫ﻴﻨﺘﺞ ﻤﻥ ﺍﺴﺘﺨﺩﺍﻡ ﺍﻟﻤﻌﺎﺩﻻﺕ )‪ (1‬ﻭ )‪ (2‬ﻭ )‪ ،(3‬ﺃﻥ‪:‬‬


‫‪‬‬
‫‪dL‬‬ ‫‪‬‬
‫‪ I1ω 1  I 3  I 2 ω 2 ω 3  I 3ω 2Se1‬‬
‫‪dt‬‬ ‫‪F‬‬
‫)‪(4‬‬ ‫‪‬‬
‫‪ I 2 ω 2  I1  I 3 ω1ω 3  I 3ω1Se 2‬‬
‫‪‬‬
‫‪ I 3 ω 3  s   I 2  I1 ω1ω 2 e 3‬‬

‫ﻭﻴﻌﻁﻰ ﻋﺯﻡ ﺩﻭﺭﺍﻥ ﻗﻭﺓ ﺍﻟﻭﺯﻥ )ﺍﻟﻘﻭﺓ ﺍﻟﻭﺤﻴﺩﺓ ﺍﻟﺘﻲ ﻟﻬﺎ ﻋﺯﻡ(‪ ،‬ﺤﻭل‬
‫ﺍﻟﻨﻘﻁﺔ ‪ ،0‬ﺒﺎﻟﻌﻼﻗﺔ ﺍﻟﺘﺎﻟﻴﺔ‪:‬‬
‫‪‬‬ ‫‪‬‬ ‫‪‬‬ ‫‪‬‬ ‫‪‬‬
‫‪MO  l e3   mg   l e3    mgk‬‬ ‫‪‬‬ ‫‪‬‬
‫‪‬‬
‫ﻨﺠﺩ ﻤﻥ ﺍﻟﺸﻜل ﺃﻥ ﻤﺘﺠﻪ ﺍﻟﻭﺤﺩﺓ ‪ k‬ﻋﻠﻰ ﺍﻟﻤﺤﻭﺭ ﺍﻟﻤﺜﺒﺕ ‪ z‬ﻴﺴﺎﻭﻱ ﺇﻟﻰ‪:‬‬
‫‪‬‬ ‫‪‬‬ ‫‪‬‬
‫‪(5) k  cosθ e 3  sinθ e 2‬‬
‫ﺒﺎﻟﺘﻌﻭﻴﺽ ﻓﻲ ﺍﻟﻌﻼﻗﺔ ﺍﻟﺴﺎﺒﻘﺔ‪ ،‬ﻴﻌﻁﻰ ﻋﺯﻡ ﺍﻟﺩﻭﺭﺍﻥ ﺒﺎﻟﺸﻜل ﺍﻟﺘﺎﻟﻲ‪:‬‬
‫‪‬‬
‫‪  ‬‬
‫‪(6) MO   mgl e 3  k  mglsinθ e1‬‬
‫‪‬‬

‫‪- 215 -‬‬

‫)‪Create PDF files without this message by purchasing novaPDF printer (http://www.novapdf.com‬‬


‫ﻭﺍﻋﺘﺒﺎﺭ‬ ‫ﺍﻟﺯﺍﻭﻴﺔ‬ ‫ﺍﻟﺤﺭﻜﺔ‬ ‫ﻜﻤﻴﺔ‬ ‫ﻨﻅﺭﻴﺔ‬ ‫ﺒﺎﺴﺘﺨﺩﺍﻡ‬
dL
MO 
dt F

.‫ ﺒﺴﺒﺏ ﺍﻟﺘﻨﺎﻅﺭ‬I1  I 2  I :‫ﺃﻥ‬


:‫ ﺃﻥ‬،(6) ‫( ﻭ‬4) ‫ﻨﺠﺩ ﺒﺎﺴﺘﺨﺩﺍﻡ ﺍﻟﻤﻌﺎﺩﻟﺘﻴﻥ‬
I1ω 1  I3  I1 ω2ω3  I3ω2s  mglsinθ
(7) I1ω 2  I1  I3 ω1ω3  I3ω1s  0
I3 ω 3  s   0
.‫ﻭﻫﻭ ﺍﻟﻤﻁﻠﻭﺏ‬

- 216 -

Create PDF files without this message by purchasing novaPDF printer (http://www.novapdf.com)
‫‪ ‬‬

‫ﺍﻟــﻤــﺮﻭﻧــﺔ‬

‫ﺧﻮﺍﺹ ﻣﺮﻭﻧﺔ ﺍﻷﺟﺴﺎﻡ ﺍﻟﺼﻠﺒﺔ‬


‫‪ELASTIC PROPERTES OF SOLIDS‬‬

‫ﻴﻌﺘﻘﺩ ﺍﻟﺒﻌﺽ ﺃﻥ ﺍﻷﺠﺴﺎﻡ ﺍﻟﺼﻠﺒﺔ ﺘﺤﺎﻓﻅ ﻋﻠﻰ ﺸﻜﻠﻬﺎ ﻋﻨﺩ ﺘﺄﺜﻴﺭ ﻗﻭﻯ‬
‫ﺨﺎﺭﺠﻴﺔ ﻋﻠﻴﻬﺎ‪ ،‬ﺒﻴﻨﻤﺎ ﺍﻟﺤﻘﻴﻘﺔ ﻏﻴﺭ ﺫﻟﻙ‪ ،‬ﺤﻴﺙ ﺃﻥ ﺠﻤﻴﻊ ﺍﻟﻤﻭﺍﺩ ﺘﺘﺸﻭﻩ ﻋﻨﺩﻤﺎ‬
‫ﺘﺨﻀﻊ ﻟﻘﻭﻯ ﺨﺎﺭﺠﻴﺔ‪ ،‬ﻓﻤﻥ ﺍﻟﻤﻤﻜﻥ ﺃﻥ ﻴﺘﻐﻴﺭ ﺸﻜﻠﻬﺎ ﺃﻭ ﻴﺘﻐﻴﺭ ﺤﺠﻤﻬﺎ ﺃﻭ ﻴﺘﻐﻴﺭ‬
‫ﻜل ﻤﻥ ﺍﻟﺤﺠﻡ ﻭﺍﻟﺸﻜل ﻋﻨﺩ ﺘﻁﺒﻴﻕ ﻗﻭﻯ ﺨﺎﺭﺠﻴﺔ ﻋﻠﻴﻬﺎ‪ ،‬ﺒﻴﻨﻤﺎ ﺘﻘﺎﻭﻡ ﺍﻟﻘﻭﻯ‬
‫ﺍﻟﺩﺍﺨﻠﻴﺔ ﻫﺫﺍ ﺍﻟﺘﺸﻭﻩ‪ .‬ﺴﻭﻑ ﻨﻨﺎﻗﺵ ﺘﺸﻭﻩ ﺍﻷﺠﺴﺎﻡ ﺍﻟﺼﻠﺒﺔ ﺍﻨﻁﻼﻗﺎﹰ ﻤﻥ ﺘﻌﺭﻴﻑ‬
‫ﻤﻔﻬﻭﻤﻲ ﺍﻹﺠﻬﺎﺩ ﻭﺍﻻﻨﻔﻌﺎل‪.‬‬
‫ﻴﻌﺭﻑ ﺍﻹﺠﻬﺎﺩ ﺒﺄﻨﻪ ﻤﻘﺩﺍﺭ ﻋﺩﺩﻱ ﻴﺘﻨﺎﺴﺏ ﻤﻊ ﺍﻟﻘﻭﺓ ﺍﻟﺘﻲ ﺘﺴﺒﺏ ﺍﻟﺘﺸﻭﻩ‪،‬‬
‫ﺒﻌﺒﺎﺭﺓ ﺃﺨﺭﻯ ﻨﻌﺭﻑ ﺍﻹﺠﻬﺎﺩ ﺒﺄﻨﻪ ﺍﻟﻨﺴﺒﺔ ﺒﻴﻥ ﺍﻟﻘﻭﺓ ﺍﻟﺨﺎﺭﺠﻴﺔ ‪ F‬ﺇﻟﻰ ﻭﺤﺩﺓ ﻤﺴﺎﺤﺔ‬
‫‪ A‬ﻤﻥ ﺍﻟﺠﺴﻡ ﺍﻟﺘﻲ ﺘﺅﺜﺭ ﻋﻠﻴﻬﺎ ﻫﺫﻩ ﺍﻟﻘﻭﺓ‪ ،‬ﺒﻴﻨﻤﺎ ﻴﻌﺭﻑ ﺍﻻﻨﻔﻌﺎل ﺒﺄﻨﻪ ﻤﻘﻴﺎﺱ ﺩﺭﺠﺔ‬
‫ﺍﻟﺘﺸﻭﻩ‪ ،‬ﻭﻴﺘﻨﺎﺴﺏ ﺍﻻﻨﻔﻌﺎل ﻤﻊ ﺍﻹﺠﻬﺎﺩ ﻭﻴﺴﻤﻰ ﺜﺎﺒﺕ ﺍﻟﺘﻨﺎﺴﺏ ﻤﻌﺎﻤل ﺍﻟﻤﺭﻭﻨﺔ‪،‬‬
‫ﻭﻴﺘﻌﻠﻕ ﻤﻌﺎﻤل ﺍﻟﻤﺭﻭﻨﺔ ﺒﻨﻭﻉ ﺍﻟﻤﺎﺩﺓ ﻭﺒﻁﺒﻴﻌﺔ ﺍﻟﺘﺸﻭﻩ‪ ،‬ﻭﻴﻌﻁﻰ ﺒﺎﻟﻌﻼﻗﺔ ﺍﻟﺘﺎﻟﻴﺔ‪:‬‬
‫ﻤﻌﺎﻤل ﺍﻟﻤﺭﻭﻨﺔ = ﺍﻹﺠﻬﺎﺩ ﻋﻠﻰ ﺍﻻﻨﻔﻌﺎل‬ ‫)‪(7.1‬‬
‫ﻨﺘﻌﺭ‪‬ﻑ ﻋﻠﻰ ﺜﻼﺜﺔ ﺃﻨﻭﺍﻉ ﻤﻥ ﺍﻟﺘﺸﻭﻫﺎﺕ‪ ،‬ﻭﻨﻌﺭ‪‬ﻑ ﻤﻌﺎﻤل ﺍﻟﻤﺭﻭﻨﺔ ﻟﻜل‬
‫ﻨﻭﻉ‪.‬‬
‫‪ .1‬ﻤﻌﺎﻤل ﻴﻭﻨﻎ )‪ ،(Young's modulus‬ﻴﻘﻴﺱ ﻫﺫﺍ ﺍﻟﻤﻌﺎﻤل ﻤﻘﺎﻭﻤﺔ ﺍﻟﻤﻭﺍﺩ‬
‫ﺍﻟﺼﻠﺒﺔ ﻋﻨﺩ ﺍﻟﺘﻐﻴﻴﺭ ﻓﻲ ﻁﻭﻟﻬﺎ‪.‬‬

‫‪- 217 -‬‬

‫)‪Create PDF files without this message by purchasing novaPDF printer (http://www.novapdf.com‬‬
‫‪ .2‬ﻤﻌﺎﻤل ﺍﻟﻘﺹ )‪ ،(Shear modulus‬ﻴﻘﻴﺱ ﻫﺫﺍ ﺍﻟﻤﻌﺎﻤل ﺤﺭﻜﺔ ﺍﻨﺯﻻﻕ‬
‫ﻤﺴﺘﻭﻴﺎﺕ ﺍﻷﺠﺴﺎﻡ ﺍﻟﺼﻠﺒﺔ ﻓﻭﻕ ﺒﻌﻀﻬﺎ‪.‬‬
‫‪ .3‬ﻤﻌﺎﻤل ﺘﻐﻴﺭ ﺍﻟﺤﺠﻡ )‪ ،(Bulk modulus‬ﻴﻘﻴﺱ ﻫﺫﺍ ﺍﻟﻤﻌﺎﻤل ﻤﻘﺎﻭﻤﺔ‬
‫ﺍﻷﺠﺴﺎﻡ ﺍﻟﺼﻠﺒﺔ ﺃﻭ ﻤﻘﺎﻭﻤﺔ ﺍﻟﺴﻭﺍﺌل ﻟﺘﻐﻴﻴﺭ ﺤﺠﻤﻬﺎ‪.‬‬

‫‪ - 51‬ﻣﻌﺎﻣﻞ ﻳﻮﻧﻎ ‪Young's Modulus :‬‬


‫ﺒﻔﺭﺽ ﺃﻥ ﻗﻀﻴﺒﺎﹰ ﻁﻭﻟﻪ ﺍﻷﺼﻠﻲ ‪ L i‬ﻭﻤﺴﺎﺤﺔ ﻤﻘﻁﻌﻪ ‪ .A‬ﺜﺒﺕ ﺃﺤﺩ ﻁﺭﻓﻴﻪ‬
‫ﻜﻤﺎ ﻋﻠﻰ ﺍﻟﺸﻜل )‪ (7.1‬ﻨﻁﺒﻕ ﻗﻭﺓ ﻋﻤﻭﺩﻴﺔ ﻋﻠﻰ ﻤﻘﻁﻌﻪ ﺍﻟﻌﺭﻀﻲ ﻤﻥ ﺠﻬﺔ ﻁﺭﻓﻪ‬
‫ﺍﻟﺤﺭ‪ ،‬ﺘﻘﺎﻭﻡ ﺍﻟﻘﻭﻯ ﺍﻟﺩﺍﺨﻠﻴﺔ ﻓﻲ ﺍﻟﻘﻀﻴﺏ ﺍﻟﺘﺸﻭﻩ‪ .‬ﻴﺘﻭﺍﺯﻥ ﺍﻟﻘﻀﻴﺏ ﻋﻨﺩﻤﺎ ﻴﺼﺒﺢ‬
‫ﻁﻭﻟﻪ ‪ ،Lf‬ﺤﻴﺙ ﺃﻥ ‪ L f  L i‬ﻭﺘﺘﻭﺍﺯﻥ ﺍﻟﻘﻭﻯ ﺍﻟﺨﺎﺭﺠﻴﺔ ﻤﻊ ﺍﻟﻘﻭﻯ ﺍﻟﺩﺍﺨﻠﻴﺔ‪.‬‬

‫ﺍﻟﺸﻜل ‪ 7.1‬ﺯﻴﺎﺩﺓ ﻁﻭل ﻗﻀﻴﺏ ﻤﺜﺒﺕ ﻤﻥ ﺃﺤﺩ ﻁﺭﻓﻴﻪ ﺒﺎﻟﻤﻘﺩﺍﺭ ‪ ΔL‬ﻨﺘﻴﺠﺔ ﺘﺄﺜﻴﺭ ﻗﻭﺓ ‪F‬‬

‫ﻨﻭﺠﺩ ﺇﺠﻬﺎﺩ ﺍﻟﺸﺩ ﻓﻲ ﺍﻟﻘﻀﻴﺏ ﻭﺫﻟﻙ ﻤﻥ ﺤﺴﺎﺏ ﻨﺴﺒﺔ ﺸﺩﺓ ﺍﻟﻘﻭﺓ ﺍﻟﺨﺎﺭﺠﻴﺔ‬
‫‪ F‬ﺇﻟﻰ ﻤﺴﺎﺤﺔ ﻤﻘﻁﻌﻪ ‪ .A‬ﻴﻤﻜﻨﻨﺎ ﺤﺴﺎﺏ ﺍﻨﻔﻌﺎل ﺍﻟﺸﺩ ﻤﻥ ﺤﺴﺎﺏ ﻨﺴﺒﺔ ﺍﻟﺘﻐﻴﺭ ﻓﻲ‬
‫ﻭﺘﺴﻤﻰ ﻫﺫﻩ ﺍﻟﻨﺴﺒﺔ ﺍﻟﺘﺸﻭﻩ(‪.‬‬ ‫)ﺃﻱ‬ ‫ﺍﻟﻁﻭل ‪ ΔL‬ﺇﻟﻰ ﺍﻟﻁﻭل ﺍﻷﺼﻠﻲ ‪Li‬‬
‫‪ΔL‬‬
‫‪Li‬‬
‫ﻴﻌﺭﻑ ﺃﺨﻴﺭﺍﹰ ﻤﻌﺎﻤل ﻴﻭﻨﻎ ﺒﺄﻨﻪ ﻨﺴﺒﺔ ﺍﻹﺠﻬﺎﺩ ﺇﻟﻰ ﺍﻻﻨﻔﻌﺎل ﻓﻲ ﺍﻟﺸﺩ‪.‬‬
‫‪F‬‬
‫ﻤﻌﺎﻤل ﻴﻭﻨﻎ‬ ‫‪(7.2) Y ‬‬ ‫‪A‬‬
‫‪ΔL‬‬
‫‪Li‬‬

‫‪- 218 -‬‬

‫)‪Create PDF files without this message by purchasing novaPDF printer (http://www.novapdf.com‬‬
‫ﻴﺴﺘﺨﺩﻡ ﻤﻌﺎﻤل ﻴﻭﻨﻎ ﻜﻨﻤﻭﺫﺝ ﻟﻭﺼﻑ ﺇﺠﻬﺎﺩ ﻗﻀﻴﺏ ﺃﻭ ﺴﻠﻙ ﺭﻓﻴﻊ ﺘﺤﺕ‬
‫ﺘﺄﺜﻴﺭ ﺍﻟﺸﺩ ﺃﻭ ﺍﻟﻀﻐﻁ‪ ،‬ﻷﻥ ﺍﻻﻨﻔﻌﺎل ﻜﻤﺎ ﻫﻭ ﻭﺍﻀﺢ ﻜﻤﻴﺔ ﻋﺩﺩﻴﺔ ﺒﺩﻭﻥ ﻭﺍﺤﺩﺓ‪،‬‬
‫ﻭﺒﺎﻟﺘﺎﻟﻲ ﻓﺈﻥ ﻭﺍﺤﺩﺓ ﻗﻴﺎﺱ ﻤﻌﺎﻤل ﻴﻭﻨﻎ ‪ Y‬ﻫﻲ ﻭﺍﺤﺩﺓ ﻗﻴﺎﺱ ﺍﻟﻘﻭﺓ ‪ F‬ﻟﻜل ﻭﺍﺤﺩﺓ‬
‫ﻤﺴﺎﺤﺔ‪ .‬ﻴﺒﻴﻥ ﺍﻟﺠﺩﻭل )‪ ،(7.1‬ﺒﻌﺽ ﺍﻟﻘﻴﻡ ﺍﻟﻨﻤﻭﺫﺠﻴﺔ ﻟﻠﻤﻌﺎﻤﻼﺕ ﺍﻟﺜﻼﺙ ﺍﻟﺴﺎﺒﻘﺔ‪،‬‬
‫ﻫﺫﺍ ﻭﺘﺒﻴﻥ ﺍﻟﺘﺠﺭﺒﺔ ﺃﻨﻪ‪:‬‬
‫أ‪ -‬ﻋﻨﺩ ﺘﻁﺒﻴﻕ ﻗﻭﺓ ﺜﺎﺒﺘﺔ ﻋﻠﻰ ﺠﺴﻡ ﺼﻠﺏ ﻁﻭﻴل‪ ،‬ﻓﺈﻥ ﺍﻟﺘﻐﻴﺭ ﻓﻲ ﺍﻟﻁﻭل‬
‫ﻴﺘﻨﺎﺴﺏ ﻁﺭﺩﺍﹰ ﻤﻊ ﺍﻟﻁﻭل ﺍﻷﺼﻠﻲ‪.‬‬
‫ب‪ -‬ﻴﺘﻨﺎﺴﺏ ﺍﻻﻨﻔﻌﺎل ﺍﻟﻨﺎﺘﺞ ﻋﻥ ﺘﺄﺜﻴﺭ ﺍﻟﻘﻭﺓ ﻁﺭﺩﺍﹰ ﻤﻊ ﻤﺴﺎﺤﺔ ﺍﻟﻤﻘﻁﻊ‪.‬‬
‫ﺘﻭﻀﺢ ﺍﻟﻤﻌﺎﺩﻟﺔ)‪ ،(7.2‬ﻫﺫﻴﻥ ﺍﻟﺸﺭﻁﻴﻥ‪.‬‬
‫ﺍﻟﺠﺩﻭل )‪ ،(7.1‬ﻴﻅﻬﺭ ﻗﻴﻡ ﻤﻌﺎﻤل ﺍﻟﻤﺭﻭﻨﺔ ﻟﺒﻌﺽ ﺍﻷﺠﺴﺎﻡ‪:‬‬
‫ﻤﻌﺎﻤل ﺘﻐﻴﺭ ﺍﻟﺤﺠﻡ‬ ‫ﻤﻌﺎﻤل ﺍﻟﻘﺹ‬ ‫‪ N‬ﻤﻌﺎﻤل ﻴﻭﻨﻎ(‬ ‫)‬ ‫ﺍﻟﻤﺎﺩﺓ‬
‫‪m2‬‬
‫‪20x1010‬‬ ‫‪14x1010‬‬ ‫‪35x1010‬‬ ‫ﺘﻨﻐﺴﺘﻥ‬
‫‪10‬‬ ‫‪10‬‬ ‫‪10‬‬
‫‪6x10‬‬ ‫‪8.4x10‬‬ ‫‪20x10‬‬ ‫ﻓﻭﻻﺫ‬
‫‪10‬‬ ‫‪10‬‬ ‫‪10‬‬
‫‪14x10‬‬ ‫‪4.2x10‬‬ ‫‪11x10‬‬ ‫ﻨﺤﺎﺱ‬
‫‪10‬‬ ‫‪10‬‬ ‫‪10‬‬
‫‪6.1x10‬‬ ‫‪3.5x10‬‬ ‫‪9.1x10‬‬ ‫ﻨﺤﺎﺱ ﺃﺼﻔﺭ‬
‫‪10‬‬ ‫‪10‬‬ ‫‪10‬‬
‫‪7x10‬‬ ‫‪2.5x10‬‬ ‫‪7x10‬‬ ‫ﺃﻟﻤﻨﻴﻭﻡ‬
‫‪10‬‬ ‫‪10‬‬
‫‪(5-5.5)x10‬‬ ‫‪(2.6-‬‬ ‫‪(6.5-7.8)x10‬‬ ‫ﺯﺠﺎﺝ‬
‫‪3.2)x1010‬‬
‫‪(5-5.5)x1010‬‬ ‫‪2.6x1010‬‬ ‫‪5.6x1010‬‬ ‫ﻜﻭﺍﺭﺘﺯ‬
‫‪0.21x1010‬‬ ‫‪-‬‬ ‫‪-‬‬
‫ﻤﺎﺀ‬
‫‪10‬‬
‫‪2.81x10‬‬ ‫‪-‬‬ ‫‪-‬‬ ‫ﺯﺌﺒﻕ‬

‫‪ - 52‬ﺣﺪ ﺍﳌﺮﻭﻧﺔ‪The elastic Limit :‬‬


‫ﻴﻌﺭﻑ ﺤﺩ ﺍﻟﻤﺭﻭﻨﺔ ﻟﻤﺎﺩﺓ ﺒﺄﻨﻪ ﺍﻹﺠﻬﺎﺩ ﺍ ﻷﻋﻅﻤﻲ ﺍﻟﺫﻱ ﻴﻤﻜﻥ ﺘﻁﺒﻴﻘﻪ ﻋﻠﻰ‬
‫ﺍﻟﻤﺎﺩﺓ ﻗﺒل ﺃﻥ ﺘﺒﺩﺃ ﺒﺎﻟﺘﺸﻭﻩ‪.‬‬

‫‪- 219 -‬‬

‫)‪Create PDF files without this message by purchasing novaPDF printer (http://www.novapdf.com‬‬
‫ﺍﻟﺸﻜل )‪ ،(7.2‬ﺍﻹﺠﻬﺎﺩ ﺒﺩﻻﻟﺔ ﺍﻻﻨﻔﻌﺎل ﻟﺠﺴﻡ ﺼﻠﺏ ﻤﺭﻥ‪.‬‬

‫ﻤﻥ ﺍﻟﻤﻤﻜﻥ ﺘﺠﺎﻭﺯ ﺤﺩ ﺍﻟﻤﺭﻭﻨﺔ ﻟﻤﺎﺩﺓ ﺇﺫﺍ ﻁﺒﻘﻨﺎ ﺇﺠﻬﺎﺩ ﻜﺒﻴﺭ ﻜﻔﺎﻴﺔ‪ ،‬ﻭﻴﺒﻴﻥ‬
‫ﺍﻟﺸﻜل )‪ ،(7.2‬ﺃﻥ ﺍﻟﻤﻨﺤﻨﻰ ﻴﻤﺜل ﺘﺯﺍﻴﺩ ﺍﻹﺠﻬﺎﺩ ﺒﺘﺯﺍﻴﺩ ﺍﻻﻨﻔﻌﺎل‪ ،‬ﻭﻋﻨﺩﻤﺎ ﻴﺘﺠﺎﻭﺯ‬
‫ﺍﻹﺠﻬﺎﺩ ﺤﺩ ﺍﻟﻤﺭﻭﻨﺔ‪ ،‬ﻓﺈﻥ ﺍﻟﺠﺴﻡ ﻴﺘﺸﻭﻩ ﺒﺎﺴﺘﻤﺭﺍﺭ ﻭﻻ ﻴﻌﻭﺩ ﺇﻟﻰ ﺍﻟﺸﻜل ﺍﻷﺼﻠﻲ‬
‫ﺤﺘﻰ ﺒﻌﺩ ﺯﻭﺍل ﺍﻹﺠﻬﺎﺩ‪ .‬ﻭﻋﻨﺩ ﺍﻻﺴﺘﻤﺭﺍﺭ ﺒﺯﻴﺎﺩﺓ ﺍﻹﺠﻬﺎﺩ ﻓﺈﻥ ﺍﻟﺠﺴﻡ ﻴﻨﻜﺴﺭ ﺃﻭ‬
‫ﻴﺘﺤﻁﻡ‪.‬‬
‫‪ - 53‬ﻣﻌﺎﻣﻞ ﺍﻟﻘﺺ ‪Shear Modulus :‬‬
‫ﻨﺫﻜﺭ ﻨﻭﻉ ﺁﺨﺭ ﻤﻥ ﺍﻟﺘﺸﻭﻫﺎﺕ ﺘﺤﺩﺙ ﻟﻸﺠﺴﺎﻡ ﺍﻟﺘﻲ ﺘﺨﻀﻊ ﻟﻘﻭﺓ ﻤﻤﺎﺴﻴﺔ‬
‫ﻋﻠﻰ ﺃﺤﺩ ﺃﻭﺠﻬﻬﺎ ﺒﻴﻨﻤﺎ ﻴﻜﻭﻥ ﺍﻟﻭﺠﻪ ﺍﻵﺨﺭ ﻤﺜﺒﺕ ﺒﻭﺍﺴﻁﺔ ﻗﻭﺓ ﺃﺨﺭﻯ ﻜﻤﺎ ﻫﻭ‬
‫ﻤﺒﻴﻥ ﻋﻠﻰ ﺍﻟﺸﻜل ) ‪ ،(7.3a‬ﻴﺩﻋﻰ ﺍﻹﺠﻬﺎﺩ ﻓﻲ ﻫﺫﻩ ﺍﻟﺤﺎﻟﺔ ﺇﺠﻬﺎﺩ ﺍﻟﻘﺹ‪.‬‬
‫ﻻ ﻴﺼﺎﺤﺏ ﺇﺠﻬﺎﺩ ﺍﻟﻘﺹ ﺘﻐﻴﺭ ﻓﻲ ﺍﻟﺤﺠﻡ‪ ،‬ﻭﻴﻌﺭﻑ ﺇﺠﻬﺎﺩ ﺍﻟﻘﺹ ﺒﺄﻨﻪ ﻨﺴﺒﺔ‬
‫ﺍﻟﻘﻭﺓ ﺍﻟﻤﻤﺎﺴﻴﺔ ‪ F‬ﺇﻟﻰ ﻤﺴﺎﺤﺔ ﻭﺠﻪ ﺍﻟﻘﺹ ‪ ، A‬ﻭﻴﻌﺭﻑ ﺍﻨﻔﻌﺎل ﺍﻟﻘﺹ ﺒﺄﻨﻪ ﺍﻟﻨﺴﺒﺔ‬

‫‪- 220 -‬‬

‫)‪Create PDF files without this message by purchasing novaPDF printer (http://www.novapdf.com‬‬
‫‪ ،‬ﺤﻴﺙ ‪ Δx‬ﺍﻟﻤﺴﺎﻓﺔ ﺍﻷﻓﻘﻴﺔ ﺍﻟﺘﻲ ﻴﺘﺤﺭﻜﻬﺎ ﻭﺠﻪ ﺍﻟﻘﺹ )ﺍﻟﻭﺠﻪ ﺍﻟﺨﺎﻀﻊ ﻟﻠﻘﻭﺓ‬
‫‪Δx‬‬
‫‪h‬‬
‫‪ (F‬ﻭ‪ h‬ﺍﺭﺘﻔﺎﻉ ﺍﻟﺠﺴﻡ‪.‬‬

‫ﺍﻟﺸﻜل‪(7.3) :‬‬
‫‪ (a‬ﺘﺸﻭﻩ ﺍﻟﻘﺹ ﺒﻭﺍﺴﻁﺔ ﻗﻭﺘﻴﻥ ﻤﺘﺴﺎﻭﻴﺘﻴﻥ ﺒﺎﻟﻘﻴﻤﺔ ﻭﻤﺘﻌﺎﻜﺴﺘﻴﻥ ﺒﺎﻻﺘﺠﺎﻩ ﺘﺅﺜﺭﺍﻥ ﻋﻠﻰ‬
‫ﻭﺠﻬﻴﻥ ﻤﺘﻭﺍﺯﻴﻴﻥ‪ (b.‬ﻜﺘﺎﺏ ﺘﺤﺕ ﺇﺠﻬﺎﺩ ﺍﻟﻘﺹ‪.‬‬

‫ﻴﻌﻁﻰ ﺃﺨﻴﺭﺍﹰ ﻤﻌﺎﻤل ﺍﻟﻘﺹ ﺒﺎﻟﻌﻼﻗﺔ ﺍﻟﺘﺎﻟﻴﺔ‪:‬‬


‫‪F‬‬
‫‪S‬‬ ‫‪A‬‬ ‫)‪(7.3‬‬
‫‪Δx‬‬
‫‪h‬‬
‫ﻴﻅﻬﺭ ﺍﻟﺠﺩﻭل‪ ،(7.1) :‬ﺒﻌﺽ ﻗﻴﻡ ﻤﻌﺎﻤل ﺍﻟﻘﺹ ﻟﺒﻌﺽ ﺍﻟﻤﻭﺍﺩ ﻭﻭﺤﺩﺓ‬
‫‪.N‬‬‫ﻗﻴﺎﺴﻪ ﻫﻲ ﻭﺤﺩﺓ ﻗﻴﺎﺱ ﺍﻟﻘﻭﺓ ﻋﻠﻰ ﻭﺤﺩﺓ ﻗﻴﺎﺱ ﺍﻟﻤﺴﺎﺤﺔ ﺃﻱ ‪ m ‬‬
‫‪2‬‬

‫‪- 221 -‬‬

‫)‪Create PDF files without this message by purchasing novaPDF printer (http://www.novapdf.com‬‬
‫‪ - 54‬ﻣﻌﺎﻣﻞ ﺗﻐﲑ ﺍﳊﺠﻢ ‪Bulk Modulus:‬‬
‫ﻴﺼﻑ ﻤﻌﺎﻤل ﺘﻐﻴﺭ ﺍﻟﺤﺠﻡ ﺍﺴﺘﺠﺎﺒﺔ ﺍﻟﺠﺴﻡ ﻟﺘﻐﻴﺭ ﺍﻟﻀﻐﻁ ﻋﻨﺩﻤﺎ ﻴﻭﻀﻊ ﻓﻲ‬
‫ﻓﺭﺍﻍ ﺠﺯﺌﻲ‪ .‬ﺒﻔﺭﺽ ﺃﻥ ﻗﻭﻯ ﺨﺎﺭﺠﻴﺔ ﺘﺅﺜﺭ ﺒﺸﻜل ﻤﻨﺘﻅﻡ ﻋﻠﻰ ﻜل ﺃﻭﺠﻪ ﺍﻟﺠﺴﻡ‬
‫ﻜﻤﺎ ﻴﻅﻬﺭ ﻋﻠﻰ ﺍﻟﺸﻜل )‪ ،(7.4‬ﻴﺤﺩﺙ ﻤﺜل ﻫﺫﺍ ﺍﻟﺘﺄﺜﻴﺭ ﺍﻟﻤﻨﺘﻅﻡ ﻟﻠﻘﻭﻯ ﻋﻨﺩﻤﺎ‬
‫ﻴﻭﻀﻊ ﺍﻟﺠﺴﻡ ﻓﻲ ﺴﺎﺌل ﺃﻭ ﻤﺎﺌﻊ‪ .‬ﻴﺘﻐﻴﺭ ﺤﺠﻡ ﺍﻟﺠﺴﻡ ﻓﻲ ﻫﺫﻩ ﺍﻟﺤﺎﻟﺔ ﺩﻭﻥ ﺃﻥ ﻴﺭﺍﻓﻕ‬
‫ﺫﻟﻙ ﺘﻐﻴﺭ ﻋﻠﻰ ﺍﻟﺸﻜل‪.‬‬

‫ﺍﻟﺸﻜل)‪ ،(7.4‬ﺠﺴﻡ ﺼﻠﺏ ﺘﺤﺕ ﻀﻐﻁ ﻤﻨﺘﻅﻡ‪ ،‬ﺤﻴﺙ ﻴﻌﺎﻨﻲ ﺘﻐﻴﺭﺍﹰً ﻓﻲ ﺍﻟﺤﺠﻡ ﺩﻭﻥ ﺃﻥ ﻴﺘﻐﻴﺭ‬
‫ﺸﻜﻠﻪ ﻨﺘﻴﺠﺔ ﺘﺄﺜﻴﺭ ﻗﻭﻯ ﻋﻤﻭﺩﻴﺔ ﻋﻠﻰ ﻭﺠﻭﻫﻪ ﺍﻟﺴﺘﺔ ‪.‬‬

‫ﻴﺘﺤﺩﺩ ﺇﺠﻬﺎﺩ ﺍﻟﺤﺠﻡ )‪ (Volume Stress‬ﺒﺄﻨﻪ ﻨﺎﺘﺞ ﻗﺴﻤﺔ ﺍﻟﻤﺭﻜﺒﺔ ﺍﻟﻌﻤﻭﺩﻴﺔ‬


‫‪ P ‬ﺒﺎﻟﻀﻐﻁ‪ ،،‬ﻓﺈﺫﺍ ﺘﻐﻴﺭ ﺍﻟﻀﻐﻁ‬ ‫ﻟﻠﻘﻭﺓ ‪ F‬ﻋﻠﻰ ﺍﻟﻤﺴﺎﺤﺔ ‪ ،A‬ﻭﺘﺩﻋﻰ ﺍﻟﻜﻤﻴﺔ‬
‫‪F‬‬
‫‪A‬‬
‫‪ ، ΔP ‬ﻓﺈﻥ ﺤﺠﻡ ﺍﻟﺠﺴﻡ ﺴﻴﻌﺎﻨﻲ ﺘﻐﻴﺭ ﺍﹰ‬ ‫ﺍﻟﻤﺅﺜﺭ ﻋﻠﻰ ﺍﻟﺠﺴﻡ ﺒﺎﻟﻤﻘﺩﺍﺭ‬
‫‪ΔF‬‬
‫‪A‬‬
‫ﺒﺎﻟﻤﻘﺩﺍﺭ ‪ ، ΔV‬ﻭﻴﺴﺎﻭﻱ ﺍﻨﻔﻌﺎل ﺍﻟﺤﺠﻡ ) ‪ (Volume Strain‬ﺘﻐﻴﺭ ﺍﻟﺤﺠﻡ ‪ΔV‬‬
‫ﻤﻘﺴﻭﻤﺎﹰ ﻋﻠﻰ ﺍﻟﺤﺠﻡ ﺍﻻﺒﺘﺩﺍﺌﻲ ‪ ، Vi‬ﺒﺎﻟﺘﻌﻭﻴﺽ ﻓﻲ ﺍﻟﻤﻌﺎﺩﻟﺔ )‪ ،(7.1‬ﻨﺤﺼل ﻋﻠﻰ‬
‫ﺘﻌﺭﻴﻑ ﻤﻌﺎﻤل ﺘﻐﻴﺭ ﺍﻟﺤﺠﻡ ﺒﺎﻟﺼﻭﺭﺓ ﺍﻵﺘﻴﺔ‪:‬‬

‫‪- 222 -‬‬

‫)‪Create PDF files without this message by purchasing novaPDF printer (http://www.novapdf.com‬‬
‫‪ΔF‬‬
‫‪B ‬‬ ‫‪A   Δp‬‬ ‫)‪(7.4‬‬
‫‪ΔV‬‬ ‫‪ΔV‬‬
‫‪Vi‬‬ ‫‪Vi‬‬
‫ﺃﺩﺨﻠﺕ ﺇﺸﺎﺭﺓ ﺍﻟﺴﺎﻟﺏ ﻓﻲ ﻫﺫﺍ ﺍﻟﺘﻌﺭﻴﻑ ﻟﺘﺒﻘﻲ ﺍﻟﻌﺩﺩ ‪ B‬ﻤﻭﺠﺒﺎﹰ‪ ،‬ﻷﻨﻪ ﻋﻨﺩ‬
‫ﺯﻴﺎﺩﺓ ﺍﻟﻀﻐﻁ‪ ،‬ﻓﺈﻥ ﺍﻟﺘﻐﻴﺭ ﻓﻲ ﺍﻟﻀﻐﻁ ‪ P‬ﻴﻜﻭﻥ ﻤﻭﺠﺒﺎﹰ‪ ،‬ﻏﻴﺭ ﺃﻥ ﺍﻟﺤﺠﻡ‬
‫ﺴﻴﺘﻨﺎﻗﺹ ﻭﻴﻜﻭﻥ ﺍﻟﺘﻐﻴﺭ ﻓﻲ ﺍﻟﺤﺠﻡ ‪ V‬ﺴﺎﻟﺒﺎﹰ ﻭﺍﻟﻌﻜﺱ ﺒﺎﻟﻌﻜﺱ‪ .‬ﻴﺘﺒﻴﻥ ﻤﻥ‬
‫ﺍﻟﺠﺩﻭل )‪ ،(7.1‬ﺇﻥ ﻟﻸﺠﺴﺎﻡ ﺍﻟﺼﻠﺒﺔ ﻭﺍﻟﺴﺎﺌﻠﺔ ﻤﻌﺎﻤل ﺤﺠﻡ ﺒﻴﻨﻤﺎ ﻟﻴﺱ ﻟﻠﺴﻭﺍﺌل‬
‫ﻤﻌﺎﻤل ﻴﻭﻨﻎ ﻭﻤﻌﺎﻤل ﻗﺹ ﻷﻨﻪ ﻻ ﻴﻭﺠﺩ ﻟﻠﺴﻭﺍﺌل ﺇﺠﻬﺎﺩ ﻗﺹ ﺃﻭ ﺇﺠﻬﺎﺩ ﺸﺩ‪.‬‬
‫ﻣﺜﺎﻝ‪:1‬‬
‫ﺘﻁﺒﻕ ﻋﻠﻰ ﻜﺒل ﻁﻭﻟﻪ ‪ ،10 m‬ﻗﻭﺓ ﺸﺩ ﻤﻘﺩﺍﺭﻫﺎ ‪ 940 N‬ﻓﻴﺯﺩﺍﺩ ﻁﻭﻟﻪ‬
‫ﺒﻤﻘﺩﺍﺭ ‪ ،0.5 cm‬ﺍﻟﻤﻁﻠﻭﺏ‪ :‬ﺇﻴﺠﺎﺩ ﻗﻁﺭ ﺍﻟﻜﺒل‪ ،‬ﻋﻠﻤﺎﹰ ﺃﻥ ﻤﻌﺎﻤل ﻴﻭﻨﻎ ﻟﻤﺎﺩﺓ ﺍﻟﻜﺒل‬
‫‪Y  20x10 10 N‬‬ ‫ﻫﻭ‬
‫‪m2‬‬
‫ﺍﳊﻞ ‪:‬‬
‫ﺘﻌﻁﻰ ﻤﺴﺎﺤﺔ ﻤﻘﻁﻊ ﺍﻟﺴﻠﻙ ﻤﻥ ﺘﻌﺭﻴﻑ ﻤﻌﺎﻤل ﻴﻭﻨﻎ‪.‬‬
‫‪F‬‬
‫‪Y‬‬ ‫‪A  A  FL i ‬‬ ‫‪940N 10m ‬‬ ‫‪ 9.4x10 6 m 2‬‬
‫‪ΔL‬‬ ‫‪YL ‬‬ ‫‪10‬‬ ‫‪‬‬
‫‪Li‬‬ ‫‪ 20x10 N 2 0.005m ‬‬
‫‪‬‬ ‫‪m ‬‬
‫ﺒﻤﺎ ﺃﻥ ﻤﻘﻁﻊ ﺍﻟﻜﺒل ﺩﺍﺌﺭﺓ‪ ،‬ﺘﻜﻭﻥ ﻤﺴﺎﺤﺔ ﻤﻘﻁﻌﻪ ﻤﺴﺎﻭﻴﺔ ﺇﻟﻰ‪:‬‬
‫‪A  π r2‬‬
‫‪A‬‬ ‫‪9.4x10  6 m 2‬‬
‫‪r‬‬ ‫‪‬‬ ‫‪ 1.7x10  3 m‬‬
‫‪π‬‬ ‫‪π‬‬
‫ﻴﻜﻭﻥ ﻗﻁﺭ ﺍﻟﻜﺒل‪:‬‬
‫‪‬‬ ‫‪‬‬
‫‪d  2r  2 1.7x10 3 m  3.4x10 3 m  3.4mm‬‬
‫ﻣﺜﺎﻝ‪:2‬‬
‫‪) 1x10 5 N‬ﺍﻟﻀﻐﻁ‬ ‫ﺘﻭﻀﻊ ﻜﺭﺓ ﺼﻠﺒﺔ ﻤﻥ ﺍﻟﻨﺤﺎﺱ ﻓﻲ ﻫﻭﺍﺀ ﻀﻐﻁﻪ‬
‫‪m2‬‬
‫‪ . 2x10 7 N‬ﺇﺫﺍ‬ ‫ﺍﻟﺠﻭﻱ ﺍﻟﻨﻅﺎﻤﻲ(‪ .‬ﻭﺘﻨﺯل ﻓﻲ ﻤﻴﺎﻩ ﺍﻟﺒﺤﺭ ﺤﺘﻰ ﻋﻤﻕ ﻀﻐﻁﻪ‬
‫‪m2‬‬
‫ﻜﺎﻥ ﺤﺠﻡ ﺍﻟﻜﺭﺓ ﻓﻲ ﺍﻟﻬﻭﺍﺀ ﻴﺴﺎﻭﻱ ﺇﻟﻰ‪ ، 0.5 m 3 :‬ﻓﻤﺎ ﻫﻭ ﺍﻟﺘﻐﻴﺭ ﺍﻟﺫﻱ ﻴﻁﺭﺃ ﻋﻠﻰ‬

‫‪- 223 -‬‬

‫)‪Create PDF files without this message by purchasing novaPDF printer (http://www.novapdf.com‬‬
‫ﺤﺠﻡ ﺍﻟﻜﺭﺓ ﻋﻨﺩﻤﺎ ﺘﻨﺯل ﻓﻲ ﻤﻴﺎﻩ ﺍﻟﺒﺤﺭ ﺤﺘﻰ ﺍﻟﻌﻤﻕ ﺍﻟﺴﺎﺒﻕ؟ ﻤﻊ ﺍﻟﻌﻠﻡ ﺃﻥ ﻤﻌﺎﻤل‬
‫‪6.1x10 10 N‬‬ ‫ﺘﻐﻴﺭ ﺤﺠﻡ ﻜﺭﺓ ﺍﻟﻨﺤﺎﺱ ﻫﻭ‬
‫‪m2‬‬
‫ﺍﳊﻞ ‪:‬‬
‫ﻨﺤﺼل ﻤﻥ ﺘﻌﺭﻴﻑ ﻤﻌﺎﻤل ﺘﻐﻴﺭ ﺍﻟﺤﺠﻡ ﻋﻠﻰ‪:‬‬
‫‪ΔP‬‬
‫‪B ‬‬
‫‪ΔV‬‬
‫‪Vi‬‬
‫ﺒﻤﺎ ﺃﻥ ﺍﻟﻀﻐﻁ ﺍﻟﻨﻬﺎﺌﻲ ﻋﻠﻰ ﺍﻟﻜﺭﺓ ﺃﻜﺒﺭ ﺒﻜﺜﻴﺭ ﻤﻥ ﺍﻟﻀﻐﻁ ﺍﻻﺒﺘﺩﺍﺌﻲ‪ ،‬ﻟﺫﻟﻙ‬
‫ﻴﻤﻜﻥ ﺇﻫﻤﺎل ﺍﻟﻀﻐﻁ ﺍﻻﺒﺘﺩﺍﺌﻲ ﺃﻤﺎﻡ ﺍﻟﻀﻐﻁ ﺍﻟﻨﻬﺎﺌﻲ‪ ،‬ﻭﻴﺼﺒﺢ ﺍﻟﺘﻐﻴﺭ ﻓﻲ ﺍﻟﻀﻐﻁ‬
‫ﻫﻭ‪:‬‬
‫‪Δp  p f  p i  p f  2x10 7 N‬‬
‫‪m2‬‬
‫ﻭﻴﻜﻭﻥ ﺍﻟﺘﻐﻴﺭ ﻓﻲ ﺍﻟﺤﺠﻡ‪:‬‬
‫‪0.5m 2x10‬‬
‫‪3‬‬ ‫‪10‬‬ ‫‪N‬‬ ‫‪‬‬
‫‪m 2   1.6x10 -4 m 3‬‬
‫‪ΔV  ‬‬
‫‪6.1x10 10 N‬‬
‫‪m2‬‬
‫ﺘﺩل ﺍﻹﺸﺎﺭﺓ ﺍﻟﺴﺎﻟﺒﺔ ﻋﻠﻰ ﺃﻥ ﺤﺠﻡ ﺍﻟﻜﺭﺓ ﻗﺩ ﺘﻨﺎﻗﺹ‪.‬‬

‫‪- 224 -‬‬

‫)‪Create PDF files without this message by purchasing novaPDF printer (http://www.novapdf.com‬‬
‫ﺍﳌﻮﺍﺩ ﺍﳌﺮﻧﺔ‬
‫‪ - 55‬ﺗﻨﺴﻮﺭ ﺍﻟﺘﺸﻮﻩ‪:‬‬
‫ﻟﻘﺩ ﻨﻭﻫﻨﺎ ﻓﻲ ﻓﻘﺭﺍﺕ ﺴﺎﺒﻘﺔ ﻋﻥ ﺇﺯﺍﺤﺔ ﺍﻷﺠﺴﺎﻡ ﺍﻟﻤﺭﻨﺔ ﻋﻨﺩ ﺘﺄﺜﻴﺭ ﻗﻭﻯ‬
‫ﺨﺎﺭﺠﻴﺔ ﻋﻠﻴﻬﺎ‪ ،‬ﻭﺴﻭﻑ ﻨﺩﺭﺱ ﻓﻲ ﻫﺫﺍ ﺍﻟﻔﺼل ﻤﺎ ﻴﺤﺼل ﺩﺍﺨل ﺍﻟﻤﺎﺩﺓ ﺍﻟﻤﺭﻨﺔ‬
‫ﺒﺸﻜل ﻋﺎﻡ‪ ،‬ﻟﻬﺫﺍ ﻨﻔﺭﺽ ﺃﻨﻪ ﻟﺩﻴﻨﺎ ﻤﺎﺩﺓ ﻏﻴﺭ ﻤﺸﻭﻫﺔ ﻭﻀﻌﺕ ﺘﺤﺕ ﺠﻬﺩ‪ ،‬ﻭﻨﺭﺍﻗﺏ‬
‫ﺤﺭﻜﺔ ﺠﺯﺀ ﻫﻼﻤﻲ ﻤﺸﻭﺏ ﺩﺍﺨل ﺍﻟﻤﺎﺩﺓ‪ .‬ﺒﻔﺭﺽ ﺃﻥ ﺍﻟﺠﺯﺀ ﺍﻟﻬﻼﻤﻲ ﻴﻘﻊ ﻓﻲ‬
‫‪‬‬
‫ﻟﺤﻅﺔ ﺍﻟﺒﺩﺀ ﻓﻲ ﺍﻟﻤﻭﻀﻊ ‪ P‬ﺍﻟﺫﻱ ﻴﺘﻌﻴﻥ ﺒﺎﻟﻤﺘﺠﻪ ‪ ، r x, y, z ‬ﻴﻨﺘﻘل ﺒﺘﺄﺜﻴﺭ ﺍﻟﺠﻬﺩ‬
‫‪‬‬
‫ﺇﻟﻰ ﺍﻟﻤﻭﻀﻊ ’‪ P‬ﺍﻟﺫﻱ ﻴﺘﻌﻴﻥ ﺒﺎﻟﻤﺘﺠﻪ ‪ ، r x , y, z ‬ﻜﻤﺎ ﻫﻭ ﻤﺒﻴﻥ ﻋﻠﻰ ﺍﻟﺸﻜل‬
‫‪‬‬
‫)‪ ،.(5.7‬ﻨﺭﻤﺯ ﻟﻤﺘﺠﻪ ﺍﻹﺯﺍﺤﺔ ﻤﻥ ﺍﻟﻨﻘﻁﺔ ‪ P‬ﺇﻟﻰ ﺍﻟﻨﻘﻁﺔ ’‪ P‬ﺒﺎﻟﺭﻤﺯ ‪ ، u‬ﻓﻴﻜﻭﻥ‪:‬‬
‫‪  ‬‬
‫‪u  r  r‬‬ ‫)‪(7.5‬‬
‫‪‬‬ ‫‪‬‬ ‫‪‬‬
‫ﺘﻌﺘﻤﺩ ﺍﻹﺯﺍﺤﺔ ‪ u‬ﻋﻠﻰ ﺍﻟﻤﻭﻀﻊ ‪ ،P‬ﺃﻱ ﺃﻥ ﺍﻹﺯﺍﺤﺔ ‪ u‬ﺘﻌﺘﺒﺭ ﺩﺍﻟﺔ ﻟﻠﻤﺘﺠﻪ ‪r‬‬
‫ﺃﻭ ﻟﻺﺤﺩﺍﺜﻴﺎﺕ ‪.x،y،z‬‬
‫ﻨﺩﺭﺱ ﺃﻭﻻﹰ ﺍﻟﺤﺎﻟﺔ ﺍﻟﺨﺎﺼﺔ ﺍﻟﺘﺎﻟﻴﺔ‪ :‬ﻋﻨﺩﻤﺎ ﻴﻜﻭﻥ ﺍﻟﺘﺸﻭﻩ ﻟﻜل ﺍﻟﻤﺎﺩﺓ ﺜﺎﺒﺘﺎﹰ‪ ،‬ﺃﻱ‬
‫ﺘﺸﻭﻫﺎﹰ ﻤﺘﺠﺎﻨﺴﺎﹰ‪.‬‬

‫ﺍﻟﺸﻜل)‪ :(7.5‬ﺒﻘﻌﺔ ﻤﺸﻭﺒﺔ ﻓﻲ ﻤﺎﺩﺓ ﻏﻴﺭ ﻤﺸﻭﻫﺔ‬


‫ﺒﻘﻌﺔ ﻫﻼﻤﻴﺔ ﻤﺸﻭﺒﺔ ﻓﻲ ﻤﺎﺩﺓ ﻏﻴﺭ ﻤﺸﻭﻫﻪ ﻓﻲ ﺍﻟﻨﻘﻁﺔ ‪ P‬ﻤﻥ ﺍﻟﻤﻜﻌﺏ ﻭﻓﻲ '‪ P‬ﺒﻌﺩ ﺍﻟﺘﺸﻭﻩ‪.‬‬

‫‪- 225 -‬‬

‫)‪Create PDF files without this message by purchasing novaPDF printer (http://www.novapdf.com‬‬
‫ﺒﻔﺭﺽ ﺃﻨﻪ ﻟﺩﻴﻨﺎ ﻋﺎﺭﻀﺔ ﻤﺘﺠﺎﻨﺴﺔ ﻤﻥ ﻤﺎﺩﺓ ﻤﺎ ﻁﺒﻘﻨﺎ ﻋﻠﻴﻬﺎ ﺇﺠﻬﺎﺩ ﺒﺤﻴﺙ‬
‫ﻴﺅﺩﻱ ﺇﻟﻰ ﺯﻴﺎﺩﺓ ﻤﻨﺘﻅﻤﺔ ﻓﻲ ﺍﻟﻁﻭل ﺒﺎﺘﺠﺎﻩ ﺍﻟﻤﺤﻭﺭ ‪ x‬ﻜﻤﺎ ﻋﻠﻰ ﺍﻟﺸﻜل )‪.(7.6‬‬
‫ﻴﺘﻨﺎﺴﺏ ﻤﻘﺩﺍﺭ ﺍﻹﺯﺍﺤﺔ ‪ ux‬ﻁﺭﺩﺍﹰ ﻤﻊ ‪ ،x‬ﻭﻴﻤﻜﻥ ﺃﻥ ﻨﻜﺘﺏ‪:‬‬
‫‪u x ‬‬
‫‪‬‬
‫‪x‬‬ ‫‪‬‬

‫ﻭﻨﻜﺘﺏ ﺍﻹﺯﺍﺤﺔ ‪ ux‬ﺒﺎﻟﺼﻭﺭﺓ‪:‬‬

‫‪u x  e xx x‬‬
‫‪‬‬
‫‪ e xx ‬ﺜﺎﺒﺕ ﺘﻨﺎﺴﺏ‪ ،‬ﻭﺴﻨﺒﻴﻥ ﻻﺤﻘﺎﹰ ﺴﺒﺏ ﻜﺘﺎﺒﺔ ﺩﻟﻴﻠﻴﻥ ﺃﺴﻔل‬ ‫ﺤﻴﺙ ﺃﻥ‬
‫‪‬‬
‫ﻭﻴﻤﻴﻥ ﺍﻟﺜﺎﺒﺕ ‪.exx‬‬

‫ﺇﺫﺍ ﻜﺎﻥ ﺍﻟﺘﺸﻭﻩ ) ﺘﻐﻴﺭ ﺍﻟﺸﻜل( ﻏﻴﺭ ﻤﺘﺠﺎﻨﺱ‪ ،‬ﻓﺈﻥ ﺍﻟﻌﻼﻗﺔ ﺒﻴﻥ ‪ x‬ﻭ ‪ ux‬ﻟﻥ‬
‫ﺘﻜﻭﻥ ﻤﻨﺘﻅﻤﺔ ﻭﺇﻨﻤﺎ ﺘﺘﻐﻴﺭ ﻤﻥ ﻨﻘﻁﺔ ﺇﻟﻰ ﺃﺨﺭﻯ‪ ،‬ﻭﻴﻌﻁﻰ ﻓﻲ ﻫﺫﻩ ﺍﻟﺤﺎﻟﺔ ﺜﺎﺒﺕ‬
‫ﺍﻟﺘﻨﺎﺴﺏ ﺒﺎﻟﻌﻼﻗﺔ ﺍﻟﺘﺎﻟﻴﺔ‪:‬‬
‫‪u x‬‬
‫‪e xx ‬‬ ‫)‪(7.6‬‬
‫‪x‬‬

‫ﻴﻜﻭﻥ ﺜﺎﺒﺕ ﺍﻟﺘﻨﺎﺴﺏ ‪ exx‬ﻓﻲ ﻫﺫﻩ ﺍﻟﺤﺎﻟﺔ ﺩﺍﻟﺔ ﻟـ ‪ ، z،y،x‬ﻭﺘﺼﻑ ﺍﻟﻌﻼﻗﺔ‬


‫ﺍﻟﺴﺎﺒﻘﺔ ﺍﻟﺘﻤﺩﺩ ﺍﻟﻁﻭﻟﻲ ﻓﻲ ﺍﺘﺠﺎﻩ ﺍﻟﻤﺤﻭﺭ ‪ ،x‬ﻭﻴﻤﻜﻥ ﻜﺫﻟﻙ ﺃﻥ ﻴﺤﺩﺙ ﺍﻟﺘﻤﺩﺩ ﻓﻲ‬
‫ﺍﺘﺠﺎﻩ ﺍﻟﻤﺤﻭﺭﻴﻥ ‪ z،y‬ﻭﻴﻭﺼﻑ ﺫﻟﻙ ﺒﺎﻟﻌﻼﻗﺘﻴﻥ ﺍﻟﻤﺸﺎﺒﻬﺘﻴﻥ ﺍﻟﺘﺎﻟﻴﺘﻴﻥ‪:‬‬
‫‪u y‬‬ ‫‪u z‬‬
‫‪e yy ‬‬ ‫‪, e zz ‬‬ ‫)‪(7.7‬‬
‫‪y‬‬ ‫‪z‬‬

‫‪- 226 -‬‬

‫)‪Create PDF files without this message by purchasing novaPDF printer (http://www.novapdf.com‬‬
‫ﺍﻟﺸﻜل)‪(7.6‬‬
‫ﺍﺯﺩﻴﺎﺩ ﺍﻟﻁﻭل ﺒﺸﻜل ﻤﻨﺘﻅﻡ‬

‫ﺒﺎﻹﻀﺎﻓﺔ ﻟﻤﺎ ﺴﺒﻕ ﻴﺠﺏ ﺃﻥ ﻨﺼﻑ ﺍﻟﺘﺸﻭﻩ ﺍﻟﻤﺭﺍﻓﻕ ﻻﻨﺯﻴﺎﺡ ﺍﻟﺠﺴﻡ‪ ،‬ﻟﺫﻟﻙ‬
‫ﻨﻔﺭﺽ ﻓﻲ ﺍﻟﺒﺩﺍﻴﺔ ﺃﻨﻪ ﻟﺩﻴﻨﺎ ﻜﻌﻭﺏ )ﻤﻜﻌﺏ ﺼﻐﻴﺭ ﺠﺩ ﺍﹰ( ﻏﻴﺭ ﺨﺎﻀﻊ ﻷﻱ ﺘﺸﻭﻩ‪،‬‬
‫ﻭﻋﻨﺩ ﺍﻟﻀﻐﻁ ﻋﻠﻴﻪ ﻴﺘﺤﻭل ﺇﻟﻰ ﻤﺘﻭﺍﺯﻱ ﺃﻀﻼﻉ‬

‫ﻜﻤﺎ ﻫﻭ ﻤﻭﻀﺢ ﻋﻠﻰ ﺍﻟﺸﻜل )‪ ،(7.6‬ﺤﻴﺙ‪ ،‬ﺘﺘﻨﺎﺴﺏ ﺍﻹﺯﺍﺤﺔ ﻓﻲ ﻤﺜل ﻫﺫﺍ‬


‫ﺍﻟﺘﺸﻭﻩ ﻟﻜل ﺠﺯﻱﺀ ﺒﺎﺘﺠﺎﻩ ﺍﻟﻤﺤﻭﺭ ‪ x‬ﻤﻊ ﺍﻻﺤﺩﺍﺜﻲ ‪ y‬ﻟﻬﺫﺍ ﺍﻟﺠﺯﻱﺀ‪ ،‬ﺃﻱ ﺃﻥ‪:‬‬
‫‪θ‬‬
‫‪ux ‬‬ ‫‪y‬‬ ‫)‪(7.8‬‬
‫‪2‬‬
‫ﻭﺘﺘﻨﺎﺴﺏ ﺍﻹﺯﺍﺤﺔ ﺒﺎﺘﺠﺎﻩ ﺍﻟﻤﺤﻭﺭ ‪ y‬ﻤﻊ ﺍﻻﺤﺩﺍﺜﻲ ‪:x‬‬
‫‪θ‬‬
‫‪uy ‬‬ ‫‪x‬‬ ‫)‪(7.9‬‬
‫‪2‬‬

‫‪- 227 -‬‬

‫)‪Create PDF files without this message by purchasing novaPDF printer (http://www.novapdf.com‬‬
‫ﻭﻴﻤﻜﻥ ﻓﻲ ﻤﺜل ﻫﺫﻩ ﺍﻟﺤﺎﻟﺔ ﻭﺼﻑ ﺍﻟﺘﺸﻭﻩ ﺃﻻﻨﺯﻴﺎﺤﻲ ﺒﻤﺴﺎﻋﺩﺓ ﺍﻟﻤﻌﺎﺩﻻﺕ‬
‫ﺍﻟﺘﺎﻟﻴﺔ‪:‬‬
‫‪u x  e xy y , u y  e yx x‬‬

‫ﺤﻴﺙ ﺃﻥ‪:‬‬
‫‪θ‬‬
‫‪e xy  e yx ‬‬
‫‪2‬‬
‫ﻴﻤﻜﻥ ﺃﺨﻴﺭﺍﹰ ﻭﺼﻑ ﺍﻟﺘﺸﻭﻩ ﺍﻟﻐﻴﺭ ﻤﺘﺠﺎﻨﺱ ﺍﻟﻤﺭﺍﻓﻕ ﻻﻨﺯﻴﺎﺡ ﻤﻥ ﺘﻌﻴﻴﻥ ﻗﻴﻡ‬
‫‪ eyx ،exy‬ﻭﺫﻟﻙ ﻤﻥ ﺍﻟﻌﻼﻗﺘﻴﻥ‪:‬‬
‫‪u x‬‬ ‫‪u y‬‬
‫‪e xy ‬‬ ‫‪, e yx ‬‬ ‫)‪(7.10‬‬
‫‪y‬‬ ‫‪x‬‬
‫ﻨﻔﺭﺽ ﺍﻵﻥ ﺇﻥ ﺍﻹﺯﺍﺤﺘﻴﻥ ‪ uy ،ux‬ﻤﻌﻴﻨﺘﺎﻥ ﺒﺎﻟﻌﻼﻗﺘﻴﻥ ﺍﻟﺘﺎﻟﻴﺘﻴﻥ‪:‬‬
‫‪θ‬‬ ‫‪θ‬‬
‫‪ux ‬‬ ‫‪y , uy   x‬‬
‫‪2‬‬ ‫‪2‬‬

‫ﺍﻟﺸﻜل)‪ (7.7‬ﺘﺸﻭﻩ ﻤﺘﺠﺎﻨﺱ ﻤﻊ ﺍﻨﺯﻴﺎﺡ‬

‫ﻭﻫﻤﺎ ﻤﺸﺎﺒﻬﺘﺎﻥ ﻟﻠﻤﻌﺎﺩﻟﺘﻴﻥ‪ ،(7.9) ،(7.8) :‬ﻤﻊ ﺍﻟﻔﺎﺭﻕ ﺃﻥ ﺇﺸﺎﺭﺓ‬


‫ﻜﻤﺎ ﻋﻠﻰ‬ ‫‪ uy‬ﻤﻌﺎﻜﺴﺔ‪ .‬ﻴﻌﺎﻨﻲ ﺍﻟﻜﻌﻭﺏ ﻓﻲ ﻫﺫﻩ ﺍﻹﺯﺍﺤﺔ ﺩﻭﺭﺍﻥ ﺒﺯﺍﻭﻴﺔ ﻤﻘﺩﺍﺭﻫﺎ‬
‫‪θ‬‬
‫‪2‬‬
‫ﺍﻟﺸﻜل )‪ ، (7.7‬ﻭﻻ ﻴﺤﺩﺙ ﺃﻱ ﺘﺸﻭﻩ ﻓﻲ ﻫﺫﻩ ﺍﻟﺤﺎﻟﺔ‪ ،‬ﻭﺇﻨﻤﺎ ﻴﺤﺩﺙ ﺩﻭﺭﺍﻥ ﻓﻲ‬
‫ﺍﻟﻔﻀﺎﺀ‪ ،‬ﻭﻻ ﻴﺤﺩﺙ ﺃﻱ ﺍﻀﻁﺭﺍﺏ ﻟﻠﻤﺎﺩﺓ ﻜﻤﺎ ﺃﻥ ﺍﻟﻤﻭﻀﻊ ﺍﻟﻨﺴﺒﻲ ﻟﻠﺫﺭﺍﺕ ﻻ‬

‫‪- 228 -‬‬

‫)‪Create PDF files without this message by purchasing novaPDF printer (http://www.novapdf.com‬‬
‫ﻴﺘﻐﻴﺭ‪ ،‬ﻭﺒﺎﻟﺘﺎﻟﻲ ﻴﺠﺏ ﺘﺼﻤﻴﻡ ﺤﺎﻟﺔ ﺒﺤﻴﺙ ﻴﺠﺏ ﺃﻻ ﻴﺩﺨل ﺍﻟﺩﻭﺭﺍﻥ ﺍﻟﺼﺭﻑ ﻓﻲ‬
‫‪ u x y‬ﻤﺘﺴﺎﻭﻴﺘﺎﻥ‬ ‫‪،‬‬ ‫ﺘﺸﻭﻴﻪ ﺍﻹﺯﺍﺤﺔ‪ ،‬ﻫﺫﺍ ﻤﻤﻜﻥ ﺇﺫﺍ ﻜﺎﻨﺕ ﺍﻟﻨﺴﺒﺘﺎﻥ‬
‫‪u y‬‬
‫‪x‬‬
‫ﻭﻤﺘﻌﺎﻜﺴﺘﺎﻥ‪ ،‬ﻭﻻ ﻴﻭﺠﺩ ﻓﻲ ﻫﺫﻩ ﺍﻟﺤﺎﻟﺔ ﺃﻱ ﺇﺠﻬﺎﺩ‪ ،‬ﻭﻴﻤﻜﻥ ﻓﻲ ﻫﺫﻩ ﺍﻟﺤﺎﻟﺔ ﺇﻀﺎﻓﺔ‬
‫ﺍﻟﺘﻌﺭﻴﻑ ﺍﻟﺘﺎﻟﻲ‪:‬‬
‫‪‬‬‫‪1  u y u x‬‬
‫‪e xy  e yx ‬‬ ‫‪‬‬ ‫‪‬‬ ‫‪‬‬
‫‪2  x‬‬
‫‪‬‬ ‫‪y‬‬

‫‪ u x‬ﻓﻲ ﺤﺎﻟﺔ ﺍﻟﺩﻭﺭﺍﻥ ﺍﻟﺼﺭﻑ ‪ ،‬ﻭﺘﺘﺤﻘﻕ‬ ‫‪،‬‬ ‫ﺘﻨﻌﺩﻡ ﺍﻟﻨﺴﺒﺘﺎﻥ‪:‬‬


‫‪u y‬‬
‫‪y‬‬ ‫‪x‬‬
‫ﺒﻴﻨﻬﻤﺎ ﻓﻲ ﺤﺎﻟﺔ ﺍﻻﻨﺘﻘﺎل ﺍﻟﺼﺭﻑ ﺍﻟﻤﺴﺎﻭﺍﺓ‪.exy= eyx :‬‬
‫ﻓﻲ ﺍﻟﺤﺎﻟﺔ ﺍﻟﻌﺎﻤﺔ ﻟﻠﺘﺸﻭﻴﺵ‪ ،‬ﺤﻴﺙ ﺘﺘﺭﺍﻓﻕ ﺍﻹﺯﺍﺤﺔ ﺇﻤﺎ ﺒﺎﻨﻀﻐﺎﻁ ﺃﻭ ﺒﺘﻤﺩﺩ‬
‫ﻟﻠﻤﺎﺩﺓ‪ ،‬ﻓﺎﻥ ﺍﻟﺘﺸﻭﻩ ﻴﻌﺭﻑ ﺒﺘﺤﺩﻴﺩ ﺘﺴﻌﺔ ﺃﻋﺩﺍﺩ ﻫﻲ‪:‬‬
‫‪u x‬‬
‫‪e xx ‬‬
‫‪x‬‬
‫‪u y‬‬
‫‪e yy ‬‬ ‫)‪(7.11‬‬
‫‪y‬‬
‫‪1  u y u x ‬‬
‫‪.e xy ‬‬ ‫‪‬‬ ‫‪‬‬ ‫‪‬‬
‫‪2  x‬‬ ‫‪y ‬‬
‫ﺘﺸﻜل ﻫﺫﻩ ﺍﻷﻋﺩﺍﺩ ﻤﺭﻜﺒﺎﺕ ﺘﻨﺴﻭﺭ ﺍﻟﺘﺸﻭﻩ‪ ،‬ﺒﻤﺎ ﺃﻥ ﻫﺫﺍ ﺍﻟﺘﻨﺴﻭﺭ‬
‫ﻤﺘﻨﺎﻅﺭ)ﺤﺴﺏ ﺍﻟﺘﻌﺭﻴﻑ ﻴﻜﻭﻥ ‪ ،( eyx=exy‬ﻓﺎﻥ ﺍﻷﻋﺩﺍﺩ ﺍﻟﻤﺨﺘﻠﻔﺔ ﻫﻲ ﺴﺘﺔ ﻓﻘﻁ‪.‬‬
‫ﻨﻜﺘﺏ ﺒﺩل ﺍﻟﻤﺭﻜﺒﺎﺕ ‪ x,y,z‬ﺍﻟﻤﺭﻜﺒﺎﺕ ‪ x1 , x 2 , x 3‬ﻭﺒﺩل ‪ u x , u y , u z‬ﺍﻟﻤﺭﻜﺒﺎﺕ‬
‫‪ ، u1 , u 2 , u 3‬ﻴﻤﻜﻨﻨﺎ ﻋﻨﺩﺌﺫ‪ ‬ﺃﻥ ﻨﻌﺒﺭ ﻋﻥ ﻋﻨﺎﺼﺭ ﺍﻟﺘﻨﺴﻭﺭ ‪ eij‬ﺒﺎﻟﻌﻼﻗﺔ ﺍﻟﺘﺎﻟﻴﺔ‪:‬‬
‫‪1  u j u i ‬‬
‫‪e ij ‬‬ ‫‪‬‬ ‫)‪(7.12‬‬
‫‪2  x i x j ‬‬

‫ﻴﻤﻜﻥ ﺃﻥ ﻴﺄﺨﺫ ﺍﻟﺩﻟﻴﻼﻥ ‪ i،j‬ﺍﻟﻘﻴﻡ ‪.1,2,3‬‬


‫ﻋﻨﺩﻤﺎ ﻴﻜﻭﻥ ﺘﺸﻭﻩ ﺍﻟﺠﺴﻡ ﻤﺘﺠﺎﻨﺴﺎ ﺴﻭﺍﺀ ﺃﻜﺎﻥ ﺘﻤﺩﺩﺍ ﺃﻡ ﺍﻨﻀﻐﺎﻁﺎﹰ ﻟﻠﺠﺴﻡ‪،‬‬
‫ﻓﺈﻥ ﻜل ﻗﻴﻡ ‪ eij‬ﺘﻜﻭﻥ ﺜﺎﺒﺘﺔ ﻭﻨﺴﺘﻁﻴﻊ ﺃﻥ ﻨﻜﺘﺏ‪:‬‬
‫‪u x  e xx x  e xy y  e xz z‬‬ ‫)‪(7.13‬‬

‫‪- 229 -‬‬

‫)‪Create PDF files without this message by purchasing novaPDF printer (http://www.novapdf.com‬‬
‫)ﺤﻴﺙ ﺍﻋﺘﺒﺭﻨﺎ ﺃﻥ ﻤﺒﺩﺃ ﺍﻹﺤﺩﺍﺜﻴﺎﺕ ﻓﻲ ﺍﻟﻨﻘﻁﺔ ﺍﻟﺘﻲ ﻴﻨﻌﺩﻡ ﻤﻥ ﺃﺠﻠﻬﺎ ﺍﻟﻤﺘﺠﻪ‬
‫‪‬‬
‫‪ .( u‬ﻴﻌﺒﺭ ﺘﻨﺴﻭﺭ ﺍﻟﺘﺸﻭﻩ ‪ eij‬ﻓﻲ ﻫﺫﻩ ﺍﻟﺤﺎﻻﺕ ﻋﻥ ﺍﻟﻌﻼﻗﺔ ﺒﻴﻥ ﻤﺘﺠﻪ ﺍﻹﺤﺩﺍﺜﻴﺎﺕ‬
‫‪‬‬ ‫‪‬‬
‫‪ r x, y, z ‬ﻭﻤﺘﺠﻪ ﺍﻹﺯﺍﺤﺔ ‪. ux, y, z ‬‬
‫ﺇﺫﺍ ﻜﺎﻥ ﺍﻟﺘﺸﻭﻩ ﻏﻴﺭ ﻤﺘﺠﺎﻨﺱ‪ ،‬ﻓﺈﻥ ﺃﻱ ﺠﺯﺀ ﻫﻼﻤﻲ ﻴﻤﻜﻥ ﺃﻥ ﻴﻌﺎﻨﻲ ﺘﺸﻭﻫﺎ‪،‬‬
‫ﺒﺎﻹﻀﺎﻓﺔ ﻟﺫﻟﻙ ﻴﻤﻜﻥ ﺃﻥ ﻴﺤﺩﺙ ﻟﻪ ﺩﻭﺭﺍﻥ ﻤﻭﻀﻌﻲ‪ ،‬ﻭﺇﺫﺍ ﻜﺎﻥ ﺍﻟﺘﺸﻭﻴﺵ ﺼﻐﻴﺭ ﺍﹰ‬
‫ﻓﺈﻥ‪:‬‬
‫‪Δu i   e ij  ω ij Δx j‬‬ ‫)‪(7.14‬‬
‫‪i‬‬

‫ﺤﻴﺙ ‪ ω ij‬ﺘﻨﺴﻭﺭ ﻻ ﺘﻨﺎﻅﺭﻱ ﻴﺼﻑ ﺍﻟﺩﻭﺭﺍﻥ‪ ،‬ﻭﻴﻌﺭﻑ ﺒﺎﻟﻌﻼﻗﺔ ﺍﻟﺘﺎﻟﻴﺔ‪:‬‬


‫‪1  u j u i ‬‬
‫‪ω ij ‬‬ ‫‪‬‬ ‫)‪(7.15‬‬
‫‪2  x i x j ‬‬

‫ﺴﻨﻘﺘﺼﺭ ﻓﻲ ﺩﺭﺍﺴﺘﻨﺎ ﻋﻠﻰ ﺍﻟﺘﺸﻭﻩ ﺍﻟﺫﻱ ﻴﻤﻜﻥ ﻭﺼﻔﻪ ﺒﺘﻨﺴﻭﺭ ﻤﺘﻨﺎﻅﺭ ‪.eij‬‬
‫‪ - 56‬ﺗﻨﺴﻮﺭ ﺍﳌﺮﻭﻧﺔ‪:‬‬
‫ﻴﻠﺯﻡ ﻟﻭﺼﻑ ﺘﺸﻭﻩ ﻤﺎﺩﺓ‪ ،‬ﺭﺒﻁ ﺍﻟﻘﻭﻯ ﺍﻟﺩﺍﺨﻠﻴﺔ ﻤﻊ ﺍﻹﺠﻬﺎﺩ ﻓﻲ ﺍﻟﻤﺎﺩﺓ‪.‬‬
‫ﻨﻔﺘﺭﺽ ﺃﻥ ﻗﺎﻨﻭﻥ ﻫﻭﻙ ﻤﺤﻘﻕ ﻤﻥ ﺃﺠل ﺃﻱ ﺠﺯﺀ ﻤﻥ ﺍﻟﻤﺎﺩﺓ‪ ،‬ﺃﻱ ﺃﻥ ﺍﻟﺘﺸﻭﻩ‬
‫ﻴﺘﻨﺎﺴﺏ ﻁﺭﺩ ﺍﹰ ﻤﻊ ﺍﻹﺠﻬﺎﺩ‪ ،‬ﻭﻟﻘﺩ ﻋﺭﻓﻨﺎ ﺴﺎﺒﻘﺎﹰ ﺘﻨﺴﻭﺭ ﺍﻹﺠﻬﺎﺩ ‪ Sij‬ﺒﺄﻨﻪ ﺍﻟﻤﺭﻜﺒﺔ ‪i‬‬
‫ﻟﻠﻘﻭﺓ ﺍﻟﻤﺅﺜﺭﺓ ﻋﻠﻰ ﻭﺤﺩﺓ ﻤﺴﺎﺤﺔ ﻋﻤﻭﺩﻴﺔ ﻋﻠﻰ ﺍﻟﻤﺤﻭﺭ ‪ .j‬ﻴﻨﺹ ﻗﺎﻨﻭﻥ ﻫﻭﻙ ﻋﻠﻰ‬
‫ﺃﻥ ﻜل ﻤﺭﻜﺒﺔ ﻟﻠﺘﻨﺴﻭﺭ ‪ Sij‬ﺘﺭﺘﺒﻁ ﺨﻁﻴﺎﹰ ﻤﻊ ﻜل ﻤﺭﻜﺒﺔ ﻟﻺﺠﻬﺎﺩ‪ .‬ﺒﻤﺎ ﺃﻥ ﻟﻜلٍ ﻤﻥ‬
‫‪ S‬ﻭ ‪ l‬ﺘﺴﻊ ﻤﺭﻜﺒﺎﺕ‪ ،‬ﻓﺈﻨﻪ ﻴﻠﺯﻡ ﻟﻭﺼﻑ ﻤﺭﻭﻨﺔ ﻤﺎﺩﺓ ‪ 9×9=81‬ﻤﻌﺎﻤل‪ ،‬ﻓﺈﺫﺍ‬
‫ﻜﺎﻨﺕ ﺍﻟﻤﺎﺩﺓ ﻤﺘﺠﺎﻨﺴﺔ ﻓﺈﻥ ﻜل ﻫﺫﻩ ﺍﻟﻤﻌﺎﻤﻼﺕ ﺘﻜﻭﻥ ﺜﻭﺍﺒﺘﺎﹰ‪ .‬ﻨﺭﻤﺯ ﻟﻬﺫﻩ ﺍﻟﺜﻭﺍﺒﺕ‬
‫ﺒﺎﻟﺭﻤﺯ ‪ ، C ijkl‬ﻭﺘﺘﻌﻴﻥ ﺒﺎﻟﻌﻼﻗﺔ‪:‬‬
‫‪Sij   C ijk l e kl‬‬ ‫)‪(7.16‬‬
‫‪kl‬‬

‫ﻴﻤﻜﻥ ﺃﻥ ﻴﺄﺨﺫ ﻜل ﺭﻤﺯ ﻤﻥ ﺍﻟﺭﻤﻭﺯ ‪ i, j, k , l‬ﺍﻟﻘﻴﻡ ‪ .3،2،1‬ﻭﺃﻥ ﺍﻟﻤﻌﺎﻤﻼﺕ‬


‫‪ C ijk l‬ﺘﺭﺒﻁ ﺘﻨﺴﻭﺭﺍﹰ ﺒﺂﺨﺭ‪ ،‬ﻭﻫﻲ ﺘﺸﻜل ﺃﻴﻀﺎﹰ ﺘﻨﺴﻭﺭﺍﹰ ﻤﻥ ﺍﻟﺭﺘﺒﺔ ﺍﻟﺭﺍﺒﻌﺔ‪ ،‬ﻴﺴﻤﻲ‬
‫ﻫﺫﺍ ﺍﻟﺘﻨﺴﻭﺭ ﺘﻨﺴﻭﺭ ﺍﻟﻤﺭﻭﻨﺔ‪.‬‬

‫‪- 230 -‬‬

‫)‪Create PDF files without this message by purchasing novaPDF printer (http://www.novapdf.com‬‬
‫ﺒﻔﺭﺽ ﺃﻥ ﻜل ﺍﻟﻤﻌﺎﻤﻼﺕ ‪ C ijkl‬ﻤﻌﻠﻭﻤﺔ‪ ،‬ﻭﺒﻔﺭﺽ ﺃﻨﻨﺎ ﻁﺒﻘﻨﺎ ﻤﺠﻤﻭﻋﺔ ﻗﻭﻯ‬
‫ﻋﻠﻰ ﺠﺴﻡ ﻟﻪ ﺸﻜل ﺍﺨﺘﻴﺎﺭﻱ‪ .‬ﺘﻅﻬﺭ ﻓﻲ ﻫﺫﻩ ﺍﻟﺤﺎﻟﺔ ﻜل ﺃﻨﻭﺍﻉ ﺘﺸﻭﻫﺎﺕ ﺍﻟﻤﺎﺩﺓ‪،‬‬
‫ﻭﻤﻥ ﺍﻟﺼﻌﺏ ﺘﺤﺩﻴﺩ ﺍﻨﺯﻴﺎﺡ ﺍﻟﺠﺴﻡ‪ ،‬ﻟﻜﻥ ﺇﺫﺍ ﻋﺭﻓﻨﺎ ﻨﻭﻉ ﺍﻟﺘﺸﻭﻩ ﻓﺈﻨﻨﺎ ﻨﺴﺘﻁﻴﻊ ﻤﻥ‬
‫ﺍﻟﻤﻌﺎﺩﻟﺔ )‪ ،(7.16‬ﺃﻥ ﻨﺤﺴﺏ ﺍﻹﺠﻬﺎﺩ ﻭﺒﺎﻟﻌﻜﺱ‪ ،‬ﻭﺃﻥ ﺍﻹﺠﻬﺎﺩ ﻭﺍﻟﺘﺸﻭﻩ ﺍﻟﻠﺫﺍﻥ‬
‫ﻴﻅﻬﺭﺍﻥ ﻓﻲ ﺃﻴﺔ ﻨﻘﻁﺔ ﻤﻥ ﺍﻟﻤﺎﺩﺓ ﻴﺘﻌﻠﻘﺎﻥ ﺒﻤﺎ ﻴﺤﺩﺙ ﻟﺒﻘﻴﺔ ﺃﺠﺯﺍﺀ ﺍﻟﻤﺎﺩﺓ‪.‬‬
‫ﺘﻜﻭﻥ ﺩﺭﺍﺴﺔ ﺍﻟﻤﺴﺄﻟﺔ ﺒﺎﺴﺘﺨﺩﺍﻡ ﻤﺒﺩﺃ ﺍﻟﻁﺎﻗﺔ ﺃﻜﺜﺭ ﺴﻬﻭﻟﺔ‪ ،‬ﻋﻨﺩﻤﺎ ﺘﺘﻨﺎﺴﺏ‬
‫ﺍﻟﻘﻭﺓ ﺍﻟﻤﺅﺜﺭﺓ ‪ F‬ﻤﻊ ﺍﻹﺯﺍﺤﺔ ‪ ،x‬ﺃﻱ ﻋﻨﺩﻤﺎ ﺘﺘﺤﻘﻕ ﺍﻟﻌﻼﻗﺔ ‪ ،F=kx‬ﻓﺈﻥ ﺍﻟﻌﻤل‬
‫‪kx 2‬‬
‫‪.‬‬ ‫ﺍﻟﻤﻘﺩﻡ ﻹﺤﺩﺍﺙ ﺇﺯﺍﺤﺔ ‪ x‬ﻴﺴﺎﻭﻱ‬
‫‪2‬‬
‫ﻭﺒﺸﻜل ﻤﻤﺎﺜل ﻓﺈﻥ ﺍﻟﻁﺎﻗﺔ ‪ ω‬ﺍﻟﻤﺨﺘﺯﻨﺔ ﻓﻲ ﻭﺤﺩﺓ ﺤﺠﻡ ﺍﺨﺘﻴﺎﺭﻴﺔ ﻤﻥ ﺍﻟﻤﺎﺩﺓ‬
‫ﺍﻟﻤﺸﻭﻫﺔ ﺘﺴﺎﻭﻱ‪:‬‬
‫‪1‬‬
‫‪ω‬‬ ‫‪ Cijk leijekl‬‬
‫‪2 ijk l‬‬
‫)‪(7.17‬‬

‫ﻭﺒﺎﻟﺘﺎﻟﻲ ﻓﺎﻟﻌﻤل ‪ w‬ﺍﻟﻤﻘﺩﻡ ﻟﺘﺸﻭﻩ ﻜﺎﻤل ﺍﻟﻤﺎﺩﺓ ﻴﺴﺎﻭﻱ ﺇﻟﻰ ﺘﻜﺎﻤل ‪ ω‬ﻋﻠﻰ‬
‫ﻜﺎﻤل ﺍﻟﺤﺠﻡ‪:‬‬
‫‪1‬‬
‫‪w‬‬ ‫‪ C ijk l e ij e kl dv‬‬
‫‪2 ijk l‬‬
‫)‪(7.18‬‬

‫ﻴﻌﺒﺭ ﻫﺫﺍ ﺍﻟﻌﻤل ﻋﻥ ﻁﺎﻗﺔ ﺍﻟﺠﻬﺩ ﺍﻟﻜﺎﻤﻨﺔ ﺩﺍﺨل ﺍﻟﻤﺎﺩﺓ‪ ،‬ﻭﺘﻜﻭﻥ ﻗﻴﻤﺔ ﻫﺫﻩ‬
‫ﺍﻟﻁﺎﻗﺔ ﺼﻐﺭﻯ ﻋﻨﺩﻤﺎ ﺘﻜﻭﻥ ﺍﻟﻤﺎﺩﺓ ﻓﻲ ﺤﺎﻟﺔ ﺍﺘﺯﺍﻥ‪ .‬ﺍﻨﻁﻼﻗﺎﹰ ﻤﻥ ﺫﻟﻙ ﻴﻤﻜﻥ ﺤل‬
‫‪‬‬
‫ﻤﺴﺄﻟﺔ ﺘﺸﻭﻩ ﺍﻟﺠﺴﻡ ﻤﻥ ﺇﻴﺠﺎﺩ ﺍﻹﺯﺍﺤﺔ ‪ u‬ﻟﻜل ﺍﻟﺠﺴﻡ ﻋﻨﺩﻤﺎ ﺘﻜﻭﻥ ﻗﻴﻤﺔ ﺍﻟﻌﻤل ‪w‬‬
‫ﺼﻐﺭﻯ‪ ،‬ﻏﻴﺭ ﺃﻨﻨﺎ ﻟﻥ ﻨﺩﺨل ﻓﻲ ﺘﻔﺎﺼﻴل ﻫﺫﻩ ﺍﻟﻤﺴﺄﻟﺔ‪.‬‬
‫ﻗﺩ ﻴﻁﺭﺡ ﺍﻟﺴﺅﺍل ﺍﻟﺘﺎﻟﻲ‪ :‬ﻤﺎ ﻫﻲ ﺍﻟﺼﻔﺎﺕ ﺍﻟﻨﺴﺒﻴﺔ ﻟﺘﻨﺴﻭﺭ ﺍﻟﻤﺭﻭﻨﺔ؟ ﻤﻥ‬
‫ﺍﻟﻭﺍﻀﺢ ﺃﻥ ‪ C ijkl‬ﻻ ﻴﺤﺘﻭﻱ ﻋﻠﻰ ‪ 81‬ﻤﻌﺎﻤﻼﹰ ﻤﺨﺘﻠﻔﺎﹰ‪ ،‬ﻷﻥ ‪ Sij‬ﻭ ‪ eij‬ﺘﻨﻨﺴﻭﺭﻴﻥ‬
‫ﻤﺘﻨﺎﻅﺭﻴﻥ ﻜلٍ ﻤﻨﻬﻤﺎ ﻴﺤﺘﻭﻱ ﻋﻠﻰ ﺴﺘﺔ ﻋﻨﺎﺼﺭ ﻤﺨﺘﻠﻔﺔ‪ ،‬ﻭﺒﺎﻟﺘﺎﻟﻲ ﻓﺈﻥ ‪ C ijkl‬ﻴﺘﺄﻟﻑ‬
‫ﻤﻥ ﺴﺕ‪ ‬ﻭﺜﻼﺜﻴﻥ ﻤﺭﻜﺒﺔ ﻤﺨﺘﻠﻔﺔ‪ ،‬ﻭﻴﻜﻭﻥ ﻋﺎﺩﺓﹰ ﺍﻟﻌﺩﺩ ﺃﻗل ﻤﻥ ﺫﻟﻙ ﺒﻜﺜﻴﺭ‪.‬‬
‫ﻨﺩﺭﺱ ﺤﺎﻟﺔ ﺨﺎﺼﺔ ﻤﺜﺎل ﻋﻠﻰ ﺫﻟﻙ ﺒﻠﻭﺭﺓ ﻤﻜﻌﺒﺔ‪ ،‬ﺤﻴﺙ ﺘﻌﻁﻰ ﻜﺜﺎﻓﺔ ﺍﻟﻁﺎﻗﺔ‬
‫‪ ω‬ﻟﻬﺫﻩ ﺍﻟﺒﻠﻭﺭﺓ ﺒﺎﻟﻌﻼﻗﺔ ﺍﻟﺘﺎﻟﻴﺔ‪:‬‬

‫‪- 231 -‬‬

‫)‪Create PDF files without this message by purchasing novaPDF printer (http://www.novapdf.com‬‬
‫‪2‬‬
‫‪1 C xxxx e xx  Cxxxy e xx e xy  Cxxxz e xx e xz  C xxyxe xx e yx  C xxyye xx e yy  ..... ‬‬
‫‪ω ‬‬ ‫‪‬‬
‫‪2  C yyyye 2yy  ........‬‬ ‫‪‬‬
‫)‪(7.19‬‬
‫ﺃﻱ ﺃﻥ ﻤﺠﻤﻭﻉ ﺍﻟﺤﺩﻭﺩ ﻫﻭ ‪ 81‬ﻭﻟﻜﻥ ﻟﻠﺒﻠﻭﺭﺓ ﺍﻟﻤﻜﻌﺒﺔ ﺼﻔﺎﺕ ﺘﻨﺎﻅﺭﻴﺔ‬
‫ﻤﺤﺩﺩﺓ‪ ،‬ﻓﻌﻨﺩﻤﺎ ﺘﺩﻭﺭ ﺍﻟﺒﻠﻭﺭﺓ ﺍﻟﻤﻜﻌﺒﺔ ﺒﺯﺍﻭﻴﺔ ﺘﺴﺎﻭﻱ ‪ ، 90‬ﻓﺈﻥ ﺨﻭﺍﺹ ﺍﻟﺒﻠﻭﺭﺓ‬
‫ﺍﻟﻔﻴﺯﻴﺎﺌﻴﺔ ﻻﺘﺘﻐﻴﺭ‪ ،‬ﻓﻤﺜﻼﹰ ﺃﺜﻨﺎﺀ ﻋﻤﻠﻴﺔ ﺍﻟﺴﺤﺏ ﺃﻭ ﺘﺒﻘﻰ ﻗﺴﺎﻭﺓ ﺍﻟﺒﻠﻭﺭﺓ ﻓﻲ ﺍﺘﺠﺎﻩ‬
‫ﺍﻟﻤﺤﻭﺭ ‪ x‬ﻤﺜل ﻗﺴﺎﻭﺘﻬﺎ ﻓﻲ ﺍﺘﺠﺎﻩ ﺍﻟﻤﺤﻭﺭ ‪ .y‬ﻴﻨﺘﺞ ﻤﻥ ﺫﻟﻙ ﺃﻨﻪ ﺇﺫﺍ ﻏﻴﺭﻨﺎ ﻤﻜﺎﻥ‬
‫ﺍﻟﻤﺤﻭﺭﻴﻥ ﺍﻻﺤﺩﺍﺜﻴﻴﻥ ‪ x‬ﻭ ‪ y‬ﻓﻲ ﺍﻟﻤﻌﺎﺩﻟﺔ )‪ ،(7.19‬ﻓﺈﻥ ﻗﻴﻤﺔ ﺍﻟﻁﺎﻗﺔ ﻴﺠﺏ ﺃﻥ ﻻ‬
‫ﺘﺘﻐﻴﺭ‪ ،‬ﻟﺫﻟﻙ ﻓﺈﻨﻪ ﻤﻥ ﺃﺠل ﺍﻟﺒﻠﻭﺭﺓ ﺍﻟﻤﻜﻌﺒﺔ ﻴﻜﻭﻥ‪:‬‬
‫‪C xxxx  C yyyy  C zzzz‬‬ ‫)‪(7.20‬‬
‫ﻭﻴﻤﻜﻥ ﻜﺫﻟﻙ ﺃﻥ ﻨﺒﻴﻥ ﺃﻥ ﺍﻟﻤﺭﻜﺒﺎﺕ ﺍﻟﻤﻤﺎﺜﻠﺔ ﻟـ ‪ Cxxxy‬ﻴﺠﺏ ﺃﻥ ﺘﺴﺎﻭﻱ‬
‫ﺍﻟﺼﻔﺭ‪ .‬ﺘﺤﻘﻕ ﺍﻟﺒﻠﻭﺭﺓ ﺍﻟﻤﻜﻌﺒﺔ ﺼﻔﺔ ﺍﻟﺘﻨﺎﻅﺭ ﻋﻨﺩ ﺍﻻﻨﻌﻜﺎﺱ ﺒﺎﻟﻨﺴﺒﺔ ﺇﻟﻰ ﺃﻱ‬
‫ﻤﺴﺘﻭﻯ ﻋﻤﻭﺩﻱ ﻋﻠﻰ ﻤﺤﻭﺭ ﻤﻥ ﺍﻟﻤﺤﺎﻭﺭ ﺍﻹﺤﺩﺍﺜﻴﺔ‪ ،‬ﻭﻋﻨﺩﻤﺎ ﻨﺒﺩل ‪ y‬ﺒـ ‪-y‬‬
‫ﻴﺠﺏ ﺃﻻ ﻴﺘﻐﻴﺭ ﺸﻲﺀ‪ ،‬ﻭﻟﻜﻥ ﻫﺫﺍ ﻴﺅﺩﻱ ﺇﻟﻰ ﺘﻐﻴﺭ ‪ exy‬ﺇﻟﻰ ‪ - exy‬ﻭﺘﺼﺒﺢ ﺍﻹﺯﺍﺤﺔ‬
‫ﻓﻲ ﺍﺘﺠﺎﻩ ﺍﻟﻤﺤﻭﺭ ‪ -y‬ﺒﺩﻻﹰ ﻤﻥ ﺍﻹﺯﺍﺤﺔ ﻓﻲ ﺍﺘﺠﺎﻩ ﺍﻟﻤﺤﻭﺭ‪ .y‬ﻟﻜﻲ ﻻ ﺘﺘﻐﻴﺭ ﺍﻟﻁﺎﻗﺔ‬
‫ﻓﻲ ﻫﺫﻩ ﺍﻟﺤﺎﻟﺔ ﻴﺠﺏ ﺘﺒﺩﻴل ‪ Cxxxy‬ﺇﻟﻰ ‪ ، - Cxxxy‬ﻭﻟﻜﻥ ﺍﻨﻌﻜﺎﺱ ﺍﻟﺒﻠﻭﺭﺓ ﻻ ﻴﺘﻐﻴﺭ‬
‫ﻜﻤﺎ ﺫﻜﺭﻨﺎ ﺴﺎﺒﻘﺎﹰ‪ ،‬ﻟﺫﻟﻙ ﻓﺄﻥ ‪ Cxxxy‬ﺘﺒﻘﻰ ﻤﺴﺎﻭﻴﺔ ﺇﻟﻰ ‪ ، - Cxxxy‬ﻭﻫﺫﺍ ﻏﻴﺭ ﻤﻤﻜﻥ‬
‫ﺇﻻ ﻋﻨﺩﻤﺎ ﻴﻜﻭﻥ ﻜلٍِ ﻤﻨﻬﻤﺎ ﻤﺴﺎﻭﻴﺎﹰ ﻟﻠﺼﻔﺭ‪.‬‬
‫ﻗﺩ ﻴﺘﺒﺎﺩﺭ ﺇﻟﻰ ﺍﻟﺫﻫﻥ ﺃﻨﻪ ﻴﻤﻜﻥ ﻭﻀﻊ ‪ ،Cyyyy=0‬ﻭﻟﻜﻥ ﻫﺫﺍ ﻏﻴﺭ ﻤﻤﻜﻥ ﻷﻥ‬
‫‪ y‬ﺘﺘﻜﺭﺭ ﺃﺭﺒﻊ ﻤﺭﺍﺕ ‪ ،‬ﺃﻱ ﺃﻥ ‪ y‬ﺘﻐﻴﺭ ﺇﺸﺎﺭﺘﻬﺎ ﺍﻟﺴﺎﻟﺒﺔ ﺃﺭﺒﻊ ﻤﺭﺍﺕ ﻤﻤﺎ ﻴﻌﻁﻲ‬
‫ﺩﺍﺌﻤﺎﹰ ﺇﺸﺎﺭﺓ ﻤﻭﺠﺒﺔ ﺍﻷﻤﺭ ﺫﺍﺘﻪ ﻋﻨﺩﻤﺎ ﺘﺘﻜﺭﺭ ‪ y‬ﻤﺭﺘﻴﻥ‪ ،‬ﻭﺃﻥ ﻤﺭﻜﺒﺎﺕ ‪ C‬ﻓﻲ ﻫﺎﺘﻴﻥ‬
‫ﺍﻟﺤﺎﻟﺘﻴﻥ ﻴﺠﺏ ﺃﻥ ﻻ ﺘﺴﺎﻭﻱ ﺼﻔﺭ‪ ،‬ﻭﺇﻨﻤﺎ ﺍﻟﻤﺭﻜﺒﺎﺕ ﺘﺴﺎﻭﻱ ﺼﻔﺭﺍﹰ ﻓﻘﻁ ﻋﻨﺩﻤﺎ‬
‫ﺘﺘﻜﺭﺭ ﻓﻴﻬﺎ ‪ y‬ﻋﺩﺩ ﺍﹰ ﻓﺭﺩﻴﺎﹰ ﻤﻥ ﺍﻟﻤﺭﺍﺕ )ﻤﺭﺓ ﺃﻭ ﺜﻼﺙ ﻤﺭﺍﺕ(‪ .‬ﻭﺒﺎﻟﺘﺎﻟﻲ ﻓﺈﻨﻪ ﻓﻲ‬
‫ﺍﻟﺒﻠﻭﺭﺓ ﺍﻟﻤﻜﻌﺒﺔ ﺘﻜﻭﻥ ‪ C‬ﻤﺨﺘﻠﻔﺔ ﻋﻥ ﺍﻟﺼﻔﺭ ﻋﻨﺩﻤﺎ ﻴﻜﻭﻥ ﺍﻟﺩﻟﻴل ﻤﻜﺭﺭ ﻋﺩﺩ ﺍﹰ‬
‫ﺯﻭﺠﻴﺎﹰ ﻤﻥ ﺍﻟﻤﺭﺍﺕ ﻭﺘﻜﻭﻥ ﻤﺴﺎﻭﻴﺔ ﻟﻠﺼﻔﺭ ﻋﻨﺩﻤﺎ ﻴﻜﻭﻥ ﺍﻟﺩﻟﻴل ﻤﻜﺭﺭ ﺍﹰ ﻋﺩﺩﺍﹰ ﻓﺭﺩﻴﺎﹰ‬

‫‪- 232 -‬‬

‫)‪Create PDF files without this message by purchasing novaPDF printer (http://www.novapdf.com‬‬
‫ﻤﻥ ﺍﻟﻤﺭﺍﺕ‪ ،‬ﻭﺒﻨﻔﺱ ﺍﻟﻁﺭﻴﻘﺔ ﻴﻤﻜﻥ ﻤﻨﺎﻗﺸﺔ ﺫﻟﻙ ﻤﻥ ﺃﺠل ‪ x‬ﻭ‪ .z‬ﺘﻜﻭﻥ ﺇﺫﻥ‬
‫ﺍﻟﻤﺭﻜﺒﺎﺕ ﺍﻟﻤﺨﺘﻠﻔﺔ ﻋﻥ ﺍﻟﺼﻔﺭ ﻫﻲ ﺍﻟﻤﺭﻜﺒﺎﺕ ﺍﻟﺘﻲ ﻟﻬﺎ ﺍﻟﺼﻴﻐﺔ ﺍﻟﺘﺎﻟﻴﺔ‪:‬‬
‫……‪ ، Cxyyx ،Cxyxy ،Cxxyy‬ﻭﻗﺩ ﻭﺠﺩﻨﺎ ﺃﻥ ﺘﺒﺩﻴل ﻜل ‪ x‬ﺒـ ‪ y‬ﺃﻭ‬
‫ﺍﻟﻌﻜﺱ )ﺃﻭ ‪ z‬ﺒـ ‪ x‬ﺍﻟﺦ ( ﻓﻲ ﺍﻟﺒﻠﻭﺭﺓ ﺍﻟﻤﻜﻌﺒﺔ‪ ،‬ﻓﺎﻥ ﻋﺩﺩ ﺍﻟﻤﺭﻜﺒﺎﺕ ﻻ ﻴﺘﻐﻴﺭ‬
‫ﻭﻫﺫﺍ ﻴﻌﻨﻲ ﺃﻨﻪ ﺘﻭﺠﺩ ﺜﻼﺙ ﺇﻤﻜﺎﻨﻴﺎﺕ ﻤﻐﺎﻴﺭﺓ ﻟﻠﺼﻔﺭ‪.‬‬
‫‪C xxxx  Cyyyy  Czzzz ‬‬
‫‪C xxyy  Cyyxx  Cxxzz  ....‬‬ ‫)‪(7.21‬‬
‫‪C xyxy  Cyxyx  Cxzxz  ....‬‬
‫ﺘﻌﻁﻰ ﺒﺎﻟﺘﺎﻟﻲ ﻜﺜﺎﻓﺔ ﺍﻟﻁﺎﻗﺔ ﻓﻲ ﺍﻟﺒﻠﻭﺭﺓ ﺍﻟﻤﻜﻌﺒﺔ ﺒﺎﻟﻌﻼﻗﺔ ﺍﻟﺘﺎﻟﻴﺔ‪:‬‬
‫‪2‬‬
‫‪‬‬ ‫‪2‬‬ ‫‪2‬‬
‫‪‬‬
‫‪1 Cxxxx e xx  e yy  e zz  2C xxyy e xx e yy  e yyzz  e zz e xx ‬‬
‫‪(7.22) ω  ‬‬ ‫‪‬‬
‫‪‬‬
‫‪2  4C xyxy e 2xy  e 2yz  e 2zx‬‬ ‫‪‬‬ ‫‪‬‬
‫ﻭﻓﻲ ﺍﻟﻤﻭﺍﺩ ﺍﻟﻤﺘﺠﺎﻨﺴﺔ ) ﺇﻴﺯﻭﺘﺭﻭﺒﻴﺔ( ﻏﻴﺭ ﺍﻟﺒﻠﻭﺭﻴﺔ‪ ،‬ﻴﺠﺏ ﺃﻥ ﻴﺒﻘﻰ ﻋﺩﺩ ‪C‬‬
‫ﻨﻔﺴﻪ ﻜﻴﻔﻤﺎ ﺘﻡ ﺍﺨﺘﻴﺎﺭ ﻤﺤﺎﻭﺭ ﺍﻹﺤﺩﺍﺜﻴﺎﺕ‪ ،‬ﻭﺘﻭﺠﺩ ﻓﻲ ﻫﺫﻩ ﺍﻟﺤﺎﻟﺔ ﻋﻼﻗﺔ ﺃﺨﺭﻯ ﺒﻴﻥ‬
‫ﺍﻟﻤﻌﺎﻤﻼﺕ ‪: C‬‬
‫‪C xxxx  C xxyy  C xyxy‬‬ ‫)‪(7.23‬‬
‫ﻴﻤﻜ ﻥ ﻤﻼﺤﻅﺔ ﺫﻟﻙ ﻤﻥ ﺍﻟﻤﻌﺎﻟﺠﺔ ﺍﻟﺘﺎﻟﻴﺔ‪ :‬ﻴﺠﺏ ﺃﻥ ﻴﺭﺘﺒﻁ ﺘﻨﺴﻭﺭ ﺍﻹﺠﻬﺎﺩ‬
‫‪ Sij‬ﻤﻊ ‪ eij‬ﺒﻁﺭﻴﻘﺔ ﺘﺠﻌل ﻫﺫﺍ ﺍﻻﺭﺘﺒﺎﻁ ﻻ ﻴﻌﺘﻤﺩ ﻋﻠﻰ ﺍﺘﺠﺎﻩ ﺍﻟﻤﺤﺎﻭﺭ ﺍﻹﺤﺩﺍﺜﻴﺔ‪،‬‬
‫ﺃﻱ ﺃﻥ ﺍﻻﺭﺘﺒﺎﻁ ﻫﻭ ﺍﺭﺘﺒﺎﻁ ﻋﺩﺩﻱ‪ .‬ﻴﻤﻜﻥ ﻤﺜﻼﹰ ﺍﻟﺤﺼﻭل ﻋﻠﻰ ‪ Sij‬ﻤﻥ ‪ eij‬ﻭﺫﻟﻙ‬
‫ﻤﻥ ﻀﺭﺏ ‪ eij‬ﺒﻌﺩﺩ ﺜﺎﺒﺕ‪ ،‬ﻓﻨﺤﺼل ﻋﻠﻰ ﻗﺎﻨﻭﻥ ﻫﻭﻙ ‪ ، Sij  const  eij‬ﻭﻟﻜﻥ‬
‫ﻫﺫﻩ ﺍﻟﻌﻼﻗﺔ ﻏﻴﺭ ﺩﻗﻴﻘﺔ ﺘﻤﺎﻤﺎﹰ‪ ،‬ﻭﺇﻨﻤﺎ ﻴﻤﻜﻥ ﺃﻥ ﻨﻠﺤﻕ ﺘﻨﺴﻭﺭ ﺍﻟﻭﺍﺤﺩﺓ ‪ δ ij‬ﻤﻀﺭﻭﺒﺎﹰ‬
‫ﻋﺩﺩﻴﺎﹰ ﺒﻌﻼﻗﺔ ﻤﺭﺘﺒﻁﺔ ﺨﻁﻴﺔ ﻤﻊ ‪ ،eij‬ﻭﺃﻥ ﺍﻟﺘﻌﺒﻴﺭ ﺍﻟﺠﺒﺭﻱ ﺍﻟﺜﺎﺒﺕ ﻭﺍﻟﺨﻁﻲ ﻟـ ‪e‬‬
‫ﻫﻭ ‪) ،  e jj‬ﻤﺸﺎﺒﻪ ﻟﻠﺘﻌﺒﻴﺭ ‪ x2+y2+z2‬ﻴﻌﺘﺒﺭ ﻤﻘﺩﺍﺭﺍﹰ ﻋﺩﺩﻴﺎﹰ(‪.‬‬
‫ﺘﻌﻁﻰ ﻋﻨﺩﺌ ﺫ‪ ‬ﺍﻟﻤﻌﺎﺩﻟﺔ ﺍﻟﻌﺎﻤﺔ ﺍﻟﺘﻲ ﺘﺭﺒﻁ ﺒﻴﻥ ﺍﻟﺘﻨﺴﻭﺭ ‪ Sij‬ﻭ ‪ eij‬ﻟﻤﺎﺩﺓ‬
‫ﺍﻴﺯﻭﺘﺭﻭﺒﻴﺔ )ﻤﺘﺠﺎﻨﺴﺔ( ﺒﺎﻟﻌﻼﻗﺔ‪:‬‬
‫‪‬‬ ‫‪‬‬
‫‪Sij  2 μ e ij  λ   e kk  δ ij‬‬ ‫)‪(7.24‬‬
‫‪ k‬‬ ‫‪‬‬

‫‪- 233 -‬‬

‫)‪Create PDF files without this message by purchasing novaPDF printer (http://www.novapdf.com‬‬
‫ﺘﺴﻤﻰ ﺍﻟﺜﻭﺍﺒﺕ ‪ μ‬ﻭ ‪ λ‬ﺜﻭﺍﺒﺕ ﻻﻭﻱ ﻟﻠﻤﺭﻭﻨﺔ‪ .‬ﻴﻨﺘﺞ ﻤﻥ ﻤﻘﺎﺭﻨﺔ ﺍﻟﻤﻌﺎﺩﻟﺘﻴﻥ‬
‫)‪ (7.24‬ﻭ )‪ ،(7.16‬ﺃﻥ‪:‬‬
‫‪C xxyy  λ‬‬
‫‪C xyxy  2μ‬‬ ‫)‪(7.25‬‬
‫‪C xxxx  2μ  λ‬‬
‫ﻭﺒﺫﻟﻙ ﻨﻜﻭﻥ ﻗﺩ ﺃﺜﺒﺘﻨﺎ ﺃﻥ ﺍﻟﻌﻼﻗﺔ )‪ ،(7.23‬ﺼﺤﻴﺤﺔ‪ .‬ﻴﺘﻀﺢ ﺃﻥ ﻤﻌﺎﻤل‬
‫ﻤﺭﻭﻨﺔ ﻤﺎﺩﺓ ﻤﺘﺠﺎﻨﺴﺔ ﻴﻤﻜﻥ ﺍﻟﺘﻌﺒﻴﺭ ﻋﻨﻪ ﺒﻭﺍﺴﻁﺔ ﺜﺎﺒﺘﻲ ﻤﺭﻭﻨﺔ‪.‬‬
‫ﻴﻤﻜﻥ ﺍﻟﺘﻌﺒﻴﺭ ﻋﻥ ﺍﻟﻤﻌﺎﻤﻼﺕ ‪ C‬ﺒﻭﺍﺴﻁﺔ ﺜﺎﺒﺘﻴﻥ ﻜﻴﻔﻴﻴﻥ ﻤﻥ ﺜﻭﺍﺒﺕ ﺍﻟﻤﺭﻭﻨﺔ‬
‫ﺍﻟﺘﻲ ﻤﺭﺕ ﺴﺎﺒﻘﺎﹰ‪ ،‬ﻤﺜﺎل ﻋﻠﻰ ﺫﻟﻙ ﺒﺎﺴﺘﺨﺩﺍﻡ ﻤﻌﺎﻤل ﻴﻭﻨﻎ ‪ Y‬ﻭﻨﺴﺒﺔ ﺒﻭﺍﺼﻭﻥ ‪،σ‬‬
‫ﻴﻜﻭﻥ‪:‬‬
‫‪Y ‬‬ ‫‪σ ‬‬
‫‪C xxxx ‬‬ ‫‪1 ‬‬ ‫‪‬‬
‫‪1  σ  1  2σ ‬‬
‫‪Y  σ ‬‬
‫‪C xxyy ‬‬ ‫‪‬‬ ‫‪‬‬ ‫)‪(7.26‬‬
‫‪1  σ  1  2σ ‬‬
‫‪Y‬‬
‫‪C xyxy‬‬ ‫‪‬‬
‫‪1 σ‬‬
‫ﺘﻌﺭﻑ ﻨﺴﺒﺔ ﺒﻭﺍﺼﻭﻥ ‪ σ‬ﻤﻥ ﺍﻟﻌﻼﻗﺔ ﺍﻟﺘﻲ ﺘﺭﺒﻁ ﺒﻴﻥ ﺍﻟﻘﻭﺓ ﻭﺍﻻﺴﺘﻁﺎﻟﺔ‪.‬‬
‫ﻋﻨﺩﻤﺎ ﻴﺨﻀﻊ ﺠﺴﻡ ﻋﻠﻰ ﺸﻜل ﻤﺘﻭﺍﺯﻱ ﻤﺴﺘﻁﻴﻼﺕ ﺇﻟﻰ ﻗﻭﻯ ﺸﺩ ﻋﻠﻰ ﺍﻤﺘﺩﺍﺩ ﺍﺤﺩ‬
‫ﺃﺒﻌﺎﺩﻩ )ﻋﻠﻰ ﺍﻤﺘﺩﺍﺩ ﺍﻟﻁﻭل ‪ l‬ﻤﺜﻼﹰ(‪ ،‬ﻓﺈﻨﻪ ﻴﺴﺘﻁﻴل ﺒﺎﻟﻤﻘﺩﺍﺭ ‪ ، l‬ﻭﻓﻲ ﻨﻔﺱ ﺍﻟﻭﻗﺕ‬
‫ﻴﻨﻀﻐﻁ ﺍﻟﺠﺴﻡ ﻭﻓﻕ ﺍﻟﺒﻌﺩﻴﻥ ﺍﻵﺨﺭﻴﻥ ﻜﻤﺎ ﻫﻭ ﻤﻭﻀﺢ ﻋﻠﻰ ﺍﻟﺸﻜل )‪،(7.8‬‬
‫ﻭﺘﺘﺤﻘﻕ ﺍﻟﻌﻼﻗﺔ ﺍﻟﺘﺎﻟﻴﺔ‪:‬‬
‫‪d h‬‬ ‫‪‬‬
‫‪‬‬ ‫‪ ‬‬
‫‪d‬‬ ‫‪h‬‬ ‫‪‬‬
‫ﺤﻴﺙ ‪ ‬ﻭ ‪ h‬ﻭ ‪ d‬ﺃﺒﻌﺎﺩ ﻤﺘﻭﺍﺯﻱ ﺍﻟﻤﺴﺘﻁﻴﻼﺕ‪ ،‬ﻭﻴﺴﻤﻰ ﺍﻟﺜﺎﺒﺕ ‪ σ‬ﻨﺴﺒﺔ‬
‫‪.‬‬ ‫ﺒﻭﺍﺼﻭﻥ‪ ،‬ﻭﻫﺫﺍ ﺍﻟﻌﺩﺩ ﻤﻭﺠﺏ ﺍﻹﺸﺎﺭﺓ ﻭﻗﻴﻤﺘﻪ ﺍﻟﻌﺩﺩﻴﺔ ﺃﻗل ﻤﻥ‬
‫‪1‬‬
‫‪2‬‬

‫‪- 234 -‬‬

‫)‪Create PDF files without this message by purchasing novaPDF printer (http://www.novapdf.com‬‬
‫ﺍﻟﺸﻜل)‪(7.8‬‬
‫ﺍﺴﺘﻁﺎﻟﺔ ﻟﻭﺡ ﻋﻠﻰ ﺸﻜل ﻤﺘﻭﺍﺯﻱ ﻤﺴﺘﻁﻴﻼﺕ ﺘﺤﺕ ﺘﺄﺜﻴﺭ ﻗﻭﺓ ﻤﻨﺘﻅﻤﺔ‪.‬‬

‫‪ - 57‬ﺍﳊﺮﻛﺔ ﰲ ﺟﺴﻢ ﺍﳌﺮﻥ‪:‬‬

‫ﺃﺸﺭﻨﺎ ﺴﺎﺒﻘﺎﹰ ﺃﻥ ﺍﻹﺠﻬﺎﺩ ﺩﺍﺨل ﺍﻟﺠﺴﻡ ﺍﻟﻤﺭﻥ ﺍﻟﻤﺘﻭﺍﺯﻥ‪ ،‬ﻴﺘﻭﺯﻉ ﺒﺤﻴﺙ ﺘﻜﻭﻥ‬
‫ﻗﻴﻤﺔ ﺍﻟﻁﺎﻗﺔ ﺼﻐﺭﻯ‪ .‬ﻨﺩﺭﺱ ﺍﻟﺤﺎﻟﺔ ﺍﻟﺘﻲ ﺘﻜﻭﻥ ﻓﻴﻬﺎ ﺍﻟﻘﻭﻯ ﺍﻟﺩﺍﺨﻠﻴﺔ ﻏﻴﺭ ﻤﺘﺯﻨﺔ‪،‬‬
‫ﻟﺫﻟﻙ ﻨﺄﺨﺫ ﺠﺯﺀﺍﹰ ﺼﻐﻴﺭﺍﹰ ﻤﻥ ﺩﺍﺨل ﺍﻟﻤﺎﺩﺓ ﻤﺴﺎﺤﺘﻪ ﺍﻟﺨﺎﺭﺠﻴﺔ ﺘﺴﺎﻭﻱ ‪ ،A‬ﻜﻤﺎ ﻫﻭ‬
‫‪‬‬
‫ﻤﺒﻴﻥ ﻋﻠﻰ ﺍﻟﺸﻜل )‪ .(7.9‬ﺇﺫﺍ ﻜﺎﻥ ﻫﺫﺍ ﺍﻟﺠﺯﺀ ﻓﻲ ﺤﺎﻟﺔ ﺍﺘﺯﺍﻥ‪ ،‬ﻓﺈﻥ ‪ F‬ﻤﺠﻤﻭﻉ‬
‫ﺍﻟﻘﻭﻯ ﺍﻟﻤﺅﺜﺭﺓ ﻋﻠﻴﻪ ﻴﺠﺏ ﺃﻥ ﻴﺴﺎﻭﻱ ﺍﻟﺼﻔﺭ‪ .‬ﻴﻤﻜﻥ ﺍﻋﺘﺒﺎﺭ ﺃﻥ ﻫﺫﻩ ﺍﻟﻘﻭﻯ ﻤﻜﻭﻨﺔ‬
‫ﻤﻥ ﺠﺯﺃﻴﻥ‪ ،‬ﺠﺯﺀ ﻨﺎﺘﺞ ﻋﻥ ﺘﺄﺜﻴﺭ ﺍﻟﻘﻭﻯ ﺍﻟﺩﺍﺨﻠﻴﺔ ‪ ، Fint‬ﻭﺠﺯﺀ ﻨﺎﺘﺞ ﻋﻥ ﺘﺄﺜﻴﺭ‬
‫ﺍﻟﻘﻭﻯ ﺍﻟﺨﺎﺭﺠﻴﺔ ﻤﺜل ﻗﻭﺓ ﺍﻟﺠﺎﺫﺒﻴﺔ‪ ،‬ﺍﻟﺘﻲ ﺘﺅﺜﺭ ﻋﻥ ﺒﻌﺩ ﻋﻠﻰ ﺠﺯﺀ ﺍﻟﻤﺎﺩﺓ‪ ،‬ﻭﻴﺭﻤﺯ‬
‫ﻟﻌﻨﺼﺭ ﺍﻟﻘﻭﺓ ﺍﻟﺨﺎﺭﺠﻴﺔ ﺍﻟﻤﺅﺜﺭﺓ ﻋﻠﻰ ﻭﺤﺩﺓ ﺍﻟﺤﺠﻡ ﺒﺎﻟﺭﻤﺯ ‪ .fext‬ﻴﻤﻜﻥ ﺇﻴﺠﺎﺩ ﺍﻟﻘﻭﺓ‬
‫ﺍﻟﺨﺎﺭﺠﻴﺔ ﺍﻟﻜﻠﻴﺔ ‪ Fext‬ﻤﻥ ﺤﺴﺎﺏ ﺘﻜﺎﻤل ﻟﻠﻘﻭﺓ ‪ fext‬ﺍﻟﻤﺅﺜﺭﺓ ﻋﻠﻰ ﻋﻨﺼﺭ ﺍﻟﺤﺠﻡ ‪dv‬‬
‫ﻭﺫﻟﻙ ﻋﻠﻰ ﻜﺎﻤل ﺤﺠﻡ ﺍﻟﺠﺯﺀ ﺍﻟﻤﺄﺨﻭﺫ ﻤﻥ ﺍﻟﻤﺎﺩﺓ‪.‬‬
‫‪Fext   f ext dv‬‬ ‫)‪(7.27‬‬

‫‪- 235 -‬‬

‫)‪Create PDF files without this message by purchasing novaPDF printer (http://www.novapdf.com‬‬
‫ﺍﻟﺸﻜل)‪ ،(7.9‬ﻋﻨﺼﺭ ﺤﺠﻡ ‪ v‬ﺴﻁﺤﻪ ‪A‬‬

‫ﺘﺘﻌﺎﺩل ﺍﻟﻘﻭﺓ ﺍﻟﺨﺎﺭﺠﻴﺔ ‪ Fext‬ﻤﻊ ﺍﻟﻘﻭﺓ ﺍﻟﺩﺍﺨﻠﻴﺔ ‪ Fint‬ﻓﻲ ﺤﺎﻟﺔ ﺍﻻﺘﺯﺍﻥ‪،‬‬


‫ﻭﻋﻨﺩﻤﺎ ﻴﻜﻭﻥ ﺠﺯﺀ ﺍﻟﻤﺎﺩﺓ ﻓﻲ ﺤﺎﻟﺔ ﻋﺩﻡ ﺍﺘﺯﺍﻥ‪ ،‬ﻓﺈﻥ ﻤﺠﻤﻭﻉ ﺍﻟﻘﻭﺘﻴﻥ ﺍﻟﺩﺍﺨﻠﻴﺔ‬
‫ﻭﺍﻟﺨﺎﺭﺠﻴﺔ ﻴﻜﻭﻥ ﻤﺴﺎﻭﻴﺎﹰ ﺇﻟﻰ ﻜﺘﻠﺔ ﺍﻟﺠﺴﻡ ﻓﻲ ﺘﺴﺎﺭﻋﻪ‪:‬‬
‫‪‬‬ ‫‪‬‬ ‫‪‬‬
‫‪Fext  Fin    r dv‬‬ ‫)‪(7.28‬‬
‫‪V‬‬

‫ﺤﻴﺙ ‪ ρ‬ﺍﻟﻜﺜﺎﻓﺔ ﺍﻟﺤﺠﻤﻴﺔ ﻟﻠﺠﺴﻡ‪ r ،‬ﺘﺴﺎﺭﻋﻪ‪ .‬ﻴﻤﻜﻥ ﺍﻻﺴﺘﻔﺎﺩﺓ ﻤﻥ ﺍﻟﻤﻌﺎﺩﻟﺔ‬


‫)‪ .(7.27‬ﻓﻲ ﻜﺘﺎﺒﺔ ﺍﻟﻤﻌﺎﺩﻟﺔ )‪ ،(7.28‬ﺒﺎﻟﺼﻭﺭﺓ ﺍﻟﺘﺎﻟﻴﺔ‪:‬‬
‫‪‬‬ ‫‪‬‬ ‫‪‬‬
‫‪‬‬ ‫‪‬‬
‫‪Fin    f ext  ρ r dv‬‬ ‫)‪(7.29‬‬
‫‪V‬‬

‫ﺒﻔﺭﺽ ﺃﻥ‪:‬‬
‫‪‬‬ ‫‪‬‬ ‫‪‬‬
‫‪f  f ext  ρ r‬‬ ‫)‪(7.30‬‬
‫ﻴﻤﻜﻥ ﻋﻨﺩﺌﺫ‪ ‬ﻜﺘﺎﺒﺔ ﺍﻟﻤﻌﺎﺩﻟﺔ )‪ ،(7.29‬ﺒﺎﻟﺼﻭﺭﺓ ﺍﻟﺘﺎﻟﻴﺔ‪:‬‬
‫‪‬‬ ‫‪‬‬
‫‪Fin   fdv‬‬ ‫)‪(7.31‬‬
‫‪v‬‬

‫ﺘﻌﺘﻤﺩ ﻗﻴﻤﺔ ﺍﻟﻘﻭﺓ ﺍﻟﺩﺍﺨﻠﻴﺔ ‪ Fin‬ﻋﻠﻰ ﺍﻹﺠﻬﺎﺩ ﻓﻲ ﺍﻟﻤﺎﺩﺓ‪ .‬ﺇﺫﺍ ﻜﺎﻥ ‪ Sij‬ﺘﻨﺴﻭﺭ‬
‫ﺍﻹﺠﻬﺎﺩ‪،‬ﺍﻟﺫﻱ ﻴﻤﻜﻥ ﺘﻌﺭﻴﻔﻪ ﺒﺎﺨﺘﺼﺎﺭ ﻋﻠﻰ ﺍﻟﺸﻜل ﺍﻟﺘﺎﻟﻲ‪:‬‬

‫‪- 236 -‬‬

‫)‪Create PDF files without this message by purchasing novaPDF printer (http://www.novapdf.com‬‬
‫‪‬‬
‫ﻨﺄﺨﺫ ﻤﻘﻁﻊ ﻤﻥ ﺍﻟﻤﺎﺩﺓ ﻋﻤﻭﺩﻱ ﻋﻠﻰ ﺍﻟﻤﺤﻭﺭ ‪ ، x‬ﻭﻨﺤﻠل ﺍﻟﻘﻭﺓ ‪F1‬‬
‫ﺍﻟﻤﺅﺜﺭﺓ ﻋﻠﻰ ﻫﺫﺍ ﺍﻟﻤﻘﻁﻊ ﺇﻟﻰ ﻤﺭﻜﺒﺎﺘﻬﺎ ‪ Fx1 , Fy1 , Fz1‬ﻜﻤﺎ ﻫﻭ ﻤﺒﻴﻥ‬
‫‪ yz‬ﺒـ‬ ‫ﻋﻠﻰ ﺍﻟﺸﻜل )‪ ،(7.10‬ﺘﺴﻤﻰ ﻨﺴﺒﺔ ﻫﺫﻩ ﺍﻟﻘﻭﻯ ﺇﻟﻰ‬
‫‪ . s xx , s yx , s zx‬ﻤﺜﻼﹰ‪:‬‬
‫‪ΔFy1‬‬
‫‪s yx ‬‬
‫‪ΔyΔz‬‬

‫ﺍﻟﺸﻜل )‪ (7.10‬ﺍﻟﻘﻭﺓ ‪ ∆F1‬ﺍﻟﻤﺅﺜﺭﺓ ﻋﻠﻰ ﻋﻨﺼﺭ ﺍﻟﻤﺴﺎﺤﺔ ‪ yz‬ﺍﻟﻌﻤﻭﺩﻱ ﻋﻠﻰ ﺍﻟﻤﺤﻭﺭ ‪،x‬‬
‫ﻭﻤﺭﻜﺒﺎﺕ ﻫﺫﻩ ﺍﻟﻘﻭﺓ ‪ Fx1 , Fy1, Fz1‬ﻋﻠﻰ ﺍﻟﻤﺤﺎﻭﺭ ﺍﻹﺤﺩﺍﺜﻴﺔ ‪.‬‬

‫ﻴﺭﻤﺯ ﺍﻟﺩﻟﻴل ﺍﻷﻭل ‪ y‬ﺇﻟﻰ ﺍﺘﺠﺎﻩ ﻤﺭﻜﺒﺔ ﺍﻟﻘﻭﺓ‪ ،‬ﻭﻴﺭﻤﺯ ﺍﻟﺩﻟﻴل ﺍﻟﺜﺎﻨﻲ ‪ x‬ﺇﻟﻰ‬
‫ﺍﺘﺠﺎﻩ ﺍﻟﻨﺎﻅﻡ ﻋﻠﻰ ﺍﻟﻤﺴﺘﻭﻯ‪ ،‬ﻭﻴﻤﻜﻥ ﻜﺘﺎﺒﺔ ﺍﻟﻌﻼﻗﺔ ﺍﻟﺴﺎﺒﻘﺔ ﺒﺎﻟﺸﻜل ﺍﻟﺘﺎﻟﻲ‪:‬‬
‫‪Fy1‬‬
‫‪s yx ‬‬
‫‪ax‬‬
‫ﺤﻴﺙ ‪ Δa x  ΔyΔz‬ﻋﻨﺼﺭ ﻤﺴﺎﺤﺔ ﻋﻤﻭﺩﻱ ﻋﻠﻰ ﺍﻟﻤﺤﻭﺭ ‪ ،x‬ﻭﻫﻜﺫﺍ‬
‫ﺒﺎﻟﻨﺴﺒﺔ ﻟﺒﻘﻴﺔ ﺍﻻﺘﺠﺎﻫﺎﺕ‪.‬‬
‫ﺒﻌﺩ ﺘﻌﺭﻴﻑ ﺘﻨﺴﻭﺭ ﺍﻹﺠﻬﺎﺩ ﻴﻤﻜﻥ ﺤﺴﺎﺏ ﺍﻟﻘﻭﺓ ﺍﻟﺩﺍﺨﻠﻴﺔ ﻜﻤﺎ ﻴﻠﻲ‪:‬‬
‫ﺒﻔﺭﺽ ﺃﻥ‪ dFx :‬ﻤﺭﻜﺒﺔ ﺍﻟﻘﻭﺓ ﻋﻠﻰ ﺍﻟﻤﺤﻭﺭ ‪ x‬ﺘﺅﺜﺭ ﻋﻠﻰ ﻋﻨﺼﺭ ﺍﻟﻤﺴﺎﺤﺔ‬
‫‪ ،da‬ﻓﺄﻨﻬﺎ ﺘﻌﻁﻰ ﺒﺎﻟﻌﻼﻗﺔ ﺍﻟﺘﺎﻟﻴﺔ‪:‬‬
‫‪dFx  S xx n x  S xy n y  S zx n z da‬‬ ‫)‪(7.32‬‬

‫‪- 237 -‬‬

‫)‪Create PDF files without this message by purchasing novaPDF printer (http://www.novapdf.com‬‬
‫‪‬‬
‫ﺤﻴﺙ ‪ n‬ﻤﺘﺠﻪ ﻋﻤﻭﺩﻱ ﻋﻠﻰ ﻋﻨﺼﺭ ﺍﻟﺴﻁﺢ ‪ .da‬ﻴﻤﻜﻥ ﻓﻲ ﻫﺫﻩ ﺍﻟﺤﺎﻟﺔ‬
‫‪‬‬
‫ﺤﺴﺎﺏ ﺍﻟﻘﻭﺓ ﺍﻟﺩﺍﺨﻠﻴﺔ ‪ Fin‬ﺍﻟﻤﺅﺜﺭﺓ ﻋﻠﻰ ﺠﺯﺀ ﺍﻟﻤﺎﺩﺓ ﺒﺎﺘﺠﺎﻩ ﺍﻟﻤﺤﻭﺭ ‪ x‬ﻤﻥ ﺤﺴﺎﺏ‬
‫ﺍﻟﺘﻜﺎﻤل ﻟﻌﻨﺼﺭ ﺍﻟﻘﻭﺓ ‪ dFx‬ﺒﺎﻟﻨﺴﺒﺔ ﻟﻜل ﺍﻟﺴﻁﺢ ‪ A‬ﻭﺫﻟﻙ ﺒﺎﻟﺘﻌﻭﻴﺽ ﻤﻥ ﺍﻟﻤﻌﺎﺩﻟﺔ‬
‫)‪ ،(7.32‬ﻓﻲ ﺍﻟﻤﻌﺎﺩﻟﺔ )‪ ،(7.31‬ﻭﺇﺠﺭﺍﺀ ﺍﻟﺘﻜﺎﻤل ﻋﻠﻰ ﻜﺎﻤل ﺍﻟﺴﻁﺢ‪.‬‬
‫‪ S‬‬ ‫‪xx‬‬ ‫‪n x  S xy n y  S xz n z da   f x dv‬‬ ‫)‪(7.33‬‬
‫‪A‬‬ ‫‪v‬‬

‫ﺘﺭﺒﻁ ﻫﺫﻩ ﺍﻟﻤﻌﺎﺩﻟﺔ ﺍﻟﻌﻼﻗﺔ ﺒﻴﻥ ﺍﻟﺘﻜﺎﻤل ﻋﻠﻰ ﺴﻁﺢ ﻭﺍﻟﺘﻜﺎﻤل ﻋﻠﻰ ﺤﺠﻡ‪،‬‬
‫ﻭﺘﺫﻜﺭﻨﺎ ﻫﺫﻩ ﺍﻟﻌﻼﻗﺔ ﺒﻘﻭﺍﻨﻴﻥ ﺍﻟﻜﻬﺭﺒﺎﺀ‪ .‬ﻓﺈﺫﺍ ﻟﻡ ﻨﺄﺨﺫ ﺒﻌﻴﻥ ﺍﻻﻋﺘﺒﺎﺭ ﺇﺸﺎﺭﺓ ‪ x‬ﻓﻲ‬
‫ﺍﻟﻁﺭﻑ ﺍﻷﻴﺴﺭ ﻭﺫﻟﻙ ﺒﺎﻟﻨﺴﺒﺔ ﻟﻜل ﻤﺭﻜﺒﺎﺕ ‪ S‬ﻓﻲ ﺍﻟﻤﻌﺎﺩﻟﺔ )‪ ،(7.33‬ﻓﺈﻥ ﻤﺎ ﺩﺍﺨل‬
‫‪‬‬
‫ﺍﻟﺘﻜﺎﻤل ﻴﺅﻭل ﺇﻟﻰ ﻀﺭﺏ ﺍﻟﻤﺘﺠﻬﻴﻥ ‪ ، S.n ‬ﻭﻴﺴﺎﻭﻱ ﻫﺫﺍ ﺍﻟﻀﺭﺏ ﺇﻟﻰ ﺘﺩﻓﻕ‬
‫‪‬‬
‫ﺍﻟﻤﺘﺠﻪ ‪ S‬ﺨﻼل ﺍﻟﺤﺠﻡ ‪ .V‬ﻴﻤﻜﻥ ﺒﺎﺴﺘﺨﺩﺍﻡ ﻨﻅﺭﻴﺔ ﻏﺎﻭﺹ‪ ،‬ﻜﺘﺎﺒﺔ ﺍﻟﺘﺩﻓﻕ ﻓﻲ‬
‫ﺍﻟﻁﺭﻑ ﺍﻷﻴﺴﺭ ﻤﻥ ﺍﻟﻤﻌﺎﺩﻟﺔ )‪ ،(7.33‬ﻜﺘﻜﺎﻤل ﺤﺠﻤﻲ ﻟﺘﻔﺎﻀل ‪ ،S‬ﻭﻫﺫﺍ ﻤﺤﻘﻕ‬
‫ﺒﻐﺽ ﺍﻟﻨﻅﺭ ﻋﻥ ﻭﺠﻭﺩ ﺍﻟﺩﻟﻴل ‪ x‬ﻷﻥ ﻨﻅﺭﻴﺔ ﻏﺎﻭﺹ ﻫﻲ ﻨﻅﺭﻴﺔ ﺭﻴﺎﻀﻴﺔ‪ ،‬ﻭﺒﻨﺎﺀ‪‬‬
‫ﻋﻠﻰ ﺫﻟﻙ ﺘﻜﺘﺏ ﺍﻟﻤﻌﺎﺩﻟﺔ )‪ ،(7.33‬ﺒﺎﻟﺼﻭﺭﺓ‪:‬‬
‫‪ S xx S xy S xz‬‬ ‫‪‬‬
‫‪v  x  y  z‬‬ ‫‪dv   f x dv‬‬
‫‪‬‬ ‫‪v‬‬
‫)‪(7.34‬‬

‫ﺒﺈﻫﻤﺎل ﻋﻤﻠﻴﺔ ﺍﻟﺘﻜﺎﻤل ﻓﻲ ﺍﻟﻁﺭﻓﻴﻥ ﺒﺎﻟﻨﺴﺒﺔ ﻟﻠﺤﺠﻡ‪ ،‬ﻨﺴﺘﻁﻴﻊ ﻜﺘﺎﺒﺔ ﻤﻌﺎﺩﻟﺔ‬


‫‪‬‬
‫ﺘﻔﺎﻀﻠﻴﺔ ﺒﺎﻟﻨﺴﺒﺔ ﺇﻟﻰ ﺃﻱ ﻤﺭﻜﺒﺔ ﻟـ ‪ f‬ﺒﺎﻟﺼﻭﺭﺓ‪:‬‬
‫‪S ij‬‬
‫‪fi  ‬‬ ‫)‪(7.35‬‬
‫‪j‬‬ ‫‪x j‬‬
‫ﺘﺭﺒﻁ ﻫﺫﻩ ﺍﻟﻤﻌﺎﺩﻟﺔ ﺒﻴﻥ ﺍﻟﻘﻭﺓ ﺍﻟﻤﺅﺜﺭﺓ ﻋﻠﻰ ﻭﺤﺩﺓ ﺍﻟﺤﺠﻡ ﻤﻊ ﺘﻨﺴﻭﺭ ﺍﻹﺠﻬﺎﺩ‬
‫‪. Sij‬‬
‫‪‬‬
‫ﺇﺫﺍ ﻋﺭﻑ ﻤﺘﺠﻪ ﺍﻹﺯﺍﺤﺔ ‪ u‬ﻓﺈﻨﻪ ﻴﻤﻜﻨﻨﺎ ﺇﻴﺠﺎﺩ ﺍﻟﺘﺸﻭﻩ ‪ ،eij‬ﺒﻌﺩ ﺫﻟﻙ ﻨﺤﺴﺏ‬
‫‪‬‬
‫ﺍﻹﺠﻬﺎﺩ ﻤﻥ ﺍﻟﻤﻌﺎﺩﻟﺔ )‪ ، (7.16‬ﺜﻡ ﻨﻭﺠﺩ ﻤﻥ ﺍﻟﻤﻌﺎﺩﻟﺔ )‪ ،(7.35‬ﻜﺜﺎﻓﺔ ﺍﻟﻘﻭﺓ ‪. f‬‬
‫‪‬‬ ‫‪‬‬
‫ﻨﺴﺘﻁﻴﻊ ﺒﻤﻌﺭﻓﺔ ﺍﻟﻘﻭﺓ ‪ f‬ﺇﻴﺠﺎﺩ ﺍﻟﺘﺴﺎﺭﻉ ‪ r‬ﻤﻥ ﺍﻟﻤﻌﺎﺩﻟﺔ )‪ ،(7.30‬ﺍﻟﺫﻱ ﻴﺒﻴﻥ ﻜﻴﻑ‬
‫ﺘﺘﻐﻴﺭ ﺍﻹﺯﺍﺤﺔ ﺩﺍﺨل ﺍﻟﻤﺎﺩﺓ‪ .‬ﻴﺘﻁﻠﺏ ﻫﺫﺍ ﺍﻟﺤﺴﺎﺏ ﺍﻟﺤﺼﻭل ﻋﻠﻰ ﻤﺠﻤﻭﻋﺔ‬
‫ﻤﻌﺎﺩﻻﺕ ﻤﻌﻘﺩﺓ ﻭﺼﻌﺒﺔ ﻟﺩﺭﺍﺴﺔ ﻤﺭﻭﻨﺔ ﺍﻟﺠﺴﻡ ﺍﻟﺼﻠﺏ‪ ،‬ﻟﺫﻟﻙ ﻨﻜﺘﻔﻲ ﺒﺈﻋﻁﺎﺀ ﻨﺘﻴﺠﺔ‬

‫‪- 238 -‬‬

‫)‪Create PDF files without this message by purchasing novaPDF printer (http://www.novapdf.com‬‬
‫ﺍﻟﺤﺴﺎﺏ ﻓﻲ ﺍﻟﻤﻭﺍﺩ ﺍﻟﻤﺘﺠﺎﻨﺴﺔ‪ .‬ﺇﺫﺍ ﺍﺴﺘﺨﺩﻤﻨﺎ ﺍﻟﻤﻌﺎﺩﻟﺔ )‪ ،(7.24‬ﻟﺤﺴﺎﺏ ‪،Sij‬‬
‫‪1  u i u j ‬‬
‫‪ ،‬ﻓﺈﻨﻨﺎ ﻨﺤﺼل ﻋﻠﻰ ﺍﻟﻤﻌﺎﺩﻟﺔ ﺍﻻﺘﺠﺎﻫﻴﺔ‬ ‫‪‬‬ ‫ﻭﻜﺘﺒﻨﺎ ‪ eij‬ﺒﺎﻟﺸﻜل‬
‫‪2  x j x i ‬‬

‫ﺍﻟﺘﺎﻟﻴﺔ‪:‬‬
‫‪‬‬ ‫‪ ‬‬ ‫‪‬‬
‫‪ ‬‬
‫‪f  λ  μ  .u  μ 2 u‬‬ ‫)‪(7.36‬‬
‫‪‬‬
‫ﻴﻤﻜﻥ ﻜﺘﺎﺒﺔ ﺍﻟﺼﻴﻐﺔ ﺍﻟﻌﺎﻤﺔ ﻟﻠﻘﻭﺓ ‪ f‬ﻋﻠﻰ ﺍﻟﺸﻜل‪:‬‬
‫‪‬‬ ‫‪ ‬‬ ‫‪‬‬
‫‪ ‬‬
‫‪f  a .u  b 2 u‬‬
‫ﻭﻫﺫﻩ ﺍﻟﺼﻴﻐﺔ ﻤﻜﺎﻓﺌﺔ ﻟﻠﺼﻴﻐﺔ ﻓﻲ ﺍﻟﻤﻌﺎﺩﻟﺔ )‪ ،(7.36‬ﻭﻟﻜﻥ ﺒﺜﻭﺍﺒﺕ ﻤﺤﺩﺩﺓ‬
‫‪‬‬ ‫‪‬‬ ‫‪‬‬
‫ﺃﺨﺭﻯ‪ .‬ﻗﺩ ﻴﺴﺄل ﺍﻟﻘﺎﺭﺉ ﻟﻤﺎﺫﺍ ﻟﻡ ﻴﻜﺘﺏ ﺤﺩ ﺜﺎﻟﺙ ‪     u‬ﻭﻫﻭ ﻤﺘﺠﻪ ﺃﻴﻀﺎﹰ؟‬
‫‪‬‬ ‫‪ ‬‬ ‫‪‬‬ ‫‪‬‬ ‫‪‬‬
‫ﺍﻟﺠﻭﺍﺏ ﻫﻭ ﺃﻨﻨﺎ ﻨﻌﻠﻡ ﺃﻥ ﻨﺎﺘﺞ ﻀﺭﺏ ‪     u‬ﻴﺴﺎﻭﻱ ﺇﻟﻰ ‪  2 u  u ‬ﻭﻫﺫﺍ‬
‫ﺘﺭﻜﻴﺏ ﺨﻁﻲ ﻟﺤﺩﻴﻥ ﻤﻭﺠﻭﺩﻴﻥ ﻓﻲ ﺍﻟﻤﻌﺎﺩﻟﺔ ﺍﻟﺴﺎﺒﻘﺔ‪ ،‬ﻭﺒﺎﻟﺘﺎﻟﻲ ﻓﺈﻥ ﻨﺎﺘﺞ ﺍﻟﻀﺭﺏ‬
‫‪‬‬ ‫‪‬‬ ‫‪‬‬
‫ﺍﻟﻤﺘﺠﻬﻲ ‪     u‬ﻻ ﻴﻀﻴﻑ ﺸﻴﺌﺎﹰ ﺠﺩﻴﺩﺍﹰ ﻟﻠﻤﻌﺎﺩﻟﺔ ﺍﻟﺴﺎﺒﻘﺔ‪ .‬ﺒﺫﻟﻙ ﻨﻜﻭﻥ ﻗﺩ ﺒﺭﻫﻨﺎ‬
‫ﻤﺭﺓ ﺜﺎﻨﻴﺔ ﺃﻨﻪ ﻓﻲ ﺍﻟﻤﻭﺍﺩ ﺍﻟﻤﺘﺠﺎﻨﺴﺔ ﻴﻭﺠﺩ ﻓﻘﻁ ﺜﺎﺒﺘﻲ ﻤﺭﻭﻨﺔ‪.‬‬
‫‪ 2u‬‬
‫‪،ρ‬‬ ‫ﻟﻠﺤﺼﻭل ﻋﻠﻰ ﻤﻌﺎﺩﻟﺔ ﺤﺭﻜﺔ ﺍﻟﻤﺎﺩﺓ‪ ،‬ﻨﺴﺎﻭﻱ ﺍﻟﻤﻌﺎﺩﻟﺔ )‪ ،(7.36‬ﺒـ‬
‫‪t 2‬‬
‫ﻭﻨﻬﻤل ﺍﻟﻘﻭﻯ ﺍﻟﺤﺠﻤﻴﺔ ﻤﺜل ﻗﻭﻯ ﺍﻟﺜﻘﺎﻟﺔ‪ ،‬ﻓﻨﺤﺼل ﻋﻠﻰ‪:‬‬
‫‪2‬‬
‫‪ u‬‬ ‫‪  ‬‬ ‫‪‬‬
‫‪ρ‬‬
‫‪t‬‬ ‫‪2‬‬
‫‪‬‬ ‫‪‬‬‫‪λ‬‬ ‫‪‬‬ ‫‪μ‬‬ ‫‪‬‬‫‪‬‬ ‫‪ ‬‬
‫‪.u  μ 2 u‬‬ ‫)‪(7.37‬‬

‫ﺘﺒﺩﻭ ﻫﺫﻩ ﺍﻟﻤﻌﺎﺩﻟﺔ ﺸﺒﻴﻬﺔ ﻟﻠﻤﻌﺎﺩﻟﺔ ﺍﻟﻤﻭﺠﻴﺔ ﺍﻟﺘﻲ ﺘﻌﺭﻓﻨﺎ ﻋﻠﻴﻬﺎ ﻓﻲ‬
‫ﺍﻟﻜﻬﺭﻁﻴﺴﻴﺔ ﻤﻊ ﻓﺎﺭﻕ ﻭﺤﻴﺩ ﻫﻭ ﻭﺠﻭﺩ ﺤﺩ ﺇﻀﺎﻓﻲ ﻴﺯﻴﺩ ﻫﺫﻩ ﺍﻟﻤﻌﺎﺩﻟﺔ ﺼﻌﻭﺒﺔ‪،‬‬
‫ﻭﻴﻜﻭﻥ ﺍﻟﺤل ﺒﺎﻟﻨﺴﺒﺔ ﻟﻠﻤﻭﺍﺩ ﺍﻟﻤﺘﺠﺎﻨﺴﺔ ﺍﻟﻤﺭﻭﻨﺔ ﻤﺸﺎﺒﻬﺎﹰ ﻟﺤل ﺍﻟﻤﻌﺎﺩﻟﺔ ﺍﻟﻤﻭﺠﻴﺔ‪.‬‬
‫ﻨﺫﻜﺭ ﺃﻥ ﺃﻱ ﻤﺘﺠﻪ ﻟﺤﻘل ﻴﻤﻜﻥ ﻜﺘﺎﺒﺘﻪ ﻤﺠﻤﻭﻉ ﻤﺘﺠﻬﻴﻥ ﺒﺤﻴﺙ ﻴﻜﻭﻥ ﺘﻔﺭﻕ ﺍﻷﻭل‬
‫ﻤﺴﺎﻭﻴﺎﹰ ﻟﻠﺼﻔﺭ ﻭﻴﻜﻭﻥ ﺩﻭﺍﺭ ﺍﻟﺜﺎﻨﻲ ﺃﻴﻀﺎﹰ ﺼﻔﺭﺍﹰ‪ ،‬ﺃﻱ ﺃﻥ‪:‬‬
‫‪  ‬‬
‫‪u  u1  u 2‬‬ ‫)‪(7.38‬‬
‫ﺤﻴﺙ‪:‬‬
‫‪ ‬‬ ‫‪ ‬‬
‫‪‬‬ ‫‪‬‬
‫‪.u1  0‬‬ ‫‪, xu 2  0‬‬ ‫)‪(7.39‬‬
‫‪  ‬‬
‫ﻨﺤﺼل ﺒﺎﻟﺘﻌﻭﻴﺽ ﻋﻥ ‪ u  u 1  u 2‬ﻓﻲ ﺍﻟﻤﻌﺎﺩﻟﺔ )‪ ،(7.37‬ﻋﻠﻰ‪:‬‬

‫‪- 239 -‬‬

‫)‪Create PDF files without this message by purchasing novaPDF printer (http://www.novapdf.com‬‬
‫‪2‬‬ ‫‪  ‬‬ ‫‪‬‬ ‫‪‬‬
‫‪t‬‬
‫‪ρ‬‬
‫‪2‬‬
‫‪‬‬‫‪u‬‬ ‫‪1‬‬ ‫‪‬‬ ‫‪u‬‬ ‫‪2‬‬ ‫‪‬‬ ‫‪‬‬ ‫‪‬‬‫‪λ‬‬ ‫‪‬‬ ‫‪μ‬‬ ‫‪‬‬‫‪‬‬ ‫‪‬‬ ‫‪‬‬
‫)‪. .u 2  μ 2 u 1  u 2  (7.40‬‬
‫‪‬‬
‫ﻨﺴﺘﻁﻴﻊ ﺒﺄﺨﺫ ﺘﻔﺭﻕ ﻫﺫﻩ ﺍﻟﻤﻌﺎﺩﻟﺔ ﺃﻥ ﻨﺘﺨﻠﺹ ﻤﻥ ﺍﻟﻤﺘﺠﻪ ‪: u1‬‬
‫‪2  ‬‬ ‫‪‬‬ ‫‪‬‬ ‫‪‬‬
‫‪‬‬ ‫‪‬‬
‫‪ρ 2 (.u 2 )  λ  μ  2 .u 2  μ. 2.u 2‬‬
‫‪t‬‬
‫‪‬‬
‫ﺒﻤﺎ ﺃﻥ ﺍﻟﻤﺅﺜﺭﻴﻥ ‪ ‬ﻭ ‪  2‬ﺘﺒﺎﺩﻟﻴﻴﻥ‪ ،‬ﻨﺴﺘﻁﻴﻊ ﺇﺨﺭﺍﺝ ‪ ‬ﻜﻤﺅﺜﺭ ﻤﺸﺘﺭﻙ‪،‬‬
‫ﻓﻨﺤﺼل ﻋﻠﻰ ﺍﻟﻤﻌﺎﺩﻟﺔ ﺍﻟﺘﺎﻟﻴﺔ‪:‬‬
‫‪   2 u 2‬‬ ‫‪ ‬‬
‫‪.ρ 2  λ  2μ  2 u 2   0‬‬ ‫)‪(7.41‬‬
‫‪ t‬‬ ‫‪‬‬
‫‪ ‬‬
‫ﺒﻤﺎ ﺃﻥ ‪ xu 2‬ﻴﺴﺎﻭﻱ ﺼﻔﺭ ﺒﺎﻟﺘﻌﺭﻴﻑ‪ ،‬ﻓﺈﻥ ﺩﻭﺍﺭ ﻤﺎ ﺩﺍﺨل ﺍﻟﻘﻭﺴﻴﻥ ﻜﺫﻟﻙ‬
‫ﻴﺴﺎﻭﻱ ﺼﻔﺭ‪ ،‬ﻭﺒﺎﻟﺘﺎﻟﻲ ﻓﺈﻥ ﺍﻟﻤﻘﺩﺍﺭ ﺒﻴﻥ ﺍﻟﻘﻭﺴﻴﻥ ﺒﺸﻜل ﻤﺘﻁﺎﺒﻕ ﻴﺴﺎﻭﻱ ﺍﻟﺼﻔﺭ‪،‬‬
‫ﺃﻱ ﺃﻥ‪:‬‬
‫‪‬‬
‫‪ 2u 2‬‬ ‫‪‬‬
‫‪ρ 2  λ  2μ  2u 2‬‬ ‫)‪(7.42‬‬
‫‪t‬‬
‫ﺒﺴﺭﻋﺔ‬ ‫ﺍﻟﻤﺘﺤﺭﻜﺔ‬ ‫ﻟﻸﻤﻭﺍﺝ‬ ‫ﻤﺘﺠﻬﺔ‬ ‫ﻤﻭﺠﻴﺔ‬ ‫ﻤﻌﺎﺩﻟﺔ‬ ‫ﻭﻫﻲ‬
‫‪‬‬
‫‪ . C 2  λ  2μ ρ‬ﺒﻤﺎ ﺃﻥ ﺩﻭﺍﺭ ‪ u 2‬ﻴﺴﺎﻭﻱ ﺼﻔﺭﺍﹰ‪ ،‬ﻓﺈﻥ ﻫﺫﻩ ﺍﻷﻤﻭﺍﺝ ﻻ ﺘﻌﺘﻤﺩ‬

‫ﻋﻠﻰ ﺍﻻﻨﺘﻘﺎل ﻭﺇﻨﻤﺎ ﻫﻲ ﺃﻤﻭﺍﺝ ﺍﻨﻀﻐﺎﻁﻴﺔ ﻤﺜل ﺍﻷﻤﻭﺍﺝ ﺍﻟﺼﻭﺘﻴﺔ‪.‬‬


‫‪‬‬
‫ﻨﺠﺩ ﺒﺸﻜل ﻤﻤﺎﺜل ﺇﺫﺍ ﺃﺨﺫﻨﺎ ﺩﻭﺍﺭ ﺍﻟﻤﻌﺎﺩﻟﺔ )‪ ،(7.40‬ﺃﻥ ﺍﻟﻤﺘﺠﻪ ‪ u1‬ﻴﺤﻘﻕ‬
‫ﺍﻟﻤﻌﺎﺩﻟﺔ ﺍﻟﺘﺎﻟﻴﺔ‪:‬‬
‫‪‬‬
‫‪ 2u1‬‬ ‫‪‬‬
‫‪ρ 2  μ 2 u1‬‬ ‫)‪(7.43‬‬
‫‪t‬‬
‫ﻭﻫﻲ ﻤﻌﺎﺩﻟﺔ ﻤﻭﺠﻴﺔ ﻟﻸﻤﻭﺍﺝ ﺍﻟﺘﻲ ﺘﻨﺘﺸﺭ ﺒﺴﺭﻋﺔ ‪ . C 2  μ ρ‬ﺒﻤﺎ ﺃﻥ‬
‫‪‬‬ ‫‪ ‬‬
‫‪ .u 1‬ﻴﺴﺎﻭﻱ ﺼﻔﺭ‪ ،‬ﻓﺈﻥ ﺘﻐﻴﺭ ﺍﻹﺯﺍﺤﺔ ‪ u 1‬ﻻ ﻴﻐﻴﺭ ﺍﻟﻜﺜﺎﻓﺔ ‪ ،ρ‬ﻭﺇﻨﻤﺎ ﻴﻭﺍﻓﻕ‬
‫‪‬‬
‫ﺍﻟﻤﺘﺠﻪ ‪ u 1‬ﺍﻻﻨﺘﻘﺎل ﺍﻟﻌﺭﻀﻲ ﻟﻸﻤﻭﺍﺝ‪.‬‬
‫ﺇﺫﺍ ﺃﺭﺩﻨﺎ ﺤﺴﺎﺏ ﺍﻹﺠﻬﺎﺩ ﺃﻟﺴﻜﻭﻨﻲ ﻓﻲ ﻤﻭﺍﺩ ﻤﺘﺠﺎﻨﺴﺔ‪ ،‬ﻴﺠﺏ ﺤل ﺍﻟﻤﻌﺎﺩﻟﺔ‬
‫‪‬‬
‫)‪ ،(7.36‬ﻭﺍﻋﺘﺒﺎﺭ ﺃﻥ ‪ f‬ﺘﺴﺎﻭﻱ ﺼﻔﺭﺍﹰ )ﺃﻭ ﺘﺴﺎﻭﻱ ﺍﻟﻘﻭﻯ ﺍﻟﺤﺠﻤﻴﺔ ﺍﻟﺴﻜﻭﻨﻴﺔ‬
‫ﺍﻟﻨﺎﺘﺠﺔ ﻋﻥ ﻗﻭﻯ ﺍﻟﺜﻘﺎﻟﺔ ﻤﺜل ‪ (ρg‬ﻓﻲ ﺸﺭﻭﻁ ﻤﺤﺩﺩﺓ ﺘﺘﻌﻠﻕ ﺒﺎﻟﻘﻭﻯ ﺍﻟﻤﺅﺜﺭﺓ ﻋﻠﻰ‬

‫‪- 240 -‬‬

‫)‪Create PDF files without this message by purchasing novaPDF printer (http://www.novapdf.com‬‬
‫ﺴﻁﺢ ﺠﺯﺀ ﻜﺒﻴﺭ ﻤﻥ ﺍﻟﻤﺎﺩﺓ‪ ،‬ﻭﻟﻜﻥ ﻫﺫﺍ ﻟﻴﺱ ﺒﺎﻷﻤﺭ ﺍﻟﺴﻬل ﻭﺫﻟﻙ ﻟﺴﺒﺒﻴﻥ‪ ،‬ﺍﻷﻭل‪:‬‬
‫ﺍﻟﻤﻌﺎﺩﻟﺔ ﺫﺍﺘﻬﺎ ﻏﻴﺭ ﺴﻬﻠﺔ‪ ،‬ﺜﺎﻨﻴﺎﹰ‪ :‬ﺃﺸﻜﺎل ﺍﻷﺠﺴﺎﻡ ﺍﻟﻤﺭﻨﺔ ﺍﻟﺘﻲ ﻨﺩﺭﺴﻬﺎ ﻗﺩ ﻻ ﺘﻜﻭﻥ‬
‫ﻤﻨﺘﻅﻤﺔ ﻫﻨﺩﺴﻴﺎﹰ‪ .‬ﻓﻔﻲ ﺍﻟﻜﻬﺭﺒﺎﺀ ﻤﺜﻼﹰ ﻨﺩﺭﺱ ﺤﻠﻭل ﻤﻌﺎﺩﻻﺕ ﻤﻜﺴﻭﻴل ﻓﻲ ﺃﻭﺴﺎﻁ‬
‫ﻫﻨﺩﺴﻴﺔ ﺴﻬﻠﺔ‪ ،‬ﻤﺜل ﻭﺴﻁ ﻟﻪ ﺸﻜل ﺃﺴﻁﻭﺍﻨﻲ‪ ،‬ﺒﻴﻨﻤﺎ ﻓﻲ ﻨﻅﺭﻴﺔ ﺍﻟﻤﺭﻭﻨﺔ ﺘﺘﻁﻠﺏ‬
‫ﺍﻟﺩﺭﺍﺴﺔ ﺘﻁﺒﻴﻘﻬﺎ ﻋﻠﻰ ﺃﺸﻜﺎل ﺃﻜﺜﺭ ﺘﻌﻘﻴﺩﺍﹰ ﻤﺜل ﺨﻁﺎﻑ ﺭﺍﻓﻌﺔ ﻭﻋﻤﻭﺩ ﻨﻘل ﺍﻟﺤﺭﻜﺔ‬
‫ﻓﻲ ﺍﻟﻌﺭﺒﺎﺕ ﻭﻋﻤﻭﺩ ﺍﻟﻤﺤﺭﻜﺎﺕ ﺍﻟﺘﻭﺭﺒﻴﻨﻴﺔ‪..............‬ﻭﺘﺤل ﻤﺜل ﻫﺫﻩ ﺍﻟﻤﺴﺎﺌل‬
‫ﻓﻲ ﻤﻌﻅﻡ ﺍﻷﺤﻴﺎﻥ ﺒﻁﺭﻴﻘﺔ ﻋﺩﺩﻴﺔ‪ ،‬ﻭﺫﻟﻙ ﺒﺎﺴﺘﺨﺩﺍﻡ ﻤﺒﺩﺃ ﺍﻟﻔﻌل ﺍﻷﺼﻐﺭﻱ ﻟﻠﻁﺎﻗﺔ‪،‬‬
‫ﻭﻨﺩﺭﺱ ﻁﺭﻴﻘﺔ ﺃﺨﺭﻯ ﻨﺴﺘﺨﺩﻡ ﻓﻴﻬﺎ ﻨﻤﺎﺫﺝ ﻤﻭﺍﺩ‪ ،‬ﺜﻡ ﻨﻘﻴﺱ ﺍﻹﺠﻬﺎﺩ ﺍﻟﺩﺍﺨﻠﻲ‬
‫ﺘﺠﺭﻴﺒﻴﺎﹰ ﺒﺎﺴﺘﺨﺩﺍﻡ ﺍﻻﺴﺘﻘﻁﺎﺏ ﺍﻟﻀﻭﺌﻲ‪.‬‬

‫ﺍﻟﺸﻜل )‪(7.11‬‬
‫ﻗﻴﺎﺱ ﺍﻹﺠﻬﺎﺩ ﺍﻟﺩﺍﺨﻠﻲ ﺒﺎﺴﺘﺨﺩﺍﻡ ﺍﻻﺴﺘﻘﻁﺎﺏ ﺍﻟﻀﻭﺌﻲ‬

‫ﻭﺘﺘﻠﺨﺹ ﻫﺫﻩ ﺍﻟﻁﺭﻴﻘﺔ ﺒﺎﻵﺘﻲ‪:‬‬


‫ﻋﻨﺩﻤﺎ ﻨﻌﺭﺽ ﻹﺠﻬﺎﺩ ﻗﻁﻌﺔ ﻤﻥ ﻤﺎﺩﺓ ﻤﺭﻨﺔ ﻤﺘﺠﺎﻨﺴﺔ ﻤﺜل ﻗﻁﻌﺔ ﺯﺠﺎﺝ‬
‫ﻋﻀﻭﻱ ﺸﻔﺎﻑ‪ ،‬ﻓﺈﻨﻪ ﻴﻅﻬﺭ ﻓﻴﻬﺎ ﻨﻭﻋﺎﻥ ﻤﻥ ﺍﻷﺸﻌﺔ ﺍﻟﻤﻨﻜﺴﺭﺓ‪.‬‬

‫‪- 241 -‬‬

‫)‪Create PDF files without this message by purchasing novaPDF printer (http://www.novapdf.com‬‬
‫ﺍﻟﺸﻜل )‪(7.12‬‬
‫ﺇﺠﻬﺎﺩ ﻤﺎﺩﺓ ﺒﻼﺴﺘﻴﻜﻴﺔ ﺒﻴﻥ ﻗﻀﻴﺒﻴﻥ ﻤﺘﻘﺎﺒﻠﻴﻥ‬
‫ﺇﺫﺍ ﻨﻔﺫ ﻋﺒﺭ ﺍﻟﺒﻼﺴﺘﻴﻙ ﻀﻭﺀ ﻤﺴﺘﻘﻁﺏ‪ ،‬ﻓﺈﻥ ﻤﺴﺘﻭﻯ ﺍﻻﺴﺘﻘﻁﺎﺏ ﻴﺩﻭﺭ‬
‫ﺒﺯﺍﻭﻴﺔ ﻗﻴﻤﺘﻬﺎ ﺘﻌﺘﻤﺩ ﻋﻠﻰ ﺍﻹﺠﻬﺎﺩ‪ ،‬ﻭﺒﻤﻌﺭﻓﺔ ﻗﻴﺎﺱ ﺯﺍﻭﻴﺔ ﻤﺴﺘﻭﻯ ﺍﻻﺴﺘﻘﻁﺎﺏ‪،‬‬
‫ﻴﻤﻜﻥ ﻗﻴﺎﺱ ﻗﻴﻤﺔ ﺍﻹﺠﻬﺎﺩ‪ ،‬ﻭﻴﺒﻴﻥ ﺍﻟﺸﻜل )‪ ،(7.11‬ﺠﻬﺎﺯ ﻗﻴﺎﺱ ﺯﺍﻭﻴﺔ ﺩﻭﺭﺍﻥ‬
‫ﺍﻻﺴﺘﻘﻁﺎﺏ‪.‬‬
‫‪ - 58‬ﺳﻠﻮﻙ ﺍﳌﻮﺍﺩ ﻏﲑ ﺍﳌﺮﻧﺔ‪:‬‬
‫ﻓﺭﻀﻨﺎ ﻓﻲ ﺍﻟﺩﺭﺍﺴﺔ ﺍﻟﺴﺎﺒﻘﺔ ﺃﻥ ﺍﻹﺠﻬﺎﺩ ﻴﺘﻨﺎﺴﺏ ﻤﻊ ﺍﻟﺘﺸﻭﻩ‪ ،‬ﻭﻟﻜﻥ ﻫﺫﺍ‬
‫ﺒﺸﻜل ﻋﺎﻡ ﻏﻴﺭ ﻤﺤﻘﻕ ﺩﺍﺌﻤﺎﹰ‪ ،‬ﻓﺎﻟﺸﻜل )‪ ،(7.13‬ﻴﻭﻀﺢ ﺘﻐﻴﺭ ﺍﻹﺠﻬﺎﺩ ﺒﺩﻻﻟﺔ ﺍﻟﺘﺸﻭﻩ‬
‫ﻟﻤﻭﺍﺩ ﻤﺭﻨﺔ‪ .‬ﻴﺘﺒﻴﻥ ﻤﻥ ﺍﻟﺭﺴﻡ ﺍﻟﺒﻴﺎﻨﻲ ﻟﺘﻐﻴﺭ ﺍﻹﺠﻬﺎﺩ ﺒﺩﻻﻟﺔ ﺍﻟﺘﺸﻭﻩ ﺃﻨﻪ ﻋﻨﺩﻤﺎ ﻴﻜﻭﻥ‬
‫ﺍﻟﺘﺸﻭﻩ ﺼﻐﻴﺭﺍﹰ ﻓﺈﻥ ﺍﻹﺠﻬﺎﺩ ﻴﺘﻨﺎﺴﺏ ﻁﺭﺩ ﺍﹰ ﻤﻌﻪ‪ ،‬ﻭﺒﻌﺩ ﺤﺩ ﻤﻌﻴﻥ ﻴﺘﻐﻴﺭ ﺍﻟﺘﻨﺎﺴﺏ‬
‫ﺍﻟﻁﺭﺩﻱ ﺒﻴﻥ ﺍﻹﺠﻬﺎﺩ ﻭﺍﻟﺘﺸﻭﻩ ﻭﻴﺒﺩﺃ ﺍﻟﺨﻁ ﺍﻟﺒﻴﺎﻨﻲ ﺒﺎﻻﻨﺤﻨﺎﺀ‪ ،‬ﻭﻓﻲ ﺍﻟﻤﻭﺍﺩ ﺍﻟﻀﻌﻴﻔﺔ‬
‫ﺍﻟﻤﺭﻭﻨﺔ ﻓﺈﻥ ﺍﻟﻤﻨﺤﻨﻲ ﺍﻟﺒﻴﺎﻨﻲ ﻴﻌﺎﻨﻲ ﺍﻨﻘﻁﺎﻋﺎﹰ ﻋﻨﺩ ﺍﻟﻨﻘﻁﺔ ﺍﻟﺘﻲ ﺘﺘﺠﺎﻭﺯ ﺤﺩ ﻤﻌﻴﻥ‬
‫ﻟﻠﻤﺭﻭﻨﺔ ﻴﺴﻤﻰ ﺍﻟﺤﺩ ﺍﻟﺤﺭﺝ‪ ،‬ﻭﺘﺴﻤﻰ ﻨﻘﻁﺔ ﺍﻻﻨﻘﻁﺎﻉ ﺍﻟﻨﻘﻁﺔ ﺍﻟﺤﺭﺠﺔ‪ ،‬ﻭﺘﺨﺘﻠﻑ ﻫﺫﻩ‬
‫ﺍﻟﻨﻘﻁﺔ ﻤﻥ ﻤﺎﺩﺓ ﺇﻟﻰ ﺃﺨﺭﻯ‪ ،‬ﻓﻔﻲ ﺍﻟﺤﺎﻟﺔ ﺍﻟﻌﺎﻤﺔ ﻴﻜﻭﻥ ﺍﻟﻤﻨﺤﻨﻰ ﺍﻟﺒﻴﺎﻨﻲ ﻟﻺﺠﻬﺎﺩ‬

‫‪- 242 -‬‬

‫)‪Create PDF files without this message by purchasing novaPDF printer (http://www.novapdf.com‬‬
‫ﻭﺍﻟﺘﺸﻭﻩ ﺃﻜﺜﺭ ﺘﻌﻘﻴﺩﺍﹰ‪ ،‬ﻭﻓﻲ ﺒﻌﺽ ﺍﻟﻤﻭﺍﺩ ﻴﻜﻭﻥ ﺇﺠﻬﺎﺩ ﺍﻟﺴﺤﺏ ﺃﺼﻐﺭ ﺒﻜﺜﻴﺭ ﻤﻥ‬
‫ﺇﺠﻬﺎﺩ ﺍﻻﻨﻀﻐﺎﻁ ﻜﻤﺎ ﻓﻲ ﺍﻟﺤﻭﺍﺭ )ﺍﻟﻁﺒﺎﺸﻴﺭ(‪.‬‬

‫ﺍﻟﺸﻜل)‪ (7.13‬ﺍﻟﻤﻨﺤﻨﻲ ﺍﻟﺒﻴﺎﻨﻲ ﻟﺘﻐﻴﺭ ﺍﻹﺠﻬﺎﺩ ﻤﻊ ﺍﻟﺘﺸﻭﻩ‬

‫‪ - 62‬ﺣﺴﺎﺏ ﺛﻮﺍﺑﺖ ﺍﳌﺮﻭﻧﺔ‪:‬‬


‫ﻨﺤﺴﺏ ﺃﺨﻴﺭ ﺍﹰ ﺜﻭﺍﺒﺕ ﻤﺭﻭﻨﺔ ﺍﻟﻤﻭﺍﺩ ﺍﻨﻁﻼﻗﺎﹰ ﻤﻥ ﺼﻔﺎﺕ ﺍﻟﺫﺭﺍﺕ ﺍﻟﻤﻜﻭﻨﺔ ﻟﻬﺫﻩ‬
‫ﺍﻟﻤﻭﺍﺩ‪ .‬ﻨﻨﻁﻠﻕ ﻤﻥ ﺤﺎﻟﺔ ﺨﺎﺼﺔ ﺒﺴﻴﻁﺔ ﺠﺴﻡ ﺒﻠﻭﺭﻱ ﻤﻜﻌﺏ ﻤﺜل ﻜﻠﻭﺭ ﺍﻟﺼﻭﺩﻴﻭﻡ‪،‬‬
‫ﺤﻴﺙ ﻴﺘﻐﻴﺭ ﺸﻜل ﺃﻭ ﻗﻴﺎﺱ ﺍﻟﺒﻠﻭﺭﺓ ﺍﻟﻤﺸﻭﻫﺔ‪ .‬ﺘﺅﺩﻱ ﻤﺜل ﻫﺫﻩ ﺍﻟﺘﻐﻴﺭﺍﺕ ﺇﻟﻰ‬
‫ﻤﻀﺎﻋﻔﺔ ﻁﺎﻗﺔ ﺠﻬﺩ ﺍﻟﺒﻠﻭﺭﺓ‪ ،‬ﻭﻟﺤﺴﺎﺏ ﺘﻐﻴﺭ ﻁﺎﻗﺔ ﺍﻟﺘﺸﻭﻩ‪ ،‬ﻴﻠﺯﻡ ﻤﻌﺭﻓﺔ ﺍﺘﺠﺎﻩ‬
‫ﺘﺤﺭﻙ ﻜل ﺫﺭﺓ‪ ،‬ﻭﻟﺤﺴﺎﺏ ﻜﺎﻤل ﻁﺎﻗﺔ ﺫﺭﺍﺕ ﺍﻟﻤﺎﺩﺓ ﻜﻘﻴﻤﺔ ﺼﻐﺭﻯ ﺃﻤﺭ ﻓﻲ ﻏﺎﻴﺔ‬
‫ﺍﻟﺼﻌﻭﺒﺔ‪ ،‬ﻟﻠﺴﻬﻭﻟﺔ ﻨﺩﺭﺱ ﺒﻠﻭﺭﺓ ﻤﻜﻌﺒﺔ ﺒﺴﻴﻁﺔ ﺒﺤﻴﺙ ﺃﻥ ﺍﻹﺯﺍﺤﺔ ﺩﺍﺨل ﺍﻟﺒﻠﻭﺭﺓ‬
‫ﺘﻜﻭﻥ ﻫﻨﺩﺴﻴﺎﹰ ﻤﻤﺎﺜﻠﺔ ﻟﻺﺯﺍﺤﺔ ﻋﻠﻰ ﺴﻁﺤﻬﺎ‪.‬‬
‫ﻴﻤﻜﻥ ﺤﺴﺎﺏ ﺜﻭﺍﺒﺕ ﻤﺭﻭﻨﺔ ﺒﻠﻭﺭﺓ ﻤﻜﻌﺒﺔ ﻜﻤﺎ ﻴﻠﻲ‪ :‬ﻨﻔﺭﺽ ﻓﻲ ﺍﻟﺒﺩﺍﻴﺔ ﺃﻨﻪ‬
‫ﻴﻭﺠﺩ ﻗﺎﻨﻭﻥ ﻤﺎ ﻴﻌﺒﺭ ﻋﻥ ﺍﻟﺘﺄﺜﻴﺭ ﺍﻟﻤﺘﺒﺎﺩل ﺒﻴﻥ ﻜل ﺯﻭﺝ ﻤﻥ ﺫﺭﺍﺕ ﺍﻟﺒﻠﻭﺭﺓ‪،‬‬
‫ﻭﻨﺤﺴﺏ ﺒﻌﺩ ﺫﻟﻙ ﺘﻐﻴﺭ ﺍﻟﻁﺎﻗﺔ ﺍﻟﺩﺍﺨﻠﻴﺔ ﺍﻟﻨﺎﺘﺠﺔ ﻤﻥ ﺍﻨﺤﺭﺍﻑ ﺍﻟﺫﺭﺍﺕ ﻋﻥ ﻭﻀﻊ‬
‫ﺍﻟﺘﻭﺍﺯﻥ‪ ،‬ﻭﺫﻟﻙ ﺒﺈﻴﺠﺎﺩ ﺍﻟﻌﻼﻗﺔ ﺒﻴﻥ ﺍﻟﻁﺎﻗﺔ ﻭﺍﻟﺘﺸﻭﻩ‪ ،‬ﺤﻴﺙ ﻨﺠﺩ ﺃﻥ ﺍﻟﻁﺎﻗﺔ ﺘﺘﻨﺎﺴﺏ‬
‫ﻤﻊ ﻤﺭﺒﻊ ﺍﻟﺘﺸﻭﻩ‪ ،‬ﺜﻡ ﻨﻘﺎﺭﻥ ﺍﻟﻁﺎﻗﺔ ﺍﻟﻤﺤﺴﻭﺒﺔ ﺒﻬﺫﻩ ﺍﻟﻁﺭﻴﻘﺔ ﻤﻊ ﺍﻟﻁﺎﻗﺔ ﺍﻟﻤﺤﺴﻭﺒﺔ‬
‫ﺒﺎﻟﻤﻌﺎﺩﻟﺔ )‪ ،(7.17‬ﻓﻨﺤﺼل ﺒﺎﻟﻤﻁﺎﺒﻘﺔ ﻋﻠﻰ ﺜﻭﺍﺒﺕ ﺍﻟﻤﺭﻭﻨﺔ ‪.Cijkl‬‬

‫‪- 243 -‬‬

‫)‪Create PDF files without this message by purchasing novaPDF printer (http://www.novapdf.com‬‬
‫ﻨﻔﺭﺽ ﻟﻠﺴﻬﻭﻟﺔ ﺃﻥ ﺍﻟﻘﻭﻯ ﺍﻟﻤﺅﺜﺭﺓ ﺒﻴﻥ ﺍﻟﺫﺭﺍﺕ ﺍﻟﻤﺘﺠﺎﻭﺭﺓ ﻫﻲ ﻗﻭﻯ‬
‫ﻤﺭﻜﺯﻴﺔ ﺘﺘﻡ ﻋﺒﺭ ﺍﻟﺨﻁﻭﻁ ﺍﻟﺘﻲ ﺘﺼل ﺒﻴﻥ ﻜل ﺫﺭﺘﻴﻥ ﻤﺘﺠﺎﻭﺭﺘﻴﻥ‪ ،‬ﻨﺘﻭﻗﻊ ﺃﻥ ﻗﻭﻯ‬
‫ﺍﻟﺘﺄﺜﻴﺭ ﺍﻟﻤﺘﺒﺎﺩل ﻓﻲ ﺍﻟﺒﻠﻭﺭﺍﺕ ﺍﻟﺸﺎﺭﺩﻴﺔ )ﺍﻷﻴﻭﻨﻴﺔ( ﻴﺠﺏ ﺃﻥ ﺘﻜﻭﻥ ﻓﻲ ﺍﻷﺴﺎﺱ ﻗﻭﻯ‬
‫ﻜﻭﻟﻭﻨﻴﺔ‪) .‬ﺘﻜﻭﻥ ﻓﻲ ﺍﻟﺭﺍﺒﻁﺔ ﺍﻟﺘﻜﺎﻓﺌﻴﺔ ﻗﻭﻯ ﺍﻟﺘﺒﺎﺩل ﺃﻜﺜﺭ ﺘﻌﻘﻴﺩﺍﹰ‪ ،‬ﺤﻴﺙ ﻴﻠﺯﻡ ﺇﻀﺎﻓﺔ‬
‫ﺇﻟﻰ ﻗﻭﻯ ﺍﻟﺘﺄﺜﻴﺭ ﺍﻟﻤﺘﺒﺎﺩل ﺒﻴﻥ ﺍﻟﺫﺭﺍﺕ ﺍﻟﻤﺘﺠﺎﻭﺭﺓ ﺇﻀﺎﻓﺔ ﺍﻟﺘﺄﺜﻴﺭﺍﺕ ﺍﻟﺠﺎﻨﺒﻴﺔ‬
‫ﺍﻷﺨﺭﻯ ﻤﻤﺎ ﻴﺅﺩﻱ ﺇﻟﻰ ﺘﻌﻘﻴﺩ ﺍﻟﺤﺴﺎﺏ ﺩﻭﻥ ﺃﻥ ﻴﻘﺩﻡ ﺃﻱ ﺠﺩﻴﺩ ﻤﻔﻴﺩ(‪.‬‬

‫ﺍﻟﺸﻜل)‪ ،(7.14‬ﻴﺒﻴﻥ ﻗﻭﻯ ﺍﻻﺭﺘﺒﺎﻁ ﺒﻴﻥ ﺍﻟﺫﺭﺍﺕ ﺍﻟﻤﺘﺠﺎﻭﺭﺓ‪.‬‬


‫‪ (a‬ﻗﻭﻯ ﺍﻻﺭﺘﺒﺎﻁ ﺒﻴﻥ ﺍﻟﺫﺭﺍﺕ‪ (b.‬ﺘﻤﺜﻴل ﻗﻭﻯ ﺍﻻﺭﺘﺒﺎﻁ ﺒﻴﻥ ﺍﻟﺫﺭﺍﺕ ﺍﻟﻤﺘﺠﺎﻭﺭﺓ ﺒﻨﺎﺒﺽ‪.‬‬
‫ﻨﺠﻤﻊ ﻓﻘﻁ ﻤﺴﺎﻫﻤﺔ ﻗﻭﻯ ﺍﻟﺫﺭﺍﺕ ﺍﻟﻤﺘﺠﺎﻭﺭﺓ‪ ،‬ﻭﻨﻬﻤل ﺘﺄﺜﻴﺭ ﺍﻟﺫﺭﺍﺕ ﺍﻟﺒﻌﻴﺩﺓ‬
‫ﺒﻌﻀﻬﺎ ﻋﻥ ﺒﻌﺽ‪ .‬ﻴﺒﻴﻥ ﺍﻟﺸﻜل )‪ ،(7,14a‬ﻗﻭﻯ ﺍﻟﺘﺄﺜﻴﺭ ﺍﻟﻤﺘﺒﺎﺩل ﺒﻴﻥ ﺍﻟﺫﺭﺍﺕ‬
‫ﺍﻟﻤﺘﺠﺎﻭﺭﺓ ﻓﻲ ﺍﻟﻤﺴﺘﻭﻯ ‪ xy‬ﺍﻟﺘﻲ ﺴﻨﺄﺨﺫﻫﺎ ﺒﻌﻴﻥ ﺍﻻﻋﺘﺒﺎﺭ‪ ،‬ﻭﻴﺠﺏ ﻜﺫﻟﻙ ﺍﻷﺨﺫ‬
‫ﺒﻌﻴﻥ ﺍﻻﻋﺘﺒﺎﺭ ﻤﺴﺎﻫﻤﺔ ﻗﻭﻯ ﺍﻟﺘﺄﺜﻴﺭ ﺍﻟﻤﺘﺒﺎﺩل ﺒﻴﻥ ﺍﻟﺫﺭﺍﺕ ﻓﻲ ﺍﻟﻤﺴﺘﻭﻯ ‪yz‬‬
‫ﻭﺍﻟﻤﺴﺘﻭﻯ ‪ zx‬ﻜﺫﻟﻙ‪.‬‬
‫ﺒﻤﺎ ﺃﻨﻨﺎ ﻨﻬﺘﻡ ﻓﻘﻁ ﺒﺤﺴﺎﺏ ﺜﻭﺍﺒﺕ ﺍﻟﻤﺭﻭﻨﺔ ﺍﻟﺘﻲ ﺘﺼﻑ ﺍﻟﺘﺸﻭﻫﺎﺕ ﺍﻟﺼﻐﻴﺭﺓ‬
‫)ﺇﺯﺍﺤﺔ ﺼﻐﻴﺭﺓ(‪ ،‬ﻟﺫﻟﻙ ﻨﻜﺘﻔﻲ ﻓﻲ ﻋﺒﺎﺭﺓ ﺍﻟﻁﺎﻗﺔ ﻓﻘﻁ ﺒﺎﻟﺤﺩﻭﺩ ﺍﻟﺘﺭﺒﻴﻌﻴﺔ ﻟﻺﺯﺍﺤﺔ‪،‬‬
‫ﺃﻱ ﻴﻤﻜﻥ ﺍﻋﺘﺒﺎﺭ ﺃﻥ ﺍﻟﻘﻭﻯ ﺒﻴﻥ ﻜل ﺯﻭﺝ ﻤﻥ ﺍﻟﺫﺭﺍﺕ ﺘﺘﻐﻴﺭ ﻭﻓﻕ ﺨﻁﻭﻁ ﺍﻻﺘﺼﺎل‬
‫ﺒﻴﻨﻬﺎ )ﺇﺯﺍﺤﺔ ﺨﻁﻴﺔ(‪ ،‬ﻟﺫﻟﻙ ﻟﻠﺘﻭﻀﻴﺢ ﻴﻤﻜﻥ ﺍﻟﻘﻭل ﺃﻥ ﻜل ﺫﺭﺓ ﺘﺭﺘﺒﻁ ﻤﻊ ﺍﻟﺫﺭﺍﺕ‬

‫‪- 244 -‬‬

‫)‪Create PDF files without this message by purchasing novaPDF printer (http://www.novapdf.com‬‬
‫ﺍﻟﻤﺠﺎﻭﺭﺓ ﺒﻨﻭﻴﺒﻀﺎﺕ )ﺍﻟﺸﻜل ‪ ،(7.14b‬ﻭﺃﻥ ﺠﻤﻴﻊ ﻫﺫﻩ ﺍﻟﻨﻭﻴﺒﻀﺎﺕ ﺒﻴﻥ ﺫﺭﺍﺕ‬
‫ﺍﻟﻜﻠﻭﺭ ﻭﺒﻴﻥ ﺫﺭﺍﺕ ﺍﻟﺼﻭﺩﻴﻭﻡ ﻴﺠﺏ ﺃﻥ ﻴﻜﻭﻥ ﻟﻬﺎ ﺜﺎﺒﺕ ﻤﺭﻭﻨﺔ ﻭﺍﺤﺩ‪ ،‬ﻨﻔﺭﺽ ﺃﻥ‬
‫ﺜﺎﺒﺕ ﻤﺭﻭﻨﺔ ﻜلٍ ﻤﻨﻬﺎ ﻫﻭ‪ .k1‬ﻜﻤﺎ ﻴﻤﻜﻥ ﺃﻥ ﻴﻜﻭﻥ ﺒﻴﻥ ﺫﺭﺍﺕ ﺍﻟﻜﻠﻭﺭ‪ -‬ﺍﻟﻜﻠﻭﺭ ﻭﺒﻴﻥ‬
‫ﺫﺭﺍﺕ ﺍﻟﺼﻭﺩﻴﻭﻡ ‪ -‬ﺍﻟﺼﻭﺩﻴﻭﻡ ﺜﻭﺍﺒﺕ ﻤﺭﻭﻨﺔ ﻤﺨﺘﻠﻔﺔ‪ ،‬ﻭﻟﻜﻥ ﻟﻠﺘﺒﺴﻴﻁ ﻭﺍﻟﺴﻬﻭﻟﺔ‬
‫ﻨﻔﺭﺽ ﺃﻨﻬﺎ ﻤﺘﻤﺎﺜﻠﺔ ﻭﻴﺴﺎﻭﻱ ﻜلٍ ﻤﻨﻬﺎ ‪.k2‬‬
‫ﻨﻔﺭﺽ ﺃﺨﻴﺭ ﺍﹰ ﺃﻥ ﺍﻟﺒﻠﻭﺭﺓ ﺨﻀﻌﺕ ﻻﻀﻁﺭﺍﺏ ﺘﺸﻭﻩ ﻤﻨﺘﻅﻡ ﻴﻭﺼﻑ‬
‫ﺒﺎﻟﺘﻨﺴﻭﺭ ‪ eij‬ﺘﺤﻭﻱ ﻤﺭﻜﺒﺎﺘﻪ ﻋﻠﻰ ‪ ، x,y,z‬ﻭﻟﻠﺘﺒﺴﻴﻁ ﻨﺩﺭﺱ ﻓﻘﻁ ﺍﻟﺘﺸﻭﻩ ﺍﻟﺫﻱ‬
‫ﻴﺤﺘﻭﻱ ﻋﻠﻰ ﺜﻼﺙ ﻤﺭﻜﺒﺎﺕ‪ ، exx , exy , eyy :‬ﺒﻔﺭﺽ ﺃﻥ ﺇﺤﺩﻯ ﺍﻟﺫﺭﺍﺕ ﺘﻘﻊ ﻓﻲ‬
‫ﻤﺭﻜﺯ ﺍﻹﺤﺩﺍﺜﻴﺎﺕ‪ ،‬ﻓﺈﻥ ﺇﺯﺍﺤﺔ ﺃﻴﺔ ﺫﺭﺓ ﺒﺎﻟﻨﺴﺒﺔ ﻟﻠﻤﺭﻜﺯ ﻴﻌﻁﻰ ﺒﻤﻌﺎﺩﻟﺔ ﻤﺸﺎﺒﻬﺔ‬
‫ﻟﻠﻤﻌﺎﺩﻟﺔ )‪.(7.14‬‬
‫‪u x  e xx x  e xy y‬‬
‫)‪(7.44‬‬
‫‪u y  e xy x  e yy y‬‬
‫ﻨﺭﻤﺯ ﻟﻠﺫﺭﺓ ﺍﻟﺘﻲ ﺇﺤﺩﺍﺜﻴﺎﺘﻬﺎ ‪ x  y  0‬ﺒﺎﻟﺫﺭﺓ ﺭﻗﻡ ‪ 1‬ﻭﺭﻤﺯ ﺒﻘﻴﺔ ﺍﻟﺫﺭﺍﺕ‬
‫ﺍﻟﻤﺠﺎﻭﺭﺓ ﻤﺒﻴﻥ ﻋﻠﻰ ﺍﻟﺸﻜل )‪ .(7.15‬ﻨﺭﻤﺯ ﻟﺜﺎﺒﺕ ﺍﻟﺸﺒﻜﺔ ﺒﺎﻟﺭﻤﺯ ‪ ،a‬ﻓﻨﺤﺼل ﻋﻠﻰ‬
‫‪ x,y‬ﻤﺭﻜﺒﺎﺕ ﺍﻹﺯﺍﺤﺘﻴﻥ ‪ ux,uy‬ﻜﻤﺎ ﻫﻭ ﻤﺒﻴﻥ ﻓﻲ ﺍﻟﺠﺩﻭل )‪.(7.2‬‬
‫ﺍﻟﺠﺩﻭل )‪ :(7.2‬ﻤﺭﻜﺒﺎﺕ ﺍﻹﺯﺍﺤﺔ ‪ux،uy‬‬
‫ﺍﻟﺫﺭﺓ‬ ‫‪x ، y‬‬ ‫‪ux‬‬ ‫‪uy‬‬ ‫‪k‬‬
‫‪1‬‬ ‫‪0 ،0‬‬ ‫‪0‬‬ ‫‪0‬‬ ‫‪-‬‬
‫‪2‬‬ ‫‪a،0‬‬ ‫‪exxa‬‬ ‫‪eyxa‬‬ ‫‪k1‬‬
‫‪3‬‬ ‫‪a،a‬‬ ‫‪(exx +e xy) a‬‬ ‫‪(eyx +eyy) a‬‬ ‫‪k2‬‬
‫‪4‬‬ ‫‪0،a‬‬ ‫‪exya‬‬ ‫‪Eyya‬‬ ‫‪k1‬‬
‫‪5‬‬ ‫‪-a ، a‬‬ ‫‪(-exx+e xy)a‬‬ ‫‪(-eyx+eyy)a‬‬ ‫‪k2‬‬
‫‪6‬‬ ‫‪-a ، 0‬‬ ‫‪-exxa‬‬ ‫‪-eyxa‬‬ ‫‪k1‬‬
‫‪7‬‬ ‫‪-a ، -a‬‬ ‫‪-(exx+e xy)a‬‬ ‫‪-(eyx+eyy)a‬‬ ‫‪k2‬‬
‫‪8‬‬ ‫‪0 ، -a‬‬ ‫‪-exya‬‬ ‫‪Eyya‬‬ ‫‪k1‬‬
‫‪9‬‬ ‫‪a ،-a‬‬ ‫‪(exx-e xy)a‬‬ ‫‪(eyx-eyy)a‬‬ ‫‪k2‬‬

‫‪- 245 -‬‬

‫)‪Create PDF files without this message by purchasing novaPDF printer (http://www.novapdf.com‬‬
‫ﻨﺤﺴﺏ ﺃﺨﻴﺭﺍﹰ ﺍﻟﻁﺎﻗﺔ ﺍﻟﻤﺨﺘﺯﻨﺔ ﻓﻲ ﺍﻟﻨﻭﻴﺒﻀﺎﺕ‪ .‬ﺇﻥ ﻫﺫﻩ ﺍﻟﻁﺎﻗﺔ ﺘﺴﺎﻭﻱ‬
‫ﺤﺎﺼل ﻀﺭﺏ ‪ k 2‬ﻓﻲ ﻤﺭﺒﻊ ﺍﺴﺘﻁﺎﻟﺔ ﻜل ﻨﻭﻴﺒﺽ‪ ،‬ﻭﻫﻜﺫﺍ ﻓﺈﻥ ﻁﺎﻗﺔ ﺍﻟﻨﻭﻴﺒﺽ‬
‫ﺍﻷﻓﻘﻲ ﺒﻴﻥ ﺍﻟﺫﺭﺘﻴﻥ ‪ 1‬ﻭ ‪ 2‬ﺘﺴﺎﻭﻱ ﺇﻟﻰ‪:‬‬
‫‪2‬‬
‫‪k 1 e xx a ‬‬
‫)‪(7.45‬‬
‫‪2‬‬

‫ﺍﻟﺸﻜل )‪ ،(7.15‬ﺇﺯﺍﺤﺔ ﺍﻟﺫﺭﺍﺕ ﺍﻟﻤﺠﺎﻭﺭﺓ ﻟﻠﺫﺭﺓ ﺭﻗﻡ‪1‬‬

‫ﻨﻼﺤﻅ ﺃﻥ ﺍﻹﺯﺍﺤﺔ ﻤﻥ ﺍﻟﻤﺭﺘﺒﺔ ﺍﻷﻭﻟﻰ ﻟﻠﺫﺭﺓ ﺭﻗﻡ ‪ 2‬ﻓﻲ ﺍﺘﺠﺎﻩ ‪ y‬ﻻ ﺘﻐﻴﺭ‬
‫ﻤﻥ ﻁﻭل ﺍﻟﻨﻭﻴﺒﺽ ﺒﻴﻥ ﺍﻟﺫﺭﺘﻴﻥ ‪ 1‬ﻭ ‪ ،2‬ﻭﻹﻴﺠﺎﺩ ﻁﺎﻗﺔ ﺍﻟﺘﺸﻭﻩ ﻟﻠﻨﻭﻴﺒﻀﺎﺕ‬
‫ﺍﻟﻘﻁﺭﻴﺔ ﺒﺎﻟﻨﺴﺒﺔ ﻟﻠﺫﺭﺓ ﺭﻗﻡ ‪ ، 3‬ﻴﺠﺏ ﺤﺴﺎﺏ ﺘﻐﻴﺭ ﺍﻟﻁﻭل ﺍﻷﻓﻘﻲ ﻭﺍﻟﻌﻤﻭﺩﻱ ﻨﺘﻴﺠﺔ‬
‫ﺍﻹﺯﺍﺤﺔ‪ .‬ﻴﻤﻜﻥ ﺍﻟﺘﻌﺒﻴﺭ ﻋﻥ ﺇﺯﺍﺤﺔ ﺼﻐﻴﺭﺓ ﻟﻠﺫﺭﺓ ﺭﻗﻡ ‪ 3‬ﻤﻥ ﻤﺒﺩﺃ ﺍﻹﺤﺩﺍﺜﻴﺎﺕ‬
‫ﺒﺩﻻﻟﺔ ‪ ux‬ﻭ ‪ uy‬ﻓﻲ ﺍﻻﺘﺠﺎﻩ ﺍﻟﻘﻁﺭﻱ ﻭﺫﻟﻙ ﻤﻥ ﺍﻟﻌﻼﻗﺔ ﺍﻟﺘﺎﻟﻴﺔ‪:‬‬
‫‪1‬‬
‫‪u‬‬ ‫‪x‬‬ ‫‪ uy ‬‬
‫‪2‬‬
‫ﻨﺴﺘﻁﻴﻊ ﺒﺎﺴﺘﻌﻤﺎل ﻗﻴﻡ ‪ ،ux,uy‬ﺤﺴﺎﺏ ﺍﻟﻁﺎﻗﺔ ﺒﺎﻟﺸﻜل ﺍﻟﺘﺎﻟﻲ‪:‬‬

‫‪- 246 -‬‬

‫)‪Create PDF files without this message by purchasing novaPDF printer (http://www.novapdf.com‬‬
‫‪2‬‬
‫‪k2‬‬ ‫‪ux  uy‬‬ ‫‪‬‬ ‫‪k 2a 2‬‬
‫‪‬‬ ‫‪ ‬‬ ‫‪e xx  e yx  e yy 2‬‬ ‫)‪(7.46‬‬
‫‪2‬‬ ‫‪‬‬ ‫‪2‬‬ ‫‪‬‬ ‫‪4‬‬
‫ﻴﻠﺯﻡ ﻟﺤﺴﺎﺏ ﻁﺎﻗﺔ ﻜل ﺍﻟﻨﻭﻴﺒﻀﺎﺕ ﻓﻲ ﺍﻟﻤﺴﺘﻭﻱ ‪ xy‬ﺠﻤﻊ ﺜﻤﺎﻨﻴﺔ ﺤﺩﻭﺩ ﻤﺜل‬
‫ﺍﻟﺤﺩ ﻓﻲ ﺍﻟﻤﻌﺎﺩﻟﺔ )‪ ،(7.45‬ﻭﺍﻟﻤﻌﺎﺩﻟﺔ )‪ .(7.46‬ﻨﺭﻤﺯ ﻟﻬﺫﻩ ﺍﻟﻁﺎﻗﺔ ﺒﺎﻟﺭﻤﺯ ‪،UO‬‬
‫ﻓﻨﺤﺼل ﻋﻠﻰ‪:‬‬

‫‪ 2 k2‬‬ ‫‪k‬‬ ‫‪2 ‬‬


‫‪k1e xx  e xx  e yx  e xy  e yy   k1e 2yy  2 e xx  e yx  e xy  e yy  ‬‬
‫‪2‬‬
‫‪‬‬
‫‪a ‬‬ ‫‪2‬‬
‫‪2‬‬ ‫‪2‬‬ ‫‪‬‬
‫‪U0  ‬‬ ‫‪‬‬
‫‪2‬‬ ‫‪k2‬‬ ‫‪k2‬‬
‫‪ k1e xx  2 e xx  e yx  e xy  e yy   k1e yy  2 e xx  e yx  e xy  e yy  ‬‬
‫‪2‬‬ ‫‪2‬‬ ‫‪2‬‬ ‫‪2‬‬

‫)‪(7.47‬‬
‫ﻭﻟﺤﺴﺎﺏ ﺍﻟﻁﺎﻗﺔ ﺍﻟﻜﻠﻴﺔ ﻟﻜل ﺍﻟﻨﻭﻴﺒﻀﺎﺕ ﺍﻟﻤﺭﺘﺒﻁﺔ ﻤﻊ ﺍﻟﺫﺭﺓ‪ ،1‬ﻴﺠﺏ ﺇﺠﺭﺍﺀ‬
‫ﺒﻌﺽ ﺍﻹﻀﺎﻓﺎﺕ ﻋﻠﻰ ﺍﻟﻤﻌﺎﺩﻟﺔ )‪ ،(7.47‬ﺘﺘﻌﻠﻕ ﻫﺫﻩ ﺍﻹﻀﺎﻓﺔ ﺒﺎﻟﺫﺭﺍﺕ ﺍﻟﻘﻁﺭﻴﺔ‬
‫ﺍﻟﻤﺘﺠﺎﻭﺭﺓ ﻏﻴﺭ ﺍﻟﻭﺍﻗﻌﺔ ﻓﻲ ﺍﻟﻤﺴﺘﻭﻯ ‪ ،xy‬ﻭﺘﺴﺎﻭﻱ ﺍﻟﻁﺎﻗﺔ ﺍﻟﻤﻀﺎﻓﺔ ﺇﻟﻰ‪:‬‬
‫‪‬‬
‫‪k 2 e 2xx a 2  e 2yy a 2‬‬ ‫‪‬‬ ‫)‪(7.48‬‬
‫ﺘﺭﺒﻁ ﺍﻟﻤﻌﺎﺩﻟﺔ )‪ ،(7.17‬ﺍﻟﻌﻼﻗﺔ ﺒﻴﻥ ﺜﺎﺒﺕ ﺍﻟﻤﺭﻭﻨﺔ ﻭﻜﺜﺎﻓﺔ ﺍﻟﻁﺎﻗﺔ ‪ ،ω‬ﻭﺒﻤﺎ‬
‫‪ .‬ﺒﻤﺎ ﺃﻥ‬ ‫ﺃﻥ ﺍﻟﻁﺎﻗﺔ ﺍﻟﺘﻲ ﺤﺴﺒﻨﺎﻫﺎ ﻫﻲ ﻟﺫﺭﺓ ﻭﺍﺤﺩﺓ‪ ،‬ﻟﺫﻟﻙ ﻴﺠﺏ ﻀﺭﺒﻬﺎ ﺒـ‬
‫‪1‬‬
‫‪2‬‬
‫ﻭﺤﺩﺓ ﺍﻟﺤﺠﻡ ﺘﺤﺘﻭﻱ ﻋﻠﻰ ‪ 1 3‬ﺫﺭﺓ‪ ،‬ﻓﺈﻥ ﺍﻟﻌﻼﻗﺔ ﺒﻴﻥ ‪ ω‬ﻭ ‪ U0‬ﻫﻲ‪:‬‬
‫‪a‬‬
‫‪U0‬‬
‫‪ω 3‬‬
‫‪2a‬‬
‫ﻹﻴﺠﺎﺩ ﺜﻭﺍﺒﺕ ﺍﻟﻤﺭﻭﻨﺔ ‪ ،Cijkl‬ﻤﻥ ﺍﻟﻀﺭﻭﺭﻱ ﺃﻥ ﻨﺭﺒﻊ ﻤﺠﻤﻭﻉ ﻤﺎ ﺒﻴﻥ‬
‫ﺍﻟﻘﻭﺴﻴﻥ ﻓﻲ ﺍﻟﻤﻌﺎﺩﻟﺔ )‪ ،(7.47‬ﺜﻡ ﻨﻀﻴﻑ ﺍﻟﻨﺎﺘﺞ ﺇﻟﻰ )‪ ،(7.48‬ﻭﻨﻘﺎﺭﻥ ﺜﻭﺍﺒﺕ‬
‫ﺃﻤﺜﺎل ‪ eijekk‬ﺍﻟﻤﻁﺎﺒﻘﺔ ﻷﻤﺜﺎﻟﻬﺎ ﻓﻲ ﺍﻟﻤﻌﺎﺩﻟﺔ )‪.(7.17‬‬
‫ﻤﺜﺎل ﻨﺠﻤﻊ ﺃﻤﺜﺎل ‪ e 2xx‬ﻭ ﺃﻤﺜﺎل ‪ ، e 2yy‬ﻨﺤﺼل ﻋﻠﻰ ﺍﻟﻤﻀﺭﻭﺏ ﺍﻟﺘﺎﻟﻲ‪:‬‬
‫‪k 1  2k 2  a 2‬‬
‫ﻟﺫﻟﻙ ﻓﺈﻥ‪:‬‬
‫‪k 1  2k 2‬‬
‫‪C xxxx  C yyyy ‬‬
‫‪a‬‬

‫‪- 247 -‬‬

‫)‪Create PDF files without this message by purchasing novaPDF printer (http://www.novapdf.com‬‬
‫ﻭﻨﺼﺎﺩﻑ ﻤﺼﺎﻋﺏ ﺒﺎﻟﻨﺴﺒﺔ ﻟﺒﻘﻴﺔ ﺍﻟﺤﺩﻭﺩ‪ .‬ﺒﻤﺎ ﺃﻨﻨﺎ ﻻ ﻨﺴﺘﻁﻴﻊ ﺘﻤﻴﻴﺯ ﺍﻟﻔﺭﻕ‬
‫ﺒﻴﻥ ﺍﻟﻤﻀﺭﻭﺒﻴﻥ ‪ exxeyy‬ﻤﻥ ‪ ،eyyexx‬ﻓﺎﻥ ﻤﻌﺎﻤل ﺍﻟﻁﺎﻗﺔ ﻟﻬﺫﻴﻥ ﺍﻟﻤﻀﺭﻭﺒﻴﻥ‬
‫ﻴﺴﺎﻭﻱ ﻤﺠﻭﻉ ﺤﺩﻴﻥ ﻓﻲ ﺍﻟﻤﻌﺎﺩﻟﺔ )‪ .(7.17‬ﻴﺴﺎﻭﻱ ﺍﻟﻤﻌﺎﻤل ﻋﻨﺩ ‪ exxeyy‬ﻓﻲ‬
‫ﺍﻟﻤﻌﺎﺩﻟﺔ )‪ (7.47‬ﺇﻟﻰ ‪ ، 2k 2‬ﻟﺫﻟﻙ ﻨﺤﺼل ﻋﻠﻰ‪:‬‬
‫‪2k 2‬‬
‫‪C xxyy  C yyxx ‬‬
‫‪a‬‬
‫ﻴﻨﺘﺞ ﻤﻥ ﻋﻼﻗﺎﺕ ﺍﻟﺘﻨﺎﻅﺭ ﺃﻥ‪ ، C xxyy  C yyxx :‬ﻭﺒﺎﻟﺘﺎﻟﻲ ﻓﺈﻥ‪:‬‬
‫‪k2‬‬
‫‪C xxyy  C yyxx ‬‬
‫‪a‬‬
‫ﻭﻨﺤﺼل ﺒﻬﺫﺍ ﺍﻷﺴﻠﻭﺏ ﻋﻠﻰ‪:‬‬
‫‪k2‬‬
‫‪C xyxy  C yxyx ‬‬
‫‪a‬‬
‫ﻭﺃﺨﻴﺭﺍﹰ ﺒﻤﻼﺤﻅﺔ ﺃﻥ ﻜل ﺍﻟﺤﺩﻭﺩ ﺍﻟﺘﻲ ﺘﺤﻭﻱ ﻋﻠﻰ ﺍﻟﺩﻟﻴل ‪ x‬ﺃﻭ ‪ y‬ﻤﺭﺓ‬
‫ﻭﺍﺤﺩﺓ ﺘﺴﺎﻭﻱ ﺍﻟﺼﻔﺭ ﻜﻤﺎ ﻭﺠﺩﻨﺎ ﺴﺎﺒﻘﺎﹰ ﻤﻥ ﻋﻼﻗﺎﺕ ﺍﻟﺘﻨﺎﻅﺭ‪ .‬ﻴﻤﻜﻥ ﺘﻠﺨﻴﺹ ﺍﻟﻨﺘﺎﺌﺞ‬
‫ﺍﻟﺴﺎﺒﻘﺔ ﺒﺎﻟﺸﻜل ﺍﻟﺘﺎﻟﻲ‪:‬‬
‫‪k1  2k 2‬‬
‫‪C xxxx  C yyyy ‬‬
‫‪a‬‬
‫‪k‬‬
‫‪C xyxy  C yxyx  2‬‬
‫‪a‬‬ ‫)‪(7.49‬‬
‫‪k2‬‬
‫‪C xxyy  C yyxx  Cxyyx  C yxxy ‬‬
‫‪a‬‬
‫‪C xxxy  C xyyy  ..........  0‬‬
‫ﺒﺫﻟﻙ ﻨﻜﻭﻥ ﻗﺩ ﻗﺩﻤﻨﺎ ﻤﺒﺩﺃ ﻴﺭﺒﻁ ﺒﻴﻥ ﺜﻭﺍﺒﺕ ﺍﻟﻤﺭﻭﻨﺔ ﻭﺼﻔﺎﺕ ﺍﻟﺫﺭﺍﺕ ﺍﻟﺘﻲ‬
‫ﺘﻅﻬﺭ ﻤﻥ ﺨﻼل ﺍﻟﺜﺎﺒﺘﻴﻥ ‪ k1‬ﻭ ‪ .k2‬ﺘﺘﺤﻘﻕ ﻓﻲ ﺍﻟﺒﻠﻭﺭﺓ ﺍﻟﻤﻜﻌﺒﺔ ﺍﻟﺤﺎﻟﺔ ﺍﻟﺨﺎﺼﺔ‬
‫‪ C xyxy  C xxyy‬ﺒﺤﻴﺙ ﻻ ﺘﺅﺨﺫ ﺃﻱ ﻗﻭﻯ ﺒﻌﻴﻥ ﺍﻻﻋﺘﺒﺎﺭ ﺴﻭﻯ ﺍﻟﻘﻭﻯ ﺍﻟﻤﺅﺜﺭﺓ ﻭﻓﻕ‬
‫ﺍﻟﺨﻁﻭﻁ ﺍﻟﺘﻲ ﺘﺼل ﺒﻴﻥ ﻜل ﺯﻭﺝ ﻤﻥ ﺍﻟﺫﺭﺍﺕ‪ ،‬ﺃﻱ ﻻ ﺘﺅﺨﺫ ﻫﻨﺎ ﺒﻌﻴﻥ ﺍﻻﻋﺘﺒﺎﺭ‬
‫ﺍﻟﻘﻭﻯ ﺒﻴﻥ ﺍﻟﺫﺭﺍﺕ ﺍﻟﻤﺸﺎﺒﻬﺔ ﻟﻘﻭﻯ ﺍﻟﻨﻭﻴﺒﻀﺎﺕ ) ﺍﻟﻤﻭﺠﻭﺩﺓ ﻓﻲ ﺍﻟﺭﺍﺒﻁﺔ ﺍﻟﺘﻜﺎﻓﺌﻴﺔ(‪.‬‬
‫ﻴﻤﻜﻥ ﻤﻘﺎﺭﻨﺔ ﻫﺫﻩ ﺍﻟﺤﺴﺎﺒﺎﺕ ﻤﻊ ﺍﻟﻨﺘﺎﺌﺞ ﺍﻟﺘﺠﺭﻴﺒﻴﺔ ﻟﻘﻴﺎﺱ ﺜﻭﺍﺒﺕ ﺍ ﻟﻤﺭﻭﻨﺔ‪،‬‬
‫ﻭﻴﺘﺒﻴﻥ ﻓﻲ ﺍﻟﺠﺩﻭل )‪ (7.3‬ﻗﻴﻡ ﺜﻼﺙ ﺜﻭﺍﺒﺕ ﻤﺭﻭﻨﺔ ﻟﺒﻌﺽ ﺍﻟﺒﻠﻭﺭﺍﺕ ﺍﻟﻤﻜﻌﺒﺔ‪ .‬ﻟﻘﺩ‬

‫‪- 248 -‬‬

‫)‪Create PDF files without this message by purchasing novaPDF printer (http://www.novapdf.com‬‬
‫ﺫﻜﺭﻨﺎ ﺃﻥ ‪ ، C xyxy  C xxyy‬ﻭﻟﻜﻥ ﻓﻲ ﺍﻟﺤﺎﻟﺔ ﺍﻟﻌﺎﻤﺔ ﻻ ﺘﻜﻭﻥ ﻫﺫﻩ ﺍﻟﻤﺴﺎﻭﺍﺓ ﻤﺤﻘﻘﺔ‪.‬‬
‫ﺍﻟﺴﺒﺏ ﻓﻲ ﺫﻟﻙ ﻫﻭ ﺃﻨﻪ ﻓﻲ ﺍﻟﻤﻌﺎﺩﻥ ﺍﻟﺸﺒﻴﻬﺔ ﻟﻠﺼﻭﺩﻴﻭﻡ ﻭﺍﻟﺒﻭﺘﺎﺴﻴﻭﻡ ﻻ ﺘﻜﻭﻥ‬
‫ﺍﻟﻘﻭﻯ ﺒﻴﻥ ﺍﻟﺫﺭﺍﺕ ﻤﻭﺠﻬﺔ ﻭﻓﻕ ﺍﻟﺨﻁﻭﻁ ﺍﻟﺘﻲ ﺘﺭﺒﻁ ﺒﻴﻨﻬﺎ‪ ،‬ﻭﻫﺫﺍ ﻋﻜﺱ ﺍﻟﻨﻤﻭﺫﺝ‬
‫ﺍﻟﺫﻱ ﺃﻭﺭﺩﻨﺎﻩ ﺴﺎﺒﻘﺎﹰ‪ .‬ﻜﺫﻟﻙ ﺍﻟﻤﺎﺱ ﻻ ﻴﺨﻀﻊ ﻟﻬﺫﺍ ﺍﻟﻘﺎﻨﻭﻥ‪ ،‬ﻷﻥ ﺍﻟﻘﻭﺓ ﺒﻴﻥ ﺫﺭﺍﺕ‬
‫ﺍﻟﻤﺎﺱ ﺘﺴﺎﻫﻤﻴﺔ‪ ،‬ﻭﺘﺤﻘﻕ ﻗﺎﻨﻭﻨﺎﹰ ﺨﺎﺼﺎﹰ ﻟﻠﺭﺒﻁ ﺒﻴﻥ ﺍﻟﺫﺭﺍﺕ ﻭﺫﻟﻙ ﻭﻓﻕ ﻨﻭﻴﺒﻀﺎﺕ‬
‫ﻤﻥ ﺭﺅﻭﺱ ﺭﺒﺎﻋﻲ ﺴﻁﻭﺡ‪ ،‬ﻤﺜل ﺍﻟﺒﻠﻭﺭﺍﺕ ﺍﻷﻴﻭﻨﻴﺔ ﻟـ ﻓﻠﻭﺭﻴﻙ ﺍﻟﻴﺜﻴﻭﻡ ﺃﻭ ﻜﻠﻭﺭ‬
‫ﺍﻟﺼﻭﺩﻴﻭﻡ ‪.......‬ﻓﻠﻬﺎ ﺠﻤﻴﻌﺎﹰ ﻨﻔﺱ ﺍﻟﺼﻔﺎﺕ ﺍﻟﻔﻴﺯﻴﺎﺌﻴﺔ ﺘﻘﺭﻴﺒﺎﹰ‪ ،‬ﻭﻜﻤﺎ ﻴﺘﺒﻴﻥ ﻓﻲ‬
‫ﺍﻟﺠﺩﻭل )‪ (7.3‬ﻓﺈﻥ ﺍﻟﺜﻭﺍﺒﺕ ‪ Cxxyy‬ﻭ ‪ Cxyxy‬ﻤﺘﺴﺎﻭﻴﺔ ﺘﻘﺭﻴﺒﺎﹰ ﻟﻤﻌﻅﻡ ﺍﻟﻤﻭﺍﺩ‪.‬‬
‫ﺍﻟﺠﺩﻭل )‪ :(7.3‬ﺜﻭﺍﺒﺕ ﺍﻟﻤﺭﻭﻨﺔ ﻟﺒﻠﻭﺭﺍﺕ ﻤﻜﻌﺒﺔ )ﻓﻲ ‪(103N.m2‬‬
‫ﺍﻟﺒﻠﻭﺭﺓ‬ ‫‪Cxxxx‬‬ ‫‪Cxxyy‬‬ ‫‪Cxyxy‬‬

‫‪N‬‬ ‫‪0.055‬‬ ‫‪0.042‬‬ ‫‪0.049‬‬


‫‪K‬‬ ‫‪0.046‬‬ ‫‪0.037‬‬ ‫‪0.026‬‬
‫‪Fe‬‬ ‫‪2.37‬‬ ‫‪1.41‬‬ ‫‪1.16‬‬
‫ﻤﺎﺱ‬ ‫‪10.76‬‬ ‫‪1.25‬‬ ‫‪5.76‬‬
‫‪Al‬‬ ‫‪1.08‬‬ ‫‪0.62‬‬ ‫‪0.28‬‬
‫‪LiF‬‬ ‫‪1.19‬‬ ‫‪0.54‬‬ ‫‪0.53‬‬
‫‪NaCl‬‬ ‫‪0.486‬‬ ‫‪0.127‬‬ ‫‪0.128‬‬
‫‪KCl‬‬ ‫‪0.40‬‬ ‫‪0.062‬‬ ‫‪0.062‬‬
‫‪NaBr‬‬ ‫‪0.33‬‬ ‫‪0.13‬‬ ‫‪0.13‬‬
‫‪KJ‬‬ ‫‪0.27‬‬ ‫‪0.043‬‬ ‫‪0.042‬‬
‫‪AgCl‬‬ ‫‪0.60‬‬ ‫‪0.36‬‬ ‫‪0.062‬‬

‫‪- 249 -‬‬

‫)‪Create PDF files without this message by purchasing novaPDF printer (http://www.novapdf.com‬‬
- 250 -

Create PDF files without this message by purchasing novaPDF printer (http://www.novapdf.com)
‫‪‬‬
‫‪ ‬‬

‫ﺍﳌﺒﺎﺩﻯء ﺍﻷﺳﺎﺳﻴﺔ ﻟﻠﻨﻈﺮﻳﺔ ﺍﻟﻨﺴﺒﻴﺔ ﺍﳋﺎﺻﺔ‬

‫ﻟﻘﺩ ﺴﺎﺩﺕ ﻤﻔﺎﻫﻴﻡ ﻨﻴﻭﺘﻥ ﻓﻲ ﺍﻟﻤﻴﻜﺎﻨﻴﻙ ﺤﺘﻰ ﻨﻬﺎﻴﺔ ﺍﻟﻘﺭﻥ ﺍﻟﺘﺎﺴﻊ ﻋﺸﺭ‪ ،‬ﺘﻠﻙ‬
‫ﺍﻟﻤﻔﺎﻫﻴﻡ ﺍﻟﺘﻲ ﺍﻋﺘﺒﺭﺕ ﺃﻥ ﺍﻟﺤﺭﻜﺔ ﺒﺎﻟﺭﻏﻡ ﻤﻥ ﺃﻨﻬﺎ ﺘﺘﻡ ﻓﻲ ﺍﻟﻔﺭﺍﻍ ﺍﻻ ﺃﻨﻬﺎ ﻻﺘﺅﺜﺭ‬
‫ﻓﻴﻪ ﻭﺃﻥ ﻫﻨﺎﻙ ﺯﻤﻥ ﻋﺎﻟﻤﻲ ﻴﺴﻴﺭ ﺒﺎﻨﺘﻅﺎﻡ ﻤﻬﻤﺎ ﻜﺎﻨﺕ ﺍﻟﺤﺭﻜﺔ‪ .‬ﻭﻋﻨﺩﻤﺎ ﻅﻬﺭﺕ‬
‫ﺍﻟﻨﻅﺭﻴﺔ ﺍﻟﻨﺴﺒﻴﺔ ﻻﻨﺸﺘﺎﻴﻥ ﺴﺎﻫﻤﺕ ﺒﺘﻐﻴﻴﺭ ﺠﺫﺭﻱ ﻟﻠﻤﻔﺎﻫﻴﻡ ﺍﻟﺴﺎﺒﻘﺔ‪ ،‬ﻭﻜﺎﻥ ﻟﻬﺎ ﺃﺜﺭ‬
‫ﺒﺎﻟﻎ ﻓﻲ ﺠﻤﻴﻊ ﻓﺭﻭﻉ ﺍﻟﻔﻴﺯﻴﺎﺀ ﻭﺨﺎﺼﺔﹰ ﺍﻟﻔﻴﺯﻴﺎﺀ ﺍﻟﻨﻭﻭﻴﺔ ﻭﺍﻟﺫﺭﻴﺔ‪ ،‬ﻜﻤﺎ ﺃﻨﻬﺎ ﺘﻨﺒﺄﺕ‬
‫ﺒﻅﻭﺍﻫﺭ ﻜﺜﻴﺭﺓ ﺃﺜﺒﺕ ﺍﻟﻌﻠﻡ ﺼﺤﺘﻬﺎ ﻓﻴﻤﺎ ﺒﻌﺩ‪ ،‬ﺒﺤﻴﺙ ﻴﻤﻜﻥ ﺃﻥ ﻨﺅﻜﺩ ﺒﺄﻥ ﻅﻬﻭﺭ‬
‫ﺍﻟﻨﻅﺭﻴﺔ ﺍﻟﻨﺴﺒﻴﺔ ﺃﺩﻯ ﺇﻟﻰ ﺒﺩﺍﻴﺔ ﻓﻴﺯﻴﺎﺀ ﺠﺩﻴﺩﺓ ﺃﻭ ﻤﻴﻜﺎﻨﻴﻙ ﺠﺩﻴﺩ ﻻﻜﻼﺴﻴﻜﻲ‪.‬‬
‫‪ -60‬ﲢﻮﻳﻼﺕ ﻏﺎﻟﻴﻠﻲ ‪:‬‬
‫ﻤﻥ ﺍﻟﻤﻌﻠﻭﻡ ﺃﻥ ﺩﺭﺍﺴﺔ ﺍﻟﺤﺭﻜﺔ ﺘﻌﻨﻲ ﺍﻴﺠﺎﺩ ﻤﻭﻀﻊ ﺍﻟﻤﺘﺤﺭﻙ ﻓﻲ ﻜل ﻟﺤﻅﺔ‬
‫ﻤﻥ ﺍﻟﺯﻤﻥ ﻭﻻﺒﺩ ﻟﺘﺤﻘﻴﻕ ﺫﻟﻙ ﻤﻥ ﺍﻴﺠﺎﺩ ﻭﺴﻴﻠﺔ ﻟﻘﻴﺎﺱ ﺍﻟﻤﺴﺎﻓﺔ ﺍﻟﻤﻘﻁﻭﻋﺔ‪ ،‬ﺜﻡ ﺍﻴﺠﺎﺩ‬
‫ﻭﺴﻴﻠﺔ ﺃﺨﺭﻯ ﻟﻘﻴﺎﺱ ﺍﻟﺯﻤﻥ‪ .‬ﻤﺜل ﻫﺫﺍ ﻨﺴﻤﻴﻪ ﺠﻤﻠﺔ ﺍﻟﻘﻴﺎﺱ‪ ،‬ﻭﻟﻴﺱ ﻤﻥ ﺍﻟﻀﺭﻭﺭﻱ‬
‫ﺃﻥ ﺘﻜﻭﻥ ﻫﺫﻩ ﺍﻟﺠﻤﻠﺔ ﺜﺎﺒﺘﺔ ﺍﺫ ﻴﻤﻜﻥ ﺃﻥ ﺘﻜﻭﻥ ﻤﺘﺤﺭﻜﺔ ﺒﻨﻅﺎﻡ ﻤﻌﻴﻥ ﻭﻟﻜﻨﻨﺎ ﺴﻨﻬﺘﻡ‬
‫ﺒﺼﻭﺭﺓ ﺨﺎﺼﺔ ﺒﺎﻟﺠﻤل ﺍﻟﻌﻁﺎﻟﻴﺔ ﻭﻫﻲ ﺍﻟﺘﻲ ﺘﺘﺤﺭﻙ ﺒﻌﻀﻬﺎ ﺒﺎﻟﻨﺴﺒﺔ ﻟﻠﺒﻌﺽ ﺒﺤﺭﻜﺔ‬
‫ﻤﺴﺘﻘﻴﻤﺔ ﻭﻤﻨﺘﻅﻤﺔ‪ ،‬ﻭﻟﻨﺄﺨﺫ ﺍﻟﺤﺎﻟﺔ ﺍﻟﺨﺎﺼﺔ ﻋﻨﺩﻤﺎ ﺘﺘﺤﺭﻙ ﺍﻟﺠﻤﻠﺔ ‪ K‬ﺒﺴﺭﻋﺔ ﺜﺎﺒﺘﺔ‬
‫)‪ (V‬ﺒﺎﻟﻨﺴﺒﺔ ﻟﻠﺠﻤﻠﺔ ﺍﻟﺜﺎﺒﺘﺔ ) ‪ (K1‬ﻭﺒﺎﻻﺘﺠﺎﻩ ‪ ox‬ﺸﻜل )‪.(8.1‬‬
‫ﻓﺈﺫﺍ ﻜﺎﻨﺕ ‪ M‬ﻨﻘﻁﺔ ﻤﺎﺩﻴﺔ ﺘﺘﺤﺭﻙ ﻓﻲ ﺍﻟﺠﻤﻠﺔ ‪ K‬ﻓﺎﻥ ﺍﻟﻌﻼﻗﺔ ﺒﻴﻥ ﺍﺤﺩﺍﺜﻴﺎﺘﻬﺎ‬
‫ﻓﻲ ﺍﻟﺠﻤﻠﺘﻴﻥ‪:‬‬
‫‪x1  x  Vt ‬‬
‫)‪(8.1‬‬
‫‪‬‬
‫‪y1  y‬‬ ‫‪‬‬
‫‪z1  z‬‬ ‫‪‬‬
‫‪‬‬

‫‪- 251 -‬‬

‫)‪Create PDF files without this message by purchasing novaPDF printer (http://www.novapdf.com‬‬
‫ﻭﺒﺎﻻﺸﺘﻘﺎﻕ ﺒﺎﻟﻨﺴﺒﺔ ﻟﻠﺯﻤﻥ ﻨﺠﺩ‪:‬‬
‫‪x1  x   V , y1  y  , z1  z ‬‬ ‫)‪(8.2‬‬
‫ﺃﻱ ﺃﻥ ﻤﺭﻜﺒﺎﺕ ﺸﻌﺎﻉ ﺍﻟﺴﺭﻋﺔ ﻋﻠﻰ ﺍﻟﻤﺤﺎﻭﺭ ‪ oz‬ﻭ ‪ o1z1‬ﻭ ‪ oy‬ﻭ ‪o1y1‬‬
‫ﻤﺘﺴﺎﻭﻴﺔ ﻓﻴﻤﺎ ﺒﻴﻨﻬﺎ‪ ،‬ﻭﻻﺘﺴﺎﻭﻱ ﻤﺭﻜﺒﺔ ﺍﻟﺴﺭﻋﺔ ﻋﻠﻰ ﺍﻟﻤﺤﻭﺭ ‪ ox‬ﻤﺭﻜﺒﺔ ﺍﻟﺴﺭﻋﺔ‬
‫ﺍﻟﻤﻘﺎﺒﻠﺔ ﻟﻬﺎ ﻋﻠﻰ ﺍﻟﻤﺤﻭﺭ ‪ o1x1‬ﻓﺎﻟﻨﻘﻁﺔ ‪ M‬ﺘﻜﻭﻥ ﺃﺴﺭﻉ ﺍﺫﺍ ﻤﺎ ﻨﻅﺭ ﺍﻟﻴﻬﺎ ﻤﻥ‬
‫ﺍﻟﺠﻤﻠﺔ ‪ . K1‬ﺃﻤﺎ ﺇﺫﺍ ﺍﺸﺘﻘﻘﻨﺎ ﺍﻟﻌﻼﻗﺔ )‪ (8.2‬ﻤﺭﺓ ﺜﺎﻨﻴﺔ ﺒﺎﻟﻨﺴﺒﺔ ﻟﻠﺯﻤﻥ ﻓﺎﻨﻨﺎ ﻨﺠﺩ‪:‬‬
‫‪x1  x  , y1  y   , z1  z  ‬‬ ‫)‪(8.3‬‬

‫ﺍﻟﺸﻜل )‪(8.1‬‬

‫ﺃﻱ ﺃﻥ ﺘﺴﺎﺭﻉ ﺍﻟﻨﻘﻁﺔ ‪ M‬ﻓﻲ ﺍﻟﺠﻤﻠﺔ ‪ K‬ﻴﺴﺎﻭﻱ ﺘﺴﺎﺭﻋﻬﺎ ﻓﻲ ﺍﻟﺠﻤﻠﺔ ‪K1‬‬


‫ﻭﺒﺎﻟﺘﺎﻟﻲ‪ ،‬ﺤﺴﺏ ﻗﺎﻨﻭﻥ ﻨﻴﻭﺘﻥ ﺍﻟﺜﺎﻨﻲ‪ ،‬ﻓﺎﻥ ﺍﻟﻘﻭﺘﻴﻥ ﺍﻟﻤﺅﺜﺭﺘﻴﻥ ﻋﻠﻰ ﺍﻟﻨﻘﻁﺔ ‪M‬‬
‫ﺘﻜﻭﻨﺎﻥ ﻤﺘﺴﺎﻭﻴﺘﻴﻥ ﻓﻲ ﻜﻠﺘﺎ ﺍﻟﺠﻤﻠﺘﻴﻥ‪ ،‬ﻭﻫﺫﺍ ﻴﻌﻨﻲ ﺃﻥ ﺤﺭﻜﺔ ﺍﻟﻨﻘﻁﺔ ‪ M‬ﺘﻜﻭﻥ‬
‫ﻤﺘﺸﺎﺒﻬﺔ ﻓﻲ ﺍﻟﺠﻤﻠﺘﻴﻥ‪ ،‬ﻓﺎﻟﺤﺠﺭ ﺍﻟﻤﺭﻤﻰ ﺍﻟﺫﻱ ﻴﺭﺴﻡ ﻗﻁﻌﺎﹰ ﻤﻜﺎﻓﺌﺎﹰ ﻓﻲ ﺍﻟﺠﻤﻠﺔ ‪K1‬‬
‫ﺴﻴﺭﺴﻡ ﺃﻴﻀﺎﹰ ﻗﻁﻌ ﺎﹰ ﻤﻜﺎﻓﺌﺎﹰ ﻓﻲ ﺍﻟﺠﻤﻠﺔ ‪ .K‬ﻭﻴﺠﺏ ﺍﻟﻤﻼﺤﻅﺔ ﻫﻨﺎ ﺃﻨﻨﺎ ﺍﻋﺘﺒﺭﻨﺎ ﺍﻟﺯﻤﻥ‬
‫ﻨﻔﺴﻪ ﻓﻲ ﺍﻟﻤﺠﻤﻭﻋﺘﻴﻥ‪ ،‬ﻭﻴﻤﻜﻨﻨﺎ ﺍﻟﺤﺼﻭل ﺃﻴﻀﺎﹰ ﻋﻠﻰ ﺍﻟﻨﺘﺎﺌﺞ ﻨﻔﺴﻬﺎ ﺇﺫﺍ ﻓﺭﻀﻨﺎ ﺃﻥ‬
‫ﺍﻟﺠﻤﻠﺔ ‪ K‬ﻫﻲ ﺍﻟﺜﺎﺒﺘﺔ ﻭﺍﻟﺠﻤﻠﺔ ‪ K1‬ﻫﻲ ﺍﻟﻤﺘﺤﺭﻜﺔ ﺒﺴﺭﻋﺔ ) ‪ (V‬ﺒﺎﻟﻨﺴﺒﺔ ﺍﻟﻰ ‪،K‬‬
‫ﺃﻱ ﺃﻥ ﻜﻠﻤﺘﻲ ﺴﺎﻜﻥ ﻭﻤﺘﺤﺭﻙ ﻫﻤﺎ ﻤﻔﻬﻭﻤﺎﻥ ﻨﺴﺒﻴﺎﻥ ﻭﻤﺎ ﻫﻭ ﺴﺎﻜﻥ ﺒﺎﻟﻨﺴﺒﺔ ﻟﻲ ﻗﺩ‬

‫‪- 252 -‬‬

‫)‪Create PDF files without this message by purchasing novaPDF printer (http://www.novapdf.com‬‬
‫ﻴﻜﻭﻥ ﻤﺘﺤﺭﻜﺎﹰ ﺒﺎﻟﻨﺴﺒﺔ ﻟﻐﻴﺭﻱ‪ .‬ﻭﻫﻜﺫﺍ ﻨﻼﺤﻅ ﺃﻥ ﺍﻟﺤﺭﻜﺔ ﺍﻟﻤﺴﺘﻘﻴﻤﺔ ﺍﻟﻤﻨﺘﻅﻤﺔ‬
‫ﻟﺠﻤﻠﺔ ﺍﻟﻘﻴﺎﺱ ﻻﺘﺅﺜﺭ ﻤﻁﻠﻘﺎﹰ ﻋﻠﻰ ﺍﻟﺤﻭﺍﺩﺙ ﺍﻟﻤﻴﻜﺎﻨﻴﻜﻴﺔ ﺍﻟﺘﻲ ﺘﺠﺭﻯ ﻓﻴﻬﺎ‪ .‬ﻭﻫﺫﺍ‬
‫ﻤﺎﻴﻌﺭﻑ ﺒﻤﺒﺩﺃ ﺍﻟﻨﺴﺒﻴﺔ ﻟﻐﺎﻟﻴﻠﻲ‪ ،‬ﻓﻼﻴﻤﻜﻨﻨﺎ ﺍﻟﻭﻗﻭﻑ ﻋﻠﻰ ﺤﻘﻴﻘﺔ ﺤﺭﻜﺔ ﺠﻤﻠﺔ ﻋﻁﺎﻟﻴﺔ‬
‫ﺒﺘﺠﺎﺭﺏ ﻤﻴﻜﺎﻨﻴﻜﻴﺔ ﺘﺠﺭﻯ ﻓﻴﻬﺎ‪.‬‬
‫‪ -61‬ﻇﺎﻫﺮﺓ ﺩﻭﺑﻠﺮ‪ ،‬ﺍﻷﺛﲑ‪ ،‬ﲡﺮﺑﺔ ﻣﺎﻳﻜﻠﺴﻮﻥ ﻭﻣﻮﺭﱄ ‪:‬‬
‫ﻟﻘﺩ ﻜﺎﻥ ﺍﻟﻁﺭﻴﻕ ﺍﻟﺫﻱ ﺃﺩﻯ ﺇﻟﻰ ﻭﺠﻭﺩ ﺍﻟﻨﻅﺭﻴﺔ ﺍﻟﻨﺴﺒﻴﺔ ﺸﺎﻗﺎﹰ ﻭﻁﻭﻴﻼﹰ‪،‬‬
‫ﻭﻻﻴﻤﻜﻨﻨﺎ ﺍﻵﻥ ﺍﻻ ﻜﺘﺎﺒﺔ ﺍﻟﺸﻲﺀ ﺍﻟﻴﺴﻴﺭ ﻋﻥ ﺫﻟﻙ‪ ،‬ﻭﺴﻨﻜﺘﻔﻲ ﺒﺎﻴﺭﺍﺩ ﺃﻫﻡ ﺍﻟﻘﻔﺯﺍﺕ‬
‫ﺍﻟﻌﻠﻤﻴﺔ ﺍﻟﺘﻲ ﺴﺎﻋﺩﺕ ﺍﻟﻌﺎﻟﻡ ﺍﻨﺸﺘﺎﻴﻥ ﻓﻲ ﺍﻜﺘﺸﺎﻑ ﻨﻅﺭﻴﺘﻪ ﺍﻟﻬﺎﻤﺔ‪.‬‬
‫ﺸﻬﺩﺕ ﻨﻬﺎﻴﺔ ﺍﻟﻘﺭﻥ ﺍﻟﺘﺎﺴﻊ ﻋﺸﺭ ﻭﺒﺩﺍﻴﺔ ﺍﻟﻘﺭﻥ ﺍﻟﻌﺸﺭﻴﻥ ﺘﻘﺩﻤ ﺎﹰ ﻜﺒﻴﺭﺍﹰ ﻓﻲ‬
‫ﻋﻠﻭﻡ ﺍﻟﻀﻭﺀ ﻭﺍﻟﻜﻬﺭﺒﺎﺀ ﻭﺍﻀﻁﺭ ﺍﻟﻌﻠﻤﺎﺀ ﻟﻔﺭﺽ ﻭﺠﻭﺩ ﻭﺴﻁ ﻤﺭﻥ‪ ،‬ﺴﻤﻭﻩ ﺍﻷﺜﻴﺭ‪،‬‬
‫ﻟﻜﻲ ﻴﻌﻠﻠﻭﺍ ﺒﻭﺍﺴﻁﺘﻪ ﺍﻟﻅﻭﺍﻫﺭ ﺍﻟﻜﻬﺭﻀﻭﺌﻴﺔ ﺍﻟﺘﻲ ﻭﺍﺠﻬﺘﻬﻡ‪.‬‬
‫ﻭﻗﺎﻟﻭﺍ ﺃﻨﻪ ﻴﻤﻸ ﻜل ﻓﺭﺍﻍ ﻭﻴﺴﺘﻁﻴﻊ ﻨﻘل ﺍﻻﻫﺘﺯﺍﺯﺍﺕ ﺒﻤﺨﺘﻠﻑ ﺃﻨﻭﺍﻋﻬﺎ‬
‫ﻭﺍﻋﺘﺒﺭﻭﻩ ﺴﺎﻜﻨﺎﹰ ﻭﻗﺎﺴﻭﺍ ﺴﺭﻋﺔ ﺍﻟﻀﻭﺀ ﺒﺎﻟﻨﺴﺒﺔ ﻟﻪ‪ ،‬ﻭﺘﺴﺎﺀل ﺍﻟﻌﻠﻤﺎﺀ ﻋﻥ ﺍﻤﻜﺎﻨﻴﺔ‬
‫ﺘﺤﺩﻴﺩ ﺤﺭﻜﺔ ﺍﻟﺠﻤﻠﺔ ﺍﻟﻌﻁﺎﻟﻴﺔ ﺒﺘﺠﺎﺭﺏ ﻀﻭﺌﻴﺔ ﺘﺠﺭﻯ ﻓﻴﻬﺎ‪) ،‬ﻁﺎﻟﻤﺎ ﺃﻨﻪ ﻟﻴﺱ ﻤﻥ‬
‫ﺍﻟﻤﻤﻜﻥ ﻤﻌﺭﻓﺔ ﺫﻟﻙ ﺒﺘﺠﺎﺭﺏ ﻤﻴﻜﺎﻨﻴﻜﻴﺔ( ﻭﻫﺫﺍ ﻤﻤﻜﻥ ﺇﺫﺍ ﻜﺎﻨﺕ ﺍﻟﺤﺭﻜﺔ ﺘﺅﺜﺭ ﻋﻠﻰ‬
‫ﺃﺤﺩ ﻋﻨﺎﺼﺭ ﺍﻟﻤﻭﺠﺔ ﺍﻟﻀﻭﺌﻴﺔ ﻭﻫﻲ ﺘﻭﺍﺘﺭ ﺍﻻﻫﺘﺯﺍﺯ ﻭﺴﺭﻋﺔ ﺍﻻﻨﺘﺸﺎﺭ ﻭﺍﺘﺠﺎﻫﻪ‪.‬‬
‫ﺍﻜﺘﺸﻑ ﺩﻭﺒﻠﺭ ﻋﺎﻡ ‪ /1842/‬ﺘﺄﺜﻴﺭ ﺤ ﺭﻜﺔ ﺍﻟﻤﻨﺒﻊ ﺍﻟﻀﻭﺌﻲ ﻋﻠﻰ ﺘﻭﺍﺘﺭ‬
‫ﺍﻫﺘﺯﺍﺯ ﺍﻟﻤﻭﺠﺔ ﺍﻟﻀﻭﺌﻴﺔ ﺍﻟﺘﻲ ﻴﺼﺩﺭﻫﺎ ﻫﺫﺍ ﺍﻟﻤﻨﺒﻊ‪ .‬ﻓﻠﻨﻔﺭﺽ ﺃﻥ ﺠﻤﻠﺔ ﺍﻟﻘﻴﺎﺱ‬
‫ﺘﺘﺤﺭﻙ ﺒﺴﺭﻋﺔ ﻤﻨﺘﻅﻤﺔ ‪ V‬ﻤﺒﺘﻌﺩﺓ ﻋﻥ ﺍﻟﻤﻨﺒﻊ ﺍﻟﻀﻭﺌﻲ ﺍﻟﺜﺎﺒﺕ ﺒﺩﻻﻟﺔ ﺍﻷﺜﻴﺭ ﺍﻟﺴﺎﻜﻥ‬
‫ﻓﻴﻤﻜﻨﻨﺎ‪ ،‬ﺍﻨﻁﻼﻗﺎﹰ ﻤﻥ ﺘﺤﻭﻴﻼﺕ ﻏﺎﻟﻴﻠﻲ ﺍﻟﺘﺄﻜﺩ ﻤﻥ ﺃﻥ ﺘﻭﺍﺘﺭ ﺍﻟﻤﻭﺠﺔ ﺍﻟﻀﻭﺌﻴﺔ‬
‫ﺴﻴﺨﺘﻠﻑ ﻋﻥ ﺍﻟﺘﻭﺍﺘﺭ ﺍﻟﺤﻘﻴﻘﻲ ﺒﺎﻟﻨﺴﺒﺔ ﻟﺸﺨﺹ ﻤﻘﻴﺩ ﺒﺎﻟﺠﻤﻠﺔ ﺍﻟﻤﺘﺤﺭﻜﺔ ﻭﺴﻴﺼﺒﺢ‪:‬‬
‫)‪1'  (1  ‬‬ ‫)‪(8.4‬‬
‫‪  ‬ﻫﻲ ﻨﺴﺒﺔ ﺴﺭﻋﺔ ﺍﻟﺠﻤﻠﺔ ﺍﻟﻤﺘﺤﺭﻜﺔ ﻋﻠﻰ ﺴﺭﻋﺔ ﺍﻟﻀﻭﺀ‪ .‬ﺃﻤﺎ‬ ‫ﺤﻴﺙ‬
‫‪V‬‬
‫‪c‬‬
‫ﻋﻨﺩﻤﺎ ﻴﺘﺤﺭﻙ ﺍﻟﻤﻨﺒﻊ ﻤﺒﺘﻌﺩ ﺍﹰ ﻋﻥ ﺍﻟﺸﺨﺹ ﺍﻟﺜﺎﺒﺕ ﺒﺩﻻﻟﺔ ﺍﻷﺜﻴﺭ ﻓﺎﻥ ﺘﻁﺒﻴﻕ ﻨﻔﺱ‬
‫ﺍﻟﺘﺤﻭﻴﻼﺕ ﻴﻌﻁﻴﻨﺎ ﺍﻟﻌﻼﻗﺔ ﺍﻟﺘﺎﻟﻴﺔ‪:‬‬

‫‪- 253 -‬‬

‫)‪Create PDF files without this message by purchasing novaPDF printer (http://www.novapdf.com‬‬
‫)‪(8.5‬‬
‫‪‬‬
‫‪ '2 ‬‬
‫‪1 ‬‬
‫ﻭﺍﻟﻤﻘﺩﺍﺭﻴﻥ '‪ 1‬ﻭ ‪  '2‬ﻏﻴﺭ ﻤﺘﺴﺎﻭﻴﻴﻥ ﺍﻻ ﺇﺫﺍ ﻜﺘﺒﻨﺎ ﺃﻥ‪:‬‬
‫)‪(8.6‬‬
‫‪1‬‬
‫‪1  ‬‬ ‫‪ 1  2  1‬‬
‫‪1‬‬
‫ﺃﻱ ﻴﺠﺏ ﺃﻥ ﻨﻬﻤل ﺍﻟﻤﻘﺩﺍﺭ ‪  2‬ﺃﻤﺎﻡ ﺍﻟﻭﺍﺤﺩ‪ ،‬ﻭﻫﺫﺍ ﻴﻌﻨﻲ ﺃﻨﻪ ﺍﺫﺍ ﺍﺴﺘﻁﻌﻨﺎ‬
‫ﺒﺘﺠﺎﺭﺏ ﻀﻭﺌﻴﺔ ﻗﻴﺎﺱ ﺍﻟﻤﻘﺎﺩﻴﺭ ﻤﻥ ﺭﺘﺒﺔ ‪  2‬ﻓﺈﻨﻪ ﻴﻤﻜﻥ ﺘﺤﺩﻴﺩ ﺍﻟﺤﺭﻜﺔ ﺍﻟﺤﻘﻴﻘﻴﺔ‬
‫ﻋﻥ ﺍﻟﺤﺭﻜﺔ ﺍﻟﻨﺴﺒﻴﺔ؛ ﺃﻱ ﻤﻌﺭﻓﺔ ﺍﻟﺠﻤﻠﺔ ﺍﻟﻤﺘﺤﺭﻜﺔ ﻭﺍﻟﺠﻤﻠﺔ ﺍﻟﺜﺎﺒﺘﺔ‪ .‬ﻭﻟﺯﻴﺎﺩﺓ‬
‫ﺍﻻﻴﻀﺎﺡ ﻨﺄﺨﺫ ﺍﻟﻤﺜﺎل ﺍﻟﺘﺎﻟﻲ ﺍﻟﻤﺘﻌﻠﻕ ﺒﻘﻴﺎﺱ ﺍﻟﺯﻤﻥ ﻭﻓﺭﻕ ﺍﻟﻤﺴﻴﺭ ﺒﻴﻥ ﺸﻌﺎﻋﻴﻥ‬
‫ﻀﻭﺌﻴﻴﻥ‪.‬‬
‫ﻟﻴﻜﻥ ﺠﺴﻡ ﻤﺘﺤﺭﻙ ﺒﺴﺭﻋﺔ ‪ V‬ﺒﺩﻻﻟﺔ ﺍﻷﺜﻴﺭ ﺍﻟﺴﺎﻜﻥ ﻭﻟﻨﻀﻊ ﻋﻠﻴﻪ ﻤﻨﺒﻌﺎﹰ ‪A‬‬
‫ﻭﻤﺴﺘﻘﺒﻼﹰ ﻟﻸﻤﻭﺍﺝ ﺍﻟﻜﻬﺭﻁﻴﺴﻴﺔ ‪ B‬ﺍﻟﺒﻌﺩ ﺒﻴﻨﻬﻤﺎ ‪ ، ‬ﻓﺈﺫﺍ ﻜﺎﻨﺕ ﺍﻟﺤﺭﻜﺔ ﺒﺎﺘﺠﺎﻩ‬
‫ﺍﻟﻤﻨﺒﻊ‪-‬ﺍﻟﻤﺴﺘﻘﺒل )ﺃﻱ ﺒﺎﺘﺠﺎﻩ ﺍﻨﺘﺸﺎﺭ ﺍﻟﻤﻭﺠﺔ( ﺸﻜل )‪ (8.2‬ﻓﻴﻤﻜﻥ ﺤﺴﺎﺏ ﺍﻟﺯﻤﻥ‬
‫ﺍﻟﻼﺯﻡ ﻟﻠﻀﻭﺀ ﻟﻴﺼل ﺒﻴﻨﻬﻤﺎ ﺒﺴﻬﻭﻟﺔ ﺍﺫﺍ ﻻﺤﻅﻨﺎ ﺃﻥ ﻋﻠﻰ ﺍﻟﻤﻭﺠﺔ ﺍﻟﻀﻭﺌﻴﺔ ﺃﻥ‬
‫ﺘﻘﻁﻊ‪ ،‬ﺨﻼل ﺯﻤﻥ ‪ ، T1‬ﺍﻟﻤﺴﺎﻓﺔ ‪ ‬ﺒﺎﻻﻀﺎﻓﺔ ﺇﻟﻰ ﺍﻟﻤﺴﺎﻓﺔ ﺍﻟﺘﻲ ﻴﻜﻭﻥ ﻗﺩ ﻗﻁﻌﻬﺎ‬
‫ﺍﻟﻤﺴﺘﻘﺒل ‪ B‬ﻓﻲ ﻫﺫﻩ ﺍﻟﻔﺘﺭﺓ ﻭﻫﻲ ‪ VT1‬ﺃﻱ ﺃﻥ‪:‬‬
‫‪CT1    VT1‬‬
‫ﻭﻤﻨﻪ‪:‬‬
‫‪‬‬
‫‪T1 ‬‬ ‫)‪(8.7‬‬
‫‪CV‬‬
‫ﻓﺈﺫﺍ ﻗﺴﻨﺎ ﺍﻟﺯﻤﻥ ‪ T1‬ﻓﻴﻤﻜﻥ ﺍﻴﺠﺎﺩ ﺴﺭﻋﺔ ﺍﻟﺠﺴﻡ ﺍﻟﻤﺘﺤﺭﻙ ‪ V‬ﺒﺩﻻﻟﺔ ﺍﻷﺜﻴﺭ‪،‬‬
‫ﻏﻴﺭ ﺃﻥ ﺍﻟﺴﺭﻋﺔ ﺍﻟﺘﻲ ﺤﺼل ﻋﻠﻴﻬﺎ ﻓﻲ ﻨﻬﺎﻴﺔ ﺍﻟﻘﺭﻥ ﺍﻟﺘﺎﺴﻊ ﻋﺸﺭ ﻜﺎﻨﺕ ﺼﻐﻴﺭﺓ‬
‫ﺠﺩﺍﹰ ﺒﺤﻴﺙ ﻴﺘﻌﺫﺭ ﻗﻴﺎﺱ ﺍﻟﻤﻘﺩﺍﺭ ‪ ، T1‬ﻭﻟﻜﻥ ﻜﺎﻥ ﻤﻥ ﺍﻟﻤﻤﻜﻥ ﻤﻘﺎﺭﻨﺔ ‪ T1‬ﺒﺯﻤﻥ‬
‫ﺁﺨﺭ ‪ T2‬ﻴﻘﻁﻊ ﺨﻼﻟﻪ ﺍﻟﻤﺴﺘﻘﺒل ‪ B‬ﺍﻟﻤﺴﺎﻓﺔ '‪ BB‬ﻜﻤﺎ ﻓﻲ ﺍﻟﺸﻜل )‪ (8.3‬ﺤﻴﺙ ﺘﺘﻡ‬
‫ﺍﻟﺤﺭﻜﺔ ﺒﺎﺘﺠﺎﻩ ﻤﺘﻌﺎﻤﺩ ﻤﻊ ﺍﺘﺠﺎﻩ ﺤﺭﻜﺔ ﺠﻤﻠﺔ ﺍﻟﻤﻨﺒﻊ‪ -‬ﺍﻟﻤﺴﺘﻘﺒل ﻭﻋﻨﺩﺌﺫ‪ ‬ﻨﺠﺩ ﺃﻥ‬
‫ﺍﻟﺯﻤﻥ ﺍﻟﻼﺯﻡ ﻟﻜﻲ ﻴﻘﻁﻊ ﺍﻟﻀﻭﺀ ﺍﻟﻤﺴﺎﻓﺔ '‪ AB‬ﻴﺴﺎﻭﻱ‪:‬‬

‫‪- 254 -‬‬

‫)‪Create PDF files without this message by purchasing novaPDF printer (http://www.novapdf.com‬‬
‫‪ 2  V 2 T22‬‬
‫‪T2 ‬‬ ‫)‪(8.8‬‬
‫‪c‬‬
‫ﻭﻤﻨﻪ‪:‬‬
‫‪‬‬
‫‪T2 ‬‬ ‫)‪(8.9‬‬
‫‪2‬‬
‫‪V‬‬
‫‪c 1‬‬
‫‪c2‬‬

‫ﺍﻟﺸﻜل )‪(8.3‬‬ ‫ﺍﻟﺸﻜل )‪(8.2‬‬

‫ﻓﺈﺫﺍ ﺠﻌﻠﻨﺎ ﺍﻟﺸﻌﺎﻋﻴﻥ ﺍﻟﻀﻭﺌﻴﻴﻥ ﺍﻟﻤﻭﺍﻓﻘﻴﻥ ﻟﻠﺤﺎﻟﺘﻴﻥ ﻴﺘﺩﺍﺨﻼﻥ ﻓﺴﻴﻜﻭﻥ ﺒﻴﻨﻬﻤﺎ‬


‫ﻓﺭﻕ ﻓﻲ ﺍﻟﻤﺴﻴﺭ ﺍﻟﻀﻭﺌﻲ ﻗﺩﺭﻩ ) ‪ (T2  T1‬ﻤﻤﺎ ﻴﺴﺒﺏ ﺘﺩﺍﺨﻠﻬﻤﺎ‪ ،‬ﻭﻤﻥ ﺩﺭﺍﺴﺔ ﻫﺫﺍ‬
‫ﺍﻟﺘﺩﺍﺨل ﻴﻤﻜﻨﻨﺎ ﺤﺴﺎﺏ ‪ .V‬ﻭﻗﺩ ﺍﺴﺘﻁﺎﻉ ﻤﺎﻴﻜﻠﺴﻭﻥ ﻋﺎﻡ ‪ 1882‬ﺘﺤﻘﻴﻕ ﺫﻟﻙ؛ ﻓﻘﺩ‬
‫ﺼﻤﻡ ﺠﻬﺎﺯﺍﹰ ﻟﻠﺘﺩﺍﺨل ﻗﺎﺩﺭﺍﹰ ﻋﻠﻰ ﺍﻅﻬﺎﺭ ﻓﺭﻭﻕ ﻓﻲ ﺍﻟﻤﺴﻴﺭ ﺃﺼﻐﺭ ﻤﻥ ﺍﻟﻘﻴﻤﺔ‬
‫ﺍﻟﺴﺎﺒﻘﺔ ﺒﻜﺜﻴﺭ )ﺍﻟﺸﻜل ‪ (8.4‬ﻭﻫﻭ ﻴﺘﺄﻟﻑ ﻤﻥ ﻤﻨﺒﻊ ﻀﻭﺌﻲ ‪ I‬ﻴﺼﺩﺭ ﺃﻤﻭﺍﺠﺎﹰ ﻀﻭﺌﻴﺔ‬
‫ﺘﺴﻘﻁ ﻋﻠﻰ ﺼﻔﻴﺤﺔ ﻨﺼﻑ ﻤﻔﻀﻀﺔ ‪ P‬ﻤﺎﺌﻠﺔ ﺒﺯﺍﻭﻴﺔ ﻤﻘﺩﺍﺭﻫﺎ ‪ 45‬ﺩﺭﺠﺔ ﻓﺘﻨﻘﺴﻡ‬
‫ﺍﻟﺤﺯﻤﺔ ﺍﻟﻀﻭﺌﻴﺔ ﺍﻟﻭﺍﺭﺩﺓ ﺇﻟﻰ ﻗﺴﻤﻴﻥ ﺃﺤﺩﻫﻤﺎ ﻴﺘﺤﺭﻙ ﺒﺎﺘﺠﺎﻩ ﻋﻤﻭﺩﻱ ﻋﻠﻰ ﺤﺭﻜﺔ‬
‫ﺍﻷﺭﺽ ﺍﻻﻨﺴﺤﺎﺒﻴﺔ‪ ،‬ﻭﻋﻨﺩﻤﺎ ﺘﻘﻁﻊ ﻤﺴﺎﻓﺔ ‪ ‬ﺘﺴﻘﻁ ﻋﻠﻰ ﻤﺭﺁﺓ ﻤﺴﺘﻭﻴﺔ ﻭﺘﻨﻌﻜﺱ‬
‫ﻋﻠﻴﻬﺎ ﻟﺘﻠﺘﻘﻲ ﻤﻊ ﺍﻟﺤﺯﻤﺔ ﺍﻷﺨﺭﻯ ﺍﻟﺘﻲ ﺘﺴﻴﺭ ﺒﺎﺘﺠﺎﻩ ﺤﺭﻜﺔ ﺍﻷﺭﺽ ﻭﺍﻟﺘﻲ ﻗﻁﻌﺕ‬
‫ﺍﻟﻤﺴﺎﻓﺔ ﻨﻔﺴﻬﺎ ‪ ‬ﻗﺒل ﺃﻥ ﺘﻨﻌﻜﺱ ﻋﻠﻰ ﻤﺭﺁﺓ ﻤﺴﺘﻭﻴﺔ ﻤﺘﻌﺎﻤﺩﺓ ﻤﻊ ﺍﻷﻭﻟﻰ‪ ،‬ﺜﻡ ﺘﻠﺘﻘﻲ‬
‫ﺍﻟﺤﺯﻤﺘﺎﻥ ﻓﻲ ﺍﻟﻨﻘﻁﺔ ‪ O‬ﻟﺘﺼﻼ ﻓﻴﻤﺎ ﺒﻌﺩ ﺇﻟﻰ ﻋﻴﻥ ‪ .S‬ﻓﺈﺫﺍ ﻜﺎﻥ ﺒﻴﻨﻬﻤﺎ ﺃﻱ ﻓﺭﻕ ﻓﻲ‬

‫‪- 255 -‬‬

‫)‪Create PDF files without this message by purchasing novaPDF printer (http://www.novapdf.com‬‬
‫ﺍﻟﻤﺴﻴﺭ ﻓﺈﻨﻨﺎ ﺴﻨﺸﺎﻫﺩ ﺃﻫﺩﺍﺏ ﺍﻟﺘﺩﺍﺨل‪ ،‬ﻭﻟﻜﻥ ﺫﻟﻙ ﻟﻡ ﻴﺤﺩﺙ ﺃﺒﺩﺍﹰ ﻭﻟﻡ ﻴﻼﺤﻅ ﺃﻱ‬
‫ﻓﺭﻕ ﻓﻲ ﺍﻟﻤﺴﻴﺭ ﺒﻴﻥ ﺍﻟﺸﻌﺎﻋﻴﻥ‪ ،‬ﻭﺒﺎﻟﺘﺎﻟﻲ ﻓﺈﻥ ﺍﻟﺯﻤﻨﻴﻥ ‪ T1‬ﻭ ‪ T2‬ﻤﺘﺴﺎﻭﻴﺎﻥ ﺘﻤﺎﻤﺎﹰ‪.‬‬
‫ﻭ ﻗﺩ ﺃﻋﻴﺩﺕ ﺘﺠﺭﺒﺔ ﻤﺎﻴﻜﻠﺴﻭﻥ ﻋﺩﺓ ﻤﺭﺍﺕ‪ ،‬ﻜﻤﺎ ﺘﻡ ﺍﺠﺭﺍﺀ ﺘﺠﺎﺭﺏ ﺃﺨﺭﻯ ﻟﻨﻔﺱ‬
‫ﺍﻟﻐﺭﺽ ﻨﺫﻜﺭ ﻤﻨﻬﺎ ﺘﺠﺭﺒﺔ ﺩﻴﺴﺘﻴﺭ ‪ 1912 Destaire‬ﺍﻟﺫﻱ ﺍﻗﺘﺭﺡ ﺍﺠﺭﺍﺀ ﺍﻟﺘﺠﺎﺭﺏ‬
‫ﻋﻠﻰ ﻤﺎﻴﺴﻤﻰ ﺒﺎﻟﻨﺠﻭﻡ ﺍﻟﻤﺯﺩﻭﺠﺔ‪ ،‬ﻭﻫﻲ ﺘﻠﻙ ﺍﻟﺘﻲ ﺘﺘﺄﻟﻑ ﻤﻥ ﻨﺠﻤﻴﻥ ﻗﺭﻴﺒﻴﻥ ﺃﺤﺩﻫﻤﺎ‬
‫ﻤﻥ ﺍﻵﺨﺭ ﻭﻤﺘﺤﺭﻜﻴﻥ ﺤﻭل ﻤﺭﻜﺯ ﻋﻁﺎﻟﺘﻬﻤﺎ‪ ،‬ﻓﺈﺫﺍ ﻗﺴﻨﺎ ﺴﺭﻋﺔ ﺍﻟﻀﻭﺀ ﻋﻨﺩﻤﺎ‬
‫ﻴﻘﺘﺭﺏ ﺍﻟﻨﺠﻡ ﻤﻥ ﺍﻷﺭﺽ ﻭﻋﻨﺩﻤﺎ ﻴﺒﺘﻌﺩ ﻋﻨﻬﺎ ﻓﻴﻤﻜﻥ ﺘﻌﻴﻴﻥ ﺴﺭﻋﺘﻬﻤﺎ ﺒﺩﻻﻟﺔ ﺍﻷﺜﻴﺭ‬
‫ﺍﻟﺴﺎﻜﻥ‪ .‬ﻏﻴﺭ ﺃﻥ ﻜل ﻫﺫﻩ ﺍﻟﻤﺤﺎﻭﻻﺕ ﻟﻡ ﺘﻨﺠﺢ ﺒﺎﻅﻬﺎﺭ ﺃﻱ ﻓﺭﻭﻕ ﻓﻲ ﺍﻟﻤﺴﻴﺭ‪،‬‬
‫ﻓﻅﻬﺭﺕ ﻋﺩﺓ ﻨﻅﺭﻴﺎﺕ ﻟﺘﻘﻠﻴل ﺴﻠﺒﻴﺎﺕ ﻫﺫﻩ ﺍﻟﺘﺠﺎﺭﺏ ﻟﻜﻨﻬﺎ ﻟﻡ ﺘﻘﺒل ﻭﺴﻘﻁﺕ ﺠﻤﻴﻌﻬﺎ‬
‫ﺃﻤﺎﻡ ﺍﻟﻨﻅﺭﻴﺔ ﺍﻟﻨﺴﺒﻴﺔ‪.‬‬

‫ﺍﻟﺸﻜل )‪(8- 4‬‬

‫‪ -62‬ﻓﺮﺿﻴﺘﺎ ﺍﻧﺘﺸﺎﻳﻦ‪The Einstein's Postulates :‬‬


‫ﺍﻥ ﺍﻟﻨﺘﺎﺌﺞ ﺍﻟﺴﻠﺒﻴﺔ ﻟﺘﺠﺎﺭﺏ ﻤﺎﻴﻜﻠﺴﻭﻥ ﻗﺎﺩﺕ ﺃﻨﺸﺘﺎﻴﻥ ﺇﻟﻰ ﺍﻋﺎﺩﺓ ﺍﻟﻨﻅﺭ‬
‫ﻭﺍﻟﺩﺭﺍﺴﺔ ﻭﺍﻟﺘﻔﻜﻴﺭ ﻓﻲ ﻤﺒﺎﺩﻯﺀ ﺍﻟﻔﻴﺯﻴﺎﺀ ﺍﻟﻜﻼﺴﻴﻜﻴﺔ ﻭﺨﺎﺼﺔﹰ ﺘﻠﻙ ﺍﻟﺘﻲ ﺘﺘﻌﻠﻕ‬
‫ﺒﺎﻟﻤﻜﺎﻥ ﻭﺍﻟﺯﻤﺎﻥ‪ ،‬ﻭﻨﺘﻴﺠﺔ ﻟﺫﻟﻙ ﺍﻜﺘﺸﻑ ﻤﺎﻴﺴﻤﻰ ﺍﻵﻥ ﺒﺎﻟﻨﻅﺭﻴﺔ ﺍﻟﻨﺴﺒﻴﺔ ﺍﻟﺨﺎﺼﺔ‬
‫ﺍﻟﺘﻲ ﺘﺭﺘﻜﺯ ﻋﻠﻰ ﻓﺭﻀﻴﺘﻴﻥ ﺃﺴﺎﺴﻴﺘﻴﻥ‪.‬‬

‫‪- 256 -‬‬

‫)‪Create PDF files without this message by purchasing novaPDF printer (http://www.novapdf.com‬‬
‫‪ -1‬ﻓﺭﻀﻴﺔ ﻨﺴﺒﻴﺔ ﺃﻨﺸﺘﺎﻴﻥ‪:‬‬
‫‪ -2‬ﻓﺭﻀﻴﺔ ﺍﻟﺴﺭﻋﺔ ﺍﻟﺤﺩﻴﺔ ﺍﻟﺜﺎﺒﺘﺔ ﻻﻨﺘﺸﺎﺭ ﺘﺄﺜﻴﺭ ﺍﻷﺠﺴﺎﻡ ﺒﻌﻀﻬﺎ ﺒﺒﻌﺽ‬
‫ﻭﺍﻟﺘﻲ ﺘﻌﻨﻲ ﺃﻥ ﺃﻱ ﺘﺄﺜﻴﺭ ﻤﺘﺒﺎﺩل ﻤﺎ ﺒﻴﻥ ﺃﻱ ﺠﺴﻤﻴﻥ ﻻﻴﻤﻜﻥ ﺃﻥ ﻴﺘﻡ ﻟﺤﻅﻴﺎﹰ ﻭﺇﻨﻤﺎ‬
‫ﻴﻨﺘﺸﺭ ﻓﻲ ﺍﻟﻔﺭﺍﻍ ﺒﺴﺭﻋﺔ ﺤﺩﻴﺔ ﺜﺎﺒﺘﺔ ﻻﺘﺘﺠﺎﻭﺯ ﺴﺭﻋﺔ ﺍﻟﻀﻭﺀ )ﺃﻭ ﺴﺭﻋﺔ ﺍﻷﻤﻭﺍﺝ‬
‫ﺍﻟﻜﻬﺭﻁﻴﺴﻴﺔ ﺒﺼﻭﺭﺓ ﻋﺎﻤﺔ( ﻭﻻﺘﺘﺄﺜﺭ ﻫﺫﻩ ﺍﻟﺴﺭﻋﺔ ﺒﺤﺭﻜﺔ ﺍﻟﻤﻨﺒﻊ ﺍﻟﺒﺎﻋﺙ ﻟﻬﺎ‪.‬‬
‫ﻁﺒﻘﺎﹰ ﻟﻠﻔﺭﻀﻴﺔ ﺍﻷﻭﻟﻰ ﻓﺈﻥ ﺍﻟﺤﺭﻜﺔ ﺍﻟﻤﺴﺘﻘﻴﻤﺔ ﺍﻟﻤﻨﺘﻅﻤﺔ ﻟﻸﺠﺴﺎﻡ ﻻﺘﺅﺜﺭ‬
‫ﻤﻁﻠﻘﺎﹰ ﻋﻠﻰ ﺍﻟﺤﻭﺍﺩﺙ ﺍﻟﺘﻲ ﺘﺠﺭﻯ ﻓﻴﻬﺎ‪ ،‬ﺃﻱ ﺃﻥ ﻗﻭﺍﻨﻴﻥ ﺍﻟﻁﺒﻴﻌﺔ ﻴﺠﺏ ﺃﻥ ﺘﻜﻭﻥ‬
‫ﻤﺘﺸﺎﺒﻬﺔ ﻓﻲ ﻜل ﺍﻟﺠﻤل ﺍﻟﻌﻁﺎﻟﻴﺔ ﻓﺈﺫﺍ ﻋﺒﺭﻨﺎ ﻋﻥ ﻗﺎﻨﻭﻥ ﻓﻲ ﺍﺤﺩﺍﻫﺎ ﺒﺩﻻﻟﺔ‬
‫ﺍﻻﺤﺩﺍﺜﻴﺎﺕ ﻭﺍﻟﺯﻤﻥ ﺒﻌﻼﻗﺔ ﺭﻴﺎﻀﻴﺔ ﻤﺎ‪ ،‬ﻓﺈﻨﻪ ﻴﺠﺏ ﺃﻥ ﻴﻌﺒﺭ ﻋﻥ ﺍﻟﻘﺎﻨﻭﻥ ﻨﻔﺴﻪ‬
‫ﺒﺎﻟﻌﻼﻗﺔ ﻨﻔﺴﻬﺎ ﻓﻲ ﺍﻟﺠﻤل ﺍﻟﻌﻁﺎﻟﻴﺔ ﺍﻷﺨﺭﻯ ﻜﺎﻓﺔﹰ‪ ،‬ﻭﻫﻜﺫﺍ ﻜﻤﺎ ﻨﻼﺤﻅ‪ ،‬ﺘﻌﻤﻴﻡ‬
‫ﻤﺒﺎﺸﺭﺓ ﻟﻤﺒﺩﺃ ﻏﺎﻟﻴﻠﻲ ﺍﻟﺫﻱ ﺩﺭﺴﻨﺎﻩ ﻓﻲ ﺒﺩﺍﻴﺔ ﻫﺫﺍ ﺍﻟﻔﺼل‪ ،‬ﻭﻟﻜﻥ ﺍﻻﺨﺘﻼﻑ ﺍﻟﺭﺌﻴﺴﻲ‬
‫ﺒﻴﻥ ﺍﻟﻨﻅﺭﻴﺘﻴﻥ ﻫﻭ ﻜﻴﻔﻴﺔ ﺍﻻﻨﺘﻘﺎل ﻤﻥ ﺠﻤﻠﺔ ﻟﻠﻘﻴﺎﺱ ﺇﻟﻰ ﺠﻤﻠﺔ ﺃﺨﺭﻯ‪ .‬ﻓﻘﺩ ﻜﺎﻨﺕ‬
‫ﺍﻟﻌﻼﻗﺎﺕ )‪ (8.1‬ﻜﺎﻓﻴﺔ ﻟﺫﻟﻙ ﺴﺎﺒﻘﺎﹰ ﺃﻤﺎ ﺍﻵﻥ ﻓﻴﺠﺏ ﺼﻴﺎﻏﺔ ﺍﻟﻌﻼﻗﺎﺕ ﺒﻴﻥ ﺍﻟﺯﻤﻥ‬
‫ﻭﺍﻟﻤﻜﺎﻥ ﻓﻲ ﺍﻟﺠﻤﻠﺘﻴﻥ ﻤﻥ ﺠﺩﻴﺩ ﺍﻨﻁﻼﻗﺎﹰ ﻤﻥ ﺍﻟﻔﺭﻀﻴﺔ ﺍﻟﺜﺎﻨﻴﺔ ﻭﺍﻟﺘﻲ ﺘﻅﻬﺭ ﺒﻭﻀﻭﺡ‬
‫ﺃﻨﻬﺎ ﺍﻟﺘﻌﺒﻴﺭ ﺍﻟﻨﻅﺭﻱ ﻋﻥ ﺘﺠﺭﺒﺔ ﻤﺎﻴﻜﻠﺴﻭﻥ‪.‬‬
‫ﺍﻥ ﺍﻟﻔﺭﻀﻴﺔ ﺍﻟﺜﺎﻨﻴﺔ ﺘﻨﻔﻲ ﺍﻤﻜﺎﻨﻴﺔ ﺤﺭﻜﺔ ﺃﺠﺴﺎﻡ ﻤﺎﺩﻴﺔ ﺒﺴﺭﻋﺔ ﺃﻜﺒﺭ ﻤﻥ‬
‫ﺴﺭﻋﺔ ﺍﻟﻀﻭﺀ ﺃﻭ ﺍﻷﻤﻭﺍﺝ ﺍﻟﻜﻬﺭﻁﻴﺴﻴﺔ ﻷﻨﻪ ﻟﻭ ﺘﺤﻘﻕ ﺫﻟﻙ ﻻﻨﺘﺸﺭ ﺍﻟﺘﺄﺜﻴﺭ ﺒﺴﺭﻋﺔ‬
‫ﺃﻜﺒﺭ ﻤﻥ ﺴﺭﻋﺔ ﺍﻟﻀﻭﺀ ﺒﻭﺍﺴﻁﺔ ﻫﺫﻩ ﺍﻷﺠﺴﺎﻡ ﺍﻟﻤﺎﺩﻴﺔ ﻭﻫﺫﺍ ﺒﺤﺩ ﺫﺍﺘﻪ ﻴﻨﺎﻗﺽ‬
‫ﺍﻟﻔﺭﻀﻴﺔ ﻨﻔﺴﻬﺎ‪.‬‬
‫ﻟﻨﻼﺤﻅ ﺃﻥ ﺍﻟﻔﺭﻀﻴﺔ ﺍﻟﺜﺎﻨﻴﺔ ﻤﺭﺘﺒﻁﺔ ﺘﻤﺎﻡ ﺍﻻﺭﺘﺒﺎﻁ ﻤﻊ ﺍﻷﻭﻟﻰ ﻷﻨﻪ ﺇﺫﺍ ﻟﻡ‬
‫ﺘﻜﻥ ﺴﺭﻋﺔ ﺍﻷﻤﻭﺍﺝ ﺍﻟﻀﻭﺌﻴﺔ ﺤﺩﻴﺔ ﻭﺜﺎﺒﺘﺔ‪ ،‬ﻜﺄﻥ ﺘﺘﺒﻊ ﻤﺜﻼﹰ ﺴﺭﻋﺔ ﺍﻟﻤﻨﺒﻊ ﻓﺈﻥ‬
‫ﺍﻟﻔﺭﻀﻴﺔ ﺍﻷﻭﻟﻰ ﻟﻥ ﺘﺘﺤﻘﻕ‪ .‬ﻓﻤﺜﻼﹰ ﺇﺫﺍ ﺘﻭﻗﻔﺕ ﺴﺭﻋﺔ ﺍﻟﻀﻭﺀ ﻋﻠﻰ ﺴﺭﻋﺔ ﺍﻟﻤﻨﺒﻊ‬
‫ﺍﻟﻤﺴﺘﻘﻴﻤﺔ ﺍﻟﻤﻨﺘﻅﻤﺔ ﻓﻴﻤﻜﻨﻨﺎ ﻋﻨﺩﺌﺫ‪ ‬ﺘﻌﻴﻴﻥ ﻫﺫﻩ ﺍﻟﺴﺭﻋﺔ ﺒﻭﺍﺴﻁﺔ ﺘﺠﺎﺭﺏ ﺃﺨﺭﻯ‬
‫ﺘﺠﺭﻯ ﻓﻲ ﺠﻤﻠﺔ ﺍﻟﻤﻨﺒﻊ ﺍﻟﻀﻭﺌﻲ‪.‬‬

‫‪- 257 -‬‬

‫)‪Create PDF files without this message by purchasing novaPDF printer (http://www.novapdf.com‬‬
‫ﺍﻥ ﻜل ﻨﺘﺎﺌﺞ ﺍﻟﻨﻅﺭﻴﺔ ﺍﻟﻨﺴﺒﻴﺔ ﻴﺠﺏ ﺃﻥ ﺘﺄﺘﻲ ﻤﻥ ﺍﻟﻔﺭﻀﻴﺘﻴﻥ ﺍﻟﺴﺎﺒﻘﺘﻴﻥ‬
‫ﻭﻟﻨﺒﺤﺙ ﺃﻭﻻﹰ ﻓﻲ ﻜﻴﻔﻴﺔ ﺍﻻﻨﺘﻘﺎل ﻤﻥ ﺠﻤﻠﺔ ﻋﻁﺎﻟﻴﺔ ﺍﻟﻰ ﺃﺨﺭﻯ ﻻﺴﺘﻨﺘﺎﺝ ﻤﺎﻴﺴﻤﻰ‬
‫ﺒﺘﺤﻭﻴﻼﺕ ﻟﻭﺭﻨﺘﺯ‪.‬‬
‫‪ -63‬ﲢﻮﻳﻼﺕ ﻟﻮﺭﻧﺘﺰ‪ :‬ﻟﺘﻜﻥ ﺍﻟﺠﻤﻠﺘﺎﻥ ‪ K‬ﻭ '‪ K‬ﺸﻜل )‪ (8.5‬ﺍﻟﻤﺘﺤﺭﻜﺘﺎﻥ‬
‫ﺃﺤﺩﺍﻫﻤﺎ ﺒﺎﻟﻨﺴﺒﺔ ﻟﻸﺨﺭﻯ ﺒﺴﺭﻋﺔ ﻨﺴﺒﻴﺔ ‪ V‬ﻭﻟﺘﻜﻥ ) ‪ ( x, y, z, t‬ﻭ ) ' ‪( x ' , y' , z' , t‬‬
‫ﺍﻻﺤﺩﺍﺜﻴﺎﺕ ﻭﺍﻟﺯﻤﻥ ﻓﻲ ﻜل ﻤﻨﻬﺎ‪ ،‬ﻓﻠﻴﺱ ﻫﻨﺎﻙ ﺃﻱ ﺴﺒﺏ ﻴﺠﻌﻠﻨﺎ ﻨﻀﻊ ﺍﻟﺯﻤﻨﻴﻥ‬
‫ﻤﺘﺸﺎﺒﻬﻴﻥ ﻓﻲ ﺍﻟﺠﻤﻠﺘﻴﻥ ﻜﻤﺎ ﻓﻌﻠﻨﺎ ﺴﺎﺒﻘﺎﹰ ﻓﻲ ﺍﻟﻤﻴﻜﺎﻨﻴﻙ ﺍﻟﻜﻼﺴﻴﻜﻲ ﺒﺩﻭﻥ ﺩﺭﺍﺴﺔ‬
‫ﻭﺘﻤﺤﻴﺹ‪ ،‬ﻭﻟﻠﺘﺴﻬﻴل ﺴﻨﻔﺭﺽ ﺃﻥ ﺍﻟﺤﺭﻜﺔ ﺍﻟﻨﺴﺒﻴﺔ ﺘﺠﺭﻯ ﻓﻲ ﺍﻻﺘﺠﺎﻩ ‪ ox‬ﻭﻟﻨﻔﺭﺽ‬
‫ﺃﻨﻪ ﻓﻲ ﺍﻟﻠﺤﻅﺔ '‪ t‬ﻤﻘﺩﺭﺓ ﻓﻲ ﺍﻟﺠﻤﻠﺔ '‪ K‬ﺘﺤﺩﺙ ﻅﺎﻫﺭﺓ ﻓﻴﺯﻴﺎﺌﻴﺔ ﻓﻲ ﻨﻘﻁﺔ '‪M‬‬
‫ﺍﺤﺩﺍﺜﻴﺎﺘﻬﺎ ﺍﻟﺩﻴﻜﺎﺭﺘﻴﺔ ' ‪ ، x' , y' , z‬ﺘﺴﻤﻰ ﻫﺫﻩ ﺍﻟﻅﺎﻫﺭﺓ ﺤﺎﺩﺜﺔ ﻓﻴﺯﻴﺎﺌﻴﺔ ﻭﻨﻘﻭل ﺃﻥ‬
‫ﺍﺤﺩﺍﺜﻴﺎﺘﻬﺎ ﻫﻲ ) ' ‪ (x ' , y' , z' , t‬ﻭﻟﺘﺤﺩﻴﺩ ﺍﺤﺩﺍﺜﻴﺎﺕ ﻫﺫﻩ ﺍﻟﺤﺎﺩﺜﺔ ) ‪ (x, y, z, t‬ﻓﻲ ﺍﻟﺠﻤﻠﺔ‬
‫‪ K‬ﻭﺍﻴﺠﺎﺩ ﺍﻟﻌﻼﻗﺔ ﺒﻴﻥ ) ‪ (x, y, z, t‬ﻭ ) ' ‪ (x ' , y' , z' , t‬ﻨﺄﺨﺫ ﺼﺩﺭ ﺍﻟﻤﻭﺠﺔ ﺍﻟﻀﻭﺌﻴﺔ‬
‫ﺍﻟﻤﻨﻁﻠﻘﺔ ﻓﻲ ﺍﻟﻠﺤﻅﺔ ‪ ، t  t '  0‬ﺃﻱ ﻋﻨﺩﻤﺎ ﻜﺎﻨﺕ ﺍﻟﺠﻤﻠﺘﺎﻥ ﻤﻨﻁﺒﻘﺘﻴﻥ‪ ،‬ﺍﻥ ﺼﺩﺭ‬
‫ﻫﺫﻩ ﺍﻟﻤﻭﺠﺔ ﺴﻴﺭﺴﻡ ﻜﺭﺓ ﻤﻌﺎﺩﻟﺘﻬﺎ‪:‬‬
‫‪x 2  y2  z2  c2 t 2  0‬‬ ‫)‪(8.10‬‬

‫‪Z‬‬
‫ﺍﻟﺸﻜل )‪(5.8‬‬
‫ﻭﺤﺴﺏ ﺍﻟﻔﺭﻀﻴﺔ ﺍﻷﻭﻟﻰ ﻓﺈﻥ ﺍﻟﻤﻭﺠﺔ ﺍﻟﻀﻭﺌﻴﺔ ﻟﻬﺫﻩ ﺍﻟﺤﺎﺩﺜﺔ ﺍﻟﻔﻴﺯﻴﺎﺌﻴﺔ‬
‫ﺴﺘﻨﺘﺸﺭ ﺃﻴﻀﺎﹰ ﻓﻲ ﺍﻟﺠﻤﻠﺔ '‪ K‬ﻭﺒﻨﻔﺱ ﺍﻟﻘﺎﻨﻭﻥ ﻭﺒﺴﺭﻋﺔ ‪ c‬ﺤﺴﺏ ﺍﻟﻔﺭﻀﻴﺔ ﺍﻟﺜﺎﻨﻴﺔ‬
‫ﺃﻱ‪:‬‬

‫‪- 258 -‬‬

‫)‪Create PDF files without this message by purchasing novaPDF printer (http://www.novapdf.com‬‬
‫)‪x' 2  y' 2 z ' 2 c 2 t ' 2  x 2  y 2  z 2  c 2 t 2  0 (8.11‬‬
‫ﻭﻋﻨﺩ ﺍﻻﻨﺘﻘﺎل ﻤﻥ ﺠﻤﻠﺔ ﺍﻟﻰ ﺃﺨﺭﻯ ﻴﺠﺏ ﺃﻭﻻﹰ ﺃﻥ ﺘﺒﻘﻰ ﺍﻟﻌﻼﻗﺎﺕ )‪،(8.10‬‬
‫)‪ (8.11‬ﻤﺤﻘﻘﺔ ﻭﺜﺎﻨﻴﺎﹰ ﺃﻥ ﻴﻜﻭﻥ ﻫﺫﺍ ﺍﻻﻨﺘﻘﺎل ﺨﻁﻴ ﺎﹰ ﻭﺫﻟﻙ ﺍﻨﻁﻼﻗﺎﹰ ﻤﻥ ﺘﺠﺎﻨﺱ‬
‫ﺍﻟﻔﺭﺍﻍ ﺍﻟﺫﻱ ﻻﻴﻭﺠﺩ ﻓﻴﻪ ﻨﻘﻁ ﻤﻤﻴﺯﺓ ﺘﺘﻤﺘﻊ ﻋﻥ ﻏﻴﺭﻫﺎ ﺒﺼﻔﺎﺕ ﺨﺎﺼﺔ‪.‬‬
‫ﻭﻟﻤﺎ ﻜﺎﻨﺕ ﺍﻟﺤﺭﻜﺔ ﺘﺤﺩﺙ ﻓﻲ ﺍﻻﺘﺠﺎﻩ ‪ ox‬ﻓﻴﻤﻜﻥ ﻤﺒﺎﺸﺭﺓﹰ ﻜﺘﺎﺒﺔ ﺃﻥ‪:‬‬
‫‪y'  y , z'  z‬‬ ‫)‪(8.12‬‬
‫ﻭﻻﻴﺠﺎﺩ ﺍﻟﻌﻼﻗﺔ ﻤﺎ ﺒﻴﻥ ‪ x‬ﻭ '‪ x‬ﻨﻼﺤﻅ ﺃﻨﻪ ﺍﺫﺍ ﺍﻨﻁﺒﻕ ﻤﺒﺩﺀﺍ ﺍﻟﺠﻤﻠﺘﻴﻥ ﻓﻲ‬
‫ﺍﻟﻠﺤﻅﺔ ‪ t  0‬ﻓﺈﻥ ﻤﻌﺎﺩﻟﺔ ﺍﻟﻤﺴﺘﻭﻱ ‪ x '  0‬ﺀ ﻓﻲ ﺍﻟﺠﻤﻠﺔ '‪ K‬ﺴﺘﺼﺒﺢ ‪x  Vt  0‬‬
‫ﻓﻲ ﺍﻟﺠﻤﻠﺔ ‪ K‬ﺃﻱ ﺃﻥ ) ‪ x'  ( x  Vt‬ﻭﻓﻲ ﺍﻟﺤﺎﻟﺔ ﺍﻟﻌﺎﻤﺔ ﺴﻴﻜﻭﻥ ﻫﺫﺍ ﺍﻟﺜﺎﺒﺕ ‪‬‬
‫ﺘﺎﺒﻌﺎﹰ ﻟﻠﺴﺭﻋﺔ ﺃﻱ‪:‬‬
‫) ‪x'  ( V ) ( x  Vt‬‬ ‫)‪(8.13‬‬
‫ﺤﻴﺙ ) ‪ (V‬ﻫﻭ ﺘﺎﺒﻊ ﻟﻠﺴﺭﻋﺔ ﻓﻘﻁ ﻭﻻﻴﻤﻜﻥ ﺃﻥ ﻴﺘﺒﻊ ﺍﻻﺤﺩﺍﺜﻴﺎﺕ‪ .‬ﺃﻤﺎ‬
‫ﺒﺎﻟﻨﺴﺒﺔ ﻟﻠﺯﻤﻥ ﻓﻲ ﻜﻠﺘﺎ ﺍﻟﺠﻤﻠﺘﻴﻥ ﻓﻴﻤﻜﻥ ﺃﻥ ﻨﻔﺭﺽ ﻋﻼﻗﺔ ﺍﻟﺘﺤﻭﻴل ﺍﻟﺨﻁﻴﺔ‪:‬‬
‫‪t '   t  x‬‬ ‫)‪(8.14‬‬
‫ﺤﻴﺙ ‪ ‬ﻭ ‪ ‬ﺘﺎﺒﻌﺎﻥ ﻴﺠﺏ ﺘﻌﻴﻴﻨﻬﻤﺎ‪ ،‬ﻭﻴﻤﻜﻥ ﻓﻲ ﺍﻟﺤﺎﻟﺔ ﺍﻟﻌﺎﻤﺔ ﺃﻥ ﻴﺘﺒﻌﺎ ﺍﻟﺴﺭﻋﺔ‪.‬‬
‫ﻭﻟﺤﺴﺎﺏ ﺍﻟﻌﻭﺍﻤل ‪  , , ‬ﻨﻌﻭﺽ ﺍﻟﻌﻼﻗﺘﻴﻥ )‪ (8.14) ، (8.13‬ﻓﻲ‬
‫ﺍﻟﻌﻼﻗﺔ )‪ (8.11‬ﻓﻨﺠﺩ‪:‬‬
‫‪ 2 ( x  Vt ) 2  y' 2 z ' 2 c 2 ( t  x ) 2  x 2  y 2  z 2  c 2 t 2‬‬ ‫)‪(8.15‬‬
‫ﻭﺒﺎﻟﻤﻁﺎﺒﻘﺔ ﺒﻴﻥ ﺍﻟﻁﺭﻓﻴﻥ ﻨﺤﺼل ﻋﻠﻰ ﺍﻟﻤﻌﺎﺩﻻﺕ ﺍﻟﺜﻼﺙ ﺍﻟﺘﺎﻟﻴﺔ‪:‬‬
‫‪2  c2  2  1‬‬ ‫‪‬‬
‫‪‬‬
‫‪2 2‬‬ ‫‪2 2‬‬
‫‪ V  c   c ‬‬ ‫‪2‬‬
‫)‪(8.16‬‬
‫‪‬‬
‫‪ 2 V  c 2  0‬‬ ‫‪‬‬
‫ﻭﻤﻨﻬﺎ ﻨﻌﻴﻥ ﺍﻟﻤﺠﺎﻫﻴل ﺍﻟﺜﻼﺜﺔ ‪  , , ‬ﻓﻨﺠﺩ‪:‬‬
‫)‪(8.17‬‬
‫‪1‬‬ ‫‪V‬‬ ‫‪V‬‬
‫‪ ‬‬ ‫‪, ‬‬ ‫‪‬‬
‫‪V‬‬ ‫‪2‬‬ ‫‪c2‬‬ ‫‪V‬‬ ‫‪2‬‬
‫‪1‬‬ ‫‪c2 1 ‬‬
‫‪2‬‬
‫‪c‬‬ ‫‪c2‬‬

‫‪- 259 -‬‬

‫)‪Create PDF files without this message by purchasing novaPDF printer (http://www.novapdf.com‬‬
‫ﻭﺒﺎﻟﺘﺒﺩﻴل ﻓﻲ )‪ (8.13‬ﻭ )‪ (8.14‬ﻨﺤﺼل ﺃﺨﻴﺭ ﺍﹰ ﺒﻌﺩ ﻤﻼﺤﻅﺔ )‪ (8.12‬ﻋﻠﻰ‬
‫ﺍﻟﺘﺤﻭﻴﻼﺕ ﺍﻟﺘﺎﻟﻴﺔ‪:‬‬
‫‪V2‬‬
‫‪t‬‬ ‫‪x‬‬
‫‪x' ‬‬
‫‪x  Vt‬‬
‫‪, y'  y , z '  z , t' ‬‬ ‫‪c2‬‬ ‫) ‪(8.18,a‬‬
‫‪V2‬‬ ‫‪V2‬‬
‫‪1‬‬ ‫‪1‬‬
‫‪c2‬‬ ‫‪c2‬‬
‫ﻭﻫﻲ ﺍﻟﻤﺴﻤﺎﺓ ﺘﺤﻭﻴﻼﺕ ﻟﻭﺭﻨﺘﺯ ﻭﺍﻟﺘﻲ ﺘﻘﺎﺒل ﺘﺤﻭﻴﻼﺕ ﻏﺎﻟﻴﻠﻲ )‪ (8.1‬ﻓﻲ‬
‫ﺍﻟﻤﻴﻜﺎﻨﻴﻙ ﺍﻟﻜﻼﺴﻴﻜﻲ‪.‬‬
‫ﻭﺍﻀﺢ ﺃﻨﻪ ﻴﻤﻜﻥ ﺍﻻﻨﺘﻘﺎل ﻤﻥ '‪ K‬ﺇﻟﻰ ‪ K‬ﺇﺫﺍ ﻻﺤﻅﻨﺎ ﺃﻥ ‪ K‬ﺘﺘﺤﺭﻙ ﺒﺎﻟﻨﺴﺒﺔ‬
‫ﺇﻟﻰ '‪ K‬ﺒﺴﺭﻋﺔ ‪ -V‬ﻭﺒﺎﻟﺘﺎﻟﻲ ﺴﻴﻜﻭﻥ‪:‬‬
‫‪V‬‬
‫‪t '‬‬ ‫'‪x‬‬
‫‪x‬‬
‫' ‪x' Vt‬‬
‫‪, y  y' , z  z' , t ‬‬ ‫‪c2‬‬ ‫)‪(8.18,b‬‬
‫‪V2‬‬ ‫‪V2‬‬
‫‪1‬‬ ‫‪1‬‬
‫‪c2‬‬ ‫‪c2‬‬
‫ﺇﻥ ﺃﻫﻤﻴﺔ ﻫﺫﺍ ﺍﻟﺘﺤﻭﻴل ﺘﺄﺘﻲ ﺃﻭﻻﹰ ﻤﻥ ﺃﻨﻪ ﻤﺴﺘﻨﺘﺞ ﻓﻘﻁ ﻤﻥ ﻓﺭﻀﻴﺘﻲ ﺃﻨﺸﺘﺎﻴﻥ‬
‫ﺍﻷﺴﺎﺴﻴﺘﻴﻥ ﻭﺃﻥ ﺘﺤﻭﻴﻼﺕ ﻏﺎﻟﻴﻠﻲ )‪ (8.1‬ﻟﻴﺴﺕ ﺇﻻ ﺤﺎﻟﺔ ﺨﺎﺼﺔ ﻤﻨﻪ‪ ،‬ﻭﻫﺫﺍ ﻴﻌﻨﻲ‬
‫ﺃﻥ ﻓﺭﻀﻴﺘﻲ ﺃﻨﺸﺘﺎﻴﻥ ﻻﺘﻨﻔﻴﺎﻥ ﺃﺒﺩ ﺍﹰ ﺘﺤﻭﻴﻼﺕ ﻏﺎﻟﻴﻠﻲ ﻭﻟﻜﻨﻬﻤﺎ ﺘﺒﻴﻨﺎﻥ ﺃﻨﻬﺎ ﺼﺤﻴﺤﺔ‬
‫ﻓﻘﻁ ﻤﻥ ﺃﺠل ﺍﻟﺴﺭﻋﺔ ﺍﻟﺼﻐﻴﺭﺓ ﺠﺩﺍﹰ ﺒﺎﻟﻨﺴﺒﺔ ﻟﺴﺭﻋﺔ ﺍﻟﻀﻭﺀ ﺃﻱ ﻋﻨﺩﻤﺎ ‪V  c‬‬
‫ﺇﺫ ﻤﻥ ﺍﻟﺴﻬل ﺍﻟﺤﺼﻭل ﻋﻠﻰ ﺍﻟﺘﺤﻭﻴﻼﺕ )‪ (8.1‬ﺇﺫﺍ ﺃﻫﻤﻠﻨﺎ ﻓﻲ ﺍﻟﻌﻼﻗﺎﺕ )‪(8.18‬‬
‫ﻭﻓﻲ ﻫﺫﻩ ﺍﻟﺤﺎﻟﺔ ﻨﺠﺩ‪ x '  x  Vt , t '  t :‬ﺃﻱ ﺃﻥ ﺘﺤﻭﻴﻼﺕ ﻏﺎﻟﻴﻠﻲ ﻫﻲ‬ ‫ﺍﻟﻤﻘﺩﺍﺭ‬
‫‪V‬‬
‫‪c‬‬
‫ﺤﺎﻟﺔ ﺨﺎ ﺼﺔ ﻤﻥ ﺘﺤﻭﻴﻼﺕ ﻟﻭﺭﻨﺘﺯ ﺘﻨﺘﺞ ﻋﻨﺩﻤﺎ ‪. V  c‬‬
‫‪ -64‬ﻧﺘﺎﺋﺞ ﲢﻮﻳﻼﺕ ﻟﻮﺭﻧﺘﺰ‪ ،‬ﺍﻟﻔﻮﺍﺻﻞ ﺍﳌﻜﺎﻧﻴﺔ ﻭﺍﻟﺰﻣﺎﻧﻴﺔ ‪:‬‬
‫ﺇﻥ ﻨﺘﺎﺌﺞ ﺍﻟﻨﻅﺭﻴﺔ ﺍﻟﻨﺴﺒﻴﺔ ﺘﺘﻨﺎﻗﺽ ﺘﻨﺎﻗﻀﺎﹰ ﺠﺫﺭﻴﺎﹰ ﻤﻊ ﺍﻟﻔﻴﺯﻴﺎﺀ ﺍﻟﻜﻼﺴﻴﻜﻴﺔ‬
‫ﺤﻭل ﺨﻭﺍﺹ ﺍﻟﻔﺭﺍﻍ ﻭﺍﻟﺯﻤﻥ‪ ،‬ﻓﻠﻴﺱ ﻤﻔﻬﻭﻤﺎ ﺍﻟﺤﺠﻡ ﻭﺍﻟﺯﻤﻥ ﻤﻁﻠﻘﻴﻥ ﻭﻻﻴﺘﻤﺘﻌﺎﻥ‬
‫"ﺒﺎﻟﻘﺩﺍﺴﺔ" ﻜﻤﺎ ﻜﺎﻥ ﺫﻟﻙ ﻋﻨﺩ ﻨﻴﻭﺘﻥ ﻭﺇﻨﻤﺎ ﻴﺘﻐﻴﺭﺍﻥ ﺒﺤﺴﺏ ﺍﻟﺠﻤﻠﺔ ﺍﻟﻌﻁﺎﻟﻴﺔ ﺍﻟﺘﻲ‬
‫ﻴﻘﺎﺴﺎﻥ ﺒﻬﺎ‪.‬‬

‫‪- 260 -‬‬

‫)‪Create PDF files without this message by purchasing novaPDF printer (http://www.novapdf.com‬‬
‫ﻟﻨﺩﺭﺱ ﺃﻭﻻﹰ ﻤﻔﻬﻭﻡ ﻁﻭل ﺠﺴﻡ ﻓﻲ ﺠﻤﻠﺘﻴﻥ ﻋﻁﺎﻟﺘﻴﻥ‪ ،‬ﻭﻟﺫﻟﻙ ﻨﻔﺭﺽ ﺃﻥ‬
‫ﻁﻭل ﺍﻟﺠﺴﻡ ﻴﺤﺩﺩ ﺒﺎﻟﻔﺭﻕ ﺒﻴﻥ ﻓﺎﺼﻠﺘﻲ ﺒﺩﺍﻴﺘﻪ ﻭﻨﻬﺎﻴﺘﻪ ﻓﻲ ﺍﻟﺠﻤﻠﺔ ﺍﻟﻤﺘﺤﺭﻜﺔ ﺃﻱ‬
‫‪ x' 2  x'1‬ﻋﻠﻰ ﺃﻥ ﺘﺅﺨﺫ ﺍﻟﻔﺎﺼﻠﺘﺎﻥ ﻓﻲ ﻨﻔﺱ ﺍﻟﻠﺤﻅﺔ ﻭﻟﻨﺤﺴﺏ ﻫﺫﺍ ﺍﻟﻁﻭل ﻓﻲ‬
‫ﺍﻟﺠﻤﻠﺔ ‪ K‬ﺃﻱ ) ‪ (x 2  x1‬ﺒﻭﺍﺴﻁﺔ ﻋﻼﻗﺘﻲ ﺍﻟﺘﺤﻭﻴل ﺍﻟﺘﺎﻟﻴﺘﻴﻥ‪:‬‬
‫‪x 2  Vt‬‬ ‫‪x1  Vt‬‬
‫‪x' 2 ‬‬ ‫‪, x'1 ‬‬ ‫)‪(8.19‬‬
‫‪2‬‬ ‫‪2‬‬
‫‪V‬‬ ‫‪V‬‬
‫‪1‬‬ ‫‪1‬‬
‫‪2‬‬
‫‪c‬‬ ‫‪c2‬‬
‫ﺒﺎﻟﻁﺭﺡ ﻨﺠﺩ‪:‬‬
‫‪x 2  x1‬‬
‫‪x' 2  x'1 ‬‬ ‫)‪(8.20‬‬
‫‪2‬‬
‫‪V‬‬
‫‪1‬‬
‫‪c2‬‬
‫ﻓﺈﺫﺍ ﺭﻤﺯﻨﺎ ﻟﻠﻁﻭل ‪ x' 2  x'1‬ﺒﺎﻟﺭﻤﺯ ‪ L0‬ﻭﻟﻠﻁﻭل ‪ x 2  x1‬ﺒﺎﻟﺭﻤﺯ ‪L‬‬
‫ﻷﻤﻜﻨﻨﺎ ﻭﻀﻊ ﺍﻟﻌﻼﻗﺔ )‪ (8.20‬ﺍﻟﻤﻤﺜﻠﺔ ﻟﻁﻭل ﺍﻟﺠﺴﻡ ﻓﻲ ﺍﻟﺠﻤﻠﺔ ‪ K‬ﺒﺎﻟﺸﻜل ﺍﻟﺘﺎﻟﻲ‪:‬‬
‫‪V2‬‬
‫‪L  L0 1 ‬‬ ‫)‪(8.21‬‬
‫‪c2‬‬
‫ﻭﻫﻲ ﻻﺘﺴﺎﻭﻱ ﺍﻟﻁﻭل ‪ L0‬ﻓﻲ ﺍﻟﺠﻤﻠﺔ '‪ K‬ﺍﺫﻥ ﻓﺎﻟﻁﻭل ‪ L0‬ﻴﺘﻘﻠﺹ ﺒﺎﻟﻨﺴﺒﺔ‬
‫‪V2‬‬
‫‪ 1 : 1 ‬ﺇﺫﺍ ﻗﻴﺱ ﻤﻥ ﻗﺒل ﻤﺭﺍﻗﺏ ﻤﻭﺠﻭﺩ ﻓﻲ ‪ ،K‬ﻭﻟﻤﺎ ﻜﺎﻨﺕ ﺍﻟﺤﺭﻜﺔ ﻓﻲ‬
‫‪c2‬‬
‫ﺍﻻﺘﺠﺎﻩ ‪ ox‬ﻻﺘﺅﺜﺭ ﻋﻠﻰ ﺍﻟﺒﻌﺩﻴﻥ ‪ y‬ﻭ ‪ z‬ﻓﻴﻤﻜﻥ ﺘﻌﻤﻴﻡ ﺍﻟﻌﻼﻗﺔ )‪ (8.21‬ﻓﻲ ﺤﺎﻟﺔ‬
‫ﺍﻟﺤﺠﻡ ﻓﻨﺠﺩ ﺃﻨﻪ ﻁﺒﻘﺎﹰ ﻟﻠﻌﻼﻗﺔ‪:‬‬
‫‪V2‬‬
‫‪V  V0 1 ‬‬ ‫)‪(8.22‬‬
‫‪c2‬‬
‫ﺒﺎﻟﺭﻏﻡ ﻤﻥ ﺃﻥ ﺫﻟﻙ ﺍﻟﺤﺠﻡ ﻻﻴﻌﺎﻨﻲ ﻤﻥ ﺘﺄﺜﻴﺭ ﺃﻱ ﻗﻭﺓ ﺨﺎﺭﺠﻴﺔ ﻓﺈﻨﻪ ﺘﻘﻠﺹ‬
‫ﺒﻨﺴﺒﺔ ﻤﻌﻴﻨﺔ‪ .‬ﻭﻫﻜﺫﺍ ﻓﺈﻥ ﻗﻭﻟﻨﺎ ﺃﻥ ﺍﻟﻤﺴﺎﻓﺔ ﺒﻴﻥ ﻨﻘﻁﺘﻴﻥ ﻫﻲ ‪ L‬ﻟﻴﺱ ﻟﻪ ﻤﻌﻨﻰ ﺇﻻ ﺇﺫﺍ‬
‫ﻗﻠﻨﺎ ﻓﻲ ﺃﻴﺔ ﺠﻤﻠﺔ ﻨﻘﻴﺱ ﻫﺫﻩ ﺍﻟﻤﺴﺎﻓﺔ‪ ،‬ﻭﻫﺫﺍ ﻤﺎﻴﻨﺎﻗﺽ ﺍﻟﻔﻴﺯﻴﺎﺀ ﺍﻟﻜﻼﺴﻴﻜﻴﺔ ﺤﻴﺙ‬
‫ﻨﻌﻠﻡ ﺃﻥ ﺍﻟﺯﻤﻥ ﻭﺍﻟﻁﻭل ﻭﺒﺎﻟﺘﺎﻟﻲ ﺍﻟﻜﺘﻠﺔ ﻤﻔﺎﻫﻴﻡ ﻤﻁﻠﻘﺔ‪.‬‬

‫‪- 261 -‬‬

‫)‪Create PDF files without this message by purchasing novaPDF printer (http://www.novapdf.com‬‬
‫ﻭﻴﻤﻜﻥ ﺍﻵﻥ ﻋﻠﻰ ﻀﻭﺀ ﻤﺎﺴﺒﻕ ﺘﻌﻠﻴل ﺍﻟﻨﺘﺎﺌﺞ ﺍﻟﺴﻠﺒﻴﺔ ﻟﺘﺠﺭﺒﺔ ﻤﺎﻴﻜﻠﺴﻭﻥ‬
‫ﻭﻤﻭﺭﻟﻲ‪ .‬ﻤﻥ ﺍﻟﻤﻌﻠﻭﻡ ﺃﻨﻪ ﻜﺎﻨﺕ ﺍﻟﻐﺎﻴﺔ ﻤﻥ ﺍﻟﺘﺠﺭﺒﺔ ﻗﻴﺎﺱ ﺴﺭﻋﺔ ﺍﻷﺭﺽ ﺍﻻﻨﺘﻘﺎﻟﻴﺔ‬
‫ﺒﺎﻟﻨﺴﺒﺔ ﻟﻸﺜﻴﺭ‪ ،‬ﻓﺈﺫﺍ ﺠﻌﻠﻨﺎ ﻤﺴﺎﺭ ﺃﺤﺩ ﺍﻟﺸﻌﺎﻋﻴﻥ ﺍﻟﻀﻭﺌﻴﻴﻥ ﺒﺎﺘﺠﺎﻩ ﺤﺭﻜﺔ ﺍﻷﺭﺽ‬
‫ﻭﺠﻌﻠﻨﺎ ﻤﺴﺎﺭ ﺍﻵﺨﺭ ﻤﺘﻌﺎﻤﺩﺍﹰ ﻤﻊ ﻫﺫﺍ ﺍﻻﺘﺠﺎﻩ ﻓﺈﻥ ﺍﻟﻤﺴﺎﻓﺔ ﺍﻟﺘﻲ ﺴﻴﻘﻁﻌﻬﺎ ﺍﻟﺸﻌﺎﻉ‬
‫ﺍﻷﻭل ﻤﺎ ﺒﻴﻥ ‪ 0‬ﻭﺍﻟﻤﺭﺁﺓ ‪) M1‬ﺸﻜل ‪ (8.4‬ﻟﻥ ﺘﺴﺎﻭﻱ ‪ ‬ﻜﻤﺎ ﺤﺴﺒﻨﺎﻫﺎ ﺴﺎﺒﻘﺎﹰ ﻓﻲ‬
‫‪‬‬
‫ﺤﻴﺙ ‪V‬‬ ‫ﺠﻤﻠﺔ ﺍﻷﺜﻴﺭ ﻭﺇﻨﻤﺎ ﺘﻜﻭﻥ ﻗﺩ ﺘﻘﻠﺼﺕ ﺒﻔﻌل ﺍﻟﺤﺭﻜﺔ ﻭﺃﺼﺒﺤﺕ‬
‫‪V2‬‬
‫‪1‬‬
‫‪c2‬‬
‫ﺴﺭﻋﺔ ﺍﻷﺭﺽ ﺒﺎﻟﻨﺴﺒﺔ ﻟﻸﺜﻴﺭ‪ ،‬ﻭﺒﺎﻟﺘﺎﻟﻲ ﻓﺎﻟﺯﻤﻥ ﺍﻟﻼﺯﻡ ﻟﻘﻁﻊ ﻫﺫﻩ ﺍﻟﻤﺴﺎﻓﺔ ﺫﻫﺎﺒﺎﹰ‬
‫‪2‬‬
‫ﺃﻤﺎ ﺍﻟﺸﻌﺎﻉ ﺍﻟﻀﻭﺌﻲ ﺍﻟﺘﺎﻟﻲ ﺍﻟﺫﻱ ﺘﺤﺭﻙ ﺒﺎﺘﺠﺎﻩ ﻤﺘﻌﺎﻤﺩ ﻤﻊ‬ ‫ﻭﺍﻴﺎﺒﺎﹰ ﻫﻭ‬
‫‪V2‬‬
‫‪1‬‬
‫‪c2‬‬
‫ﺍﺘﺠﺎﻩ ﺤﺭﻜﺔ ﺍﻟﺸﻌﺎﻉ ﺍﻷﻭل ﻓﻴﻠﺯﻤﻪ ﻜﻤﺎ ﺭﺃﻴﻨﺎ ﻓﻲ )‪ (8.9‬ﺯﻤﻨﺎﹰ ‪ 2T2‬ﻭﻫﺫﺍ ﻴﺴﺎﻭﻱ‬
‫ﻭ ﻴﺒﺩﻭ ﺒﻭﻀﻭﺡ ﺃﻥ ﺍﻟﺯﻤﻨﻴﻥ ﻤﺴﺘﺎﻭﻴﺎﻥ ﺘﻤﺎﻤﺎﹰ ﻭﺒﺎﻟﺘﺎﻟﻲ ﻟﻥ ﻴﻅﻬﺭ ﺃﻱ‬ ‫‪2‬‬
‫‪V2‬‬
‫‪C 1‬‬
‫‪c2‬‬
‫ﻓﺭﻕ ﻓﻲ ﺍﻟﻤﺴﻴﺭ ﺒﻴﻨﻬﻤﺎ ﻭﻟﻥ ﻴﻅﻬﺭ ﻓﻲ ﺍﻟﺠﻬﺎﺯ ﺃﻱ ﺃﻫﺩﺍﺏ ﺘﺩﺍﺨل‪.‬‬
‫ﻭﻴﺠﺏ ﺍﻟﺘﺄﻜﻴﺩ ﻫﻨﺎ ﺃﻥ ﺘﻘﻠﺹ ﺍﻷﻁﻭﺍل ﻫﻭ ﻅﺎﻫﺭﺓ ﺤﺭﻜﻴﺔ ﺼﺭﻓﺔ ﻭﻟﻴﺱ‬
‫ﻤﻴﻜﺎﻨﻴﻜﻴﺔ‪ ،‬ﻭﻨﻌﻨﻲ ﺒﺫﻟﻙ ﺃﻥ ﺍﻟﺠﺴﻡ ﺍﻟﻤﺘﺤﺭﻙ ﺒﺴﺭﻋﺔ ﻜﺒﻴﺭﺓ ﻟﻥ ﻴﺨﻀﻊ ﻟﻘﻭﻯ ﻤﻌﻴﻨﺔ‬
‫)ﺍﺠﻬﺎﺩﺍﺕ( ﺘﺴﺒﺏ ﺘﺸﻭﻫﺎﹰ ﻓﻲ ﺸﻜﻠﻪ ﺇﺫ ﻻﻴﺴﺘﻁﻴﻊ ﺍﻟﺸﺨﺹ ﺍﻟﻤﺭﺍﻓﻕ ﻟﻬﺫﺍ ﺍﻟﺠﺴﻡ ﺃﻥ‬
‫ﻴﻼﺤﻅ ﻫﺫﻩ ﺍﻟﺘﻐﻴﺭﺍﺕ‪ .‬ﻭﺒﻬﺫﻩ ﺍﻟﻤﻨﺎﺴﺒﺔ ﻨﻘﻭل ﺃﻥ ﻤﻔﻬﻭﻡ ﺍﻟﺠﺴﻡ ﺍﻟﺼﻠﺏ )ﻭﻫﻭ ﺍﻟﺠﺴﻡ‬
‫ﺍﻟﺫﻱ ﻻﻴﺘﺸﻭﻩ ﺒﺤﻴﺙ ﺘﺒﻘﻰ ﺍﻟﻤﺴﺎﻓﺔ ﺒﻴﻥ ﺃﻱ ﻨﻘﻁﺘﻴﻥ ﻤﻨﻪ ﺜﺎﺒﺘﺔ( ﻴﻨﺎﻗﺽ ﺍﻟﻨﻅﺭﻴﺔ‬
‫ﺍﻟﻨﺴﺒﻴﺔ ﻭﻟﺒﺭﻫﺎﻥ ﺫﻟﻙ ﻨﻘﻭل ﺃﻨﻪ ﻟﻭ ﻭﺠﺩ ﻫﺫﺍ ﺍﻟﺠﺴﻡ ﻷﻤﻜﻨﻨﺎ ﺒﻭﺍﺴﻁﺘﻪ ﻨﻘل ﺍﻟﺘﺄﺜﻴﺭ‬
‫ﻤﻥ ﻤﻜﺎﻥ ﺇﻟﻰ ﺁﺨﺭ ﺒﺴﺭﻋﺔ ﻻﻨﻬﺎﺌﻴﺔ‪ ،‬ﺇﺫ ﺃﻥ ﺼﺩﻤﺔ ﺼﻐﻴﺭﺓ ﻓﻲ ﺍﺤﺩﻯ ﻨﻬﺎﻴﺘﻴﻪ ﻴﻤﻜﻥ‬
‫ﺃﻥ ﺘﻨﺘﻘل ﻟﺤﻅﻴﺎﹰ ﺇﻟﻰ ﺍﻟﻨﻬﺎﻴﺔ ﺍﻷﺨﺭﻯ ﻭﻫﺫﺍ ﻁﺒﻌﺎﹰ ﻴﺨﺎﻟﻑ ﺍﻟﻔﺭﻀﻴﺔ ﺍﻟﺜﺎﻨﻴﺔ‪.‬‬

‫‪- 262 -‬‬

‫)‪Create PDF files without this message by purchasing novaPDF printer (http://www.novapdf.com‬‬
‫ﻭﺒﻨﻔﺱ ﺍﻟﻁﺭﻴﻘﺔ ﻴﻤﻜﻥ ﺃﻥ ﻨﺒﺭﻫﻥ ﻅﺎﻫﺭﺓ ﺘﻤﺩﺩ ﺍﻷﺯﻤﻨﺔ‪ ،‬ﻭﻟﺫﻟﻙ ﻨﺄﺨﺫ ﺤﺎﺩﺜﺔ‬
‫ﺘﺤﺩﺙ ﻓﻲ ﺍﻟﻤﺠﺎل ﺍﻟﺯﻤﻨﻲ ‪ t 0  t ' 2  t '1‬ﻤﻘﺎﺴﺎﹰ ﻓﻲ ﺍﻟﺠﻤﻠﺔ '‪ K‬ﻭﻓﻲ ﻨﻘﻁﺔ ﻤﻌﻴﻨﺔ‪،‬‬
‫ﺤﻴﺙ ‪ t ' 2 , t '1‬ﺯﻤﻥ ﺍﺒﺘﺩﺍﺀ ﻭﺍﻨﺘﻬﺎﺀ ﺍﻟﺤﺎﺩﺜﺔ ﻭﺤﺴﺏ ﺍﻟﺘﺤﻭﻴﻼﺕ )‪ (8.18,b‬ﻴﻜﻭﻥ‪:‬‬
‫'‪Vx‬‬ ‫'‪Vx‬‬
‫‪t'2 ‬‬ ‫‪t '1 ‬‬
‫‪t2 ‬‬ ‫‪c2‬‬ ‫‪, t1 ‬‬ ‫‪c2‬‬ ‫)‪(8.24‬‬
‫‪2‬‬ ‫‪2‬‬
‫‪V‬‬ ‫‪V‬‬
‫‪1‬‬ ‫‪1‬‬
‫‪c2‬‬ ‫‪c2‬‬
‫ﻭﺍﻟﻤﺠﺎل ﺍﻟﺯﻤﻨﻲ ﺍﻟﻤﻘﺎﺒل ﻟﻠﺤﺎﺩﺜﺔ ﻓﻲ ﺍﻟﺠﻤﻠﺔ ‪ K‬ﻫﻭ ‪ t  t 2  t1‬ﺴﻴﺼﺒﺢ‪:‬‬
‫‪t 0‬‬
‫‪t  t 2  t1 ‬‬ ‫)‪(8.25‬‬
‫‪V2‬‬
‫‪1‬‬
‫‪c2‬‬
‫ﻓﺎﻟﺯﻤﻥ ﺍﻟﻤﻘﺎﺱ ﻓﻲ ﺍﻟﺠﻤﻠﺔ '‪ K‬ﻻﻴﺴﺎﻭﻱ ﺍﻟﺯﻤﻥ ﺍﻟﻤﻘﺎﺱ ﻓﻲ ﺍﻟﺠﻤﻠﺔ ‪K‬‬
‫ﻭﺒﺎﻟﺘﺎﻟﻲ ﻓﺈﻥ ﻤﻔﻬﻭﻡ ﺍﻟﺯﻤﻥ ﻨﺴﺒﻲ ﺃﻴﻀﺎﹰ‪ ،‬ﻭﻴﺠﺏ ﻋﻠﻴﻨﺎ ﺩﺍﺌﻤﺎﹰ ﺘﺤﺩﻴﺩ ﺍﻟﺠﻤﻠﺔ ﺍﻟﺘﻲ‬
‫ﻗﻴﺱ ﻓﻴﻬﺎ ﻫﺫﺍ ﺍﻟﺯﻤﻥ‪ ،‬ﺇﺫﺍ ﺃﺭﺩﻨﺎ ﺃﻥ ﻨﻜﻭﻥ ﻋﻠﻤﻴﻴﻥ ﻓﻲ ﻗﻴﺎﺴﺎﺘﻨﺎ‪ ،‬ﻭﻴﻤﻜﻥ ﺃﻴﻀﺎﹰ‬
‫ﺍﻟﺤﺼﻭل ﻋﻠﻰ ﻋﻼﻗﺔ ﻤﺸﺎﺒﻬﺔ ﻋﻨﺩ ﺍﻻﻨﺘﻘﺎل ﻤﻥ ‪ K‬ﺇﻟﻰ '‪ K‬ﻓﻨﺠﺩ ﻤﻥ )‪ (8.18,a‬ﺃﻥ‬
‫ﺍﻟﺯﻤﻥ ' ‪ t‬ﺍﻟﻤﻘﺎﺱ ﻓﻲ ﺍﻟﺠﻤﻠﺔ ‪ K‬ﻴﺴﺎﻭﻱ‪:‬‬
‫)‪(8.26‬‬
‫‪t‬‬
‫‪t ' ‬‬
‫‪2‬‬
‫‪V‬‬
‫‪1‬‬
‫‪c2‬‬
‫ﻭﻟﻘﺩ ﻅﻬﺭﺕ ﻋﺩﺓ ﺘﺠﺎﺭﺏ ﻟﻠﺘﺤﻘﻕ ﻤﻥ ﺼﺤﺔ ﺘﻤﺩﺩ ﺍﻷﺯﻤﻨﺔ‪ ،‬ﻨﺫﻜﺭ ﻤﻨﻬﺎ ﺘﻠﻙ‬
‫ﺍﻟﻤﺘﻌﻠﻘﺔ ﺒﺘﻔﻜﻙ ﺒﻌﺽ ﺍﻟﺠﺴﻴﻤﺎﺕ ﺍﻷﺴﺎﺴﻴﺔ ﻭﺒﺼﻭﺭﺓ ﺨﺎﺼﺔ ﺍﻟﻤﻴﺯﻭﻨﺎﺕ ‪‬؛ ﻓﻤﻥ‬
‫ﺍﻟﻤﻌﻠﻭﻡ ﺃﻥ ﺍﻟﻤﻴﺯﻭﻥ ﺍﻟﻤﻭﺠﺏ ‪  ‬ﻴﺘﻔﻜﻙ ﺇﻟﻰ ﺒﻭﺯﻴﺘﺭﻭﻥ )ﺍﻟﻜﺘﺭﻭﻥ ﻤﻭﺠﺏ(‬
‫ﻭﺠﺯﻴﺌﻲ ﻨﻴﺘﺭﻴﻨﻭ ﺃﻤﺎ ﺍﻟﻤﻴﺯﻭﻥ ﺍﻟﺴﺎﻟﺏ ‪  ‬ﻓﻴﺘﻔﻜﻙ ﺇﻟﻰ ﺍﻟﻜﺘﺭﻭﻥ ﻭﺠﺯﻴﺌﻲ ﻨﻴﺘﺭﻴﻨﻭ‬
‫ﺤﺴﺏ ﺍﻟﻤﻌﺎﺩﻟﺔ‪:‬‬
‫‪ ‬‬
‫‪ e   2‬‬

‫ﻭﻴﺘﻡ ﻫﺫﺍ ﺍﻟﺘﻔﻜﻙ ﻤﻬﻤﺎ ﻜﺎﻨﺕ ﺴﺭﻋﺔ ﺍﻟﻤﻴﺯﻭﻥ ﺍﻟﻤﺘﺤﺭﻙ‪ ،‬ﻓﺈﺫﺍ ﻋﻠﻤﻨﺎ ﺃﻥ‬
‫ﺴﺭﻋﺔ ﻫﺫﺍ ﺍﻟﻤﻴﺯﻭﻥ ﻗﺩ ﺘﺼل ﺇﻟﻰ ﻤﻘﺩﺍﺭ ﻗﺭﻴﺏ ﻤﻥ ﺴﺭﻋﺔ ﺍﻟﻀﻭﺀ ﻓﺈﻥ ﺍﻟﻔﺭﻕ ﺒﻴﻥ‬

‫‪- 263 -‬‬

‫)‪Create PDF files without this message by purchasing novaPDF printer (http://www.novapdf.com‬‬
‫ﺯﻤﻥ ﺤﻴﺎﺓ ﺍﻟﻤﻴﺯﻭﻥ ﺍﻟﺴﺭﻴﻊ ﻭﺍﻟﻤﻴﺯﻭﻥ ﺍﻟﺒﻁﻲﺀ ﺴﻴﻜﻭﻥ ﻤﻠﺤﻭﻅﺎﹰ‪ ،‬ﺍﻟﻌﻼﻗﺔ ﺍﻟﺯﻤﻨﻴﺔ‬
‫ﺴﺘﻜﻭﻥ ﻤﻥ ﺍﻟﺸﻜل‪:‬‬
‫‪s‬‬
‫‪K ‬‬
‫‪V2‬‬
‫‪1‬‬
‫‪c2‬‬
‫ﺤﻴﺙ ‪  k‬ﺯﻤﻥ ﺤﻴﺎﺓ ﺍﻟﻤﻴﺯﻭﻥ ﺍﻟﻤﺘﺤﺭﻙ ﺒﺴﺭﻋﺔ ‪ V‬ﻭ ‪  s‬ﺯﻤﻥ ﺤﻴﺎﺓ‬
‫ﺍﻟﻤﻴﺯﻭﻥ ﺍﻟﺴﺎﻜﻥ )ﺍﻟﺒﻁﻲﺀ( ﻭﻤﻥ ﺍﻟﻤﻌﻠﻭﻡ ﻤﻥ ﺍﻟﺘﺠﺎﺭﺏ ﺍﻟﻔﻴﺯﻴﺎﺌﻴﺔ ﺍﻟﺩﻗﻴﻘﺔ ﺃﻥ‬
‫‪  s  2  10 6 sec‬ﻓﺈﺫﺍ ﻜﺎﻥ ﺯﻤﻥ ﺤﻴﺎﺓ ﺍﻟﻤﻴﺯﻭﻥ ﻻﻴﺘﻌﻠﻕ ﺒﺴﺭﻋﺘﻪ ﻓﺈﻨﻪ ﻴﺠﺏ ﺃﻥ‬
‫ﻴﻘﻁﻊ ﻤﺴﺎﻓﺔ ‪ x‬ﺘﻌﻁﻰ ﺒﺎﻟﻌﻼﻗﺔ‪:‬‬
‫‪x  V   3  108  2  10 6  600 m‬‬ ‫) ‪(V  c‬‬
‫ﻏﻴﺭ ﺃﻥ ﺍﻟﺘﺠﺭﺒﺔ ﺘﺜﺒﺕ ﺃﻥ ﺍﻟﻤﻴﺯﻭﻨﺎﺕ ﺘﻘﻁﻊ ﺤﻭﺍﻟﻲ ‪ 20 km‬ﻗﺒل ﺃﻥ ﺘﺘﻔﻜﻙ‬
‫ﻭﺒﺎﻟﺘﺎﻟﻲ ﻓﻬﺫﺍ ﻴﻭﺍﻓﻕ ﺯﻤﻨﺎﹰ ﻗﺩﺭﻩ‪:‬‬
‫‪20  10 3‬‬
‫‪k ‬‬ ‫‪ 7  10  5 sec  50  s‬‬
‫‪8‬‬
‫‪3  10‬‬
‫ﻭﻫﺫﺍ ﺍﻟﺘﺼﺤﻴﺢ ﻴﺠﺏ ﺃﻥ ﻴﺅﺨﺫ ﺒﻌﻴﻥ ﺍﻻﻋﺘﺒﺎﺭ ﻭﻻﻴﻤﻜﻥ ﺍﻫﻤﺎﻟﻪ‪) ،‬ﻭﻴﺒﺩﻭ ﺃﻥ‬
‫ﺍﻟﻤﻴﺯﻭﻥ ﻴﺘﺤﺭﻙ ﺒﺴﺭﻋﺔ ﻫﺭﺒﺎﹰ ﻤﻥ ﺍﻟﻤﻭﺕ ﺍﻟﺫﻱ ﻴﻠﻲ ﺍﻟﺘﻔﻜﻙ(‪.‬‬
‫ﻭﻓﻲ ﻨﻬﺎﻴﺔ ﻫﺫﻩ ﺍﻟﻔﻘﺭﺓ ﻻﺒﺩ ﻟﻨﺎ ﺒﻐﻴﺔ ﺯﻴﺎﺩﺓ ﺍﻻﻴﻀﺎﺡ ﻤﻥ ﺍﻻﺠﺎﺒﺔ ﻋﻠﻰ‬
‫ﺍﻟﺴﺅﺍﻟﻴﻥ ﺍﻟﺘﺎﻟﻴﻴﻥ‪:‬‬
‫‪ -1‬ﻫل ﻜﺎﻨﺕ ﺍﻟﻨﺘﺎﺌﺞ ﺍﻟﺘﺠﺭﻴﺒﻴﺔ ﻤﺘﻭﺍﻓﻘﺔ ﻤﻊ ﺍﻟﻤﻔﺎﻫﻴﻡ ﺍﻟﻨﻴﻭﺘﻴﻨﻴﺔ ﺤﻭل ﺍﻟﻁﻭل‬
‫ﺍﻟﻤﻁﻠﻕ ﻟﻠﺠﺴﻡ ﻭﺍﻟﺯﻤﻥ ﺍﻟﻌﺎﻟﻤﻲ ﻗﺒل ﻅﻬﻭﺭ ﺍﻟﻨﻅﺭﻴﺔ ﺍﻟﻨﺴﺒﻴﺔ؟‬
‫‪ -2‬ﻫل ﺘﻘﻠﺹ ﺍﻷﻁﻭﺍل ﻭﺘﻤﺩﺩ ﺍﻷﺯﻤﻨﺔ ﺤﺎﺩﺜﺔ ﺤﻘﻴﻘﻴﺔ ﺃﻡ ﻅﺎﻫﺭﻴﺔ؟‬
‫ﺇﻥ ﺍﻟﺠﻭﺍﺏ ﻋﻠﻰ ﺍﻟﺴﺅﺍل ﺍﻷﻭل ﻭﺍﻀﺢ‪ ،‬ﻭﻫﻭ ﺃﻨﻪ ﻗﺒل ﺘﺠﺭﺒﺔ ﻤﺎﻴﻜﻠﺴﻭﻥ‬
‫ﻭﻤﺤﺎﻭﻟﺔ ﺘﺤﺩﻴﺩ ﺴﺭﻋﺔ ﺍﻷﺭﺽ ﺒﺎﻟﻨﺴﺒﺔ ﻟﻸﺜﻴﺭ ﺍﻟﺴﺎﻜﻥ ﻟﻡ ﻴﺘﻌﺎﻤل ﺍﻟﻌﻠﻤﺎﺀ ﻤﻊ‬
‫ﺼﻐﻴﺭﺓ ﺩﺍﺌﻤﺎﹰ‪ ،‬ﻭﻟﺫﻟﻙ ﻜﺎﻥ ﻤﻥ ﺍﻟﻤﻤﻜﻥ‬ ‫ﺴﺭﻋﺎﺕ ﻜﺒﻴﺭﺓ ﺒﺤﻴﺙ ﻜﺎﻨﺕ ﺍﻟﻨﺴﺒﺔ‬
‫‪V‬‬
‫‪c‬‬
‫ﺩﺍﺌﻤﺎﹰ ﺍﻟﻘﺒﻭل ﺒﺎﻟﻨﺘﺎﺌﺞ ﺍﻟﻜﻼﺴﻴﻜﻴﺔ‪ ،‬ﺃﻤﺎ ﺍﻵﻥ ﻭﺒﻌﺩ ﺘﻁﻭﺭ ﺍﻟﻌﻠﻭﻡ ﺍﻟﻔﻴﺯﻴﺎﺌﻴﺔ ﻓﻘﺩ‬
‫ﺃﺼﺒﺤﺕ ﺍﻟﻨﻅﺭﻴﺔ ﺍﻟﻨﺴﺒﻴﺔ ﺃﺤﺩ ﺃﺭﻜﺎﻥ ﺍﻟﻔﻴﺯﻴﺎﺀ ﺍﻟﺤﺩﻴﺜﺔ ﺍﻟﺘﻲ ﻻﺒﺩ ﻓﻴﻬﺎ ﻟﺘﻔﺴﻴﺭ ﻜﺜﻴﺭ‬

‫‪- 264 -‬‬

‫)‪Create PDF files without this message by purchasing novaPDF printer (http://www.novapdf.com‬‬
‫ﻤﻥ ﺍﻟﻅﻭﺍﻫﺭ ﻓﻲ ﻫﺫﻩ ﺍﻟﻔﻴﺯﻴﺎﺀ‪ ،‬ﻜﻤﺎ ﺃﻥ ﻜﺜﻴﺭ ﺍﹰ ﻤﻥ ﺍﻟﺘﺠﺎﺭﺏ ﺍﻟﻌﻠﻤﻴﺔ ﺍﻟﺘﻲ ﻻﻴﺘﺴﻊ‬
‫ﺍﻟﻤﺠﺎل ﻟﺫﻜﺭﻫﺎ ﻫﻨﺎ ﺒﺭﻫﻨﺕ ﻋﻠﻰ ﺼﺤﺔ ﻨﻅﺭﻴﺔ ﺃﻨﺸﺘﺎﻴﻥ‪.‬‬
‫ﻭﻟﻺﺠﺎﺒﺔ ﻋﻠﻰ ﺍﻟﺴﺅﺍل ﺍﻟﺜﺎﻨﻲ ﻨﻘﻭل ﺃﻨﻪ ﻜﺜﻴﺭ ﺍﹰ ﻤﺎ ﻨﺴﺘﻌﻤل ﻜﻠﻤﺔ ﻅﺎﻫﺭﻱ‬
‫ﻟﻠﺩﺍﻟﺔ ﻋﻠﻰ ﺃﻥ ﺘﻘﻠﺹ ﺍﻷﻁﻭﺍل ﻭﺘﻤﺩﺩ ﺍﻷﺯﻤﻨﺔ ﻫﻭ ﻅﺎﻫﺭﺓ ﺤﺭﻜﻴﺔ ﻭﻟﻴﺱ ﻤﻴﻜﺎﻨﻴﻜﻴﺔ‪.‬‬
‫ﻗﺩ ﻴﻜﻭﻥ ﻤﻥ ﺍﻟﻤﻔﻴﺩ ﺍﻟﺘﻨﻭﻴﻪ ﺃﻥ ﺍﻟﺘﺄﺜﻴﺭﺍ ﺕ ﺍﻟﺴﺎﺒﻘﺔ ﻫﻲ ﺤﻘﻴﻘﻴﺔ ﺒﻤﻌﻨﻰ ﺃﻨﻬﺎ ﺘﺤﺩﺙ‬
‫ﻓﻌﻼﹰ ﻭﻟﻴﺱ ﻤﺭﺘﺒﻁﺔ ﺒﺄﻱ ﻭﻫﻡ ﺃﻭ ﺨﻴﺎل ﺍﻟﻤﺭﺍﻗﺏ‪ ،‬ﻓﻬﻲ ﻟﻴﺴﺕ )ﺴﺭﺍﺒﺎﹰ( ﻭﻟﻜﻥ‬
‫ﻻﻴﺴﺘﻁﻴﻊ ﻤﺭﺍﻗﺒﺘﻬﺎ ﺇﻻ ﺍﻟﺸﺨﺹ ﺍﻟﺜﺎﺒﺕ ﺒﺎﻟﻨﺴﺒﺔ ﻟﻬﺎ‪ ،‬ﻭﺼﻌﻭﺒﺔ ﻓﻬﻡ ﺫﻟﻙ ﺘﺄﺘﻲ ﻤﻥ‬
‫ﺃﻨﻨﺎ ﺘﻌﻭﺩﻨﺎ ﻋﻠﻰ ﻁﻭل ﺜﺎﺒﺕ ﻟﻸﺸﻴﺎﺀ ﻭﺯﻤﻥ ﻋﺎﻟﻤﻲ ﻴﺴﻴﺭ ﺒﺎﻨﺘﻅﺎﻡ‪ ،‬ﺒﻴﻨﻤﺎ ﻫﻲ ﻓﻲ‬
‫ﺍﻟﺤﻘﻴﻘﺔ ﻏﻴﺭ ﺫ ﻟﻙ‪ ،‬ﺇﺫ ﺃﻥ ﺍﻷﻁﻭﺍل ﻭﺍﻷﺯﻤﻨﺔ ﺘﺘﻐﻴﺭ ﺒﻌﻀﻬﺎ ﺒﺎﻟﻨﺴﺒﺔ ﻟﻠﺒﻌﺽ ﺍﻵﺨﺭ‬
‫ﻭﻻﻨﺴﺘﻁﻴﻊ ﺃﻥ ﻨﺅﻜﺩ ﺃﻱ ﻤﻨﻬﺎ ﺍﻟﻁﻭل ﺍﻟﺤﻘﻴﻘﻲ ﺍﻷﺼﻠﻲ ﺍﻟﺫﻱ ﻟﻡ ﻴﺘﻐﻴﺭ ﻷﻥ ﺫﻟﻙ‬
‫ﻤﺸﺎﺒﻪ ﻟﻠﺴﺅﺍل‪ ،‬ﺃﻱ ﻤﻥ ﺍﻟﺠﻤﻠﺘﻴﻥ ﺜﺎﺒﺘﺔ ﻭﺃﻱ ﻤﻨﻬﺎ ﻤﺘﺤﺭﻜﺔ؟ ﻭﻫﻜﺫﺍ ﻓﺈﻥ ﻤﻔﻬﻭﻤﻲ‬
‫ﺍﻟﻁﻭل ﻭﺍﻟﺯﻤﻥ ﻨﺴﺒﻴﻴﻥ ﺘﻤﺎﻤﺎﹰ ﻜﻤﻔﻬﻭﻤﻲ ﺍﻟﺤﺭﻜﺔ ﻭﺍﻟﺴﻜﻭﻥ‪.‬‬
‫ﻭﻟﺯﻴﺎﺩﺓ ﺍﻻﻴﻀﺎﺡ ﻨﻭﺭﺩ ﺍﻟﻤﺜﺎل ﺍﻟﺘﺎﻟﻲ‪:‬‬
‫ﻟﺘﻜﻥ ﺍﻟﻜﺭﺘﺎﻥ ﺍﻟﻤﺸﺤﻭﻨﺘﺎﻥ '‪ A‬ﻭ ‪ A‬ﺍﺤﺩﺍﻫﻤﺎ ﻤﺭﺘﺒﻁﺔ ﻤﻊ ﺍﻟﺠﻤﻠﺔ ‪ ،K‬ﻭﺍﻟﺜﺎﻨﻴﺔ‬
‫ﻤﻊ '‪ ،K‬ﻤﻥ ﺍﻟﻭﺍﻀﺢ ﺃﻥ ﺍﻟﻜﺭﺓ '‪ A‬ﺴﺘﺘﺤﻭل ﻤﻥ ﻭﺠﻬﺔ ﻤﺭﺍﻗﺏ ﻤﻭﺠﻭﺩ ﻓﻲ ﺍﻟﺠﻤﻠﺔ‬
‫‪ ،K‬ﺇﻟﻰ ﻤﺠﺴﻡ ﻗﻁﻊ ﻨﺎﻗﺹ ﻷﻨﻬﺎ ﺘﺘﻘﻠﺹ ﺒﺎﺘﺠﺎﻩ ﺍﻟﺤﺭﻜﺔ ﻭﺒﺎﻟﺘﺎﻟﻲ ﻓﺈﻥ ﺍﻟﺘﺄﺜﻴﺭ‬
‫ﺍﻟﻤﺘﺒﺎﺩل ﺒﻴﻥ ﺍﻟﻜﺭﺘﻴﻥ ﺴﺘﺤﺴﺏ ﻜﻤﺎ ﻟﻭ ﻜﺎﻥ ﺒﻴﻥ ﻤﺠﺴﻡ ﻗﻁﻊ ﻨﺎﻗﺹ '‪ A‬ﻭﻜﺭﺓ ‪.A‬‬
‫ﻭﺒﺎﻟﻌﻜﺱ ﻓﺈﻥ ﻤﺭﺍﻗﺒﺎﹰ ﻓﻲ ﺍﻟﺠﻤﻠﺔ '‪ K‬ﺴﻴﻼﺤﻅ ﺃﻥ ‪ A‬ﺘﺤﻭﻟﺕ ﺇﻟﻰ ﻤﺠﺴﻡ ﻗﻁﻊ ﻨﺎﻗﺹ‬
‫ﻭ '‪ ،A‬ﺍﻟﻤﺭﺘﺒﻁﺔ ﻤﻌﻪ‪ ،‬ﺒﻘﻴﺕ ﻜﻤﺎ ﻫﻲ ﻭﺒﺎﻟﺘﺎﻟﻲ ﻴﻤﻜﻥ ﺤﺴﺎﺏ ﺍﻟﺘﺄﺜﻴﺭ ﺍﻟﻤﺘﺒﺎﺩل ﻤﻥ‬
‫ﻭﺠﻬﺔ ﻨﻅﺭﻩ ﺃﻴﻀﺎﹰ ﻜﻤﺎ ﻓﻲ ﺍﻟﺤﺎﻟﺔ ﺍﻷﻭﻟﻰ ﻭﻟﻜﻥ ﺒﺘﺒﺩﻴل ﻤﻜﺎﻨﻲ ﺍﻟﺠﺴﻤﻴﻥ )ﻭﻜﻤﺎ‬
‫ﺘﺭﺍﻨﻲ ﻴﺎﺠﻤﻴل ﺃﺭﺍﻙ(‪ ،‬ﻭﻫﻜﺫﺍ ﺘﺒﻘﻰ ﺍﻟﺤﺎﺩﺜﺔ ﺍﻟﻔﻴﺯﻴﺎﺌﻴﺔ ﻭﺍﺤﺩﺓ ﻓﻲ ﺍﻟﺤﺎﻟﺘﻴﻥ‪.‬‬
‫‪ -65‬ﻗﺎﻧﻮﻥ ﲨﻊ ﺍﻟﺴﺮﻉ‪ ،‬ﲢﻮﻳﻼﺕ ﺍﻟﺰﻭﺍﻳﺎ ‪:‬‬
‫ﻴﺴﻬل ﺍﻴﺠﺎﺩ ﻗﺎﻨﻭﻥ ﺠﻤﻊ ﺍﻟﺴﺭﻉ ﺇﺫﺍ ﻁﺒﻘﻨﺎ ﺍﻟﻌﻼﻗﺎﺕ )‪ (8.18,b‬ﻤﻥ ﺃﺠل‬
‫ﻋﻨﺼﺭ ﺘﻔﺎﻀﻠﻲ ‪ dx‬ﺤﻴﺙ ﻨﺠﺩ‪:‬‬

‫‪- 265 -‬‬

‫)‪Create PDF files without this message by purchasing novaPDF printer (http://www.novapdf.com‬‬
dx'V dt'
dx  , dy  dy'
V2
1
c2 (8.27)
V
dt ' dx'
dz  dz' , dt  c2
c2
‫ ﻓﺈﻥ‬K' ‫ ﻫﻲ ﺍﺤﺩﺍﺜﻴﺎﺕ ﺍﻟﻨﻘﻁﺔ ﺍﻟﻤﺎﺩﻴﺔ ﻓﻲ ﺍﻟﺠﻤﻠﺔ‬x' ‫ ﻭ‬y' ‫ ﻭ‬z' ‫ﻭﺇﺫﺍ ﻜﺎﻨﺕ‬
:‫ﻤﺭﻜﺒﺎﺕ ﺴﺭﻋﺘﻬﺎ ﻋﻠﻰ ﺍﻟﻤﺤﺎﻭﺭ ﺍﻻﺤﺩﺍﺜﻴﺔ ﺴﺘﻜﻭﻥ‬
dx' dy' dz'
u' x  , u' y  , u' z  (8.28)
dt ' dt ' dt'
:‫ ﺴﻴﻜﻭﻥ‬K ‫ﻭﻓﻲ ﺍﻟﺠﻤﻠﺔ‬
ux 
dx
, uy 
dy
, uz 
dz
(8.29)
dt dt dt
:‫ ﻓﻨﺠﺩ‬dt ‫( ﻋﻠﻰ ﺍﻟﺯﻤﻥ‬8.27) ‫ﻨﻘﺴﻡ ﻁﺭﻓﻲ ﺍﻟﻤﻌﺎﺩﻻﺕ‬

dx u'x V 
ux   
dt u' 
1 V x 
c2 
V2 
u' y 1  
c2 
(8.30)
dy
uy   
dt u' x 
1 V
c2 

V2 
u' z 1  
dz c2 
uz  
dt u'x 
1 V 
c2 
‫ ﻭﻨﺠﺩ‬،‫ﻭﻫﺫﻩ ﺍﻟﻤﻌﺎﺩﻻﺕ ﺘﻌﺒﺭ ﻋﻥ ﻗﺎﻨﻭﻥ ﺠﻤﻊ ﺍﻟﺴﺭﻉ ﻓﻲ ﺍﻟﻨﻅﺭﻴﺔ ﺍﻟﻨﺴﺒﻴﺔ‬
:‫ ﺃﻥ‬V  c ‫ﺒﺴﻬﻭﻟﺔ ﻋﻨﺩﻤﺎ‬
u x  u' x  V , u y  u' y , u z  u' z

- 266 -

Create PDF files without this message by purchasing novaPDF printer (http://www.novapdf.com)
‫ﻭﻫﻭ ﻗﺎﻨﻭﻥ ﺘﺭﻜﻴﺏ ﺍﻟﺴﺭﻉ ﻓﻲ ﺍﻟﻤﻴﻜﺎﻨﻴﻙ ﺍﻟﺘﻘﻠﻴﺩﻱ‪ ،‬ﻭﻻﻴﺠﺎﺩ ﻗﺎﻨﻭﻥ ﺘﻐﻴﻴﺭ‬
‫ﺍﻟﺯﻭﺍﻴﺎ ﻨﺄﺨﺫ ﺍﻟﺯﺍﻭﻴﺔ ‪ ‬ﺍﻟﺘﻲ ﻅﻠﻬﺎ ﻴﻌﻁﻰ ﺒﺎﻟﻌﻼﻗﺔ‪:‬‬
‫‪uy‬‬
‫‪tg  ‬‬ ‫)‪(8.31‬‬
‫‪ux‬‬
‫ﻭﻟﻨﺤﺴﺏ ﻗﻴﻤﺔ ﺍﻟﺯﺍﻭﻴﺔ ﺍﻟﻤﻘﺎﺒﻠﺔ ﻟﻬﺎ ﻓﻲ ﺍﻟﺠﻤﻠﺔ '‪ K‬ﻭﻟﺫﻟﻙ ﻨﺒﺩل ‪ u y‬ﻭ ‪u x‬‬

‫ﺒﻘﻴﻤﺘﻴﻬﻤﺎ ﻤﻥ )‪ (8.30‬ﻓﻨﺠﺩ ﺃﺨﻴﺭﺍﹰ‪:‬‬


‫‪V2‬‬
‫‪u' 1 ‬‬
‫‪c2‬‬
‫‪tg  ‬‬ ‫'‪sin ‬‬ ‫)‪(8.32‬‬
‫‪V  u ' cos ‬‬
‫ﺤﻴﺙ ﻭﻀﻌﻨﺎ‪:‬‬
‫'‪u' x  u' cos ' , u' y  u' sin ‬‬

‫ﻭﺘﺩل ﺍﻟﻌﻼﻗﺔ )‪ (8.32‬ﻋﻠﻰ ﺃﻥ ﻤﻔﻬﻭﻡ ﺍﻟﺯﺍﻭﻴﺔ ﻨﺴﺒﻲ ﺃﻴﻀﺎﹰ‪ ،‬ﻭﻴﺠﺏ ﺩﺍﺌﻤ ﺎﹰ‬
‫ﻤﻌﺭﻓﺔ ﻓﻲ ﺃﻱ ﺠﻤﻠﺔ ﻗﻴﺴﺕ ﻫﺫﻩ ﺍﻟﺯﺍﻭﻴﺔ؟‬
‫‪ -66‬ﺗﻮﺍﻗﺖ ﺣﺎﺩﺛﺘﲔ ‪:‬‬
‫ﻟﻨﻔﺭﺽ ﺍﻵﻥ ﺤﺩﻭﺙ ﺤﺎﺩﺜﺘﻴﻥ ﻓﻲ ﻨﻔﺱ ﺍﻟﻭﻗﺕ '‪ t‬ﻓﻲ ﺍﻟﺠﻤﻠﺔ '‪ K‬ﺍﻟﻤﺘﺤﺭﻜ ﺔ‬
‫ﺒﺎﻟﻨﺴﺒﺔ ‪ :K‬ﺍﻷﻭﻟﻰ ﻭﻗﻌﺕ ﻓﻲ ﺍﻟﻨﻘﻁﺔ ﺍﻟﺘﻲ ﻓﺎﺼﻠﺘﻬﺎ ‪ x'1‬ﻭﺍﻟﺜﺎﻨﻴﺔ ﻓﻲ ﺍﻟﻨﻘﻁﺔ ﺍﻟﺘﻲ‬
‫ﻓﺎﺼﻠﺘﻬﺎ ‪ x' 2‬ﻭﻟﻨﺒﺤﺙ ﻓﻴﻤﺎ ﺇﺫﺍ ﻜﺎﻨﺘﺎ ﻤﺘﻭﺍﻗﺘﺘﻴﻥ ﻓﻲ ﺍﻟﺠﻤﻠﺔ ‪ .K‬ﻭﻏﻨﻲ ﻋﻥ ﺍﻟﺒﻴﺎﻥ ﺃﻥ‬
‫ﺍﻟﺤﺎﺩﺜﺘﻴﻥ ﺍﻟﻤﺘﻭﺍﻗﺘﺘﻴﻥ ﻓﻲ ﺠﻤﻠﺔ ﻤﺎ ﻓﻲ ﺍﻟﻤﻴﻜﺎﻨﻴﻙ ﺍﻟﻜﻼﺴﻴﻜﻲ ﺴﺘﺒﻘﻴﺎﻥ ﻤﺘﻭﺍﻗﺘﺘﻴﻥ ﻓﻲ‬
‫ﻜل ﺍﻟﺠﻤل ﺍﻷﺨﺭﻯ‪ ،‬ﻭﻟﻜﻥ ﺍﻷﻤﺭ ﻤﺨﺘﻠﻑ ﻓﻲ ﺍﻟﻨﻅﺭﻴﺔ ﺍﻟﻨﺴﺒﻴﺔ‪ ،‬ﻭﻟﺒﺭﻫﺎﻥ ﺫﻟﻙ‬
‫ﻨﻔﺭﺽ ﺃﻥ ‪ t 1‬ﻫﻭ ﺯﻤﻥ ﻭﻗﻭﻉ ﺍﻟﺤﺎﺩﺜﺔ ﺍﻷﻭﻟﻰ ﻓﻲ ﺍﻟﺠﻤﻠﺔ ‪ K‬ﻭﺍﻟﺫﻱ ﻴﻌﻁﻰ‬
‫ﺒﺎﻟﻌﻼﻗﺔ‪:‬‬
‫'‪x‬‬
‫‪t ' V 1‬‬
‫‪t1 ‬‬ ‫‪c2‬‬
‫‪V2‬‬
‫‪1‬‬
‫‪c2‬‬
‫ﺃﻤﺎ ﺍﻟﺤﺎﺩﺜﺔ ﺍﻟﺜﺎﻨﻴﺔ ﻓﺘﺤﺩﺙ ﻓﻲ ﺍﻟﺯﻤﻥ ‪ t 2‬ﺤﻴﺙ‪:‬‬

‫‪- 267 -‬‬

‫)‪Create PDF files without this message by purchasing novaPDF printer (http://www.novapdf.com‬‬
‫‪V‬‬
‫‪t' ‬‬ ‫‪x' 2‬‬
‫‪t2 ‬‬ ‫‪c2‬‬
‫‪V2‬‬
‫‪1‬‬
‫‪c2‬‬
‫ﻭﺍﻟﻔﺭﻕ ﺍﻟﺯﻤﻨﻲ ﺒﻴﻥ ﺤﺩﻭﺜﻬﻤﺎ ﻫﻭ‪:‬‬
‫‪V x' 2 x '1‬‬
‫‪t  t 2  t1 ‬‬ ‫)‪(8.33‬‬
‫‪c2‬‬ ‫‪V2‬‬
‫‪1‬‬
‫‪c2‬‬
‫ﻭﻫﻭ ﻻﻴﺴﺎﻭﻱ ﺍﻟﺼﻔﺭ‪ ،‬ﻫﺫﺍ ﺒﺎﻻﻀﺎﻓﺔ ﺇﻟﻰ ﺃﻨﻪ ﻴﺘﺒﻊ ﺍﺸﺎﺭﺓ ﺍﻟﻤﻘﺩﺍﺭ‬
‫) ‪ (x ' 2  x'1‬ﻓﻘﺩ ﻴﻜﻭﻥ ﻤﻭﺠﺒﺎﹰ ﺃﻭ ﺴﺎﻟﺒﺎﹰ ﻭﺒﺎﻟﺘﺎﻟﻲ ﻓﻘﺩ ﺘﺤﺩﺙ ﺍﻟﺤﺎﺩﺜﺔ ﺍﻟﺜﺎﻨﻴﺔ ﻗﺒل‬
‫ﺍﻷﻭﻟﻰ ﻓﻲ ﺍﻟﺠﻤﻠﺔ ‪ .K‬ﻭﻓﻲ ﺍﻟﺤﺎﻟﺔ ﺍﻟﺨﺎﺼﺔ ﻋﻨﺩﻤﺎ ﺘﻘﻊ ﺍﻟﺤﺎﺩﺜﺘﺎﻥ ﻓﻲ ﺍﻟﺠﻤﻠﺔ '‪ K‬ﻓﻲ‬
‫ﺍﻟﻨﻘﻁﺔ ﻨﻔﺴﻬﺎ ﺃﻱ‪ x '1  x' 2 :‬ﻓﺈﻥ ‪ t  0‬ﻤﻬﻤﺎ ﻜﺎﻨﺕ ‪ V‬ﻭﺘﻜﻭﻥ ﺍﻟﺤﺎﺩﺜﺘﺎﻥ‬
‫ﻤﺘﻭﺍﻗﺘﺘﻴﻥ ﻓﻲ ﻜل ﺍﻟﺠﻤل ﺍﻟﻌﻁﺎﻟﻴﺔ ﻋﻠﻰ ﺍﻻﻁﻼﻕ‪.‬‬
‫‪ -67‬ﺍﻟﻘﻴﻢ ﺍﳌﻄﻠﻘﺔ ﰲ ﺍﻟﻨﻈﺮﻳﺔ ﺍﻟﻨﺴﺒﻴﺔ –ﺍ‪‬ﺎﻝ‪ -‬ﺍﻟﺰﻣﻦ ﺍﳋﺎﺹ ‪:‬‬
‫ﻗﺩ ﺘﻜﻭﻥ ﺍﻟﻨﻅﺭﻴﺔ ﺍﻟﻨﺴﺒﻴﺔ ﻤﺴﺅﻭﻟﺔ ﻋﻥ ﺘﺸﻭﻴﺵ ﺃﻓﻜﺎﺭﻨﺎ ﺍﻟﻌﻠﻤﻴﺔ‪ ،‬ﻓﺎﻟﻁﻭل‬
‫ﻨﺴﺒﻲ ﻭ ﺍﻟﺯﻤﻥ ﻨﺴﺒﻲ ﻭﺍﻟﺘﻭﺍﻗﺕ ﻨﺴﺒﻲ ﻭﺍﻟﺯﺍﻭﻴﺔ ﻨﻔﺴﻬﺎ ﻨﺴﺒﻴﺔ‪ ،‬ﻓﻤﺎ ﻫﻭ ﺍﻟﻤﻁﻠﻕ ﺍﺫﻥ‬
‫ﻓﻲ ﻫﺫﺍ ﺍﻟﻜﻭﻥ؟‬
‫ﻟﻌل ﻓﻲ ﻫﺫﺍ ﺍﻟﻘﻭل ﺍﻟﺴﺎﺒﻕ ﺒﻌﺽ ﺍﻟﺘﺠﻨﻲ ﻋﻠﻰ ﻫﺫﻩ ﺍﻟﻨﻅﺭﻴﺔ ﺍﻟﺭﺍﺌﻌﺔ ﺍﻟﺘﻲ‪،‬‬
‫ﻋﻠﻰ ﺍﻟﻌﻜﺱ‪ ،‬ﺤﺎﻭﻟﺕ ﺃﻥ ﺘﺤﺭﺭﻨﺎ ﻤﻥ ﻗﻴﻭﺩ ﺍﻟﺯﻤﺎﻥ ﻭﺍﻟﻤﻜﺎﻥ‪ ،‬ﻭﺘﺒﻴﻥ ﻟﻨﺎ ﻤﺎﻨﺤﻥ ﻓﻴﻪ‬
‫ﻤﻥ ﺨﻁﺄ‪ ،‬ﻟﺘﺴﺎﻋﺩﻨﺎ ﻋﻠﻰ ﺍﻟﺒﺤﺙ ﻋﻥ ﺍﻟﻤﻔﺎﻫﻴﻡ ﺍﻟﻤﻁﻠﻘﺔ ﺍﻟﺘﻲ ﻭﺠﺩ ﺍﻟﻜﺜﻴﺭ ﻤﻨﻬﺎ‬
‫ﺍﻨﻁﻼﻗﺎﹰ ﻤﻥ ﺍﻟﻨﻅﺭﻴﺔ ﺍﻟﻨﺴﺒﻴﺔ‪ ،‬ﻭﺍﻟﺘﻲ ﺘﺴﻤﻰ ﺃﺤﻴﺎﻨﺎﹰ ﺒﻨﻅﺭﻴﺔ ﺍﻟﻼﻤﺘﻐﻴﺭﺍﺕ ﺍﻟﻔﻴﺯﻴﺎﺌﻴﺔ‪،‬‬
‫ﻭﻟﻨﺒﺤﺙ ﺍﻵﻥ ﻓﻲ ﺒﻌﺽ ﻫﺫﻩ ﺍﻟﻼﻤﺘﻐﻴﺭﺍﺕ‪.‬‬
‫ﻟﺘﻜﻥ ﺍﻟﺤﺎﺩﺜﺔ ﻓﻲ ﺍﻟﺯﻤﻥ ‪ t‬ﻭﻓﻲ ﺍﻟﻨﻘﻁﺔ ) ‪ ( x, y, z‬ﻭﻓﻲ ﻟﺤﻅﺔ ﺃﺨﺭﻯ ‪t 1‬‬
‫ﻭﻨﻘﻁﺔ ﺃﺨﺭﻯ ) ‪ (x 1, y1, z1‬ﺘﺤﺩﺙ ﺤﺎﺩﺜﺔ ﺜﺎﻨﻴﺔ ﻓﻲ ﺍﻟﻠﺤﻅﺔ ﻨﻔﺴﻬﺎ ﻭﻋﻨﺩﺌﺫ‪ ‬ﻨﻌﺭﻑ‬
‫ﺍﻟﻤﺠﺎل ﺒﻴﻥ ﺍﻟﺤﺎﺩﺜﺘﻴﻥ ﺒﺎﻟﻌﻼﻗﺔ‪:‬‬
‫‪S  c 2 ( t1  t )2  ( x1  x )2  ( y1  y) 2  (z1  z )2‬‬ ‫)‪(8.34‬‬

‫‪- 268 -‬‬

‫)‪Create PDF files without this message by purchasing novaPDF printer (http://www.novapdf.com‬‬
‫ﻨﻘﻭل ﺃﻥ ﻫﺫﺍ ﺍﻟﻤﺠﺎل ﻻﻤﺘﻐﻴﺭ ‪ Invariant‬ﺒﻤﻌﻨﻰ ﺃﻨﻪ ﻻﻴﺘﻐﻴﺭ ﻋﻨﺩ ﺍﻻﻨﺘﻘﺎل‬
‫ﻤﻥ ﺠﻤﻠﺔ ﺇﻟﻰ ﺃﺨﺭﻯ‪ ،‬ﻭﻴﻤﻜﻥ ﺍﻟﺘﺄﻜﺩ ﻤﻥ ﺫﻟﻙ ﺒﻭﺍﺴﻁﺔ ﺘﺤﻭﻴﻼﺕ ﻟﻭﺭﻨﺘﺯ‪ ،‬ﻟﻨﺄﺨﺫ‬
‫ﺍﻟﻤﺠﺎل ﻨﻔﺴﻪ ﻓﻲ ﺍﻟﺠﻤﻠﺔ '‪ K‬ﻓﻨﺠﺩ‪:‬‬
‫‪S'  c 2 ( t '1  t ' )2  ( x'1  x' )2  ( y'1  y' ) 2  (z'1 z ' ) 2‬‬ ‫)‪(8.35‬‬
‫ﺜﻡ ﻟﻨﺤﺴﺏ '‪ S‬ﻓﻲ ﺍﻟﺠﻤﻠﺔ '‪ K‬ﺍﻨﻁﻼﻗﺎﹰ ﻤﻥ ﺘﺤﻭﻴﻼﺕ ﻟﻭﺭﻨﺘﺯ )‪ (8.18‬ﻓﻨﺠﺩ‬
‫ﺃﻭﻻﹰ‪:‬‬
‫‪2‬‬ ‫‪2‬‬ ‫‪V2‬‬
‫‪c ( t 1  t )  2V(x1  x )( t 1  t ) ‬‬ ‫‪( x1  x ) 2‬‬
‫‪2‬‬
‫‪c 2 ( t '1  t ' )2 ‬‬ ‫‪c‬‬
‫‪2‬‬
‫‪V‬‬
‫‪1‬‬
‫‪c2‬‬
‫‪2‬‬ ‫) ‪( x1  x )2  V 2 ( t1  t )2  2V( x1  x )( t1  t‬‬ ‫)‪(8.36‬‬
‫‪(x '1  x' ) ‬‬
‫‪2‬‬
‫‪V‬‬
‫‪1‬‬
‫‪c2‬‬
‫‪( y'1  y' ) 2  ( y1  y)2‬‬
‫‪(z'1 z ' ) 2  (z1  z ) 2‬‬
‫ﻭﺒﺎﻟﺘﺒﺩﻴل ﻓﻲ )‪ (8.35‬ﻨﺠﺩ ﺃﻥ ‪ '  S S‬ﻭﺒﺎﻟﺘﺎﻟﻲ ﻓﺎﻟﻤﺠﺎل ﻫﻭ ﻻﻤﺘﻐﻴﺭ‬
‫ﻓﻴﺯﻴﺎﺌﻲ ‪.Physical Invariant‬‬
‫ﻜﺜﻴﺭﺍﹰ ﻤﺎﻴﺅﺨﺫ‪ ،‬ﻋﻭﻀﺎﹰ ﻋﻥ ﺍﻟﻤﺠﺎل ‪ ،S‬ﻋﻨﺼﺭ ﺍﻟﻤﺠﺎل ‪ dS‬ﺍﻟﺫﻱ ﻴﻜﺘﺏ‬
‫ﺒﺎﻟﺸﻜل‪:‬‬
‫‪dS  c 2dt 2  dx 2  dy 2  dz 2  c 2 dt' 2 dx' 2 dy'2 dz' 2‬‬ ‫)‪(8.37‬‬
‫ﺍﻟﺫﻱ ﻗﺩ ﻴﻜﻭﻥ ﻤﻭﺠﺒﺎﹰ ﺃﻭ ﺴﺎﻟﺒﺎﹰ ﻓﻔﻲ ﺍﻟﺤﺎﻟﺔ ﺍﻷﻭﻟﻰ‪) :‬ﻭﺒﻌﺩ ﺍﺴﺘﺒﺩﺍل ‪ d‬ﺒـ ‪(‬‬
‫ﺃﻱ ﻋﻨﺩﻤﺎ‪:‬‬
‫‪c 2 ( t )2  ( x )2  (y)2   z ) 2‬‬ ‫)‪(8.38‬‬
‫ﻓﻴﻤﻜﻨﻨﺎ ﻋﻨﺩﺌﺫ‪ ‬ﺩﺍﺌﻤﺎﹰ ﺍﻴﺠﺎﺩ ﺠﻤﻠﺔ ﻗﻴﺎﺱ '‪ K‬ﺒﺤﻴﺙ ﺘﺤﺩﺙ ﻓﻴﻬﺎ ﺤﺎﺩﺜﺘﺎﻥ ﻓﻲ‬
‫ﻨﻔﺱ ﺍﻟﻤﻜﺎﻥ ﺃﻱ‪:‬‬
‫‪x'  y'  z'  0‬‬

‫‪- 269 -‬‬

‫)‪Create PDF files without this message by purchasing novaPDF printer (http://www.novapdf.com‬‬
‫ﻭﻟﻤﺎ ﻜﺎﻥ '‪ dS  dS‬ﻭ '‪ S  S‬ﻓﺈﻨﻨﺎ ﻨﺠﺩ ﺒﺎﻟﺘﺒﺩﻴل ﻓﻲ )‪ (8.37‬ﺃﻥ‪:‬‬
‫' ‪c 2 ( t ) 2  (x )2  (y)2  ( z )2  c t‬‬ ‫)‪(8.39‬‬
‫ﻭﻴﺘﺤﻘﻕ ﺫﻟﻙ ﻤﻥ ﺃﺠل ﺠﻤﻴﻊ ﺍﻟﻘﻴﻡ ﺍﻟﺤﻘﻴﻘﻴﺔ ﻟﻠﻌﺒﺎﺭﺓ ﺍﻟﺘﻲ ﺘﺤﺕ ﺍﻟﺠﺫﺭ‪ ،‬ﻭﻟﺫﻟﻙ‬
‫ﺘﺴﻤﻰ ﻫﺫﻩ ﺍﻟﻤﺠﺎﻻﺕ‪ :‬ﺍﻟﻤﺠﺎﻻﺕ ﺍﻟﺯﻤﻨﻴﺔ ﺍﻟﻤﺘﺸﺎﺒﻬﺔ‪.‬‬
‫ﺃﻤﺎ ﻓﻲ ﺤﺎل ﺍﻟﻤﺠﺎل ﺍﻟﻌﻘﺩﻱ ﺤﻴﺙ ﺘﻜﻭﻥ ﺍﻟﻌﺒﺎﺭﺓ ﺍﻟﻤﺠﺫﻭﺭﺓ ﺴﺎﻟﺒﺔ ﻓﻲ‬
‫)‪ (8.37‬ﻓﺈﻨﻪ ﻴﻤﻜﻥ ﺍﻴﺠﺎﺩ ﺠﻤﻠﺔ ﻗﻴﺎﺱ ﺘﺤﺩﺙ ﻓﻴﻬﺎ ﺤﺎﺩﺜﺘﺎﻥ ﻓﻲ ﻤﻜﺎﻨﻴﻥ ﻤﺨﺘﻠﻔﻴﻥ‬
‫ﻭﻟﻜﻥ ﻓﻲ ﺍﻟﻭﻗﺕ ﻨﻔﺴﻪ ‪ t  0‬ﻭﺸﺭﻁ ﺘﺤﻘﻴﻕ ﺫﻟﻙ ﻫﻭ‪:‬‬
‫)‪c 2 ( t ) 2  (x ) 2  (y)2  ( z )2  i ( x' ) 2  (y' )2  (z ' ) 2 ( 8.40‬‬
‫ﻭﻫﺫﺍ ﻤﻤﻜﻥ ﺩﺍﺌﻤﺎﹰ ﻋﻨﺩﻤﺎ ﺘﻜﻭﻥ ﺍﻟﻌﺒﺎﺭﺓ ﺍﻟﻤﺠﺫﻭﺭﺓ ﺴﺎﻟﺒﺔ ﻭﺘﺴﻤﻰ ﻫﺫﻩ‬
‫ﺍﻟﻤﺠﺎﻻﺕ ﺍﻟﻤﺠﺎﻻﺕ ﺍﻟﻤﻜﺎﻨﻴﺔ ﺍﻟﻤﺘﺸﺎﺒﻬﺔ‪.‬‬
‫ﻭﺃﺨﻴﺭﺍﹰ ﻟﺘﻌﺭﻴﻑ ﺍﻟﺯﻤﻥ ﺍﻟﺨﺎﺹ ﻨﻔﺭﺽ ﺠﻤﻠﺔ ﻋﻁﺎﻟﻴﺔ '‪ K‬ﺘﺘﺤﺭﻙ ﺒﺴﺭﻋﺔ ‪V‬‬
‫ﺒﺎﻟﻨﺴﺒﺔ ﺍﻟﻰ ‪ K‬ﻭﻟﻨﻔﺭﺽ ﺃﻨﻪ ﻓﻲ ﻨﻘﻁﺔ ﻤﺎ ) '‪ (x ' , y' , z‬ﺤﺩﺜﺕ ﺤﺎﺩﺜﺘﺎﻥ ﻤﺘﻌﺎﻗﺒﺘﺎﻥ‬
‫ﺒﻔﺭﻕ ﺯﻤﻨﻲ ﻗﺩﺭﻩ ‪ ، dt 0‬ﻭﻨﺅﻜﺩ ﻫﻨﺎ ﺃﻥ ﻫﺫﺍ ﺍﻟﺯﻤﻥ ﻤﻘﺎﺱ ﺒﺎﻟﺴﺎﻋﺎﺕ ﻓﻲ ﺍﻟﺠﻤﻠﺔ '‪K‬‬
‫ﻭﻟﺫﻟﻙ ﻴﺴﻤﻰ ﺒﺎﻟﺯﻤﻥ ﺍﻟﺨﺎﺹ‪ ،‬ﻭﻟﻨﺤﺴﺏ ﻋﻨﺼﺭ ﺍﻟﻤﺠﺎل ‪ dS‬ﺒﻴﻥ ﺍﻟﺤﺎﺩﺜﺘﻴﻥ ﻓﻨﺠﺩ‪:‬‬
‫)‪dS  dS'  c 2 (dt 0 ) 2  (dx' ) 2  (dy' ) 2  (dz' ) 2  c dt 0 (8.41‬‬
‫ﻭﻤﻨﻪ‪:‬‬
‫‪dS‬‬
‫‪dt 0 ‬‬
‫‪c‬‬
‫ﻭﻟﻤﺎ ﻜﺎﻥ ﻜل ﻤﻥ ‪ dS‬ﻭ ‪ c‬ﻤﻘﺩﺍﺭﺍﹰ ﻻﻤﺘﻐﻴﺭﺍﹰ ﻓﺈﻥ ‪ dt 0‬ﻫﻭ ﻤﻘﺩﺍﺭ ﻻﻤﺘﻐﻴﺭ‬
‫ﺒﻘﻴﻤﺘﻬﺎ ﻓﻨﺠﺩ‪:‬‬ ‫‪dS‬‬ ‫‪ Invariant‬ﻭﻟﻜﻲ ﻨﻌﺒﺭ ﻋﻥ ﻫﺫﺍ ﺍﻟﺯﻤﻥ ﻓﻲ ﺍﻟﺠﻤﻠﺔ ‪ K‬ﻨﺒﺩل‬
‫‪1 2‬‬
‫‪dt 0 ‬‬ ‫‪c (dt )2  (dx )2  (dy)2  (dz )2 ‬‬
‫‪c‬‬
‫‪2‬‬ ‫‪2‬‬ ‫‪2‬‬
‫‪1  dx ‬‬ ‫‪ dy ‬‬ ‫‪ dz  ‬‬ ‫‪v2‬‬
‫‪ dt 1 ‬‬ ‫‪          1 ‬‬ ‫‪dt‬‬
‫‪c 2  dt ‬‬ ‫‪ dt ‬‬ ‫‪ dt  ‬‬ ‫‪c2‬‬

‫ﻭﻤﻨﻪ ﻨﺠﺩ ﺒﺎﻻﺴﺘﻜﻤﺎل‪:‬‬

‫‪- 270 -‬‬

‫)‪Create PDF files without this message by purchasing novaPDF printer (http://www.novapdf.com‬‬
‫‪v2‬‬
‫‪t0 ‬‬ ‫‪‬‬ ‫‪1‬‬ ‫‪dt‬‬ ‫)‪(8.42‬‬
‫‪c2‬‬
‫ﺤﻴﺙ ‪ v‬ﺴﺭﻋﺔ ﺍﻟﺠﺴﻡ‪ ،‬ﻤﻘﺎﺴﺔ ﻓﻲ ﺍﻟﺠﻤﻠﺔ ﺍﻟﺜﺎﺒﺘﺔ ‪.K‬‬
‫‪" -68‬ﻻﺗﻐﲑ" ﺍﻟﻘﻮﺍﻧﲔ ﺍﻟﻔﻴﺰﻳﺎﺋﻴﺔ ﺑﺎﻟﻨﺴﺒﺔ ﻟﺘﺤﻮﻳﻼﺕ ﻟﻮﺭﻧﺘﺰ‪،‬‬
‫ﺍﻟﺼﻴﻐﺔ ﺍﻟﺮﺑﺎﻋﻴﺔ ﺍﻷﺑﻌﺎﺩ ﻟﻠﻨﻈﺮﻳﺔ ﺍﻟﻨﺴﺒﻴﺔ‪:‬‬
‫ﻁﺒﻘﺎﹰ ﻟﻠﻨﻅﺭﻴﺔ ﺍﻟﻨﺴﺒﻴﺔ ﻓﺈﻥ ﻜل ﺍﻟﻘﻭﺍﻨﻴﻥ ﺍﻟﻔﻴﺯﻴﺎﺌﻴﺔ ﻴﺠﺏ ﺃﻥ ﻴﻌﺒﺭ ﻋﻨﻬﺎ‬
‫ﺒﺎﻟﺸﻜل ﻨﻔﺴ ﻪ ﻓﻲ ﻜل ﺍﻟﺠﻤل ﺍﻟﻌﻁﺎﻟﻴﺔ‪ ،‬ﻭﻫﺫﺍ ﻴﻌﻨﻲ ﺃﻨﻬﺎ ﻴﺠﺏ ﺃﻥ ﺘﺼﺎﻍ ﺒﺤﻴﺙ‬
‫ﻻﻴﺘﻐﻴﺭ ﺸﻜﻠﻬﺎ ﻋﻨﺩ ﺍﻻﻨﺘﻘﺎل ﻤﻥ ﺠﻤﻠﺔ ﻋﻁﺎﻟﻴﺔ ﺇﻟﻰ ﺃﺨﺭﻯ ﺒﻭﺍﺴﻁﺔ ﺘﺤﻭﻴﻼﺕ‬
‫ﻟﻭﺭﻨﺘﺯ‪.‬‬
‫ﺇﻥ ﻤﻌﺎﺩﻻﺕ ﺍﻟﻤﻴﻜﺎﻨﻴﻙ ﺍﻟﻨﻴﻭﺘﻭﻨﻲ‪ ،‬ﻤﺜﻼﹰ‪ ،‬ﻻﺘﺘﻐﻴﺭ ﻋﻨﺩﻤﺎ ﻨﻁﺒﻕ ﻋﻠﻴﻬﺎ‬
‫ﺘﺤﻭﻴﻼﺕ ﻏﺎﻟﻴﻠﻲ ﻜﻤﺎ ﺭﺃﻴﻨﺎ ﺴﺎﺒﻘﺎﹰ‪ .‬ﻭﻟﻜﻨﻬﺎ ﺘﺄﺨﺫ ﺸﻜﻼﹰ ﺁﺨﺭ ﻋﻨﺩ ﺘﻁﺒﻴﻕ ﺘﺤﻭﻴﻼﺕ‬
‫ﻟﻭﺭﻨﺘﺯ ﻋﻠﻴﻬﺎ‪ ،‬ﺇﺫﻥ ﻴﺠﺏ ﺍﺠﺭﺍﺀ ﺒﻌﺽ ﺍﻟﺘﻌﺩﻴﻼﺕ ﻋﻠﻴﻬﺎ ﻋﻨﺩ ﻜﺘﺎﺒﺘﻬﺎ ﻓﻲ ﺍﻟﻨﻅﺭﻴﺔ‬
‫ﺍﻟﻨﺴﺒﻴﺔ‪ ،‬ﺃﻤﺎ ﻤﻌﺎﺩﻻﺕ ﻤﻜﺴﻭﻴل ﻓﻲ ﺍﻟﻜﻬﺭﻁﻴﺴﻴﺔ ﻓﻬﻲ ﻻﺘﺘﺄﺜﺭ ﺒﺘﺤﻭﻴﻼﺕ ﻟﻭﺭﻨﺘﺯ ﺃﻱ‬
‫ﺃﻨﻬﺎ ﺼﺤﻴﺤﺔ ﻭﻻﺤﺎﺠﺔ ﻻﺠﺭﺍﺀ ﺃﻱ ﺘﻌﺩﻴل ﻋﻠﻴﻬﺎ‪ .‬ﻭﻨﺅﻜﺩ ﻫﻨﺎ ﺃﻥ ﺍﻟﻔﺭﻀﻴﺔ ﺍﻷﻭﻟﻰ‬
‫ﻗﺩ ﻟﻌﺒﺕ ﺩﻭﺭﺍﹰ ﻫﺎﻤﺎﹰ ﻓﻲ ﺼﻴﺎﻏﺔ ﻜﺜﻴﺭ ﻤﻥ ﺍﻟﻘﻭﺍﻨﻴﻥ ﻓﻲ ﻨﻅﺭﻴﺔ ﺍﻟﺤﻘل ﻭﻤﻴﻜﺎﻨﻴﻙ ﺍﻟﻜﻡ‬
‫ﻭﺍﻟﻨﻅﺭﻴﺔ ﺍﻟﻤﻭﺠﻴﺔ ﻟﻠﺤﻘﻭل ﻭﺃﺴﻬﻤﺕ ﺇﻟﻰ ﺤﺩ ﻜﺒﻴﺭ ﻓﻲ ﺘﻘﺩﻡ ﻫﺫﻩ ﺍﻟﻌﻠﻭﻡ‪ ،‬ﻷﻥ ﻫﺫﻩ‬
‫ﺍﻟﻔﺭﻀﻴﺔ ﺘﺅﻜﺩ ﺃﻥ ﺠﻤﻴﻊ ﺍﻟﻘﻭﺍﻨﻴﻥ ﺍﻟﻔﻴﺯﻴﺎﺌﻴﺔ ﻴﺠﺏ ﺃﻥ ﺘﺼﺎﻍ ﺒﺎﻟﺸﻜل ﻨﻔﺴﻪ ﻓﻲ ﻜﺎﻓﺔ‬
‫ﺍﻟﺠﻤل ﺍﻟﻌﻁﺎﻟﻴﺔ‪.‬‬
‫ﻭﻴﻤﻜﻨﻨﺎ ﺍﻟﻘﻭل ﺃﻥ ﺨﺎﺼﺔ "ﻻﺘﻐﻴﺭ" ﺍﻟﻘﻭﺍﻨﻴﻥ ﺍﻟﻔﻴﺯﻴﺎﺌﻴﺔ ﻟ ﻴﺴﺕ ﻭﻟﻴﺩﺓ ﺍﻟﻨﻅﺭﻴﺔ‬
‫ﺍﻟﻨﺴﺒﻴﺔ‪ ،‬ﻓﻘﺩ ﻋﺭﻓﺕ ﺍﻟﻔﻴﺯﻴﺎﺀ ﺍﻟﻜﻼﺴﻴﻜﻴﺔ ﻨﻭﻋﺎﹰ ﻤﻥ ﺫﻟﻙ ﻓﻬﺫﻩ ﺍﻟﻘﻭﺍﻨﻴﻥ‪ ،‬ﻤﺜﻼﹰ‪ ،‬ﻴﺠﺏ‬
‫ﺃﻥ ﻻﺘﺘﻐﻴﺭ ﺇﺫﺍ ﺩﻭﺭﻨﺎ ﺍﻟﻤﺤﺎﻭﺭ ﺍﻻﺤﺩﺍﺜﻴﺔ ﺒﺯﺍﻭﻴﺔ ﻤﺎ ‪) ‬ﺍﻴﺯﻭﺘﺭﻭﺒﻴﺔ ﺍﻟﻔﺭﺍﻍ( ﻭﻓﻲ‬
‫ﺍﻟﺤﻘﻴﻘﺔ ﺘﺼﺎﻍ ﺒﺄﺤﺩ ﺸﻜﻠﻴﻥ ﺍﺜﻨﻴﻥ‪:‬‬
‫‪‬‬ ‫)‪(8.43‬‬
‫‪‬‬ ‫‪‬‬
‫‪ ‬‬ ‫)‪(8.44‬‬

‫‪- 271 -‬‬

‫)‪Create PDF files without this message by purchasing novaPDF printer (http://www.novapdf.com‬‬
‫ﻭﻴﺘﻀﺢ ﻤﺒﺎﺸﺭﺓﹰ ﻤﻥ ﺍﻟﺤﺎﻟﺔ ﺍﻷﻭﻟﻰ‪ ،‬ﻋﻨﺩﻤﺎ ﻴﻌﺒﺭ ﻋﻥ ﺍﻟﻘﻴﻡ ﺍﻟﻔﻴﺯﻴﺎﺌﻴﺔ ﺒﻜﻤﻴﺎﺕ‬
‫ﺴﻠﻤﻴﺔ‪ ،‬ﺃﻥ ﺍﻟﻘﺎﻨﻭﻥ ﻻﻴﺘﻐﻴﺭ ﻋﻨﺩﻤﺎ ﺘﺩﻭﺭ ﺍﻟﻤﺤﺎﻭﺭ ﺍﻻﺤﺩﺍﺜﻴﺔ ﺃﻤﺎ ﻓﻲ ﺍﻟﺤﺎﻟﺔ ﺍﻟﺜﺎﻨﻴﺔ‬
‫‪‬‬ ‫‪‬‬
‫ﺤﻴﺙ ﻴﻌﺒﺭ ﻋﻥ ﺍﻟﻘﻴﻡ ﺍﻟﻔﻴﺯﻴﺎﺌﻴﺔ ﺒﻜﻤﻴﺎﺕ ﺸﻌﺎﻋﻴﺔ‪ ،‬ﻓﺈﻥ ﻜﻼ ﻤﻥ ‪ ‬ﻭ ‪ ‬ﻴﺘﻐﻴﺭ‪ ،‬ﻓﺈﺫﺍ‬
‫ﻓﺭﻀﻨﺎ ﺃﻥ ﺍﻟﺩﻭﺭﺍﻥ ﻴﺘﻡ ﺤﻭل ﺍﻟﻤﺤﻭﺭ ‪ oz‬ﻓﺈﻥ ﻜﻼ ﻤﻥ ﻤﺭﻜﺒﺘﻲ ﻨﺼﻑ ﺍﻟﻘﻁﺭ‬
‫ﺍﻟﺸﻌﺎﻋﻲ ﻋﻠﻰ ‪ ox‬ﻭ ‪ oy‬ﺴﺘﺼﺒﺢ ﻓﻲ ﺍﻟﺤﺎﻟﺔ ﺍﻟﻌﺎﻤﺔ‪:‬‬
‫)‪x  x' cos   y' sin  , y  y' cos   x' sin  (8.45‬‬
‫ﻭﻴﺘﺤﻭل ﻜﻼ ﻤﻥ ﻁﺭﻓﻲ ﺍﻟﻌﻼﻗﺘﻴﻥ )‪ (8.44‬ﺒﺎﻟﻌﻼﻗﺘﻴﻥ ﺍﻟﺴﺎﺒﻘﺘﻴﻥ ﻨﻔﺴﻬﻤﺎ‬
‫ﺒﺤﻴﺙ ﺘﺒﻘﻰ ﻫﺫﻩ ﺍﻟﻤﻌﺎﺩﻟﺔ ﺼﺤﻴﺤﺔ؛ ﻓﺈﺫﺍ ﺃﺨﺫﻨﺎ ﻤﺜﻼﹰ ﻤﻌﺎﺩﻟﺔ ﺍﻟﺘﺤﺭﻴﻙ ﺍﻷﺴﺎﺴﻴﺔ‪:‬‬
‫‪‬‬ ‫‪‬‬
‫‪mW  F‬‬
‫ﺃﻭ ﺒﺎﻟﺸﻜل ﺍﻟﺘﺤﻠﻴﻠﻲ‪:‬‬
‫‪mx   Fx , my   Fy , mz    Fz‬‬ ‫)‪(8.46‬‬
‫ﻓﺈﻥ ﻜﻼ ﻤﻥ ﺍﻟﻁﺭﻓﻴﻥ ﺴﻴﺘﺤﻭل ﺤﺴﺏ ﺍﻟﻌﻼﻗﺎﺕ )‪ (8.45‬ﻭﺒﺎﻟﺘﺎﻟﻲ ﻴﺒﻘﻰ‬
‫ﻗﺎﻨﻭﻥ ﻨﻴﻭﺘﻥ ﻓﻲ ﺍﻟﺘﺤﺭﻴﻙ ﺼﺤﻴﺤﺎﹰ‪.‬‬
‫ﻫﺫﺍ ﻭﻴﻤﻜﻥ ﺒﺎﺨﺘﺼﺎﺭ‪ ،‬ﻜﺘﺎﺒﺔ ﺍﻟﺸﺭﻭﻁ ﻜﺎﻓﺔ ﺍﻟﺘﻲ ﻴﺠﺏ ﺃﻥ ﺘﺤﻘﻘﻬﺎ ﻗﻭﺍﻨﻴﻥ‬
‫ﺍﻟﻔﻴﺯﻴﺎﺀ ﺍﻟﻜﻼﺴﻴﻜﻴﺔ ﻜﻤﺎﻴﻠﻲ‪:‬‬
‫‪ -1‬ﻻﺘﺘﻐﻴﺭ ﺒﺎﻟﻨﺴﺒﺔ ﻟﺘﺤﻭﻴﻼﺕ ﻏﺎﻟﻴﻠﻲ‪.‬‬
‫‪ -2‬ﻻﺘﺘﻐﻴﺭ ﺒﺎﻟﻨﺴﺒﺔ ﻻﻨﺘﻘﺎﻻﺕ ﻭﺩﻭﺭﺍﻨﺎﺕ ﺍﻟﻤﺤﺎﻭﺭ ﺍﻻﺤﺩﺍﺜﻴﺔ‪.‬‬
‫‪ -3‬ﻻﺘﺘﻐﻴﺭ ﺇﺫ ﻏﻴﺭﻨﺎ ﺍﻟﺯﻤﻥ ‪ t‬ﺒـ ‪ t  t‬ﺃﻱ ﺃﻥ ﺍﻟﺯﻤﻥ ﻴﺴﻴﺭ ﺒﺼﻭﺭﺓ‬
‫ﻤﺴﺘﻤﺭﺓ ﻭﻤﻨﺘﻅﻤﺔ‪.‬‬
‫‪ -4‬ﻻﺘﺘﻐﻴﺭ ﻋﻨﺩﻤﺎ ﻨﺒﺩل ‪ t‬ﺒـ ‪ -t‬ﻭﻫﺫﺍ ﻴﻌﻨﻲ ﺘﻨﺎﻅﺭ ﺍﻟﻘﻭﺍﻨﻴﻥ ﺍﻟﻔﻴﺯﻴﺎﺌﻴﺔ‬
‫ﺒﺎﻟﻨﺴﺒﺔ ﻟﻠﻤﺎﻀﻲ ﻭﺍﻟﻤﺴﺘﻘﺒل‪.‬‬
‫ﺃﻤﺎ ﻓﻲ ﺍﻟﻨﻅﺭﻴﺔ ﺍﻟﻨﺴﺒﻴﺔ ﻓﻨﺠﺩ ﻋﻭﻀﺎﹰ ﻋﻥ ﺍﻟﺸﺭﻁ ﺍﻷﻭل ﺍﻟﺸﺭﻁ ﺍﻟﺘﺎﻟﻲ‪:‬‬
‫ﻻﺘﻐﻴﺭ ﺍﻟﻘﻭﺍﻨﻴﻥ ﺍﻟﻔﻴﺯﻴﺎﺌﻴﺔ ﺒﺎﻟﻨﺴﺒﺔ ﻟﺘﺤﻭﻴﻼﺕ ﻟﻭﺭﻨﺘﺯ‪ ،‬ﻭﻟﻜﻨﻬﺎ ﺘﻭﺍﻓﻕ ﻋﻠﻰ ﺍﻟﺸﺭﻁﻴﻥ‬
‫ﺍﻟﺜﺎﻨﻲ ﻭﺍﻟﺭﺍﺒﻊ‪ ،‬ﻭﻤﻥ ﺍﻟﻭﺍﻀﺢ ﺃﻨﻪ ﻟﻴﺱ ﻟﻠﺸﺭﻁ ﺍﻟﺜﺎﻟﺙ ﻤﻌﻨﻰ ﻓﻲ ﺍﻟﻨﻅﺭﻴﺔ ﺍﻟﻨﺴﺒﻴﺔ‬
‫ﻷﻥ ﺍﻟﺯﻤﻥ ﻤﺭﺘﺒﻁ ﺒﺎﻟﺤﺭﻜﺔ ﻨﻔﺴﻬﺎ‪.‬‬

‫‪- 272 -‬‬

‫)‪Create PDF files without this message by purchasing novaPDF printer (http://www.novapdf.com‬‬
‫ﻻﻴﺠﺎﺩ ﺍﻟﻁﺭﻴﻘﺔ ﺍﻟﻌﺎﻤﺔ ﻓﻲ ﺍﻟﺤﺼﻭل ﻋﻠﻰ "ﺍﻟﻼﻤﺘﻐﻴﺭﺍﺕ" ﺍﻟﻨﺴﺒﻴﺔ ﻨﻌﻭﺩ ﺇﻟﻰ‬
‫ﻓﻜﺭﺓ ﺍﻟﻤﺠﺎل ‪ dS‬ﻭﻨﻔﺭﺽ ﻤﺘﺤﻭﻻﹰ ﺠﺩﻴﺩﹰﺍ ﻋﻘﺩﻴﺎﹰ ﻫﻭ‪:‬‬
‫‪  i ct‬‬ ‫)‪(8.47‬‬
‫ﻓﻨﺠﺩ ﺃﻥ ﺍﻟﻤﺠﺎل ‪ dS 2‬ﻴﺼﺒﺢ‪:‬‬
‫‪ dS 2  dx 2  dy 2  dz 2  d 2‬‬ ‫)‪(8.48‬‬
‫ﻭﻟﻨﻔﺭﺽ‪ ،‬ﺒﺩﻭﻥ ﺍﻟﺘﻭﺴﻊ ﻓﻲ ﺍﻟﻤﻔﺎﻫﻴﻡ ﺍﻟﺭﻴﺎﻀﻴﺔ‪ ،‬ﻭﺠﻭﺩ ﻤﺤﺎﻭﺭ ﻤﻌﺎﻤﺩﺓ‬
‫) ‪ (x, y, z, t‬ﻓﻲ ﻓﺭﺍﻍ ﻤﺎ ﺭﺒﺎﻋﻲ ﺍﻷﺒﻌﺎﺩ‪ ،‬ﻭﻨﻘﻭل ﺒﺄﻥ ﺘﺤﻭﻴﻼﺕ ﻟﻭﺭﻨﺘﺯ ﺘﺄﺨﺫ ﺸﻜﻼﹰ‬
‫ﺨﻁﻴﺎﹰ ﻓﻲ ﻫﺫﺍ ﺍﻟﻔﺭﺍﻍ ﺍﻟﺠﺩﻴﺩ ﺒﺤﻴﺙ ﻴﺒﻘﻰ ﺍﻟﻤﺠﺎل ‪ dS 2‬ﻻﻤﺘﻐﻴﺭ ﺍﹰ‪ ،‬ﻭﻟﺒﺭﻫﺎﻥ ﺫﻟﻙ‬
‫ﻨﻔﺭﺽ ﻜﻤﺎ ﻓﻲ ﺍﻟﺴﺎﺒﻕ ﺃﻥ ﺤﺭﻜﺔ ﺍﻟﺠﻤﻠﺔ '‪ K‬ﺒﺎﻟﻨﺴﺒﺔ ﻟـ ‪ K‬ﺘﺘﻡ ﺒﺎﻻﺘﺠﺎﻩ ﺍﻟﻤﺸﺘﺭﻙ‬
‫ﻟﻠﻤﺤﻭﺭﻴﻥ ‪ x‬ﻭ '‪ x‬ﻭﻫﺫﺍ ﻴﻌﻨ ﻲ ﺃﻥ ‪ z‬ﻭ ‪ y‬ﻴﺒﻘﻴﺎﻥ ﺜﺎﺒﺘﻴﻥ ﻓﻲ ﺍﻟﺠﻤﻠﺘﻴﻥ ﻭﺃﻥ ﺍﻟﺘﻐﻴﺭ‬
‫ﻴﻁﺭﺃ ﻓﻘﻁ ﻋﻠﻰ ﺍﻟﻤﺭﻜﺒﺘﻴﻥ ‪ ‬ﻭ ‪ ،x‬ﻓﻠﻨﻔﺭﺽ ﺩﻭﺭﺍﻨﺎﹰ ﺒﺯﺍﻭﻴﺔ ﻤﺎ ‪ ‬ﻓﻲ ﺍﻟﻤﺴﺘﻭﻯ‬
‫) '‪ (x ' , ‬ﺘﻜﻭﻥ ﺍﻟﻌﻼﻗﺔ ﻓﻴﻪ ﺒﻴﻥ ) ‪ (x, ‬ﻭ ) '‪ (x ' , ‬ﺒﺎﻟﺸﻜل )ﺍﻨﻅﺭ )‪.((8.45‬‬
‫‪x  x' cos   ' sin  ,   ' cos   x' sin ‬‬ ‫)‪(8.49‬‬
‫ﻭﻫﺎﺘﺎﻥ ﺍﻟﻌﻼﻗﺘﺎﻥ ﺼﺤﻴﺤﺘﺎﻥ ﺒﺎﻟﻨﺴﺒﺔ ﻟﻜل ﻨﻘﺎﻁ ﺍﻟﺠﻤﻠﺔ '‪ K‬ﻓﻠﻨﻜﺘﺒﻬﺎ ﺒﺎﻟﻨﺴﺒﺔ‬
‫ﻟﻤﺭﻜﺯ ﻫﺫﻩ ﺍﻟﺠﻤﻠﺔ ﺃﻱ ﻓﻲ ﺍﻟﻨﻘﻁﺔ ‪ x '  0‬ﻓﻨﺠﺩ‪:‬‬
‫‪x   ' sin  ,   ' cos ‬‬ ‫)‪(8.50‬‬
‫ﻭﻤﻨﻪ‪:‬‬
‫)‪(8.51‬‬
‫‪x‬‬ ‫‪x‬‬ ‫‪ix i‬‬ ‫‪x‬‬ ‫‪V‬‬
‫‪tg   ‬‬ ‫‪‬‬ ‫‪‬‬ ‫‪‬‬ ‫‪i‬‬
‫‪‬‬ ‫‪ic t c t c‬‬ ‫‪t‬‬ ‫‪c‬‬
‫ﺤﻴﺙ ‪ V‬ﻫﻲ ﺍﻟﺴﺭﻋﺔ ﺍﻟﻤﻨﺘﻅﻤﺔ ﻟﻠﺠﻤﻠﺔ '‪ K‬ﺒﺎﻟﻨﺴﺒﺔ ﻟﻠﺠﻤﻠﺔ ‪ K‬ﻭﻟﻴﺱ ﻤﻥ‬
‫ﺍﻟﺼﻌﺏ ﺤﺴﺎﺏ ‪ sin ‬ﻭ ‪ cos ‬ﺇﺫ ﻨﺠﺩ ﺒﺴﻬﻭﻟﺔ ﺃﻥ‪:‬‬

‫‪- 273 -‬‬

‫)‪Create PDF files without this message by purchasing novaPDF printer (http://www.novapdf.com‬‬
‫‪1‬‬ ‫‪1‬‬ ‫‪‬‬
‫‪cos  ‬‬ ‫‪‬‬ ‫‪‬‬
‫‪1  tg 2 ‬‬ ‫‪V2‬‬ ‫‪‬‬
‫‪1‬‬ ‫‪‬‬
‫‪c2‬‬
‫)‪(8.52‬‬
‫‪‬‬
‫‪V‬‬ ‫‪‬‬
‫‪i‬‬ ‫‪‬‬
‫‪sin   tg  cos  ‬‬ ‫‪c‬‬ ‫‪‬‬
‫‪V2‬‬ ‫‪‬‬
‫‪1‬‬ ‫‪‬‬
‫‪c2‬‬ ‫‪‬‬
‫ﺜﻡ ﻨﺠﺩ ﺒﺎﻟﺘﺒﺩﻴل ﻓﻲ )‪ (8.49‬ﺃﻥ‪:‬‬
‫‪V2‬‬ ‫‪V‬‬
‫‪x'i‬‬ ‫'‪‬‬ ‫' ‪'i x‬‬
‫‪x‬‬ ‫‪c‬‬ ‫‪, ‬‬ ‫‪c‬‬ ‫)‪(8.53‬‬
‫‪2‬‬
‫‪V‬‬ ‫‪V2‬‬
‫‪1‬‬ ‫‪1‬‬
‫‪c2‬‬ ‫‪c2‬‬
‫ﻭﺇﺫﺍ ﺒﺩﻟﻨﺎ ‪ ‬ﻭ '‪ ‬ﺒﻘﻴﻤﺘﻴﻬﻤﺎ ﺒﺎﻟﻨﺴﺒﺔ ﺇﻟﻰ ‪ t‬ﻭ '‪ t‬ﻨﺠﺩ ﻨﻔﺱ ﺍﻟﻌﻼﻗﺎﺕ )‪(8.18‬‬
‫ﺃﻱ ﺘﺤﻭﻴﻼﺕ ﻟﻭﺭﻨﺘﺯ ﻭﻫﻭ ﺍﻟﻤﻁﻠﻭﺏ‪.‬‬
‫‪ -69‬ﺍﳌﺘﺠﻬﺎﺕ ﰲ ﺍﻟﻔﺮﺍﻍ ﺭﺑﺎﻋﻲ ﺍﻷﺑﻌﺎﺩ‪:‬‬
‫‪Vectors in four demensions continum‬‬
‫ﻟﻘﺩ ﺭﺃﻴﻨﺎ ﻓﻲ ﺍﻟﻔﻘﺭﺓ ﺍﻟﺴﺎﺒﻘﺔ ﻜﻴﻑ ﺃﻨﻪ ﺒﻔﺭﺽ ﻤﺤﻭﺭ ﺠﺩﻴﺩ ‪ ‬ﻟﻘﻴﺎﺱ ﺍﻟﺯﻤﻥ‬
‫ﺃﻤﻜﻥ ﺍﻟﺘﻌﺒﻴﺭ ﻋﻥ ﻋﻨﺼﺭ ﺍﻟﻤﺠﺎل ‪ dS‬ﻓﻲ ﺍﻟﻔﺭﺍﻍ ﺍﻟﺭﺒﺎﻋﻲ ﺒﺸﻜل ﻤﺠﻤﻭﻉ ﻤﺭﺒﻌﺎﺕ‬
‫ﻷﺭﺒﻊ ﻗﻴﻡ ﺜﻼﺙ ﻤﻨﻬﺎ ‪ dz, dy, dx‬ﻟﻠﻤﻜﺎﻥ ﻭﻭﺍﺤﺩﺓ ‪ d‬ﻟﻠﺯﻤﻥ‪ ،‬ﻭﺒﺭﻫﻨﺎ ﺃﻥ ﺘﺤﻭﻴﻼﺕ‬
‫ﻟﻭﺭﻴﻨﺘﺯ ﺘﺅﻭل ﺇﻟﻰ ﺩﻭﺭﺍﻨﺎﺕ ﻓﻲ ﻫﺫﺍ ﺍﻟﻔﺭﺍﻍ ﺍﻟﺭﺒﺎﻋﻲ‪.‬‬
‫ﻭﺴﻨﺒﺤﺙ ﺍﻵﻥ ﻓﻲ ﺨﻭﺍﺹ ﺍﻟﻤﺘﺠﻬﺎﺕ ﺍﻟﻤﻌﺭﻓﺔ ﻓﻲ ﺍﻟﺴﺎﺤﺔ ﺍﻟﺠﺩﻴﺩﺓ ﻭﻟﺫﻟﻙ‬
‫ﻨﻔﺭﺽ ﻨﺼﻑ ﺍﻟﻘﻁﺭ ﺍﻟﻤﺘﺠﻪ ‪ r‬ﻜﺸﻌﺎﻉ ﻓﻲ ﺍﻟﻔﺭﺍﻍ ﺍﻟﺭﺒﺎﻋﻲ ﻤﺭﻜﺒﺎﺘﻪ )‪( x, y, z, ‬‬
‫ﻭﻟﻨﻁﺒﻕ ﺘﺤﻭﻴﻼﺕ ﻟﻭﺭﻨﺘﺯ ﻋﻠﻴﻪ ﻓﻨﺠﺩ ﺃﻨﻪ ﻴﺘﺤﻭل ﻁﺒﻘﺎﹰ ﻟﻠﻌﻼﻗﺔ‪:‬‬
‫‪r      r' ‬‬ ‫)‪(8.54‬‬
‫‪‬‬

‫ﺇﻥ ﻓﻬﻡ ﻫﺫﺍ ﺍﻟﺭﻤﺯ ﻴﻘﺘﻀﻲ ﺍﻟﺭﺠﻭﻉ ﺇﻟﻰ ﺍﻟﻔﺭﺍﻍ ﺍﻟﺜﻼﺜﻲ ﺍﻷﺒﻌﺎﺩ؛ ﻓﺎﻟﺸﻌﺎﻉ‬
‫‪‬‬
‫'‪ r‬ﺍﻟﻤﻌﺭﻑ ﻓﻲ ﻫﺫﺍ ﺍﻟﻔﺭﺍﻍ ﺒﻤﺭﻜﺒﺎﺘﻪ ' ‪ z' , y' , x‬ﻴﺅﻭل ﺒﻌﺩ ﺩﻭﺭﺍﻥ ﺍﻟﻤﺤﺎﻭﺭ‬
‫‪‬‬
‫ﺍﻻﺤﺩﺍﺜﻴﺔ '‪ oz' , oy' , ox‬ﺇﻟﻰ ﻤﺘﺠﻪ ‪ r‬ﻤﺭﻜﺒﺎﺘﻪ ﻓﻲ ﺍﻟﺠﻤﻠﺔ ﺍﻟﺠﺩﻴﺩﺓ ‪ z, y, x‬ﻭﻴﻤﻜﻥ‬

‫‪- 274 -‬‬

‫)‪Create PDF files without this message by purchasing novaPDF printer (http://www.novapdf.com‬‬
‫ﺃﻥ ﺘﺘﻌﻴﻥ ﺍﻟﻌﻼﻗﺔ ﺒﻴﻥ ﻤﺭﻜﺒﺎﺕ ﺍﻟﻤﺘﺠﻪ ﻓﻲ ﺍﻟﻤﺠﻤﻭﻋﺘﻴﻥ )‪ (K‬ﻭ )'‪ (K‬ﺒﺎﺴﻘﺎﻁ‬
‫‪‬‬ ‫‪‬‬ ‫‪‬‬ ‫‪‬‬
‫ﻋﻠﻰ ﺍﻟﻤﺤﺎﻭﺭ ﺍﻟﺠﺩﻴﺩﺓ ﺤﻴﺙ ﻨﺠﺩ ﺒﺴﻬﻭﻟﺔ‪:‬‬ ‫'‪r '  x ' i' y' j'  z ' k‬‬ ‫ﺍﻟﻤﺘﺠﻪ‬
‫‪x  11x' 12 y' 13z' ‬‬
‫)‪(8.55‬‬
‫‪‬‬
‫‪y   21x'  22 y'  23z '‬‬
‫‪z   31x'   32 y'  33z' ‬‬
‫ﺃﻭ ﺒﺎﻟﺸﻜل ﺍﻟﻤﺼﻔﻭﻓﻲ ﺍﻟﺘﺎﻟﻲ‪:‬‬
‫‪ x   11  21  31   x ' ‬‬
‫‪  ‬‬ ‫‪ ‬‬
‫‪ y     21  22  23   y' ‬‬ ‫)‪(8.56‬‬
‫‪z  ‬‬ ‫‪ ‬‬
‫‪   31  32  33   z' ‬‬
‫ﻤﻊ ﺍﻟﻌﻠﻡ ﺃﻥ ‪  ‬ﻫﻲ ﺠﻴﻭﺏ ﺍﻟﺘﻤﺎﻡ ﺍﻟﻤﻭﺠﻬﺔ ﻟﻠﻤﺤﺎﻭﺭ ﺍﻟﺠﺩﻴﺩﺓ ﺒﺩﻻﻟﺔ‬
‫ﺍﻟﻘﺩﻴﻤﺔ‪ .‬ﻭﻴﻤﻜﻥ ﺍﺨﺘﺼﺎﺭ ﺍﻟﻤﻌﺎﺩﻻﺕ ﺍﻟﺜﻼﺙ ﺍﻟﺴﺎﺒﻘﺔ ﺒﻤﻌﺎﺩﻟﺔ ﻭﺍﺤﺩﺓ ﻫﻲ ﺍﻟﻌﻼﻗﺔ‬
‫)‪ (8.54‬ﻭﻴﻤﻜﻥ ﺃﺤﻴﺎﻨﺎﹰ ﺍﺨﺘﺼﺎﺭ ﺍﻟﺭﻤﺯ ‪ ‬ﻋﻠﻰ ﺃﻥ ﻴﺘﻡ ﺍﻟﺠﻤﻊ ﺒﺎﻟﻭﺴﻴﻁ ﺍﻟﻤﻜﺭﺭ ﺃﻱ‬
‫ﺃﻥ ﺍﻟﻌﻼﻗﺔ ﺍﻟﺴﺎﺒﻘﺔ ﺘﻭﻀﻊ ﺒﺎﻟﺸﻜل‪:‬‬
‫‪r    r'‬‬ ‫)‪(8.57‬‬
‫‪‬‬
‫ﻭﻋﻨﺩ ﺩﻭﺭﺍﻥ ﺍﻟﻤﺤﺎﻭﺭ ﺍﻻﺤﺩﺍﺜﻴﺔ ﻓﺈﻥ ﺍﻟﻘﻴﻤﺔ ﺍﻟﻤﻁﻠﻘﺔ ﻟﻠﻤﺘﺠﻪ ‪ r‬ﻻﺘﺘﻐﻴﺭ ﺃﻱ‬
‫ﺃﻥ‪:‬‬
‫‪‬‬ ‫‪‬‬
‫)‪| r | x 2  y 2  z 2  x' 2  y' 2  z' 2 | r' | (8.58‬‬
‫ﻭﺒﺎﻟﺘﺎﻟﻲ ﺇﺫﺍ ﺒﺩﻟﻨﺎ ﺍﻟﻤﺭﻜﺒﺎ ﺕ ‪ x, y, z‬ﺒﻘﻴﻤﻬﺎ ﻤﻥ )‪ (8.55‬ﻓﺈﻥ ﺍﻟﺸﺭﻁ )‪(8.58‬‬
‫ﻴﺼﺒﺢ ﻤﺠﺭﺩ ﻋﻼﻗﺎﺕ ﻴﺠﺏ ﺃﻥ ﺘﺤﻘﻘﻬﺎ ﺍﻟﻤﻘﺎﺩﻴﺭ ‪  ‬ﻭﻤﻥ ﺍﻟﺴﻬل ﻤﻼﺤﻅﺔ ﺃﻥ‬
‫ﺃﻤﺜﺎل ﺍﻟﺠﺩﺍﺀﺍﺕ ' ‪ z' x ' , y' z‬ﻴﺠﺏ ﺃﻥ ﺘﺴﺎﻭﻱ ﺍﻟﺼﻔﺭ ﻭﺃﻥ ﺃﻤﺜﺎل ‪z' 2 , y' 2 , x ' 2‬‬
‫ﺘﺴﺎﻭﻱ ﺍﻟﻭﺍﺤﺩ ﻭﻫﻜﺫﺍ ﻨﺤﺼل ﻋﻠﻰ ﺍﻟﻌﻼﻗﺎﺕ ﺍﻟﺘﺎﻟﻴﺔ‪:‬‬
‫‪3‬‬
‫‪  1   2  0 ,   2  3  0 ,   3  1  0‬‬ ‫)‪(8.59‬‬
‫‪ 1‬‬ ‫‪‬‬ ‫‪‬‬
‫‪3‬‬
‫‪  1  1  1   2  2  1   3  3  1‬‬ ‫)‪(8.60‬‬
‫‪ 1‬‬ ‫‪‬‬ ‫‪‬‬
‫ﻭﻴﻤﻜﻥ ﺃﻴﻀﺎﹰ ﺍﺨﺘﺼﺎﺭ ﺍﻟﻌﻼﻗﺎﺕ ﺍﻟﺴﺎﺒﻘﺔ ﺒﻌﻼﻗﺘﻴﻥ ﻤﻥ ﺍﻟﺸﻜل‪:‬‬

‫‪- 275 -‬‬

‫)‪Create PDF files without this message by purchasing novaPDF printer (http://www.novapdf.com‬‬
‫‪     0‬‬ ‫‪(   )‬‬
‫‪‬‬ ‫)‪(8.61‬‬
‫‪     1‬‬ ‫‪(   ) ‬‬

‫ﺃﻭ ﺤﺘﻰ ﺒﻌﻼﻗﺔ ﻭﺍﺤﺩﺓ ﺘﺴﻤﻰ ﺃﺤﻴﺎﻨﺎﹰ ﻋﻼﻗﺔ ﺍﻟﺘﻭﺍﻤﺩ‪:‬‬


‫‪      ‬‬ ‫)‪(8.62‬‬
‫ﺃﻤﺎ ﻓﻲ ﺍﻟﻔﺭﺍﻍ ﺍﻟﺭﺒﺎﻋﻲ ﻓﻴﻤﻜﻥ ﺘﻁﺒﻴﻕ ﺍﻟﻤﻨﺎﻗﺸﺔ ﺍﻟﺴﺎﺒﻘﺔ ﻨﻔﺴﻬﺎ ﺒﻔﺎﺭﻕ ﻭﺤﻴﺩ‬
‫ﻓﻘﻁ ﻭﻫﻭ ﺃﻥ ﺍﻷﺩﻟﺔ ﺍﻟﺴﺎﺒﻘﺔ ‪ , , ‬ﺴﺘﺄﺨﺫ ﺃﺭﺒﻊ ﻗﻴﻡ ﻤﻭﺍﻓﻘﺔ ﻟﻸﺒﻌﺎﺩ ﺍﻷﺭﺒﻌﺔ‬
‫) ‪. (, z, y, x‬‬
‫ﻟﻨﺤﺴﺏ ﻤﺭﺒﻊ ﺍﻟﻤﺘﺠﻪ ‪ r‬ﻭﻨﺒﺭﻫﻥ ﺃﻨﻪ ﻻﻴﺘﻐﻴﺭ ﺒﺎﻟﺩﻭﺭﺍﻥ ﻓﻨﺠﺩ ﺒﺎﻻﻋﺘﻤﺎﺩ‬
‫ﻋﻠﻰ )‪ (8.57‬ﻭﻋﻠﻰ )‪ (8.62‬ﺃﻥ‪:‬‬
‫‪r2  r r  (  r'  )(  r ' )     r' r'   r' r'    r'  2‬‬ ‫)‪(8.63‬‬
‫ﻟﻨﻌﺩ ﺍﻵﻥ ﺇﻟﻰ ﺍﻟﻔﺭﺍﻍ ﺍﻟﺭﺒﺎﻋﻲ ﻭﺘﺩﻭﺭ ﺍﻟﻤﺤﺎﻭﺭ ﻓﻲ ﺍﻟﻤﺴﺘﻭﻯ ) ‪(x  , ‬‬
‫ﺒﺯﺍﻭﻴﺔ ‪ ‬ﻓﻨﺤﺼل ﻨﺘﻴﺠﺔ ﻟﺫﻟﻙ ﻋﻠﻰ ﺍﻟﻌﻼﻗﺎﺕ )‪ (8.53‬ﻨﻔﺴﻬﺎ‪ ،‬ﻭﺒﺎﻟﺘﺎﻟﻲ ﻓﺎﻻﻨﺘﻘﺎل‬
‫ﻤﻥ ﺍﻟﻤﺤﺎﻭﺭ ﺍﻟﻘﺩﻴﻤﺔ ﺍﻟﻰ ﺍﻟﺠﺩﻴﺩﺓ ﻴﻜﺎﻓﻰﺀ ﺍﻟﺘﺤﻭﻴل ﺍﻟﺘﺎﻟﻲ‪:‬‬
‫‪‬‬ ‫‪V‬‬ ‫‪‬‬
‫‪‬‬ ‫‪i‬‬ ‫‪‬‬
‫‪‬‬ ‫‪1‬‬ ‫‪c‬‬ ‫‪‬‬
‫‪0 0‬‬
‫‪‬‬ ‫‪V2‬‬ ‫‪2 ‬‬
‫'‪x‬‬
‫‪x ‬‬ ‫‪V‬‬
‫‪  ‬‬
‫‪ ‬‬ ‫‪1‬‬ ‫‪1‬‬
‫‪  ‬‬ ‫‪c2‬‬ ‫‪c‬‬ ‫‪2 ‬‬
‫‪ ‬‬
‫‪  ‬‬ ‫‪ ‬‬
‫‪  ‬‬ ‫‪ ‬‬
‫‪y   0‬‬ ‫‪1 0‬‬ ‫‪0‬‬ ‫‪  y' ‬‬ ‫)‪(8.64‬‬
‫‪z   0‬‬ ‫‪ ‬‬
‫‪  ‬‬ ‫‪0 1‬‬ ‫‪0‬‬ ‫‪  z' ‬‬
‫‪  ‬‬ ‫‪V‬‬ ‫‪ ‬‬
‫‪   i‬‬ ‫‪1‬‬
‫‪ ‬‬
‫‪  ‬‬ ‫‪c‬‬ ‫‪0‬‬ ‫‪0‬‬ ‫‪  ' ‬‬
‫‪‬‬ ‫‪V2‬‬ ‫‪V‬‬ ‫‪2 ‬‬
‫‪ 1‬‬ ‫‪1‬‬ ‫‪‬‬
‫‪‬‬ ‫‪c2‬‬ ‫‪c 2 ‬‬
‫‪‬‬

‫ﺃﻱ ﺃﻥ‪:‬‬

‫‪- 276 -‬‬

‫)‪Create PDF files without this message by purchasing novaPDF printer (http://www.novapdf.com‬‬
‫‪‬‬ ‫‪V‬‬ ‫‪‬‬
‫‪‬‬ ‫‪i‬‬ ‫‪‬‬
‫‪‬‬ ‫‪1‬‬ ‫‪c‬‬ ‫‪‬‬
‫‪0 0‬‬
‫‪‬‬ ‫‪V2‬‬ ‫‪V‬‬ ‫‪2 ‬‬
‫‪ 1‬‬ ‫‪1‬‬ ‫‪‬‬
‫‪‬‬ ‫‪c2‬‬ ‫‪c2 ‬‬
‫‪‬‬ ‫‪‬‬
‫‪‬‬ ‫‪‬‬
‫‪    0‬‬ ‫‪1 0‬‬ ‫‪0‬‬ ‫‪‬‬ ‫)‪(8.65‬‬
‫‪‬‬ ‫‪‬‬
‫‪ 0‬‬ ‫‪0 1‬‬ ‫‪0‬‬ ‫‪‬‬
‫‪‬‬ ‫‪V‬‬ ‫‪‬‬
‫‪ i‬‬ ‫‪‬‬
‫‪‬‬ ‫‪c‬‬ ‫‪1‬‬ ‫‪‬‬
‫‪0‬‬ ‫‪0‬‬
‫‪‬‬ ‫‪V2‬‬ ‫‪V‬‬ ‫‪2 ‬‬
‫‪ 1‬‬ ‫‪1‬‬ ‫‪‬‬
‫‪‬‬ ‫‪c2‬‬ ‫‪c 2 ‬‬
‫‪‬‬
‫ﻭﻴﻤﻜﻥ ﺍﻟﺘﺄﻜﺩ ﺒﺴﻬﻭﻟﺔ ﺃﻥ ﻫﺫﻩ ﺍﻟﻤﺼﻔﻭﻓﺔ ﺘﺤﻘﻕ ﺍﻟﺸﺭﻁ )‪ ،(8.62‬ﻨﻌﻤﻡ ﺍﻵﻥ‬
‫ﺘﻌﺭﻴﻑ ﺍﻟﻤﺘﺠﻪ ﺭﺒﺎﻋﻲ ﺍﻷﺒﻌﺎﺩ ﺒﻘﻭﻟﻨﺎ ﺃﻨﻪ ﻜل ﻤﺘﺠﻪ ‪ a ‬ﺘﺘﺤﻭل ﻤﺭﻜﺒﺎﺘﻪ‬
‫‪ a , a z , a y , a x‬ﻋﻨﺩ ﺘﺩﻭﻴﺭ ﺍﻟﻤﺤﺎﻭﺭ ﺍﻻﺤﺩﺍﺜﻴﺔ ﺒﻨﻔﺱ ﺍﻟﻘﺎﻨﻭﻥ ﺍﻟﺫﻱ ﺘﺘﺤﻭل ﻓﻴﻪ‬
‫ﻤﺭﻜﺒﺎﺕ ﺍﻟﻤﺘﺠﻪ ‪ r‬ﺃﻱ‪:‬‬
‫‪a     a ‬‬ ‫)‪(8.66‬‬
‫ﻭﺇﺫﺍ ﺍﻗﺘﺼﺭﻨﺎ ﻋﻠﻰ ﺍﻟﺩﻭﺭﺍﻥ ﻓﻲ ﺍﻟﻤﺴﺘﻭﻯ ) ‪ (x, ‬ﻓﺈﻥ ﺍﻟﻤﺭﻜﺒﺎﺕ ﺍﻟﺠﺩﻴﺩﺓ‬
‫ﺘﺤﻘﻕ ﺍﻟﻌﻼﻗﺎﺕ‪:‬‬
‫‪V‬‬
‫‪a ' x i‬‬ ‫‪a' ‬‬
‫‪ax ‬‬ ‫‪c‬‬ ‫‪, a y  a' y‬‬
‫‪2‬‬
‫‪V‬‬
‫‪1‬‬
‫‪c2‬‬
‫)‪(8.67‬‬
‫‪V‬‬
‫‪a '  i a ' x‬‬
‫‪a z  a' z , a  ‬‬ ‫‪c‬‬
‫‪V2‬‬
‫‪1‬‬
‫‪c2‬‬
‫ﻭﻟﻬﺫﻩ ﺍﻟﻤﺘﺠﻬﺎﺕ ﺭﺒﺎﻋﻴﺔ ﺍﻷﺒﻌﺎﺩ‪ ،‬ﻜﻤﺎ ﻟﻠﻤﺘﺠﻬﺎﺕ ﻓﻲ ﺍﻟﻔﺭﺍﻍ ﺍﻟﺜﻼﺜﻲ ﻨﻌﺭﻑ‬
‫ﺍﻟﺠﺩﺍﺀ ﺍﻟﻌﺩﺩﻱ‪:‬‬
‫‪ ‬‬
‫‪a  b   a . b  a b   c t e‬‬ ‫)‪(8.68‬‬

‫‪- 277 -‬‬

‫)‪Create PDF files without this message by purchasing novaPDF printer (http://www.novapdf.com‬‬
‫ﺤﻴﺙ ‪ c‬ﻜﻤﻴﺔ ﺴﻠﻤﻴﺔ‪ ،‬ﻭﻨﻘﻭل ﻫﻨﺎ ﺃﻥ ﺍﻟﻤﺘﺠﻬﻴﻥ ‪ a ‬ﻭ ‪ b ‬ﻤﺘﻌﺎﻤﺩﺍﻥ ﻓﻲ‬
‫ﺍﻟﻔﺭﺍﻍ ﺍﻟﺭﺒﺎﻋﻲ ﺇﺫﺍ ﻜﺎﻥ ﺠﺩﺍﺅﻫﻤﺎ ﺍﻟﻌﺩﺩﻱ ﻓﻲ ﻫﺫﺍ ﺍﻟﻔﺭﺍﻍ ﻴﺴﺎﻭﻱ ﺍﻟﺼﻔﺭ‪.‬‬
‫ﺇﻥ ﺃﻫﻡ ﺨﺎﺼﺔ ﻟﻠﻤﺘﺠﻬﺎﺕ ‪ a ‬ﺍﻟﻤﻌﺭﻓﺔ ﻓﻲ ﺍﻟﻔﺭﺍﻍ ﺍﻟﺭﺒﺎﻋﻲ ﻫﻲ ﺃﻥ‬
‫ﻤﺭﻜﺒﺎﺘﻬﺎ ﻜﻤﻴﺎﺕ ﺴﻠﻤﻴﺔ ﻭﺒﺎﻟﺘﺎﻟﻲ ﻻﺘﺘﻐﻴﺭ ﺒﺩﻭﺭﺍﻥ ﺍﻟﻤﺤﺎﻭﺭ ﻻﺤﺩﺍﺜﻴﺔ ﺃﻱ ﺃﻥ‪:‬‬
‫‪a  a   a x a x  a y a y  a z a z  a a   c t e‬‬ ‫)‪(8.69‬‬
‫ﻭﻴﻼﺤﻅ ﺃﻴﻀﺎﹰ ﺃﻥ ﻤﺭﺒﻊ ﻤﺘﺠﻪ ﻤﻌﺭﻑ ﻓﻲ ﺍﻟﻔﺭﺍﻍ ﺍﻟﺭﺒﺎﻋﻲ ﻻﻴﻜﻭﻥ ﻤﻭﺠﺒ ﺎﹰ‬
‫ﺒﺎﻟﻀﺭﻭﺭﺓ‪.‬‬
‫‪ -70‬ﺍﻟﺴﺮﻋﺔ ﻭﺍﻟﺘﺴﺎﺭﻉ ﰲ ﺍﻟﻔﺮﺍﻍ ﺍﻟﺮﺑﺎﻋﻲ ‪:‬‬
‫ﺘﻌﺭﻑ ﺴﺭﻋﺔ ﻤﺘﺤﺭﻙ ﻓﻲ ﺍﻟﻔﺭﺍﻍ ﺍﻟﺭﺒﺎﻋﻲ ﺒﺎﻟﻌﻼﻗﺔ‪:‬‬
‫‪dr‬‬
‫‪u ‬‬ ‫)‪(8.70‬‬
‫‪dt 0‬‬

‫‪V2‬‬
‫‪. dt 0  1 ‬‬ ‫ﺤﻴﺙ ‪ r‬ﻫﻭ ﻤﺘﺠﻪ ﻤﺭﻜﺒﺎﺘﻪ ‪ , z, y, x‬ﻭ ‪dt‬‬
‫‪c2‬‬
‫ﻤﻥ ﺍﻟﺴﻬل ﺍﻟﺤﺼﻭل ﻋﻠﻰ ﻤﺭﻜﺒﺎﺕ ‪ u ‬ﻭﺫﻟﻙ ﺒﺘﺒﺩﻴل ‪ dt 0‬ﺒﻘﻴﻤﺘﻬﺎ ﺤﻴﺙ‬
‫ﻨﺠﺩ‪:‬‬
‫‪dx‬‬ ‫‪dx‬‬ ‫‪vx‬‬
‫‪ux ‬‬ ‫‪‬‬ ‫‪‬‬
‫‪dt 0‬‬ ‫‪v2‬‬ ‫‪v2‬‬
‫‪1‬‬ ‫‪dt‬‬ ‫‪1‬‬
‫‪c2‬‬ ‫‪c2‬‬
‫)‪(8.71‬‬
‫‪vy‬‬ ‫‪vz‬‬ ‫‪ic‬‬
‫‪uy ‬‬ ‫‪, uz ‬‬ ‫‪, u ‬‬
‫‪2‬‬ ‫‪2‬‬
‫‪v‬‬ ‫‪v‬‬ ‫‪v2‬‬
‫‪1‬‬ ‫‪1‬‬ ‫‪1‬‬
‫‪c2‬‬ ‫‪c2‬‬ ‫‪c2‬‬
‫ﻭﻴﺒﺩﻭ ﻭﺍﻀﺤﺎﹰ ﺃﻥ ﺍﻟﻌﻼﻗﺎﺕ ﺍﻟﺴﺎﺒﻘﺔ ﺘﺅﻭل ﺇﻟﻰ ﻤﺭﻜﺒﺎﺕ ﺍﻟﺴﺭﻋﺔ ﺍﻟﻤﻌﺭﻭﻓﺔ‬
‫ﻓﻲ ﺍﻟﻤﻴﻜﺎﻨﻴﻙ ﺍﻟﺘﻘﻠﻴﺩﻱ ﻋﻨﺩﻤﺎ ‪ ، v  c‬ﺃﻤﺎ ﺍﻟﻤﺭﻜﺒﺔ ﺍﻟﺭﺍﺒﻌﺔ ﻓﻼﻴﻤﻜﻥ ﺃﻥ ﺘﺠﺩ‬
‫ﻤﺎﻨﺅﻭل ﺇﻟﻴﻪ ﻓﻲ ﺍﻟﻔﺭﺍﻍ ﺍﻟﺜﻼﺜﻲ ﺍﻻﻗﻠﻴﺩﻱ ﻷﻨﻬﺎ ﻋﻘﺩﻴﺔ‪ .‬ﻭﻤﻥ ﺍﻟﺴﻬل ﺍﻟﺒﺭﻫﺎﻥ ﺃﻥ‬
‫ﻤﺭﺒﻊ ﻤﺘﺠﻪ ﺍﻟﺴﺭﻋﺔ ﻴﺴﺎﻭﻱ ﻤﻘﺩﺍﺭﺍﹰ ﺜﺎﺒﺘﺎﹰ ﺃﻱ ﺃﻥ‪:‬‬
‫‪2‬‬
‫‪u‬‬ ‫)‪ u  u   u 2x  u 2y  u 2z  u 2   c 2  ivariant (8.72‬‬

‫ﺘﻌﺭﻑ ﺃﺨﻴﺭﺍﹰ ﻤﺘﺠﻪ ﺍﻟﺘﺴﺎﺭﻉ ﻓﻲ ﺍﻟﻔﺭﺍﻍ ﺍﻟﺭﺒﺎﻋﻲ ﺒﺎﻟﻌﻼﻗﺔ‪:‬‬

‫‪- 278 -‬‬

‫)‪Create PDF files without this message by purchasing novaPDF printer (http://www.novapdf.com‬‬
du 
W  (8.73)
dt 0
:‫ ﻓﻨﺠﺩ‬W ‫ﻟﻨﺤﺴﺏ ﻤﺭﻜﺒﺎﺕ‬
du x du x dt 1 d vx
Wx   
dt 0 dt dt 0 v2 dt v2
1 1
c2 c2
:‫ﻭﻤﻨﻪ‬
 
vx vx ( v .v ) 
Wx   (8.74)
v2 v2 2
1 c 2 (1  )
c2 c2
:‫ﻭﻜﺫﻟﻙ ﻨﺠﺩ‬
 
v y vy( v .v ) 
Wy  
v2 v2 2
1 c 2 (1  )
c2 c2 (8.75)
 
vz vz ( v . v ) 
Wz  
2
v 2 v2 2
1 c (1  )
c2 c2
:‫ﺃﻤﺎ ﺍﻟﻤﺭﻜﺒﺔ ﺍﻟﺭﺍﺒﻌﺔ ﻟﻠﺘﺴﺎﺭﻉ ﻓﻨﺤﺼل ﻋﻠﻴﻬﺎ ﺒﺎﻟﻁﺭﻴﻘﺔ ﻨﻔﺴﻬﺎ ﻭﻴﻜﻭﻥ‬
 
du  du  
i ( v .v )
(8.76)
dt 1 d ic
W    . 
dt 0 dt dt 0 v 2 dt c2 c v2 2
1 1 (1  )
c2 c2 c2
‫( ﺍﺸﺘﻘﺎﻗﺎﹰ ﻜﻠﻴﺎﹰ ﺒﺎﻟﻨﺴﺒﺔ ﻟﻠﺯﻤﻥ ﺍﻟﺨﺎﺹ‬8.72) ‫ﻭﺃﺨﻴﺭﺍﹰ ﻨﻼﺤﻅ ﺃﻨﻪ ﺒﺎﺸﺘﻘﺎﻕ‬
:‫ﻨﺤﺼل ﻋﻠﻰ ﺍﻟﻌﻼﻗﺔ ﺍﻟﻬﺎﻤﺔ‬
du y
(8.77)
du x du z du
ux  uy  uz  u   0
dt 0 dt 0 dt 0 dt 0
‫ ﻷﻥ ﺠﺩﺍﺅﻫﻤﺎ‬،‫ﺃﻱ ﺃﻥ ﺍﻟﺴﺭﻋﺔ ﻭﺍﻟﺘﺴﺎﺭﻉ ﻤﺘﻌﺎﻤﺩﺍﻥ ﻓﻲ ﺍﻟﻔﺭﺍﻍ ﺍﻟﺭﺒﺎﻋﻲ‬
.‫ ﻴﺴﺎﻭﻱ ﺍﻟﺼﻔﺭ‬،(8.68) ‫ ﻁﺒﻘﺎﹰ ﻟﻠﺘﻌﺭﻴﻑ‬،‫ﺍﻟﻌﺩﺩﻱ‬

- 279 -

Create PDF files without this message by purchasing novaPDF printer (http://www.novapdf.com)
- 280 -

Create PDF files without this message by purchasing novaPDF printer (http://www.novapdf.com)
‫ﻣﺴﺎﺋﻞ ﺍﻟﻔﺼﻞ ﺍﻟﺜﺎﻣﻦ‬

‫‪ -1‬ﺒﺭﻫﻥ ﺼﺤﺔ ﺍﻟﻌﻼﻗﺎﺕ )‪ (8.18‬ﺍﻨﻁﻼﻗﺎﹰ ﻤﻥ )‪.(8.18,a‬‬

‫‪ -2‬ﺘﺴﻴﺭ ﺴﻔﻴﻨﺔ ﻓﻀﺎﺌﻴﺔ ﺒﺴﺭﻋﺔ ‪ 300 m / sec‬ﻤﺎﻫﻭ ﺍﻟﺯﻤﻥ ﺍﻟﻼﺯﻡ ﻟﻜﻲ‬


‫ﻴﺨﺘﻠﻑ ﺘﻭﻗﻴﺕ ﺴﺎﻋﺔ ﻤﻭﺠﻭﺩﺓ ﻓﻴﻬﺎ ﻋﻥ ﺴﺎﻋﺔ ﻤﻭﺠﻭﺩﺓ ﻋﻠﻰ ﺍﻷﺭﺽ ﺒﻤﻘﺩﺍﺭ ﺜﺎﻨﻴﺔ‬
‫ﻭﺍﺤﺩﺓ؟‬

‫‪ -3‬ﺍﺫﺍ ﻜﺎﻨﺕ ‪ ‬ﻜﺘﻠﺔ ﻭﺍﺤﺩﺓ ﺍﻟﺤﺠﻭﻡ )ﺍﻟﻜﺘﻠﺔ ﺍﻟﻨﻭﻋﻴﺔ( ﻟﺠﺴﻡ ﺒﺎﻟﻨﺴﺒﺔ ﺍﻟﻰ‬
‫ﻤﺭﺍﻗﺏ ﺜﺎﺒﺕ ﻤﻊ ﺍﻟﺠﺴﻡ ﻓﺎﻭﺠﺩ ﻫﺫﻩ ﺍﻟﻜﺘﻠﺔ ﺍﻟﻨﻭﻋﻴﺔ ﻋﻨﺩﻤﺎ ﻴﺘﺤﺭﻙ ﺍﻟﺠﺴﻡ ﺒﺎﻟﻨﺴﺒﺔ‬
‫ﻟﻠﻤﺭﺍﻗﺏ ﺒﺴﺭﻋﺔ ‪.0,9 C‬‬
‫ﺃﻭﺠﺩ ﺍﻟﻜﺘﻠﺔ ﺍﻟﻨﻭﻋﻴﺔ ﻟﻌﻨﺼﺭ ﺍﻟﺫﻫﺏ ﻋﻨﺩﻤﺎ ﻴﺘﺤﺭﻙ ﺒﺎﻟﻨﺴﺒﺔ ﻟﻤﺭﺍﻗﺏ ﺒﺴﺭﻋﺔ‬
‫‪ 0,9C‬ﻤﻊ ﺍﻟﻌﻠﻡ ﺃﻥ ﻜﺘﻠﺘﻪ ﺍﻟﻨﻭﻋﻴﺔ ﻓﻲ ﺤﺎﻟﺔ ﺍﻟﺴﻜﻭﻥ ﺘﺴﺎﻭﻱ ‪.19,3 gr / cm 3‬‬

‫‪ -4‬ﻴﻁﻠﻕ ﺍﺴﻡ ﻅﺎﻫﺭﺓ ﺩﻭﺒﻠﺭ ﻋﻠ ﻰ ﺍﻟﺘﻐﻴﺭ ﺍﻟﺤﺎﺼل ﻓﻲ ﺘﻭﺍﺘﺭ ﺃﻤﻭﺍﺝ ﺤﺭﻜﺔ‬


‫ﺍﻫﺘﺯﺍﺯﻴﺔ ﻨﺘﻴﺠﺔ ﺤﺭﻜﺔ ﺍﻟﻤﻨﺒﻊ ﺍﻟﺒﺎﻋﺙ ﻟﻬﺎ ﺃﻭ ﺤﺭﻜﺔ ﺍﻟﻤﺭﺍﻗﺏ ﺍﻟﺫﻱ ﻴﺘﻠﻘﺎﻫﺎ‪.‬‬
‫ﺍﺩﺭﺱ ﻫﺫﻩ ﺍﻟﻅﺎﻫﺭﺓ ﺒﺎﻟﻨﺴﺒﺔ ﻟﻸﻤﻭﺍﺝ ﺍﻟﻀﻭﺌﻴﺔ ﺍﻨﻁﻼﻗﺎﹰ ﻤﻥ ﺘﺤﻭﻴﻼﺕ ﻏﺎﻟﻴﻠﻲ‬
‫ﻤﻤﻴﺯ ﺍﹰ ﺠﻤﻴﻊ ﺤﺎﻻﺕ ﺍﻗﺘﺭﺍﺏ ﻭﺍﺒﺘﻌﺎﺩ ﺍﻟﻤﻨﺒﻊ ﻭﺍﻟﻤﺭﺍﻗﺏ ﻋﻥ ﺒﻌﻀﻬﻤﺎ‪.‬‬

‫‪ -5‬ﻅﺎﻫﺭﺓ ﺩﻭﺒﻠﺭ ﻓﻲ ﺍﻟﻤﻴﻜﺎﻨﻴﻙ ﺍﻟﻨﺴﺒﻲ‪:‬‬


‫ﺃ‪ -‬ﻅﺎﻫﺭﺓ ﺩﻭﺒﻠﺭ ﺍﻟﻁﻭﻻﻨﻴﺔ‪ :‬ﻴﻨﺒﻌﺙ ﻀﻭﺀ ﺘﻭﺍﺘﺭﻩ ‪ ‬ﻤﻥ ﻤﻨﺒﻊ ﻀﻭﺌﻲ ﺴﺎﻜﻥ‬
‫‪ I‬ﻭﻴﻘﻴﺱ ﻤﺭﺍﻗﺏ ﻤﺘﺤﺭﻙ ﺒﺴﺭﻋﺔ ‪ v‬ﺒﺎﺘﺠﺎﻩ ﺍﻨﺘﺸﺎﺭ ﺍﻟﻀﻭﺀ ﻤﺒﺘﻌﺩﺍﹰ ﻋﻥ ﺍﻟﻤﻨﺒﻊ ‪،I‬‬
‫ﺘﻭﺍﺘﺭﺍﹰ ' ‪ ‬ﻟﻠﻀﻭﺀ ﺍﻟﺼﺎﺩﺭ ﻋﻥ ﻫﺫﺍ ﺍﻟﻤﻨﺒﻊ‪ ،‬ﻓﺈﺫﺍ ﺍﻋﺘﺒﺭﻨﺎ ﺃﻥ ‪ I‬ﻫﻭ ﺴﺎﻋﺔ ﺘﺩﻕ ‪‬‬
‫ﺩﻗﺔ ﻜل ﺜﺎﻨﻴﺔ ﻭﺘﺼﺩﺭ ﻭﻤﻀﺔ ﻀﻭﺌﻴﺔ ﻓﻲ ﻜل ﻤﻨﻬﺎ ﻓﺒﺭﻫﻥ ﺃﻥ‪:‬‬

‫‪- 281 -‬‬

‫)‪Create PDF files without this message by purchasing novaPDF printer (http://www.novapdf.com‬‬
‫‪V‬‬
‫‪1‬‬
‫'‪  ‬‬ ‫‪c‬‬
‫‪V‬‬
‫‪1‬‬
‫‪c‬‬
‫ﺏ‪ -‬ﻅﺎﻫﺭﺓ ﺩﻭﺒﻠﺭ ﺍﻟﻌﺭﻀﺎﻨﻴﺔ‪ :‬ﺍﺫﺍ ﺤﺩﺜﺕ ﺍﻟﺤﺭﻜﺔ ﺒﺎﺘﺠﺎﻩ ﻤﺘﻌﺎﻤﺩ ﻤﻊ ﺍﻨﺘﺸﺎﺭ‬
‫ﺍﻟﻀﻭﺀ ﻭﺒﺎﻟﺴﺭﻋﺔ ﻨﻔﺴﻬﺎ ‪ v‬ﻓﺒﺭﻫﻥ ﺃﻥ‪:‬‬
‫‪V2‬‬
‫‪'   1 ‬‬
‫‪c2‬‬

‫‪ -6‬ﺘﺤﻠﻕ ﻁﺎﺌﺭﺓ ﻁﻭﻟﻬﺎ ‪ 10 m‬ﺒﺴﺭﻋﺔ ﻗﺩﺭﻫﺎ ‪ ،300 m/sec‬ﻤﺎﻫﻭ ﻁﻭل ﻫﺫﻩ‬


‫ﺍﻟﻁﺎﺌﺭﺓ ﺍﻟﺫﻱ ﻴﻘﻴﺴﻪ ﻤﺭﺍﻗﺏ ﻋﻠﻰ ﺍﻷﺭﺽ ﻭﺒﻌﺩ ﻜﻡ ﻤﻥ ﺍﻟﺯﻤﻥ ﺘﻘﺼﺭ ﺴﺎﻋﺔ ﺍﻟﻁﻴﺎﺭ‬
‫‪ 10 3 sec‬ﻋﻥ ﺴﺎﻋﺔ ﻤﺭﺍﻗﺏ ﻋﻠﻰ ﺍﻷﺭﺽ‪.‬‬

‫‪ -7‬ﻴﺒﺘﻌﺩ ﻨﺠﻡ ﻋﻥ ﺍﻷﺭﺽ ﺒﺴﺭﻋﺔ ‪ 50 km / sec‬ﻭﻴﻁﻠﻕ ﻀﻭﺀﺍﹰ ﻁﻭل‬


‫ﻤﻭﺠﺘﻪ ‪ ، 6563 A ‬ﻤﺎﻫﻭ ﻤﻘﺩﺍﺭ ﺘﻐﻴﺭ ﻁﻭل ﻤﻭﺠﺔ ﻫﺫﺍ ﺍﻟﻀﻭﺀ ﻋﻨﺩﻤﺎ ﻴﻘﺎﺱ ﻋﻠﻰ‬
‫ﺍﻷﺭﺽ؟‪.‬‬

‫‪ -8‬ﻴﻤﻴل ﺍﻟﻘﻀﻴﺏ ‪ AB‬ﺍﻟﺫﻱ ﻁﻭﻟﻪ ‪ L‬ﺒﺯﺍﻭﻴﺔ ‪ ‬ﻋﻠﻰ ﺍﻟﻤﺤﻭﺭ ‪ ox‬ﻓﻲ‬


‫ﺍﻟﺠﻤﻠﺔ ‪ ،K‬ﺍﺤﺴﺏ ﻁﻭل ﻫﺫﺍ ﺍﻟﻘﻀﻴﺏ '‪ L‬ﻭﺯﺍﻭﻴﺔ ﻤﻴﻼﻨﻪ '‪ ‬ﻋﻠﻰ '‪ ox‬ﻓﻲ ﺍﻟﺠﻤﻠﺔ‬
‫'‪ ،K‬ﺍﻟﺘﻲ ﺘﺘﺤﺭﻙ ﺒﺴﺭﻋﺔ ‪ V‬ﺒﺎﻟﻨﺴﺒﺔ ﻟﻠﺠﻤﻠﺔ ‪.K‬‬

‫‪ -9‬ﺒﺭﻫﻥ ﺃﻥ ﻋﻨﺼﺭ ﺍﻟﺤﺠﻡ ﻓﻲ ﺍﻟﻔﺭﺍﻍ ﺍﻟﺭﺒﺎﻋﻲ ﻫﻭ ﻻﻤﺘﻐﻴﺭ ﻓﻴﺯﻴﺎﺌﻲ ﺃﻱ‬


‫ﺃﻥ‪:‬‬
‫'‪x y z   x' y' z' ‬‬

‫‪ -10‬ﻨﻔﺭﺽ ﻨﺠﻤﺎﹰ ﻤﻭﺠﻭﺩﺍﹰ ﻓﻲ ﻤﺴﺘﻭﻯ ﺍﻷﻓﻕ ﻭﻋﻠﻰ ﻤﺴﺎﻓﺔ ‪ 10 6‬ﺴﻨﺔ‬


‫ﻀﻭﺌﻴﺔ ﻭﻟﻨﻔﺭﺽ ﺃﻥ ﻤﺭﺍﻗﺒﺎﹰ ﻤﻭﺠﻭﺩﺍﹰ ﻋﻠﻰ ﺍﻷﺭﺽ ﻴﺭﻯ ﻓﻲ ﻨﻔﺱ ﺍﻟﻠﺤﻅﺔ‬

‫‪- 282 -‬‬

‫)‪Create PDF files without this message by purchasing novaPDF printer (http://www.novapdf.com‬‬
‫ﻅﺎﻫﺭﺘﻴﻥ ﺍﺤﺩﺍﻫﻤﺎ ﺘﺤﺩﺙ ﻋﻠﻰ ﺴﻁﺢ ﺍﻷﺭﺽ ﻭﺍﻷﺨﺭﻯ ﻋﻠﻰ ﺴﻁﺢ ﺍﻟﻨﺠﻡ‪ .‬ﺍﺤﺴﺏ‬
‫ﺍﻟﻔﺎﺼل ﺍﻟﺯﻤﻨﻲ ﺒﻴﻥ ﺍﻟﻅﺎﻫﺭﺘﻴﻥ ﺒﺎﻟﻨﺴﺒﺔ ﻹﻨﺴﺎﻥ ﻴﺘﺤﺭﻙ ﺒﺴﺭﻋﺔ ‪6 km / sec‬‬
‫ﺒﺎﺘﺠﺎﻩ ﺍﻟﻨﺠﻡ‪.‬‬

‫‪ -11‬ﺘﺘﺤﺭﻙ ﻤﺴﻁﺭﺘﺎﻥ‪ ،‬ﻁﻭل ﻜل ﻤﻨﻬﻤﺎ ‪ ‬ﻓﻲ ﺍﻟﺠﻤﻠﺔ ﺍﻟﺴﺎﻜﻨﺔ‪ ،‬ﺒﺴﺭﻋﺔ ‪v‬‬


‫ﻟﻜل ﻤﻨﻬﻤﺎ ﺒﺎﺘﺠﺎﻫﻴﻥ ﻤﺨﺘﻠﻔﻴﻥ‪ ،‬ﺍﻭﺠﺩ ﻁﻭل ﻜل ﻤﻨﻬﻤﺎ ﻓﻲ ﺠﻤﻠﺔ ﺍﺤﺩﺍﺜﻴﺔ ﻤﺭﺘﺒﻁﺔ‬
‫ﺒﺎﻷﺨﺭﻯ‪.‬‬

‫‪ -12‬ﻴﻨﻁﻠﻕ ﺒﺭﻭﺘﻭﻥ ﻤﻥ ﺍﻟﺸﻤﺱ ﺒﺎ ﺘﺠﺎﻩ ﺍﻷﺭﺽ ﻗﺎﻁﻌ ﺎﹰ ﻤﺴﺎﻓﺔ‬


‫‪ .‬ﻜﻡ ﺘﺒﻠﻎ ﻫﺫﻩ ﺍﻟﻤﺴﺎﻓﺔ ﺒﺎﻟﻨﺴﺒﺔ ﻟﻤﺭﺍﻗﺏ ﻤﺭﺘﺒﻁ‬ ‫‪ 1,5.10 8 km‬ﺒﺴﺭﻋﺔ‬
‫‪4c‬‬
‫‪5‬‬
‫ﺒﺎﻟﺒﺭﻭﺘﻭﻥ ﻭﻤﺎﻫﻭ ﺍﻟﺯﻤﻥ ﺍﻟﻼﺯﻡ ﻟﻘﻁﻌﻬﺎ ﺒﺎﻟﻨﺴﺒﺔ ﻟﻤﺭﺍﻗﺒﻴﻥ ﺃﺤﺩﻫﻤﺎ ﻤﺭﺘﺒﻁ ﺒﺎﻷﺭﺽ‬
‫ﻭﺍﻵﺨﺭ ﻤﺭﺘﺒﻁ ﺒﺎﻟﺒﺭﻭﺘﻭﻥ؟‪.‬‬

‫‪- 283 -‬‬

‫)‪Create PDF files without this message by purchasing novaPDF printer (http://www.novapdf.com‬‬
- 284 -

Create PDF files without this message by purchasing novaPDF printer (http://www.novapdf.com)
‫‪‬‬
‫‪ ‬‬
‫‪‬‬
‫ﻣﻴﻜﺎﻧﻴﻚ ﺍﻟﻨﻈﺮﻳﺔ ﺍﻟﻨﺴﺒﻴﺔ‬

‫‪ -71‬ﻛﻤﻴﺔ ﺍﳊﺮﻛﺔ ﰲ ﺍﻟﻔﺮﺍﻍ ﺍﻟﺮﺑﺎﻋﻲ ‪:‬‬


‫ﺇﻥ ﻤﻔﻬﻭﻡ ﺍﻟﻨﻘﻁﺔ ﺍﻟﻤﺎﺩﻴﺔ ﻓﻲ ﺍﻟﻤﻴﻜﺎﻨﻴﻙ ﻴﻌﻨﻲ ﺠﺴﻤﺎﹰ ﺃﺒﻌﺎﺩﻩ ﺼﻐﻴﺭﺓ ﺠﺩ ﺍﹰ‬
‫ﺒﺤﻴﺙ ﻴﻤﻜﻥ ﺍﻫﻤﺎﻟﻬﺎ ﻭﻫﺫﺍ ﻤﺎﻨﺴﻤﻴﻪ ﺠﺴﻴﻤﺎﹰ‪.‬‬
‫ﺍﺫﺍ ﺘﺤﺭﻙ ﺠﺴﻴﻡ ﺒﺤﺭﻜﺔ ﻤﻨﺘﻅﻤﺔ ﻓﻲ ﺍﺤﺩﻯ ﺍﻟﺠﻤل ﺍﻟﻌﻁﺎﻟﻴﺔ ﻓﺈﻥ ﺍﻟﺤﺭﻜﺔ‬
‫ﺴﺘﺒﻘﻰ ﻤﻨﺘﻅﻤﺔ ﻋﻨﺩ ﺍﻻﻨﺘﻘﺎل ﺍﻟﻰ ﺠﻤﻠﺔ ﺃﺨﺭﻯ ﺒﻭﺍ ﺴﻁﺔ ﺘﺤﻭﻴﻼﺕ ﻟﻭﺭﻨﺘﺯ ﻭﻫﺫﺍ‬
‫ﻤﺎﻴﺅﻜﺩ ﺃﻥ ﻗﺎﻨﻭﻥ ﺍﻟﻌﻁﺎﻟﺔ ﻫﻭ ﻻﻤﺘﻐﻴﺭ ﻨﺴﺒﻲ‪ .‬ﻭﻴﻤﻜﻥ ﺍﻟﺘﺄﻜﺩ ﻤﻥ ﺫﻟﻙ ﺒﺴﻬﻭﻟﺔ‬
‫‪d2x‬‬
‫ﺜﻡ ﺒﺘﺒﺩﻴل ‪ x‬ﺒﻘﻴﻤﺘﻬﺎ ﺒﺩﻻﻟﺔ '‪ x‬ﻤﻥ )‪ (8.18b‬ﻓﻨﺤﺼل ﻋﻠﻰ‬ ‫ﺒﻭﻀﻊ‬
‫‪dt 2‬‬
‫'‪d 2 x‬‬
‫‪ .‬ﻭﻟﻜﻥ ﻤﻌﺎﺩﻻﺕ ﺍﻟﺤﺭﻜﺔ ﺍﻟﻤﻁﺒﻘﺔ ﻓﻲ ﺍﻟﻤﻴﻜﺎﻨﻴﻙ ﺍﻟﺘﻘﻠﻴﺩﻱ ﺘﺘﻐﻴﺭ ﻋﻨﺩ‬ ‫‪0‬‬
‫‪dt' 2‬‬
‫ﺍﻻﻨﺘﻘﺎل ﻤﻥ ﺠﻤﻠﺔ ﻋﻁﺎﻟﻴﺔ ﺇﻟﻰ ﺃﺨﺭﻯ ﺒﻭﺍ ﺴﻁﺔ ﺘﺤﻭﻴﻼﺕ ﻟﻭﺭﻨﺘﺯ )ﻤﻊ ﺍﻟﻌﻠﻡ ﺃﻨﻬﺎ‬
‫ﻻﺘﺘﻐﻴﺭ ﻋﻨﺩ ﺘﻁﺒﻴﻕ ﺘﺤﻭﻴﻼﺕ ﻏﺎﻟﻴﻠﻲ ﻋﻠﻴﻬﺎ – ﺍﻨﻅﺭ ﺍﻟﻔﺼل ﺍﻟﺴﺎﺒﻕ(‪ .‬ﻭﻴﻤﻜﻥ‬
‫ﺍﻟﺘﺄﻜﺩ ﻤﻥ ﺫﻟﻙ ﺒﺴﻬﻭﻟﺔ ﺒﻜﺘﺎﺒﺔ ﻤﺴﻘﻁ ﻤﻌﺎﺩﻟﺔ ﺍﻟﺤﺭﻜﺔ ﻋﻠﻰ ﺍﻟﻤﺤﻭﺭ ‪ ox‬ﻤﺜﻼﹰ‪،‬‬
‫‪md 2 x‬‬
‫‪ Fx ‬ﺜﻡ ﺘﺒﺩﻴل ‪ x‬ﻭﻤﺸﺘﻘﺎﺘﻬﺎ ﺒﺩﻻﻟﺔ '‪ x‬ﻭ '‪ t‬ﻤﻥ ﺘﺤﻭﻴﻼﺕ ﻟﻭﺭﻨﺘﺯ ﻓﻨﺠﺩ‬
‫‪dt 2‬‬
‫'‪d 2 x‬‬
‫‪ ، F' x  m‬ﻭﻟﻬﺫﺍ‪ ،‬ﻁﺒﻘﺎﹰ ﻟﻠﻔﺭﻀﻴﺔ‬ ‫ﺃﻨﻨﺎ ﻻﻨﺤﺼل ﻋﻠﻰ ﺼﻴﻐﺔ ﻤﻥ ﺍﻟﺸﻜل‬
‫‪dt ' 2‬‬
‫ﺍﻟﺜﺎﻨﻴﺔ‪ ،‬ﻻﻴﻤﻜﻥ ﺍﺴﺘﺨﺩﺍﻡ ﻫﺫﻩ ﺍﻟﻤﻌﺎﺩﻟﺔ ﻓﻲ ﺍﻟﻨﻅﺭﻴﺔ ﺍﻟﻨﺴﺒﻴﺔ‪ ،‬ﻭﻻﺒﺩ ﻤﻥ ﺍﻟﺒﺤﺙ ﻋﻥ‬
‫ﺼﻴﻐﺔ ﺠﺩﻴﺩﺓ ﻟﻬﺎ‪ ،‬ﻭﻟﺘﺤﻘﻴﻕ ﺫﻟﻙ ﻨﻠﺠﺄ ﺇﻟﻰ ﺍﻟﻔﺭﺍﻍ ﺍﻟﺭﺒﺎﻋﻲ ﻭﻨﻔﺭﺽ ﺠﺴﻴﻤﺎﹰ ﻜﺘﻠﺘﺔ‬
‫ﺍﻟﺴﻜﻭﻨﻴﺔ ﻟﻪ ‪ m‬ﻭﻨﺅﻜﺩ ﻋﻠﻰ ﻜﺘﻠﺔ ﺍﻟﺴﻜﻭﻥ ﻫﺫﻩ ﻷﻨﻬﺎ ﻜﻤﺎ ﺴﻨﺭﻯ ﻓﻴﻤﺎ ﺒﻌﺩ‪ ،‬ﺘﻌﺘﺒﺭ‬

‫‪- 285 -‬‬

‫)‪Create PDF files without this message by purchasing novaPDF printer (http://www.novapdf.com‬‬
‫ﺼﻔﺔ ﺃﺴﺎﺴﻴﺔ ﻤﻥ ﺼﻔﺎﺕ ﺍﻟﺠﺴﻴﻤﺎﺕ‪ ،‬ﻓﻬﻲ ﺍﺤﺩﻯ ﻤﻴﺯﺍﺕ ﺍﻟﺠﺴﻴﻡ‪ ،‬ﺍﻟﺸﺨﺼﻴﺔ‪ ،‬ﺃﻭ‬
‫ﺍﺤﺩ ﻋﻨﺎﺼﺭ ﻫﻭﻴﺘﻪ‪ ،‬ﻭﻻﻴﻤﻜﻥ ﻤﻁﻠﻘﺎﹰ ﺃﻥ ﻴﺸﺘﺭﻙ ﺠﺴﻴﻤﺎﻥ ﻓﻲ ﻨﻔﺱ ﺍﻟﻜﺘﻠﺔ‪ ،‬ﻭﻨﻌﺭﻑ‬
‫ﻜﻤﻴﺔ ﺍﻟﺤﺭﻜﺔ ﻓﻲ ﺍﻟﻔﺭﺍﻍ ﺍﻟﺭﺒﺎﻋﻲ ﺒﺎﻟﻌﻼﻗﺔ‪:‬‬
‫‪P  m u ‬‬ ‫)‪(9.1‬‬
‫ﺤﻴﺙ ‪ u ‬ﻫﻲ ﺴﺭﻋﺔ ﺍﻟﺠﺴﻴﻡ ﻓﻲ ﺍﻟﻔﺭﺍﻍ ﺍﻟﺭﺒﺎﻋﻲ ﺍﻟﻤﻌﺭﻓﺔ ﻓﻲ ﺍﻟﻔﺼل‬
‫ﺍﻟﺴﺎﺒﻕ )ﺍﻟﻌﻼﻗﺔ )‪ ، ((8.70‬ﻭﺒﺎﻻﺴﻘﺎﻁ ﻋﻠﻰ ﺍﻟﻤﺤﺎﻭﺭ ﺍﻷﺭﺒﻌﺔ ﻟﻠﻔﺭﺍﻍ ﺍﻟﺭﺒﺎﻋﻲ‬
‫ﻨﺤﺼل ﻋﻠﻰ ﺍﻟﻤﺭﻜﺒﺎﺕ ﺍﻷﺭﺒﻊ ﺍﻟﺘﺎﻟﻴﺔ‪:‬‬
‫‪m vx‬‬ ‫‪m vy ‬‬
‫‪Px  m u x ‬‬ ‫‪, Py  m u y ‬‬
‫‪‬‬
‫‪v2‬‬ ‫‪v2 ‬‬
‫‪1‬‬ ‫‪1‬‬ ‫‪‬‬
‫‪c2‬‬
‫‪m vz‬‬
‫‪c 2 ‬‬ ‫)‪(9.2‬‬
‫‪imC ‬‬
‫‪Pz  m u z ‬‬ ‫‪, P  m u  ‬‬
‫‪v2‬‬ ‫‪v 2 ‬‬
‫‪1‬‬ ‫‪1‬‬
‫‪c2‬‬ ‫‪c 2 ‬‬
‫ﻭﻴﺒﺩﻭ ﻭﺍﻀﺤﺎﹰ ﺃﻥ ﻜﻼﹰ ﻤﻥ ﻫﺫﻩ ﺍﻟﻤﺭﻜﺒﺎﺕ ﻴﺅﻭل ﺇﻟﻰ ﻤﺎ ﻴﻘﺎﺒﻠﻪ ﻓﻲ ﺍﻟﻨﻅﺭﻴﺔ‬
‫ﺍﻟﻜﻼﺴﻴﻜﻴﺔ ﺃﻱ ﺇﻟﻰ ‪ mv z , mv y , mv x‬ﻋﻨﺩﻤﺎ ‪ v  c‬ﻋﻠﻰ ﺍﻟﺘﺭﺘﻴﺏ ﺃﻤﺎ ﺍﻟﻤﺭﻜﺒﺔ‬
‫ﺍﻟﺭﺍﺒﻌﺔ ﻓﻠﻴﺱ ﻟﻬﺎ ﻤﺎ ﻴﻘﺎﺒﻠﻬﺎ ﻓﻲ ﺍﻟﻔﺭﺍﻍ ﺍﻻﻗﻠﻴﺩﻱ ﻷﻨﻬﺎ ﻋﻘﺩﻴﺔ ﺼﺭﻓﺔ‪.‬‬
‫‪ -72‬ﻣﻌﺎﺩﻟﺔ ﺍﳊﺮﻛﺔ ﰲ ﻣﻴ ﻜﺎﻧﻴﻚ ﺍﻟﻨﻈﺮﻳﺔ ﺍﻟﻨﺴﺒﻴﺔ ‪:‬‬
‫ﻤﻥ ﺍﻟﻁﺒﻴﻌﻲ ﺍﻵﻥ ﺘﻌﻤﻴﻡ ﻤﻌﺎﺩﻟﺔ ﺍﻟﺤﺭﻜﺔ ﺒﺎﻟﺸﻜل‪:‬‬
‫‪dp ‬‬
‫)‪mu   F (  1, 2, 3, 4  x, y, z,  ) (9.3‬‬
‫‪d‬‬
‫‪‬‬
‫‪dt 0‬‬ ‫‪dt 0‬‬
‫ﺤﻴﺙ ‪ F‬ﻤﺘﺠﻪ ﻤﻌﺭﻑ ﻓﻲ ﺍﻟﻔﺭﺍﻍ ﺍﻟﺭﺒﺎﻋﻲ ﻴﺴﻤﻰ ﺍﻟﻘﻭﺓ ﺭﺒﺎﻋﻴﺔ ﺍﻷﺒﻌﺎﺩ‪ ،‬ﺃﻭ‬
‫ﻗﻭﺓ ﻤﻴﻨﻜﻭﻓﺴﻜﻲ‪ .‬ﻟﻨﻜﺘﺏ ﻤﻌﺎﺩﻟﺔ ﺍﻟﺤﺭﻜﺔ ﻋﻠﻰ ﺍﻟﻤﺤﻭﺭ ‪ ox‬ﺍﻋﺘﻤﺎﺩﺍﹰ ﻋﻠﻰ )‪(9.3‬‬
‫ﻓﻨﺠﺩ‪:‬‬
‫‪dp x‬‬ ‫‪1‬‬ ‫‪d‬‬ ‫‪mv x‬‬
‫‪‬‬ ‫‪ Fx‬‬
‫‪dt 0‬‬ ‫‪v 2 dt‬‬ ‫‪v‬‬ ‫‪2‬‬
‫‪1‬‬ ‫‪1‬‬
‫‪2‬‬
‫‪c‬‬ ‫‪c2‬‬
‫ﺃﻭ ﺒﺎﻟﺸﻜل‪:‬‬

‫‪- 286 -‬‬

‫)‪Create PDF files without this message by purchasing novaPDF printer (http://www.novapdf.com‬‬
‫‪v2‬‬
‫)‪(9.4‬‬
‫‪d‬‬ ‫‪mv x‬‬
‫‪ Fx 1 ‬‬
‫‪dt‬‬ ‫‪v‬‬ ‫‪2‬‬ ‫‪c2‬‬
‫‪1‬‬
‫‪c2‬‬
‫ﺤﻴﺙ ‪ Fx‬ﻫﻲ ﻤﺭﻜﺒﺔ ﺍﻟﻘﻭﺓ ﺍﻟﻤﺅﺜﺭﺓ ﻓﻲ ﺍﻟﺠﺴﻴﻡ‪ ،‬ﻋﻠﻰ ﺍﻟﻤﺤﻭﺭ ‪ ox‬ﻓﻲ‬
‫ﺍﻟﻔﺭﺍﻍ ﺍﻟﺭﺒﺎﻋﻲ ﻭﻻﻴﺠﺎﺩ ﺍﻟﻌﻼﻗﺔ ﺒﻴﻥ ‪ Fx‬ﻭ ‪) Fx‬ﻤﺭﻜﺒﺔ ﺍﻟﻘﻭﺓ ﻓﻲ ﺍﻟﻔﺭﺍﻍ ﺍﻟﺜﻼﺜﻲ‬
‫ﻋﻠﻰ ﺍﻟﻤﺤﻭﺭ ‪ (ox‬ﺘﺘﻁﻠﺏ ﺃﻥ ﻴﺘﺤﻭل ﺍﻟﻁﺭﻑ ﺍﻟﺜﺎﻨﻲ ﻋﻨﺩﻤﺎ ‪ v  c‬ﺍﻟﻰ ﺍﻟﻘﻭﺓ ‪F‬‬
‫ﻭﻟﻬﺫﺍ ﻴﺠﺏ ﺃﻥ ﻴﻜﻭﻥ‪:‬‬
‫‪v2‬‬
‫‪Fx  Fx 1 ‬‬ ‫)‪(9.5‬‬
‫‪c2‬‬
‫ﻭﻋﻨﺩﺌﺫ‪ ‬ﺘﻭﻀﻊ ﺍﻟﻤﻌﺎﺩﻟﺔ )‪ (9.4‬ﺒﺎﻟﺸﻜل ﺍﻟﺘﺎﻟﻲ‪:‬‬
‫‪mv x‬‬
‫)‪(9.6‬‬
‫‪d‬‬
‫‪ Fx‬‬
‫‪dt‬‬ ‫‪v‬‬ ‫‪2‬‬
‫‪1‬‬
‫‪c2‬‬
‫ﻭﻴﺒﺩﻭ ﺒﻭﻀﻭﺡ ﺃﻥ ﺍﻟﻤﻌﺎﺩﻟﺔ )‪ (9.6‬ﺘﺅﻭل ﺇﻟﻰ ﻤﻌﺎﺩﻟﺔ ﻨﻴﻭﺘﻥ ﺍﻟﻤﻘﺎﺒﻠﺔ‬
‫‪ mx   Fx‬ﻋﻨﺩﻤﺎ ‪. v  c‬‬
‫ﺃﻤﺎ ﻤﻌﺎﺩﻟﺘﺎ ﺍﻟﺤﺭﻜﺔ ﻋﻠﻰ ﺍﻟﻤﺤﻭﺭﻴﻥ ‪ oy‬ﻭ ‪ oz‬ﻓﺘﻭﻀﻌﺎﻥ ﺒﺸﻜل ﻤﺸﺎﺒﻪ ﺤﻴﺙ‬
‫ﻨﺠﺩ‪:‬‬
‫‪mv y‬‬ ‫‪mv z‬‬
‫)‪(9.7‬‬
‫‪d‬‬ ‫‪d‬‬
‫‪ Fy ,‬‬ ‫‪ Fz‬‬
‫‪dt‬‬ ‫‪v2‬‬ ‫‪dt‬‬ ‫‪v2‬‬
‫‪1‬‬ ‫‪1‬‬
‫‪c2‬‬ ‫‪c2‬‬
‫ﺒﻘﻴﻤﺘﻬﺎ‬ ‫‪P‬‬ ‫ﻭﻟﻠﺤﺼﻭل ﻋﻠﻰ ﻤﻌﺎﺩﻟﺔ ﺍﻟﺤﺭﻜﺔ ﻋﻠﻰ ﺍﻟﻤﺤﻭﺭ ﺍﻟﺘﺨﻴﻠﻲ ‪ ‬ﻨﺒﺩل‬
‫ﻤﻥ )‪ (9.2‬ﻓﻨﺠﺩ‪:‬‬
‫‪v2‬‬
‫)‪(9.8‬‬
‫‪d‬‬ ‫‪icm‬‬
‫‪ F 1 ‬‬
‫‪dt‬‬ ‫‪v‬‬ ‫‪2‬‬ ‫‪c2‬‬
‫‪1‬‬
‫‪c2‬‬
‫ﻟﻨﺒﺤﺙ ﻋﻥ ﺍﻟﻤﻌﻨﻰ ﺍﻟﻔﻴﺯﻴﺎﺌﻲ ﻟﻤﺭﻜﺒﺔ ﺍﻟﻘﻭﺓ ﻋﻠﻰ ﺍﻟﻤﺤﻭﺭ ‪ ‬ﻟﺫﺍ ﻨﻀﺭﺏ‬
‫ﺜﻡ ﻨﺠﻤﻊ ﻓﻨﺠﺩ ﺒﺎﻻﻋﺘﻤﺎﺩ ﻋﻠﻰ )‪:(8.77‬‬ ‫‪u‬‬ ‫)‪ (9.3‬ﻋﺩﺩﻴﺎﹰ‬

‫‪- 287 -‬‬

‫)‪Create PDF files without this message by purchasing novaPDF printer (http://www.novapdf.com‬‬
‫‪dmu‬‬
‫‪u‬‬ ‫)‪ F u   Fx u x  Fy u y  Fz u z  F u   0 (9.9‬‬
‫‪dt 0‬‬
‫ﻭﺍﺫﺍ ﺒﺩﻟﻨﺎ ﺍﻟﻤﻘﺎﺩﻴﺭ ‪ F‬ﻭ ‪ u ‬ﺒﻘﻴﻤﻬﺎ ﻤﻥ ﺍﻟﻌﻼﻗﺎﺕ ﺍﻟﺴﺎﺒﻘﺔ ﻓﺈﻨﻨﺎ ﻨﺤﺼل‬
‫ﻋﻠﻰ ﺍﻟﻤﻌﺎﺩﻟﺔ‪:‬‬
‫‪vx‬‬ ‫‪Fy v y‬‬ ‫‪Fz v z‬‬
‫)‪(9.10‬‬
‫‪ic‬‬
‫‪Fx‬‬ ‫‪‬‬ ‫‪‬‬ ‫‪ F‬‬ ‫‪0‬‬
‫‪2‬‬ ‫‪2‬‬ ‫‪2‬‬
‫‪1‬‬
‫‪v‬‬
‫‪1‬‬
‫‪v‬‬
‫‪1‬‬
‫‪v‬‬ ‫‪v2‬‬
‫‪1‬‬
‫‪c2‬‬ ‫‪c2‬‬ ‫‪c2‬‬ ‫‪c2‬‬
‫‪ ‬‬
‫ﻭﻤﻥ ﺍﻟﻭﺍﻀﺢ ﺃﻥ ﺍﻟﺤﺩﻭﺩ ﺍﻟﺜﻼﺜﺔ ﺍﻷﻭﻟﻰ ﻟﻴﺴﺕ ﺇﻻ ﺍﻟﺠﺩﺍﺀ ﺍﻟﻌﺩﺩﻱ ‪F . v‬‬

‫ﻓﻲ ﺍﻟﻔﺭﺍﻍ ﺍﻟﺜﻼﺜﻲ‪ ،‬ﻭﻋﻨﺩﺌﺫ‪ ‬ﻨﺠﺩ ﻤﻥ )‪ (9.10‬ﺒﻌﺩ ﺍﻻﺨﺘﺼﺎﺭ ﻋﻠﻰ‬


‫‪1‬‬
‫‪2‬‬
‫‪v‬‬
‫‪1‬‬
‫‪c2‬‬
‫ﺍﻟﻤﻌﺎﺩﻟﺔ‪:‬‬ ‫ﻭﻀﺭﺏ ﺍﻟﻁﺭﻓﻴﻥ ﺒـ‬
‫‪i‬‬
‫‪c‬‬
‫‪v2‬‬ ‫‪i  ‬‬
‫‪F 1 ‬‬ ‫) ‪ (F . v‬‬ ‫)‪(9.11‬‬
‫‪c2 c‬‬
‫‪‬‬ ‫‪ ‬‬
‫ﻏﻴﺭ ﺃﻥ ﺍﻟﻤﻘﺩﺍﺭ ‪ F . v‬ﻫﻭ ﺍﻟﻌﻤل ﺍﻟﺫﻱ ﺘﻨﺠﺯﻩ ﺍﻟﻘﻭﺓ ‪ F‬ﻓﻲ ﻭﺍﺤﺩﺓ ﺍﻟﺯﻤﻥ‬
‫‪‬‬
‫)ﺍﻻﺴﺘﻁﺎﻋﺔ(‪ .‬ﻭﻫﻜﺫﺍ ﺘﺭﺘﺒﻁ ﺍﻟﻤﺭﻜﺒﺔ ‪ F‬ﺒﻌﻤل ﺍﻟﻘﻭﺓ ‪ F‬ﻓﻲ ﺍﻟﻔﺭﺍﻍ ﺍﻟﺜﻼﺜﻲ‬
‫ﺒﺎﻟﻌﻼﻗﺔ‪:‬‬
‫‪ ‬‬
‫) ‪i (F . v‬‬
‫‪F ‬‬ ‫)‪(9.12‬‬
‫‪c‬‬ ‫‪v2‬‬
‫‪1-‬‬
‫‪c2‬‬
‫ﻭﻴﻤﻜﻥ ﺍﻵﻥ ﻭﻀﻊ ﺍﻟﻤﻌﺎﺩﻟﺔ )‪ (9.8‬ﺒﻌﺩ ﺘﺒﺩﻴل ‪ F‬ﺒﻘﻴﻤﺘﻬﺎ ﻤﻥ )‪(9.12‬‬
‫ﺒﺎﻟﺸﻜل‪:‬‬
‫‪mc 2‬‬ ‫‪ ‬‬
‫)‪(9.13‬‬
‫‪d‬‬ ‫‪dA‬‬
‫‪ (F . v ) ‬‬
‫‪dt‬‬ ‫‪v‬‬ ‫‪2‬‬ ‫‪dt‬‬
‫‪1‬‬
‫‪c2‬‬

‫‪- 288 -‬‬

‫)‪Create PDF files without this message by purchasing novaPDF printer (http://www.novapdf.com‬‬
‫ﻭﺒﻤﺎ ﺃﻥ ﺍﻟﻁﺭﻑ ﺍﻷﻴﻤﻥ ﻫﻭ ﺍﻻﺴﺘﻁﺎﻋﺔ ﻓﻤﻥ ﺍﻟﻁﺒﻴﻌﻲ ﺃﻥ ﻨﻔﺭﺽ‪ ،‬ﺍﻨﻁﻼﻗﺎﹰ‬
‫‪mc 2‬‬
‫ﻫﻭ ﺍﻟﻁﺎﻗﺔ ﺍﻟﺤﺭﻜﻴﺔ ﺍﻟﺘﻲ‬ ‫ﻤﻥ ﻤﻔﺎﻫﻴﻡ ﺍﻟﻤﻴﻜﺎﻨﻴﻙ ﺍﻟﺘﻘﻴﺩﻱ‪ ،‬ﺃﻥ ﺍﻟﻤﻘﺩﺍﺭ‬
‫‪2‬‬
‫‪v‬‬
‫‪1‬‬
‫‪c2‬‬
‫ﻨﺭﻤﺯ ﻟﻬﺎ ﺒﺎﻟﺭﻤﺯ ‪ E‬ﻫﻨﺎ )ﻭﻟﻴﺱ ‪ T‬ﻜﻤﺎ ﻓﻌﻠﻨﺎ ﺴﺎﺒﻘﺎﹰ ﻷﺴﺒﺎﺏ ﺘﺘﻀﺢ ﻓﻴﻤﺎ ﺒﻌﺩ ( ﺃﻱ‬
‫ﺃﻥ‪:‬‬
‫‪mc 2‬‬
‫‪E‬‬ ‫)‪(9.14‬‬
‫‪2‬‬
‫‪v‬‬
‫‪1‬‬
‫‪c2‬‬
‫‪‬‬ ‫‪‬‬
‫ﻟﻨﺒﺤﺙ ﺃﺨﻴﺭ ﺍﹰ ﻋﻥ ﺼﻴﻐﺔ ﻟﻠﺘﺴﺎﺭﻉ ‪ W‬ﻭﻋﻼﻗﺘﻪ ﺒﺎﻟﻘﻭﺓ ‪ ، F‬ﻟﺫﺍ ﻨﻀﻊ‬
‫ﻤﻌﺎﺩﻻﺕ ﺍﻟﺤﺭﻜﺔ )‪ (9.6‬ﻭ )‪ (9.7‬ﺒﺎﻟﺸﻜل ﺍﻟﺸﻌﺎﻋﻲ ﺍﻟﺘﺎﻟﻲ‪:‬‬
‫‪‬‬ ‫‪‬‬
‫‪mc 2‬‬ ‫‪‬‬
‫)‪ F (9.15‬‬
‫‪d‬‬ ‫‪mv‬‬ ‫‪m‬‬ ‫‪dv‬‬ ‫‪v d‬‬
‫‪‬‬ ‫‪‬‬
‫‪dt‬‬ ‫‪2‬‬
‫‪v2‬‬ ‫‪v 2 dt c dt‬‬ ‫‪v2‬‬
‫‪1‬‬ ‫‪1‬‬ ‫‪1‬‬
‫‪c2‬‬ ‫‪c2‬‬ ‫‪c2‬‬
‫ﺃﻭ ﺒﺎﻟﺸﻜل‪:‬‬
‫‪‬‬ ‫‪‬‬
‫‪v dE ‬‬
‫)‪(9.16‬‬
‫‪m‬‬ ‫‪dv‬‬
‫‪‬‬ ‫‪F‬‬
‫‪2 dt‬‬ ‫‪2 dt‬‬
‫‪v‬‬ ‫‪c‬‬
‫‪1‬‬
‫‪c2‬‬
‫‪‬‬
‫‪‬‬
‫‪ W ‬ﻭﻨﻀﻌﻪ ﺍﻋﺘﻤﺎﺩﺍﹰ ﻋﻠﻰ )‪ (9.13‬ﺒﺎﻟﺸﻜل‪:‬‬ ‫ﻭﻤﻨﻪ ﻨﺤﺼل ﻋﻠﻰ‬
‫‪dv‬‬
‫‪dt‬‬
‫‪v2‬‬
‫‪‬‬ ‫‪1‬‬ ‫‪‬‬ ‫‪‬‬ ‫‪‬‬
‫‪‬‬
‫‪c 2  v   ‬‬
‫‪W‬‬
‫‪dv‬‬
‫‪‬‬ ‫‪ F  2 ( F . v )‬‬ ‫)‪(9.17‬‬
‫‪dt‬‬ ‫‪m‬‬ ‫‪c‬‬
‫‪‬‬ ‫‪‬‬
‫‪F‬‬ ‫‪‬‬
‫؛ ﺃﻱ ﺇﻟﻰ ﺍﻟﻘﻴﻤﺔ ﺍﻟﻤﻘﺎﺒﻠﺔ ﻓﻲ ﺍﻟﻤﻴﻜﺎﻨﻴﻙ‬ ‫ﻭﻴﺒﺩﻭ ﻭﺍﻀﺤﺎﹰ ﺃﻥ ‪ W‬ﻴﺅﻭل ﺇﻟﻰ‬
‫‪m‬‬
‫ﺍﻟﺘﻘﻠﻴﺩﻱ ﻋﻨﺩﻤﺎ ‪. v  c‬‬

‫‪- 289 -‬‬

‫)‪Create PDF files without this message by purchasing novaPDF printer (http://www.novapdf.com‬‬
‫‪ -73‬ﺍﻟﻌﻼﻗﺔ ﺑﲔ ﻛﻤﻴﺔ ﺍﳊﺮﻛﺔ ﻭﺍﻟﻄﺎﻗﺔ ﻭﺍﻟﻜﺘﻠﺔ ﰲ ﻣﻴﻜﺎﻧﻴﻚ‬
‫ﺍﻟﻨﻈﺮﻳﺔ ﺍﻟﻨﺴﺒﻴﺔ ‪:‬‬
‫ﺴﻨﺩﺭﺱ ﺍﻵﻥ ﺨﻭﺍﺹ ﺍﻟﻜﻤﻴﺎﺕ ﺍﻟﺘﻲ ﻋﺭ‪‬ﻓﻨﺎﻫﺎ ﻓﻲ ﺍﻟﻔﻘﺭﺘﻴﻥ ﺍﻟﺴﺎﺒﻘﺘﻴﻥ‪:‬‬
‫ﻟﺫﺍ ﻨﻜﺘﺏ ﺃﻭﻻﹰ ﺍﻟﻌﻼﻗﺔ ﺒﻴﻥ ﻜﺘﻠﺔ ﺍﻟﺴﻜﻭﻥ ‪ m‬ﻭﻜﻤﻴﺔ ﺍﻟﺤﺭﻜﺔ ﻓﻲ ﺍﻟﻔﺭﺍﻍ‬
‫ﺍﻟﺜﻼﺜﻲ ﻁﺒﻘﺎﹰ ﻟـ )‪ (9.2‬ﺤﻴﺙ ﻨﺠﺩ‪:‬‬
‫‪‬‬
‫‪‬‬
‫)‪(9.18‬‬
‫‪mv‬‬
‫‪P ‬‬
‫‪2‬‬
‫‪v‬‬
‫‪1‬‬
‫‪c2‬‬
‫ﻭﻫﻲ ﺘﺅﻭل ﺇﻟﻰ ﺍﻟﻌﻼﻗﺔ ﺍﻟﻜﻼﺴﻴﻜﻴﺔ ﻋﻨﺩﻤﺎ ‪ . v  c‬ﻭﻤﻥ ﻫﻨﺎ ﻴﻤﻜﻥ ﺃﻥ‬
‫ﻨﻔﻜﺭ ﺃﻥ ‪ m‬ﻗﺩ ﺘﺘﻁﺎﺒﻕ ﻤﻊ ﻜﺘﻠﺔ ﺍﻟﺠﺴﻴﻡ ﺍﻟﻤﺘﺤﺭﻙ ﺒﺴﺭﻋﺔ ﻏﻴﺭ ﻜﺒﻴﺭﺓ‪ ،‬ﻭﻟﻜﻥ‬
‫ﺍﻟﺤﻘﻴﻘﺔ ﻏﻴﺭ ﺫﻟﻙ‪ ،‬ﻭﻟﻴﺴﺕ ﺒﻬﺫﻩ ﺍﻟﺒﺴﺎﻁﺔ ﻜﻤﺎ ﺴﻴﺘﻀﺢ ﻓﻴﻤﺎﻴﻠﻲ‪:‬‬
‫ﺍﻥ ﺨﻭﺍﺹ ﻜﺘﻠﺔ ﺍﻟﺴﻜﻭﻥ ﻟﻠﺠﺴﻴﻡ ﺘﺨﺘﻠﻑ ﺘﻤﺎﻤﺎﹰ ﻋﻥ ﺨﻭﺍﺼﻬﺎ ﻓﻲ ﺍﻟﻤﻴﻜﺎﻨﻴﻙ‬
‫ﺍﻟﺘﻘﻠﻴﺩﻱ ﻓﻤﺜﻼﹰ ﺃﻥ ‪ m‬ﻻﺘﺤﻘﻕ ﻗﺎﻨﻭﻥ ﺤﻔﻅ ﺍﻟﻜﺘﻠﺔ‪ ،‬ﻭﻫﻨﺎﻙ ﺍﻟﻜﺜﻴﺭ ﻤﻥ ﺍﻟﻅﻭﺍﻫﺭ‬
‫ﺍﻟﻔﻴﺯﻴﺎﺌﻴﺔ ﺍﻟﺘﻲ ﺘﺒﻴﻥ ﺫﻟﻙ‪ ،‬ﻫﺫﺍ ﺒﺎﻹﻀﺎﻓﺔ ﺇﻟﻰ ﺃﻥ ﺍﻟﻜﺘﻠﺔ ‪ m‬ﻨﻔﺴﻬﺎ ﻻﺘﻭﺠﺩ ﻋﻨﺩ ﺒﻌﺽ‬
‫ﺍﻟﺠﺴﻴﻤﺎﺕ ﻜﺎﻟﻔﻭﺘﻭﻨﺎﺕ ﻭﺒﻌﺽ ﺍﻟﺠﺴﻴﻤﺎﺕ ﺍﻷﺴﺎﺴﻴﺔ ﻜﺎﻟﻨﻴﺘﺭﻴﻨﻭ ﻤﺜﻼﹰ‪ .‬ﻭﻫﺫﺍ ﻴﻌﻨﻲ‬
‫ﺃﻨﻪ ﺇﺫﺍ ﺤﺩﺙ ﺘﻔﺎﻋل ﻤﺎ ﺒﺤﻴﺙ ﻨﺘﺞ ﻓﻲ ﻨﻬﺎﻴﺘﻪ ﺠﺴﻴﻤﺎﺕ ﻜﺘﻠﻬﺎ ﺍﻟﺴﺎﻜﻨﺔ ﺘﺴﺎﻭﻱ‬
‫ﺍﻟﺼﻔﺭ ﻓﺈﻥ ﻗﺎﻨﻭﻥ ﺤﻔﻅ ﺍﻟﻜﺘﻠﺔ ﺍﻟﻤﻌﺭﻭﻑ ﻟﻥ ﻴﺘﺤﻘﻕ‪ .‬ﻭﻨﺫﻜﺭ ﻤﺜﺎﻻﹰ ﻋﻠﻰ ﺫﻟﻙ‪ :‬ﺍﻨﺘﺎﺝ‬
‫ﺍﻷﺯﻭﺍﺝ ﻓﻲ ﻏﺭﻓﺔ ﻭﻴﻠﺴﻭﻥ ﻭﻓﻨﺎﺀ ﺍﻻﻟﻜﺘﺭﻭﻥ ﻭﺍﻟﺒﻭﺯﻴﺘﺭﻭﻥ ﻟﻴﻜﻭﻨﺎ ﻤﻌﺎﹰ ﻓﻭﺘﻭﻨﺎﹰ‬
‫ﻀﻭﺌﻴﺎﹰ‪ .‬ﻭﻗﺩ ﺒﺭﻫﻨﺕ ﺘﺠﺎﺭﺏ ﺍﻟﻔﻴﺯﻴﺎﺀ ﺍﻟﻨﻭﻭﻴﺔ‪ ،‬ﻭﺨﺼﻭﺼﺎﹰ ﺘﻠﻙ ﺍﻟﺘﻲ ﺃﺠﺭﻴﺕ ﻓﻲ‬
‫ﻤﺠﺎﻻﺕ ﺍﻟﺠﺴﻴﻤﺎﺕ ﺍﻷﺴﺎﺴﻴﺔ‪ ،‬ﺃﻥ ﻜﺘﻠﺔ ﺍﻟﺴﻜﻭﻥ ﺍﻟﻤﻌﺭﻓﺔ ﻓﻲ ﺒﺩﺍﻴﺔ ﻫﺫﺍ ﺍﻟﻔﺼل‬
‫ﺘﺅﻟﻑ ﺍﺤﺩﻯ ﺍﻟﺨﻭﺍﺹ ﺍﻷﺴﺎﺴﻴﺔ ﻟﻠﺠﺴﻴﻤﺎﺕ ﺒﺸﻜل ﻋﺎﻡ ﻭﻫﻲ ﻻﺘﺘﻐﻴﺭ ﻓﻲ ﻜل‬
‫ﺍﻟﺘﻔﺎﻋﻼﺕ ﺍﻟﺘﻲ ﺘﺸﺎﺭﻙ ﻓﻴﻬﺎ‪.‬‬
‫ﻟﺫﻟﻙ ﻜﻠﻪ ﻜﺎﻥ ﻻﺒﺩ ﻤﻥ ﺘﻭﻀﻴﺢ ﻤﻔﻬﻭﻡ ﺍﻟﻜﺘﻠﺔ ﻓﻲ ﺍﻟﻨﻅﺭﻴﺔ ﺍﻟﻨﺴﺒﻴﺔ ﺒﺸﻜل‬
‫ﺠﻴﺩ‪ ،‬ﻟﺫﺍ ﻨﻔﺭﺽ ﺃﻥ ﻜﺘﻠﺔ ﺍﻟﺠﺴﻴﻡ ﺘﺎﺒﻌﺔ ﻟﺴﺭﻋﺘﻪ ﺃﻱ )‪ m  m(v‬ﻭﺘﻌﺭﻑ ﺒﺄﻨﻬﺎ‬
‫ﻋﺎﻤل ﺍﻟﺘﻨﺎﺴﺏ ﻤﺎ ﺒﻴﻥ ﺍﻟﺴﺭﻋﺔ ﻭﻜﻤﻴﺔ ﺍﻟﺤﺭﻜﺔ ﻭﺘﻭﻀﻊ ﺒﺎﻟﺸﻜل‪:‬‬

‫‪- 290 -‬‬

‫)‪Create PDF files without this message by purchasing novaPDF printer (http://www.novapdf.com‬‬
‫‪‬‬ ‫‪‬‬ ‫‪m‬‬
‫‪P  m( v) v , m( v) ‬‬ ‫)‪(9.19‬‬
‫‪2‬‬
‫‪v‬‬
‫‪1‬‬
‫‪c2‬‬
‫ﻭﻫﻜﺫﺍ ﻓﺈﻥ ﻤﻔﻬﻭﻡ ﺍﻟﻜﺘﻠﺔ ﻴﺘﻭﻗﻑ ﻋﻠﻰ ﺍﻟﺴﺭﻋﺔ‪ ،‬ﻭﻟﻜﻥ ﻴﺠﺏ ﺍﻟﻤﻼﺤﻅﺔ ﺃﻴﻀﺎﹰً‬
‫‪v2‬‬
‫‪ . 1‬ﻓﺈﺫﺍ ﺘﺤﺭﻙ ﺠﺴﻴﻡ‬ ‫ﺃﻥ )‪ m(v‬ﻟﻴﺴﺕ ﻤﻘﺩﺍﺭﺍﹰ ﻻﻤﺘﻐﻴﺭ ﺍﹰ ﺒﺴﺒﺏ ﺘﻐﻴﺭ‬
‫‪c2‬‬
‫ﺒﺴﺭﻋﺘﻴﻥ ﻤﺨﺘﻠﻔﻴﻥ ﺒﺎﻟﻨﺴﺒﺔ ﺇﻟﻰ ﺠﻤﻠﺘﻲ ﻗﻴﺎﺱ ﻤﺨﺘﻠﻔﻴﻥ ﻓﺈﻥ ﻜﺘﻠﺘﻪ ﺴﺘﺨﺘﻠﻑ ﻓﻲ‬
‫ﺍﻟﺠﻤﻠﺘﻴﻥ‪.‬‬
‫ﻟﻨﺩﺭﺱ ﺍﻵﻥ ﺨﻭﺍﺹ ﺍﻟﻁﺎﻗﺔ ﺍﻟﻜﻠﻴﺔ ﺍﻟﻤﻌﺭﻓﺔ ﺒﺎﻟﻌﻼﻗﺔ )‪ (9.14‬ﻓﻨﺠﺩ ﻤﻨﻬﺎ‬
‫ﻭﻤﻥ ﺍﻟﻌﻼﻗﺔ ﺍﻷﺨﻴﺭﺓ ﻤﻥ )‪ (9.2‬ﻤﺎﻴﻠﻲ‪:‬‬
‫)‪(9.20‬‬
‫‪E‬‬
‫‪F  i‬‬
‫‪c‬‬
‫ﻭﻫﻜﺫﺍ ﻓﺈﻥ ﺍﻟﻤﺭﻜﺒﺔ ﺍﻟﺯﻤﻨﻴﺔ ﻟﻜﻤﻴﺔ ﺍﻟﺤﺭﻜﺔ ﺘﺴﺎﻭﻱ ﺍﻟﻁﺎﻗﺔ )ﺒﺎﻟﺘﻘﺭﻴﺏ ﺇﻟﻰ‬
‫ﺜﺎﺒﺕ ﺍﻟﺘﻨﺎﺴﺏ(‪ .‬ﻭﺃﻫﻤﻴﺔ ﻫﺫﻩ ﺍﻟﻌﻼﻗﺔ )‪ (9.20‬ﺘﻜﻤﻥ ﻓﻲ ﺃﻥ ﻜﻤﻴﺔ ﺍﻟﺤﺭﻜﺔ ﻓﻲ‬
‫ﺍﻟﻔﺭﺍﻍ ﺍﻟﺭﺒﺎﻋﻲ ﺘﻭﻀﻊ ﺒﺎﻟﺸﻜل‪:‬‬
‫)‪(9.21‬‬
‫‪E‬‬
‫) ‪P (Px , Py , Pz , i‬‬
‫‪c‬‬
‫ﻭﻫﺫﻩ ﺍﻟﻤﺭﻜﺒﺎﺕ ﻟﻴﺴﺕ ﻻﻤﺘﻐﻴﺭﺍﺕ ﻨﺴﺒﻴﺔ ﻭﺇﻨﻤﺎ ﺘﺘﻐﻴﺭ ﺒﺘﺤﻭﻴﻼﺕ ﻁﺒﻘﺎﹰ‬
‫ﻟﻠﻤﻌﺎﺩﻻﺕ ﺍﻟﻌﺎﻤﺔ )‪(8.67‬؛ ﺃﻱ ﺒﺎﻟﺸﻜل‪:‬‬
‫'‪E‬‬
‫‪P'x ‬‬ ‫‪V‬‬
‫‪Px ‬‬ ‫‪c2‬‬ ‫‪, Py  P ' y , Pz  P' z‬‬
‫‪2‬‬
‫‪V‬‬
‫‪1‬‬
‫‪c2‬‬ ‫)‪(9.22‬‬
‫‪P c‬‬ ‫‪E' VP' x‬‬
‫‪E  ‬‬
‫‪i‬‬ ‫‪V2‬‬
‫‪1‬‬
‫‪c2‬‬
‫ﺤﻴﺙ ‪ V‬ﻫﻲ ﺍﻟﺴﺭﻋﺔ ﺍﻟﻨﺴﺒﻴﺔ ﻟﻠﺠﻤﻠﺘﻴﻥ‪.‬‬

‫‪- 291 -‬‬

‫)‪Create PDF files without this message by purchasing novaPDF printer (http://www.novapdf.com‬‬
‫ﺃﻤﺎ ﺍﻟﻼﻤﺘﻐﻴﺭﺍﺕ ﺍﻟﻨﺴﺒﻴﺔ ﻫﻨﺎ ﻓﻬﻲ ﻤﺭﺒﻊ ﺍﻟﻤﺘﺠﻪ ‪ P‬ﺍﻟﻤﻌﺭﻑ ﻓﻲ ﺍﻟﻔﺭﺍﻍ‬
‫ﺍﻟﺭﺒﺎﻋﻲ )ﺍﻨﻅﺭ )‪ ((8.69‬ﺃﻱ‪:‬‬
‫‪E2‬‬
‫‪invariant  P2  Px2  Py2  Pz2  P2  Px2  Py2  Pz2 ‬‬ ‫)‪ (9.23‬‬
‫‪2‬‬
‫‪‬‬ ‫‪c‬‬
‫ﻭﻟﺤﺴﺎﺏ ﻗﻴﻤﺔ ﻫﺫﺍ ﺍﻟﻼﻤﺘﻐﻴﺭ ﻨﺒﺩل ﻤﺭﻜﺒﺎﺕ ‪ P‬ﺒﻘﻴﻤﻬﺎ ﻓﻨﺠﺩ‪:‬‬
‫‪m 2v 2‬‬ ‫‪m2c2‬‬
‫‪ P2 ‬‬ ‫‪2‬‬
‫‪‬‬
‫‪2‬‬
‫‪ m2c 2‬‬ ‫)‪(9.24‬‬
‫‪‬‬ ‫‪v‬‬ ‫‪v‬‬
‫‪1‬‬ ‫‪1‬‬
‫‪c2‬‬ ‫‪c2‬‬
‫ﻟﻨﻌﺩ ﺍﻵﻥ ﺇﻟﻰ ﻋﺒﺎﺭﺓ ﺍﻟﻁﺎﻗﺔ ﻭ ﻨﻨﺸﺭ ‪ E‬ﻋﻨﺩﻤﺎ ‪ v  c‬ﻓﻨﺠﺩ‪:‬‬
‫‪1‬‬
‫‪v2  2‬‬ ‫‪v2‬‬ ‫‪mv 2‬‬
‫‪2‬‬
‫‪E  mc (1 ‬‬ ‫)‬ ‫‪ mc 2 (1 ‬‬ ‫‪ ....)  mc 2 ‬‬ ‫)‪(9.25‬‬
‫‪c2‬‬ ‫‪2c 2‬‬ ‫‪2‬‬
‫ﻭﺍﻟﺤﺩ ﺍﻟﺜﺎﻨﻲ ﻴﺘﻁﺎﺒﻕ ﻤﻊ ﺍﻟﻁﺎﻗﺔ ﺍﻟﺤﺭﻜﻴﺔ ﻟﻠﺠﺴﻴﻡ‪ ،‬ﻭﻟﻜﻥ ﻋﻨﺩﻤﺎ ‪ v  0‬ﻓﺈﻥ‬
‫ﻁﺎﻗﺔ ﺍﻟﺠﺴﻴﻡ ‪ E 0‬ﻏﻴﺭ ﺍﻟﻤﻭﺠﻭﺩ ﻓﻲ ﺃﻱ ﺤﻘل ﻗﻭﻯ ﺘﻌﻁﻰ ﺒﺎﻟﻌﻼﻗﺔ‪:‬‬
‫‪E 0  mc 2  0‬‬ ‫)‪(9.26‬‬
‫ﻭﻫﻲ ﺍﺤﺩﻯ ﺍﻟﻨﺘﺎﺌﺞ ﺍﻟﻬﺎﻤﺔ ﻟﻠﻨﻅﺭﻴﺔ ﺍﻟﻨﺴﺒﻴﺔ‪ ،‬ﺇﺫ ﺃﻥ ﻁﺎﻗﺔ ﺍﻟﺠﺴﻴﻡ ﺍﻟﺴﺎﻜﻥ‬
‫ﻻﺘﺴﺎﻭﻱ ﺍﻟﺼﻔﺭ‪ ،‬ﻭﻜﺜﻴﺭﺍﹰ ﻤﺎ ﺘﺴﻤﻰ ﺍﻟﻁﺎﻗﺔ ‪ E 0‬ﻁﺎﻗﺔ ﺍﻟﺴﻜﻭﻥ ‪ .Rest Energy‬ﻭﻗﺩ‬
‫ﺘﻅﻬﺭ ﻫﺫﻩ ﺍﻟﻨﺘﻴﺠﺔ ﻏﺭﻴﺒﺔ ﻟﻠﻭﻫﻠﺔ ﺍﻷﻭﻟﻰ ﻭﻗﺩ ﻴﺼﻌﺏ ﻋﻠﻴﻨﺎ ﻓﻬﻡ ﺃﻥ ﻁﺎﻗﺔ ﺍﻟﺠﺴﻴﻡ‬
‫ﺍﻟﺴﺎﻜﻥ ﻏﻴﺭ ﺍﻟﺨﺎﻀﻊ ﻷﻱ ﺘﺄﺜﻴﺭ ﺨﺎﺭﺠﻲ ﻻﺘﺴﺎﻭﻱ ﺍﻟﺼﻔﺭ‪ ،‬ﻤﻤﺎ ﻴﺩﻋﻭﻨﺎ ﺇﻟﻰ ﺍﻟﺸﻙ‬
‫ﺃﻥ ﺘﻌﺭﻴﻑ ﺍﻟﻁﺎﻗﺔ ﺒﺎﻟﻌﻼﻗﺔ )‪ (9.14‬ﻏﻴﺭ ﺼﺤﻴﺢ ﻭﺃﻨﻪ ﻜﺎﻥ ﻴﺠﺏ ﺍﻀﺎﻓﺔ ﺜﺎﺒﺕ ﻤﺎ‬
‫ﻫﻭ ‪  mc2‬ﺒﺤﻴﺙ ﻴﺨﺘﺼﺭ ﻤﻊ ﺍﻟﺜﺎﺒﺕ ‪ mc2‬ﺍﻟﻤﻭﺠﻭﺩ ﻓﻲ )‪ (9.25‬ﻭﺘﻨﻌﺩﻡ ﻁﺎﻗﺔ‬
‫ﺍﻟﺠﺴﻴﻡ ﺍﻟﺴﺎﻜﻥ ﻋﻨﺩﻤﺎ ‪ . v  0‬ﻭﻟﻜﻥ ﺍﻟﺤﻘﻴﻘﺔ ﻫﻲ ﻏﻴﺭ ﺫﻟﻙ ﻭﻻﻴﻤﻜﻥ ﺃﻥ ﻨﻁﻠﺏ‬
‫ﺩﺍﺌﻤﺎﹰ ﻤﻥ ﺍﻟﻌﻼﻗﺎﺕ ﺍﻟﻨﺴﺒﻴﺔ ﺃﻥ ﺘﺅﻭل ﺇﻟﻰ ﺍﻟﻌﻼﻗﺎﺕ ﺍﻟﻜﻼﺴﻴﻜﻴﺔ ﺍﻟﻤﻘﺎﺒﻠﺔ ﻟﻬﺎ ﻋﻨﺩﻤﺎ‬
‫‪ ، v  c‬ﻭﻟﻜﻥ‪ ،‬ﻭﺒﺩﻭﻥ ﺸﻙ‪ ،‬ﻴﻤﻜﻥ ﺃﻥ ﻨﺅﻜﺩ ﺃﻥ ﺘﺤﻭﻴﻼﺕ ﻟﻭﺭﻨﺘﺯ ﻴﺠﺏ ﺃﻥ ﺘﺅﻭل‬
‫ﺇﻟﻰ ﺘﺤﻭﻴﻼﺕ ﻏﺎﻟﻴﻠﻲ )ﻫﺫﺍ ﺍﻟﺸﻲﺀ ﺍﻟﻤﻭﺠﻭﺩ ﻓﻲ ﺼﻠﺏ ﺍﻟﻨﻅﺭﻴﺔ ﺍﻟﻨﺴﺒﻴﺔ( ﻭﺒﺼﻭﺭﺓ‬
‫ﺨﺎﺼﺔ ﻴﺠﺏ ﺃﻥ ﺘﺘﺤﻭل ﺍﻟﻤﻌﺎﺩﻻﺕ )‪ (9.22‬ﺇﻟﻰ ﻋﻼﻗﺎﺕ ﺘﺭﻜﻴﺏ ﺍﻟﺴﺭﻉ ﻓﻲ‬
‫ﺍﻟﻤﻴﻜﺎﻨﻴﻙ ﺍﻟﺘﻘﻠﻴﺩﻱ‪ ،‬ﻓﺈﺫﺍ ﻓﺭﻀﻨﺎ ﺠﺴﻴﻤ ﹰﺎ ﺴﺭﻋﺘﻪ '‪ v‬ﻓﻲ ﺠﻤﻠﺔ ﻋﻁﺎﻟﻴﺔ '‪ K‬ﺘﺘﺤﺭﻙ‬

‫‪- 292 -‬‬

‫)‪Create PDF files without this message by purchasing novaPDF printer (http://www.novapdf.com‬‬
‫ﺒﺩﻭﺭﻫﺎ ﺒﺴﺭﻋﺔ ‪ (V  c ) V‬ﻭﻜﺎﻨﺕ ‪ v'  c‬ﺃﻴﻀﺎﹰ ﻓﺈﻥ ﻁﺎﻗﺘﻪ '‪ ،E‬ﻁﺒﻘﺎﹰ ﻟﻠﻌﻼﻗﺔ‬
‫) ‪ . (E  mc2‬ﻭﺇﺫﺍ ﻋﻭﻀﻨﺎ ﻫﺫﻩ ﺍﻟﻘﻴﻤﺔ ﻓﻲ ﺍﻟﻌﻼﻗﺔ‬ ‫‪mc 2‬‬ ‫)‪ ،(9.25‬ﺘﺴﺎﻭﻱ ﺘﻘﺭﻴﺒﺎﹰ‬
‫ﺍﻷﻭﻟﻰ ﻤﻥ ﺘﺤﻭﻴﻼﺕ ﻟﻭﺭﻨﺘﺯ )‪ (9.22‬ﻓﻲ ﺍﻟﻔﺭﺍﻍ ﺍﻟﺭﺒﺎﻋﻲ ﻓﺈﻨﻨﺎ ﻨﺠﺩ‪:‬‬
‫‪mc 2‬‬
‫‪Px  P' x ‬‬ ‫‪V  P' x  mV‬‬ ‫)‪(9.27‬‬
‫‪c2‬‬
‫ﻭﻤﻨﻪ ﻨﺤﺼل ﻋﻠﻰ ﻋﻼﻗﺔ ﺘﺭﻜﻴﺏ ﺍﻟﺴﺭﻉ ﺍﻟﻤﻌﺭﻭﻓﺔ ﻓﻲ ﺍﻟﻤﻴﻜﺎﻨﻴﻙ ﺍﻟﻜﻼﺴﻴﻜﻲ‬
‫ﻭﻫﻲ‪:‬‬
‫‪v x  v' x  V‬‬
‫ﺃﻤﺎ ﺇﺫﺍ ﺘﺠﺎﻫﻠﻨﺎ ﻁﺎﻗﺔ ﺍﻟﺠﺴﻴﻡ ﺍﻟﺴﺎﻜﻥ ‪ mc2‬ﻭﻭﻀﻌﻨﺎﻫﺎ ﺘﺴﺎﻭﻱ ﺍﻟﺼﻔﺭ ﻓﺈﻨﻨﺎ‬
‫ﻨﺠﺩ ﻤﻥ )‪ (9.22‬ﺃﻥ ‪ P x  P' x‬ﻭ ‪ v x  v' x‬ﻭﻫﺫﺍ ﻏﻴﺭ ﻤﻌﻘﻭل‪ ،‬ﻤﻤﺎ ﻴﺅﻜﺩ ﺃﻥ‬
‫ﻁﺎﻗﺔ ﺍﻟﺠﺴﻴﻡ ﺍﻟﺴﺎﻜﻥ ﻻﺘﺴﺎﻭﻱ ﺍﻟﺼﻔﺭ ﻓﻌﻼﹰ‪ ،‬ﺃﻤﺎ ﻁﺎﻗﺔ ﺍﻟﺠﺴﻴﻡ ﺍﻟﻤﺘﺤﺭﻙ ﺒﺴﺭﻋﺔ ‪v‬‬
‫ﻓﺘﻭﻀﻊ‪ ،‬ﻁﺒﻘﺎﹰ ﻟـ )‪ ،(9.14‬ﺒﺎﻟﺸﻜل‪:‬‬
‫‪E  m(v )c 2‬‬ ‫)‪(9.28‬‬
‫ﻭﻜﺜﻴﺭﺍﹰ ﻤﺎﺘﺴﻤﻰ ﺍﻟﻤﻌﺎﺩﻟﺘﺎﻥ )‪ (9.26‬ﻭ )‪ (9.28‬ﻤﻌﺎﺩﻟﺘﻲ ﺍﻴﻨﺸﺘﺎﻴﻥ‪ ،‬ﻭﻫﻤﺎ‬
‫ﺘﺅﻜﺩﺍﻥ ﺃﻥ ﻟﻠﺠﺴﻴﻡ ﺍﻟﺫﻱ ﻜﺘﻠﺘﻪ ‪ m‬ﻁﺎﻗﺔ ‪ E‬ﻭﺃﻥ ﺍﻟﻜﺘﻠﺔ ﻭﺍﻟﻁﺎﻗﺔ ﻤﺘﻼﺯﻤﺎﻥ ﺘﻤﺎﻤ ﺎﹰ‬
‫ﻭﻤﺘﻨﺎﺴﺒﺎﻥ ﻤﻊ ﺒﻌﻀﻬﻤﺎ ﺍﻟﺒﻌﺽ ﻭﺍﻨﻬﻤﺎ ﻭﺠﻬﺎﻥ ﻤﺨﺘﻠﻔﺎﻥ ﻟﺸﻲﺀ ﻭﺍﺤﺩ‪ ،‬ﻭﻫﺫﺍ‬
‫ﻤﺎﻴﺴﻤﻰ ﺃﺤﻴﺎﻨﺎﹰ ﺒﻘﺎﻨﻭﻥ ﺘﻜﺎﻓﺅ ﺍﻟﻜﺘﻠﺔ ﻭﺍﻟﻁﺎﻗﺔ‪ ،‬ﺫﻟﻙ ﺍﻟﻘﺎﻨﻭﻥ ﺍﻟﺫﻱ ﺃﺜﺒﺘﺕ ﺍﻟﺘﺠﺎﺭﺏ‬
‫ﺼﺤﺘﻪ ﺒﺤﻴﺙ ﻻﻴﺭﻗﻰ ﺇﻟﻴﻪ ﺍﻟﺸﻙ ﺤﺘﻰ ﺍﻵﻥ‪.‬‬
‫ﺇﻥ ﺍﺤﺩﻯ ﺍﻟﻨﺘﺎﺌﺞ ﺍﻟﻬﺎﻤﺔ ﺃﻴﻀﺎﹰ ﻟﻠﻨﻅﺭﻴﺔ ﺍﻟﻨﺴﺒﻴﺔ ﻫﻲ ﺠﻤﻊ ﻗﺎﻨﻭﻨﻲ ﻤﺼﻭﻨﻴﺔ‬
‫ﺍﻟﻁﺎﻗﺔ ﻭﻤﺼﻭﻨﻴﺔ ﻜﻤﻴﺔ ﺍﻟﺤﺭﻜﺔ ﻓﻲ ﻗﺎﻨﻭﻥ ﻭﺍﺤﺩ‪ ،‬ﻭﻟﺘﻭﻀﻴﺢ ﺫﻟﻙ ﻨﻔﺭﺽ ﻤﺠﻤﻭﻋﺔ‬
‫ﻤﻌﺯﻭﻟﺔ )ﻻﺘﺅﺜﺭ ﻋﻠﻴﻬﺎ ﺃﻴﺔ ﻗﻭﻯ ﺨﺎﺭﺠﻴﺔ( ﻭﺒﺎﻟﺘﺎﻟﻲ ﺴﺘﻜﻭﻥ ﻁﺎﻗﺘﻬﺎ ﺍﻟﻜﻠﻴﺔ ﻭﻜﻤﻴﺔ‬
‫ﺤﺭﻜﺘﻬﺎ ﻤﺼﻭﻨﺘﺎﻥ ﺃﻱ‪:‬‬
‫)‪(9.29‬‬
‫‪dE‬‬
‫‪ 0  E  E0‬‬
‫‪dt‬‬
‫)‪(9.30‬‬
‫‪dP‬‬
‫‪ 0  P  P0‬‬
‫‪dt‬‬

‫‪- 293 -‬‬

‫)‪Create PDF files without this message by purchasing novaPDF printer (http://www.novapdf.com‬‬
‫ﻭﻴﻤﻜﻥ ﺍﻟﺘﺄﻜﺩ ﺃﻥ ﺍﻟﻘﺎﻨﻭﻥ ﺍﻟﺜﺎﻨﻲ ﺴﻴﻜﻭﻥ ﻨﺘﻴﺠﺔ ﻁﺒﻴﻌﻴﺔ ﻟﻠﻘﺎﻨﻭﻥ ﺍﻷﻭل ﺍﺫﺍ‬
‫ﻻﺤﻅﻨﺎ ﺍﻟﻌﻼﻗﺔ )‪ (9.28‬ﺍﻟﺘﻲ ﺘﺒﻴﻥ ﺃﻨﻪ ﻋﻨﺩﻤﺎ ‪ E  const‬ﻓﺈﻥ ‪m( v)  const.‬‬
‫‪‬‬ ‫‪‬‬
‫ﻤﻤﺎ ﻴﻨﺘﺞ ﻋﻨﻪ ﺜﺒﺎﺕ ﺍﻟﺴﺭﻋﺔ ‪ v‬ﻭ ﺒﺎﻟﺘﺎﻟﻲ ﻤﺼﻭﻨﻴﺔ ‪ . m v‬ﺃﻤﺎ ﻓﻲ ﺍﻟﻨﻅﺭﻴﺔ‬
‫ﺍﻟﻜﻼﺴﻴﻜﻴﺔ ﻓﻜﻼﹰ ﻤﻥ ﺍﻟﻘﺎﻨﻭﻨﻴﻥ ﻤﺴﺘﻘل ﻋﻥ ﺍﻵﺨﺭ‪.‬‬
‫ﻭﺍﻟﺠﺩﻴﺭ ﺒﺎﻟﺫﻜﺭ ﺃﻨﻪ ﻜﺜﻴﺭﺍﹰ ﻤﺎﻨﺴﺘﺨﺩﻡ ﺍﻟﺭﻤﺯ ‪ E kin‬ﻟﻠﺘﻌﺒﻴﺭ ﻋﻥ ﺍﻟﻁﺎﻗﺔ‬
‫ﺍﻟﺤﺭﻜﻴﺔ ﻓﻲ ﺍﻟﻨﻅﺭﻴﺔ ﺍﻟﻨﺴﺒﻴﺔ ﺒﺩﻻﹰ ﻤﻥ ﺍﻟﺭﻤﺯ ‪ T‬ﺍﻟﻤﺴﺘﺨﺩﻡ ﻓﻲ ﺍﻟﻤﻴﻜﺎﻨﻴﻙ ﺍﻟﺘﻘﻠﻴﺩﻱ‬
‫ﺤﻴﺙ‪:‬‬
‫‪‬‬ ‫‪‬‬
‫‪‬‬ ‫‪‬‬
‫‪‬‬ ‫‪‬‬
‫)‪ 1  [ m( v)  m]c 2 (9.31‬‬
‫‪1‬‬
‫‪E kin  E  mc 2  mc 2 ‬‬
‫‪‬‬ ‫‪v2‬‬ ‫‪‬‬
‫‪ 1 2‬‬ ‫‪‬‬
‫‪‬‬ ‫‪c‬‬ ‫‪‬‬
‫ﻭﻤﻥ ﺍﻟﻤﻔﻴﺩ ﻓﻲ ﻨﻬﺎﻴﺔ ﻫﺫﻩ ﺍﻟﻔﻘﺭﺓ ﺍﺴﺘﻨﺘﺎﺝ ﺍﻟﻌﻼﻗﺔ ﺍﻟﻬﺎﻤﺔ ﺍﻟﺘﻲ ﺘﺭﺒﻁ ﺍﻟﻁﺎﻗﺔ‬
‫ﻭﻜﻤﻴﺔ ﺍﻟﺤﺭﻜﺔ ﻭﺫﻟﻙ ﻤﻥ ﺍﻟﻌﻼﻗﺘﻴﻥ )‪ (9.14‬ﻭ )‪ (9.18‬ﺤﻴﺙ ﻨﺠﺩ‪:‬‬
‫‪‬‬ ‫‪E ‬‬
‫‪P ‬‬ ‫‪v‬‬ ‫)‪(9.32‬‬
‫‪c2‬‬
‫‪‬‬ ‫‪‬‬
‫ﺤﻴﺙ ‪ P‬ﻜﻤﻴﺔ ﺤﺭﻜﺔ ﺍﻟﺠﺴﻴﻡ ﻓﻲ ﺍﻟﻔﺭﺍﻍ ﺍﻟﺜﻼﺜﻲ‪ E ،‬ﻁﺎﻗﺘﻪ ﻭ ‪ v‬ﺴﺭﻋﺘﻪ‪.‬‬
‫‪ -74‬ﺗﺎﺑﻊ ﻻﻏﺮﺍﻧﺞ‪ ،‬ﻣﻌﺎﺩﻻﺕ ﻻﻏﺮﺍﻧﺞ ﻭﺗﺎﺑﻊ ﻫﺎﻣﻠﺘﻮﻥ ﰲ ﺍﻟﻨﻈﺮﻳﺔ‬
‫ﺍﻟﻨﺴﺒﻴﺔ‪:‬‬
‫ﻟﻘﺩ ﺭﺃﻴﻨﺎ ﺴﺎﺒﻘﺎﹰ ﻜﻴﻑ ﺘﻜﺘﺏ ﻤﻌﺎﺩﻻﺕ ﺍﻟﺤﺭﻜﺔ ﺒﺸﻜل ﺒﺴﻴﻁ ﻓﻲ ﺍﻟﻤﻴﻜﺎﻨﻴﻙ‬
‫ﺍﻟﻜﻼﺴﻴﻜﻲ ﻭﺴﻨﻌﻤﻡ ﺫﻟﻙ ﻓﻲ ﺍﻟﻨﻅﺭﻴﺔ ﺍﻟﻨﺴﺒﻴﺔ‪ ،‬ﻟﺫﺍ ﻨﺸﻜل ﺘﺎﺒﻊ ﻻﻏﺭﺍﻨﺞ ﺍﻟﺫﻱ‬
‫ﻴﺴﺎﻭﻱ ‪ T  V‬ﺘﻌﺭﻴﻔﺎﹰ ﻭﺍﻟﺫﻱ ﻤﺸﺘﻘﻪ ﺒﺎﻟﻨﺴﺒﺔ ﻟﻼﺤﺩﺍﺜﻴﺎﺕ ﻴﺴﺎﻭﻱ ﺍﻟﻘﻭﺓ ﺍﻟﻤﺅﺜﺭﺓ ﺃﻤﺎ‬
‫ﻤﺸﺘﻘﻪ ﺒﺎﻟﻨﺴﺒﺔ ﻟﻠﺴﺭﻉ ﺍﻟﻤﻌﻤﻤﺔ ﻓﻴﺴﺎﻭﻱ ﻜﻤﻴﺔ ﺍﻟﺤﺭﻜﺔ‪ ،‬ﻓﻤﺜﻼﹰ ﺇﺫﺍ ﺃﺨﺫﻨﺎ ﺘﺎﺒﻊ‬
‫ﻻﻏﺭﺍﻨﺞ ﻟﺠﺴﻴﻡ ﻴﺘﺤﺭﻙ ﻋﻠﻰ ﺍﻟﻤﺤﻭﺭ ‪ ox‬ﺤﺭﻜﺔ ﺫﺍﺕ ﺒﻌﺩ ﻭﺍﺤﺩ( ﻓﺈﻥ ﺘﺎﺒﻊ‬
‫ﻻﻏﺭﺍﻨﺞ ﻟﻪ ﻫﻭ‪:‬‬
‫‪1‬‬
‫‪L  TV ‬‬ ‫) ‪mx  2  V(x‬‬
‫‪2‬‬

‫‪- 294 -‬‬

‫)‪Create PDF files without this message by purchasing novaPDF printer (http://www.novapdf.com‬‬
‫ﺤﻴﺙ ) ‪ V(x‬ﻫﻭ ﺍﻟﻜﻤﻭﻥ ﺍﻟﺘﻲ ﺘﺸﺘﻕ ﻤﻨﻪ ﺍﻟﻘﻭﺓ ﺍﻟﻤﺴﺒﺒﺔ ﻟﻠﺤﺭﻜﺔ‪ ،‬ﻨﻼﺤﻅ ﺃﻥ‪:‬‬
‫‪L‬‬ ‫‪L‬‬ ‫‪V‬‬
‫‪ mx  Px ,‬‬ ‫‪‬‬ ‫‪ (grad V ) x  Fx‬‬
‫‪x ‬‬ ‫‪x‬‬ ‫‪x‬‬
‫ﻭﺴﻨﺴﺘﻔﻴﺩ ﻤﻥ ﺍﻟﺨﺎﺼﺘﻴﻥ ﺍﻟﻤﺫﻜﻭﺭﺘﻴﻥ ﻻﻴﺠﺎﺩ ‪ L‬ﻓﻲ ﺍﻟﻨﻅﺭﻴﺔ ﺍﻟﻨﺴﺒﻴﺔ ﻟﺫﺍ ﻨﺄﺨﺫ‬
‫ﺍﻟﺘﺎﺒﻊ‪:‬‬
‫‪v2‬‬
‫‪L  mc 2 1 ‬‬ ‫) ‪ V(q j‬‬ ‫)‪(9.33‬‬
‫‪c2‬‬
‫ﻭﺒﺎﻟﻔﻌل ﻨﻼﺤﻅ ﺃﻥ ‪ L‬ﻫﺫﺍ ﻴﺤﻘﻕ ﻫﺎﺘﻴﻥ ﺍﻟﺨﺎﺼﺘﻴﻥ ﻷﻥ‪:‬‬
‫‪‬‬
‫‪‬‬
‫)‪ (grad V ) j  Fj (9.34‬‬
‫‪L‬‬ ‫‪mv‬‬ ‫‪L‬‬ ‫‪V‬‬
‫‪‬‬ ‫‪ P,‬‬ ‫‪‬‬
‫‪‬‬ ‫‪q j‬‬ ‫‪q j‬‬
‫‪v‬‬ ‫‪v2‬‬
‫‪1‬‬
‫‪c2‬‬
‫ﻭﺒﺎﻟﺘﺎﻟﻲ ﻴﻤﻜﻥ ﺍﻋﺘﺒﺎﺭ ‪ L‬ﺍﻟﻤﻌﺭﻑ ﺒﺎﻟﻌﻼﻗﺔ )‪ (9.33‬ﺒﻤﺜﺎﺒﺔ ﺘﺎﺒﻊ ﻻﻏﺭﺍﻨﺞ ﻓﻲ‬
‫ﺍﻟﻨﻅﺭﻴﺔ ﺍﻟﻨﺴﺒﻴﺔ‪ ،‬ﺃﻤﺎ ﻤﻌﺎﺩﻟﺔ ﺍﻟﺤﺭﻜﺔ ﻓﺘﻭﻀﻊ ﺒﺎﻟﺸﻜل‪:‬‬
‫)‪(9.35‬‬
‫‪d L‬‬ ‫‪L‬‬
‫‪‬‬ ‫‪0‬‬
‫‪dt q j q j‬‬

‫ﻭﻟﻠﺤﺼﻭل ﻋﻠﻰ ﺘﺎﺒﻊ ﻫﺎﻤﻠﺘﻭﻥ ﻨﻨﻁﻠﻕ ﻤﻥ ﺘﻌﺭﻴﻑ )‪ (8.45‬ﺤﻴﺙ ﻨﺠﺩ‪:‬‬


‫‪mv 2‬‬ ‫‪v2‬‬
‫‪H‬‬ ‫‪‬‬ ‫‪P jq j  L ‬‬ ‫‪ mc 2 1 ‬‬ ‫) ‪ V(q j‬‬
‫‪v‬‬ ‫‪2‬‬ ‫‪c2‬‬
‫‪1‬‬
‫‪c2‬‬
‫ﻭﻤﻨﻪ‪:‬‬
‫‪mc 2‬‬
‫‪H‬‬ ‫) ‪ V(q j‬‬ ‫)‪(9.36‬‬
‫‪2‬‬
‫‪v‬‬
‫‪1‬‬
‫‪c2‬‬
‫ﻭﻗﺩ ﺭﺃﻴﻨﺎ ﺴﺎﺒﻘﺎﹰ ﺃﻨﻪ ﻻﺒﺩ ﻤﻥ ﺍﻟﺘﻌﺒﻴﺭ ﻋﻥ ﺘﺎﺒﻊ ﻫﺎﻤﻠﺘﻭﻥ ﺒﺩﻻﻟﺔ ﺍﻟﻤﺘﺤﻭﻻﺕ‬
‫ﺍﻟﻘﺎﻨﻭﻨﻴﺔ ‪ P j‬ﻭ ‪ q j‬ﻟﺫﺍ ﻴﺠﺏ ﺤﺴﺎﺏ ‪ v 2‬ﺒﺩﻻﻟﺔ ‪ P 2‬ﻤﻥ ﺍﻟﻌﻼﻗﺎﺕ )‪ (9.2‬ﺤﻴﺙ‬
‫ﻨﺠﺩ ﺒﺘﺭﺒﻴﻊ ﺍﻟﺜﻼﺙ ﺍﻷﻭﻟﻰ‪:‬‬

‫‪- 295 -‬‬

‫)‪Create PDF files without this message by purchasing novaPDF printer (http://www.novapdf.com‬‬
‫‪m 2v 2‬‬
‫‪P 2  Px2  Py2  Pz2 ‬‬
‫‪v2‬‬
‫‪1‬‬
‫‪c2‬‬
‫ﻭﻤﻨﻪ‪:‬‬
‫‪v2‬‬ ‫‪p2‬‬
‫‪‬‬
‫‪c2‬‬ ‫‪P 2  m2c2‬‬
‫ﻭﺒﺎﻟﺘﺒﺩﻴل ﻓﻲ )‪ (9.36‬ﻨﺤﺼل ﺃﺨﻴﺭﺍﹰ ﻋﻠﻰ ﺘﺎﺒﻊ ﻫﺎﻤﻠﺘﻭﻥ ﺍﻟﺘﺎﻟﻲ‪:‬‬
‫) ‪H  P 2 c 2  m 2 c 4  V( q j‬‬ ‫)‪(9.37‬‬
‫ﻭﻫﺫﺍ ﺍﻟﺘﺎﺒﻊ ﻴﺠﺏ ﺃﻥ ﻴﺘﻁﺎﺒﻕ ﻤﻊ ﺍﻟﻁﺎﻗﺔ ﺍﻟﻜﻠﻴﺔ ﻷﻥ ﺍﻟﻘﻭﻯ ﻜﻤﻭﻨﻴﺔ‪.‬‬
‫ﻟﻨﺩﺭﺱ ﺍﻟﺤﺎﻟﺔ ﺍﻟﺨﺎﺼﺔ ﺍﻟﻬﺎﻤﺔ ﻭﻫﻲ ﻋﻨﺩﻤﺎ ﻴﻜﻭﻥ ‪ P  mc‬ﺤﻴﺙ ﻴﻤﻜﻥ‬
‫ﻭﻀﻊ ﺘﺎﺒﻊ ﻫﺎﻤﻠﺘﻭﻥ ﺒﺎﻟﺸﻜل‪:‬‬
‫‪H  Pc‬‬ ‫)‪(9.38‬‬
‫ﺇﻥ ﻫﺫﻩ ﺍﻟﺤﺎﻟﺔ ﺍﻟﺘﻲ ﺘﺤﺩﺙ ﻋﻨﺩﻤﺎ ﻴﻜﻭﻥ ﺍﻨﺩﻓﺎﻉ ﺍﻟﺠﺴﻴﻡ )ﻭﺒﺎﻟﺘﺎﻟﻲ ﺴﺭﻋﺘﻪ(‬
‫ﻜﺒﻴﺭ ﺠﺩﺍﹰ ﺘﺴﻤﻰ ﺍﻟﺤﺎﻟﺔ ﻓﻭﻕ ﺍﻟﻨﺴﺒﻴﺔ ‪ aultra-relativistic‬ﻭﻤﻥ ﺍﻟﺒﺩﻴﻬﻲ ﺃﻥ ﺍﻟﻌﻼﻗﺔ‬
‫)‪ (9.38‬ﻫﻲ ﻋﻼﻗﺔ ﺼﺤﻴﺤﺔ ﺘﻤﺎﻤﺎﹰ ﻭﻟﻴﺴﺕ ﺘﻘﺭﻴﺒﻴﺔ ﻋﻨﺩﻤﺎ ﺘﻨﻌﺩﻡ ﻜﺘﻠﺔ ﺍﻟﺠﺴﻴﻡ‬
‫ﻭﺘﺴﺘﺨﺩﻡ ﺍﻟﺼﻭﺭﺓ ﺨﺎﺼﺔ‪ ،‬ﻓﻲ ﺍﻟﺩﺭﺍﺴﺔ ﺍﻟﺤﺭﻜﻴﺔ ﻟﻠﺤﻭﺍﺩﺙ ﺍﻟﻔﻴﺯﻴﺎﺌﻴﺔ ﺍﻟﺘﻲ ﺘﺸﺎﺭﻙ‬
‫ﻓﻴﻬﺎ ﺍﻟﻔﻭﺘﻭﻨﺎﺕ ﺍﻟﻀﻭﺌﻴﺔ‪.‬‬
‫‪ -75‬ﻗﺎﻧﻮﻥ ﻣﺼﻮﻧﻴﺔ ﺍﻟﻄﺎﻗﺔ‪ -‬ﻛﻤﻴﺔ ﺍﳊﺮﻛﺔ ﰲ ﺍﻟﻔﻴﺰﻳﺎء ﺍﻟﻨﻮﻭﻳﺔ ‪:‬‬
‫ﻟﻡ ﺘﺒﺭﻫﻥ ﺍﻟﻨﺘﺎﺌﺞ ﺍﻟﺘﺠﺭﻴﺒﻴﺔ ﻓﻲ ﺍﻟﻔﻴﺯﻴﺎﺀ ﺍﻟﺤﺩﻴﺜﺔ ﻋﻠﻰ ﺼﺤﺔ ﻗﺎﻨﻭﻥ ﺤﻔﻅ‬
‫ﺍﻟﻁﺎﻗﺔ ﻭﻜﻤﻴﺔ ﺍﻟﺤﺭﻜﺔ ﻓﻲ ﺍﻟﻨﻅﺭﻴﺔ ﺍﻟﻨﺴﺒﻴﺔ ﻓﺤﺴﺏ‪ ،‬ﻭﺍﻨﻤﺎ ﺘﻌﺩﻯ ﺫﻟﻙ ﺇﻟﻰ ﺃﻥ ﻫﺫﺍ‬
‫ﺍﻟﻘﺎﻨﻭﻥ ﺃﺼﺒﺢ ﺃﺤﺩ ﺍﻟﻤﺒﺎﺩﻯﺀ ﺍﻷﺴﺎﺴﻴﺔ ﻓﻲ ﺍﻟﻔﻴﺯﻴﺎﺀ ﺍﻟﻨﻭﻭﻴﺔ‪ ،‬ﻭﺴﻨﻘﺘﺼﺭ ﻫﻨﺎ ﻋﻠﻰ‬
‫ﺒﻌﺽ ﺍﻷﻤﺜﻠﺔ ﺍﻟﺘﻲ ﺘﺒﻴﻥ ﻀﺭﻭﺭﺓ ﺍﻻﻋﺘﻤﺎﺩ ﻋﻠﻰ ﻨﺘﺎﺌﺞ ﺍﻟﻤﻴﻜﺎﻨﻴﻙ ﺍﻟﻨﺴﺒﻲ ﻟﺩﺭﺍﺴﺔ‬
‫‪Elementary‬‬ ‫ﺍﻟﺤﻭﺍﺩﺙ ﺍﻟﺘﻲ ﺘﺠﺭﻯ ﻓﻲ ﺍﻟﻨﻭﺍﺓ ﻭﺩﺭﺍﺴﺔ ﺍﻟﺠﺴﻴﻤﺎﺕ ﺍﻷﺴﺎﺴﻴﺔ‬
‫‪.Particles‬‬

‫‪- 296 -‬‬

‫)‪Create PDF files without this message by purchasing novaPDF printer (http://www.novapdf.com‬‬
‫ﺁ‪ -‬ﺍﻨﻘﺴﺎﻡ ﺍﻟﺠﺴﻴﻤﺎﺕ ﺍﻟﺫﺍﺘﻲ‪:‬‬
‫ﺴﻨﺭﻯ ﻜﻴﻑ ﺃﻥ ﻗﺎﻨﻭﻥ ﺤﻔﻅ ﺍﻟﻁﺎﻗﺔ ﻴﻀﻊ ﺸﺭﻭﻁﺎﹰ ﻤﻌﻴﻨﺔ ﻻﻤﻜﺎﻨﻴﺔ ﻫﺫﺍ‬
‫ﺍﻻﻨﻘﺴﺎﻡ‪ ،‬ﻟﺫﺍ ﻨﺩﺭﺱ ﺍﻨﻘﺴﺎﻡ ﺠﺴﻴﻡ ﻤﺎ ﻜﺘﻠﺘﻪ ‪ m‬ﺍﻟﻰ ﺠﺴﻤﻴﻥ ﺫﻭﻯ ﻜﺘﻠﺘﻴﻥ ﻤﺨﺘﻠﻔﺘﻴﻥ‬
‫‪ m1‬ﻭ ‪ . m 2‬ﻭﻟﻨﺩﺭﺱ ﺍﻟﺤﺎﺩﺜﺔ ﻓﻲ ﻤﺠﻤﻭﻋﺔ ﺍﺤﺩﺍﺜﻴﺔ ﻤﺭﻜﺯﻫﺎ ﻴﻘﻊ ﻓﻲ ﻤﺭﻜﺯ ﺜﻘل‬
‫ﺍﻟﺠﺴﻴﻡ ﺍﻟﻤﻨﻘﺴﻡ ﻭﻫﻭ ﺴﺎﻜﻥ )ﻤﺠﻤﻭﻋﺔ ﻤﺭﻜﺯ ﺍﻟﻜﺘﻠﺔ(‪.‬‬
‫ﻭﻫﻜﺫﺍ ﺘﻨﻌﺩﻡ ﻜﻤﻴﺔ ﺍﻟﺤﺭﻜﺔ ﺍﻟﻜﻠﻴﺔ ﻗﺒل ﺍﻻﻨﻘﺴﺎﻡ ﻭﺒﻌﺩﻩ‪ ،‬ﻭﻴﺒﻘﻰ ﻤﺭﺒﻊ ﺍﻟﻤﺘﺠﻪ‬
‫‪ ، P‬ﺍﻟﻤﻌﺭﻑ ﻓﻲ ﺍﻟﻔﺭﺍﻍ ﺍﻟﺭﺒﺎﻋﻲ ﺜﺎﺒﺘﺎﹰ ﺃﻴﻀﺎﹰ ﻭﻴﺴﺎﻭﻱ‪ ،‬ﻁﺒﻘﺎﹰ ﻟـ )‪:(9.2‬‬
‫‪E2‬‬
‫‪I  P2 ‬‬ ‫‪ m2c 2‬‬
‫‪2‬‬
‫‪c‬‬
‫ﻭﻤﻨﻪ‪:‬‬
‫‪P 2 c 2  E 2  m 2 c 4‬‬ ‫)‪(9.39‬‬
‫‪‬‬ ‫‪‬‬
‫ﻓﺈﺫﺍ ﺭﻤﺯﻨﺎ ﻟﻜﻤﻴﺔ ﺤﺭﻜﺔ ﻜل ﻤﻥ ﺍﻟﺠﺴﻴﻤﻴﻥ ﺍﻟﻨﺎﺘﺠﻴﻥ ﺒـ ‪ P1‬ﻭ ‪) P2‬ﻓﻲ‬
‫ﺍﻟﻔﺭﺍﻍ ﺍﻟﺜﻼﺜﻲ( ﻓﺈﻥ ﻤﺠﻤﻭﻋﻬﻤﺎ ﻴﺠﺏ ﺃﻥ ﻴﺴﺎﻭﻱ ﺍﻟﺼﻔﺭ ﺃﻴﻀﺎﹰ ﺒﻌﺩ ﺍﻻﻨﻘﺴﺎﻡ ﺃﻱ‪:‬‬
‫‪‬‬ ‫‪‬‬
‫‪P1  P2  0‬‬
‫ﺒﻔﺭﺽ ﺃﻥ ‪ E1‬ﻭ ‪ E 2‬ﻫﻤﺎ ﻁﺎﻗﺘﺎ ﺍﻟﺠﺴﻴﻤﻴﻥ‪ ،‬ﺜﻡ ﺒﺎﻟﺘﺒﺩﻴل ﻓﻲ )‪ (9.39‬ﻨﺠـﺩ‬
‫ﺍﻟﻤﻌﺎﺩﻟﺔ‪:‬‬
‫‪‬‬ ‫‪‬‬
‫‪(P1  P2 ) 2 c 2  (E1  E 2 ) 2  m 2 c 4‬‬
‫ﺃﻭ‪:‬‬
‫‪mc 2  E1  E 2‬‬ ‫)‪(9.40‬‬
‫ﻭﺇﺫﺍ ﻓﺭﻀﻨﺎ ﺃﻥ‪:‬‬
‫‪E(1)  E (kin‬‬
‫)‪1‬‬
‫‪ m1c 2 , E ( 2)  E(kin‬‬
‫)‪2‬‬
‫‪ m 2c 2‬‬ ‫)‪(9.41‬‬
‫ﻓﺈﻥ )‪ (9.40‬ﺘﻭﻀﻊ ﺒﺎﻟﺸﻜل‪:‬‬
‫‪mc 2  (m1  m 2 )c 2  E (kin‬‬
‫)‪1‬‬
‫‪ E (kin‬‬
‫)‪2‬‬
‫)‪(9.42‬‬
‫‪ E(1‬ﻤﻘﺩﺍﺭﺍﻥ ﻤﻭﺠﺒﺎﻥ ﻓﻼﺒﺩ ﻟﻜﻲ ﻴﺤﺩﺙ‬
‫ﻭﺒﻤﺎ ﺃﻥ ﻜﻼﹰ ﻤﻥ ‪ E kin‬ﻭ ‪kin‬‬
‫)‬ ‫)‪(2‬‬

‫ﺍﻻﻨﻘﺴﺎﻡ ﺍﻟﺴﺎﺒﻕ ﺃﻥ ﻴﻜﻭﻥ‪:‬‬

‫‪- 297 -‬‬

‫)‪Create PDF files without this message by purchasing novaPDF printer (http://www.novapdf.com‬‬
‫‪mc 2  (m1  m 2 )c 2  m  m1  m 2‬‬ ‫)‪(9.43‬‬
‫ﻭﻫﺫﺍ ﻴﻌﻨﻲ ﺃﻨﻪ ﻻﻴﻤﻜﻥ ﻟﻠﺠﺴﻴﻡ ﺃﻥ ﻴﻨﻘﺴﻡ ﻤﻥ ﺘﻠﻘﺎﺀ ﻨﻔﺴﻪ ﺇﻻ ﺇﺫﺍ ﻜﺎﻨﺕ ﻜﺘﻠﺘﻪ‬
‫ﺃﻜﺒﺭ ﻤﻥ ﻤﺠﻤﻭﻋﺔ ﻜﺘﻠﺘﻲ ﺍﻟﺠﺴﻴﻤﻴﻥ ﺍﻟﻨﺎﺘﺠﻴﻥ ﻋﻥ ﺍﻨﻘﺴﺎﻤﻪ‪ .‬ﻭﻤﻥ ﺍﻟﻭﺍﻀﺢ ﺃﻥ ﻗﺎﻨﻭﻥ‬
‫ﺤﻔﻅ ﺍﻟﻜﺘﻠﺔ ﺒﺎﻟﻤﻌﻨﻰ ﺍﻟﻜﻼﺴﻴﻜﻲ ﻏﻴﺭ ﻤﺤﻘﻕ ﻫﻨﺎ‪.‬‬
‫ﺃﻤﺎ ﻋﻨﺩﻤﺎ ‪ m  m1  m 2‬ﻓﻼﺒﺩ ﻤﻥ ﺼﺭﻑ ﻁﺎﻗﺔ ﺨﺎﺭﺠﻴﺔ ﻟﺘﺤﻘﻴﻕ ﺍﻻﻨﻘﺴﺎﻡ‬
‫ﺍﻟﺫﺍﺘﻲ ﺒﺤﻴﺙ ﻴﺘﻡ ﺍﻟﺘﻐﻠﺏ ﺒﻭﺍﺴﻁﺘﻬﺎ ﻋﻠﻰ ﻁﺎﻗﺔ ﺭﺒﻁ ﺍﻟﺠﺴﻴﻤﻴﻥ ‪ m1‬ﻭ ‪ m 2‬ﺩﺍﺨل‬
‫ﺍﻟﺠﺴﻴﻡ ‪ ،m‬ﻭﻴﺼﺭﻑ ﺍﻟﻤﻘﺩﺍﺭ ﺍﻟﺒﺎﻗﻲ ﻤﻥ ﺍﻟﻁﺎﻗﺔ ﻋﻠﻰ ﺍﻜﺴﺎﺒﻬﻤﺎ ﺍﻟﻁﺎﻗﺔ ﺍﻟﺤﺭﻜﻴﺔ‬
‫‪. E(2‬‬
‫‪ E kin‬ﻭ ‪kin‬‬
‫)‬ ‫)‪(1‬‬

‫ﻟﻨﺤﺴﺏ ﺍﻵﻥ ﻁﺎﻗﺘﻲ ﺍﻟﺠﺴﻴﻤﻴﻥ ﺍﻟﻨﺎﺘﺠﻴﻥ ﻋﻥ ﺍﻻﻨﻘﺴﺎﻡ ﻓﻨﻼﺤﻅ ﺃﻭﻻﹰ ﺃﻥ‬


‫| ‪ | P1 || P2‬ﻭﺒﺎﻟﺘﺎﻟﻲ ‪ P12  P22‬ﻭﻨﺴﺘﻔﻴﺩ ﻤﻥ ﺍﻟﻌﻼﻗﺘﻴﻥ‪:‬‬
‫‪E1  P12 c 2  m12 c 4‬‬ ‫‪,‬‬ ‫‪E 2  P22c 2  m 22 c 4‬‬
‫ﻓﻤﻥ ﺍﻟﺜﺎﻨﻴﺔ ﻨﺠﺩ‪:‬‬
‫‪E 22  P22 c 2  m 22 c 4  P12 c 2  m 22 c 4  E12  m12 c 4  m 22 c 4‬‬
‫ﺃﻭ‪:‬‬
‫‪E 22  E12  m 22 c 4  m12 c 4‬‬ ‫)‪(9.44‬‬
‫ﻭﻤﻥ )‪ (9.40‬ﻨﺠﺩ‪:‬‬
‫‪E12  (mc 2  E 2 ) 2‬‬ ‫)‪(9.45‬‬
‫ﻭﺍﻟﻤﻌﺎﺩﻟﺘﺎﻥ )‪ (9.44‬ﻭ )‪ ( 9.45‬ﻫﻤﺎ ﻤﻌﺎﺩﻟﺘﺎﻥ ﺠﺒﺭﻴﺘﺎﻥ ﺒﺎﻟﻨﺴﺒﺔ ‪ E1‬ﻭ ‪E 2‬‬
‫ﺒﺤﻠﻬﺎ ﻨﺤﺼل ﻋﻠﻰ ﻜل ﻤﻨﻬﻤﺎ ﺤﻴﺙ ﻨﺠﺩ ﺃﺨﻴﺭﺍﹰ‪:‬‬
‫‪(m 2  m 22  m12 )c 2‬‬ ‫‪(m 2  m12  m 22 )c 2‬‬
‫‪E1 ‬‬ ‫‪, E2 ‬‬ ‫)‪(9.46‬‬
‫‪2m‬‬ ‫‪2m‬‬
‫ﺏ‪ -‬ﺍﺴﺘﻘﺭﺍﺭ ﺍﻟﻨﻭﻴﺎﺕ ﺍﻟﺫﺭﻴﺔ‪:‬‬
‫ﺍﻥ ﺍﺴﺘﻘﺭﺍﺭ ﻨﻭﺍﺓ ﻤﺎ ﻴﻌﻨﻲ ﻋﺩﻡ ﺍﻤﻜﺎﻨﻴﺔ ﺍﻨﻘﺴﺎﻤﻬﺎ ﻓﻲ ﺍﻟﻭﻗﺕ ﺍﻟﺤﺎﻀﺭ ﻋﻠﻰ‬
‫ﺍﻷﻗل‪ ،‬ﻭﺴﻨﺭﻯ ﺍﻵﻥ‪ ،‬ﺍﻨﻁﻼﻗﺎﹰ ﻤﻥ ﻗﻭﺍﻨﻴﻥ ﺍﻟﻤﻴﻜﺎﻨﻴﻙ ﺍﻟﻨﺴﺒﻲ ﺃﻨﻪ ﻴﻤﻜﻥ ﺍﻴﺠﺎﺩ ﺍﻟﺸﺭﻁ‬
‫ﺍﻷﺴﺎﺴﻲ ﻟﻬﺫﺍ ﺍﻻﺴﺘﻘﺭﺍﺭ‪.‬‬

‫‪- 298 -‬‬

‫)‪Create PDF files without this message by purchasing novaPDF printer (http://www.novapdf.com‬‬
‫ﻟﺘﻜﻥ ﺍﻟﻨﻭﺍﺓ ‪ X‬ﺍﻟﺘﻲ ﻜﺘﻠﺘﻬﺎ ‪ m‬ﻭﺘﺤﻭﻱ ‪ Z‬ﺒﺭﻭﺘﻭﻨﺎ ﻭ ‪ A‬ﻨﻭﻜﻠﻭﻨﺎﹰ )ﺒﺭﻭﺘﻭﻥ ﺃﻭ‬
‫ﻨﺘﺭﻭﻥ( ﻭ ‪ A-Z‬ﻨﺘﺭﻭﻨﺎﹰ‪ ،‬ﻭﻤﻥ ﺍﻟﻤﻌﻠﻭﻡ ﺃﻥ ﺍﻟﻨﻭﻜﻠﻭﻨﺎﺕ ﺘﺒﻘﻰ ﻓﻲ ﺤﺭﻜﺔ ﺩﺍﺌﻤﺔ ﻀﻤﻥ‬
‫ﺍﻟﻨﻭﺍﺓ ﻭﺒﺎﻟﺭﻏﻡ ﻤﻥ ﺫﻟﻙ ﺘﻅل ﻤﺭﺘﺒﻁﺔ ﻤﻊ ﺒﻌﻀﻬﺎ ﺍﻟﺒﻌﺽ ﺘﺤﺕ ﺘﺄﺜﻴﺭ ﺍﻟﻜﻤﻭﻥ‬
‫ﺍﻟﻨﻭﻭﻱ ﺍﻟﻘﻭﻱ‪.‬‬
‫ﻭﺍﺫﺍ ﻜﺎﻨﺕ ﻁﺎﻗﺔ ﺍﻟﺴﻜﻭﻥ ﻟﻠﻨﻭﺍﺓ ﻜﻜل ﺘﺴﺎﻭﻱ ‪ mc2‬ﻭﻁﺎﻗﺔ ﺍﻟﺴﻜﻭﻥ ﻟﻜل‬
‫ﻓﻤﻥ ﺍﻟﻭﺍﻀﺢ ﺃﻥ ﺸﺭﻁ ﺍﻨﻘﺴﺎﻡ ﻫﺫﻩ‬ ‫‪ mic 2‬‬ ‫ﺍﻟﻨﻭﻜﻠﻭﻨﺎﺕ ﻓﻲ ﺩﺍﺨﻠﻬﺎ ﺘﺴﺎﻭﻱ‬
‫ﺍﻟﻨﻭﺍﺓ ﺍﻟﻰ ﻤﻜﻭﻨﺎﺘﻬﺎ ﺫﺍﺕ ﺍﻟﻜﺘل ‪ mi‬ﻟﻴﺱ ﺍﻻ ﺘﻌﻤﻴﻤﺎﹰ ﻤﺒﺎﺸﺭﺍﹰ ﻟﻤﺎ ﺭﺃﻴﻨﺎﻩ ﻓﻲ ﺍﻟﻔﻘﺭﺓ‬
‫ﺍﻟﺴﺎﺒﻘﺔ ﺃﻱ‪:‬‬
‫‪mc 2   m i c 2‬‬ ‫)‪(9.47‬‬
‫ﺍﻥ ﺸﺭﻁ ﺍﺴﺘﻘﺭﺍﺭﻫﺎ ﻓﻬﻭ‪:‬‬
‫‪ mi c 2  mc2‬‬ ‫)‪(9.48‬‬
‫ﻭﻴﻌﺭﻑ ﻁﺎﻗﺔ ﺍﺭﺘﺒﺎﻁ ﺍﻟﻨﻭﺍﺓ ﺒﺎﻟﻌﻼﻗﺔ‪:‬‬
‫‪mc 2   m i c 2  mc 2‬‬
‫ﻓﺈﺫﺍ ﻜﺎﻥ ﺴﺎﻟﺒﺎﹰ ﻓﺈﻥ ﺍﻟﻨﻭﺍﺓ ﻴﻤﻜﻥ ﺃﻥ ﺘﻨﻘﺴﻡ ﺫﺍﺘﻴﺎﹰ‪ ،‬ﺃﻤﺎ ﺇﺫﺍ ﻜﺎﻥ ﻤﻭﺠﺒﺎﹰ ﻓﻼﺘﻨﻘﺴﻡ‬
‫ﺇﻻ ﺒﺎﻀﺎﻓﺔ ﻁﺎﻗﺔ ﻤﻥ ﺍﻟﺨﺎﺭﺝ ﻭﻴﺘﻡ ﺫﻟﻙ ﺒﻘﺫﻓﻬﺎ ﺒﺠﺴﻴﻡ ﺁﺨﺭ ﻁﺎﻗﺘﻪ ﺃﻜﺒﺭ ﻤﻥ ﻁﺎﻗﺔ‬
‫ﺍﻻﺭﺘﺒﺎﻁ‪ .‬ﻭﻴﺴﻤﻰ ﺍﻟﻤﻘﺩﺍﺭ‪:‬‬
‫‪m   m i  m‬‬ ‫)‪(9.49‬‬
‫ﻨﻘﺹ ﺍﻟﻜﺘﻠﺔ‪ ،‬ﻭﻫﻭ ﻴﻌﻁﻴﻨﺎ ﻓﻜﺭﺓ ﺃﻴﻀﺎﹰ ﻋﻥ ﺍﺴﺘﻘﺭﺍﺭ ﺍﻟﻨﻭﺍﺓ‪ ،‬ﻭﻨﺅﻜﺩ ﻫﻨﺎ ﺃﻥ‬
‫ﺍﻟﺸﺭﻁ )‪ (9.47‬ﻏﻴﺭ ﻜﺎﻑ ﻻﻨﻘﺴﺎﻡ ﺍﻟﻨﻭﺍﺓ ﻭﻴﺠﺏ ﺃﻥ ﻴﺘﺤﻘﻕ ﺃﻴﻀﺎﹰ ﻤﺎﻴﺴﻤﻰ ﻗﺎﻨﻭﻥ‬
‫‪‬‬
‫ﻤﺼﻭﻨﻴﺔ ﺍﻟﺸﺤﻨﺔ ﺍﻟﻜﻬﺭﺒﺎﺌﻴﺔ ﻭﻋﺩﺩ ﺍﻟﻨﻭﻜﻠﻭﻨﺎﺕ ﻭﺍﻟﻌﺯﻡ ﺍﻟﻜﺘﻠﻲ ‪ j‬ﺃﻱ ﺃﻨﻪ ﻋﻨﺩ‬
‫ﺍﻨﻘﺴﺎﻡ ﻨﻭﺍﺓ ﻜﺘﻠﺘﻬﺎ ‪ m‬ﺇﻟﻰ ﻗﺴﻤﻴﻥ ﻜﺘﻠﺘﺎﻫﻤﺎ ‪ m1‬ﻭ ‪ m 2‬ﻴﺠﺏ ﺃﻥ ﺘﺘﺤﻘﻕ ﺍﻟﻌﻼﻗﺎﺕ‬
‫ﺍﻟﺘﺎﻟﻴﺔ‪:‬‬
‫‪‬‬ ‫‪‬‬ ‫‪‬‬
‫)‪J  J1  J 2 , Z1  Z 2  Z , A  A 1  A 2 (9.50‬‬

‫‪- 299 -‬‬

‫)‪Create PDF files without this message by purchasing novaPDF printer (http://www.novapdf.com‬‬
‫‪‬‬
‫ﻭ ‪ J‬ﺍﻟﻌﺯﻡ ﺍﻟﻜﻠﻲ ﻟﻠﺠﺴﻴﻡ‬ ‫‪(i  1,2) i‬‬ ‫ﺤﻴﺙ ‪ ji‬ﻫﻭ ﺍﻟﻌﺯﻡ ﺍﻟﻜﻠﻲ ﻟﻠﺠﺴﻴﻡ‬
‫ﺍﻟﻤﻨﻘﺴﻡ‪ ،‬ﻭ ‪ A‬ﻋﺩﺩ ﺍﻟﻨﻭﻜﻠﻭﻨﺎﺕ‪ ،‬ﻭ ‪ Z‬ﻋﺩﺩ ﺍﻟﺒﺭﻭﺘﻭﻨﺎﺕ‪.‬‬
‫ﻟﻨﺄﺨﺫ ﻤﺜﺎﻻﹰ ﻋﻠﻰ ﻤﺎﺴﺒﻕ‪:‬‬
‫ﻟﺘﻜﻥ ﻨﻭﺍﺓ ﺍﻟﺒﻴﺭﻴﻠﻴﻭﻡ ‪ Be 84‬ﺍﻟﺘﻲ ﻜﺘﻠﺘﻬﺎ ‪ m  8,00785 a.u.m‬ﺃﺼﻐﺭ ﻤﻥ‬
‫ﻤﺠﻤﻭﻉ ﺍﻟﺒﺭﻭﺘﻭﻨﺎﺕ ﺍﻷﺭﺒﻌﺔ ﻭﺍﻟﻨﺘﺭﻭﻨﺎﺕ ﺍﻷﺭﺒﻌﺔ ﺍﻟﻤﻜﻭﻨﺔ ﻟﻬﺎ‪ ،‬ﺃﻱ ﺃﻥ‪:‬‬
‫‪ m i  4  1,008123  4  1,00893  8,0682121‬‬ ‫‪m‬‬
‫ﻭﺒﺎﻟﺘﺎﻟﻲ ﺘﻌﺘﺒﺭ ﻫﺫﻩ ﺍﻟﻨﻭﺍﺓ ﻤﺴﺘﻘﺭﺓ ﺒﺎﻟﻨﺴﺒﺔ ﻟﻼﻨﻘﺴﺎﻡ ﺍﻟﻰ ﺜﻤﺎﻨﻴﺔ ﻨﻭﻜﻠﻭﻨﺎﺕ ﺃﻤﺎ‬
‫ﺇﺫﺍ ﺤﺴﺒﻨﺎ ﺍﻤﻜﺎﻨﻴﺔ ﺍﻨﻘﺴﺎﻤﻬﺎ ﺍﻟﻰ ﻨﻭﺍﺘﻲ ﻫﻴﻠﻴﻭﻡ ﻜﺘﻠﺔ ﻜلٍ ﻤﻨﻬﻤﺎ ‪ 4,0039‬ﺃﻱ ﺃﻥ‬
‫ﻤﺠﻤﻭﻉ ﻜﺘﻠﺘﻴﻬﻤﺎ‪:‬‬
‫‪ m i  2  40039  8,00780 a.u.m‬‬
‫‪ Be 84‬ﻗﻠﻘﺔ‬ ‫‪  m i  m‬ﻭﺒﺎﻟﺘﺎﻟﻲ ﺘﻌﺒﺘﺭ ﻨﻭﺍﺓ ﺍﻟﺒﻴﺭﻴﻠﻴﻭﻡ‬ ‫ﻭﻫﻜﺫﺍ ﻨﺠﺩ ﺃﻥ‬
‫ﺒﺎﻟﻨﺴﺒﺔ ﻻﻨﻘﺴﺎﻤﻬﺎ ﺇﻟﻰ ﻨﻭﺍﺘﻲ ﻫﻴﻠﻴﻭﻡ‪ ،‬ﻭﻫﻲ ﺘﻨﻘﺴﻡ ﻁﻭﻋﻴﺎﹰ ﺇﻟﻰ ‪ . 2He 42‬ﻭﻟﻜﻥ ﺇﺫﺍ‬
‫ﺃﺨﺫﻨﺎ ﻨﻅﻴﺭﺍﹰ ﺁﺨﺭ ﻟﻠﻬﻠﻴﻭﻡ ﻫﻭ ‪ Be 94‬ﻭﺤﺴﺒﻨﺎ ﺍﻤﻜﺎﻨﻴﺔ ﺍﻨﻘﺴﺎﻤﻪ ﻷﻱ ﻤﻥ ﺍﻷﺠﺯﺍﺀ ﺍﻟﺘﻲ‬
‫ﻭﺒﺎﻟﺘﺎﻟﻲ ﻓﺈﻥ ﻨﻭﺍﺓ ‪ Be 94‬ﺘﻜﻭﻥ‬ ‫‪ mi  m‬‬ ‫ﻴﺘﻜﻭﻥ ﻤﻨﻬﺎ ﻓﺈﻨﻨﺎ ﻨﻼﺤﻅ ﺩﺍﺌﻤ ﺎﹰ ﺃﻥ‬
‫ﻤﺴﺘﻘﺭﺓ ﻭﻻﺘﻨﻘﺴﻡ ﻤﻥ ﺘﻠﻘﺎﺀ ﻨﻔﺴﻬﺎ ﺃﺒﺩ ﺍﹰ‪ .‬ﻭﻫﻜﺫﺍ ﻴﻤﻜﻥ ﻤﻌﺭﻓﺔ ﺍﻤﻜﺎﻨﻴﺔ ﺍﻨﻘﺴﺎﻡ‬
‫ﻭﺍﺴﺘﻘﺭﺍﺭ ﺍﻟﻨﻭﻴﺎﺕ ﺍﻟﺫﺭﻴﺔ ﺒﻌﺩ ﻤﻌﺭﻓﺔ ﺍﻟﻜﺘل ﺍﻟﺘﻲ ﺘﺘﺄﻟﻑ ﻤﻨﻬﺎ‪.‬‬
‫ﺝ‪ -‬ﺍﻟﻁﺎﻗﺔ ﺍﻟﻨﺎﺘﺠﺔ ﻋﻥ ﺍﻟﺘﻔﺎﻋﻼﺕ ﺍﻟﻨﻭﻭﻴﺔ‪:‬‬
‫ﻟﻴﻜﻥ ﺍﻟﺘﻔﺎﻋل‪:‬‬
‫‪AB  C D‬‬ ‫)‪(9.51‬‬
‫ﻭﻟﻨﺤﺴﺏ ﺍﻟﻁﺎﻗﺔ ﺍﻟﻨﺎﺘﺠﺔ ﻋﻨﻪ ﺍﻋﺘﻤﺎﺩﺍﹰ ﻋﻠﻰ ﻤﻔﺎﻫﻴﻡ ﺍﻟﻨﻅﺭﻴﺔ ﺍﻟﻨﺴﺒﻴﺔ‪ ،‬ﻟﺫﺍ‬
‫ﻨﺴﺘﻔﻴﺩ ﻤﻥ ﻗﺎﻨﻭﻥ ﺤﻔﻅ ﺍﻟﻁﺎﻗﺔ ﺍﻟﺫﻱ ﻴﻭﻀﻊ ﺒﺎﻟﺸﻜل‪:‬‬
‫‪E(kin‬‬
‫)‪A‬‬
‫‪ M A C 2  E (kin‬‬
‫)‪B‬‬
‫‪ m B C 2  E (kin‬‬
‫)‪C‬‬
‫‪ m C C 2  E(kin‬‬
‫)‪D‬‬
‫)‪ m D c 2 (9.52‬‬
‫ﻭﻴﻬﻤﻨﺎ ﻤﻥ ﺫﻟﻙ ﺍﻟﻤﻘﺩﺍﺭ ‪ Q‬ﺍﻟﻨﺎﺘﺞ ﻋﻥ ﻓﺭﻕ ﺍﻟﻜﺘﻠﺔ ﺍﻟﺫﻱ ﻴﺴﺎﻭﻱ‪:‬‬
‫‪‬‬
‫‪Q  (M A  M B )  (M C  M D )C 2‬‬ ‫‪‬‬ ‫)‪(9.53‬‬

‫‪- 300 -‬‬

‫)‪Create PDF files without this message by purchasing novaPDF printer (http://www.novapdf.com‬‬
‫ﺍﻟﺫﻱ ﻴﺤﺴﺏ ﺒﺴﻬﻭﻟﺔ ﺇﺫﺍ ﻋﻠﻤﻨﺎ ﻜﺘل ﺍﻟﻨﻭﻴﺎﺕ ﺍﻟﻤﺸﺎﺭﻜﺔ ﻋﻥ ﺍﻟﺘﻔﺎﻋل ﻭﺍﻟﻨﺎﺘﺠﺔ‬
‫‪E (kin‬‬
‫)‪B‬‬
‫ﻋﻨﻪ‪ .‬ﻴﺘﺤﻘﻕ ﺍﻟﺘﻔﺎﻋل ﺍﻟﺴﺎﺒﻕ ﻋﺎﺩﺓﹰ ﺒﻘﺫﻑ ﺍﻟﻨﻭﺍﺓ ‪ B‬ﺍﻟﺘﻲ ﺘﻌﺘﺒﺭ ﺴﺎﻜﻨﺔ ‪ 0‬‬
‫ﺒﻭﺍﺴﻁﺔ ﺍﻟﻨﻭﺍﺓ ﺫﺍﺕ ﺍﻟﻁﺎﻗﺔ ﺍﻟﻤﻌﻠﻭﻤﺔ‪ .‬ﺃﻤﺎ ﺍﻟﻁﺎﻗﺔ ﺍﻟﺤﺭﻜﻴﺔ ﻟﻜل ﻤﻥ ﺍﻟﻨﻭﺍﺘﻴﻥ‬
‫ﺍﻟﻨﺎﺘﺠﺘﻴﻥ ﻓﻴﻤﻜﻥ ﻗﻴﺎﺴﻬﺎ ﻭﺒﺎﻟﺘﺎﻟﻲ ﺤﺴﺎﺏ ﺍﻟﻤﻘﺩﺍﺭ ‪ Q‬ﻤﻥ ﺍﻟﻌﻼﻗﺔ )‪ (9.52‬ﻭﺒﻤﻘﺎﺭﻨﺘﻪ‬
‫ﻤﻊ ﺍﻟﻌﻼﻗﺔ )‪ (9.53‬ﻴﻤﻜﻥ ﺍﻟﺘﺤﻘﻕ ﻤﻥ ﻗﺎﻨﻭﻥ ﺘﻌﺎﺩل ﺍﻟﻁﺎﻗﺔ ﻭﺍﻟﻜﺘﻠﺔ‪.‬‬
‫ﻟﻨﺩﺭﺱ ﻋﻠﻰ ﺴﺒﻴل ﺍﻟﻤﺜﺎل ﺍﻟﺘﻔﺎﻋل ﺍﻟﻨﻭﻭﻱ‪:‬‬
‫‪Li 73  H11  2He 42‬‬ ‫)‪(9.54‬‬
‫ﺇﻥ ﻜﺘل ﺍﻟﻨﻭﻯ ﺍﻟﻤﺸﺎﺭﻜﺔ ﻓﻲ ﺍﻟﺘﻔﺎﻋل ﻭﻫﻲ ﺍﻟﻠﻴﺘﻴﻭﻡ ﻭﺍﻟﻬﻴﺩﺭﻭﺠﻴﻥ‪ ،‬ﻤﻘﺎﺴﺔ‬
‫ﺒﺩﺭﺠﺔ ﻤﻤﺘﺎﺯﺓ ﻤﻥ ﺍﻟﺩﻗﺔ ﻫﻲ‪:‬‬
‫‪m(Li 73 )  7,01822 a.u.m , m( H11 )  1,00812 a.u.m‬‬
‫ﻭﻤﺠﻤﻭﻉ ﻜﺘﻠﺘﻴﻬﻤﺎ ﻴﺴﺎﻭﻱ ‪ 8,02634 a.u.m‬ﺃﻤﺎ ﺍﻟﻜﺘﻠﺔ ﺍﻟﻨﺎﺘﺠﺔ ﻋﻥ ﺍﻟﺘﻔﺎﻋل‬
‫ﻓﻬﻲ‪:‬‬
‫‪2m(He 22 )  2  4,0039  8,0078 a.u.m‬‬
‫ﺃﻤﺎ ﻓﺭﻕ ﺍﻟﻜﺘﻠﺔ ﻓﻬﻭ‪ m  0,0185 a.u.m :‬ﻭﺍﻟﻁﺎﻗﺔ ﺍﻟﻤﻘﺎﺒﻠﺔ ﻟﻪ‪:‬‬
‫‪E  mc 2  17,2 MeV‬‬ ‫)‪(9.55‬‬
‫ﻭﻫﺫﻩ ﺍﻟﻘﻴﻤﺔ ﺘﺘﻭﺍﻓﻕ ﻤﻊ ﺍﻟﻘﻴﻤﺔ ﺍﻟﻤﻘﺎﺴﺔ ﺘﺠﺭﻴﺒﻴﺎﹰ‪ ،‬ﻤﻊ ﺍﻟﻌﻠﻡ ﺃﻨﻨﺎ ﺍﺴﺘﺨﺩﻤﻨﺎ ﻓﻲ‬
‫ﺍﻟﺤﺴﺎﺒﺎﺕ ﺍﻟﻤﻌﻁﻴﺎﺕ ﺍﻟﺘﺎﻟﻴﺔ‪:‬‬
‫‪1aum  1,66  10 24 g. 1 eV  10 6 MeV  1,602  10 12 erg‬‬
‫ﻭﺍﻀﺢ ﺃﻴﻀﺎﹰ ﻤﻥ ﻫﺫﺍ ﺍﻟﻤﺜﺎل ﺃﻨﻪ ﻟﻴﺱ ﻟﻘﺎﻨﻭﻥ ﻤﺼﻭﻨﻴﺔ ﺍﻟﻜﺘﻠﺔ ﺃﻱ ﻤﻌﻨﻰ؛ ﻷﻥ‬
‫ﻜﺘﻠﺔ ‪ m‬ﺘﺨﺘﻔﻲ ﺘﻤﺎﻤﺎﹰ ﻭﺘﺘﺤﻭل ﺇﻟﻰ ﻁﺎﻗﺔ‪.‬‬
‫ﺩ‪ -‬ﺍﻨﻘﺴﺎﻡ ﺍﻟﺠﺴﻴﻤﺎﺕ ﺍﻷﺴﺎﺴﻴﺔ‪ :‬ﻟﻘﺩ ﻟﻭﺤﻅ ﺃﻥ ﻜﺜﻴﺭﺍﹰ ﻤﻥ ﺍﻟﺠﺴﻴﻤﺎﺕ‬
‫ﺍﻷﺴﺎﺴﻴﺔ ﻏﻴﺭ ﻤﺴﺘﻘﺭ ﺒﺎﻟﻨﺴﺒﺔ ﻻﻨﻘﺴﺎﻤﻪ ﺍﻟﻰ ﺠﺴﻴﻤﺎﺕ ﺃﺨﺭﻯ ﺃﺒﺴﻁ ﻤﻨﻪ‪ .‬ﻭﻟﻥ‬
‫ﻨﺩﺭﺱ ﻫﺫﻩ ﺍﻟﻅﺎﻫﺭﺓ ﺒﺎﺴﻬﺎﺏ ﻷﻨﻬﺎ ﻤﻥ ﺍﺨﺘﺼﺎﺹ ﺍﻟﻔﻴﺯﻴﺎﺀ ﺍﻟﻨﻭﻭﻴﺔ‪ .‬ﻭﺴﻨﻜﺘﻔﻲ‬
‫ﺒﺎﻴﺭﺍﺩ ﺒﻌﺽ ﺍﻷﻤﺜﻠﺔ ﺍﻟﺘﻲ ﺘﺒﻴﻥ ﺃﻫﻤﻴﺔ ﺘﻁﺒﻴﻕ ﻗﻭﺍﻨﻴﻥ ﺤﻔﻅ ﺍﻟﻁﺎﻗﺔ ﻭﻜﻤﻴﺔ ﺍﻟﺤﺭﻜﺔ‬
‫ﻋﻨﺩ ﺩﺭﺍﺴﺔ ﺍﻨﻘﺴﺎﻡ ﻤﺎ‪ ،‬ﻭﺴﻨﻘﺘﺼﺭ ﻋﻠﻰ ﺍﻨﻘﺴﺎﻡ ﺍﻟﻤﻴﺯﻭﻨﺎﺕ ‪ ‬ﻭ ‪ . ‬ﻓﻤﻥ ﺍﻟﻤﻌﻠﻭﻡ‬

‫‪- 301 -‬‬

‫)‪Create PDF files without this message by purchasing novaPDF printer (http://www.novapdf.com‬‬
‫ﺘﻭﺠﺩ ﻋﻠﻰ ﺜﻼﺜﺔ ﺃﻨﻭﺍﻉ ‪  ‬ﻭ ‪  ‬ﻭ ‪)  0‬ﻤﺸﺤﻭﻨﺔ ﺍﻴﺠﺎﺒﻴﺎﹰ‬ ‫‪‬‬ ‫ﺃﻥ ﺍﻟﻤﻴﺯﻭﻨﺎﺕ‬
‫ﻭﺴﻠﺒﻴﺎﹰ ﻭﻤﻌﺘﺩﻟﺔ( ﻭﺃﻥ ﻜﺘﻠﺔ ﺍﻟﻤﻴﺯﻭﻥ ﺍﻟﻤﺸﺤﻭﻥ ﺘﺴﺎﻭﻱ ‪ 273‬ﻤﺭﺓ ﻜﺘﻠﺔ ﺍﻻﻟﻜﺘﺭﻭﻥ‬
‫ﻭﻟﻜﻨﻪ ﻻﻴﻠﺒﺙ ﺃﻥ ﻴﻨﻘﺴﻡ ﺒﻌﺩ ﻭﻗﺕ ﻗﺼﻴﺭ ﺠﺩ ﺍﹰ ﻤﻥ ﻭﻻﺩﺘﻪ‪ ،‬ﻭﻗﺩ ﺃﺜﺒﺘﺕ ﺍﻟﺘﺠﺎﺭﺏ ﺃﻥ‬
‫ﻋﻤﺭﻩ ﻴﺴﺎﻭﻱ ‪ . 2,6  10 8 sec‬ﻭﻟﻜﻥ ﻜﺘﻠﺔ ﺍﻟﻤﻴﺯﻭﻥ ﺍﻟﻤﻌﺘﺩل ﺘﺴﺎﻭﻱ ‪ 264‬ﻤﺭﺓ‬
‫ﻤﻥ ﻜﺘﻠﺔ ﺍﻻﻟﻜﺘﺭﻭﻥ ﺃﻤﺎ ﺍﻟﻤﻴﺯﻭﻨﺎﺕ ﺍﻟﺒﻁﻴﺌﺔ ‪  ‬ﻓﻜﺘﻠﺘﻬﺎ ﺘﺴﺎﻭﻱ ‪ 207‬ﻤﺭﺍﺕ ﻭﻟﻜﻥ‬
‫ﻋﻤﺭﻫﺎ ﺃﻁﻭل ﻓﻬﻭ ﻴﺴﺎﻭﻱ ‪. 2  10 6 sec‬‬
‫ﻟﻨﺒﺩﺃ ﺒﺩﺭﺍﺴﺔ ﺍﻨﻘﺴﺎﻡ ﺍﻟﻤﻴﺯﻭﻥ ‪  ‬ﺍﻟﻰ ﺍﻟﻤﻴﺯﻭﻥ ‪  ‬ﻭﺍﻟﻨﻴﺘﺭﻴﻨﻭ ﻁﺒﻘﺎﹰ‬
‫ﻟﻠﻤﻌﺎﺩﻟﺔ‪:‬‬
‫‪      ‬‬ ‫)‪(9.56‬‬
‫ﺤﻴﺙ ﺍﻟﻨﻴﺘﺭﻴﻨﻭ ‪ ‬ﻫﻭ ﺠﺴﻴﻡ ﻜﺘﻠﺘﻪ‪ ،‬ﻋﻠﻰ ﻤﺎﻴﺒﺩﻭ‪ ،‬ﻗﺭﻴﺒﺔ ﺠﺩﺍﹰ ﻤﻥ ﺍﻟﺼﻔﺭ‬
‫ﻜﺎﻟﻔﻭﺘﻭﻥ ﻏﻴﺭ ﺃﻨﻪ ﻻﻴﺅﻴﻥ ﻓﻲ ﻁﺭﻴﻘﺔ ﺃﻱ ﺫﺭﺍﺕ‪ ،‬ﻭﻴﻤﻜﻥ‪ ،‬ﺒﺩﺭﺍﺴﺔ ﺍﻻﻨﻘﺴﺎﻡ ﺍﻟﺴﺎﺒﻕ‬
‫ﺤﺴﺎﺏ ﻜﺘﻠﺔ ﺍﻟﻨﻴﺘﺭﻴﻨﻭ ﺍﺫﺍ ﻋﻠﻤﺕ ﻜﺘل ﺍﻟﻤﻴﺯﻭﻨﺎﺕ ﺍﻟﻤﺸﺎﺭﻜﺔ ﻓﻲ ﺍﻟﺘﻔﺎﻋل ﻓﺎﺫﺍ‬
‫ﺍﻋﺘﺒﺭﻨﺎ ﺃﻥ ﺍﻟﻤﺠﻤﻭﻉ ﺍﻟﻜﻠﻲ ﻟﻜﻤﻴﺔ ﺍﻟﺤﺭﻜﺔ ﻓﻲ ﻤﺠﻤﻭﻋﺔ ﻤﺭﻜﺯ ﺍﻟﻜﺘﻠﺔ ﺍﻻﺤﺩﺍﺜﻴﺔ‬
‫ﻴﺴﺎﻭﻱ ﺍﻟﺼﻔﺭ ﺃﻱ ﺃﻥ ‪ | P || P | P‬ﺜﻡ ﻜﺘﺒﻨﺎ ﻗﺎﻨﻭﻥ ﻤﺼﻭﻨﻴﺔ ﺍﻟﻁﺎﻗﺔ ﻓﻲ ﻫﺫﺍ‬
‫ﺍﻟﺘﻔﺎﻋل ﻓﺈﻨﻨﺎ ﻨﺠﺩ‪:‬‬
‫‪m  C 2  E   E   E kin  m  c 2  P2c 2  m 2 c 2 ‬‬
‫)‪(9.57‬‬
‫‪‬‬ ‫‪2‬‬
‫‪ E kin‬‬ ‫‪ m c ‬‬ ‫‪P2 c 2‬‬ ‫‪ m 2 c 4‬‬

‫ﻭﺒﺎﻻﺴﺘﻔﺎﺩﺓ ﻤﻥ ﺍﻟﻌﻼﻗﺔ ﺍﻟﺘﻲ ﺘﻌﺒﺭ ﻋﻥ ﺍﻤﻜﺎﻨﻴﺔ ﻜﺘﺎﺒﺔ ﺍﻟﻁﺎﻗﺔ ﺒﺸﻜﻠﻴﻥ‪:‬‬


‫‪E kin  mc 2  P 2 c 2  m 2 c 4‬‬
‫ﺤﻴﺙ ﻨﺠﺩ ﻤﻨﻬﺎ ﺒﺎﻟﺘﺭﺒﻴﻊ‪:‬‬
‫‪E 2kin  2mc 2 E kin  P 2c 2‬‬
‫ﺜﻡ ﺒﺘﺒﺩﻴل ‪ P 2c 2‬ﺒﻘﻴﻤﺘﻬﺎ ﻓﻲ )‪ (9.57‬ﻨﺤﺼل ﻋﻠﻰ ﺍﻟﻤﻌﺎﺩﻟﺔ‪:‬‬
‫‪‬‬ ‫‪ 2‬‬
‫‪m  c 2  E kin‬‬ ‫‪ m  c 2  ( E kin‬‬ ‫‪)  2m c 2 E kin  m 2 c 4‬‬

‫ﻭﻫﻲ ﻤﻌﺎﺩﻟﺔ ﺠﺒﺭﻴﺔ ﺒﺎﻟﻨﺴﺒﺔ ﺍﻟﻰ ‪ E kin‬ﻟﺤﻠﻬﺎ ﻨﺭﺒﻊ ﺍﻟﻁﺭﻓﻴﻥ ﻓﻨﺠﺩ‪:‬‬

‫‪- 302 -‬‬

‫)‪Create PDF files without this message by purchasing novaPDF printer (http://www.novapdf.com‬‬
‫‪(m  c 2  m  c 2 ) 2  ( E kin ) 2  2E kin ( m  c 2  m c 2 ) ‬‬
‫‪ ( E kin ) 2  2m  c 2 E kin  m 2 c 4‬‬

‫ﻭﻤﻨﻪ ﻨﺠﺩ ﺃﺨﻴﺭﺍﹰ ﻗﻴﻤﺔ ‪ E kin‬ﺍﻟﺘﺎﻟﻴﺔ‪:‬‬


‫‪(m  c 2  m  c 2 )2  m 2 c 4‬‬ ‫‪[(m   m  )2  m 2 ]c 2‬‬
‫‪E kin‬‬ ‫‪‬‬ ‫‪‬‬ ‫)‪(9.58‬‬
‫‪2m  c 2‬‬ ‫‪2m ‬‬

‫ﻭﺒﻘﻴﺎﺱ ﻫﺫﻩ ﺍﻟﻁﺎﻗﺔ ‪ E kin‬ﻴﻤﻜﻥ ﺤﺴﺎﺏ ‪ m ‬ﻭﻗﺩ ﻭﺠﺩ ﻨﺘﻴﺠﺔ ﻟﺫﻟﻙ ﺃﻥ‬
‫‪ E kin‬ﺘﺴﺎﻭﻱ ﺩﻭﻤﺎﹰ ﻗﻴﻤﺔ ﻤﻌﻴﻨﺔ ﺜﺎﺒﺘﺔ ﻭﺃﻥ ‪ m   0‬ﻤﻤﺎ ﻴﺅﻜﺩ ﺼﺤﺔ ﺍﻟﺩﺭﺍﺴﺔ‬
‫ﺍﻟﺤﺭﻜﻴﺔ ﺍﻟﺴﺎﺒﻘﺔ ﻟﻼﻨﻘﺴﺎﻡ ﺍﻟﻤﺫﻜﻭﺭ‪.‬‬
‫ﻟﻨﺩﺭﺱ ﺃﺨﻴﺭﺍﹰ ﺍﻨﻘﺴﺎﻡ ﺍﻟﻤﻴﺯﻭﻥ ‪  0‬ﺍﻟﻤﻌﺘﺩل ﺍﻟﻰ ﺠﺴ ﻴﻤﻲ ﻁﺎﻗﺔ )ﻓﻭﺘﻭﻨﻴﻥ(‬
‫ﺸﻜل )‪ (9.1‬ﻁﺒﻘﺎﹰ ﻟﻠﻤﻌﺎﺩﻟﺔ‪:‬‬
‫‪ 0    ‬‬ ‫)‪(9.59‬‬
‫ﺤﻴﺙ ﻴﻤﻜﻥ ﺍﻟﺘﺄﻜﺩ ﻤﻥ ﻭﺠﻭﺩ ﻜل ﻤﻥ ﺍﻟﻔﻭﺘﻭﻨﻴﻥ ﺒﻭﺍﺴﻁﺔ ﻤﺎ ﻴﺅﻴﻨﺎﻨﻪ ﻓﻲ‬
‫ﻁﺭﻴﻘﻬﻤﺎ ﻤﻥ ﺫﺭﺍﺕ‪ ،‬ﻭﻗﺩ ﻟﻭﺤﻅﺕ ﻋﻼﻗﺔ ﻤﺎ ﺒﻴﻥ ﻁﺎﻗﺔ ﺍﻟﻔﻭﺘﻭﻨﻴﻥ ﺍﻟﻨﺎﺘﺠﻴﻥ‬
‫ﻭﺍﻟﺯﻭﺍﻴﺔ ﺍﻟﺘﻲ ﻴﻨﻁﻠﻘﺎﻥ ﺒﻬﺎ‪ .‬ﻭﻴﻤﻜﻥ ﺍﻴﺠﺎﺩ ﻫﺫﻩ ﺍﻟﻌﻼﻗﺔ ﻨﻅﺭﻴﺎﹰ ﺍﻨﻁﻼﻗﺎﹰ ﻤﻥ ﻗﻭﺍﻨﻴﻥ‬
‫ﻤﺼﻭﻨﻴﺔ ﺍﻟﻁﺎﻗﺔ‪.‬‬

‫ﺸﻜل )‪(9.1‬‬
‫ﻭﺴﻨﺩﺭﺱ ﺫﻟﻙ ﻓﻲ ﻤﺎﻴﺴﻤﻰ ﻤﺠﻤﻭﻋﺔ ﺍﻻﺤﺩﺍﺜﻴﺎﺕ ﺍﻟﻤﺨﺒﺭﻴﺔ )ﻭﻫﻲ‬
‫ﺍﻟﻤﺠﻤﻭﻋﺔ ﺍﻟﺘﻲ ﺘﺘﺤﺭﻙ ﺒﺎﻟﻨﺴﺒﺔ ﺍﻟﻰ ﻤﺠﻤﻭﻋﺔ ﻤﺭﻜﺯ ﺍﻟﻜﺘﻠﺔ ‪ c‬ﺒﺴﺭﻋﺔ ﻤﻌﻴﻨﺔ )‪،v(c‬‬

‫‪- 303 -‬‬

‫)‪Create PDF files without this message by purchasing novaPDF printer (http://www.novapdf.com‬‬
‫ﻭﺘﺘﻡ ﻜل ﺍﻟﻘﻴﺎﺴﺎﺕ ﻓﻴﻬﺎ‪ ،‬ﻟﺫﺍ ﻨﺴﺘﻔﻴﺩ ﻤﻥ ﺜﺒﺎﺕ ﻤﺭﺒﻊ ﺸﻌﺎﻉ ﻜﻤﻴﺔ ﺍﻟﺤﺭﻜﺔ ﻓﻲ ﺍﻟﻔﺭﺍﻍ‬
‫ﺍﻟﺭﺒﺎﻋﻲ ﺍﻟﻤﻌﺒﺭ ﻋﻨﻪ ﺒﺎﻟﻤﻌﺎﺩﻟﺔ‪:‬‬
‫‪‬‬ ‫‪‬‬
‫‪I  m 2 c 4  ( P1  P2 ) 2 c 2  ( E1  E 2 )2‬‬ ‫)‪(9.60‬‬
‫ﺤﻴﺙ ‪ E 2 , E1, P2 , P1‬ﻫﻲ ﻜﻤﻴﺘﺎ ﺤﺭﻜﺘﻲ ﺍﻟﻔﻭﺘﻭﻨﻴﻥ ﻭﻁﺎﻗﺘﺎﻫﻤﺎ ﻋﻠﻰ‬
‫ﺍﻟﺘﺭﺘﻴﺏ‪ .‬ﻭﺒﻤﺎ ﺃﻥ ﻜﺘﻠﺔ ﺍﻟﻔﻭﺘﻭﻥ ﺘﺴﺎﻭﻱ ﺍﻟﺼﻔﺭ ﻓﺈﻥ ﻁﺎﻗﺘﻪ ﺘﺴﺎﻭﻱ ‪ ،pc‬ﻭﻋﻨﺩﺌﺫ‪‬‬
‫ﻨﺠﺩ ﺒﺎﻟﺘﺒﺩﻴل ﻓﻲ )‪ (9.60‬ﺍﻟﻤﻌﺎﺩﻟﺔ‪:‬‬

‫‪‬‬ ‫‪‬‬
‫‪m 2 c 4  ( p1  p 2 )2 c 2  (p1  p 2 )2 c 2 ‬‬
‫)‪(9.61‬‬
‫‪2‬‬ ‫‪‬‬
‫‪2‬‬ ‫‪2‬‬
‫‪ 2p1p 2 c (1  cos  )  4p1p 2 c sin‬‬
‫‪2‬‬
‫ﺤﻴﺙ ‪ ‬ﺍﻟﺯﺍﻭﻴﺔ ﺒﻴﻥ ﺍﺘﺠﺎﻫﻲ ﺍﻟﻔﻭﺘﻭﻨﻴﻥ ﺍﻟﻤﻨﻁﻠﻘﻴﻥ‪ .‬ﻭﻤﻨﻪ ﻨﺠﺩ ﺍﻟﻘﻴﻤﺔ ﺍﻟﺘﺎﻟﻴﺔ‬
‫ﻟﻠﺯﺍﻭﻴﺔ‪:‬‬
‫‪mc 2‬‬
‫)‪(9.62‬‬
‫‪‬‬
‫‪sin‬‬ ‫‪‬‬
‫‪2 2 E1E 2‬‬

‫ﻭﻫﻲ ﺘﺘﻭﺍﻓﻕ ﺘﻭﺍﻓﻘﺎﹰ ﺠﻴﺩﺍﹰ ﻤﻊ ﺍﻟﺘﺠﺭﺒﺔ‪ ،‬ﻤﻤﺎ ﻴﺅﻜﺩ ﻤﺭﺓ ﺃﺨﺭﻯ ﺼﺤﺔ ﺍﻟﻨﻅﺭﻴﺔ‬
‫ﺍﻟﻨﺴﺒﻴﺔ‪.‬‬
‫‪ -76‬ﺍﺻﻄﺪﺍﻡ ﺍﳉﺴﻴﻤﺎﺕ ﰲ ﺍﻟﻨﻈﺮﻳﺔ ﺍﻟﻨﺴﺒﻴﺔ ‪:‬‬
‫ﺴﻨﻬﺘﻡ ﻓﻲ ﻫﺫﻩ ﺍﻟﻔﻘﺭﺓ ﺒﺎﻻﺼﻁﺩﺍﻤﺎﺕ ﺍﻟﻤﺭﻨﺔ ﺒﻴﻥ ﺍﻟﺠﺴﻴﻤﺎﺕ ﻭﻫﻭ ﺍﻻﺼﻁﺩﺍﻡ‬
‫ﺍﻟﺫﻱ ﻻﻴﺤﺩﺙ ﺃﻱ ﺘﻔﺎﻋل ﻭﻻﻴﻐﻴﺭ ﺍﻟﺒﻨﻴﺔ ﺍﻟﺩﺍﺨﻠﻴﺔ ﻷﻱ ﻤﻥ ﺍﻟﺠﺴﻴﻤﺎﺕ ﺍﻟﻤﺼﻁﺩﻤﺔ‬
‫ﻭﻫﺫﺍ ﻤﺎ ﻴﺤﺩﺙ ﻜﺜﻴﺭﺍﹰ ﻓﻲ ﻋﺎﻟﻡ ﺍﻟﺠﺴﻴﻤﺎﺕ ﺍﻷﺴﺎﺴﻴﺔ‪.‬‬
‫‪‬‬
‫ﻟﻴﻜﻥ ﺍﻟﺠﺴﻴﻡ ﺍﻟﺴﺭﻴﻊ ﺫﻱ ﺍﻟﻜﺘﻠﺔ ‪ ‬ﻭﻜﻤﻴﺔ ﺍﻟﺤﺭﻜﺔ ‪ p‬ﻭﻟﻨﺩﺭﺱ ﺍﺼﻁﺩﺍﻤﻪ‬
‫ﺒﺠﺴﻴﻡ ﺁﺨﺭ ﻜﺘﻠﺘﻪ ‪ ،m‬ﻴﻤﻜﻥ ﺍﻋﺘﺒﺎﺭﻩ ﺴﺎﻜﻨﺎﹰ ﺍﺫﺍ ﻜﺎﻨﺕ ﺴﺭﻋﺔ ﺍﻟﺠﺴﻴﻡ ﺍﻟﺼﺎﺩﻡ ﺃﻜﺒﺭ‬
‫ﺒﻜﺜﻴﺭ ﻤﻥ ﺴﺭﻋﺘﻪ‪ .‬ﺃﻤﺎ ﺒﻌﺩ ﺍﻻﺼﻁﺩﺍﻡ ﻓﺈﻨﻪ ﺴﻴﻨﻁﻠﻕ ﺒﻜﻤﻴﺔ ﺤﺭﻜﺔ ‪ p1‬ﺘﺼﻨﻊ ﺯﺍﻭﻴﺔ‬
‫‪‬‬ ‫‪‬‬
‫ﺘﺼﻨﻊ ﺯﺍﻭﻴﺔ ‪ ‬ﻤﻊ ‪) p‬ﺸﻜل‬ ‫‪p2‬‬ ‫ﻤﺎ ‪ .‬ﻭﻴﻨﻁﻠﻕ ﺍﻟﺠﺴﻴﻡ ﺍﻟﺴﺭﻴﻊ ﺒﻜﻤﻴﺔ ﺤﺭﻜﺔ‬

‫‪- 304 -‬‬

‫)‪Create PDF files without this message by purchasing novaPDF printer (http://www.novapdf.com‬‬
‫‪‬‬
‫‪ ،(9.2‬ﻭﻟﻨﺤﺴﺏ ﻁﺎﻗﺔ ﺍﻟﺠﺴﻴﻡ ﺍﻟﺫﻱ ﺍﻨﻁﻠﻕ ﺒﻜﻤﻴﺔ ﺤﺭﻜﺔ ‪ p1‬ﺒﻌﺩ ﺍﻻﺼﻁﺩﺍﻡ )ﻭﻫﻭ‬
‫ﺍﻟﺫﻱ ﻜﺎﻥ ﺴﺎﻜﻨﺎﹰ ﻓﻲ ﺍﻟﺒﺩﺀ(‪.‬‬
‫ﻟﺫﺍ ﻨﻜﺘﺏ ﻗﺎﻨﻭﻨﻲ ﻤﺼﻭﻨﻴﺔ ﺍﻟﻁﺎﻗﺔ ﻭﻜﻤﻴﺔ ﺍﻟﺤﺭﻜﺔ ﺍﻟﺘﺎﻟﻴﻴﻥ‪:‬‬
‫‪‬‬ ‫‪‬‬ ‫‪‬‬
‫‪p  p1  p 2‬‬ ‫)‪(9.63‬‬
‫‪mc 2  E  E1  E 2‬‬ ‫)‪(9.64‬‬

‫ﺍﻟﺸﻜل )‪(9.2‬‬

‫ﻭﻫﻤﺎ ﺍﻟﻤﻌﺎﺩﻟﺘﺎﻥ ﺍﻷﺴﺎﺴﻴﺘﺎﻥ ﺍﻟﻼﺯﻤﺘﺎﻥ ﻟﺤﺴﺎﺏ ‪ E1‬ﻭ ‪ E 2‬ﻟﺤﻠﻬﻤﺎ ﻨﻀﻊ‬


‫)‪ (9.64‬ﺒﺎﻟﺸﻜل‪:‬‬
‫‪p 2 c 2   2 c 4  p 22 c 2   2 c 4  E (1‬‬‫)‬
‫‪kin‬‬ ‫)‪(9.65‬‬
‫ﺜﻡ ﻨﺭﺒﻊ )‪ (9.63‬ﺒﻌﺩ ﻨﻘل ‪ p1‬ﺍﻟﻰ ﺍﻟﻁﺭﻑ ﺍﻷﻴﺴﺭ ﻓﻨﺠﺩ‪:‬‬
‫‪p 22  p12  p 2  2p1p cos ‬‬ ‫)‪(9.66‬‬
‫ﻭﺒﺤﺫﻑ ‪ p 2‬ﻤﻥ )‪ (9.65‬ﻭ )‪ (9.66‬ﺜﻡ ﺍﻟﺘﻌﺒﻴﺭ ﻋﻥ ‪ p1‬ﺒﻘﻴﻤﺘﻬﺎ ﻤﻥ‬
‫ﺍﻟﻌﻼﻗﺔ‪:‬‬
‫‪p12 c 2  ( E(kin‬‬
‫‪1) 2‬‬
‫‪)  2mc 2 E (kin‬‬
‫)‪1‬‬
‫)‪(9.67‬‬
‫ﻨﺠﺩ ﺃﺨﻴﺭﺍﹰ‪:‬‬
‫‪pp1c 2 cos   E (kin‬‬
‫)‪1‬‬
‫‪( p 2 c 2   2 c 4  mc 2‬‬ ‫)‪(9.68‬‬

‫‪- 305 -‬‬

‫)‪Create PDF files without this message by purchasing novaPDF printer (http://www.novapdf.com‬‬
‫ﻭﺒﺘﺭﺒﻴﻊ ﺍﻟﻌﻼﻗﺔ ﺍﻟﺴﺎﺒﻘﺔ ﺘﻡ ﺍﻟﺘﻌﺒﻴﺭ ﻋﻥ ‪ p1‬ﺒﻘﻴﻤﺘﻬﺎ ﺃﻴﻀﺎﹰ ﻤﻥ )‪(9.67‬‬
‫ﻨﺤﺼل ﻋﻠﻰ ﺍﻟﻁﺎﻗﺔ ﺍﻟﺤﺭﻜﻴﺔ ﻟﻠﺠﺴﻴﻡ ﺍﻟﺫﻱ ﻜﺎﻥ ﺴﺎﻜﻨ ﺎﹰ ﻗﺒل ﺍﻻﺼﻁﺩﺍﻡ‪ ،‬ﻭﻫﻲ‬
‫ﺍﻟﺘﺎﻟﻴﺔ‪:‬‬
‫‪2 4‬‬ ‫‪2‬‬
‫‪E (kin‬‬
‫)‪1‬‬
‫‪‬‬
‫‪2mp c cos ‬‬
‫)‪(9.69‬‬
‫‪2 2‬‬ ‫‪2 4‬‬ ‫‪2 2‬‬ ‫‪2 2‬‬ ‫‪2‬‬
‫‪( p c   c  ( mc )  p c cos ‬‬

‫ﺃﻱ ﺃﻥ‪:‬‬ ‫‪0‬‬ ‫ﻭﻫﻲ ﺘﺒﻴﻥ ﺃﻥ ﺃﻜﺒﺭ ﻁﺎﻗﺔ ﻴﻜﺘﺴﺒﻬﺎ ﻫﺫﺍ ﺍﻟﺠﺴﻴﻡ ﺘﻨﺘﺞ ﻋﻨﺩﻤﺎ‬
‫‪p 2c 2‬‬
‫‪(E (kin‬‬
‫)‪1‬‬
‫‪)max  2mc 2‬‬ ‫)‪(9.70‬‬
‫‪2 2‬‬ ‫‪2 4‬‬ ‫‪2 2‬‬ ‫‪2 2‬‬
‫‪( p c   c  ( mc )  p c‬‬
‫ﻟﻨﺩﺭﺱ ﺒﻌﺽ ﺍﻟﺤﺎﻻﺕ ﺍﻟﺨﺎﺼﺔ‪.‬‬
‫‪ -1‬ﺍﺫﺍ ﻜﺎﻥ ﺍﻟﺠﺴﻴﻡ ﺍﻟﻭﺍﺭﺩ ﺒﺭﻭﺘﻭﻨﺎﹰ ﺃﻭ ﻨﺘﺭﻭﻨﺎﹰ ﻭﺍﻟﺠﺴﻴﻡ ﺍﻟﺴﺎﻜﻥ ﺍﻟﻜﺘﺭﻭﻨﺎﹰ‬
‫ﻓﻴﻤﻜﻥ ﺃﻥ ﻨﻀﻊ ‪ .   m‬ﻭﻜﺫﻟﻙ ﺴﻨﻌﺘﺒﺭ ﺃﻥ ﺍﻟﺠﺴﻴﻡ ﺍﻟﻭﺍﺭﺩ ﺴﺭﻴﻊ ﺠﺩﺍﹰ ﺒﺤﻴﺙ‬
‫ﺘﺘﺤﻘﻕ ﺍﻟﻌﻼﻗﺔ ‪ pc  c 2‬ﻭﻋﻨﺩﺌﺫ‪ ‬ﻨﺤﺼل ﻤﻥ )‪ (9.70‬ﻋﻠﻰ ﺍﻟﻁﺎﻗﺔ ﺍﻟﺘﺎﻟﻴﺔ‪:‬‬
‫‪p 2c 2‬‬
‫‪(E (kin‬‬
‫)‪1‬‬
‫‪)max  2mc 2‬‬ ‫)‪(9.71‬‬
‫‪2mc 2 pc   2 c 4‬‬
‫ﻓﺈﺫﺍ ﺘﺤﻘﻘﺕ‪ ،‬ﺒﺎﻻﻀﺎﻓﺔ ﺇﻟﻰ ﻤﺎﺴﺒﻕ‪ ،‬ﺍﻟﻌﻼﻗﺔ ﺍﻟﺘﺎﻟﻴﺔ‪:‬‬
‫‪2c4‬‬ ‫‪‬‬
‫‪pc ‬‬ ‫‪‬‬ ‫‪ c2‬‬
‫‪2‬‬ ‫‪m‬‬
‫‪mc‬‬
‫‪ E(1‬ﺍﻟﻤﻌﺒﺭ ﻋﻨﻬﺎ ﺒﺎﻟﻌﻼﻗﺔ )‪ (9.71‬ﺘﻭﻀﻊ ﺒﺎﻟﺸﻜل ﺍﻟﻨﻬﺎﺌﻲ‬
‫ﻓﺈﻥ ﺍﻟﻁﺎﻗﺔ ‪kin‬‬
‫)‬

‫ﺍﻟﺘﺎﻟﻲ‪:‬‬
‫‪(E (kin‬‬
‫)‪1‬‬
‫‪)max  p c  E‬‬ ‫)‪(9.72‬‬
‫ﻭﻫﻲ ﺘﺴﺎﻭﻱ ﻁﺎﻗﺔ ﺍﻟﺠﺴﻴﻡ ﺍﻟﻭﺍﺭﺩ‪.‬‬
‫ﺏ‪ -‬ﺍﺫﺍ ﻜﺎﻥ ﺍﻟﺠﺴﻴﻡ ﺍﻟﻭﺍﺭﺩ ﺨﻔﻴﻔﺎﹰ ﺠﺩﺍﹰ )ﺍﻟﻜﺘﺭﻭﻥ ﻤﺜﻼﹰ( ﻭﺍﻟﺠﺴﻴﻡ ﺍﻟﺴﺎﻜﻥ‬
‫ﺜﻘﻴل ﺠﺩﺍﹰ ﺃﻱ ‪   m‬ﻭﺘﺤﻘﻕ ﺃﻴﻀﺎﹰ‪ ،‬ﻜﻤﺎ ﺫﻜﺭﻨﺎ ﺴﺎﺒﻘﺎﹰ ‪ pc  c 2‬ﻓﻴﻤﻜﻥ ﻤﻥ‬
‫)‪ (9.70‬ﺍﻟﺤﺼﻭل ﻋﻠﻰ ﺍﻟﻘﻴﻤﺔ ﺍﻟﻌﻅﻤﻰ ﻟﻁﺎﻗﺔ ﺍﻟﺠﺴﻴﻡ ﺍﻟﺫﻱ ﻜﺎﻥ ﺴﺎﻜﻨﺎﹰ ﻗﺒل‬
‫ﺍﻻﺼﻁﺩﺍﻡ ﻋﻠﻰ ﺍﻟﻘﻴﻤﺔ ﺍﻟﻌﻅﻤﻰ ﻟﻁﺎﻗﺔ ﺍﻟﺠﺴﻴﻡ ﺍﻟﺫﻱ ﻜﺎﻥ ﺴﺎﻜﻨﺎﹰ ﻗﺒل ﺍﻻﺼﻁﺩﺍﻡ‬
‫ﻭﻫﻲ ﺍﻟﺘﺎﻟﻴﺔ‪:‬‬

‫‪- 306 -‬‬

‫)‪Create PDF files without this message by purchasing novaPDF printer (http://www.novapdf.com‬‬
‫‪p2c2‬‬
‫‪(E (kin‬‬
‫)‪1‬‬
‫‪)max‬‬ ‫‪ 2mc‬‬ ‫‪2‬‬
‫)‪(9.73‬‬
‫‪2mc 2 pc  m 2 c 4‬‬
‫ﻭﺍﺫﺍ ﺘﺤﻘﻕ ﺒﺎﻻﻀﺎﻓﺔ ﺍﻟﻰ ﺫﻟﻙ‪ ،‬ﺃﻥ ‪ pc  mc2‬ﻓﺈﻨﻨﺎ ﻨﺠﺩ ﻋﻨﺩﺌ ﺫ‪ ‬ﻤﻥ‬
‫)‪ (9.73‬ﺍﻟﻁﺎﻗﺔ ﺍﻟﺘﺎﻟﻴﺔ‪:‬‬
‫‪E(kin‬‬
‫)‪1‬‬
‫‪ pc  E‬‬ ‫)‪(9.74‬‬
‫ﺃﻱ ﺃﻥ ﺍﻟﻁﺎﻗﺔ ﺍﻟﻌﻅﻤﻰ ﻟﻠﺠﺴﻴﻡ‪ ،‬ﺍﻟﺫﻱ ﻜﺎﻥ ﺴﺎﻜﻨﺎﹰ‪ ،‬ﺘﺼﺒﺢ ﻤﺴﺎﻭﻴﺔ ﻟﻁﺎﻗﺔ‬
‫ﺍﻟﺠﺴﻴﻡ ﺍﻟﻭﺍﺭﺩ ﻓﻬﻭ ﻴﺄﺨﺫ ﻜل ﺍﻟﻁﺎﻗﺔ‪.‬‬
‫‪ -77‬ﻇﺎﻫﺮﺓ ﻛﻮﻣﺒﺘﻮﻥ‪Compton Effect :‬‬
‫ﺍﻥ ﺃﺤﺩ ﺍﻟﺘﻁﺒﻴﻘﺎﺕ ﺍﻟﻬﺎﻤﺔ ﻟﺩﺭﺍﺴﺔ ﺍﻻﺼﻁﺩﺍﻡ ﻓﻲ ﺍﻟﻤﻴﻜﺎﻨﻴﻙ ﺍﻟﻨﺴﺒﻲ ﻫﻭ‬
‫ﻤﺎﻴﺴﻤﻰ ﻅﺎﻫﺭﺓ ﻜﻭﻤﺒﺘﻭﻥ ﻭﻫﻲ ﺘﺤﺩﺙ ﻋﻨﺩﻤﺎ ﻴﻜﻭﻥ ﺍﻟﺠﺴﻴﻡ ﺍﻟﻭﺍﺭﺩ ﻓﻭﺘﻭﻨﺎﹰ ﻁﺎﻗﺘﻪ‬
‫‪ E1  E(kin‬ﻭﻴﻜﻭﻥ ﻟﻠﻔﻭﺘﻭﻥ‬
‫)‪1‬‬
‫‪ E  pc‬ﻭﺍﻟﺠﺴﻴﻡ ﺍﻟﺴﺎﻜﻥ ﺍﻟﻜﺘﺭﻭﻨﺎﹰ ﻁﺎﻗﺘﻪ ‪ mc 2‬‬
‫ﺍﻟﻨﺎﺘﺞ ﻁﺎﻗﺔ ‪ E 2  p 2 c‬ﺸﻜل )‪.(9.3‬‬

‫ﺍﻟﺸﻜل )‪(9.3‬‬

‫‪- 307 -‬‬

‫)‪Create PDF files without this message by purchasing novaPDF printer (http://www.novapdf.com‬‬
‫ﻟﻨﺩﺭﺱ ﻫﺫﻩ ﺍﻟﻅﺎﻫﺭﺓ ﺒﺎﻟﺘﻔﺼﻴل‪ ،‬ﻟﺫﺍ ﻨﻜﺘﺏ ﻤﻌﺎﺩﻟﺘﻲ ﻤﺼﻭﻨﻴﺔ ﺍﻟﻁﺎﻗﺔ ﻭﻜﻤﻴﺔ‬
‫ﺍﻟﺤﺭﻜﺔ ﺍﻟﺘﺎﻟﻴﺘﻴﻥ‪:‬‬
‫‪E  E 2  E(1‬‬‫)‬
‫‪kin‬‬ ‫)‪(9.75‬‬
‫‪‬‬ ‫‪‬‬ ‫‪‬‬
‫‪p  p1  p 2‬‬ ‫)‪(9.76‬‬
‫ﻓﻤﻥ ﺍﻟﺜﺎﻨﻴﺔ ﻨﺠﺩ‪:‬‬
‫‪p12  p 2  p 22  2pp 2 cos ‬‬ ‫) ‪(9.77,a‬‬
‫ﺤﻴﺙ ‪ ‬ﻫﻲ ﺍﻟﺯﺍﻭﻴﺔ ﺒﻴﻥ ﺍﺘﺠﺎﻩ ﺍﻟﻔﻭﺘﻭﻥ ﺍﻟﻭﺍﺭﺩ ﻭﺍﻟﻔﻭﺘﻭﻥ ﺍﻟﻤﺘﺸﺘﺕ ﺫﻱ‬
‫‪E‬‬
‫‪ p ‬ﻓﻴﻤﻜﻥ ﻭﻀﻊ ﺍﻟﻌﻼﻗﺔ ﺍﻟﺴﺎﺒﻘﺔ‬ ‫ﺍﻟﻁﺎﻗﺔ ‪ . E 2‬ﻓﺈﺫﺍ ﻋﻠﻤﻨﺎ ﺃﻥ‬
‫‪E‬‬
‫‪, p2  2‬‬
‫‪c‬‬ ‫‪c‬‬
‫ﺒﺎﻟﺸﻜل‪:‬‬
‫‪E‬‬ ‫‪2E E 2 cos ‬‬
‫)‪(9.77b‬‬
‫‪E‬‬
‫‪p12  ( ) 2  ( 2 ) 2 ‬‬
‫‪c‬‬ ‫‪c‬‬ ‫‪c2‬‬
‫ﻭﺒﺎﻻﺴﺘﻔﺎﺩﺓ ﻤﻥ )‪ (9.67‬ﻨﺤﻭل )‪ (9.77b‬ﺍﻟﻰ ﺍﻟﺸﻜل‪:‬‬
‫‪(E (kin‬‬
‫‪1) 2‬‬
‫‪)  2m e c 2 E(kin‬‬
‫)‪1‬‬
‫‪ E 2  E 22  2EE 2 cos ‬‬ ‫)‪(9.78‬‬
‫‪ E(1‬ﻤﻥ‬
‫ﻭﻴﻤﻜﻥ ﺍﻵﻥ ﺍﻟﺤﺼﻭل ﻋﻠﻰ ﻁﺎﻗﺔ ﺍﻟﻔﻭﺘﻭﻥ ﺍﻟﻤﺘﺸﺘﺕ ‪ E 2‬ﺒﺤﺫﻑ ‪kin‬‬
‫)‬

‫ﺍﻟﻤﻌﺎﺩﻟﺘﻴﻥ )‪ (9.75‬ﻭ )‪ (9.78‬ﺤﻴﺙ ﻴﻌﺒﺭ ﻋﻥ ﻫﺫﻩ ﺍﻟﻁﺎﻗﺔ ﺒﺩﻻﻟﺔ ﻁﺎﻗﺔ ﺍﻟﻔﻭﺘﻭﻥ‬


‫ﺍﻟﻭﺍﺭﺩ ‪ E‬ﻭﺯﺍﻭﻴﺔ ﺍﻟﺘﺸﺘﺕ ‪ ‬ﺒﺎﻟﻌﻼﻗﺔ‪:‬‬
‫‪m ec2 E‬‬
‫‪E2 ‬‬ ‫)‪(9.79‬‬
‫) ‪m e c 2  E(1  cos ‬‬
‫ﻟﻨﺤﺴﺏ ﺍﻟﺘﻐﻴﺭ ﻓﻲ ﻁﻭل ﻤﻭﺠﺔ ﺍﻟﻔﻭﺘﻭﻥ ﺍﻟﻭﺍﺭﺩ‪ ،‬ﺍﻟﻨﺎﺘﺞ ﻋﻥ ﺍﻻﺼﻁﺩﺍﻡ ﻟﺫﺍ‬
‫ﻨﻼﺤﻅ ﺃﻥ‪:‬‬
‫‪hc‬‬ ‫‪hc‬‬
‫‪E  h ‬‬ ‫‪ ‬‬
‫‪‬‬ ‫‪E‬‬
‫ﻭﻴﻜﻭﻥ‪:‬‬
‫)‪(1  cos )  (1  cos  ) (9.80‬‬
‫‪1‬‬ ‫‪1‬‬ ‫‪hc‬‬
‫(‪   2  1  hc‬‬ ‫‪ )‬‬
‫‪E 2 E1‬‬ ‫‪mec‬‬
‫ﺤﻴﺙ ﺭﻤﺯﻨﺎ ﺒـ ‪ ‬ﻟﻠﻤﻘﺩﺍﺭ‪:‬‬

‫‪- 308 -‬‬

‫)‪Create PDF files without this message by purchasing novaPDF printer (http://www.novapdf.com‬‬
‫‪h‬‬
‫‪‬‬ ‫‪ 0,0242 A ‬‬
‫‪m ec‬‬
‫ﻭﻫﻭ ﻤﺎﻴﺴﻤﻰ ﻁﻭل ﻤﻭﺠﺔ ﻜﻭﻤﺒﺘﻭﻥ‪.‬‬
‫ﻭﻤﻥ ﺍﻟﻁﺭﻴﻑ ﺃﻥ ﻨﻼﺤﻅ ﺃﻥ ﺍﻟﺘﻐﻴﺭ ﻓﻲ ﺍﻟﻁﻭل ﺍﻟﻤﻭﺠﻲ ﻏﻴﺭ ﺘﺎﺒﻊ ﻟﻁﺎﻗﺔ‬
‫ﺍﻟﻔﻭﺘﻭﻥ ﺍﻟﻭﺍﺭﺩ ﻭﺃﻥ ‪.  max  2‬‬
‫ﻭﻟﺤﺴﺎﺏ ﻁﺎﻗﺔ ﺍﻻﻟﻜﺘﺭﻭﻥ ﺍﻟﺤﺭﻜﻴﺔ ﺍﻟﻨﺎﺘﺠﺔ ﺒﻌﺩ ﺍﻻﺼﻁﺩﺍﻡ ﻨﺴﺘﺨﺩﻡ ﺍﻟﻌﻼﻗﺘﻴﻥ‬
‫)‪ (9.75‬ﻭ )‪ (9.79‬ﻓﻨﺠﺩ‪:‬‬
‫)‪E(1  cos ‬‬
‫‪E(kin‬‬
‫)‪1‬‬
‫‪ E  E2 ‬‬ ‫‪E‬‬ ‫)‪(9.81‬‬
‫) ‪m c c 2 (1  cos ‬‬

‫ﺜﻡ ﻗﺴﻤﻨﺎ ﺤﺩﻱ ﺍﻟﻜﺴﺭ ﻋﻠﻰ‬ ‫ﻓﺈﺫﺍ ﺒﺩﻟﻨﺎ ‪ E‬ﻓﻲ ﺍﻟﺒﺴﻁ ﻭﺍﻟﻤﻘﺎﻡ ﺒﻘﻴﻤﺘﻬﺎ‬
‫‪hc‬‬
‫‪‬‬
‫‪ m e c 2‬ﻓﺈﻨﻨﺎ ﻨﺤﺼل ﻋﻠﻰ ﺍﻟﻁﺎﻗﺔ ﺍﻟﺤﺭﻜﻴﺔ ﺍﻟﺘﺎﻟﻴﺔ‪:‬‬
‫)‪(9.82‬‬
‫) ‪(1  cos ‬‬
‫‪E (kin‬‬
‫)‪1‬‬
‫‪‬‬ ‫‪E‬‬
‫) ‪  (1  cos ‬‬
‫ﻭﻫﻲ ﺼﻐﻴﺭﺓ ﺠﺩﺍﹰ ﻋﻨﺩﻤﺎ ‪   ‬ﺃﻤﺎ ﻋﻨﺩﻤﺎ ‪  ~ ‬ﻓﺈﻥ ﻫﺫﻩ ﺍﻟﻁﺎﻗﺔ‬
‫ﺘﺼﺒﺢ ﻤﻥ ﺭﺘﺒﺔ ﻁﺎﻗﺔ ﺍﻟﻔﻭﺘﻭﻥ ﺍﻟﻭﺍﺭﺩ‪.‬‬
‫ﺘﻔﻴﺩ ﺍﻟﻌﻼﻗﺔ ﺍﻟﺴﺎﺒﻘﺔ )‪ (9.82‬ﻓﻲ ﺩﺭﺍﺴﺔ ﺘﺸﺘﺕ ﺍﻟﻀﻭﺀ ﻋﻠﻰ ﻤﺨﺘﻠﻑ ﺍﻟﻤﻭﺍﺩ‪،‬‬
‫ﻓﺈﺫﺍ ﻁﺒﻘﺕ ﻋﻠﻰ ﺍﻟﻀﻭﺀ ﺍﻟﻤﺭﺌﻲ ﺍﻟﺫﻱ ﻁﻭل ﻤﻭﺠﺘﻪ ‪ ‬ﻜﺒﻴﺭ ﺠﺩﺍﹰ ﺒﺎﻟﻨﺴﺒﺔ ﺍﻟﻰ ‪A‬‬
‫)ﺃﻱ ) ‪ ( (7000    4000    0,042 A ‬ﻨﺠﺩ ﺃﻥ ﺍﻟﻁﺎﻗﺔ ﺍﻟﺘﻲ ﺘﻜﺘﺴﺒﻬﺎ‬
‫ﺍﻟﻜﺘﺭﻭﻨﺎﺕ ﺍﻟﻤﺎﺩﺓ ﺼﻐﻴﺭﺓ ﺠﺩ ﺍﹰ ﻁﺒﻘﺎﹰ ﻟـ )‪ (9.82‬ﻭﺴﻴﻨﻁﻠﻕ ﺍﻟﻔﻭﺘﻭﻥ ﺒﺎﻟﻁﺎﻗﺔ ﻨﻔﺴﻬﺎ‬
‫ﺘﻘﺭﻴﺒﺎﹰ ﻤﻤﺎ ﻻﻴﺴﺒﺏ ﺃﻱ ﺘﻐﻴﺭ ﻓﻲ ﻨﻭﻉ ﺍﻟﻀﻭﺀ ﺍﻟﻭﺍﺭﺩ‪ ،‬ﻭﻴﻤﻜﻥ ﻓﻲ ﻫﺫﻩ ﺍﻟﺤﺎﻟﺔ‬
‫ﺍﺴﺘﻨﺘﺎﺝ ﻋﻼﻗﺔ ﺭﻴﺎﻀﻴﺔ ﻟﺩﺭﺍﺴﺔ ﺘﺸﺘﺕ ﺍﻟﻀﻭﺀ ﺍﻨﻁﻼﻗﺎﹰ ﻤﻥ ﻤﻔﺎﻫﻴﻡ ﻜﻼﺴﻴﻜﻴﺔ‪.‬‬
‫ﺃﻤﺎ ﺇﺫﺍ ﻁﺒﻘﺕ ﺍﻟﻌﻼﻗﺔ )‪ (9.82‬ﻋﻠﻰ ﺍﻟﻔﻭﺘﻭﻨﺎﺕ ﺫﺍﺕ ﺍﻷﻁﻭﺍل ﺍﻟﻤﻭﺠﻴﺔ‬
‫ﺍﻟﺼﻐﻴﺭﺓ ﻜﺄﺸﻌﺔ ﺭﻭﻨﺘﺠﻥ ﻭﺃﺸﻌﺔ ‪ ‬ﻓﺈﻥ ﺍﻟﺘﻐﻴﺭ ﻓﻲ ﺍﻟﻁﻭل ﺍﻟﻤﻭﺠﻲ ﻴﺼﺒﺢ ﻤﻥ‬
‫ﺭﺘﺒﺔ ﻁﻭل ﺍﻟﻤﻭﺠﺔ ﺍﻟﻭﺍﺭﺩﺓ ﻜﻤﺎ ﺃﻥ ﺍﻟﻁﺎﻗﺔ ﺍﻟﺘﻲ ﻴﻜﺘﺴﺒﻬﺎ ﺍﻻﻟﻜﺘﺭﻭﻥ ﺘﺼﺒﺢ ﻜﺒﻴﺭﺓ‬
‫ﻤﻤﺎ ﻴﺴﺒﺏ ﺘﻐﻴﺭﺍﹰ ﻓﻲ ﺍﻷﺸﻌﺔ ﺍﻟﻭﺍﺭﺩﺓ ﻭﻻﺘﺴﺘﻁﻴﻊ ﺍﻟﻌﻼﻗﺎﺕ ﺍﻟﻜﻼﺴﻴﻜﻴﺔ ﻭﺼﻑ‬
‫ﺘﺸﺘﺕ ﻫﺫﻩ ﺍﻷﺸﻌﺔ ﻋﻠﻰ ﺍﻟﺒﻠﻭﺭﺍﺕ ﺍﻟﻤﻌﺩﻨﻴﺔ ﻭﻻﺒﺩ ﻟﺫﻟﻙ ﻤﻥ ﺘﻁﺒﻴﻕ ﻨﻅﺭﻴﺔ ﺍﻟﻜﻡ‪.‬‬

‫‪- 309 -‬‬

‫)‪Create PDF files without this message by purchasing novaPDF printer (http://www.novapdf.com‬‬
‫ﻭﻫﻜﺫﺍ ﻨﺭﻯ ﺃﻥ ﺍﻟﻨﻅﺭﻴﺔ ﺍﻟﻨﺴﺒﻴﺔ ﺘﺒﻴﻥ ﻟﻨﺎ ﺤﺩﻭﺩ ﺘﻁﺒﻴﻕ ﺍﻟﻤﻌﺎﺩﻻﺕ ﺍﻟﻜﻼﺴﻴﻜﻴﺔ‬
‫ﻟﺩﺭﺍﺴﺔ ﺒﻌﺽ ﺍﻟﻅﻭﺍﻫﺭ ﺍﻟﻔﻴﺯﻴﺎﺌﻴ ﺔ‪.‬‬
‫‪ -78‬ﻣﻔﺎﻫﻴﻢ ﺃﻭﻟﻴﺔ ﻋﻦ ﺍﻟﻨﻈﺮﻳﺔ ﺍﻟﻨﺴﺒﻴﺔ ﺍﻟﻌﺎﻣﺔ ‪:‬‬
‫ﻟﻘﺩ ﺤﺎﻭﻟﺕ ﺍﻟﻨﻅﺭﻴﺔ ﺍﻟﻨﺴﺒﻴﺔ ﺍﻟﺘﻲ ﺩﺭﺴﻨﺎﻫﺎ ﻓﻲ ﺍﻟﻔﺼﻠﻴﻥ ﺍﻟﺴﺎﺒﻘﻴﻥ ﺃﻥ ﺘﻭﺤﺩ‬
‫ﻓﻴﺯﻴﺎﺀ ﺍﻟﺠﻤل ﺍﻟﻌﻁﺎﻟﻴﺔ‪ ،‬ﻓﻌﻤﻠﺕ ﻋﻠﻰ ﺼﻴﺎﻏﺔ ﻜﺎﻓﺔ ﺍﻟﻘﻭﺍﻨﻴﻥ ﺍﻟﻔﻴﺯﻴﺎﺌﻴﺔ ﻓﻲ ﻗﺎﻟﺏ‬
‫ﻭﺍﺤﺩ )ﺍﻟﻔﺭﻀﻴﺔ ﺍﻷﻭﻟﻰ(‪ ،‬ﻭﻟﻜﻥ ﻤﺎﺫﺍ ﻋﻥ ﺍﻟﺠﻤل ﻏﻴﺭ ﺍﻟﻌﻁﺎﻟﻴﺔ؟ )ﻭﻫﻲ ﺘﻠﻙ ﺍﻟﺠﻤل‬
‫ﺍﻻﺤﺩﺍﺜﻴﺔ ﺍﻟﺘﻲ ﺘﺘﺤﺭﻙ ﺒﺎﻟﻨﺴﺒﺔ ﻟﺒﻌﻀﻬﺎ ﺒﺤﺭﻜﺔ ﻏﻴﺭ ﻤﻨﺘﻅﻤﺔ( ﻭﻫل ﻟﻬﺫﻩ ﺍﻟﺠﻤل‬
‫ﺫﺍﺘﻴﺔ ﺨﺎﺼﺔ ﺒﺤﻴﺙ ﻨﺤﺘﺎﺝ ﺍﻟﻰ ﻓﻴﺯﻴﺎﺀ ﻤﺴﺘﻘﻠﺔ ﻟﺩﺭﺍﺴﺔ ﺍﻟﻅﻭﺍﻫﺭ ﺍﻟﻔﻴﺯﻴﺎﺌﻴﺔ ﺍﻟﺘﻲ‬
‫ﺘﺠﺭﻯ ﻀﻤﻥ ﻜل ﻤﻨﻬﺎ؟ ﻟﻘﺩ ﻓﻜﺭ ﺃﻨﻴﺸﺘﺎﻴﻥ ﻁﻭﻴﻼﹰ ﺒﻬﺫﻩ ﺍﻟﺠﻤل ﻓﻲ ﺍﻟﻔﺘﺭﺓ ﺍﻟﺘﻲ ﺘﻠﺕ‬
‫ﻅﻬﻭﺭ ﺍﻟﻨﻅﺭﻴﺔ ﺍﻟﻨﺴﺒﻴﺔ ﺍﻟﻌﺎﻤﺔ ﺍﻟﺘﻲ ﻟﻥ ﻨﺴﺘﻁﻴﻊ ﺩﺭﺍﺴﺘﻬﺎ ﻫﻨﺎ ﺒﺎﻟﺘﻔﺼﻴل ﻷﻥ ﻫﺫﺍ‬
‫ﻴﺤﺘﺎﺝ ﺇﻟﻰ ﻜﺜﻴﺭ ﻤﻥ ﺍﻟﺭﻴﺎﻀﻴﺎﺕ ﺍﻟﻌﺎﻟﻴﺔ ﻭﺴﻨﻜﺘﻔﻲ ﺒﺎﻋﻁﺎﺀ ﻟﻤﺤﺔ ﺴﺭﻴﻌﺔ ﻋﻨﻬﺎ‪.‬‬
‫ﻟﻴﺴﺕ ﺍﻟﻨﺴﺒﻴﺔ ﺍﻟﻌﺎﻤﺔ‪ ،‬ﻜﻤﺎ ﻴﺩل ﺍﺴﻤﻬﺎ‪ ،‬ﺍﻻ ﺘﻌﻤﻴﻤﺎﹰ ﻤﺒﺎﺸﺭﺍﹰ ﻟﻠﻨﻅﺭﻴﺔ ﺍﻟﺨﺎﺼﺔ‬
‫ﺍﻟﺘﻲ ﻅﻬﺭﺕ ﺴﻨﺔ ‪ 1905‬ﻭﺍﻟﺘﻲ ﺘﻬﺘﻡ ﺒﺎﻟﺠﻤل ﺍﻟﻌﻁﺎﻟﻴﺔ ﺒﻴﻨﻤﺎ ﺘﻬﺘﻡ ﺍﻟﻌﺎﻤﺔ ﺒﺎﻟﺠﻤل‬
‫ﺍﻟﻼﻋﻁﺎﻟﻴﺔ‪ ،‬ﻭﻟﻬﺎ ﻓﺭﻀﻴﺘﺎﻥ ﺍﺜﻨﺘﺎﻥ‪:‬‬
‫ﺍﻟﻔﺭﻀﻴﺔ ﺍﻷﻭﻟﻰ‪ :‬ﻴﺠﺏ ﺼﻴﺎﻏﺔ ﻜﺎﻓﺔ ﺍﻟﻘﻭﺍﻨﻴﻥ ﺍﻟﻔﻴﺯﻴﺎﺌﻴﺔ ﺒﺸﻜل ﻭﺍﺤﺩ ﻓﻲ‬
‫ﻜﺎﻓﺔ ﺍﻟﺠﻤل ﺍﻟﻤﺘﺤﺭﻜﺔ ﻭﺒﺎﻟﺘﺎﻟﻲ ﻓﺈﻥ ﺼﻴﻐﺔ ﺃﻱ ﻗﺎﻨﻭﻥ ﻓﻴﺯﻴﺎﺌﻲ ﻴﺼﻑ ﻅﺎﻫﺭﺓ‬
‫ﻓﻴﺯﻴﺎﺌﻴﺔ ﻤﺎ‪ ،‬ﻴﺠﺏ ﺃﻥ ﺘﻜﻭﻥ ﻤﺴﺘﻘﻠﺔ ﻋﻥ ﺍﻟﺠﻤﻠﺔ ﺍﻻﺤﺩﺍﺜﻴﺔ ﺍﻟﺘﻲ ﺘﺤﺩﺙ ﻓﻴﻬﺎ ﻫﺫﻩ‬
‫ﺍﻟﻅﺎﻫﺭﺓ‪.‬‬
‫ﺍﻟﻔﺭﻀﻴﺔ ﺍﻟﺜﺎﻨﻴﺔ‪ :‬ﻻﻴﻤﻜﻥ ﻟﻤﺭﺍﻗﺏ ﻤﻭﺠﻭﺩ ﻓﻲ ﻤﺨﺘﺒﺭ ﻤﻐﻠﻕ ﺃﻥ ﻴﻤﻴﺯ ﺒﻴﻥ‬
‫ﺍﻟﻅﻭﺍﻫﺭ ﺍﻟﻤﺘﻌﻠﻘﺔ ﺒﺤﻘل ﺠﺎﺫﺒﻴﺔ ﻭﺘﻠﻙ ﺍﻟﻤﺘﻌﻠﻘﺔ ﺒﺘﺴﺎﺭﻉ ﺍﻟﻤﺨﺘﺒﺭ‪ ،‬ﻭﻫﺫﻩ ﺍﻟﻔﺭﻀﻴﺔ‬
‫ﺘﺠﺴﻴﺩ ﻟﻤﺎ ﻴﺴﻤﻰ ﻤﺒﺩﺃ ﺍﻟﺘﻌﺎﺩل ﺒﻴﻥ ﺍﻟﻜﺘﻠﺔ ﺍﻟﺜﻘﻴﻠﺔ ﻭﺍﻟﻜﺘﻠﺔ ﺍﻟﻌﺎﻁﻠﺔ ﺍﻟﺫﻱ ﻴﺒﺭﻫﻥ‬
‫ﻨﻅﺭﻴﺎﹰ ﻜﻤﺎ ﻴﻠﻲ‪:‬‬
‫ﺘﺘﻌﻴﻥ ﺍﻟﻜﺘﻠﺔ ﺍﻟﻌﺎﻁﻠﺔ ‪ mi‬ﻟﺠﺴﻴﻡ ﻤﺎ ﺒﻘﻴﺎﺱ ﺍﻟﺘﺴﺎﺭﻉ ﺍﻟﺫﻱ ﻴﻜﺘﺴﺒﻪ ﻫﺫﺍ ﺍﻟﺠﺴﻡ‬
‫ﻋﻨﺩﻤﺎ ﻨﻁﺒﻕ ﻋﻠﻴﻪ ﻗﻭﺓ ﻤﻌﻴﻨﺔ ‪ F‬ﻁﺒﻘﺎﹰ ﻟﻠﻘﺎﻨﻭﻥ ﺍﻟﺘﺎﻟﻲ‪:‬‬
‫‪‬‬ ‫‪‬‬
‫‪F  mi W‬‬ ‫)‪(9.83‬‬

‫‪- 310 -‬‬

‫)‪Create PDF files without this message by purchasing novaPDF printer (http://www.novapdf.com‬‬
‫ﻭﺘﺘﻌﻴﻥ ﺍﻟﻜﺘﻠﺔ ﺍﻟﺜﻘﻴﻠﺔ ‪ m g‬ﻟﻠﺠﺴﻴﻡ ﺒﻌﺩ ﻤﻌﺭﻓﺔ ﻗﻭﺓ ﺍﻟﺠﺫﺏ ﺍﻟﺘﻲ ﻴﺘﺄﺜﺭ ﺒﻬﺎ‬
‫ﻋﻨﺩ ﻭﻀﻌﻪ ﺒﺎﻟﻘﺭﺏ ﻤﻥ ﺠﺴﻡ ﺁﺨﺭ ﻜﺘﻠﺘﻪ ‪ M‬ﺤﺴﺏ ﻗﺎﻨﻭﻥ ﺍﻟﺠﺎﺫﺒﻴﺔ ﺍﻟﻌﺎﻟﻤﻴﺔ ﺍﻟﺫﻱ‬
‫ﺭﺃﻴﻨﺎﻫﺎ ﺴﺎﺒﻘﺎﹰ ﻭﻫﻭ‪:‬‬
‫‪Mm g‬‬
‫‪Fg  f‬‬ ‫)‪(9.84‬‬
‫‪r2‬‬
‫ﻓﺎﺫﺍ ﺍﻋﺘﺒﺭﻨﺎ ﺠﺴﻴﻤﺎﹰ ﻴﺴﻘﻁ ﺴﻘﻭﻁﺎﹰ ﺤﺭﺍﹰ ﺒﺠﻭﺍﺭ ﺍﻷﺭﺽ ﺍﻟﻤﺒﺎﺸﺭ ﻭﻁﺒﻘﻨﺎ‬
‫ﺍﻟﻘﺎﻨﻭﻥ ﺍﻟﺴﺎﺒﻕ ﻋﻠﻴﻪ ﻓﺈﻨﻪ ﻴﺠﺏ ﺘﺒﺩﻴل ‪ M‬ﺒﻜﺘﻠﺔ ﺍﻷﺭﺽ ﻭ ‪ r‬ﺒﻨﺼﻑ ﻗﻁﺭﻫﺎ‬
‫ﻭﺴﻴﻜﺘﺴﺏ ﻫﺫﺍ ﺍﻟﺠﺴﻡ ﺘﺴﺎﺭﻋﺎﹰ ‪ g‬ﻴﺤﺴﺏ ﻤﻥ ﺍﻟﻌﻼﻗﺔ )‪ (9.83‬ﻜﻤﺎﻴﻠﻲ‪:‬‬
‫‪mg‬‬
‫)‪(9.85‬‬
‫‪F‬‬ ‫‪M‬‬
‫‪g‬‬ ‫‪ f earth ‬‬
‫‪mi‬‬ ‫‪2‬‬ ‫‪mi‬‬
‫‪rearth‬‬
‫ﻭﺒﻤﺎ ﺃﻥ ﺍﻟﺘﺴﺎﺭﻉ ﺍﻷﺭﻀﻲ ‪ g‬ﺜﺎﺒﺕ ﻋﻠﻰ ﺨﻁ ﻋﺭﺽ ﻤﻌﻴﻥ‪ ،‬ﻜﻤﺎ ﺘﺩل‬
‫‪mg‬‬
‫ﻴﺠﺏ ﺃﻥ ﻴﺴﺎﻭﻱ ﻤﻘﺩﺍﺭﺍﹰ ﺜﺎﺒﺘﺎﹰ‪ ،‬ﻭﺒﺎﻟﻔﻌل ﻓﻘﺩ ﺃﺜﺒﺘﺕ‬ ‫ﺍﻟﺘﺠﺎﺭﺏ‪ ،‬ﻓﺈﻥ ﺍﻟﻤﻘﺩﺍﺭ‬
‫‪mi‬‬
‫ﺍﻟﺘﺠﺎﺭﺏ ﺍﻟﺤﺩﻴﺜﺔ ﺃﻥ ﺍﻟﻨﺴﺒﺔ ﺍﻟﺴﺎﺒﻘﺔ ﺘﺴﺎﻭﻱ ﺍﻟﻭﺍﺤﺩ ﺒﺨﻁﺄ ﻨﺴﺒﻲ ﻻﻴﺘﺠﺎﻭﺯ ‪.10 10‬‬
‫ﻟﻘﺩ ﺭﺃﻴﻨﺎ ﻓﻲ ﺍﻟﻔﻘﺭﺍﺕ ﺍﻟﺴﺎﺒﻘﺔ ﺃﻥ ﺍﻟﻤﺎﺩﺓ ﻭﺍﻟﻁﺎﻗﺔ ﻭﺠﻬﺎﻥ ﻤﺨﺘﻠﻔﺎﻥ ﻟﺸﻲﺀ‬
‫ﻭﺍﺤﺩ ﻓﻠﻴﺴﺕ ﺍﻟﻜﺘﻠﺔ ﺴﻭﻯ ﻋﺎﻤل ﻴﺸﻴﺭ ﺇﻟﻰ ﻤﻘﺩﺍﺭ ﺍﻟﻁﺎﻗﺔ ﻭﻟﻴﺴﺕ ﺍﻟﻤﺎﺩﺓ ﺴﻭﻯ‬
‫ﺘﺭﻜﻴﺯ ﻟﻠﻁﺎﻗﺔ ﻭﻴﺘﻡ ﺍﻟﺭﺒﻁ ﺒﻴﻨﻬﻤﺎ ﺒﺎﻟﻌﻼﻗﺔ ﺍﻟﺸﻬﻴﺭﺓ ‪ . E  mc2‬ﻭﻴﻨﺘﺞ ﻋﻥ ﺫﻟﻙ ﺃﻥ‬
‫ﻴﻤﻜﻥ ﺃﻥ ﻴﻜﻭﻥ ﻟﻬﺎ ﺨﻭﺍﺹ ﺘﺠﺎﺫﺒﻴﺔ‪ .‬ﻓﺈﺫﺍ‬ ‫ﻜل ﻁﺎﻗﺔ ‪ E‬ﺘﻌﺎﺩل ﻜﺘﻠﺔ ﻤﻘﺩﺍﺭﻫﺎ‬
‫‪E‬‬
‫‪c2‬‬
‫ﻁﺒﻘﻨﺎ ﺫﻟﻙ ﻋﻠﻰ ﺍﻟﻔﻭﺘﻭﻨﺎﺕ )ﺠﺴﻴﻤﺎﺕ ﺍﻟﻁﺎﻗﺔ( ﺍﻟﺘﻲ ﻁﺎﻗﺔ ﻜل ﻤﻨﻬﺎ ﺘﺴﺎﻭﻱ ‪h‬‬

‫ﻭﺒﺎﻟﺘﺎﻟﻲ ﻴﻤﻜﻥ ﺍﻟﻘﻭل ﺃﻥ ﻜﺘﻠﺔ ﺍﻟﻔﻭﺘﻭﻥ ﺘﺯﺩﺍﺩ‬ ‫ﻨﺠﺩ ﺃﻥ ﻟﻜل ﻤﻨﻬﺎ ﻜﺘﻠﺔ ﺘﻌﺎﺩل‬
‫‪h‬‬
‫‪2‬‬
‫ﺒﺎﺯﺩﻴﺎﺩ ﺘﻭﺍﺘﺭﻩ‪ ،‬ﻓﻜﺘﻠﺔ ﺍﻟﻔﻭﺘﻭﻥ ﺍﻟﺒﻨﻔﺴﺠﻲ ﺃﻜﺒﺭ ﺒﻤﺭﺘﻴﻥ ﺘﻘﺭﻴﺒﺎﹰ ﻤﻥ ﻜﺘﻠﺔ ﺍﻟﻔﻭﺘﻭﻥ‬
‫ﺍﻷﺤﻤﺭ‪ ،‬ﻭﻜﻤﺎ ﺘﺘﺠﺎﺫﺏ ﺍﻟﻜﺘل ﺍﻟﻤﺎﺩﻴﺔ ﻁﺒﻘﺎﹰ ﻟـ )‪ (9.84‬ﻓﻼﺒﺩ ﺃﻥ ﺘﻌﺎﻨﻲ ﺍﻟﻔﻭﺘﻭﻨﺎﺕ‬
‫ﺍﻟﻀﻭﺌﻴﺔ ﻨﻭﻋﺎﹰ ﻤﻥ ﺍﻟﺠﺫﺏ ﻋﻨﺩ ﻤﺭﻭﺭﻫﺎ ﺒﺎﻟﻘﺭﺏ ﻤﻥ ﺃﺠﺴﺎﻡ ﻤﺎﺩﻴﺔ ﻜﺎﻟﺸﻤﺱ ﻤﺜﻼﹰ‪،‬‬
‫ﻭﻟﻤﺎ ﻜﺎﻥ ﻤﻥ ﺍﻟﻤﺘﻌﺫﺭ ﺍﻟﺘﺤﻘﻕ ﻤﻥ ﺫﻟﻙ ﺘﺠﺭﻴﺒﻴﺎﹰ ﺒﺴﺒﺏ ﺴﻁﻭﻉ ﺍﻟﺸﻤﺱ ﻭﻋﺩﻡ‬
‫ﺍﻤﻜﺎﻨﻴﺔ ﺍﻟﻨﻅﺭ ﺍﻟﻴﻬﺎ ﻓﻲ ﺍﻟﺤﺎﻟﺔ ﺍﻟﻌﺎﺩﻴﺔ‪ ،‬ﻓﻘﺩ ﺍﻨﺘﻅﺭ ﺍﻟﻌﻠﻤﺎﺀ ﻜﺴﻭﻓﺎﹰ ﻜﻠﻴﺎﹰ ﻟﻬﺎ ﺤﺩﺙ‬

‫‪- 311 -‬‬

‫)‪Create PDF files without this message by purchasing novaPDF printer (http://www.novapdf.com‬‬
‫ﻋﺎﻡ ‪ 1919‬ﻭﺘﺤﻘﻘﻭﺍ ﺘﺠﺭﻴﺒﻴ ﺎﹰ ﻤﻥ ﻭﺠﻭﺩ ﺍﻨﺤﺭﺍﻑ ﻴﻘﺎﺭﺏ "‪ 1,75‬ﻟﻸﺸﻌﺔ ﺍﻟﻀﻭﺌﻴﺔ‬
‫ﺍﻟﻤﻨﺒﺜﻘﺔ ﻤﻥ ﺍﻟﻨﺠﻭﻡ ﻋﻨﺩ ﻤﺭﻭﺭﻫﺎ ﺒﺎﻟﻘﺭﺏ ﻤﻥ ﺍﻟﺸﻤﺱ‪.‬‬
‫ﺍﻥ ﺍﻻﻨﺤﺭﺍﻑ ﺍﻟﺴﺎﺒﻕ ﻴﺠﻌﻠﻨﺎ ﻨﻌﻴﺩ ﺍﻟﻨﻅﺭ ﻓﻲ ﺒﻨﻴﺔ ﺍﻟﻔﺭﺍﻍ ﺍﻟﺜﻼﺜﻲ ﻭﺍﻟﺭﺒﺎﻋﻲ‬
‫ﺤﻴﺙ ﻜﻨﺎ ﻨﻌﺘﺒﺭ ﺃﻥ ﺍﻟﺨﻁ ﺍﻟﻤﺴﺘﻘﻴﻡ ﻫﻭ ﻤﺴﺎﺭ ﺍﻟﺸﻌﺎﻉ ﺍﻟﻀﻭﺌﻲ‪ .‬ﻭﻟﻤﺎ ﻜﺎﻥ ﺍﻟﻤﺴﺎﺭ‬
‫ﻻﻴﻤﻜﻥ ﺃﻥ ﻴﻜﻭﻥ ﻤﺴﺘﻘﻴﻤﺎﹰ ﺍﻻ ﻓﻲ ﺍﻟﻔﻀﺎﺀ ﺍﻟﺒﻌﻴﺩ ﻋﻥ ﺃﻱ ﻤﺎﺩﺓ‪ ،‬ﺒﺤﻴﺙ ﻴﻨﻌﺩﻡ ﺃﻱ‬
‫ﺤﻘل ﻟﻠﺠﺎﺫﺒﻴﺔ‪ ،‬ﻓﺈﻨﻪ ﻴﺴﺘﺤﻴل ﺍﻟﻘﻭل ﺒﻭﺠﻭﺩ ﺍﻟﻤﺴﺎﺭ ﺍﻟﻤﺴﺘﻘﻴﻡ ﻟﻠﺸﻌﺎﻉ ﺍﻟﻀﻭﺌﻲ ﻋﻨﺩ‬
‫ﻤﺭﻭﺭﻩ ﺒﺎﻟﻘﺭﺏ ﻤﻥ ﺃﺠﺴﺎﻡ ﻤﺎﺩﻴﺔ ﻭﻻﺒﺩ ﺃﻥ ﻴﻨﺤﻨﻲ ﻫﺫﺍ ﺍﻟﻤﺴﺎﺭ‪ .‬ﻭﻫﻜﺫﺍ ﻴﺼﺒﺢ‬
‫ﺍﻟﻔﺭﺍﻍ ﻻﺍﻗﻠﻴﺩﻴﺎﹰ ﻭﻟﻡ ﻴﻌﺩ ﺍﻟﺨﻁ ﺍﻟﻤﺴﺘﻘﻴﻡ ﻭﺍﻟﺩﺍﺌﺭﺓ ﺃﺴﺎﺴﺎﹰ ﻟﻬﻨﺩﺴﺘﻪ ﻜﻤﺎ ﻜﺎﻥ ﻓﻲ‬
‫ﺍﻟﻔﺭﺍﻍ ﺍﻻﻗﻠﻴﺩﻱ‪ ،‬ﺤﻴﺙ ﺍﺴﺘﻁﻌﻨﺎ ﺘﻤﺜﻴل ﺍﻟﺤﺭﻜﺔ ﺍﻟﻤﺴﺘﻘﻴﻤﺔ ﺒﻤﻨﺤﻨﻲ ﺒﻴﺎﻨﻲ ﻤﺴﺘﻭ‬
‫ﻤﺭﺴﻭﻡ‪ .‬ﺒﺎﻟﻨﺴﺒﺔ ﻟﻤﺤﻭﺭﻴﻥ ﻤﺘﻌﺎﻤﺩﻴﻥ ‪ ox‬ﻭ ‪ .ot‬ﻭﺃﺼﺒﺢ ﻟﺯﺍﻤ ﺎﹰ ﻋﻠﻴﻨﺎ‪ ،‬ﻁﺒﻘﺎﹰ ﻟﻠﻨﻅﺭﻴﺔ‬
‫ﺍﻟﻨﺴﺒﻴﺔ ﺍﻟﻌﺎﻤﺔ ﺃﻥ ﻨﺩﺭﺱ ﻫﺫﻩ ﺍﻟﺤﺭﻜﺔ ﻻ ﺒﻭﺍﺴﻁﺔ ﻤﺤﻭﺭﻴﻥ ﻤﺘﻌﺎﻤﺩﻴﻥ ﻭﺍﻨﻤﺎ‬
‫ﺒﻭﺍﺴﻁﺔ ﺨﻁﻴﻥ ﺠﻴﻭﺩﻴﺯﻴﻴﻥ ﻤﺘﻌﺎﻤﺩﻴﻴﻥ ﻤﺭﺴﻭﻤﻴﻥ ﻋﻠﻰ ﺴﻁﺢ ﻓﺭﺍﻏﻲ‪ .‬ﻭﻗﺩ ﺒﺤﺙ‬
‫ﺃﻴﻨﺸﺘﺎﻴﻥ ﻓﻲ ﺍﻟﺒﻨﻴﺔ ﺍﻟﻬﻨﺩﺴﻴﺔ ﻟﻬﺫﺍ ﺍﻟﻔﺭﺍﻍ ﻭﻭﺠﺩ ﻀﺎﻟﺘﻪ ﻓﻴﻤﺎ ﻴﺴﻤﻰ ﻫﻨﺩﺴﺔ ﺭﻴﻤﺎﻥ‬
‫ﺍﻟﺘﻲ ﺘﻌﺘﺒﺭ ﺃﻥ ﺍﻟﻔﺭﺍﻍ ﻤﺤﺩﺏ ﺒﺤﻴﺙ ﻴﻜﻭﻥ ﻤﺠﻤﻭﻉ ﺯﻭﺍﻴﺎ ﺍﻟﻤﺜﻠﺙ ﻓﻴﻪ ﺃﻜﺜﺭ ﻤﻥ‬
‫‪ .180‬ﻭﻟﻥ ﻨﺨﻭﺽ ﻓﻲ ﺍﻟﺘﻔﺎﺼﻴل ﻷﻥ ﺫﻟﻙ ﻤﻥ ﺍﺨﺘﺼﺎﺹ ﺍﻟﺭﻴﺎﻀﻴﻴﻥ ﻭﺍﻟﺒﺎﺤﺜﻴﻥ‬
‫ﻓﻲ ﺍﻟﻨﻅﺭﻴﺔ ﺍﻟﻨﺴﺒﻴﺔ‪ .‬ﻭﻨﻜﺘﻔﻲ ﺒﺎﻟﻘﻭل ﺃﻥ ﺍﻟﺠﻤل ﺍﻻﺤﺩﺍﺜﻴﺔ ﺍﻟﻤﺘﻌﻠﻘﺔ ﺒﻬﺫﺍ ﺍﻟﻔﺭﺍﻍ‪،‬‬
‫ﻭﺍﻟﺘﻲ ﺘﻘﻴﺱ ﻓﻴﻬﺎ ﺍﻟﻤﻜﺎﻥ ﻭﺍﻟﺯﻤﺎﻥ‪ ،‬ﻫﻲ ﻤﺎﻴﺴﻤﻰ ﺠﻤل ﻏﻭﺹ ﺤﻴﺙ ﺘﺄﺨﺫ ﺍﻟﻘﻭﺍﻨﻴﻥ‬
‫ﺍﻟﻔﻴﺯﻴﺎﺌﻴﺔ ﺸﻜﻼﹰ ﻤﻭﺤﺩ ﺍﹰ ﻓﻲ ﻜﺎﻓﺔ ﻫﺫﻩ ﺍﻟﺠﻤل ﻭﻟﻜﻨﻬﺎ ﺘﺤﻭﻱ ﻋﻠﻰ ﺃﻤﺜﺎل ﺜﺎﺒﺘﺔ ﻴﺤﺩﺩﻫﺎ‬
‫ﺍﻟﺸﻜل ﺍﻟﻬﻨﺩﺴﻲ ﻟﻠﻔﺭﺍﻍ ﺍﻟﻼﺍﻗﻠﻴﺩﻱ ﺍﻟﺫﻱ ﺘﺤﺩﺩ ﺨﻭﺍﺼﻪ ﺍﻟﻤﺠﺎﻻﺕ ﺍﻟﺠﺎﺫﺒﻴﺔ ﺍﻟﺘﻲ‬
‫ﻴﻭﺠﺩ ﻀﻤﻨﻬﺎ ﻭﻴﺘﺄﺜﺭ ﺒﻬﺎ‪.‬‬
‫‪ -79‬ﻧﺘﺎﺋﺞ ﻭﲢﻘﻴﻘﺎﺕ ﲡﺮﻳﺒﻴﺔ‪:‬‬
‫ﺁ‪ -‬ﺤﺭﻜﺔ ﻜﻭﻜﺏ ﻋﻁﺎﺭﺩ‪ :‬ﺩﺭﺴﻨﺎ ﻓﻲ ﺍﻟﻤﻴﻜﺎﻨﻴﻙ ﺍﻟﻔﻴﺯﻴﺎﺌﻲ ﺍﻟﺤﺭﻜﺔ ﻓﻲ ﺤﻘل‬
‫ﻤﺭﻜﺯﻱ ﺍﺴﺘﻨﺎﺩﺍﹰ ﺍﻟﻰ ﻤﻜﺎﻨﻴﻙ ﻨﻴﻭﺘﻥ ﻭﺒﺭﻫﻨﺎ ﺃﻥ ﻤﺴﺎﺭﺍﺕ ﺍﻟﻜﻭﺍﻜﺏ ﺍﻟﺴﻴﺎﺭﺓ ﺤﻭل‬
‫ﺍﻟﺸﻤﺱ ﻫﻲ ﻗﻁﻭﻉ ﻨﺎﻗﺼﺔ ﺘﻘﻊ ﺍﻟﺸﻤﺱ ﻓﻲ ﺃﺤﺩ ﻤﺤﺭﻗﻴﻬﺎ‪ .‬ﻭﻗﺩ ﻟﻭﺤﻅ ﻓﻴﻤﺎ ﺒﻌﺩ ﺃﻥ‬
‫ﺍﻟﻤﺴﺎﺭﺍﺕ ﺍﻟﺴﺎﺒﻘﺔ ﺘﺘﺄﺭﺠﺢ ﻗﻠﻴﻼﹰ ﻭﻻﺘﺒﻘﻰ ﻓﻲ ﻤﺴﺘﻭ ﺜﺎﺒﺕ‪ ،‬ﻭﻫﺫﺍ ﻤﺎ ﻴﺨﺎﻟﻑ ﻁﺒﻘﺎﹰ‬

‫‪- 312 -‬‬

‫)‪Create PDF files without this message by purchasing novaPDF printer (http://www.novapdf.com‬‬
‫ﻟﻨﻅﺭﻴﺔ ﺍﻟﻌﺯﻡ ﺍﻟﺤﺭﻜﻲ ﻟﻠﺤﺭﻜﺔ ﺍﻟﻤﺭﻜﺯﻴﺔ ﻭﺍﻟﺘﻲ ﺘﺅﻜﺩ ﺃﻥ ﺍﻟﻤﺴﺎﺭ ﻴﺠﺏ ﺃﻥ ﻴﻜﻭﻥ‬
‫‪‬‬
‫ﻭﺍﻗﻌﺎﹰ ﻓﻲ ﻤﺴﺘﻭ ﻤﺘﻌﺎﻤﺩ ﻤﻊ ‪ . L 0‬ﻭﻜﺎﻨﺕ ﻅﺎﻫﺭﺓ ﺍﻟﺘﺄﺭﺠﺢ ﻫﺫﻩ ﻭﺍﻀﺤﺔ ﺠﺩﺍﹰ‬
‫ﺒﺎﻟﻨﺴﺒﺔ ﻟﻜﻭﻜﺏ ﻋﻁﺎﺭﺩ‪ ،‬ﺃﻗﺭﺏ ﺍﻟﻜﻭﺍﻜﺏ ﺍﻟﻰ ﺍﻟﺸﻤﺱ‪ ،‬ﻭﺍﻟﺫﻱ ﻴﺩﻭﺭ ‪ 414‬ﺩﻭﺭﺓ‬
‫ﺤﻭل ﺍﻟﺸﻤﺱ ﺨﻼل ‪ 100‬ﻋﺎﻡ ﺃﺭﻀﻲ ﻭﻗﺩ ﺒﻠﻎ ﺯﺍﻭﻴﺔ ﺘﺄﺭﺠﺢ ﺍﻟﻤﺴﺎﺭ ﺨﻼل ﻫﺫﻩ‬
‫ﺍﻟﻔﺘﺭﺓ "‪) 574‬ﺸﻜل ‪ .(9.4‬ﻭﻗﺩ ﺤﺎﻭل ﺍﻟﻌﻠﻤﺎﺀ ﻋﻨﺩﺌﺫ‪ ‬ﺘﻔﺴﻴﺭ ﻫﺫﻩ ﺍﻟﻅﺎﻫﺭﺓ ﻓﻔﺭﻀﻭﺍ‬
‫ﻭﺠﻭﺩ ﺘﻔﺎﻋل ﺒﻴﻥ ﻜﻭﻜﺏ ﻋﻁﺎﺭﺩ ﻭﻜﻭﺍﻜﺏ ﺍﻟﻤﺠﻤﻭﻋﺔ ﺍﻟﺸﻤﺴﻴﺔ ﺍﻷﺨﺭﻯ ﺒﺎﻻﻀﺎﻓﺔ‬
‫ﺍﻟﻰ ﻗﻭﺓ ﺠﺫﺏ ﺍﻟﺸﻤﺱ ﻟﻪ ﻭﺃﺠﺭﻭﺍ ﺤﺴﺎﺒﺎﺕ ﺩﻗﻴﻘﺔ ﻁﺒﻘﺎﹰ ﻟﻬﺫﻩ ﺍﻟﻔﺭﻀﻴﺔ ﻓﻭﺠﺩﻭﺍ ﺃﻥ‬
‫ﺍﻻﻨﺤﺭﺍﻑ ﺍﻟﻤﺫﻜﻭﺭ ﻴﺠﺏ ﺃﻥ ﻴﺴﺎﻭﻱ "‪ 531‬ﺃﻱ ﺒﻔﺭﻕ ﻗﺩﺭﻩ "‪.43‬‬
‫ﻭﻟﻤﺎ ﻭﻀﻊ ﺍﻨﻴﺸﺘﻴﻥ ﺍﻟﻨﻅﺭﻴﺔ ﺍﻟﻨﺴﺒﻴﺔ ﺍﻟﻌﺎﻤﺔ ﺩﺭﺱ ﺤﺭﻜﺔ ﺠﺴﻴﻡ ﻓﻲ ﺤﻘل‬
‫ﺠﺎﺫﺒﻴﺔ ﺍﻟﺸﻤﺱ ﻭﺍﺴﺘﻨﺘﺞ ﺭﻴﺎﻀﻴﺎﹰ ﻗﺎﻨﻭﻨﻴﻥ ﺠﺩﻴﺩﻴﺩﻥ ﺒﺩﻻﹰ ﻤﻥ ﻗﺎﻨﻭﻨﻲ ﺜﺒﺎﺕ ﺍﻟﺴﺭﻋﺔ‬
‫ﺍﻟﺴﻁﺤﻴﺔ ﻭﻗﺎﻨﻭﻥ ﺒﻴﻨﻪ ﺍﻟﺜﺎﻨﻲ ﻫﻤﺎ‪:‬‬
‫‪d 2‬‬
‫)‪(9.86‬‬
‫‪‬‬ ‫‪d‬‬
‫‪u ‬‬ ‫‪ 3u 2 ,‬‬ ‫‪u2‬‬ ‫‪c‬‬
‫‪2‬‬ ‫‪2‬‬ ‫‪dS‬‬
‫‪d‬‬ ‫‪c‬‬

‫‪ u ‬ﻭ ‪ ‬ﺜﺎﺒﺕ ﻏﻭﺹ ﻭ ‪ ‬ﺍﻟﺯﺍﻭﻴﺔ ﺍﻟﺴﻤﺘﻴﺔ‬ ‫ﺤﻴﺙ ‪ c‬ﺜﺎﺒﺕ ﺍﻟﺴﻁﻭﺡ ﻭ‬


‫‪1‬‬
‫‪‬‬
‫ﺃﻤﺎ ‪ dS‬ﻓﻬﻭ ﻋﻨﺼﺭ ﺍﻟﻤﺠﺎل ﺍﻟﺫﻱ ﻋﺭﻑ ﺴﺎﺒﻘﺎﹰ‪ ،‬ﻭﻗﺩ ﻁﺒﻕ ﺍﻨﻴﺸﺘﻴﻥ ﺍﻟﻤﻌﺎﺩﻟﺘﻴﻥ‬
‫ﺍﻟﺴﺎﺒﻘﺘﻴﻥ ﻟﺩﺭﺍﺴﺔ ﻜﻭﻜﺏ ﻋﻁﺎﺭﺩ ﻓﻭﺠﺩ ﺃﻥ ﺍﻻﻨﺤﺭﺍﻑ ﺍﻟﻤﻨﻭﻩ ﻋﻨﻪ ﺴﺎﺒﻘﺎﹰ ﻴﺠﺏ ﺃﻥ‬
‫ﻴﺴﺎﻭﻱ "‪ 573‬ﺨﻼل ﻗﺭﻥ ﻭﺍﺤﺩ ﺃﻱ ﺒﺎﺨﺘﻼﻑ ﺜﺎﻨﻴﺔ ﻭﺍﺤﺩﺓ ﻓﻘﻁ ﻋﻥ ﺍﻟﺘﺠﺭﺒﺔ‪ .‬ﻭﻗﺩ‬
‫ﻜﺎﻥ ﻫﺫﺍ ﺩﻋﻤﺎﹰ ﻜﺒﻴﺭﺍﹰ ﻟﻨﻅﺭﻴﺘﻪ ﺍﻟﺸﻬﻴﺭﺓ‪.‬‬

‫‪- 313 -‬‬

‫)‪Create PDF files without this message by purchasing novaPDF printer (http://www.novapdf.com‬‬
‫ﺍﻟﺸﻜل )‪(9.4‬‬
‫ﺏ‪ -‬ﺘﺄﺜﻴﺭ ﺍﻟﺠﺎﺫﺒﻴﺔ ﻋﻠﻰ ﺴﻴﺭ ﺍﻟﺯﻤﻥ‪:‬‬
‫ﻟﻘﺩ ﺘﻨﺒﺄﺕ ﺍﻟﻨﻅﺭﻴﺔ ﺍﻟﻨﺴﺒﻴﺔ ﺍﻟﻌﺎﻤﺔ ﺒﺘﻘﺩﻴﻡ ﺃﻭ ﺘﺄﺨﻴﺭ ﺍﻟﺴﺭﻋﺎﺕ ﻋﻨﺩﻤﺎ ﺘﻨﺘﻘل‬
‫ﻤﻥ ﻤﺠﺎل ﺠﺫﺏ ﺇﻟﻰ ﺁﺨﺭ ﺸﺩﺘﻪ ﺃﺼﻐﺭ ﺃﻭ ﺃﻜﺒﺭ ﻤﻥ ﺍﻟﺤﻘل ﺍﻟﺫﻱ ﻜﺎﻨﺕ ﻓﻴﻪ ﻋﻠﻰ‬
‫ﺍﻟﺘﺭﺘﻴﺏ‪ .‬ﻓﺈﺫﺍ ﻜﺎﻨﺕ ‪ t 0‬ﺍﻟﻔﺘﺭﺓ ﺍﻟﺯﻤﻨﻴﺔ ﺒﻴﻥ ﺩﻗﺎﺕ ﺴﺎﻋﺔ ﻤﺎ ﻋﻨﺩﻤﺎ ﻻﺘﻜﻭﻥ ﻤﺘﺄﺜﺭﺓ‬
‫ﺒﺄﻱ ﺤﻘل ﺠﺎﺫﺏ ﻓﺈﻥ ﻫﺫﻩ ﺍﻟﻔﺘﺭﺓ ﺘﺼﺒﺢ ‪ t‬ﻋﻨﺩﻤﺎ ﺘﻭﻀﻊ ﻓﻲ ﺤﻘل ﺠﺫﺏ ﻨﺎﺘﺞ ﻋﻥ‬
‫ﻜﺘﻠﺔ ‪ m‬ﻨﺼﻑ ﻗﻁﺭﻫﺎ ‪ R‬ﺒﺤﻴﺙ ﻴﻜﻭﻥ‪:‬‬
‫)‪(9.87‬‬
‫‪t0‬‬
‫‪t‬‬
‫‪M‬‬
‫‪1  2f‬‬
‫‪R‬‬
‫ﺤﻴﺙ ‪ f‬ﺜﺎﺒﺕ ﺍﻟﺠﺫﺏ ﻟﻠﻜﺘﻠﺔ ‪ .M‬ﻭﻴﻨﺘﺞ ﻋﻥ ﺍﻟﺘﻐﻴﺭ ﻓﻲ ﺍﻟﺯﻤﻥ ﺘﻐﻴﺭ ﺍﹰ ﻓﻲ‬
‫ﺘﻭﺍﺘﺭ ﺍﻻﺸﻌﺎﻋﺎﺕ ) ‪ ( ‬ﺍﻟﻭﺍﺭﺩﺓ ﻤﻥ ﻨﺠﻡ ﻤﺎ‪ ،‬ﻜﺎﻟﺸﻤﺱ ﻤﺜﻼﹰ‪ ،‬ﺍﻟﻰ ﻜﻭﻜﺏ ﻤﺜل‬
‫‪1‬‬
‫‪T‬‬
‫ﺍﻷﺭﺽ‪ ،‬ﻤﻤﺎ ﻴﺴﺒﺏ ﺍﻨﺯﻴﺎﺤﺎﹰ ﻓﻲ ﺍﻟﺨﻁﻭﻁ ﺍﻟﻁﻴﻔﻴﺔ ﻨﺤﻭ ﺍﻷﺤﻤﺭ ﻭﻴﺒﺭﻫﻥ ﺫﻟﻙ‬
‫ﻨﻅﺭﻴﺎﹰ ﻜﻤﺎﻴﻠﻲ‪:‬‬

‫‪- 314 -‬‬

‫)‪Create PDF files without this message by purchasing novaPDF printer (http://www.novapdf.com‬‬
‫ﻟﻴﻜﻥ ﺍﻟﻔﻭﺘﻭﻥ ﺫﻭ ﺍﻟﻁﺎﻗﺔ ‪ h‬ﺍﻟﺼﺎﺩﺭ ﻋﻥ ﻨﺠﻡ ﻜﺘﻠﺘﻪ ‪ M‬ﻭﻨﺼﻑ ﻗﻁﺭﻩ ‪.R‬‬
‫‪ ، m ‬ﺃﻤﺎ ﻁﺎﻗﺘﻪ ﺍﻟﻜﺎﻤﻨﺔ ﺍﻟﻨﺎﺘﺠﺔ‬ ‫ﺍﻥ ﻜﺘﻠﺔ ﻫﺫﺍ ﺍﻟﻔﻭﺘﻭﻥ ﺍﻟﻤﻌﺎﺩﻟﺔ ﻟﻁﺎﻗﺘﻪ ﻫﻲ‬
‫‪h‬‬
‫‪2‬‬
‫ﻋﻥ ﻭﺠﻭﺩﻩ ﻓﻲ ﺤﻘل ﺠﺫﺏ ﺍﻟﻨﺠﻡ ﺍﻟﺴﺎﺒﻕ ﻓﻬﻲ‪ ،‬ﻁﺒﻘﺎﹰ ﻟﻤﺎ ﺭﺃﻴﻨﺎﻩ ﺴﺎﺒﻘﺎﹰ‪.‬‬
‫‪ ‬‬
‫)‪(9.88‬‬
‫‪fMm fMh‬‬
‫‪‬‬
‫‪V   F . dr  ‬‬
‫‪h‬‬
‫‪‬‬
‫‪c2R‬‬
‫ﻭﺘﻜﻭﻥ ﺍﻟﻁﺎﻗﺔ ﺍﻟﻜﻠﻴﺔ ﻟﻠﻔﻭﺘﻭﻥ ﻋﻨﺩ ﺍﻨﺒﻌﺎﺜﻪ ﻤﻥ ﺍﻟﻨﺠﻡ‪:‬‬
‫)‪(9.89‬‬
‫‪fMh‬‬ ‫‪fM‬‬
‫‪E  h ‬‬ ‫‪ h(1 ‬‬ ‫)‬
‫‪c2R‬‬ ‫‪c2 R‬‬
‫ﺃﻤﺎ ﻁﺎﻗﺔ ﻫﺫﺍ ﺍﻟﻔﻭﺘﻭﻥ ﺍﻟﻜﻠﻴﺔ ﻋﻨﺩ ﻭﺼﻭﻟﻪ ﺇﻟﻰ ﻜﻭﻜﺏ ﺍﻷﺭﺽ ﺒﺘﻭﺍﺘﺭ ' ‪‬‬
‫ﻓﻬﻲ '‪) h‬ﺒﺎﻫﻤﺎل ﻁﺎﻗﺘﻪ ﺍﻟﻜﺎﻤﻨﺔ ﻋﻠﻰ ﺍﻷﺭﺽ ﺒﺎﻟﻨﺴﺒﺔ ﻟﻬﺫﻩ ﺍﻟﻁﺎﻗﺔ ﻋﻠﻰ ﺍﻟﻨﺠﻡ(‬
‫ﻭﺍﺫﺍ ﺍﺴﺘﻔﺩﻨﺎ ﻤﻥ ﻗﺎﻨﻭﻥ ﻤﺼﻭﻨﻴﺔ ﺍﻟﻁﺎﻗﺔ ﻓﺈﻨﻨﺎ ﻨﺠﺩ‪:‬‬
‫‪f'M‬‬
‫‪h '  h(1 ‬‬ ‫)‬
‫‪c2R‬‬
‫ﻭﻤﻨﻪ‪:‬‬
‫)‪(9.90‬‬
‫'‪‬‬ ‫‪fM‬‬
‫‪ 1‬‬
‫‪‬‬ ‫‪c2R‬‬
‫ﻓﻴﺴﺎﻭﻱ‪:‬‬‫ﺃﻤﺎ ﺍﻟﺘﻐﻴﺭ ﺍﻟﻨﺴﺒﻲ ﻓﻲ ﺍﻟﺘﻭﺍﺘﺭ ﺃﻱ‬
‫‪‬‬
‫‪‬‬
‫'‪   ‬‬ ‫‪ ' fM‬‬
‫‪‬‬ ‫‪ 1 ‬‬ ‫)‪(9.91‬‬
‫‪‬‬ ‫‪‬‬ ‫‪ c2R‬‬
‫ﻓﺈﻥ ﺍﻟﺘﻭﺍﺘﺭ ﻴﺯﺩﺍﺩ ﻭﺒﺎﻟﺘﺎﻟﻲ ﺘﺘﻨﺎﻗﺹ ﻁﻭل ﺍﻟﻤﻭﺠﺔ ﻭﻴﻨﺯﺍﺡ‬ ‫‪  0‬‬ ‫ﻭﺒﻤﺎ ﺃﻥ‬
‫ﺍﻟﺨﻁ ﺍﻟﻁﻴﻔﻲ ﻨﺤﻭ ﺍﻷﺤﻤﺭ‪ ،‬ﻭﻴﻤﻜﻥ ﺍﻟﺘﺤﻘﻕ ﻤﻥ ﺫﻟﻙ ﺘﺠﺭﻴﺒﻴﺎﹰ ﺒﻤﻘﺎﺭﻨﺔ ﺍﻟﻀﻭﺀ ﺍﻵﺘﻲ‬
‫ﻤﻥ ﺍﻟﻨﺠﻭﻡ ﺒﺫﻟﻙ ﺍﻟﺘﻲ ﻨﺤﺼل ﻋﻠﻴﻪ ﻓﻲ ﺍﻟﻤﺨﺘﺒﺭ‪ .‬ﻭﺇﺫﺍ ﺃﺨﺫﻨﺎ ﻨﺠﻡ ﺍﻟﺸﻤﺱ ﻤﺜﻼﹰ‬
‫ﻓﺈﻨﻨﺎ ﻨﺠﺩ‪:‬‬ ‫ﻭﺤﺴﺒﻨﺎ‬
‫‪‬‬
‫‪‬‬
‫‪6,67  10 11  1,99  10 30‬‬
‫)‪ 2,12  10  6 (9.92‬‬
‫‪ fM‬‬
‫‪‬‬ ‫‪‬‬
‫‪‬‬ ‫‪2‬‬ ‫‪8‬‬ ‫‪2‬‬ ‫‪8‬‬
‫‪c R‬‬ ‫‪(3.10 )  6.4  10‬‬
‫ﻭﻗﺩ ﺤﺴﺒﺕ ‪ ‬ﺍﻟﻤﻘﺎﺒﻠﺔ ﻟﻠﻀﻭﺀ ﺍﻷﺨﻀﺭ ) ‪ (  5000 A ‬ﻓﻭﺠﺩ ﺃﻥ‬
‫‪   0,01 A ‬ﻭﻫﻲ ﻗﻴﻤﺔ ﺼﻐﻴﺭﺓ ﺠﺩﺍﹰ ﻻﻴﻤﻜﻥ ﻤﻼﺤﻅﺘﻬﺎ ﺒﺴﺒﺏ ﻋﺭﺽ ﺍﻟﺨﻁ‬

‫‪- 315 -‬‬

‫)‪Create PDF files without this message by purchasing novaPDF printer (http://www.novapdf.com‬‬
‫ﺍﻟﻁﻴﻔﻲ ﺍﻟﻨﺎﺘﺞ ﻋﻥ ﻅﺎﻫﺭﺓ ﺩﻭﺒﻠﺭ‪ .‬ﻭﻟﻜﻥ ﺍﺫﺍ ﺃﺠﺭﻴﻨﺎ ﺍﻟﺤﺴﺎﺏ ﺍﻟﺴﺎﺒﻕ ﺒﺎﺴﺘﺒﺩﺍل‬
‫ﺍﻟﺸﻤﺱ ﺒﻨﺠﻡ ﺁﺨﺭ ﻤﻥ ﺘﻠﻙ ﺍﻟﻨﺠﻭﻡ ﺍﻟﺘﻲ ﻜﺜﺎﻓﺘﻬﺎ ﻜﺒﻴﺭﺓ ﺠﺩﺍﹰ ﺒﺤﻴﺙ ﺘﺒﻠﻎ ﺍﻟﻨﺴﺒﺔ‬
‫ﺍﻟﻴﻤﺎﻨﻴﺔ"‬ ‫ﺍﻟﺸﻌﺭﻱ‬ ‫"ﺭﻓﻴﻕ‬ ‫ﺍﻟﻤﺴﻤﻰ‬ ‫ﺍﻟﻨﺠﻡ‬ ‫)ﻤﺜل‬ ‫ﻜﺒﻴﺭﺍﹰ‬ ‫ﺤﺩﺍﹰ‬
‫‪M‬‬
‫‪R‬‬
‫)‪ (Sompanion of Syius‬ﻭﺍﻟﺫﻱ ﺘﺼل ﻜﺜﺎﻓﺘﻪ ﺍﻟﻰ ‪ ،1,5  108 kg / m 3‬ﺃﻱ ﻜﺘﻠﺔ‬
‫ﺍﻟﺴﻨﺘﻤﺘﺭ ﺍﻟﻤﻜﻌﺏ ﺍﻟﻭﺍﺤﺩ ﻤﻨﻪ ﺘﺴﺎﻭﻱ ‪ (150 kg‬ﻓﻴﻤﻜﻥ ﻤﻼﺤﻅﺔ ﺍﻻﻨﺯﻴﺎﺡ ﺍﻟﺴﺎﺒﻕ‬
‫ﺘﺠﺭﻴﺒﻴﺎﹰ‪.‬‬
‫ﻭﻗﺩ ﺍﺴﺘﻁﺎﻉ ﺍﻟﻌﻠﻤﺎﺀ ﻓﻴﻤﺎ ﺒﻌﺩ ﺘﺤﻘﻴﻕ ﺍﻻﻨﺯﻴﺎﺡ ﺍﻟﻁﻴﻔﻲ ﺍﻟﺴﺎﺒﻕ ﻓﻲ ﺍﻟﻤﺨﺘﺒﺭ‪،‬‬
‫ﻋﻨﺩﻤﺎ ﺃﺠﺭﻭﺍ ﺘﺠﺭﺒﺔ ﻟﻘﻴﺎﺱ ﺘﻭﺍﺘﺭﺍﺕ ﺍﺸﻌﺎﻋﺎﺕ ‪" ‬ﺍﻟﺴﺎﻗﻁﺔ" ﻤﻥ ﺍﺭﺘﻔﺎﻉ ‪ ،20 m‬ﺍﺫ‬
‫ﻤﻥ ﺍﻟﻤﻌﻠﻭﻡ ﺃﻥ ﻁﺎﻗﺔ ﺍﻟﺠﺴﻴﻡ ﺍﻟﺴﺎﻗﻁ ﻤﻥ ﺍﺭﺘﻔﺎﻉ ‪ ‬ﺘﺘﻐﻴﺭ ﺒﻤﻘﺩﺍﺭ ‪ ، mg‬ﻓﺈﺫﺍ ﻁﺒﻘﻨﺎ‬
‫ﻗﺎﻨﻭﻥ ﻤﺼﻭﻨﻴﺔ ﺍﻟﻁﺎﻗﺔ ﻋﻠﻰ ﺍﻟﻔﻭﺘﻭﻨﺎﺕ ﺍﻟﻤﺫﻜﻭﺭﺓ ﻓﺈﻨﻨﺎ ﻨﺠﺩ ﺒﺴﻬﻭﻟﺔ‪:‬‬
‫‪g‬‬
‫)‪(9.93‬‬
‫‪h‬‬
‫‪h '  h  mg ‬‬ ‫‪g  h(1 ‬‬ ‫)‬
‫‪2‬‬
‫‪c‬‬ ‫‪c2‬‬
‫ﺤﻴﺙ ‪ ‬ﺘﻭﺍﺘﺭ ﺍﻟﻔﻭﺘﻭﻥ ﻋﻠﻰ ﺍﺭﺘﻔﺎﻉ ‪ ‬ﻭ ' ‪ ‬ﺘﻭﺍﺘﺭﻩ ﺍﻟﻨﻬﺎﺌﻲ ﻓﺈﺫﺍ ﻜﺎﻥ‬
‫‪   20 m‬ﻓﺈﻨﻨﺎ ﻨﺤﺼل ﻋﻠﻰ ﺍﻟﺘﻐﻴﺭ ﺍﻟﻨﺴﺒﻲ ﺍﻟﺘﺎﻟﻲ ﻓﻲ ﺍﻟﺘﻭﺍﺘﺭ‪:‬‬
‫‪ g‬‬ ‫‪9,8  20‬‬
‫‪‬‬ ‫‪‬‬ ‫‪ 2,2  10 15‬‬
‫‪‬‬ ‫‪2‬‬ ‫‪8‬‬ ‫‪2‬‬
‫‪c‬‬ ‫) ‪(3  10‬‬
‫ﻭﻗﺩ ﺃﺜﺒﺘﺕ ﺍﻟﺘﺠﺭﺒﺔ ﺼﺤﺔ ﺍﻟﻌﻼﻗﺔ ﺍﻟﺴﺎﺒﻘﺔ ﺒﻤﺎ ﻻﻴﺩﻉ ﻤﺠﺎﻻﹰ ﻟﻠﺸﻙ ﻓﻲ ﺘﺄﺜﻴﺭ‬
‫ﺍﻟﺠﺎﺫﺒﻴﺔ ﻋﻠﻰ ﺍﻟﺯﻤﻥ‪.‬‬

‫‪- 316 -‬‬

‫)‪Create PDF files without this message by purchasing novaPDF printer (http://www.novapdf.com‬‬
‫ﻣﺴﺎﺋﻞ ﺍﻟﻔﺼﻞ ﺍﻟﺘﺎﺳﻊ‬

‫‪ -1‬ﻤﺴﺭﻉ ﻟﻠﺒﺭﻭﺘﻭﻨﺎﺕ ﻴﺴﺘﻁﻴﻊ ﺍﻜﺴﺎﺒﻬﺎ ﻁﺎﻗﺔ ﻜﺎﻓﻴﺔ ﻟﺘﺸﻜﻴل ﺃﺯﻭﺍﺝ ﻤﻥ‬


‫ﺍﻟﺒﺭﻭﺘﻭﻨﺎﺕ ﻭﻀﺩﻴﺩﺍﺘﻬﺎ ‪ proton-antiproton pairs‬ﻁﺒﻘﺎﹰ ﻟﻠﺘﻔﺎﻋل‪:‬‬
‫‪ ‬‬ ‫‪‬‬
‫) ‪p  p  p  p  (p  p‬‬

‫ﺍﺤﺴﺏ ﺃﺼﻐﺭ ﻁﺎﻗﺔ ﻻﺯﻤﺔ ﻟﻠﺒﺭﻭﺘﻭﻨﺎﺕ ﺍﻟﻭﺍﺭﺩﺓ ﻟﻜﻲ ﻴﺘﻡ ﺍﻟﺘﻔﺎﻋل؟‬


‫ﺍﻟﺤل‪ :‬ﻤﻥ ﺍﻟﻤﻌﻠﻭﻡ ﻓﻲ ﺍﻟﻔﻴﺯﻴﺎﺀ ﺍﻟﻨﻭﻭﻴﺔ ﺃﻨﻪ ﻋﻨﺩﻤﺎ ﺘﻜﻭﻥ ﻁﺎﻗﺔ ﺍﻟﺒﺭﻭﺘﻭﻥ‬
‫ﺍﻟﻭﺍﺭﺩ ﺼﻐﺭﻯ ﺒﺤﻴﺙ ﻴﺘﻡ ﺍﻟﺘﻔﺎﻋل ﻓﺈﻥ ﺍﻟﺠﺴﻴﻤﺎﺕ ﺍﻷﺭﺒﻌﺔ ﺍﻟﻨﺎﺘﺠﺔ ﻋﻥ ﺍﻟﺘﻔﺎﻋل‬
‫ﺴﺘﻨﻁﻠﻕ ﺒﺴﺭﻋﺔ ﺘﻜﺎﺩ ﺘﻜﻭﻥ ﻤﻌﺩﻭﻤﺔ‪ ،‬ﻭﺒﺎﻟﺘﺎﻟﻲ ﺒﻁﺎﻗﺔ ﺘﺴﺎﻭﻱ ‪ 4mc2‬ﺤﻴﺙ ‪ m‬ﻜﺘﻠﺔ‬
‫ﺍﻟﺴﻜﻭﻥ ﻟﻠﺒﺭﻭﺘﻭﻥ‪ ،‬ﻓﺈﺫﺍ ﺍﺴﺘﺨﺩﻤﻨﺎ ﻤﺠﻤﻭﻋﺔ ﺍﺤﺩﺍﺜﻴﺎﺕ ﻤﺭﻜﺯ ﺍﻟﻜﺘﻠﺔ ﺤﻴﺙ ﻴﻘﺘﺭﺏ‬
‫ﺍﻟﺒﺭﻭﺘﻭﻨﺎﻥ ﻤﻥ ﺒﻌﻀﻬﻤﺎ ﺒﺴﺭﻋﺔ ‪ v cM‬ﻟﻜل ﻤﻨﻬﻤﺎ ﺒﺎﺘﺠﺎﻫﻴﻥ ﻤﺘﻌﺎﻜﺴﻴﻥ ﺃﻱ ﺃﻥ ﻜﻼﹰ‬
‫ﻤﻨﻬﻤﺎ ﻴﻘﺘﺭﺏ ﻤﻥ ﺍﻵﺨﺭ ﻗﺒل ﺍﻟﺘﻔﺎﻋل ﺒﺴﺭﻋﺔ ‪ v cM‬ﻭﻴﻜﻭﻥ ﻁﺎﻗﺔ ﻜل ﻤﻨﻬﻤﺎ‬
‫‪mc 2‬‬
‫ﻭﻫﻜﺫﺍ ﻨﺠﺩ ﺒﺘﻁﺒﻴﻕ ﻗﺎﻨﻭﻥ ﻤﺼﻭﻨﻴﺔ ﺍﻟﻁﺎﻗﺔ‪:‬‬
‫‪v2‬‬
‫‪1‬‬
‫‪c2‬‬
‫‪2mc 2‬‬
‫‪ 4mc 2‬‬
‫‪v2‬‬
‫‪1  cM‬‬
‫‪c2‬‬
‫ﻭﻤﻨﻪ ﻨﺤﺴﺏ ‪ v cM‬ﺤﻴﺙ ﻨﺠﺩ‪:‬‬
‫‪3‬‬
‫‪v cM ‬‬ ‫‪c‬‬
‫‪2‬‬
‫ﻭﻟﻠﺤﺼﻭل ﻋﻠﻰ ﻫﺫﻩ ﺍﻟﺴﺭﻋﺔ ﺜﻡ ﺤﺴﺎﺏ ﺍﻟﻁﺎﻗﺔ ﺍﻟﻤﻁﻠﻭﺒﺔ ﻓﻲ ﺍﻟﻤﺠﻤﻭﻋﺔ‬
‫ﺍﻟﻤﺨﺒﺭﻴﺔ ﻨﺴﺘﻔﻴﺩ ﻤﻥ ﻋﻼﻗﺔ ﺘﺭﻜﻴﺏ ﺍﻟﺴﺭﻉ ﺍﻟﻤﻌﺭﻭﻓﺔ )‪ (8.30‬ﺤﻴﺙ ﻨﺠﺩ ﻤﻥ‬
‫ﺍﻷﻭﻟﻰ ﺒﺎﻋﺘﺒﺎﺭ ﺃﻥ ‪ u' x‬ﻫﻲ ﺴﺭﻋﺔ ﺍﻟﺠﺴﻴﻡ ﻓﻲ ﻤﺠﻤﻭﻋﺔ ﻤﺭﻜﺯ ﺍﻟﻜﺘﻠﺔ ﻭﺘﺴﺎﻭﻱ‬

‫‪- 317 -‬‬

‫)‪Create PDF files without this message by purchasing novaPDF printer (http://www.novapdf.com‬‬
‫‪ v cM‬ﻭ ‪ V‬ﻫﻲ ﺴﺭﻋﺔ ﻤﺭﻜﺯ ﺍﻟﻜﺘﻠﺔ ﻨﻔﺴﻪ ﺒﺎﻟﻨﺴﺒﺔ ﻟﻠﻤﺠﻤﻭﻋﺔ ﺍﻟﻤﺨﺒﺭﻴﺔ ﺃﻱ ﺘﺴﺎﻭﻱ‬
‫‪ v cM‬ﻭﻫﻜﺫﺍ ﻨﺠﺩ‪:‬‬
‫‪v‬‬ ‫‪ v cM 4 3‬‬
‫‪v  cM‬‬ ‫‪‬‬ ‫‪C‬‬
‫‪c 2cM‬‬ ‫‪7‬‬
‫‪1‬‬
‫‪c2‬‬
‫ﻭﻋﻨﺩﺌﺫ‪ ‬ﻨﺤﺴﺏ ﺍﻟﻁﺎﻗﺔ ﺍﻟﺤﺭﻜﻴﺔ ﻁﺒﻘﺎﹰ ﻟﻠﻌﻼﻗﺔ )‪ (9.31‬ﺤﻴﺙ ﻨﺠﺩ‪:‬‬
‫‪‬‬ ‫‪‬‬
‫‪ m‬‬ ‫‪‬‬
‫‪E kin  [ m( v)  M ]c 2  ‬‬ ‫‪ m   6 mc 2‬‬
‫‪‬‬ ‫‪48‬‬ ‫‪‬‬
‫‪ 1‬‬ ‫‪‬‬
‫‪‬‬ ‫‪49‬‬ ‫‪‬‬

‫‪ -2‬ﺃﻭﺠﺩ ﻗﺎﻨﻭﻥ ﺍﻟﺤﺭﻜﺔ ﺍﻟﻤﺴﺘﻘﻴﻤﺔ ﺍﻟﻤﺘﺴﺎﺭﻋﺔ ﺒﺎﻨﺘﻅﺎﻡ )ﻓﻲ ﺠﻤﻠﺘﻬﺎ‬


‫ﺍﻟﺨﺎﺼﺔ( ﻓﻲ ﺍﻟﻨﻅﺭﻴﺔ ﺍﻟﻨﺴﺒﻴﺔ ﻭﺒﺭﻫﻥ ﺃﻥ ﻫﺫﺍ ﺍﻟﻘﺎﻨﻭﻥ ﻴﺅﻭل ﺇﻟﻰ ﻤﺎ ﻴﻘﺎﺒﻠﻪ ﻓﻲ‬
‫ﺍﻟﻤﻴﻜﺎﻨﻴﻙ ﺍﻟﺘﻘﻠﻴﺩﻱ ﻋﻨﺩﻤﺎ ‪. v  c‬‬

‫‪ -3‬ﻤﻥ ﺍﻟﻤﻌﻠﻭﻡ ﺃﻨﻪ ﻴﻤﻜﻥ ﻻﺸﻌﺎﻋﺎﺕ ‪ ‬ﻓﻲ ﺸﺭﻭﻁ ﻤﻨﺎﺴﺒﺔ ﺃﻥ ﺘﺸﻜل‬


‫ﺃﺯﻭﺍﺠﺎﹰ ﻤﻥ ﺍﻟﻴﻭﺯﻴﺘﺭﻭﻨﺎﺕ ﻭﺍﻻﻟﻜﺘﺭﻭﻨﺎﺕ ) ‪ (e  , e ‬ﺒﺭﻫﻥ ﺃﻥ ﺫﻟﻙ ﻻﻴﻤﻜﻥ ﺃﻥ ﻴﺘﻡ‬
‫ﺍﻻ ﺒﻭﺠﻭﺩ ﺠﺴﻴﻡ ﺜﺎﻟﺙ ﻴﺄﺨﺫ ﻜﻤﻴﺔ ﻤﻥ ﺍﻟﻁﺎﻗﺔ ﺒﺤﻴﺙ ﻴﺘﺤﻘﻕ ﻗﺎﻨﻭﻥ ﻤﺼﻭﻨﻴﺔ ﺍﻟﻁﺎﻗﺔ‬
‫ﻓﻲ ﺍﻟﻔﺭﺍﻍ ﺍﻟﺭﺒﺎﻋﻲ‪ ،‬ﺜﻡ ﺃﻭﺠﺩ ﺃﺼﻐﺭ ﻁﺎﻗﺔ ﻻﺯﻤﺔ ﻟﻠﻔﻭﺘﻭﻨﺎﺕ ﻟﻜﻲ ﻴﺘﻡ ﺍﻟﺘﻔﺎﻋل‬
‫ﺍﻟﺴﺎﺒﻕ‪.‬‬

‫‪ -4‬ﻴﻨﻘﺴﻡ ﺠﺴﻴﻡ ﻜﺘﻠﺘﻪ ‪ m‬ﺇﻟﻰ ﺜﻼﺙ ﺠﺴﻴﻤﺎﺕ ﻜﺘﻠﻬﺎ ‪ m 3, m 2 , m1‬ﺃﻭﺠﺩ‬


‫‪ E (max‬ﻨﺘﻴﺠﺔ ﻟﻬﺫﺍ ﺍﻻﻨﻘﺴﺎﻡ‪.‬‬
‫)‪1‬‬
‫ﺃﻜﺒﺭ ﻁﺎﻗﺔ ﻴﻤﻜﻥ ﺃﻥ ﻴﺄﺨﺫﻫﺎ ﺍﻟﺠﺴﻴﻡ ﺍﻷﻭل‬

‫‪ -5‬ﻴﺘﺤﺭﻙ ﺠﺴﻴﻡ ﻜﺘﻠﺘﻪ ﺍﻟﺴﺎﻜﻨﺔ ‪ m‬ﻋﻠﻰ ﺍﻟﻤﺤﻭﺭ ‪ ox‬ﺤﺴﺏ ﻗﺎﻨﻭﻥ ﺍﻟﺤﺭﻜﺔ‬


‫ﺍﻟﺘﺎﻟﻲ‪:‬‬
‫‪x  b 2  c2t 2  b‬‬

‫‪- 318 -‬‬

‫)‪Create PDF files without this message by purchasing novaPDF printer (http://www.novapdf.com‬‬
‫ﺍﺤﺴﺏ ﺍﻟﻘﻭﺓ ﺍﻟﺘﻲ ﺘﺴﺒﺏ ﻫﺫﻩ ﺍﻟﺤﺭﻜﺔ‪.‬‬

‫ﻭﻴﺼﻁﺩﻡ ﺍﺼﻁﺩﺍﻤﺎﹰ ﻏﻴﺭ‬ ‫‪ -6‬ﻴﺘﺤﺭﻙ ﺠﺴﻴﻡ ﻜﺘﻠﺘﻪ ﺍﻟﺴﺎﻜﻨﺔ ‪ m‬ﺒﺴﺭﻋﺔ‬


‫‪4c‬‬
‫‪5‬‬
‫ﻤﺭﻥ ﺒﺠﺴﻴﻡ ﻟﻪ ﻨﻔﺱ ﺍﻟﻜﺘﻠﺔ‪.‬‬
‫ﺁ‪ -‬ﺍﺤﺴﺏ ﺴﺭﻋﺔ ﺍﻟﺠﺴﻴﻡ ﺍﻟﻨﺎﺘﺞ‪.‬‬
‫ﺏ‪ -‬ﺍﺤﺴﺏ ﻜﺘﻠﺘﻪ ﺍﻟﺴﺎﻜﻨﺔ‪.‬‬

‫‪ -7‬ﻴﻤﻜﻥ ﺃﻥ ﺘﺘﺸﻜل ﺍﻟﻤﻴﺯﻭﻨﺎﺕ ‪  ‬ﻋﻨﺩ ﺍﺼﻁﺩﺍﻡ ﺒﺭﻭﺘﻭﻥ ﺴﺭﻴﻊ ﺒﺂﺨﺭ‬


‫ﻁﺒﻘﺎﹰ ﻟﻠﻤﻌﺎﺩﻟﺔ‪:‬‬
‫‪p  p  D   ‬‬
‫ﺍﺤﺴﺏ ﺃﺼﻐﺭ ﻁﺎﻗﺔ ﻟﻠﺒﺭﻭﺘﻭﻨﺎﺕ ﻟﻜﻲ ﻴﺘﻡ ﺍﻟﺘﻔﺎﻋل ﺍﻟﺴﺎﺒﻕ‪.‬‬

‫‪ -8‬ﺍﺤﺴﺏ ﻁﺎﻗﺔ ﺍﺭﺘﺒﺎﻁ ﻨﻭﺍﺓ ﺍﻟﻠﻴﺜﻴﻭﻡ ﺍﻟﻤﺅﻟﻔﺔ ﻤﻥ ﺜﻼﺜﺔ ﺒﺭﻭﺘﻭﻨﺎﺕ ﻭﺃﺭﺒﻌﺔ‬


‫ﻨﺘﺭﻭﻨﺎﺕ ﺍﺫﺍ ﻋﻠﻤﺕ ﺃﻥ ﻜﺘﻠﺘﻬﺎ ﺍﻟﺴﺎﻜﻨﺔ ﺘﺴﺎﻭﻱ ‪ 1,6445 .10 25 kg‬ﻭﺃﻥ ﻜﺘﻠﺔ ﻜل‬
‫ﻤﻥ ﺍﻟﺒﺭﻭﺘﻭﻥ ﻭﺍﻟﻨﺘﺭﻴﻥ ﻫﻲ ﻋﻠﻰ ﺍﻟﺘﺭﺘﻴﺏ‪:‬‬
‫‪ 1,67265 .10 27 kg‬ﻭ ‪1,67495 .10 27 kg‬‬

‫‪ -9‬ﺍﺤﺴﺏ ﺍﻟﺘﻐﻴﺭ ﻓﻲ ﺴﺭﻋﺔ ﺠﺴﻴﻡ ﺸﺤﻨﺘﻪ ‪ e‬ﻭﻜﺘﻠﺘﻪ ‪ m‬ﺒﻌﺩ ﺃﻥ ﻴﺠﺘﺎﺯ ﻓﺭﻗﺎﹰ‬


‫ﻓﻲ ﺍﻟﻜﻤﻭﻥ ﻗﺩﺭﻩ ‪.V‬‬

‫‪ -10‬ﻴﻨﻘﺴﻡ ﺠﺴﻴﻡ ﻜﺘﻠﺘﻪ ‪ m‬ﻤﻥ ﺍﻟﺴﻜﻭﻥ ﺇﻟﻰ ﺠﺴﻴﻤﻴﻥ ﻜﺘﻠﺘﺎﻫﻤﺎ ‪ m1‬ﻭ ‪. m 2‬‬
‫ﺍﺤﺴﺏ ﻁﺎﻗﺔ ﻜل ﻤﻥ ﺍﻟﺠﺴﻴﻤﻴﻥ ﺍﻟﻨﺎﺘﺠﻴﻥ ﺜﻡ ﺍﺤﺴﺏ ﺍﻨﺩﻓﺎﻉ ﻜل ﻤﻨﻬﻤﺎ‪.‬‬

‫‪ -11‬ﻴﻁﻠﺏ ﺍﻟﺘﻌﺒﻴﺭ ﻋﻥ ﺴﺭﻋﺔ ﺠﺴﻴﻡ ﺒﺩﻻﻟﺔ ﺍﻨﺩﻓﺎﻋﻪ ﻭﻜﺘﻠﺘﻪ ﺍﻟﺴﺎﻜﻨﺔ‪.‬‬

‫‪- 319 -‬‬

‫)‪Create PDF files without this message by purchasing novaPDF printer (http://www.novapdf.com‬‬
‫‪ ‬‬
‫‪ ‬‬
‫‪ACCELERATED COORDINATE SYSTEMS‬‬

‫ﺃﻧﻈﻤﺔ ﺍﶈﺎﻭﺭ ﻏﲑ ﺍﻟﻘﺼﻮﺭﻳﺔ ) ﺍﻟﻼﻋﻄﺎﻟﻴﺔ (‪:‬‬


‫‪:Non Inertial Axis Systems‬‬
‫ﻫﻲ ﺃﻨﻅﻤﺔ ﺍﻟﻤﺤﺎﻭﺭ ﺍﻹﺤﺩﺍﺜﻴﺔ ﺍﻟﺘﻲ ﺘﺘﺤﺭﻙ ﺤﺭﻜﺔ ﻤﺴﺘﻘﻴﻤﺔ ﻤﻨﺘﻅﻤﺔ ﺒﺎﻟﻨﺴـﺒﺔ‬
‫ﻟﻠﻔﺭﺍﻍ‪ .‬ﻋﻠﻰ ﺴﺒﻴل ﺍﻟﻤﺜﺎل‪ ،‬ﻤﺠﻭﻋﺔ ﺍﻹﺤـﺩﺍﺜﻴﺎﺕ ﺍﻟﻤﺜﺒﺘـﺔ ﻋﻠـﻰ ﺍﻷﺭﺽ ﻟﻴﺴـﺕ‬
‫ﻋﻁﺎﻟﻴﺔ ﻷﻥ ﺍﻷﺭﺽ ﺘﺩﻭﺭ ﺤﻭل ﻨﻔﺴﻬﺎ ﻭﺤﻭل ﺍﻟﺸﻤﺱ‪ .‬ﻭﺒﻨـﺎﺀ ﻋﻠـﻰ ﺫﻟـﻙ ﺇﺫﺍ‬
‫ﺍﺴﺘﺨﺩﻤﻨﺎ ﻤﺠﻤﻭﻋﺔ ﻤﺤﺎﻭﺭ ﻟﻭﺼﻑ ﺤﺭﻜﺔ ﺠﺴﻡ ﺒﺎﻟﻨﺴﺒﺔ ﺇﻟﻰ ﺍﻷﺭﺽ ﻓﺈﻨﻨﺎ ﺴـﻭﻑ‬
‫ﻨﺤﺼل ﻋﻠﻰ ﻨﺘﺎﺌﺞ ﻏﻴﺭ ﺩﻗﻴﻘﺔ‪ .‬ﻟﻬﺫﺍ ﺍﻟﺴﺒﺏ ﺴﻨﺭﻯ ﻓﻲ ﻫﺫﺍ ﺍﻟﻔﺼل ﻜﻴﻑ ﺘﺘﻡ ﺤﺭﻜﺔ‬
‫ﺍﻷﺠﺴﺎﻡ ﺒﺎﻟﻨﺴﺒﺔ ﻟﻤﺠﻤﻭﻋﺔ ﻤﺤﺎﻭﺭ ﻤﺘﺤﺭﻜﺔ‪.‬‬
‫ﺃﻧﻈﻤﺔ ﺍﶈﺎﻭﺭ ﺍﻟﺪﻭﺍﺭﺓ ‪:Revolving Axis Systems‬‬
‫ﺒﻔﺭﺽ ‪ XYZ‬ﻤﺠﻤﻭﻋﺔ ﻤﺤﺎﻭﺭ ﺜﺎﺒﺘﺔ )ﻋﻁﺎﻟﻴﺔ( ﻓﻲ ﺍﻟﻔﻀﺎﺀ‪ .‬ﻭﺒﻔـﺭﺽ ﺃﻥ‬
‫ﻤﺠﻤﻭﻋﺔ ﺍﻟﻤﺤﺎﻭﺭ ‪ xyz‬ﻟﻬﺎ ﻨﻔﺱ ﻨﻘﻁﺔ ﺍﻷﺼل ‪ .o‬ﻭﻟﻜﻨﻬﺎ ﺘﺩﻭﺭ ﺒﺴﺭﻋﺔ ﺯﺍﻭﻴﺔ ‪ω‬‬
‫ﺒﺎﻟﻨﺴﺒﺔ ﺇﻟﻰ ﺍﻟﻤﺠﻤﻭﻋﺔ ﺍﻟﺜﺎﺒﺘﺔ‪ .XYZ‬ﻨﻌﺘﺒﺭ ﺃﻥ ﺠﺴـﻡ ﻴﺘﺤـﺭﻙ ﻓـﻲ ﻤﺠﻤﻭﻋـﺔ‬
‫ﺍﻟﻤﺤﺎﻭﺭ ‪ xyz‬ﺍﻟﻤﺘﺤﺭﻜﺔ‪.‬‬

‫ﺍﻟﺸﻜل )‪ :(1‬ﺍﻟﻤﺤﺎﻭﺭ ﺍﻹﺤﺩﺍﺜﻴﺔ ﺍﻟﺩﻭﺍﺭﺓ‬

‫‪- 320 -‬‬

‫)‪Create PDF files without this message by purchasing novaPDF printer (http://www.novapdf.com‬‬
‫ﻴﻁﻠﺏ ﺇﻴﺠﺎﺩ ﺴﺭﻋﺔ ﻭﺘﺴﺎﺭﻉ ﺍﻟﺠﺴﻡ ﺒﺎﻟﻨﺴﺒﺔ ﻟﻜلٍ ﻤﻥ ﻤﺠﻤﻭﻋـﺔ ﺍﻟﻤﺤـﺎﻭﺭ‬
‫ﺍﻟﻤﺘﺤﺭﻜﺔ ‪) xyz‬ﺍﻟﻼﻋﻁﺎﻟﻴﺔ( ﻭﺍﻟﻤﺜﺒﺘﺔ ‪) XYZ‬ﺍﻟﻌﻁﺎﻟﻴﺔ(‪.‬‬
‫ﺍﻟﺴﺮﻋﺔ ﻭﺍﻟﺘﺴﺎﺭﻉ ﺑﺎﻟﻨﺴﺒﺔ ﺇﱃ ﻧﻈﺎﻡ ﺇﺣﺪﺍﺛﻴﺎﺕ ﻣﺘﺤ ﺮﻙ‬
‫‪Velocity and Acceleration in Moving System Coordinates‬‬
‫‪‬‬
‫ﺇﺫﺍ ﻜﺎﻥ ‪ r‬ﻤﺘﺠﻪ ﻤﻭﻀﻊ ﺍﻟﺠﺴﻡ ﺍﻟﻤﺘﺤﺭﻙ‪ ،‬ﻓﺈﻨﻪ ﻴﺘﻌﻴﻥ ﻓﻲ ﺍﻟﻤﺠﻤﻭﻋـﺔ ‪xyz‬‬
‫ﺒﺎﻟﻌﻼﻗﺔ ﺍﻟﺘﺎﻟﻴﺔ‪:‬‬
‫‪‬‬ ‫‪‬‬ ‫‪‬‬ ‫‪‬‬
‫‪r  x i  y j  zk‬‬ ‫)‪(1‬‬
‫ﻭﺘﻌﻁﻰ ﺴﺭﻋﺔ ﺍﻟﺠﺴﻡ ﺒﺎﻟﻨﺴﺒﺔ ﺇﻟﻰ ﻤﺸـﺎﻫﺩ ﻴﻘـﻑ ﻓـﻲ ﺍﻟﻤﺠﻤﻭﻋـﺔ ‪xyz‬‬
‫ﺒﺎﻟﻤﻌﺎﺩﻟﺔ‪:‬‬
‫‪ dr‬‬ ‫‪dx  dy  dz ‬‬ ‫‪ ‬‬ ‫‪‬‬
‫‪V‬‬ ‫‪‬‬ ‫‪i‬‬ ‫‪j  k  x i  y j  zk‬‬ ‫)‪(2‬‬
‫‪dt‬‬ ‫‪M‬‬ ‫‪dt‬‬ ‫‪dt‬‬ ‫‪dt‬‬
‫ﻴﺩل ﺍﻟﺭﻤﺯ‪ M‬ﻋﻠﻰ ﺃﻥ ﺍﻻﺸﺘﻘﺎﻕ ﻴﺘﻡ ﻓﻲ ﺍﻟﻤﺠﻤﻭﻋﺔ ﺍﻟﻤﺘﺤﺭﻜﺔ‪.‬‬
‫ﺘﺴﻤﻰ ﺍﻟﺴﺭﻋﺔ ﻓﻲ ﻫﺫﻩ ﺍﻟﻤﺠﻤﻭﻋﺔ ﺒﺎﻟﺴﺭﻋﺔ ﺍﻟﻅﺎﻫﺭﻴﺔ‪ ،‬ﻭﻴـﺘﻡ ﺍﻟﺤﺼـﻭل‬
‫ﻋﻠﻰ ﺍﻟﺘﺴﺎﺭﻉ ﻓﻲ ﻫﺫﻩ ﺍﻟﻤﺠﻤﻭﻋﺔ ﺒﺎﺸﺘﻘﺎﻕ ﺍﻟﺴﺭﻋﺔ ﻓﻲ ﺍﻟﻤﻌﺎﺩﻟﺔ )‪ (2‬ﺒﺎﻟﻨﺴﺒﺔ ﻟﻠﺯﻤﻥ‬
‫ﺒﺎﻟﺼﻭﺭﺓ‪:‬‬
‫‪‬‬
‫‪ dV‬‬ ‫‪dx  dy  dz ‬‬ ‫‪  ‬‬
‫‪a‬‬ ‫‪‬‬ ‫‪i‬‬ ‫‪j  k  x i  yj  zk‬‬ ‫)‪(3‬‬
‫‪dt‬‬ ‫‪M‬‬
‫‪dt‬‬ ‫‪dt‬‬ ‫‪dt‬‬

‫ﻴﺴﻤﻰ ﻫﺫﺍ ﺍﻟﺘﺴﺎﺭﻉ ﺒﺎﻟﺘﺴﺎﺭﻉ ﺍﻟﻅﺎﻫﺭﻱ‪.‬‬


‫ﺍﻟﺴﺮﻋﺔ ﻭﺍﻟﺘﺴﺎﺭﻉ ﰲ ﻧﻈﺎﻡ ﺇﺣﺪﺍﺛﻴﺎﺕ ﻣﺜﺒﺖ ‪:‬‬
‫‪Velocity in Fixed System Coordinates:‬‬
‫ﺃﻭﻻﹰ ﺍﻟﺴﺭﻋﺔ ﻓﻲ ﻤﺠﻤﻭﻋﺔ ﻤﺤﺎﻭﺭ ﺇﺤﺩﺍﺜﻴﺔ ﻤﺜﺒﺘﺔ‪:‬‬
‫ﻹﻴﺠﺎﺩ ﺍﻟﺴﺭﻋﺔ ﻭﺍﻟﺘﺴﺎﺭﻉ ﻟﺤﺭﻜﺔ ﺠﺴﻡ ﺒﺎﻟﻨﺴﺒﺔ ﺇﻟﻰ ﻤﺠﻤﻭﻋﺔ ﻤﺤﺎﻭﺭ ﺇﺤﺩﺍﺜﻴـﺔ‬
‫‪‬‬
‫ﻤﺜﺒﺘﺔ ‪ XYZ‬ﻴﺠﺏ ﻤﻼﺤﻅﺔ ﺃﻨﻪ ﻟﻴﺱ ﻤﺘﺠﻪ ﺍﻟﻤﻭﻀﻊ ‪ r‬ﻭﺤﺩﻩ ﻫﻭ ﺍﻟﻤﺘﻐﻴﺭ ﺒل ﺘﺘﻐﻴﺭ‬
‫‪  ‬‬
‫ﻤﺘﺠﻬﺎﺕ ﺍﻟﻭﺤﺩﺓ ‪ k , j , i‬ﻨﻔﺴﻬﺎ ﻷﻨﻬﺎ ﻤﺤﻤﻭﻟﺔ ﻋﻠﻰ ﻤﺤﺎﻭﺭ‪ z ،y،x‬ﺘﺩﻭﺭ )ﺘﺘﻐﻴـﺭ‬
‫ﻤﻊ ﺍﻟﺯﻤﻥ( ﺒﺎﻟﻨﺴﺒﺔ ﺇﻟﻰ ﺍﻟﻤﺠﻤﻭﻋﺔ ﺍﻟﻤﺜﺒﺘﺔ‪.‬‬
‫ﻴﻤﻜﻥ ﺍﻟﺤﺼﻭل ﻋﻠﻰ ﻤﺘﺠﻪ ﺴﺭﻋﺔ ﺍﻟﺠﺴﻡ ﺍﻟﻤﺘﺤﺭﻙ ﺒﺎﻟﻨﺴﺒﺔ ﺇﻟﻰ ﺍﻟﻤﺠﻤﻭﻋﺔ ﺍﻟﻤﺜﺒﺘﺔ‪،‬‬
‫ﻤــﻥ ﺍﺸــﺘﻘﺎﻕ ﺍﻟﻌﻼﻗــﺔ )‪ ،(1‬ﺒﺎﻟﻨﺴــﺒﺔ ﻟﻠــﺯﻤﻥ ﻭﻓــﻕ ﺍﻟﻤﻌﺎﺩﻟــﺔ‪:‬‬

‫‪- 321 -‬‬

‫)‪Create PDF files without this message by purchasing novaPDF printer (http://www.novapdf.com‬‬
‫‪‬‬ ‫‪‬‬ ‫‪‬‬ ‫‪‬‬
‫‪‬‬ ‫‪dr‬‬ ‫‪dx  dy  dz ‬‬ ‫‪di‬‬ ‫‪dj‬‬ ‫‪dk‬‬
‫‪V‬‬ ‫‪F‬‬
‫‪‬‬ ‫‪F‬‬
‫‪‬‬ ‫‪i ‬‬ ‫‪j‬‬ ‫‪k  x‬‬ ‫‪ y‬‬ ‫‪ z‬‬
‫‪dt‬‬ ‫‪dt‬‬ ‫‪dt‬‬ ‫‪dt‬‬ ‫‪dt‬‬ ‫‪dt‬‬ ‫‪dt‬‬
‫‪‬‬ ‫‪‬‬ ‫‪‬‬ ‫‪‬‬ ‫‪‬‬ ‫‪‬‬ ‫‪‬‬
‫‪dr‬‬ ‫‪di‬‬ ‫‪dj‬‬ ‫‪dk‬‬ ‫‪‬‬ ‫‪di‬‬ ‫‪dj‬‬ ‫‪dk‬‬
‫‪‬‬ ‫‪ x‬‬ ‫‪ y‬‬ ‫‪ z‬‬ ‫‪ V‬‬ ‫‪ x‬‬ ‫‪ y‬‬ ‫‪ z‬‬
‫‪dt‬‬ ‫‪M‬‬ ‫‪dt‬‬ ‫‪dt‬‬ ‫‪dt‬‬ ‫‪M‬‬ ‫‪dt‬‬ ‫‪dt‬‬ ‫‪dt‬‬
‫)‪(4‬‬
‫ﻴﺩل ﺍﻟﺭﻤﺯ‪ F‬ﻋﻠﻰ ﺃﻥ ﺍﻻﺸﺘﻘﺎﻕ ﻴﺘﻡ ﺒﺎﻟﻨﺴﺒﺔ ﻟﻠﻤﺠﻤﻭﻋـﺔ ﺍﻟﻤﺜﺒﺘـﺔ‪ .‬ﻭﺍﻵﻥ ﻨﺤﺴـﺏ‬
‫‪  ‬‬
‫ﻤﺸﺘﻘﺎﺕ ﻤﺘﺠﻬﺎﺕ ﺍﻟﻭﺤﺩﺓ ‪ k , j , i‬ﺒﺎﻟﻨﺴﺒﺔ ﻟﻠﺯﻤﻥ ﻭﻨﻌﻭﺽ ﻓﻲ ﺍﻟﻤﻌﺎﺩﻟﺔ )‪ .(4‬ﺒﻤـﺎ‬
‫‪  ‬‬
‫ﺃﻥ ‪ k , j , i‬ﻤﺘﺠﻬﺎﺕ ﻭﺤﺩﺓ ﻤﺘﻌﺎﻤﺩﺓ ﻓﺈﻥ ﻤﺸﺘﻕ ﻜل ﻤﻨﻬﺎ ﻫـﻭ ﻤﺘﺠـﻪ ﻭﺍﻗـﻊ ﻓـﻲ‬
‫‪‬‬
‫‪‬‬
‫ﻫﻭ ﻤﺘﺠﻪ ﻤﺘﻌﺎﻤﺩ ﻤﻊ ﺍﻟﻤﺘﺠـﻪ ‪i‬‬ ‫ﻤﺴﺘﻭﻯ ﺍﻟﻤﺘﺠﻬﻴﻥ ﺍﻵﺨﺭﻴﻥ‪ .‬ﻤﺜﻼ ﺇﻥ ﺍﻟﻤﺸﺘﻕ‬
‫‪di‬‬
‫‪dt‬‬
‫‪ ‬‬
‫ﻭﺒﺎﻟﺘﺎﻟﻲ ﻴﻘﻊ ﻓﻲ ﻤﺴﺘﻭﻯ ﺍﻟﻤﺘﺠﻬﻴﻥ ‪ k , j‬ﻭﻟﻬﺫﺍ ﻴﻤﻜﻥ ﺃﻥ ﻴﻜﺘﺏ ﺒﺎﻟﺼﻭﺭﺓ‪:‬‬
‫‪‬‬
‫‪di‬‬ ‫‪‬‬ ‫‪‬‬
‫‪ a1 j  a 2 k‬‬ ‫)‪(5‬‬
‫‪dt‬‬
‫ﻭﺒﻨﻔﺱ ﺍﻟﺸﻜل ﻴﻤﻜﻥ ﻜﺘﺎﺒﺔ ﺍﻟﻤﺘﺠﻬﻴﻥ ﺍﻵﺨﺭﻴﻥ‪ ،‬ﺤﻴﺙ ﻨﺠـﺩ‪:‬‬
‫‪‬‬
‫‪dj‬‬ ‫‪‬‬ ‫‪‬‬
‫‪ a3k  a 4 i‬‬ ‫)‪(6‬‬
‫‪dt‬‬
‫‪‬‬
‫‪dk‬‬ ‫‪‬‬ ‫‪‬‬
‫‪ a5i  a6 j‬‬ ‫)‪(7‬‬
‫‪dt‬‬
‫ﺒﺴﻬﻭﻟﺔ‪.‬‬ ‫ﺒﺒﻌﻀﻬﺎ ﺒﻌﻼﻗﺎﺕ ﺒﺴﻴﻁﺔ ﻴﻤﻜﻥ ﺇﻴﺠﺎﺩﻫﺎ‬ ‫‪a 1 , a 2 ,....., a 6‬‬ ‫ﺘﺭﺘﺒﻁ ﺍﻟﻤﺭﻜﺒﺎﺕ‬
‫‪‬‬
‫ﻨﻌﻠﻡ ﺃﻥ‪ ، i . j  0 :‬ﺒﺎﺸﺘﻘﺎﻕ ﻫﺫﻩ ﺒﺎﻟﻨﺴﺒﺔ ﻟﻠﺯﻤﻥ‪ ،‬ﻴﻨﺘﺞ ﺃﻥ‪:‬‬
‫‪‬‬ ‫‪‬‬
‫‪ dj  di‬‬
‫‪i j‬‬ ‫‪0‬‬ ‫)‪(8‬‬
‫‪dt‬‬ ‫‪dt‬‬
‫ﺒﻀﺭﺏ ﺍﻟﻌﻼﻗﺔ )‪ (5‬ﺴﻠﻤﻴﺎﹰ ﺒـ ‪ j‬ﻭﺍﻟﻌﻼﻗﺔ )‪ (6‬ﺒـ ‪ i‬ﻴﻨﺘﺞ ﺃﻥ‪:‬‬
‫‪‬‬
‫‪ dj‬‬
‫‪i  a4‬‬
‫‪dt‬‬ ‫)‪(9‬‬
‫‪‬‬
‫‪ di‬‬
‫‪j‬‬ ‫‪ a1‬‬
‫‪dt‬‬
‫ﻭﺒﻨﻔﺱ ﺍﻟﻁﺭﻴﻘﺔ ﻴﻤﻜﻥ ﺍﻟﺤﺼـﻭل ﻋﻠـﻰ‬ ‫ﻭﻤﻥ )‪ (8‬ﻭ )‪ (9‬ﻴﻨﺘﺞ ﺃﻥ ‪a4=-a1 :‬‬
‫ﺃﻥ‪ a5=- a2 :‬ﻭ‪ .a6=-a3‬ﻭﻋﻨﺩﺌﺫ ﺘﺼﺒﺢ ﺍﻟﻌﻼﻗﺎﺕ‪ (7)، (6) ، (5) :‬ﺒﺎﻟﺼﻭﺭﺓ‪:‬‬

‫‪- 322 -‬‬

‫)‪Create PDF files without this message by purchasing novaPDF printer (http://www.novapdf.com‬‬
‫‪‬‬
‫‪di‬‬ ‫‪‬‬ ‫‪‬‬
‫‪ a1 j  a 2k‬‬ ‫)‪(10‬‬
‫‪dt‬‬
‫‪‬‬
‫‪dj‬‬ ‫‪‬‬ ‫‪‬‬
‫‪ a3k  a1i‬‬ ‫)‪(11‬‬
‫‪dt‬‬
‫‪‬‬
‫‪dk‬‬ ‫‪‬‬ ‫‪‬‬
‫‪  a2i  a3 j‬‬ ‫)‪(12‬‬
‫‪dt‬‬
‫ﺜﻼﺙ ﻤﻌﺎﺩﻻﺕ ﺒﺜﻼﺜﺔ ﻤﺠﺎﻫﻴل ‪ ،a1،a2،a3‬ﻴﻤﻜﻥ ﺇﻴﺠﺎﺩﻫﺎ ﻤـﻥ ﺤـل ﺍﻟﻤﻌـﺎﺩﻻﺕ‬
‫ﺍﻟﺜﻼﺙ ﺍﻟﺴﺎﺒﻘﺔ ﻭﺫﻟﻙ ﺒﻀﺭﺏ ﻁﺭﻓﻲ ﺍﻟﻤﻌﺎﺩﻟﺔ )‪ (10‬ﺒـ ‪ x‬ﻭﺍﻟﻤﻌﺎﺩﻟﺔ )‪ (11‬ﺒـ ‪y‬‬
‫ﻭﺍﻟﻤﻌﺎﺩﻟﺔ )‪ (12‬ﺒـ ‪ z‬ﻭﺠﻤﻊ ﺍﻟﻤﻌﺎﺩﻻﺕ ﺍﻟﻨﺎﺘﺠﺔ‪ ،‬ﻴﻜﻭﻥ‪:‬‬
‫‪‬‬ ‫‪‬‬ ‫‪‬‬
‫‪di‬‬ ‫‪dj‬‬ ‫‪dk‬‬ ‫‪‬‬ ‫‪‬‬ ‫‪‬‬
‫‪x y z‬‬ ‫‪  a 1 y  a 2 z i  a 1x  a 3 z  j  a 2 x  a 3 y k‬‬
‫‪dt‬‬ ‫‪dt‬‬ ‫‪dt‬‬
‫)‪(13‬‬
‫ﻴﻤﻜﻥ ﻜﺘﺎﺒﺔ ﻫﺫﻩ ﺍﻟﻤﻌﺎﺩﻟﺔ ﺒﺎﻟﺼﻭﺭﺓ‪:‬‬
‫‪‬‬ ‫‪‬‬ ‫‪‬‬
‫‪di‬‬ ‫‪dj‬‬ ‫‪dk‬‬ ‫‪‬‬
‫‪x‬‬ ‫‪ y‬‬ ‫‪z‬‬ ‫‪  a 1 y  a 2 z i‬‬
‫‪dt‬‬ ‫‪dt‬‬ ‫‪dt‬‬
‫‪‬‬ ‫‪‬‬
‫‪ a 1 x  a 3 z  j  a 2 x  a 3 y k‬‬
‫‪‬‬ ‫‪‬‬ ‫‪‬‬ ‫‪‬‬ ‫‪‬‬ ‫‪‬‬
‫‪i‬‬ ‫‪j‬‬ ‫‪k‬‬ ‫‪i‬‬ ‫‪j‬‬ ‫‪k‬‬
‫‪ ‬‬
‫‪ a3  a 2 a1  ω1 ω2 ω3  ωΧ r‬‬
‫‪x‬‬ ‫‪y‬‬ ‫‪z‬‬ ‫‪x‬‬ ‫‪y‬‬ ‫‪z‬‬
‫)‪(14‬‬
‫‪‬‬
‫ﻴﺴﻤﻰ ﺍﻟﻤﺘﺠﻪ ‪ ‬ﺒﻤﺘﺠﻪ ﺍﻟﺴﺭﻋﺔ ﺍﻟﺯﺍﻭﻴﺔ ﻟﻠﻤﺠﻤﻭﻋﺔ ﺍﻟﻤﺘﺤﺭﻜﺔ ﺒﺎﻟﻨﺴﺒﺔ ﻟﻠﻤﺠﻤﻭﻋـﺔ‬
‫ﺍﻟﺜﺎﺒﺘﺔ‪.‬‬
‫‪‬‬ ‫‪‬‬ ‫‪‬‬ ‫‪‬‬
‫‪ω  ω1 i  ω2 j  ω3k‬‬ ‫ﺤﻴﺙ‪:‬‬
‫‪ω1  a 3, ω 2  a 2, ω 3  a1‬‬ ‫ﻭﺒﺎﻋﺘﺒﺎﺭ ﺃﻥ‪:‬‬
‫ﺒﺎﻟﺘﻌﻭﻴﺽ ﻤﻥ ﺍﻟﻤﻌﺎﺩﻟﺔ)‪ (14‬ﻓﻲ ﺍﻟﻤﻌﺎﺩﻟﺔ )‪ (4‬ﻨﺤﺼل ﺃﺨﻴﺭﺍ ﻋﻠﻰ ﻤﺘﺠﻪ ﺍﻟﺴـﺭﻋﺔ‬
‫ﻟﻠﺠﺴﻡ ﺍﻟﻤﺘﺤﺭﻙ ﺒﺎﻟﻨﺴﺒﺔ ﺇﻟﻰ ﺍﻟﻤﺠﻤﻭﻋﺔ ﺍﻟﻤﺜﺒﺘﺔ ﺒﺸﻜﻠﻪ ﺍﻟﻨﻬﺎﺌﻲ ﺒﺎﻟﺼﻭﺭﺓ‪:‬‬
‫‪‬‬
‫‪‬‬ ‫‪dr‬‬ ‫‪ ‬‬
‫‪vF ‬‬ ‫‪ ωxr‬‬ ‫)‪(15‬‬
‫‪dt‬‬ ‫‪M‬‬
‫‪ ‬‬
‫ﻴﻤﺜل ﺍﻟﻤﺘﺠﻪ ‪ ωΧ r‬ﺍﻟﺴﺭﻋﺔ ﺍﻟﻠﺤﻅﻴﺔ ﻟﻨﻘﻁﺔ ﺜﺎﺒﺘﺔ ﻓﻲ ﺍﻟﻤﺠﻤﻭﻋﺔ ﺍﻟﻤﺘﺤﺭﻜﺔ ﻭﺘﻨﻁﺒﻕ‬

‫‪- 323 -‬‬

‫)‪Create PDF files without this message by purchasing novaPDF printer (http://www.novapdf.com‬‬
‫ﻓﻲ ﺍﻟﻠﺤﻅﺔ ‪ t‬ﻋﻠﻰ ﺍﻟﻨﻘﻁﺔ ﺍﻟﻤﺘﺤﺭﻜﺔ ‪ P‬ﻭﺘﺘﺤﺭﻙ ﻋﻠﻰ ﻤﺤﻴﻁ ﺩﺍﺌﺭﺓ ﻨﺼﻑ ﻗﻁﺭﻫـﺎ‬
‫‪ .r‬ﻭﻨﺴﻤﻴﻬﺎ ﻟﺫﻟﻙ ﺒﺴﺭﻋﺔ ﻤﻭﻀﻊ ﺍﻟﻨﻘﻁـﺔ ﺍﻟﻤﺘﺤﺭﻜـﺔ ﻓـﻲ ﺍﻟﻠﺤﻅـﺔ ‪ t‬ﺒﺎﻟﻨﺴـﺒﺔ‬
‫ﻟﻠﻤﺠﻤﻭﻋﺔ ﺍﻟﻤﺜﺒﺘﺔ‪.،‬ﻴﻤﻜﻨﻨﺎ ﺃﻥ ﻨﺴﺘﻨﺘﺞ ﺃﻥ ﺴﺭﻋﺔ ﺍﻟﺠﺴﻡ ﺒﺎﻟﻨﺴﺒﺔ ﻟﻤﺠﻤﻭﻋﺔ ﻤﺤـﺎﻭﺭ‬
‫ﻤﺜﺒﺘﺔ ﺘﺴﺎﻭﻱ ﺴﺭﻋﺔ ﺍﻟﺠﺴﻡ ﺒﺎﻟﻨﺴﺒﺔ ﻟﻤﺠﻤﻭﻋﺔ ﻤﺤﺎﻭﺭ ﺘﺩﻭﺭ ﺒﺎﻟﻨﺴـﺒﺔ ﻟﻠﻤﺠﻤﻭﻋـﺔ‬
‫ﺍﻟﻤﺜﺒﺘﺔ ﻤﻀﺎﻓﺎ ﺇﻟﻴﻬﺎ ﺍﻟﻀﺭﺏ ﺃﻻﺘﺠﺎﻫﻲ ﻟﻤﺘﺠﻪ ﺍﻟﺴﺭﻋﺔ ﺍﻟﺯﺍﻭﻴﺔ ﻟﺩﻭﺭﺍﻥ ﺍﻟﻤﺠﻤﻭﻋﺔ‬
‫‪‬‬ ‫‪‬‬
‫ﻟﻠﺠﺴﻡ ﺍﻟﻤﺘﺤﺭﻙ‪ .‬ﻭﺘﺴﻤﻰ ﺍﻟﺴﺭﻋﺔ ﻓـﻲ ﻫـﺫﻩ‬ ‫ﺍﻟﻤﺘﺤﺭﻜﺔ ‪ ω‬ﻓﻲ ﻤﺘﺠﻪ ﺍﻟﻤﻭﻀﻊ ‪r‬‬
‫ﺍﻟﻤﺠﻤﻭﻋﺔ ﺒﺎﻟﺴﺭﻋﺔ ﺍﻟﺤﻘﻴﻘﻴﺔ‪.‬‬
‫ﺛﺎﻧﻴﺎ ً ﺍﻟﺘﺴﺎﺭﻉ ﰲ ﻧﻈﺎﻡ ﺇﺣﺪﺍﺛﻴﺎﺕ ﻣﺜﺒﺖ ‪:‬‬
‫‪Acceleration in Fixed System Coordinates:‬‬
‫ﻴﻤﻜﻥ ﺇﻴﺠﺎﺩ ﺍﻟﺘﺴﺎﺭﻉ ﻓﻲ ﻤﺠﻤﻭﻋﺔ ﻤﺤﺎﻭﺭ ﺇﺤﺩﺍﺜﻴﺔ ﻤﺜﺒﺘﺔ ﺒـﻨﻔﺱ ﺍﻟﻁﺭﻴﻘـﺔ ﺍﻟﺘـﻲ‬
‫ﻭﺠﺩﻨﺎ ﺒﻬﺎ ﺍﻟﺴﺭﻋﺔ ﻓﻲ ﺍﻟﻔﻘﺭﺓ ﺍﻟﺴﺎﺒﻘﺔ‪ ،‬ﺤﻴﺙ ﻨﺠﺩ‪:‬‬
‫‪‬‬ ‫‪2‬‬ ‫‪‬‬ ‫‪‬‬
‫‪ d r‬‬ ‫‪d dr‬‬ ‫‪d  dr‬‬ ‫‪ ‬‬
‫‪a 2‬‬ ‫‪‬‬ ‫‪‬‬ ‫‪‬‬
‫‪‬‬ ‫‪ ωΧ r  ‬‬
‫‪d t‬‬ ‫‪F‬‬
‫‪dt F dt F dt F  dt M‬‬ ‫‪‬‬
‫‪‬‬ ‫‪‬‬
‫‪d  dr‬‬ ‫‪     dr‬‬ ‫‪ ‬‬
‫‪‬‬ ‫‪ ωΧ r   ωΧ ‬‬ ‫‪ ωΧ r ‬‬
‫‪dt M  dt M‬‬ ‫‪‬‬ ‫‪ dt M‬‬ ‫‪‬‬

‫‪‬‬
‫‪‬‬ ‫‪d2 r‬‬ ‫‪      ‬‬
‫‪aF  2‬‬ ‫‪ 2ωΧ r  ω Χ r  ωΧ ωΧ r ‬‬ ‫)‪(16‬‬
‫‪dt‬‬ ‫‪M‬‬
‫‪‬‬
‫‪ d2 r‬‬
‫‪ a  2‬ﺒﺎﻟﺘﺴﺎﺭﻉ ﺍﻟﺤﻘﻴﻘﻲ ﻭﻫﻭ ﻤﺘﺠﻪ ﻤﻭﻀـﻊ ﺍﻟﻨﻘﻁـﺔ ‪P‬‬ ‫ﻴﺴﻤﻰ ﺍﻟﺘﺴﺎﺭﻉ‬
‫‪dt‬‬ ‫‪F‬‬

‫ﺒﺎﻟﻨﺴﺒﺔ ﻟﻠﻤﺠﻤﻭﻋﺔ ﺍﻟﻤﺜﺒﺘﺔ‪.‬‬


‫‪‬‬
‫‪ dV‬‬
‫‪ a ‬ﺒﺎﻟﺘﺴﺎﺭﻉ ﺍﻟﻅﺎﻫﺭﻱ ﻟﻠﻨﻘﻁﺔ ‪ P‬ﻭﻫﻭ ﺘﺴﺎﺭﻋﻬﺎ ﻓﻲ‬ ‫‪M‬‬
‫ﻭ ﻴﺴﻤﻰ ﺍﻟﺘﺴﺎﺭﻉ‬
‫‪dt‬‬
‫ﺍﻟﻤﺠﻤﻭﻋﺔ ﺍﻟﻤﺘﺤﺭﻜﺔ‪ .‬ﻭﻤﺠﻤﻭﻉ ﻗﻴﻡ ﺤﺩﻭﺩ ﺍﻟﻁﺭﻑ ﺍﻷﻴﻤﻥ ﻟﻠﻌﻼﻗـﺔ )‪ (16‬ﻴﺸـﻜل‬
‫ﺘﺴﺎﺭﻉ ﺍﻟﻤﺠﻭﻋﺔ ﺍﻟﻤﺘﺤﺭﻜﺔ ﺒﺎﻟﻨﺴﺒﺔ ﻟﻠﻤﺠﻤﻭﻋﺔ ﺍﻟﻤﺜﺒﺘﺔ‪ .‬ﺤﻴﺙ‪:‬‬
‫‪‬‬ ‫‪‬‬
‫‪ ω Χ r‬ﺍﻟﺘﺴﺎﺭﻉ ﺍﻟﺨﻁﻲ ﻟﻠﻨﻘﻁﺔ ﺍﻟﺜﺎﺒﺘﺔ ﻓﻲ ﺍﻟﻤﺠﻤﻭﻋـﺔ ﺍﻟﻤﺘﺤﺭﻜـﺔ )ﺍﻟﺘﺴـﺎﺭﻉ‬
‫ﺍﻟﺨﻁﻲ ﻟﻤﻭﻀﻊ ﺍﻟﻤﺘﺤﺭﻙ(‬

‫‪- 324 -‬‬

‫)‪Create PDF files without this message by purchasing novaPDF printer (http://www.novapdf.com‬‬
‫‪‬‬ ‫‪‬‬ ‫‪‬‬
‫‪ ω Χ ω Χ r ‬ﺍﻟﺘﺴﺎﺭﻉ ﺍﻟﺠﺎﺫﺏ ﻟﻠﻨﻘﻁﺔ ﺍﻟﺜﺎﺒﺘﺔ ﻓﻲ ﺍﻟﻤﺠﻤﻭﻋﺔ ﺍﻟﻤﺜﺒﺘـﺔ )ﺍﻟﺘﺴـﺎﺭﻉ‬
‫ﺍﻟﺠﺎﺫﺏ ﻟﻤﻭﻀﻊ ﺍﻟﻤﺘﺤﺭﻙ(‬
‫‪‬‬ ‫‪‬‬
‫‪ 2 ω Χ r‬ﺍﻟﺘﺴﺎﺭﻉ ﺍﻟﻤﺘﻤﻡ )ﺘﺴﺎﺭﻉ ﻜﻭﺭﻴﻭ ﻟﻴﺱ( ﻟﻠﻨﻘﻁﺔ ﻤﻊ ﺍﻟﻤﺠﻤﻭﻋﺔ ﺍﻟﻤﺜﺒﺘـﺔ‬
‫ﻭﺍﻟﺘﻲ ﺘﻨﻁﺒﻕ ﻓﻲ ﺍﻟﻠﺤﻅﺔ ‪ t‬ﻋﻠﻰ ﺍﻟﻨﻘﻁﺔ ﺍﻟﻤﺘﺤﺭﻜﺔ‪.‬‬
‫ﺃﻧﻈﻤﺔ ﺍﶈﺎﻭﺭ ﺍﳌﺘﺤﺮﻛﺔ ﰲ ﺍﳊﺎﻟﺔ ﺍﻟﻌﺎﻣﺔ ‪:‬‬
‫‪Axis Systems Moving in General:‬‬
‫ﺍﻓﺘﺭﻀﺘﺎ ﻓﻲ ﺍﻟﻔﻘﺭﺓ ﺍﻟﺴﺎﺒﻘﺔ ﺃﻥ ﻤﺠﻤﻭﻋﺘﻲ ﺍﻟﻤﺤﺎﻭﺭ ‪ xyz‬ﻭ ‪ XYZ‬ﻟﻬﻤﺎ ﻨﻔـﺱ‬
‫‪‬‬
‫ﻨﻘﻁﺔ ﺍﻷﺼل ‪ .O‬ﻨﻔﺭﺽ ﻓﻲ ﺤﺎﻟﺔ ﻋﺩﻡ ﻭﺠﻭﺩ ﻨﻘﻁﺔ ﺃﺼل ﻤﺸـﺘﺭﻜﺔ‪ ،‬ﺃﻥ ‪ R‬ﻫـﻭ‬
‫ﻤﺘﺠﻪ ﻤﻭﻀﻊ ﺍﻟﻨﻘﻁﺔ ‪ O/‬ﺃﺼل ﺍﻟﻤﺠﻭﻋﺔ ‪، xyz‬ﺍﻟﺸﻜل )‪.(2‬‬

‫ﺍﻟﺸﻜل )‪ :(2‬ﺍﻟﻤﺤﺎﻭﺭ ﺍﻟﻤﺘﺤﺭﻜﺔ ﻓﻲ ﺍﻟﺤﺎﻟﺔ ﺍﻟﻌﺎﻤﺔ‬

‫‪ ‬‬
‫ﺇﺫﺍ ﻜﺎﻨﺘﺎ ‪ R‬و‪‬‬
‫‪ R‬ﺘﺭﻤﺯﺍﻥ ﺇﻟﻰ ﺍﻟﺴﺭﻋﺔ ﻭﺍﻟﺘﺴﺎﺭﻉ ﺍﻟﻠﺘﺎﻥ ﺘﺘﺤﺭﻙ ﺒﻬﻤﺎ ﻨﻘﻁﺔ ﺍﻷﺼل‬
‫‪ O/‬ﺒﺎﻟﻨﺴﺒﺔ ﺇﻟﻰ ﺍﻟﻨﻘﻁﺔ ﺍﻟﻤﺜﺒﺘﺔ ‪ ،O‬ﺘﻌﻁﻰ ﻋﻨﺩﺌﺫ‪ ‬ﺍﻟﺴﺭﻋﺔ ﻭﺍﻟﺘﺴﺎﺭﻉ ﺒﺎﻟﻨﺴـﺒﺔ ﺇﻟـﻰ‬
‫ﻤﺠﻤﻭﻋﺔ ﺍﻟﻤﺤﺎﻭﺭ ﺍﻟﻤﺜﺒﺘﺔ ﺒﺎﻟﻤﻌﺎﺩﻟﺘﻴﻥ ﺍﻟﺘﺎﻟﻴﺘﻴﻥ‪:‬‬
‫ﺍﻟﺴﺭﻋﺔ‪:‬‬
‫‪‬‬
‫‪ dr1 dR dr‬‬ ‫‪ ‬‬
‫‪V‬‬ ‫‪‬‬ ‫‪‬‬ ‫‪ ωΧ r‬‬ ‫)‪(17‬‬
‫‪dt‬‬ ‫‪dt dt‬‬ ‫‪M‬‬

‫ﺍﻟﺘﺴﺎﺭﻉ‪:‬‬
‫‪‬‬ ‫‪‬‬ ‫‪‬‬ ‫‪‬‬
‫‪d 2 r1 d 2 r   ‬‬ ‫‪ dr   d 2R‬‬
‫‪ 2  ωx ωxr   2ωx  ω x r  2‬‬ ‫)‪(18‬‬
‫‪dt 2‬‬ ‫‪dt‬‬ ‫‪dt‬‬ ‫‪d t‬‬

‫‪- 325 -‬‬

‫)‪Create PDF files without this message by purchasing novaPDF printer (http://www.novapdf.com‬‬
‫‪‬‬ ‫‪‬‬
‫‪d2r‬‬
‫ﻴﻤﻜﻥ ﺍﻟﺤﺼﻭل ﻋﻠﻰ ﻗﺎﻨﻭﻥ ﻨﻴﻭﺘﻥ ﺍﻟﺜﺎﻨﻲ ‪ F  m 2‬ﻓﻲ ﻨﻅﺎﻡ ﺇﺤﺩﺍﺜﻴﺎﺕ‬
‫‪dt‬‬
‫ﻻﻋﻁﺎﻟﻴﺔ ﻤﻥ ﺍﻟﻤﻌﺎﺩﻟﺔ ﺍﻷﺨﻴﺭﺓ ﺒﻌﺩ ﻀﺭﺒﻬﺎ ﺒﻜﺘﻠﺔ ﺍﻟﺠﺴﻡ ﺍﻟﻤﺘﺤﺭﻙ‪.‬‬
‫‪‬‬ ‫‪‬‬
‫‪d2r ‬‬ ‫‪  ‬‬ ‫‪    d 2R ‬‬
‫‪m 2  F  m ωx ωxr   2ωx r  ω x r  2 ‬‬
‫‪dt‬‬ ‫‪‬‬ ‫‪dt ‬‬

‫ﺍﻟﻘﻮﻯ ﺍﻟﻮﻫﻤﻴﺔ ‪Fictitious Forces‬‬


‫ﻴﻤﻜﻨﻨﺎ ﻜﺘﺎﺒﺔ ﻤﻌﺎﺩﻟﺔ ﺍﻟﺤﺭﻜﺔ )‪ (18‬ﻓﻲ ﻨﻅﺎﻡ ﺇﺤﺩﺍﺜﻴﺎﺕ ﻤﺘﺴﺎﺭﻉ ﺒﺼﻭﺭﺓ ﻤﻤﺎﺜﻠـﺔ‬
‫ﻟﺼﻭﺭﺓ ﻗﺎﻨﻭﻥ ﻨﻴﻭﺘﻥ ﺍﻟﺜﺎﻨﻲ ﻓﻲ ﻨﻅﺎﻡ ﻋﻁﺎﻟﻲ ﻭﺫﻟﻙ ﺒﺎﻟﺼﻭﺭﺓ ﺍﻟﺘﺎﻟﻴﺔ‪:‬‬
‫‪‬‬
‫‪d2r ‬‬
‫‪m‬‬ ‫‪ Feff‬‬ ‫)‪(19‬‬
‫‪dt 2‬‬
‫‪‬‬
‫‪d2 r‬‬
‫‪ 2‬ﺍﻟﺘﺴﺎﺭﻉ ﺍﻟﻤﻼﺤﻅ ﻓﻲ ﻨﻅﺎﻡ ﺍﻹﺤﺩﺍﺜﻴﺎﺕ ﺍﻟﻤﺘﺤﺭﻙ ﻭﺍﻟﻨﺎﺘﺞ ﻋﻥ ﺍﻟﻘﻭﺓ‬ ‫‪M‬‬
‫ﺤﻴﺙ‬
‫‪dt‬‬
‫ﺍﻟﻔﻌﺎﻟﺔ ﺍﻟﺘﺎﻟﻴﺔ‪:‬‬
‫‪‬‬
‫‪‬‬ ‫‪‬‬ ‫‪  ‬‬ ‫‪    d2R ‬‬
‫‪Feff  F  m ωx ωxr   2ωxv  ω x r  2 ‬‬ ‫)‪(20‬‬
‫‪‬‬ ‫‪dt ‬‬
‫ﻴﺘﻀﻤﻥ ﺍﻟﺤﺩ ﺍﻟﺜﺎﻨﻲ ﻤﻥ ﺍﻟﻁﺭﻑ ﺍﻷﻴﻤﻥ ﻤﺠﻤﻭﻋﺔ ﻗﻭﻯ ﺘﺴﻤﻰ ﺒـﺎﻟﻘﻭﻯ ﺍﻟﺨﻴﺎﻟﻴـﺔ‪،‬‬
‫ﻭﻫﻲ‪:‬‬
‫ﻗﻭﺓ ﺍﻟﺠﺫﺏ ﺍﻟﻤﺭﻜﺯﻱ‪:centrifugal force‬‬
‫‪‬‬ ‫‪  ‬‬
‫‪Fcf  mωx ωx r ‬‬ ‫)‪(21‬‬
‫ﻗﻭﺓ ﻜﻭﺭﻴﻭﻟﻴﺱ ‪Coriolis force‬‬
‫‪‬‬ ‫‪ ‬‬
‫‪FCor  2mωxv‬‬ ‫)‪(22‬‬
‫ﻗﻭﺓ ﺴﻤﺘﻴﺔ ‪Azimuthal Force‬‬
‫‪‬‬ ‫‪ ‬‬
‫‪Faz   mω xr‬‬ ‫)‪(23‬‬

‫ﻗﻭﺓ ﺍﻨﺴﺤﺎﺒﻴﺔ ‪Translational Force‬‬


‫‪‬‬ ‫‪‬‬
‫‪d2r‬‬
‫‪Ftr   m 2‬‬ ‫)‪(24‬‬
‫‪dt‬‬

‫‪- 326 -‬‬

‫)‪Create PDF files without this message by purchasing novaPDF printer (http://www.novapdf.com‬‬
‫ﺘﻨﺘﺞ ﻗﻭﺓ ﺍﻟﺠﺫﺏ ﺍﻟﻤﺭﻜﺯﻱ ﻓﻲ ﺍﻟﻤﻌﺎﺩﻟﺔ )‪ (21‬ﻤﻥ ﺍﻟﺤﺭﻜﺔ ﺍﻟﺩﻭﺭﺍﻨﻴـﺔ ﻟﻨﻅـﺎﻡ‬
‫‪ ‬‬
‫ﺍﻹﺤﺩﺍﺜﻴﺎﺕ ‪ xyz‬ﻭﺃﻥ ‪ ω.Fcf  0‬ﻻﻥ ﻗﻭﺓ ﺍﻟﺠﺫﺏ ﺍﻟﻤﺭﻜﺯﻱ ﻋﻤﻭﺩﻴﺔ ﻋﻠﻰ ﺍﻟﻤﺤﻭﺭ‬
‫‪‬‬ ‫‪‬‬
‫ﺍﻟﺤﺎﻤل ﻟﻤﺘﺠﻪ ﺍﻟﺴﺭﻋﺔ ﺍﻟﺯﺍﻭﻴﺔ ‪ ، ω‬ﻓﺈﺫﺍ ﺍﺨﺘﻴﺭ ﻤﺘﺠﻪ ﺍﻟﺴﺭﻋﺔ ﺍﻟﺯﺍﻭﻴﺔ ‪ ω‬ﻋﻠﻰ ﻁﻭل‬
‫ﺍﻟﻤﺤﻭﺭ ‪) z‬ﻤﺤﻭﺭ ﺍﻟﺤﺭﻜﺔ( ﻜﻤﺎ ﻫﻭ ﻤﺒﻴﻥ ﻓﻲ ﺍﻟﺸﻜل )‪.(3‬‬

‫ﺍﻟﺸﻜل )‪ :(3‬ﻗﻭﺓ ﺍﻟﺠﺫﺏ ﺃﺜﻨﺎﺀ ﺍﻟﺩﻭﺭﺍﻥ ﺒﺴﺭﻋﺔ ﺯﺍﻭﻴﺔ‬

‫ﺘﻌﻁﻰ ﻓﻲ ﻫﺫﻩ ﺍﻟﺤﺎﻟﺔ ﻗﻭﺓ ﺍﻟﺠﺫﺏ ﺍﻟﻤﺭﻜﺯﻱ ﺒﺎﻟﻌﻼﻗﺔ ﺍﻟﺘﺎﻟﻴﺔ‪:‬‬


‫‪‬‬ ‫‪  ‬‬ ‫‪ ‬‬
‫‪‬‬ ‫‪‬‬ ‫‪‬‬
‫‪Fcf  m ωω. r   r ω 2  mω 2 x i  yj‬‬ ‫‪‬‬ ‫)‪(25‬‬
‫‪‬‬
‫‪ m 2 ‬‬
‫‪‬‬
‫ﺤﻴﺙ ‪ ρ‬ﻤﺘﺠﻪ ﻤﻭﻀﻊ ﺍﻟﺠﺴﻡ ﻤﻥ ﺍﻟﻤﺤﻭﺭ ‪ .z‬ﻭﺘﻘﻊ ﻫﺫﻩ ﺍﻟﻘﻭﺓ ﻓﻲ ﺍﻟﻤﺴﺘﻭﻯ ‪xy‬‬
‫ﻭﺠﻬﺘﻬﺎ ﻤﻥ ﻤﺤﻭﺭ ﺍﻟﺩﻭﺭﺍﻥ‪.‬‬

‫‪- 327 -‬‬

‫)‪Create PDF files without this message by purchasing novaPDF printer (http://www.novapdf.com‬‬
‫ﺍﳊﺮﻛﺔ ﻋﻠﻰ ﺳﻄﺢ ﺍﻷﺭﺽ ‪The Motion on the Earth :‬‬
‫ﻟﺩﺭﺍﺴﺔ ﺤﺭﻜﺔ ﺠﺴﻡ ﺒﺎﻟﻘﺭﺏ ﻤﻥ ﺴﻁﺢ ﺍﻷﺭﺽ‪ ،‬ﻤﻥ ﺍﻟﻤﻨﺎﺴﺏ ﺍﺨﺘﻴـﺎﺭ ﻨﻅـﺎﻡ‬
‫ﺇﺤﺩﺍﺜﻴﺎﺕ ﻤﺜﺒﺕ ﻋﻠﻰ ﺴﻁﺢ ﺍﻷﺭﺽ‪ .‬ﺒﻔﺭﺽ ﺃﻥ ﻨﻅﺎﻡ ﺇﺤﺩﺍﺜﻲ ‪ S/‬ﻴـﺩﻭﺭ ﺒﺴـﺭﻋﺔ‬
‫‪‬‬
‫ﺯﺍﻭﻴﺔ ﺜﺎﺒﺘﺔ ‪ ω‬ﺒﺎﻟﻨﺴﺒﺔ ﻨﻅﺎﻡ ﺇﺤﺩﺍﺜﻴﺎﺕ ﻋﻁﺎﻟﻴﺔ ‪ SI‬ﻭﻟﻬﻤﺎ ﻨﻔـﺱ ﻨﻘﻁـﺔ ﺍﻷﺼـل‬
‫)ﻤﺭﻜﺯ ﺍﻷﺭﺽ( ‪ CM‬ﻜﻤﺎ ﻫﻭ ﻤﺒﻴﻥ ﻓﻲ ﺍﻟﺸﻜل )‪،(4‬‬

‫ﺍﻟﺸﻜل )‪ :(4‬ﻨﻅﺎﻡ ﺍﻹﺤﺩﺍﺜﻴﺎﺕ ﺍﻟﻤﺜﺒﺕ ﻭﺍﻟﻤﺘﺤﺭﻙ‬

‫ﻴﺘﺒﻴﻥ ﻤﻥ ﺍﻟﺸﻜل ﺃﻥ ‪ R‬ﻴﺴﺎﻭﻱ ﺍﻟﺼﻔﺭ ﻻﻨﻁﺒﺎﻕ ﻤﺭﻜﺯﻱ ﺍﻟﻨﻅـﺎﻤﻴﻥ‪ ،‬ﻭﺃﻥ ﻤﺘﺠـﻪ‬


‫ﻤﻭﻀﻊ ﺍﻟﺠﺴﻡ ﺍﻟﻤﺘﺤﺭﻙ ﻫﻭ‪ ،r/‬ﻭﺘﻜﻭﻥ ﺤﺴﺏ ﺍﻟﻤﻌﺎﺩﻟﺔ )‪ ،(20‬ﻤﻌﺎﺩﻟﺔ ﺍﻟﺤﺭﻜﺔ ﻓـﻲ‬
‫ﻫﺫﻩ ﺍﻟﺤﺎﻟﺔ ﻫﻲ‪:‬‬
‫‪‬‬
‫‪d 2 r ‬‬ ‫‪‬‬ ‫‪  ‬‬ ‫‪ ‬‬
‫‪m 2  F  mg  mωx ωx r   2ωxv‬‬ ‫)‪(26‬‬
‫‪dt‬‬
‫ﺘﻜﻭﻥ ﺍﻟﻘﻭﻯ ﺍﻟﻤﺅﺜﺭﺓ ﻋﻠﻰ ﺍﻟﻜﺘﻠﺔ ‪ m‬ﻓﻲ ﻨﻅﺎﻡ ﺍﻹﺤﺩﺍﺜﻴﺎﺕ ﺍﻟﻌﻁﺎﻟﻴـﺔ ﻫـﻲ ﻗـﻭﺓ‬
‫‪‬‬ ‫‪‬‬
‫ﺍﻟﺠﺎﺫﺒﻴﺔ ﺍﻷﺭﻀﻴﺔ ‪ mg‬ﻭﻗﻭﻯ ﺃﺨﺭﻯ ‪ . F‬ﺃﻥ ﻤﺭﻜﺯ ﻨﻅﺎﻡ ﺍﻹﺤﺩﺍﺜﻴﺎﺕ ‪ S‬ﺍﻟﻤﺜﺒـﺕ‬
‫‪‬‬
‫ﻋﻠﻰ ﺍﻷﺭﺽ ﻴﺘﻌﻴﻥ ﺒﻨﺼﻑ ﻗﻁﺭ ﺍﻷﺭﺽ ‪ . R E‬ﻭﻴﺘﻌﻴﻥ ﻤﻭﻀﻊ ﺍﻟﻜﺘﻠﺔ ‪ m‬ﻓﻲ ﻫﺫﺍ‬
‫‪‬‬
‫ﺍﻟﻨﻅﺎﻡ ﺒﺎﻟﻤﺘﺠﻪ ‪ r‬ﻭﻴﻜﻭﻥ‪:‬‬
‫‪ ‬‬ ‫‪‬‬
‫‪r  R E  r‬‬ ‫)‪(27‬‬

‫‪- 328 -‬‬

‫)‪Create PDF files without this message by purchasing novaPDF printer (http://www.novapdf.com‬‬
‫‪‬‬ ‫‪‬‬ ‫‪‬‬
‫‪dr  d r‬‬
‫ﺒﻤﺎ ﺃﻥ ‪ R E‬ﻤﺘﺠﻪ ﺜﺎﺒـﺕ‪ ،‬ﻓـﺎﻥ ‪ ، ‬ﺒﺎﻻﺴـﺘﻔﺎﺩﺓ ﻤـﻥ ﺍﻟﻤﻌﺎﺩﻟـﺔ )‪،(27‬‬
‫‪dt dt‬‬
‫ﻭﺍﻟﻤﻌﺎﺩﻟﺔ )‪ ،(26‬ﻨﺠﺩ ﺃﻥ ﺍﻟﻘﻭﻯ ﺍﻟﻤﺅﺜﺭﺓ ﻋﻠﻰ ﺍﻟﻜﺘﻠﺔ ‪ m‬ﻫﻲ‪:‬‬
‫‪‬‬
‫‪d r ‬‬
‫‪2‬‬
‫‪‬‬ ‫‪  ‬‬ ‫‪‬‬ ‫‪ ‬‬
‫‪  ‬‬
‫‪m 2  F  mg  m ωx ωx R E  r  2ωxv‬‬
‫‪dt‬‬
‫‪‬‬
‫)‪(28‬‬ ‫‪‬‬
‫‪‬‬
‫ﺒﻤﺎ ﺃﻥ ﺴﺭﻋﺔ ﺩﻭﺭﺍﻥ ﺍﻷﺭﺽ ﺼﻐﻴﺭﺓ ﻨﺴﺒﻴﺎﹰ ﻭﻨﺼﻑ ﻗﻁﺭﻫﺎ ‪ R E‬ﻜﺒﻴﺭ ﻤﻘﺎﺭﻨﺔﹰً ﻤﻊ‬
‫‪‬‬ ‫‪‬‬
‫‪ r‬ﻟﺫﻟﻙ ﻴﻤﻜﻥ ﺇﻫﻤﺎل ‪ ، r‬ﻭﺘﻌﻁﻰ ﻤﻌﺎﺩﻟﺔ ﺍﻟﺤﺭﻜﺔ ﺒﺎﻟﻨﺴﺒﺔ ﺇﻟﻰ ﺍﻟﻨﻅـﺎﻡ ‪ S‬ﺒﺸـﻜل‬
‫ﺘﻘﺭﻴﺒﻲ ﺒﺎﻟﺼﻭﺭﺓ‪:‬‬
‫‪‬‬
‫‪d2r ‬‬ ‫‪‬‬ ‫‪  ‬‬ ‫‪ ‬‬
‫‪m 2  F  mg  m ωx ωxR E  2ωxv‬‬
‫‪dt‬‬
‫‪ ‬‬ ‫‪‬‬ ‫‪‬‬ ‫)‪(29‬‬

‫ﺇﺫﺍ ﻜﺎﻨﺕ ﺍﻷﺭﺽ ﻜﺭﻭﻴﺔ ﺘﻤﺎﻤﺎﹰ‪ ،‬ﻓﺎﻥ ﺘﺴﺎﺭﻉ ﺍﻟﺠﺎﺫﺒﻴﺔ ‪ g‬ﻴﻜﻭﻥ ﻟـﻪ ﻗﻴﻤـﺔ ﻭﺍﺤـﺩﺓ‬
‫ﻭﺍﺘﺠﺎﻫﻪ ﻨﺤﻭ ﻤﺭﻜﺯ ﺍﻷﺭﺽ‪ .‬ﻭﻟﻜﻥ ﺍﻟﻭﺍﻗﻊ ﺨﻼﻑ ﺫﻟﻙ ﺤﻴﺙ ﺃﻥ ﺍﻷﺭﺽ ﻟﻴﺴـﺕ‬
‫ﻜﺭﻭﻴﺔ ﺘﻤﺎﻤﺎﹰ ﻜﻤﺎ ﺃﻥ ﻜﺜﺎﻓﺘﻬﺎ ﺘﺘﻐﻴﺭ ﻤﻥ ﻤﻜﺎﻥ ﻵﺨﺭ ﻤﻤﺎ ﻴﺅﺩﻱ ﺇﻟﻰ ﺘﻐﻴـﺭ ‪ g‬ﻤـﻥ‬
‫ﻨﻘﻁﺔ ﺇﻟﻰ ﺃﺨﺭﻯ ﻋﻠﻰ ﺴﻁﺢ ﺍﻷﺭﺽ‪ .‬ﻓﻤﻥ ﺃﺠل ﺠﺴﻡ ﺴﺎﻜﻥ ﻋﻠﻰ ﺴـﻁﺢ ﺍﻷﺭﺽ‬
‫‪‬‬
‫‪d 2r‬‬
‫)‪ (v=0‬ﻭﻟﻜﻲ ﻴﻜﻭﻥ ﻤﺘﺯﻥ ﻴﺠﺏ ﺃﻥ ﻴﻜﻭﻥ ‪ 2  0‬ﻭﻴﻨﺘﺞ ﻤـﻥ ﺍﻟﻤﻌﺎﺩﻟـﺔ )‪(29‬‬
‫‪dt‬‬
‫ﺃﻥ‪:‬‬
‫‪‬‬ ‫‪   ‬‬
‫‪‬‬
‫‪F   m g  ωx ωxR E‬‬ ‫‪‬‬ ‫‪‬‬ ‫)‪(30‬‬
‫ﺒﻔﺭﺽ ﺃﻥ‪:‬‬
‫‪‬‬ ‫‪   ‬‬
‫‪g eff‬‬ ‫‪‬‬
‫‪ g  ωx ωxR E‬‬ ‫‪‬‬ ‫)‪(31‬‬
‫ﻴﺴﻤﻰ ﻫﺫﺍ ﺍﻟﺘﺴﺎﺭﻉ ﺒﺎﻟﺘﺴﺎﺭﻉ ﺍﻟﻔﻌﺎل ﻟﻠﺠﺎﺫﺒﻴﺔ ﺍﻷﺭﻀﻴﺔ‪ ،‬ﻭﺠﻬﺘـﻪ ﻋﻜـﺱ ﺠﻬـﺔ‬
‫‪‬‬
‫ﺍﻟﻘﻭﺓ ‪ ، F‬ﻜﻤﺎ ﻭﻴﻤﻜﻥ ﺤﺴﺎﺏ ﻗﻴﻤﺔ ﺍﻟﺘﺼﺤﻴﺢ ﻓﻲ ﺘﺴﺎﺭﻉ ﺍﻟﺠﺎﺫﺒﻴﺔ ﺍﻷﺭﻀـﻴﺔ ﻤـﻥ‬
‫ﺍﻟﻌﻼﻗﺔ‪:‬‬
‫‪  ‬‬
‫‪‬‬
‫‪ωx ωxR E  ω 2 R E sinθ‬‬ ‫‪‬‬ ‫)‪(32‬‬
‫‪‬‬
‫ﺤﻴﺙ ‪ θ‬ﺍﻟﺯﺍﻭﻴﺔ ﺍﻟﺭﺃﺴﻴﺔ ﺒﻴﻥ ﻤﺘﺠﻪ ﺍﻟﺴﺭﻋﺔ ﺍﻟﺯﺍﻭﻴﺔ ‪ ω‬ﻭﻤﺘﺠـﻪ ﻨﺼـﻑ ﻗﻁـﺭ‬
‫‪‬‬
‫ﺍﻷﺭﺽ ‪ . R E‬ﻭﺒﻤﺎ ﺃﻥ ﺍﻟﺴﺭﻋﺔ ﺍﻟﺯﺍﻭﻴﺔ ﻟﺩﻭﺭﺍﻥ ﺍﻷﺭﺽ ﺤﻭل ﻨﻔﺴﻬﺎ ﺘﺴﺎﻭﻱ ﺇﻟﻰ‪:‬‬
‫‪2π‬‬ ‫‪2π‬‬ ‫‪rad‬‬
‫‪ω‬‬ ‫‪‬‬ ‫‪ 0.727x10  4‬‬ ‫)‪(33‬‬
‫‪τ‬‬ ‫‪24x3600‬‬ ‫‪sec‬‬

‫‪- 329 -‬‬

‫)‪Create PDF files without this message by purchasing novaPDF printer (http://www.novapdf.com‬‬
‫ﺒﺎﻟﺘﻌﻭﻴﺽ ﻓﻲ ﺍﻟﻤﻌﺎﺩﻟﺔ )‪ (32‬ﻨﺤﺼل ﻋﻠﻰ ﻗﻴﻤﺔ ﺍﻟﺘﺼﺤﻴﺢ ﻓﻲ ﺘﺴـﺎﺭﻉ ﺍﻟﺠﺎﺫﺒﻴـﺔ‬
‫ﺍﻷﺭﻀﻴﺔ‪:‬‬
‫‪‬‬
‫‪ω 2 R E sinθ  0.727x10  4‬‬ ‫‪ 6.371x10 sinθ  0.03sinθ sm‬‬
‫‪2‬‬ ‫‪3‬‬
‫‪2‬‬
‫)‪(34‬‬

‫ﺘﻜﻭﻥ ﻫﺫﻩ ﺍﻟﻨﺘﻴﺠﺔ ﺃﻗل ‪ 0.3 0 0‬ﻤﻥ ﻗﻴﻤﺔ ‪ ،g‬ﻭﻤﻥ ﻏﻴﺭ ﺍﻟﻤﻤﻜﻥ ﻗﻴﺎﺴﻬﺎ‪ ،‬ﻭﺠﻬﺔ ﺤـﺩ‬
‫ﺍﻟﺘﺼﺤﻴﺢ ﺒﺎﺘﺠﺎﻩ ﻨﺼﻑ ﺍﻟﻘﻁﺭ ﺍﻟﺫﺍﻫﺏ ﻤﻥ ﻤﺤﻭﺭ ﺍﻟﺩﻭﺭﺍﻥ ﻜﻤﺎ ﻫﻭ ﻤﻭﻀـﺢ ﻓـﻲ‬
‫ﺍﻟﺸﻜل ) ‪.(5‬‬

‫ﺍﻟﺸﻜل )‪ :(5‬ﺘﺴﺎﺭﻉ ﺍﻟﺠﺎﺫﺒﻴﺔ ﺍﻟﻔﻌﺎل ﻭﺍﺘﺠﺎﻫﺎﺕ ﺍﻟﻘﻭﻯ‬

‫ﺒﺈﺩﺨﺎل ‪ geff‬ﻓﻲ ﺍﻟﻤﻌﺎﺩﻟﺔ ﺍﻟﺘﻔﺎﻀﻠﻴﺔ )‪ ،(29‬ﺍﻟﺘﻲ ﺘﺼﻑ ﺤﺭﻜﺔ ﺠﺴﻡ ﻋﻠـﻰ ﺴـﻁﺢ‬
‫ﺍﻷﺭﺽ ﻨﺤﺼل ﻋﻠﻰ ﺍﻟﻤﻌﺎﺩﻟﺔ ﺍﻵﺘﻴﺔ‪:‬‬
‫‪d r ‬‬
‫‪2‬‬
‫‪‬‬ ‫‪ ‬‬
‫‪m‬‬ ‫‪2‬‬
‫‪ F  mg eff  2mωxv‬‬ ‫)‪(35‬‬
‫‪dt‬‬
‫ﻭﻤﻥ ﺍﻟﻤﻼﺌﻡ ﻓﻲ ﺩﺭﺍﺴﺔ ﻫﺫﻩ ﺍﻟﻤﺴﺄﻟﺔ ﺍﺨﺘﻴﺎﺭ ﺍﻟﻤﺤﺎﻭﺭ ﺍﻹﺤﺩﺍﺜﻴﺔ ﺍﻟﻤﺘﻌﺎﻤـﺩﺓ ﻋﻠـﻰ‬
‫ﺴﻁﺢ ﺍﻷﺭﺽ ﺒﺤﻴﺙ ﻴﻜﻭﻥ ﺍﻟﻤﺤﻭﺭ ‪ z‬ﺤﺎﻤل ﻟﻠﻤﺘﺠﻪ ‪ geff‬ﺒﺤﻴﺙ‪:‬‬
‫‪‬‬ ‫‪‬‬
‫‪g eff  g eff ez‬‬ ‫)‪(36‬‬
‫ﻭﺍﺨﺘﻴﺎﺭ ﺍﻟﻤﺤﻭﺭ ‪ y‬ﺇﻟﻰ ﺍﻟﺸﻤﺎل ﺃﻤﺎ ﺍﻟﻤﺤﻭﺭ ‪ x‬ﻓﻴﺨﺘﺎﺭ ﺇﻟﻰ ﺍﻟﺸﺭﻕ ﻜﻤﺎ ﻫـﻭ ﻤﺒـﻴﻥ‬
‫‪‬‬
‫ﻓﻲ ﺍﻟﺸﻜل )‪ .(5‬ﻭﻴﻜﺘﺏ ﻤﺘﺠﻪ ﺍﻟﺴﺭﻋﺔ ﺍﻟﺯﺍﻭﻴﺔ ‪ ω‬ﺒﺩﻻﻟﺔ ﻤﺭﻜﺒﺎﺘـﻪ ﻋﻠـﻰ ﻫـﺫﻩ‬
‫ﺍﻟﻤﺤﺎﻭﺭ ﻭﻓﻕ ﺍﻟﻤﻌﺎﺩﻟﺔ ﺍﻟﺘﺎﻟﻴﺔ‪:‬‬
‫‪‬‬ ‫‪‬‬ ‫‪‬‬ ‫‪‬‬
‫‪ω  0 i  ωsinθ j  ωcosθk‬‬ ‫)‪(37‬‬

‫‪- 330 -‬‬

‫)‪Create PDF files without this message by purchasing novaPDF printer (http://www.novapdf.com‬‬
‫ﺒﺎﻟﺘﻌﻭﻴﺽ ﻤﻥ ﺍﻟﻤﻌﺎﺩﻟﺔ )‪ ،(37‬ﻓﻲ ﻋﻼﻗﺔ ﻗﻭﺓ ﻜﻭﺭﻴﻭﻟﻴﺱ ﺍﻟﺘﺎﻟﻴﺔ‪:‬‬
‫‪‬‬ ‫‪ ‬‬
‫‪Fcor  2mωxv‬‬ ‫)‪(38‬‬
‫ﻨﺤﺼل ﻋﻠﻰ‪:‬‬
‫‪‬‬ ‫‪‬‬ ‫‪‬‬ ‫‪‬‬
‫‪‬‬
‫‪Fcor  2mω v y cosθ  v z sinθ i  v x cosθ j  v x sinθk‬‬ ‫‪‬‬ ‫)‪(39‬‬
‫ﻭﻴﻨﺘﺞ ﻋﻥ ﺘﺄﺜﻴﺭ ﻗﻭﺓ ﻜﻭﺭﻴﻭﻟﻴﺱ ﻫﺫﻩ ﺍﻨﺤﺭﺍﻑ ﺍﻟﺠﺴﻡ ﻋﻥ ﺍﺘﺠـﺎﻩ ﺤﺭﻜﺘـﻪ‪ .‬ﻓﻔـﻲ‬
‫‪ ، 0  θ ‬ﻴﻜـﻭﻥ‬ ‫ﻨﺼﻑ ﺍﻟﻜﺭﺓ ﺍﻷﺭﻀﻴﺔ ﺍﻟﺸﻤﺎﻟﻲ ﻭﺍﻟﺯﺍﻭﻴﺔ ‪ θ‬ﻀﻤﻥ ﺍﻟﻤﺠـﺎل‬
‫‪π‬‬
‫‪2‬‬
‫ﺍﻨﺤﺭﺍﻑ ﺍﻟﺠﺴﻡ ﻋﻥ ﺍﺘﺠﺎﻩ ﺤﺭﻜﺘﻪ ﻤﺒﻴﻥ ﻓﻲ ﺍﻟﺠﺩﻭل ﺍﻟﺘﺎﻟﻲ‪:‬‬

‫ﺠﻬﺔ ﺍﻻﻨﺤﺭﺍﻑ‬ ‫ﺠﻬﺔ ﺍﻟﺴﺭﻋﺔ‬


‫ﺸﺭﻕ‬ ‫ﺸﻤﺎل)‪(vy>0‬‬
‫ﺠﻨﻭﺏ ﻭﺇﻟﻰ ﺍﻷﻋﻠﻰ‬ ‫ﺸﺭﻕ )‪(vx>0‬‬
‫ﻏﺭﺏ‬ ‫ﺠﻨﻭﺏ )‪(vy<0‬‬
‫ﺸﻤﺎل ﻭﺍﻟﻰ ﺍﻷﺴﻔل‬ ‫ﻏﺭﺏ )‪(vx<0‬‬
‫ﻏﺭﺏ‬ ‫ﺃﻋﻠﻰ )‪(vz>0‬‬
‫ﺸﺭﻕ‬ ‫ﺃﺴﻔل )‪(vz<0‬‬
‫ﺣﺮﻛﺔ ﺟﺴﻢ ﺑﺎﻟﻨﺴﺒﺔ ﻟﻸﺭﺽ ‪:Motion of Body Compared the Earth‬‬
‫ﻻ ﻴﻤﻜﻥ ﺘﻁﺒﻴﻕ ﻗﺎﻨﻭﻥ ﻨﻴﻭﺘﻥ ﺍﻟﺜﺎﻨﻲ ﺇﻻ ﻓﻲ ﻤﺠﻤﻭﻋﺔ ﻤﺤﺎﻭﺭ ﻗﺼـﻭﺭﻴﺔ ﻓﻘـﻁ‪،‬‬
‫ﻭﻟﻜﻥ ﺇﺫﺍ ﺍﺴﺘﺨﺩﻤﻨﺎ ﺍﻟﻌﻼﻗﺔ‬
‫‪‬‬ ‫‪ ‬‬
‫‪‬‬ ‫‪d2 r‬‬ ‫‪    ‬‬
‫‪aF  2‬‬ ‫‪ ω Χ r  2ωΧ r  ωΧ ωΧ r ‬‬ ‫)‪(40‬‬
‫‪dt‬‬ ‫‪M‬‬

‫ﻨﺤﺼل ﻋﻠﻰ ﻨﺘﻴﺠﺔ ﻗﺎﺒﻠﺔ ﻟﻠﺘﻁﺒﻴﻕ ﻓﻲ ﻤﺠﻤﻭﻋﺎﺕ ﺍﻟﻤﺤﺎﻭﺭ ﻏﻴﺭ ﺍﻟﻘﺼﻭﺭﻴﺔ‪ .‬ﻴﻤﻜـﻥ‬
‫ﻜﺘﺎﺒﺔ ﻫﺫﻩ ﺍﻟﻨﺘﻴﺠﺔ ﺒﻌﺩ ﻀﺭﺒﻬﺎ ﺒﻜﺘﻠﺔ ﺍﻟﺠﺴﻡ ‪ m‬ﺒﺎﻟﺸﻜل ﺍﻵﺘﻲ‪:‬‬
‫‪‬‬
‫‪d2r ‬‬ ‫‪ ‬‬ ‫‪ ‬‬ ‫‪  ‬‬
‫‪dt‬‬
‫‪‬‬ ‫‪‬‬ ‫‪‬‬ ‫‪‬‬
‫‪m 2  F - m ω Χ r - 2m ω ΧV - mωΧ ω Χ r ‬‬ ‫)‪(41‬‬
‫‪‬‬ ‫‪‬‬
‫ﺒﻤﺎ ﺃﻥ ﺍﻷﺭﺽ ﺘﺩﻭﺭ ﺒﺴﺭﻋﺔ ﺯﺍﻭﻴﺔ ‪ ω‬ﺜﺎﺒﺘﺔ ﺤﻭل ﻤﺤﻭﺭﻫﺎ‪ ،‬ﻓﺄﻥ ‪ ، ω  0‬ﻋﻨﺩﺌﺫ‪‬‬
‫ﺘﺼﺒﺢ ﺍﻟﻤﻌﺎﺩﻟﺔ ﺍﻟﺴﺎﺒﻘﺔ ﺒﺎﻟﺼﻭﺭﺓ‬

‫‪- 331 -‬‬

‫)‪Create PDF files without this message by purchasing novaPDF printer (http://www.novapdf.com‬‬
‫‪‬‬
‫‪d2r ‬‬ ‫‪ ‬‬ ‫‪  ‬‬
‫‪‬‬ ‫‪‬‬
‫‪m 2  F  2m ΧV  mωΧωΧ r ‬‬
‫‪dt‬‬
‫)‪(42‬‬

‫ﺤﻴﺙ‪:‬‬
‫‪ ‬‬
‫ﻗﻭﺓ ﻜﻭﺭﻴﻭ ﻟﻴﺱ‪.‬‬ ‫‪2m V‬‬ ‫‪‬‬ ‫‪‬‬
‫‪  ‬‬
‫‪ m r ‬ﺍﻟﻘﻭﺓ ﺍﻟﺠﺎﺫﺒﺔ ﺍﻟﻤﺭﻜﺯﻴﺔ‪.‬‬
‫‪  ‬‬
‫‪  mr ‬ﺍﻟﻘﻭﺓ ﺍﻟﻁﺎﺭﺩﺓ ﺍﻟﻤﺭﻜﺯﻴﺔ‪.‬‬

‫ﺃﻣﺜﻠﺔ ‪Examples‬‬
‫‪ -1‬ﺘﺩﻭﺭ ﻤﺠﻤﻭﻋﺔ ﻤﺤﺎﻭﺭ ‪ xyz‬ﺒﺎﻟﻨﺴﺒﺔ ﻟﻤﺠﻤﻭﻋﺔ ﺃﺨﺭﻯ ﻤﺜﺒﺘﺔ ‪ XYZ‬ﻟﻬﻤـﺎ‬
‫ﻨﻔﺱ ﻨﻘﻁﺔ ﺍﻷﺼل‪ .‬ﻴﻌﻁﻰ ﻤﺘﺠﻪ ﺍﻟﺴﺭﻋﺔ ﺍﻟﺯﺍﻭﻴﺔ ﻟﻠﻤﺠﻤﻭﻋﺔ ‪ xyz‬ﺒﺎﻟﻨﺴـﺒﺔ ﺇﻟـﻰ‬
‫ﺍﻟﻤﺠﻤﻭﻋﺔ ‪ XYZ‬ﺒﺎﻟﻌﻼﻗﺔ‪:‬‬
‫‪‬‬ ‫‪‬‬ ‫‪‬‬ ‫‪‬‬
‫‪ ω  2t i - t 2 j  2t  4k‬ﺤﻴﺙ ‪t‬ﻫﻭ ﺍﻟﺯﻤﻥ‪ .‬ﻴﻌﻁﻰ ﻤﺘﺠﻪ ﻤﻭﻀﻊ ﺠﺴﻡ ﻋﻨﺩ‬
‫ﺍﻟﻠﺤﻅﺔ ‪ ،t‬ﻜﻤﺎ ﻴﺸﺎﻫﺩ ﻓﻲ ﺍﻟﻤﺠﻤﻭﻋﺔ ‪ xyz‬ﺒﺎﻟﻌﻼﻗﺔ‪:‬‬
‫‪‬‬ ‫‪‬‬ ‫‪‬‬ ‫‪‬‬
‫‪r  t 2 i  6t j   4t 3  4 k‬‬
‫ﺃﻭﺠﺩ ﻋﻨﺩ ﺍﻟﻠﺤﻅﺔ ‪ t=1s‬ﻜلٍ ﻤﻥ‪:‬‬
‫ﺍﻟﺴﺭﻋﺔ ﺍﻟﻅﺎﻫﺭﻴﺔ ﻭﺍﻟﺘﺴﺎﺭﻉ ﺍﻟﻅﺎﻫﺭﻱ‪.‬‬ ‫أ‪-‬‬
‫ﺍﻟﺴﺭﻋﺔ ﺍﻟﺤﻘﻴﻘﻴﺔ‪.‬‬ ‫ب‪-‬‬
‫ﺘﺴﺎﺭﻉ ﻜﻭﺭﻴﻭ ﻟﻴﺱ‪.‬‬ ‫ت‪-‬‬
‫ﺘﺴﺎﺭﻉ ﺍﻟﺠﺎﺫﺒﻴﺔ ﺍﻟﻤﺭﻜﺯﻴﺔ‪.‬‬ ‫ث‪-‬‬
‫ﺍﻟﺤل‪:‬‬
‫أ‪ -‬ﺍﻟﺴﺭﻋﺔ ﺍﻟﻅﺎﻫﺭﻴﺔ ﻋﻨﺩ ﺍﻟﻠﺤﻅﺔ ‪ t‬ﻫﻲ‪:‬‬
‫‪ dr‬‬ ‫‪ ‬‬ ‫‪‬‬
‫‪V‬‬ ‫‪M‬‬
‫‪ 2t i - 6 j  12t 2 k‬‬ ‫)‪(1‬‬
‫‪dt‬‬
‫ﻭﻋﻨﺩﻤﺎ ‪ t=1s‬ﺘﻜﻭﻥ ﺍﻟﺴﺭﻋﺔ ﺍﻟﻅﺎﻫﺭﻴﺔ ﻤﺴﺎﻭﻴﺔ ﺇﻟﻰ‪.‬‬
‫‪‬‬ ‫‪ ‬‬ ‫‪‬‬
‫‪V  2 i - 6 j  12k‬‬

‫‪- 332 -‬‬

‫)‪Create PDF files without this message by purchasing novaPDF printer (http://www.novapdf.com‬‬
‫( ﺒﺎﻟﻨﺴـﺒﺔ ﻟﻠـﺯﻤﻥ‬1) ‫ﻹﻴﺠﺎﺩ ﺍﻟﺘﺴﺎﺭﻉ ﺍﻟﻅﺎﻫﺭﻱ ﻨﺸﺘﻕ ﺍﻟﺴﺭﻋﺔ ﺍﻟﻅﺎﻫﺭﻴﺔ ﺍﻟﻌﻼﻗﺔ‬
:‫ﻓﻨﺤﺼل ﻋﻠﻰ‬
  
a  2 i  24tk (2)
:‫ ﻴﺴﺎﻭﻱ‬t=1 s ‫ﺍﻟﺘﺴﺎﺭﻉ ﺍﻟﻅﺎﻫﺭﻱ ﻋﻨﺩ‬
  
a  2 i  24k
:‫ ﻤﻥ‬t ‫ ﺘﻌﻁﻰ ﺍﻟﺴﺭﻋﺔ ﺍﻟﺤﻘﻴﻘﻴﺔ ﻋﻨﺩ ﺃﻱ ﺯﻤﻥ‬-‫ب‬

 
dr  
V F
 ωΧ r (3)
dt M
   
 dr  
vF 
dt M

 ω Χ r  2 i - 6 j  12k 
 2    
   
 2t i - t j  2t  4 k Χ t 2  1 i - 6t j  4t 3 k 
:(3) ‫ ﺘﺤﺴﺏ ﺍﻟﺴﺭﻋﺔ ﺍﻟﺤﻘﻴﻘﻴﺔ ﻤﻥ ﺍﻟﻌﻼﻗﺔ ﺭﻗﻡ‬t=1s ‫ﻭﻋﻨﺩ‬
  
i j k
      
V  2 i - 6 j  12k  2 - 1 6  30 i - 16 j  k (4)
1 -6 4

‫ﺘﺴﺎﺭﻉ ﻜﻭﺭﻴﻭ ﻟﻴﺱ‬ -‫ت‬


      
  dr
a C  2ωΧ
dt
 
 4 i  2 j  12k Χ 2 i  6 j  12k 
  
 48 i  24 j  20k
(5)
:‫ﺍﻟﺘﺴﺎﺭﻉ ﺍﻟﺠﺎﺫﺏ‬ -‫ث‬
        
 
ωΧ ωΧ r   2 i  j  6k Χ 32 i  4 j  10k
  

 71i  190 j  8k

‫ ﺒﺴــﺭﻋﺔ ﺯﺍﻭﻴــﺔ‬xyz ‫ﺘﺘﺤــﺭﻙ ﻤﺠﻤﻭﻋــﺔ ﻤﺤــﺎﻭﺭ‬ .a


   
‫ ﻤﺜﺒﺘﺔ )ﻗﺼﻭﺭﻴﺔ( ﻟﻬﻤﺎ ﻨﻔﺱ‬XYZ ‫ ﺒﺎﻟﻨﺴﺒﺔ ﻟﻤﺠﻤﻭﻋﺔ ﻤﺤﺎﻭﺭ‬ω  2 i  3 j  5k

- 333 -

Create PDF files without this message by purchasing novaPDF printer (http://www.novapdf.com)
‫‪‬‬
‫ﻨﻘﻁﺔ ﺍﻷﺼل‪ .‬ﺇﺫﺍ ﻜﺎﻥ ‪ r‬ﻤﺘﺠﻪ ﻤﻭﻀﻊ ﺍﻟﺠﺴﻡ ﺍﻟﻤﺘﺤﺭﻙ‪ ،‬ﻴﻌﻁﻰ ﻓـﻲ ﺍﻟﻤﺠﻤﻭﻋـﺔ‬
‫‪ xyz‬ﺒﺩﻻﻟﺔ ﺍﻟﺯﻤﻥ ‪ t‬ﺒﺎﻟﻌﻼﻗﺔ‪:‬‬
‫‪‬‬ ‫‪‬‬ ‫‪‬‬ ‫‪‬‬
‫‪r  sint i  cost j  e  t k‬‬
‫ﺃﻭﺠﺩ ‪:‬‬
‫‪‬‬
‫ﺒﺎﻟﻨﺴﺒﺔ ﻟﻠﻤﺠﻤﻭﻋﺔ ﺍﻟﻤﺘﺤﺭﻜﺔ‪.‬‬
‫‪dr‬‬
‫أ‪-‬‬
‫‪dt‬‬
‫‪‬‬
‫ﺒﺎﻟﻨﺴﺒﺔ ﻟﻠﻤﺠﻤﻭﻋﺔ ﺍﻟﻤﺜﺒﺘﺔ‪.‬‬
‫‪dr‬‬
‫ب‪-‬‬
‫‪dt‬‬
‫‪‬‬
‫‪d2r‬‬
‫‪ 2‬ﺒﺎﻟﻨﺴﺒﺔ ﻟﻠﻤﺠﻤﻭﻋﺔ ﺍﻟﻤﺘﺤﺭﻜﺔ‪.‬‬ ‫ت‪-‬‬
‫‪dt‬‬
‫‪‬‬
‫‪d2r‬‬
‫‪ 2‬ﺒﺎﻟﻨﺴﺒﺔ ﻟﻠﻤﺠﻤﻭﻋﺔ ﺍﻟﻤﺜﺒﺘﺔ‪.‬‬ ‫ث‪-‬‬
‫‪dt‬‬
‫‪‬‬ ‫‪‬‬ ‫‪‬‬ ‫‪‬‬
‫‪ -3‬ﺘﺩﻭﺭ ﻤﺠﻤﻭﻋﺔ ﻤﺤﺎﻭﺭ ‪ xyz‬ﺒﺴـﺭﻋﺔ ﺯﺍﻭﻴـﺔ ‪ ω  5 i  4 j  10k‬ﺒﺎﻟﻨﺴـﺒﺔ‬
‫ﻟﻤﺠﻤﻭﻋﺔ ﻤﺤﺎﻭﺭ ‪ XYZ‬ﻤﺜﺒﺘﺔ ﻟﻬﺎ ﻨﻔﺱ ﻨﻘﻁﺔ ﺍﻷﺼل‪ .‬ﺃﻭﺠﺩ ﺴﺭﻋﺔ ﺠﺴﻡ ﻤﺜﺒـﺕ‬
‫ﻓﻲ ﺍﻟﻤﺠﻤﻭﻋﺔ ‪ xyz‬ﻋﻨﺩ ﺍﻟﻨﻘﻁﺔ )‪ (-2,1,3‬ﻜﻤـﺎ ﻴﺭﺍﻫـﺎ ﻤﺸـﺎﻫﺩ ﻤﺜﺒـﺕ ﻓـﻲ‬
‫ﺍﻟﻤﺠﻤﻭﻋﺔ ‪.XYZ‬‬

‫‪ -4‬ﻤﺠﻤﻭﻋــﺔ ﻤﺤــﺎﻭﺭ ‪ xyz‬ﺘــﺩﻭﺭ ﺤــﻭل ﺍﻟﻤﺤــﻭﺭ ‪ z‬ﺒﺴــﺭﻋﺔ ﺯﺍﻭﻴــﺔ‪:‬‬


‫‪‬‬ ‫‪‬‬ ‫‪‬‬
‫‪   cos t i  sin t j‬ﺒﺎﻟﻨﺴﺒﺔ ﻟﻤﺠﻤﻭﻋﺔ ﺍﻟﻤﺤﺎﻭﺭ ‪ XYZ‬ﺍﻟﻤﺜﺒﺘﺔ ‪ ،‬ﺤﻴﺙ ‪ t‬ﻫﻲ‬
‫ﺍﻟﺯﻤﻥ ‪ .‬ﻤﺘﺠﻪ ﺍﻟﻤﻭﻀﻊ ﻟﻨﻘﻁﺔ ﺃﺼل ﺍﻟﻤﺠﻭﻋﺔ ‪ xyz‬ﺒﺎﻟﻨﺴﺒﺔ ﻟﻠﻤﺠﻤﻭﻋﺔ ‪XYZ‬‬
‫‪‬‬ ‫‪ ‬‬ ‫‪‬‬
‫ﻫﻭ ‪ . R  t i  j  t 2 k‬ﺇﺫﺍ ﻜﺎﻥ ﻤﺘﺠﻪ ﻤﻭﻀـﻊ ﺠﺴـﻡ ﺒﺎﻟﻨﺴـﺒﺔ ﻟﻠﻤﺠﻤﻭﻋـﺔ‬
‫‪‬‬ ‫‪‬‬ ‫‪‬‬ ‫‪‬‬
‫‪r  3t  1i  2t j  5k‬‬ ‫ﺍﻟﻤﺘﺤﺭﻜﺔ ﻫﻭ‪:‬‬
‫ﺃﻭﺠﺩ‪:‬‬
‫ﺍﻟﺴﺭﻋﺔ ﺍﻟﻅﺎﻫﺭﻴﺔ ﻟﻠﺠﺴﻡ ﻋﻨﺩ ﺃﻱ ﺯﻤﻥ ‪.t‬‬ ‫أ‪-‬‬
‫ﺍﻟﺴﺭﻋﺔ ﺍﻟﺤﻘﻴﻘﻴﺔ ﻟﻠﺠﺴﻡ ﻋﻨﺩ ﺃﻱ ﺯﻤﻥ ‪.t‬‬ ‫ب‪-‬‬
‫ﺍﻟﺘﺴﺎﺭﻉ ﺍﻟﻅﺎﻫﺭﻱ ﻟﻠﺠﺴﻡ ﻋﻨﺩ ﺃﻱ ﺯﻤﻥ ‪.t‬‬ ‫ت‪-‬‬
‫ﺍﻟﺘﺴﺎﺭﻉ ﺍﻟﺤﻘﻴﻘﻲ ﻟﻠﺠﺴﻡ ﻋﻨﺩ ﺃﻱ ﺯﻤﻥ ‪.t‬‬ ‫ث‪-‬‬
‫ﺍﻟﺤل‪:‬‬

‫‪- 334 -‬‬

‫)‪Create PDF files without this message by purchasing novaPDF printer (http://www.novapdf.com‬‬
:‫ﺍﻟﺴﺭﻋﺔ ﺍﻟﻅﺎﻫﺭﻴﺔ‬ -‫أ‬
 dr  
V  3 i  2j
dt M

‫ﺍﻟﺴﺭﻋﺔ ﺍﻟﺤﻘﻴﻘﻴﺔ‬ -‫ب‬



 dR dr 
dr      
V    ωΧ r  i  2tk  3 i  2 j
dt dt F dt M
  
i j k
 cost sint 0
3t  1  2t 5
     
 i  2t k  3 i  2 j  5sint i  5cost  j

  2t.cost  3t  1sint k
   
 v  4  5sint  i  2  5cost  j  2t  2t.cost  3t  1sint k
:‫ﺍﻟﺘﺴﺎﺭﻉ ﺍﻟﻅﺎﻫﺭﻱ‬ -‫ت‬

 d2 r
a 2 0
dt M

:‫ﺍﻟﺘﺴﺎﺭﻉ ﺍﻟﺤﻘﻴﻘﻲ‬ -‫ث‬


 
 d 2R d 2 r       
a 2  2  ω Χ r  2ωΧV  ωΧ ωΧ r 
dt dt M

- 335 -

Create PDF files without this message by purchasing novaPDF printer (http://www.novapdf.com)
     
i j k i j k
 
a  2k   sint cost 0  2 cost sint 0
3t  1  2t 5 3 2 0
  
i j k
 2 cost sint 0
5sint  5cost  2t.cost  3t  1sint 
   
a  2k  5cost i   5sint  j
 
 2t.sint   3t  1cost k  2 2cost  3sint k


  2tcost.sin t  3t  1sin 2 t i 
 

   
 
  2t.cos 2 t  3t  1sint.cost j   5cos 2 t  5sin 2 t k
   


a  2k  5cost i  5sint j  2tsint k  3tcostk  costk  4costk  6sint k
     
 2tcost.sint i  3tsin 2t i  sin 2t i  2tcos 2 t j  3tsint.cos t j  5k

 

a  5cost  2tsint.cos t  3tsin 2 t  sin 2 t i 


 5sint  2tcos 2 t  3tsint.cos t j  
 2  2tsint  3tcost  cost  4cost  6sint  5k
  
  
a  5cost  2tsint.cost  3tsin2t  sin2t i  5sint  2tcos2t  3tsint.cost j  

2tsint  3tcost  5cost  6sint  3k

- 336 -

Create PDF files without this message by purchasing novaPDF printer (http://www.novapdf.com)
 
‫ ﻋﺮﺑﻲ‬- ‫ﺇﻧﻜﻠﻴﺰﻱ‬

-A-
  Absorbance  
Active Force  
Addition of Velocities  
Amplitude  
Amplitude of Vibration  
Amplitude of Simple Harmonic  
Motion
Analytical Mechanics  
Angular Momentum=moment of  
Momentum
Anti-Rolling  
Aperiodic  
Arm of Couple=Moment Arm  
Asymptotical  
Asymptotical  
Atom  
Attraction (Gravitational)  
Attractive Force  
Axis  
-B-
Balance (Spring)  
Ballistica 
 
Binormal  

- 337 -

Create PDF files without this message by purchasing novaPDF printer (http://www.novapdf.com)

You might also like